Вы находитесь на странице: 1из 429

Thank you

for purchasing this e-book.


To receive special offers and news
about our latest products,
sign up below.

Sign Up
Or visit LWW.com

W olters Kluwer
Health
L ippincott’s
ANESTHESIA
REVIEW:
1,001
QUESTIONS AND ANSWERS
L ippincott’s
ANESTHESIA
REVIEW:
1,001
QUESTIONS AND ANSWERS
Paul Sikka, MD, PhD
Department of Anesthesia and
Perioperative Medicine
Signature Healthcare Brockton Hospital,
Brockton, Massachusetts
Affiliate of Beth Israel Deaconess Medical
Center, Boston, Massachusetts (Former
Faculty—Brigham and Women’s Hospital,
Harvard Medical School)

Edward A. Bittner, MD, PhD, FCCP, FCCM


Program Director, Critical Care Medicine-Anesthesiology
Fellowship, Associate
Director, Surgical Intensive Care Unit,
Assistant Professor of Anaesthesia,
Harvard Medical School, Massachusetts
General Hospital, Department of
Anesthesia, Critical Care, and Pain
Medicine, Boston, Massachusetts

Thomas M. Halaszynski, DMD, MD, MBA


Associate Professor of Anesthesiology,
Director of Regional Anesthesia/
Acute Pain Medicine, Department of
Anesthesiology, Yale University School of
Medicine, Yale New Haven Hospital, New
Haven, Connecticut

T hoha M. Pham, MD
Associate Clinical Professor, University
of California, San Francisco (UCSF),
Department of Anesthesia and
Perioperative Care, San Francisco, California

Ashish C. Sinha, MD, PhD, DABA


Vice Chairman, Anesthesiology &
Critical Care, Drexel University College
of Medicine, Hahnemann University
Hospital, Philadelphia, Pennsylvania

iX}‫'״‬Wolters Kluwer
Philadelphia j Baltimore j New Yodt l London
Buenos Aires 1 Hong Kong - Sydney - ToJtyo
Acquisitions Editor: Brian Brown
Product Development Editor: Nicole Dernoski
Editorial Assistant: Lindsay Burgess
Production Project Manager: Bridgett Dougherty
Design Coordinator: Stephen Druding
Manufacturing Coordinator: Beth Welsh
Marketing Manager: Dan Dressler
Prepress Vendor: S4C Publishing Services

Copyright © 2015 Wolters Kluwer Health

All rights reserved. This book is protected by copyright. No part of this book may be reproduced or transmitted in any form or by any
means, including as photocopies or scanned-in or other electronic copies, or utilized by any information storage and retrieval system
without written permission from the copyright owner, except for brief quotations embodied in critical articles and reviews. Materials
appearing in this book prepared by individuals as part of their official duties as U.S. government employees are not covered by the
above-mentioned copyright. To request permission, please contact Wolters Kluwer Health at Two Commerce Square, 2001 Market
Street, Philadelphia, PA 19103, via email at permissions@lww.com, or via our website at lww.com (products and services).

98765432 1

Library of Congress Cataloging-in-Publication Data

Sikka, Paul, author.


Lippincott’s anesthesia review : 1001 questions and answers / Paul Sikka, Edward Bittner, Thomas Halaszynski, Thoha Pham, Ashish
Sinha.
p. ; cm.
Anesthesia review
E-ISBN: 978-1-4698-3101-5
I. Bittner, Edward A., 1967- author. II. Halaszynski, Thomas, author. III. Pham, Thoha, author. IV. Sinha, Ashish, author. V. Title. VI. Title:
Anesthesia review.
[DNLM: 1. Anesthesia--Examination Questions. 2. Anesthetics—Examination Questions. WO 218.2]
RD82.3
617.9'6076—dc23

2014019574

This work is provided “as is,” and the publisher disclaims any and all warranties, express or implied, including any warranties as to
accuracy, comprehensiveness, or currency of the content of this work.

This work is no substitute for individual patient assessment based upon healthcare professionals’ examination of each patient and
consideration of, among other things, age, weight, gender, current or prior medical conditions, medication history, laboratory data, and
other factors unique to the patient. The publisher does not provide medical advice or guidance, and this work is merely a reference tool.
Healthcare professionals, and not the publisher, are solely responsible for the use of this work, including all medical judgments and for
any resulting diagnosis and treatments.

Given continuous, rapid advances in medical science and health information, independent professional verification of medical diagnoses,
indications, appropriate pharmaceutical selections and dosages, and treatment options should be made and healthcare professionals
should consult a variety of sources. When prescribing medication, healthcare professionals are advised to consult the product information
sheet (the manufacturer’s package insert) accompanying each drug to verify, among other things, conditions of use, warnings, and side
effects and identify any changes in dosage schedule or contradictions, particularly if the medication to be administered is new,
infrequently used, or has a narrow therapeutic range. To the maximum extent permitted under applicable law, no responsibility is assumed
by the publisher for any injury and/or damage to persons or property, as a matter of products liability, negligence law or otherwise, or
from any reference to or use by any person of this work.

LWW.com
“Dedicated to our Parents and Teachers ”
who selflessly pass on their values and knowledge to us
CONTRIBUTORS

Mian Ahmad, MD
D e p a r tm e n t o f A n e s th e s io lo g y a n d P e r io p e r a tiv e M e d ic in e , D r e x e l U n iv e r s ity C o lle g e o f M e d ic in e ,
P h ila d e lp h ia , P e n n s y lv a n ia

Sheri M. Berg, MD
I n s tru c to r, D e p a r tm e n t o f A n e s th e s ia , C r itic a l C a r e , a n d P a in M e d ic in e , M a s s a c h u s e tts G e n e r a l
H o s p ita l, B o s to n , M a s s a c h u s e tts

Edward A. Bittner, MD, PhD, FCCP, FCCM


P r o g r a m D ir e c to r , C r itic a l C a r e M e d ic in e - A n e s th e s io lo g y F e llo w s h ip , A s s o c ia te D ir e c to r , S u r g ic a l
In te n s iv e C a r e U n it, A s s is ta n t P r o f e s s o r o f A n a e s th e s ia , H a r v a r d M e d ic a l S c h o o l, M a s s a c h u s e tts
G e n e r a l H o s p ita l, D e p a r tm e n t o f A n e s th e s ia , C r itic a l C a r e , a n d P a in M e d ic in e , B o s to n ,
M a s s a c h u s e tts

Yuriy S. Bronshteyn, MD
S u r g ic a l C r itic a l C a r e F e llo w , M a s s a c h u s e tts G e n e r a l H o s p ita l, D e p a r tm e n t o f A n e s th e s ia , C r itic a l
C a r e , a n d P a in M e d ic in e , B o s to n , M a s s a c h u s e tts

Thomas M. Halaszynski, DMD, MD, MBA


A s s o c ia te P r o f e s s o r o f A n e s th e s io lo g y , D ir e c to r o f R e g io n a l A n e s th e s ia /A c u te P a in M e d ic in e ,
D e p a r tm e n t o f A n e s th e s io lo g y , Y a le U n iv e r s ity S c h o o l o f M e d ic in e , Y a le N e w H a v e n H o s p ita l,
N e w H a v e n , C o n n e c tic u t

D arrin J. H yatt, MD
A n e s th e s ia C h ie f R e s id e n t, D e p a r tm e n t o f A n e s th e s ia , C r itic a l C a r e , a n d P a in M e d ic in e , M a s s a c h u s e tts
G e n e r a l H o s p ita l, B o s to n , M a s s a c h u s e tts

Daniel W. Johnson, MD
A s s is ta n t P r o f e s s o r , F e llo w s h ip D ir e c to r , C r itic a l C a r e A n e s th e s io lo g y , D e p a r tm e n t o f
A n e s th e s io lo g y , U n iv e r s ity o f N e b r a s k a M e d ic a l C e n te r, O m a h a , N e b r a s k a

Rebecca Kalman, MD
C lin ic a l I n s tr u c to r in A n e s th e s ia , M a s s a c h u s e tts G e n e r a l H o s p ita l, B o s to n , M a s s a c h u s e tts

Jean Kwo, MD
A n e s th e s io lo g is t, D e p a r tm e n t o f A n e s th e s ia , C r itic a l C a r e , a n d P a in M e d ic in e , M a s s a c h u s e tts G e n e r a l
H o s p ita l, A s s is ta n t P r o f e s s o r o f A n a e s th e s ia , H a r v a r d M e d ic a l S c h o o l, B o s to n , M a s s a c h u s e tts
Jinlei Li, MD
A s s is ta n t P r o f e s s o r o f A n e s th e s io lo g y , Y a le U n iv e r s ity S c h o o l o f M e d ic in e , Y a le N e w H a v e n
H o s p ita l, N e w H a v e n , C o n n e c tic u t

Dipty Mangla, MD
S ta ff A n e s th e s io lo g is t, C u m b e r la n d P a in M a n a g e m e n t, C u m b e r la n d , M a r y la n d

Ala Nozari, MD
A s s is ta n t P r o f e s s o r , D e p a r tm e n t o f A n e s th e s ia , C r itic a l C a r e , a n d P a in M e d ic in e , M a s s a c h u s e tts
G e n e r a l H o s p ita l, B o s to n , M a s s a c h u s e tts

Thoha M. Pham, MD
A s s o c ia te C lin ic a l P r o f e s s o r , U n iv e r s ity o f C a lif o r n ia , S a n F r a n c is c o (U C S F ), D e p a r tm e n t o f
A n e s th e s ia a n d P e r io p e r a tiv e C a r e , S a n F r a n c is c o , C a l if o r n ia

Manish Purohit, MD
D e p a r tm e n t o f A n e s th e s io lo g y a n d P e r io p e r a tiv e M e d ic in e , D r e x e l U n iv e r s ity C o lle g e o f M e d ic in e ,
P h ila d e lp h ia , P e n n s y lv a n ia

Paul Sikka, MD, PhD


D e p a r tm e n t o f A n e s th e s ia a n d P e r io p e r a tiv e M e d ic in e , S ig n a tu r e H e a lth c a r e B r o c k to n H o s p ita l,
B r o c k to n , M a s s a c h u s e tts , A f f ilia te o f B e th I s r a e l D e a c o n e s s M e d ic a l C e n te r, B o s to n , M a s s a c h u s e tts
( F o r m e r F a c u lty — B r ig h a m a n d W o m e n ’s H o s p ita l, H a r v a r d M e d ic a l S c h o o l)

Ashish C. Sinha, MD, PhD, DABA


V ic e C h a ir m a n , A n e s th e s io lo g y & C r itic a l C a r e , D r e x e l U n iv e r s ity C o lle g e o f M e d ic in e , H a h n e m a n n
U n iv e r s ity H o s p ita l, P h ila d e lp h ia , P e n n s y lv a n ia

Preet Mohinder Singh, MD


D e p a r tm e n t o f A n e s th e s ia , A ll In d ia In s titu te o f M e d ic a l S c ie n c e s , N e w D e lh i, In d ia

David L. Stahl, MD
C lin ic a l F e llo w , D e p a r tm e n t o f A n e s th e s ia , C r itic a l C a r e a n d P a in M e d ic in e , M a s s a c h u s e tts G e n e r a l
H o s p ita l, B o s to n , M a s s a c h u s e tts

Deppu Ushakumari, MD
D e p a r tm e n t o f A n e s th e s io lo g y a n d P e r io p e r a tiv e M e d ic in e , D r e x e l U n iv e r s ity C o lle g e o f M e d ic in e ,
P h ila d e lp h ia , P e n n s y lv a n ia
PREFACE

T h e p r a c tic e o f a n e s th e s io l o g y r e q u ir e s a s o lid f o u n d a tio n o f k n o w le d g e . It is w ith e x tr e m e p le a s u r e


th a t w e in tr o d u c e Lippincott’s Anesthesia Review: 1,001 Questions and Answers. T h e b o o k is d e s ig n e d
to r a p id ly r e v ie w a n e s th e s io l o g y to h e lp r e s id e n ts p a s s th e w r itte n e x a m in a tio n s ta k e n d u r in g a n d
a fte r re s id e n c y . T h e b o o k is b r o a d l y d iv id e d in to 21 c h a p te rs to c o v e r a lm o s t a ll r e le v a n t to p ic s
te ste d . E a c h q u e s tio n is f o ll o w e d b y f o u r p o s s ib le a n s w e rs , a m o n g w h ic h o n e is th e b e s t o r m o s t
lik e ly a n s w e r.
T h e e d ito r s a c k n o w le d g e th e w o r k o f a ll w h o h a v e g iv e n th e ir v a lu a b le tim e a n d e f f o r t to
c o m p le te th is b o o k . T h e s e in c lu d e a ll a u th o r s , p r o o f r e a d e r s ( in c lu d in g S h ilp a S h a h , M D ), a n d th e
te a m a t L ip p in c o tt W illia m s & W ilk in s . W e w o u ld a ls o lik e to th a n k o u r f a m ilie s f o r th e ir s u p p o r t
w h ile w e p r e p a r e d th is m a n u s c r ip t.
W e h o p e th a t th is r e v ie w b o o k p r o v e s to b e a v a lu a b le e d u c a tio n a l r e s o u r c e f o r a n e s th e s ia
r e s id e n ts a n d y o u n g p r a c titio n e r s to h e lp th e m p a s s th e b o a r d s . F o r a n y c o n s tr u c tiv e s u g g e s tio n s ,
p le a s e c o n ta c t u s b y e m a il: A n e s 1 0 0 1 @ o u tlo o k .c o m .
The Editors
CONTENTS

Contributors
Preface

1. Perioperative Evaluation and Management


PREET SINGH, MANISH PUROHIT, ASHISH SINHA, AND PAUL SIKKA

2. Airway Management
YURIY BRONSHTEYN AND EDWARD BITTNER

3. Anesthesia Machine
PAUL SIKKA

4. Patient Monitoring
DARREN HYATT, ALA NOZARI, AND EDWARD BITTNER

5. Fluid Management and Blood Transfusion


REBECCA KALMAN AND EDWARD BITTNER

6. Anesthetic Pharmacology
MIAN AHMAD AND ASHISH SINHA

7. Spinal and Epidural Anesthesia


THOMAS HALASZYN SKI

8. Peripheral Nerve Blocks


THOMAS HALASZYN SKI

9. Pain Management
THOMAS HALASZYN SKI

10. Orthopedic Anesthesia


THOMAS HALASZYN SKI

11. Cardiovascular Anesthesia


DEPPU USHAKUMARI AND ASHISH SINHA
12 . Thoracic Anesthesia
DEPPU USHAKUMARI AND ASHISH SINHA

13 . Neuroanesthesia
DIPTY MANGLA AND ASHISH SINHA

14 . Gastrointestinal, Liver, and Renal Diseases


THOHA PHAM

15 . Endocrine Diseases
JEAN KWO AND EDWARD BITTNER

16 . Ophthalmic, Ear, Nose, and Throat Surgery


THOHA PHAM

17 . Obstetric Anesthesia
THOHA PHAM

18 . Pediatric Anesthesia
DIPTY MANGLA AND ASHISH SINHA

19 . Critical Care
DAVID STAHL, DANIEL JOHNSON, AND EDWARD BITTNER

20 . Postoperative Anesthesia Care


SHERI BERG AND EDWARD BITTNER

21 . Miscellaneous Topics
PAUL SIKKA AND THOMAS HALASZYN SKI
Perioperative Evaluation and Management
Preet Singh, Manish Purohit, Ashish Sinha, and Paul Sikka

1. Preoperative application of scopolamine patch to prevent postoperative nausea and vomiting


should be avoided in
A. Female, 35 years old
B. Smoker, 20 years old
C. Patient with a blood pressure of 160/96 mm Hg
D. Male, 70 years old

2. Which of the following drugs is least likely to be effective for prophylaxis for postoperative
nausea and vomiting?
A. Ondansetron
B. Scopolamine patch
C. Aprepitant
D. Metoclopramide

3. Famotidine, when used for stress ulcer prophylaxis, must be avoided preoperatively in which of
the following patients?
A. Patients with replaced mitral valve on warfarin
B. Patients with idiopathic thrombocytopenic purpura (ITP) for splenectomy
C. Patients with achalasia cardia for esophageal myotomy
D. Patients with a history of coronary stenting on aspirin

4. Which of the following drugs antagonizes substance P in the central nervous system and is used
as premedication to prevent postoperative nausea and vomiting?
A. Palonosetron
B. Aprepitant
C. Metoclopramide
D. Prochlorperazine

5. Which of the following predictors is likely to be associated with lower incidence of


perioperative nausea and vomiting?
A. Female gender
B. Use of fentanyl for pain relief
C. Patients with a history of smoking
D. Patients undergoing laparoscopic surgery

6. All of the following have an antiemetic action, except


A. Promethazine
B. Propofol
C. Etomidate
D. Haloperidol

7. Cefazolin, as a component of perioperative antimicrobial prophylaxis for surgery, must begin


within what time before incision?
A. Simultaneously with incision
B. Within 30 minutes prior to incision
C. Within 60 minutes prior to incision
D. Within 120 minutes prior to incision

8. Vancomycin, as a component of perioperative antimicrobial prophylaxis for surgery, must


begin within what time before incision?
A. Simultaneously with incision
B. Within 30 minutes prior to incision
C. Within 60 minutes prior to incision
D. Within 120 minutes prior to incision

9. A 65-year-old male with a history of hypertension and diabetes presents to emergency


department with altered sensation with a likely subdural hematoma. To assess his
cardiorespiratory status, he is asked about his level of physical activity. If he is capable of
performing at least which of the following activities independently, he is less likely to have
significant cardiopulmonary ailment during surgery?
A. Walk to washroom on level floor
B. Play the accordion
C. Walk one block
D. Climb a flight of stairs

10. In preoperative assessment of patients, physical activity is graded in terms of metabolic


equivalents (METs). The value that corresponds to oxygen consumption of 1 MET in an adult is
A. 2 mL/kg/min
B. 7 mL/kg/min
C. 3.5 mL/kg/min
D. 5.5 mL/kg/min
11. As per American Society of Regional Anesthesia (ASRA) guidelines, intravenous infusion of
unfractionated heparin should be stopped how long prior to a planned epidural?
A. 1 to 1.5 hours
B. 2 to 4 hours
C. at least 12 hours
D. at least 24 hours

12. For emergent surgery, anticoagulation produced by warfarin can be reversed by using
A. Fresh-frozen plasma (FFP)
B. Injectable vitamin K
C. Prothrombin complex concentrate
D. Factor VIII concentrate

13. Neuraxial block is not contraindicated for patients on which of the following drugs?
A. Warfarin
B. Low-molecular-weight heparin
C. Aspirin
D. Clopidogrel

14. All of the following are risk factors for obstructive sleep apnea, except
A. Obesity
B. Short neck
C. Enlarged tonsils
D. Female gender

15. A 70-year-old male, who is diabetic for the last 20 years, is scheduled for an elective surgery.
Which of the following is not a sign of autonomic diabetic neuropathy?
A. History of recurrent diarrhea
B. History of postural hypotension
C. History of recurrent constipation
D. History of urinary retention

16. Which of the following perioperative factors in patients undergoing dialysis prior to surgery
predicts the possibility of hypotension (due to increased volume removed)?
A. Change in serum sodium
B. Change in body weight
C. Change in serum potassium
D. Change in pH after dialysis

17. A patient with a history of severe asthma is scheduled for an appendectomy. Which of the
following induction agents will cause the least respiratory depression?
A. Ketamine
B. Propofol
C. Etomidate
D. Thiopental

18. Which of the following drugs can significantly prolong the QT interval on the ECG?
A. Dexamethasone
B. Droperidol
C. Aprepitant
D. Glycopyrrolate

19. Which of the following tests is used to confirm coagulation after stopping low-molecular-
weight heparin (LMWH)?
A. PT
B. aPTT
C. ACT
D. None of the above

20. Effect of combined administration of midazolam and fentanyl is


A. Additive
B. Synergistic
C. Competitively antagonistic
D. Noncompetitively antagonistic

21. Preoperative anesthetic evaluation is likely to bring down the incidence of all the following,
except
A. Case cancellations
B. Patient morbidity
C. Preoperative anxiety
D. Direct procedural costs

22. For elective procedures, an anesthesia provider must obtain informed and preferably written
consent
A. Just prior to transferring the patient to the operating room for surgery
B. During preoperative anesthetic evaluation
C. At the same time that a surgeon obtains consent for the surgical procedure
D. Just prior to induction of anesthesia in the operating room

23. An optimal preoperative evaluation is designed


A. To screen for and properly manage comorbid conditions
B. To assess the risk of anesthesia and surgery and lower it
C. To identify patients who may require special anesthetic techniques or postoperative care
D. All the above

24. ASA classification for risk stratification is validated for predicting preoperative morbidity
associated with the following, except
A. General or regional anesthesia
B. Conscious sedation
C. Monitored anesthesia care
D. Surgical procedure

25. A healthy pregnant patient in labor has which of the following ASA classifications?
A. I
B. II
C. III
D. IV

26. Sedatives, as premedication, must be avoided in which of the following patients?


A. Uncontrolled hypertensive
B. Toddler for tonsillectomy
C. Brain tumor patients
D. Patients with alcohol abuse

27. As per the American Society of Regional Anesthesia (ASRA) guidelines, which of the
following drugs can be continued preoperatively in patients planned for neuraxial blockade for
an elective procedure?
A. Aspirin
B. Clopidogrel
C. Warfarin
D. Low-molecular-weight heparin

28. As per ASA classification, a controlled hypertensive patient with no target end-organ damage
scheduled for elective surgery will be classified as
A. ASA I
B. ASA II
C. ASA III
D. ASA VI

29. A brain-dead organ donor undergoing laparotomy for “kidney harvesting” will be classified as
an
A. ASA III
B. ASA IV
C. ASA V
D. ASA VI

30. A moribund patient who is not expected to survive without the operation is categorized as an
A. ASA III
B. ASA IV
C. ASA V
D. ASA VI

31. A patient with a history of uncontrolled hypertension, diabetes, and angina, who is to undergo a
laparoscopic cholecystectomy, will be classified as an
A. ASA II
B. ASA III
C. ASA IV
D. ASA V

32. A 65-year-old male with a history of mitral valve replacement 2 years back presents for a knee
replacement. He is on warfarin since the time of valve replacement. As per ASRA guidelines,
the ideal time to stop his warfarin prior to surgery would be
A. 12 hours
B. 3 days
C. 5 days
D. 10 days

33. A 26-year-old female, with a history of rheumatic mitral stenosis, is scheduled for an elective
cesarean section at 38 weeks of gestation. Just prior to surgery, she is diagnosed to have atrial
fibrillation (AF) with no hemodynamic instability. The first step in preparation for surgery is
A. Perform an echocardiogram to rule out left-atrial clot
B. Synchronized DC cardioversion under sedation
C. Antiarrhythmic medication
D. Plan for therapy postdelivery

34. A 72-year-old patient with a history of hypertension and angina at moderate activity is to
undergo a laparoscopic cholecystectomy. Due to decreased effort tolerance and a significant
blockade of left anterior descending coronary artery onstress thallium, a preprocedure
coronary intervention is planned. Which of the following procedures performed prior to the
elective surgery is least likely to delay the laparoscopic surgery?
A. Coronary artery bypass graft (CABG)
B. Percutaneous coronary stenting—bare-metallic stent
C. Percutaneous coronary stenting—drug-eluting stent
D. Percutaneous balloon dilatation
35. Which of the following is not seen as a result of primary renal disease in patients with chronic
renal failure?
A. Hypocoagulable state
B. Hypercoagulable state
C. Hyper proteinemia
D. Anemia

36. A 2-year-old child is to undergo a tonsillectomy. The child had formula milk 2 hours ago. As
per ASA guidelines, optimal NPO status would be to wait another_____before proceeding to
surgery:
A. No waiting, since it is a child
B. 2 hours
C. 4 hours
D. 6 hours

37. A 45-year-old patient is scheduled for an abdominal hysterectomy. She states that her aunt had a
severe reaction to anesthesia and was in the ICU for 1 week. You would avoid which of the
following drugs for her general anesthesia?
A. Droperidol
B. Ketamine
C. Sevoflurane
D. Etomidate

38. Elective surgery should be postponed after a myocardial infarction for at least
A. 30 days
B. 6 weeks
C. 3 months
D. 6 months

39. The most significant risk factor for developing pulmonary complications is
A. Site of surgery (abdominal/thoracic)
B. Presence of respiratory infection
C. Presence of obstructive sleep apnea
D. Smoking

40. Maximum international normalized ratio (INR) before proceeding for elective surgery should
be
A. 1.0
B. 1.2
C. 1.4
D. 1.6
41. A 73-year-old patient has residual weakness on the right arm and leg following a stroke 5 years
ago. He is now scheduled for laparoscopic cholecystectomy under general anesthesia. Which of
the following sites should be preferably used to monitor the train of four muscle twitches for
estimating neuromuscular blockade?
A. Right ulnar nerve-innervated muscles
B. Right posterior tibial nerve-innervated muscles
C. Left ulnar nerve-innervated muscles
D. Left facial nerve

42. A 32-year-old patient after being involved in a road traffic accident due to alcohol intoxication
is taken to the operating room for open fracture reduction of an ankle fracture. His blood
alcohol level is above the legal limit. Compared to a patient who is not intoxicated with alcohol,
you would expect the minimum alveolar concentration (MAC) of sevoflurane to be
A. Higher
B. Lower
C. Equal
D. Unpredictable due to pharmacodynamic variations

43. A 55-year-old patient with a history of asthma and heart failure is to undergo a hernia repair.
On physical examination, you notice that the patient is wheezing. Following treatment with
albuterol, the patient should be monitored for which electrolyte?
A. Potassium
B. Calcium
C. Sodium
D. Chloride

44. Smoking cessation for 24 hours before a scheduled surgery will lead to
A. Improvement of ciliary function
B. Decrease in mucous production
C. Decrease in airway irritability
D. Decrease in level of carboxyhemoglobin

45. Which of the following tests is likely to detect clinically relevant bleeding tendency most
efficiently?
A. Activated partial thromboplastin time
B. Prothrombin time
C. Activated clotting time
D. Thromboelastogram (TEG)

46. As per AHA guidelines, which of the following is not a major clinical risk predictor in a patient
with cardiac disease scheduled for noncardiac surgery?
A. Recent myocardial infarction
B. Symptomatic mitral stenosis
C. Presence of congestive cardiac failure
D. Uncontrolled systolic hypertension

47. Glycopyrrolate, when given preoperatively, can cause all of the following, except
A. Skin flushing
B. Dry mouth
C. Bronchoconstriction
D. Tachycardia

48. Which of the following is true about metoclopramide?


A. Decreases lower esophageal sphincter tone
B. Delays gastric emptying
C. Can cause extrapyramidal side effects
D. Useful in preventing postoperative nausea

49. Which of the following occurs during the preoxygenation of a patient?


A. Increase in functional residual capacity
B. Denitrogenation
C. Increase in CO2 clearance from lungs
D. Increase in closing capacity of lungs

50. Which of the following agents is associated with the highest incidence of hepatitis
postoperatively?
A. Halothane
B. Isoflurane
C. Desflurane
D. Sevoflurane

51. The inhalation agent of choice in a 2-year-old child for ophthalmologic surgery is
A. Halothane
B. Desflurane
C. Sevoflurane
D. Nitrous oxide

52. Which of the following is true of nitrous oxide?


A. Acts on central nervous system GABA receptors
B. Lowers pulmonary vascular resistance
C. Suppresses EEG pattern in the cerebral cortex
D. Precipitates vitamin B12 deficiency anemia
53. The antiemetic effect of propofol is thought to occur due to
A. Depressant effect on the chemoreceptor trigger zone
B. Inhibition of dopamine activity
C. Inhibition of glutamate release
D. All of the above

54. Which of the following is the preferred intravenous agent of induction of anesthesia for
maintaining spontaneous breathing and airway tone?
A. Midazolam
B. Propofol
C. Ketamine
D. Diazepam

55. Succinylcholine is contraindicated in a patient with


A. Chronic renal failure
B. Duchene muscular dystrophy
C. Myasthenia gravis
D. Patient with full stomach

56. A 75-year-old patient with a history of hypertension is to undergo laparoscopic colectomy for
carcinoma colon. Continuing of which of the following antihypertensive drugs, preoperatively,
in the geriatric age group, can be associated with profound hypotension on induction of general
anesthesia?
A. Metoprolol
B. Angiotensin-converting-enzyme (ACE) inhibitors
C. Hydrochlorothiazide
D. Furosemide

57. Which of the following findings in the preoperative evaluation cannot be attributed to obesity
with obstructive sleep apnea (OSA) in a patient planned for bariatric surgery?
A. Pulmonary artery hypertension
B. Congestive heart failure
C. Peripheral neuropathy
D. Dementia

58. All of the following medications can be administered via an epidural anesthesia, except
A. Fentanyl
B. Sufentanil
C. Alfentanil
D. Remifentanil
59. Ondansetron causes its antiemetic effect by acting as an
A. Agonist at 5-HT2 receptors
B. Antagonist at 5-HT2 receptors
C. Agonist at 5-HT3 receptors
D. Antagonist at 5-HT3 receptors

60. Which of the following statements is false regarding scopolamine patch applied
preoperatively?
A. May produce sedation
B. Decreases the risk of nausea
C. Adds to the analgesia
D. Inhibits muscarinic receptors

61. Overdose with dexmedetomidine results in


A. Hypertension
B. Bradycardia
C. Hypertension and bradycardia
D. Hypotension and bradycardia

62. Abrupt withdrawal of steroids can lead to


A. Malignant hypertension
B. Sickle cell crisis
C. Addisonian crisis
D. Psychosis

63. Promethazine primarily inhibits which of the following receptors?


A. Serotonin
B. Dopamine
C. Muscarinic
D. Acetylcholine

64. All of the following surgeries are associated with an increased risk of postoperative nausea and
vomiting, except
A. Shoulder arthroscopy
B. Laparoscopic surgery
C. Strabismus repair
D. Tympanoplasty

65. Abrupt stoppage of total parenteral nutrition (TPN) would most likely cause
A. Hypoglycemia
B. Hyperglycemia
C. Hyperphosphatemia
D. Hypophosphatemia

66. Glycopyrrolate causes all of the following, except


A. Sedation
B. Tachycardia
C. Antisialagogue effect
D. Lowers lower esophageal sphincter tone

67. In general, herbal medications should be stopped before surgery for at least_____days:
A. 3
B. 7
C. 10
D. 14

68. Which of the following antibiotics can prolong the action of neuromuscular-blocking drugs?
A. Gentamicin
B. Penicillin
C. Levofloxacin
D. Cephalexin

69. Estrogen in birth control pills increases the perioperative risk of


A. Diarrhea
B. Thromboembolism
C. Stroke
D. Myocardial infarction

70. A 42-year-old patient is scheduled for a hernia repair under general anesthesia. His medications
include fluoxetine, alprazolam, and lithium for bipolar disorder. In the preoperative area, he
appears confused, has tremors, and is ataxic. Your next step would be to
A. Cancel the case
B. Proceed with the case
C. Order a lithium blood level
D. Consult a psychiatrist

71. A 34-year-old patient is to undergo an appendectomy under general anesthesia. He is taking a


monoamine oxidase inhibitor (MAOI) for depression. Intraoperatively, his blood pressure
drops to 72/36 mm Hg and a medication is administered. His blood pressure suddenly increases
to 220/120 mm Hg. The most likely medicine that was administered is
A. Ephedrine
B. Meperidine
C. Phenylephrine
D. Norepinephrine

72. All of the following are true about diabetic patients, except
A. Patients should take half or one-third of their insulin dose the morning of the surgery
B. Patients should continue their oral hypoglycemic agents the morning of the surgery
C. Finger-stick blood glucose should be tested before taking the patient to the operating room
D. Patient with an insulin pump should continue the insulin at their basal rate

73. Digoxin toxicity is most likely exacerbated by


A. Hyperkalemia
B. Hypokalemia
C. Hypercalcemia
D. Hypocalcemia

74. The most common complication of inserting a central venous catheter is


A. Carotid artery puncture
B. Thrombosis
C. Cardiac arrhythmias
D. Air embolism

75. A patient is administered cephalexin preoperatively. Within 5 minutes of starting the antibiotic,
the patient starts to wheeze and develops tachycardia, and the blood pressure drops to 78/42 mm
Hg. Your next step would be to administer
A. Ephedrine
B. Phenylephrine
C. Epinephrine
D. Oxygen

76. All of the following may occur with an interscalene block, except
A. Subarachnoid injection
B. Radial nerve blockade
C. Median nerve blockade
D. Ulnar nerve blockade

77. An axillary nerve block would not produce loss of sensation of the
A. Lateral aspect of the forearm
B. Medial aspect of the forearm
C. The entire forearm
D. None of the above
78. The femoral nerve lies
A. Medial to the femoral artery
B. Anterior to the femoral artery
C. Posterior to the femoral artery
D. Lateral to the femoral artery

79. All of the following nerves are blocked by an ankle block, except
A. Sural
B. Superficial peroneal
C. Deep peroneal
D. Anterior tibial

80. Sore throat is


A. More common after using an endotracheal tube
B. More common after using a laryngeal mask airway
C. Similar incidence with either endotracheal tube or a laryngeal mask airway
D. More common after using an oral airway

81. A patient with hypertrophic obstructive cardiomyopathy (HOCM) presents with dyspnea and
angina on exertion. Which of the following is the best agent to treat these symptoms?
A. Hydrochlorothiazide
B. Metoprolol
C. Morphine
D. Nitroglycerin

82. St. John wort (Hypericum perforatum) potentiates the effects of


A. Heparin
B. Warfarin
C. Aspirin
D. Clopidogrel

83. The most powerful predictor of atrial fibrillation post-cardiac surgery is


A. History of diabetes
B. History of hypertension
C. Age
D. Time on bypass

84. A patient with Parkinson disease undergoes a general anesthetic. Your plan to treat his nausea
would include all of the following, except
A. Dexamethasone
B. Scopolamine patch
C. Metoclopramide
D. Ondansetron

85. A 65-year-old patient is being treated for congestive cardiac failure. He is able to take a shower
but gets dyspneic on mowing the lawn. His New York Heart Association classification is
A. Class 1
B. Class 2
C. Class 3a
D. Class 3b

86. The percentage of postdural puncture headaches that would resolve spontaneously by 1 week is
approximately
A. 30%
B. 50%
C. 50%
D. 70%

87. A 46-year-old lady is seen at the preoperative assessment clinic. She is taking 180 mg/day
methadone. The most likely change to be found in her preoperative ECG is
A. Prolonged PR interval
B. Prolonged QTc
C. U wave
D. Tented T-waves

88. You are about to anesthetize a 55-year-old man who is undergoing liver resection for removal
of metastatic carcinoid tumor. The drug of choice to treat intraoperative hypotension is
A. Octreotide
B. Dobutamine
C. Milrinone
D. Vasopressin

89. You are performing an interscalene brachial plexus block on an awake 40-year-old patient who
is healthy with no significant medical history. Soon after injecting 20 mL of 0.25% bupivacaine
the patient becomes agitated, has a seizure, and loses consciousness. Your first step in
management is
A. Administer intralipid
B. Administer midazolam or propofol to control the seizure
C. Establish airway and give 100% O2 via a face mask
D. Administer epinephrine
90. Patients with dilated cardiomyopathy exhibit all of the following, except
A. Decreased myocardial contractility
B. Afterload should be maximized
C. Increased preload
D. Left ventricular hypertrophy

91. A septic patient has a central venous pressure of 10 mm Hg, a blood pressure of 80/40 mm Hg,
and a pulse rate of 96 beats/min. The best agent to treat the hypotension is
A. Dopamine
B. Dobutamine
C. Noradrenaline
D. Epinephrine

92. Which of the following organs is least tolerant of ischemia for removal for transplantation?
A. Cornea
B. Heart
D. Kidney

E. Pancreas
93. You have administered a patient 1.2 mg/kg of rocuronium to do an intubation. You are unable to
intubate or ventilate the patient and decide to reverse the patient’s paralysis with sugammadex.
The dosage you would use is
A. 2 mg/kg
B. 4 mg/kg
C. 8 mg/kg
D. 16 mg/kg

94. A young female patient with anorexia nervosa has just started eating again. After 4 days, she
develops dyspnea and is found to have cardiac failure. Which of the following is most
important to correct?
A. Potassium
B. Phosphate
C. Glucose
D. Sodium

95. A pregnant lady is to undergo general anesthesia for acute appendicitis. At what gestational age
should you monitor fetal heart rate?
A. 16 weeks
B. 18 weeks
C. 24 weeks
D. 28 weeks
96. Which of the following is the best predictor of a difficult intubation in a morbidly obese
patient?
A. Pretracheal tissue volume
B. Body mass index
C. Mallampati score
D. Thyromental distance

97. A patient with a history of chronic obstructive pulmonary disease presents for lung volume-
reduction surgery. Which of the following is a contraindication for surgery?
A. Age >60 years
B. Chronic asthma
C. FEV <25%
D. Evidence of bullous disease

98. All of the following help increase the excretion of calcium, except
A. Bisphosphonates
B. Calcitonin
C. Furosemide
D. IV crystalloids

99. Which of the following is contraindicated to use during pregnancy?


A. Aspirin
B. Enalapril
C. Metoprolol
D. Hydralazine

100. During scoliosis surgery, monitoring of somatosensory-evoked potentials indicates


monitoring of
A. Anterior horn
B. Anterior corticospinal tract
C. Dorsal column
D. Spinothalamic tract

101. The desflurane vaporizer is heated because of desflurane’s


A. High vapor pressure
B. High boiling point
C. High minimum alveolar concentration
D. High volatility

102. Which of the following is the most effective way to reduce renal failure in a patient having an
abdominal aortic aneurysm repair?
A. F lu id b o lu s p r i o r to a o r tic c la m p in g
B. F lu id b o lu s a fte r a o r tic c la m p r e le a s e
C. A d m in is tr a tio n o f m a n n ito l
D. M in im iz a tio n o f c r o s s - c la m p tim e
CHAPTER 1 ANSWERS

1. D. S c o p o la m in e , a n a n tic h o lin e r g ic d r u g , is o f te n a p p lie d a s a tr a n s d e r m a l p a tc h


p r e o p e r a t iv e ly f o r th e p r e v e n tio n o f p o s to p e r a tiv e n a u s e a a n d v o m itin g . H o w e v e r, lik e a tr o p in e ,
a n d u n lik e g ly c o p y r r o la te , s c o p o la m in e p a s s e s th r o u g h th e b l o o d - b r a i n b a r r i e r a n d c a n c a u s e
c o n f u s io n , e s p e c ia lly in th e e ld e r ly . H e n c e , a p p lic a tio n o f s c o p o la m in e p a tc h s h o u ld b e a v o id e d
in th e e ld e r ly . T r e a tm e n t o f s c o p o la m in e - in d u c e d c o n f u s io n m a y r e q u i r e a d m in is tr a tio n o f
p h y s o s tig m in e .

2. D. M e to c lo p r a m id e is a p r o k in e tic a g e n t a n d h e lp s to in c r e a s e g a s tr ic m o tility . T h e A S A d o e s
n o t r e c o m m e n d p r e o p e r a t iv e a d m in is tr a tio n o f m e to c lo p r a m id e f o r p r e v e n tio n o f p o s to p e r a tiv e
n a u s e a a n d v o m itin g . A ll th e o th e r a g e n ts h a v e p r o v e n b e n e f it in p r e v e n tin g p o s to p e r a tiv e
n a u s e a a n d v o m itin g .

3. B. F a m o tid in e is k n o w n to c a u s e th r o m b o c y to p e n ia ( b o th q u a n tita tiv e a n d q u a lita tiv e p la te le t


d y s f u n c tio n ) . P a tie n ts w ith IT P a lr e a d y h a v e l o w p la te le ts ; th u s , s u c h p r e m e d ic a tio n s h o u ld b e
a v o id e d . W a r f a r in d o e s n o t a f f e c t p la te le t f u n c tio n o r n u m b e r, th u s h a s n o r e la tio n to
p e r io p e r a tiv e b le e d in g d u e to p la te le t p a th o lo g y ; h o w e v e r, it is a n in d e p e n d e n t r i s k f a c to r f o r
b le e d in g .

4. B. A p r e p ita n t is a n N K 1 r e c e p to r a n ta g o n is t th a t a n ta g o n iz e s th e a c tio n o f s u b s ta n c e P in th e
c e n tr a l n e r v o u s s y s te m to p r e v e n t n a u s e a a n d v o m itin g . P a lo n o s e t r o n is a 5 - H T 3 a n ta g o n is t,
m e to c lo p r a m id e is a n a n tid o p a m in e r g ic a g e n t, a n d p r o c h l o r p e r a z i n e is a d o p a m in e (D 2)
r e c e p to r a n ta g o n is t ( a n tip s y c h o tic d r u g ) w ith a d d itio n a l a n tie m e tic a c tiv ity .

5. C. T h e A p fe l s c o r e c a n b e u s e d to p r e d ic t p a tie n ts w ith a h ig h r i s k f o r p e r io p e r a tiv e n a u s e a


a n d v o m itin g (P O N V ). It in c lu d e s f o u r f a c to r s : f e m a le g e n d e r , n o n s m o k in g , p o s to p e r a tiv e u s e
o f o p io id s , a n d p r e v io u s P O N V o r m o tio n s ic k n e s s in th e p a tie n ts ’ h is to r y . S u r g e r ie s lik e
la p a r o s c o p y , m id d le - e a r s u r g e r y , a n d s tr a b is m u s s u r g e r y a r e a s s o c ia te d w ith a h ig h e r r i s k o f
PO N V .

6. C. E to m id a te a d m in is tr a tio n c a n c a u s e a n in c r e a s e in th e in c id e n c e o f p e r io p e r a tiv e n a u s e a a n d
v o m itin g (P O N V ). P r o m e th a z in e , h a lo p e r id o l, a n d p r o p o f o l a ll a r e u s e d in th e tr e a tm e n t o f
P O N V . T h e la tte r tw o a r e u s u a lly u s e d f o r th e tr e a tm e n t o f r e f r a c t o r y P O N V .

7. C. P -L a c ta m a n tib io tic s m u s t b e g iv e n w ith in 6 0 m in u te s p r i o r to in c is io n . V a n c o m y c in a n d


f l u o r o q u in o lo n e s r e q u i r e a d m in is tr a tio n w ith in 1 2 0 m in u te s p r i o r to in c is io n .

8. D. V a n c o m y c in a n d f l u o r o q u in o lo n e s r e q u i r e a d m in is tr a tio n w ith in 1 2 0 m in u te s p r i o r to
in c is io n . P -L a c ta m a n tib io tic s m u s t b e g iv e n w ith in 6 0 m in u te s p r i o r to in c is io n .
9. D. E f f o r t to le r a n c e o f a r o u n d 4 M E T s ( m e ta b o lic e q u iv a le n t o f ta s k s ) o r m o r e is s u g g e s te d to
b e a g o o d p r e d ic to r f o r p o s to p e r a tiv e c a r d io p u lm o n a r y o u tc o m e . T h e s e a c tiv itie s a r e c la s s if ie d
a s p e r p h y s ic a l s tr a in in v o lv e d .

10. C. O n e m e ta b o lic e q u iv a le n t is d e f in e d a s th e a m o u n t o f o x y g e n c o n s u m e d a t re s t, a n d is e q u a l
to 3.5 m L O 2/k g /m in . T h e e n e r g y c o s t o f a n y a c tiv ity c a n b e d e te r m in e d b y m u ltip ly in g 3.5 to
th e o x y g e n c o n s u m p tio n (m L O 2/k g /m in ) . M E T s c a n b e a s s e s s e d a s f o llo w s :
• 1 M E T — c a n ta k e c a r e o f s e lf (e a tin g , d r e s s in g , to ile t)
• 4 M E T s — c a n w a lk u p a f li g h t o f s te p s o r a h ill
• 4 to 10 M E T s — c a n d o h e a v y h o u s e h o ld w o r k ( s c r u b b in g f l o o r s , lif tin g h e a v y f u r n itu r e )
• > 1 0 M E T s — c a n p a r tic ip a te in s tr e n u o u s s p o r ts (s w im m in g , te n n is , b a s k e tb a ll, s k iin g )

11. B. A s p e r A S R A g u id e lin e s 2 0 1 0 , h e p a r in in f u s io n s h o u ld b e s to p p e d a t le a s t 2 to 4 h o u r s
b e f o r e p la c in g a n e p id u r a l. T h is is to p r e v e n t th e p o te n tia l f o r m a ti o n o f a n e p id u r a l h e m a to m a .

12. A. A s p e r th e A H A /A C C S c ie n tific S ta te m e n t, r e v e r s a l o f w a r f a r i n c a n b e a c h ie v e d b y u s in g
a ll, e x c e p t c h o ic e D . H o w e v e r, f o r e m e r g e n t s u r g e r y th e fa s te s t m e th o d is th e a d m in is tr a tio n o f
f r e s h - f r o z e n p la s m a . P e a k a c tio n o f in je c ta b le v ita m in K ta k e s u p to 6 to 12 h o u r s .

13. C. A s p e r A S R A g u id e lin e s (2 0 1 0 ), a s p ir in in ta k e b y th e p a tie n t is n o m o r e c o n s id e r e d a s a


c o n tr a in d ic a tio n to p e r f o r m i n g a n e u r a x ia l b lo c k .

14. D. T h e a s s e s s m e n t o f p r e o p e r a t iv e p r e d ic ta b ility f o r o b s tr u c tiv e s le e p a p n e a c a n b e d o n e b y


u s in g th e “ S T O P - B A N G ” q u e s tio n n a ir e . In th is s c o r in g , m a le g e n d e r , a n d n o t f e m a le g e n d e r , is
c la s s if ie d a s a r i s k f a c to r (S , s n o r in g ; T , tir e d d u r in g d a y tim e ; O , o b s e r v e d f o r a p n e a d u r in g
s le e p ; P, h ig h b l o o d p r e s s u r e ; B , B M I > 3 5 k g / m 2; A , a g e > 5 0 y e a r s ; N , n e c k c ir c u m f e r e n c e > 4 0
c m ; G , m a le g e n d e r ) . In a d d itio n to th e q u e s tio n n a ir e , u p p e r a ir w a y a n a to m ic a l a b n o r m a litie s
th a t in c r e a s e th e l i k e lih o o d o f o b s tr u c tio n a r e to n s illa r h y p e r tr o p h y , tu m o r s o f th e u p p e r
a irw a y , o r f a c io m a x illa r y a b n o r m a litie s .

15. D. A ll, e x c e p t c h o ic e D , a r e s ig n s o f d ia b e tic a u to n o m ic n e u ro p a th y . U r in a r y r e te n tio n a t th is


a g e is m o r e lik e ly d u e to p r o s ta te h y p e r tr o p h y .

16. B. W e ig h t lo s s d u e to d ia ly s is is a ttrib u te d to a c tu a l v o lu m e ( u ltr a f iltr a te ) r e m o v e d f r o m th e


b o d y . T h u s , a h ig h w e ig h t lo s s c a n p r e d ic t h ig h e r c ir c u l a t o r y v o lu m e lo s t, w h ic h c a n le a d to
p o o r c o m p e n s a tio n o f h y p o te n s io n in p a tie n ts u n d e r g o in g s u r g e r y .

17. A. K e ta m in e c a u s e s th e le a s t r e s p i r a t o r y d e p r e s s io n a m o n g th e in tr a v e n o u s in d u c tio n a g e n ts .
T h e r e f o r e , it m a y b e b e n e f ic ia l a s a n in d u c tio n a g e n t in p a tie n ts w ith s e v e r e a s th m a . H o w e v e r,
k e ta m in e c a u s e s a n in c r e a s e in s e c r e tio n s , a n d m a y p r o d u c e e m e r g e n c e d e lir iu m (v iv id
d r e a m s ) . P r e tr e a tm e n t w ith g ly c o p y r r o la te a n d m id a z o la m a lle v ia te s th e s e e ffe c ts o f k e ta m in e .
T h e o th e r in d u c tio n a g e n ts c a u s e d o s e - d e p e n d e n t r e s p i r a t o r y d e p r e s s io n .
18. B. D r o p e r id o l c a n c a u s e a s ig n if ic a n t p r o l o n g a t i o n o f th e Q T in te r v a l o n th e E C G . P a tie n ts
s h o u ld h a v e a p r e o p e r a t iv e E C G , a n d E C G m o n ito r in g s h o u ld b e c o n tin u e d p o s to p e r a ti v e ly f o r
a t le a s t 2 h o u r s , b e f o r e d is c h a r g in g th e p a tie n t.

19. D. A t p re s e n t, n o c o n v e n tio n a l te s t (P T , P T T ) c a n b e u s e d to q u a n tif y th e c lin ic a l e ffe c ts o f


L M W H o n th e c o a g u la tio n s y s te m . A n ti- F a c to r X a e s tim a tio n m a y b e u s e d in s p e c if ic p a tie n ts
to m o n ito r th e c o a g u la tiv e e ffe c ts o f L M W H .

20. B. A s th e s e d r u g s a c t o n d if f e r e n t r e c e p to r s , th e ir e ffe c ts a r e g e n e r a lly c o n s id e r e d to b e


s y n e r g is tic . P a tie n ts r e c e iv in g b o th th e s e d r u g s m a y b e p r o n e to g r e a te r s e d a tio n a n d
r e s p i r a t o r y d e p r e s s io n th a n w h e n r e c e iv in g th e d r u g a lo n e .

21. D. P r e o p e r a tiv e e v a lu a tio n in f a c t in c lu d e s a b a tte r y o f te sts a n d a d d s a d d itio n a l c o s ts to th e


to ta l p e r io p e r a tiv e c o s ts . H o w e v e r, p r e o p e r a t iv e e v a lu a tio n is v ita l, a s it r e c o g n iz e s p a tie n t
c o m o r b id iti e s , w h ic h c a n w o r s e n p e r io p e r a tiv e l y a n d c a u s e in c r e a s e d p a tie n t m o r b id ity .
P r e o p e r a tiv e e v a lu a tio n e v e n tu a lly lo w e r s in d ir e c t c o s ts th a t m a y b e in c u r r e d to tr e a t th e
w o r s e n i n g a lim e n t, p o s to p e r a tiv e ly . D u r in g p r e o p e r a t iv e in te r a c tio n , p a tie n t a n x ie ty is u s u a lly
lo w e r e d a s th e r is k s a n d p r o c e d u r e a r e e x p la in e d to th e p a tie n t.

22. B. A n a n e s th e s ia c o n s e n t s h o u ld b e o b ta in e d d u r in g p r e a n e s th e tic e v a lu a tio n , w h e n e v e r


p o s s ib le . T h is is o n e o f th e p r im e a im s th a t n e e d to b e f u lf ille d a s a c o m p o n e n t o f p r e o p e r a t iv e
a n e s th e tic e v a lu a tio n .

23. D. T h e g o a ls o f p r e a n e s th e tic e v a lu a tio n in c lu d e a ll th o s e lis te d in th e q u e s tio n . In a d d itio n ,


o th e r ta r g e ts o f p r e a n e s th e tic e v a lu a tio n in c lu d e e d u c a tio n o f p a tie n ts a n d f a m ilie s a b o u t
a n e s th e s ia a n d th e a n e s th e s io l o g is t’s r o le , o b ta in in g in f o r m e d c o n s e n t, m o tiv a tio n o f p a tie n ts to
s to p s m o k in g a n d lo s e w e ig h t, o r c o m m it to o th e r p r e v e n tiv e c a r e .

24. D. A S A c la s s if ic a tio n d o e s n o t in c lu d e th e n a tu r e o f p r o c e d u r e in p r e d ic tin g p e r io p e r a tiv e


m o r b i d ity a n d m o r ta lity . It o n ly in c lu d e s p a tie n t- b a s e d m o r b i d ity r a th e r th a n ty p e o f s u r g e r y .

25. B. H e a lth y p r e g n a n t p a tie n ts in la b o r a r e c la s s if ie d a s a n A S A II. P a tie n ts w ith c o n tr o lle d


d ia b e te s o r e s s e n tia l h y p e r te n s io n a r e s till c la s s if ie d a s a n A S A II. P r e s e n c e o f p r e e c la m p s ia w ill
s te p u p th e c la s s if ic a tio n to a n A S A III.

26. C. S e d a tiv e s ty p ic a lly a lle v ia te a n x ie ty in h y p e r te n s iv e p a tie n ts ( p r e v e n tin g b l o o d p r e s s u r e


e le v a tio n s d u e to s u r g e r y - r e la te d a n x ie ty ), in p a tie n ts w ith c h r o n ic a lc o h o l a b u s e , a n d in
c h ild r e n to m a in ta in c o o p e r a tio n f o r in d u c tio n o f a n e s th e s ia . In n e u r o s u r g i c a l p a tie n ts ,
s e d a tiv e s c a n le a d to d e p r e s s io n o f r e s p i r a t o r y d r iv e , w h ic h c a n c a u s e h y p e r c a r b i a a n d a n
in c r e a s e in in tr a c r a n ia l p r e s s u r e .

27. A. A s p e r A S R A g u id e lin e s , w a r f a r i n m u s t b e s to p p e d a t le a s t 5 d a y s p r i o r a n d c l o p i d o g r e l 7
to 10 d a y s p r i o r to e le c tiv e s u r g e r y . L o w - m o le c u la r - w e ig h t h e p a r in in th e r a p e u tic d o s e s m u s t
b e s to p p e d a t le a s t 2 4 h o u r s p r io r , a n d w h e n b e in g u s e d in p r o p h y la c tic d o s e s , it m u s t b e
s to p p e d a t le a s t 12 h o u r s p r i o r to a n e le c tiv e s u r g e r y r e q u i r i n g c e n tr a l n e u r a x ia l b lo c k a d e .
A s p ir in u s e is n o m o r e c o n s id e r e d a s a c o n tr a in d ic a tio n to p e r f o r m i n g a n e u r a x ia l b lo c k .

28. B. A S A c la s s if ie s a n y m e d ic a l c o m o r b id ity w ith o u t f u n c tio n a l lim ita tio n (i.e., h y p e r te n s iv e


w ith o u t c o r o n a r y a r te r y d is e a s e o r a n g in a ) a s a n A S A II. O n c e th e p a tie n t’s a c tiv ity is lim ite d
d u e to th e d is e a s e , th e p a tie n t is th e n c a te g o r iz e d a s a n A S A III.

29. D. B y d e f in itio n , s u c h p a tie n ts a r e c a te g o r iz e d a s A S A C la s s V I.

30. C. B y d e f in itio n , th e s e p a tie n ts r e q u i r e s u r g e r y d e s p ite b e in g r e a l l y s ic k . M o s t o fte n , th e


s u r g ic a l c o r r e c t i o n o f th e u n d e r ly in g p a th o lo g y (th a t m a y h a v e le d to m u l tio r g a n in v o lv e m e n t)
m a y b e th e o n ly o p tio n o f im p r o v in g th e ir c h a n c e s o f s u r v iv a l. A h e m o d y n a m ic a lly u n s ta b le
p a tie n t s e c o n d a r y to p e r f o r a t i o n p e r ito n itis , w ith a n a c u te k id n e y in ju r y , w o u ld b e a n e x a m p le .
A lth o u g h th e p a tie n t m a y b e e x tr e m e ly s ic k , u n til th e p e r f o r a t i o n p e r ito n itis is s u r g ic a ll y
tre a te d , th e c h a n c e s o f s u r v iv a l m a y n o t im p r o v e .

31. B. A S A III is a p a tie n t w ith s e v e r e s y s te m ic d is e a s e th a t is a c o n s ta n t th r e a t to lif e ( f u n c tio n a lity


in c a p a c ita te d ) .

32. C. W a r f a r in s h o u ld b e s to p p e d a t le a s t 5 d a y s p r i o r to s u r g e r y . O n th e d a y o f th e s u r g e r y , th e
p r o t h r o m b i n tim e ( in te r n a tio n a l n o r m a liz e d r a tio o r IN R ) is c h e c k e d . A n IN R o f 1.4 o r le s s is
d e s ir a b le to p e r f o r m th e s u r g e r y .

33. A. B e f o r e a n y r a te /r h y th m c o n tr o l in p a tie n ts lik e ly to h a v e A F f o r m o r e th a n 4 8 h o u r s , le ft-


a tr ia l c lo ts m u s t b e r u le d o u t. A n u n d ia g n o s e d c lo t c a n le a d to c a ta s tr o p h ic e m b o lic
co n seq u en ces.

34. D. F o r a d r u g - e lu tin g ste n t, it is a d v is e d to a v o id e le c tiv e s u r g e r y f o r a y e a r (to c o n tin u e d u a l


a n tip la te le t m e d ic a tio n ) , a n d f o r a b a r e - m e ta ll ic sten t, it is a d v is e d to a v o id e le c tiv e s u r g e r y f o r
a b o u t 4 w e e k s . P e r f o r m i n g la p a r o s c o p ic s u r g e r y p o s t- C A B G s u r g e r y is h ig h ly ris k y . S o w h e n
s u r g e r y n e e d s to b e p la n n e d in th e n e a r fu tu r e , th e p a tie n t s h o u ld b e a d v is e d to u n d e r g o b a llo o n
d ila ta tio n a n d th e n d e la y th e e le c tiv e p r o c e d u r e f o r 2 to 3 w e e k s th e re a f te r .

35. C. R e n a l f a ilu r e c a n in d u c e p la te le t d y s fu n c tio n , a n d th e r e f o r e , c e n tr a l n e u r a x ia l b lo c k a d e is


s till d e b a te d in th e s e p a tie n ts . T h e y a ls o h a v e c o a g u la tio n f a c to r a b n o r m a litie s th a t m a y
p r e d is p o s e th e m to d e e p v e in th r o m b o s is . A n e m ia is a r e s u lt o f d e c r e a s e d e r y th r o p o ie tin
p r o d u c tio n a n d is o f te n la b e le d a s “ a n e m ia o f c h r o n ic d is e a s e .”

36. C. A s p e r A S A g u id e lin e s , it is r e c o m m e n d e d to w a it a t le a s t 6 h o u r s a fte r in g e s t io n o f


n o n h u m a n m ilk b e f o r e p e r f o r m i n g a n e le c tiv e o p e r a tio n in a c h ild .
37. C. V o la tile in h a la tio n a g e n ts a n d s u c c in y lc h o lin e a r e c o n s id e r e d t r i g g e r s f o r m a lig n a n t
h y p e r th e r m ia (M H ) r e a c tio n . M H h a s a g e n e tic c o m p o n e n t, a n d r u n s in f a m ilie s . S in c e h e r a u n t
h a d a s e v e r e r e a c tio n to a n e s th e s ia , f u r th e r d e ta ils s h o u ld b e o b ta in e d f r o m th e h is to r y . If a n y
d o u b t a b o u t th e h is to r y , th e p a tie n t s h o u ld b e a s s u m e d to b e p r o n e to d e v e lo p in g M H . V o la tile
a g e n ts a n d s u c c in y lc h o lin e s h o u ld b e a v o id e d in th is p a tie n t.

38. B. E le c tiv e s u r g e r y s h o u ld b e p o s tp o n e d f o r a t le a s t 6 w e e k s a fte r a m y o c a r d ia l in f a r c tio n .


R is k o f r e i n f a r c t i o n is a p p r o x im a te ly 5 .5 % f o r s u r g e r i e s b e tw e e n 0 a n d 3 m o n th s , 2 .5 % b e tw e e n
3 a n d 6 m o n th s , a n d 2 % a fte r 6 m o n th s o f a m y o c a r d ia l in f a r c tio n .

39. A. T h e m o s t s ig n if ic a n t r i s k f a c to r f o r d e v e lo p in g p u lm o n a r y c o m p lic a tio n s is th e u p p e r


a b d o m in a l o r th o r a c ic s ite o f s u r g e r y . A s s u c h , a ll p a tie n ts u n d e r g o in g s u c h s u r g e r i e s s h o u ld b e
o p tim a lly p r e p a r e d f o r th e s u r g e r y . T h is in c lu d e s p u lm o n a r y to ile t: c h e s t
p h y s io th e r a p y /e x e r c is e s , a n d p o s tu r a l d r a in a g e o f m u c u s a n d s e c r e tio n s .

40. C. T h e r e is n o s p e c if ic v a lu e o f IN R b e f o r e a p a tie n t is ta k e n to th e O R f o r e le c tiv e s u r g e r y .


H o w e v e r, it is r e c o m m e n d e d th a t a n IN R v a lu e o f 1.4 o r le s s s h o u ld b e a im e d f o r b e f o r e ta k in g
th e p a tie n t to th e O R f o r e le c tiv e s u r g e r y . In c a s e o f e m e r g e n c y , th e IN R c a n b e n o r m a liz e d b y
in f u s in g f r e s h - f r o z e n p la s m a .

41. C. T h e p a r a ly z e d m u s c le s d u e to c e n tr a l d e n e r v a tio n e v e n tu a lly d e v e lo p a tro p h y .


E x tr a ju n c tio n a l r e c e p to r s a r e th e n s y n th e s iz e d a t th e m u s c le s ite s , w h ic h r e m a in r e s is ta n t to th e
e ffe c ts o f n e u r o m u s c u la r b lo c k a d e f o r v a r y in g d e g r e e s . T h u s , th e s e p a r a ly z e d m u s c le s g iv e a n
e x a g g e r a te d r e s p o n s e o n d ir e c t s tim u la tio n w ith a n e r v e s tim u la to r . T h e r e f o r e , m u s c le tw itc h
m o n ito r in g s h o u ld b e d o n e o n th e n o n a f f e c te d s ite s to c o r r e c t l y m o n ito r th e d e g r e e o f
n e u r o m u s c u la r b lo c k a d e .

42. B. M A C ty p ic a lly is f o u n d to b e lo w e r f o r p a tie n ts o n s e d a tiv e s , a n x io ly tic s , a lc o h o l


in to x ic a tio n , h y p o th e r m ia , e x tr e m e s o f a g e , m o r i b u n d /s ic k p a tie n ts , a n d p a tie n ts w ith o b tu n d e d
c o n s c io u s n e s s . C h r o n ic a lc o h o l a b u s e , h o w e v e r, in c r e a s e s M A C .

43. A. A ll p 2 a g o n is ts a r e k n o w n to c a u s e in te r n a liz a tio n o f p o ta s s iu m ( f r o m p la s m a to c e ll), th u s


c a u s in g h y p o k a le m ia . T h is p r in c ip le is s o m e tim e s u s e d in th e tr e a tm e n t o f p a tie n ts w ith
h y p e r k a le m ia .

44. D. S m o k in g c e s s a tio n f o r 2 4 h o u r s b e f o r e s u r g e r y r e d u c e s c a r b o x y h e m o g lo b i n (C O H b )
le v e ls . R e d u c e d le v e ls o f C O H b in c r e a s e s le v e ls o f o x y g e n a te d H b , w h ic h d e c r e a s e s th e r i s k o f
m y o c a r d ia l is c h e m ia a n d p e r io p e r a tiv e c a r d ia c m o r b id ity . D e la y e d b e n e f its ( c e s s a tio n m o r e
th a n 8 w e e k s ) a r e k n o w n to im p r o v e a ir w a y i m m u n o lo g i c a n d c i l i a r y fu n c tio n .

45. D. A m o n g a ll th e s e te sts, T E G h a s th e h ig h e s t p o s itiv e p r e d ic tiv e v a lu e f o r d ia g n o s i n g a


b le e d in g te n d e n c y . D e r a n g e d v a lu e s f r o m o th e r te sts lis te d h a v e n o t s h o w n to a lw a y s c o r r e la te
w e ll w ith b le e d in g te n d e n c y . F o r e x a m p le , th e o th e r te sts w ill b e d e r a n g e d in a p a tie n t w ith
s e p s is b u t m a y n o t s h o w a c lin ic a ll y r e le v a n t b le e d in g te n d e n c y .

46. D. A ll th e o th e r c h o ic e s n e e d e v a lu a tio n /o p tim iz a tio n p r i o r to e le c tiv e n o n c a r d ia c s u r g e r y .


U n c o n tr o lle d s y s to lic h y p e r te n s io n w ith o u t t a r g e t e n d - o r g a n d a m a g e is a m in o r p r e d i c t o r / r i s k
fa c to r. It c a n b e u s u a lly c o n tr o lle d w ith in tr a o p e r a tiv e a n tih y p e r te n s iv e m e d ic a tio n s w ith o u t
e v id e n c e o f s ig n if ic a n t a d v e r s e o u tc o m e s .

47. C. G ly c o p y r r o la te is a s y n th e tic q u a te r n a r y a m in e w ith a n tim u s c a r in ic p r o p e r tie s a n d n o


c e n tr a l s id e e ffe c ts l ik e s e d a tio n . A ll th e o th e r c h o ic e s a r e a s a r e s u lt o f d ir e c t c o n s e q u e n c e o f
c h o lin e r g ic b lo c k a d e .

48. C. M e to c lo p r a m id e is a p r o k in e tic a g e n t th a t e n h a n c e s g a s tr ic c le a r a n c e a n d in c r e a s e s lo w e r
e s o p h a g e a l s p h in c te r to n e , p r e v e n tin g v o m itin g , b u t m a y n o t a c tu a lly w o r k f o r n a u s e a
( v o m itin g r a th e r th a n n a u s e a is p r e v e n te d ) . It b lo c k s th e d o p a m in e r g ic r e c e p to r s to c a u s e
p a r k in s o n is m - lik e e x tr a p y r a m id a l s id e e ffe c ts .

49. B. P r e o x y g e n a tio n o f lu n g s p r i m a r i l y a c ts to in c r e a s e s a f e a p n e a tim e b y d e n itr o g e n a tin g


f u n c tio n a l r e s id u a l c a p a c ity (F R C ) a n d in c r e a s in g d is s o lv e d o x y g e n c o n te n t in th e b lo o d . It d o e s
n o t a lte r a n y p h y s ic a l m e a s u r e m e n ts o f lu n g s ; th a t is , it h a s n o e f f e c t o n F R C o r o n c lo s in g
v o lu m e /c a p a c ity .

50. A. H a lo th a n e , e s p e c ia lly o n r e p e a te d a d m in is tr a tio n , c a n c a u s e tw o s u b ty p e s o f h e p a titis (ty p e


1 is im m u n o g e n ic — m ild — a n d ty p e 2 is d u e to d ir e c t e f f e c t o f h a lo th a n e o n liv e r c e lls ) . T h e
in c id e n c e o f h a lo th a n e h e p a titis is a r o u n d 1 in 1 0 ,0 0 0 to 1 in 3 5 ,0 0 0 h a lo th a n e a n e s th e tic s .

51. C. B o th h a lo th a n e a n d s e v o f lu r a n e h a v e b e e n u s e d f o r in h a la tio n in d u c tio n in th e p e d ia tr ic


p o p u la tio n . S e v o f lu r a n e h a s l a r g e l y r e p la c e d h a lo th a n e d u e to a b e tte r s a f e ty p r o f ile , a n d h a s
e m e r g e d a s th e in d u c tio n a g e n t o f c h o ic e in p e d ia tr ic p o p u la tio n .

52. D. N itr o u s o x id e is k n o w n to in h ib it th e e n z y m e “ m e th io n in e s y n th a s e ,” in h ib itin g D N A


s y n th e s is a n d p r e c ip ita tin g B 12 d e fic ie n c y , c a u s in g p e r n ic io u s m e g a l o b la s tic a n e m ia . N itr o u s
o x id e is a ls o k n o w n to a c t o n N M D A r e c e p to r s a n d a ls o in c r e a s e p u lm o n a r y v a s c u la r
r e s is ta n c e .

53. D. A ll th e m e c h a n is m s h a v e b e e n p r o p o s e d f o r p r o p o f o l in p r e v e n tin g n a u s e a a n d v o m itin g in


th e p o s to p e r a tiv e p e r i o d (P O N V ). P r o p o f o l, w h e n u s e d , is u s e d in r e f r a c t o r y c a s e s o f P O N V
a n d in l o w d o s e s .

54. C. K e ta m in e p r e s e r v e s s p o n ta n e o u s r e s p i r a t i o n a n d a ir w a y to n e w ith o u t c a u s in g a p n e a a t
in d u c tio n d o s e s . P r o p o f o l a n d b e n z o d ia z e p in e s a r e a s s o c ia te d w ith r e s p i r a t o r y d e p r e s s io n a t
in d u c tio n d o s e s a n d c a u s e a p n e a .
55. B. S u c c in y lc h o lin e s h o u ld n o t b e u s e d in p a tie n ts w ith a h is to r y o f m u s c u la r d y s tr o p h y o r
p a tie n ts w ith a h is to r y o f m a lig n a n t h y p e r th e r m ia . M y a s th e n ia g r a v is p a tie n ts m a y s h o w
r e s is ta n c e to P h a s e I b lo c k o f s u c c in y lc h o lin e . In p a tie n ts w ith fu ll s to m a c h , s u c c in y lc h o lin e is
u s e d in “ r a p id s e q u e n c e in tu b a tio n ” to p r e v e n t a s p ir a tio n .

56. B. M u ltip le s tu d ie s h a v e s h o w n p r o p e n s ity o f A C E in h ib ito r s to p r e c ip ita te p r o f o u n d


h y p o te n s io n a t in d u c tio n o f g e n e r a l a n e s th e s ia , e s p e c ia lly in th e g e r ia tr ic a g e g r o u p . H e n c e ,
A C E in h ib ito r s s h o u ld b e w ith h e ld o n th e d a y o f th e s u r g e r y , e s p e c ia lly in th e e ld e r ly a n d f o r
m a jo r s u r g e r ie s .

57. D. M o r b id ly o b e s e p a tie n ts w ith O S A a r e o f te n s u b je c t to p e r s is te n t h y p o x ia , w h ic h le a d s to


in c r e a s e d p u lm o n a r y v a s c u la r r e s is ta n c e , e v e n tu a lly le a d in g to p u lm o n a r y a r te r y h y p e r te n s io n .
O b e s e p a tie n ts a r e a ls o k n o w n to h a v e a h ig h e r in c id e n c e o f c a r d ia c p r o b le m s , in c lu d in g a
d ila te d h e a r t a n d h e a r t f a ilu r e . C o m p r e s s io n n e u r o p a th ie s a r e a ls o c o m m o n in th is
s u b p o p u la tio n . D e m e n tia is a c e n tr a l- n e r v o u s - s y s te m - r e la te d c o m p lic a tio n n o t a s s o c ia te d
d ir e c tly w ith o b e s ity .

58. D. R e m if e n ta n il p r e p a r a t io n s a v a ila b le in th e m a r k e t h a v e g ly c in e a s th e p r e s e r v a tiv e , w h ic h


c a n c a u s e d ir e c t n e u r o to x ic ity . T h u s , it is r e c o m m e n d e d th a t r e m if e n ta n il p r e p a r a t io n s b e n o t
u s e d f o r c e n tr a l n e u r a x ia l b lo c k a d e .

59. D. O n d a n s e tr o n e x e r ts its a n tie m e tic e f f e c t b y a c tin g a s a n a n ta g o n is t o n th e 5 - H T 3 r e c e p to r s .


D r u g s in th e s a m e c a te g o r y in c lu d e p a lo n o s e tr o n a n d g r a n is e tr o n . R a r e ly r e p o r te d s id e e ffe c ts
o f th e s e a g e n ts in c lu d e Q T p r o lo n g a tio n , h y p o te n s io n , a n d h e a d a c h e .

60. C. S c o p o la m in e is a n a n tim u s c a r in ic d r u g th a t c a n c r o s s th e b l o o d - b r a i n b a r r i e r a n d c a u s e
s e d a tio n a n d c o n f u s io n , e s p e c ia lly in th e e ld e r ly . It d o e s n o t p r o d u c e a n a lg e s ia .

61. D. D e x m e d e to m id in e is a n a 2 r e c e p to r a g o n is t, w ith a b o u t e ig h t tim e s g r e a te r a f f in ity f o r th e


r e c e p to r th a n c lo n id in e . C o n tin u o u s in f u s io n is m o r e lik e ly to r e s u lt in h y p o te n s io n a n d
b r a d y c a r d ia .

62. C. A d d is o n ia n c r is is o r a c u te a d r e n a l in s u f f ic ie n c y d u r in g th e p e r io p e r a tiv e p e r i o d o c c u r s in
p a tie n ts w ith k n o w n a d r e n a l in s u f f ic ie n c y o r in th o s e r e c e iv in g c h r o n ic s te r o id th e ra p y . T h e
la tte r c a u s e s h y p o th a la m ic - p itu ita r y a x is s u p p r e s s io n . P a tie n ts w ith a d r e n a l in s u f f ic ie n c y m a y
p r e s e n t w ith r e f r a c t o r y s h o c k w ith e le c tr o ly te a n d g lu c o s e a b n o r m a litie s . T r e a tm e n t c o n s is ts o f
a d m in is tr a tio n o f h y d r o c o r tis o n e a n d c o r r e c t i o n o f a s s o c ia te d d e r a n g e m e n ts .

63. B. P r o m e th a z in e is c o m m o n ly u s e d a s a n a n tie m e tic . It h a s a n tid o p a m in e r g ic a c tiv ity , a n d in


a d d itio n a ls o h a s a n tih is ta m in ic a n d a n t i - a - a d r e n e r g i c a c tiv ity .

64. A. F a c to r s th a t a r e a s s o c ia te d w ith a n in c r e a s e d r i s k o f p o s to p e r a tiv e n a u s e a a n d v o m itin g


in c lu d e p r e v io u s h is to r y o f p o s to p e r a tiv e n a u s e a a n d v o m itin g , f e m a le g e n d e r , o b e s ity ,
n o n s m o k in g , p a in , e y e o r e a r s u r g e r y , la p a r o s c o p ic s u r g e r y , a n e s th e tic d r u g s , a n d g a s tr ic
d is te n tio n .

65. A. A b r u p t w ith d r a w a l o f T P N w ill m o s t c o m m o n ly r e s u lt in h y p o g ly c e m ia d u e to th e h ig h


c ir c u la tin g in s u lin le v e ls .

66. A. G ly c o p y r r o la te is a n a n tic h o lin e r g ic d r u g w ith a q u a te r n a r y a m m o n iu m s tr u c tu r e , w h ic h


p r e v e n ts it f r o m c r o s s in g th e b l o o d - b r a i n b a r r i e r . T h e r e f o r e , it h a s n o c e n tr a l n e r v o u s s y s te m
e ffe c ts (s e d a tio n ) . G ly c o p y r r o la te in c r e a s e s th e h e a r t ra te , c a u s e s d r y n e s s o f s e c r e tio n s , a n d
lo w e r s th e lo w e r e s o p h a g e a l s p h in c te r to n e . T h e la tte r m a y p r e d is p o s e a p a tie n t to p u lm o n a r y
a s p ir a ti o n o f g a s tr ic c o n te n ts .

67. B. P a tie n ts ta k in g h e r b a l m e d ic a tio n s f o r th e ir a lle g e d b e n e f its a r e o f te n u n a w a r e o f th e ir


p o te n tia l s id e e ffe c ts ( b le e d in g te n d e n c y , p la te le t d y s fu n c tio n , e tc.). M o s t m e d ic a tio n s m u s t b e
s to p p e d f o r a t le a s t 7 d a y s p r i o r to s u r g e r y .

68. A. G e n ta m ic in is a n a m in o g ly c o s id e a n tib io tic th a t b lo c k s a c e ty lc h o lin e r e le a s e f r o m th e


p r e s y n a p tic te r m in a ls a n d r e d u c e s p o s ts y n a p tic r e s p o n s iv e n e s s . T h is m a y p r o l o n g
n e u r o m u s c u la r b lo c k a d e a s s o c ia te d w ith n o n d e p o la r iz i n g m u s c le r e la x a n ts .

69. B. E s tr o g e n in ta k e c a n le a d to a h y p e r c o a g u la b le sta te , p r e d is p o s in g w o m e n to
t h r o m b o e m b o li c e v e n ts . O th e r r i s k f a c to r s f o r th r o m b o e m b o li s m in c lu d e m a jo r s u r g e r y ,
m u ltip le tr a u m a (h ip f r a c tu r e ) , lo w e r e x tr e m ity p a r a ly s is , in c r e a s in g a g e , c a r d ia c o r r e s p i r a t o r y
f a ilu r e , p r o l o n g e d im m o b ility , p r e s e n c e o f c e n tr a l v e n o u s lin e s , a n d a w id e v a r ie ty o f
h e m a t o lo g ic c o n d itio n s ( in h e r ite d o r a c q u ir e d ) .

70. C. B e c a u s e o f its n a r r o w th e r a p e u tic in d e x , lith iu m d o s in g r e q u ir e s c o n s ta n t s u r v e illa n c e w ith


m o n ito r in g o f le v e ls a n d d o s a g e a d ju s tm e n t. T h r e e ty p e s o f lith iu m in to x ic a tio n c a n o c c u r —
a c u te , a c u te o r c h r o n ic , a n d c h r o n ic . C h r o n ic lith iu m in to x ic a tio n o c c u r s in th o s e p a tie n ts o n
lo n g - t e r m lith iu m th e ra p y .
• M ild to x ic ity : m a n if e s ts a s le th a r g y , d r o w s in e s s , c o a r s e h a n d tr e m o r , m u s c le w e a k n e s s ,
n a u s e a , v o m itin g , a n d d ia r r h e a
• M o d e r a te to x ic ity : m a n if e s ts a s c o n f u s io n , d y s a r th r ia , n y s ta g m u s , a ta x ia , m y o c lo n ic tw itc h e s ,
a n d f la t o r in v e r te d T -w a v e s o n E C G
• S e v e re to x ic ity : m a y b e lif e - th r e a te n in g . It m a y p r e s e n t w ith g r o s s l y im p a ir e d c o n s c io u s n e s s ,
in c r e a s e d d e e p te n d o n r e f le x e s , s e iz u r e s , s y n c o p e , r e n a l in s u f f ic ie n c y , c o m a , a n d d e a th .

71. A. P a tie n ts u n d e r tr e a tm e n t w ith M A O Is h a v e a n in c r e a s e d a v a ila b ility o f e n d o g e n o u s


n o r e p in e p h r in e . T h e r e f o r e , tr e a tm e n t w ith a n in d ir e c t- a c tin g d r u g s u c h a s e p h e d r in e c a n le a d to
a n e x a g g e r a te d r e s p o n s e . H y p o te n s io n in th e s e p a tie n ts is b e tte r m a n a g e d w ith a d ir e c t- a c tin g
d r u g s u c h a s p h e n y le p h r in e .
72. B. P a tie n ts ta k in g o r a l h y p o g ly c e m ic a g e n ts m a y e x p e r ie n c e d e la y e d h y p o g ly c e m ia in th e
a b s e n c e o f c a l o r i c in ta k e in th e in tr a o p e r a tiv e a n d p o s to p e r a tiv e p e r io d s . H e n c e , p a tie n ts s h o u ld
b e a d v is e d n o t to ta k e o r a l h y p o g ly c e m ic a g e n ts th e m o r n i n g o f th e s u r g e r y . In a d d itio n ,
m e tf o r m in s h o u ld b e s to p p e d a t le a s t 4 8 h o u r s b e f o r e s u r g e r y a s it m a y p r e c ip ita te th e
d e v e lo p m e n t o f la c tic a c id o s is d u r in g s u r g e r y . P a tie n ts o n a n in s u lin p u m p s h o u ld c o n tin u e th e
in s u lin a t th e b a s a l ra te .

73. B. D ig o x in is a n in o tr o p e th a t b lo c k s th e N a+ /K + A T P a s e p u m p o n th e m y o c a r d ia l c e ll. It
c a u s e s c a lc iu m io n s to e n te r th e c e lls , b u t c a u s e s a n e t K+ lo s s f r o m th e c e ll. T h u s , h y p o k a le m ia ,
m o r e so th a n h y p e r c a lc e m ia , w ill e x a c e r b a te d ig ita lis to x ic ity . S ig n s a n d s y m p to m s o f d ig o x in
to x ic ity in c lu d e d r o w s in e s s o r c o n f u s io n , n a u s e a /v o m itin g , lo s s o f a p p e tite , d ia r r h e a , d is tu r b e d
c o l o r v is io n ( y e llo w o r g r e e n h a lo s a r o u n d o b je c ts ) , a g ita tio n , a n d c a r d ia c d y s r h y th m ia s .
C h a r a c te r is tic E K G c h a n g e s in c lu d e b r a d y c a r d ia , a p r o l o n g e d P R in te r v a l, o r a n a c c e le r a te d
ju n c tio n a l rh y th m .

74. C. D u r in g c e n tr a l lin e in s e r tio n , th e g u id e w ir e o r th e tip o f th e c a th e te r e n te r s th e r i g h t a tr iu m


a n d m a y r e s u lt in a n a r r h y th m ia , w h ic h r e tu r n s to s in u s r h y th m w h e n th e g u id e w ir e /c a th e te r tip
is w ith d r a w n o u t o f th e h e a rt.

75. C. A n tib io tic a lle r g ie s m a y r e s u lt in a n a n a p h y la c tic o r a n a p h y la c to id r e a c tio n . B a s e d o n th e


p a tie n t’s p r e s e n ta tio n , a n a p h y la c tic s h o c k is th e m o s t c o n s is te n t d ia g n o s is a n d n e e d s to b e
tr e a te d w ith e p in e p h r in e f ir s t, w h ic h r e v e r s e s m o s t o f th e m a n if e s ta tio n s o f a n a p h y la x is .

76. D. T h e u ln a r n e r v e is f r e q u e n tly s p a r e d w ith a n in te r s c a le n e b lo c k . C o m p lic a tio n s o f a n


in te r s c a le n e b lo c k in c lu d e s te lla te g a n g lio n b lo c k , p h r e n ic n e r v e b lo c k , r e c u r r e n t la r y n g e a l
n e r v e b lo c k , H o r n e r s y n d r o m e , v e r te b r a l a r te r y in je c tio n , e p id u r a l/s u b a r a c h n o id /s u b d u r a l
in je c tio n , a n d p n e u m o th o r a x .

77. A. A n a x illa r y n e r v e b lo c k p r o d u c e s b lo c k a d e o f th e m e d ia n , u ln a r, a n d th e r a d ia l n e r v e s .
S e n s a tio n to th e la te r a l a s p e c t o f th e f o r e a r m is p r o v id e d b y th e m u s c u lo c u ta n e o u s n e r v e , w h ic h
m u s t b e b lo c k e d s e p a r a te ly (d e e p in je c tio n in to th e c o r a c o b r a c h ia lis m u s c le ) .

78. D. T h e f e m o r a l n e r v e lie s la te r a l to th e f e m o r a l a rte ry , w h ic h is la te r a l to th e f e m o r a l v e in


(V A N — v e in , a rte ry , n e r v e ; m e d ia l to la te r a l) .

79. D. T h e a n k le b lo c k b lo c k s th e d e e p p e r o n e a l n e r v e , th e s a p h e n o u s n e r v e , th e p o s t e r i o r tib ia l
n e r v e , th e s u r a l n e r v e , a n d th e s u p e r f ic ia l p e r o n e a l n e rv e .

80. A. L a r y n g o p h a r y n g iti s is m o r e c o m m o n a fte r a n e n d o tr a c h e a l in tu b a tio n th a n w h e n u s in g a


la r y n g e a l m a s k a irw a y . T h e in c id e n c e o f s o r e th r o a t c a n v a r y f r o m 1 5 % to 4 0 % , a n d d e p e n d s
o n o p e r a to r e x p e r ie n c e (le s s tr a u m a ) . U s e o f s m a lle r e n d o tr a c h e a l tu b e s , s m a lle r c u f f s iz e s (le s s
a r e a o f c o n ta c t w ith tr a c h e a l m u c o s a ) , a n d l o w p r e s s u r e in th e tr a c h e a l c u f f d e c r e a s e th e
in c id e n c e o f p o s to p e r a tiv e s o r e th r o a t. U s in g lid o c a in e j e l l y to lu b r ic a te th e e n d o tr a c h e a l tu b e
( r a th e r th a n lu b r ic a tin g je lly ) in c r e a s e s th e in c id e n c e o f s o r e th r o a t. M o s t c a s e s o f s o r e th r o a t
r e s o lv e s p o n ta n e o u s ly .

81. B. In H O C M , o b s tr u c tio n o f th e v e n tr ic u la r o u tf lo w tr a c t c a n o c c u r f r o m s y s to lic a n te r io r


m o tio n o f th e m itr a l v a lv e a g a in s t th e h y p e r tr o p h ie d s e p tu m . In p a tie n ts w ith a s e v e r e H O C M ,
m y o c a r d ia l d e p r e s s io n is b e n e f ic ia l, w h ic h c a n b e o b ta in e d b y u s in g P - b lo c k e r s ( m e to p r o l o l) o r
c a lc iu m c h a n n e l b lo c k e r s .

82. D. St. J o h n w o r t is a c o m m o n ly u s e d h e r b a l m e d ic a tio n th a t is a C Y P 2 C 1 9 - a n d C Y P 3 A 4


in d u c e r . A s c l o p i d o g r e l is a c tiv a te d b y th e c y to c h r o m e P 4 5 0 s y s te m , St. J o h n w o r t m a y b e u s e d
to in c r e a s e th e e f f e c t o f c l o p i d o g r e l in h y p o r e s p o n d e r s . It r e d u c e s th e e f f e c t o f w a r f a r i n a n d
h e p a r in , w ith little e f f e c t o n a s p ir in .

83. C. A d v a n c e d a g e is th e m o s t im p o r ta n t p r e d ic to r o f a tr ia l f i b r illa tio n n o t o n ly in p a tie n ts


f o ll o w i n g c a r d ia c s u r g e r y b u t a ls o in th e g e n e r a l p o p u la tio n .

84. C. P a r k in s o n d is e a s e is c h a r a c te r iz e d b y a lo s s o f d o p a m in e in th e n ig r o s tr i a tu m , r e s u ltin g in
b r a d y k in e s ia , r ig id ity , p o s tu r a l in s ta b ility , a n d p i l l - r o l l i n g r e s tin g tr e m o r . M e to c lo p r a m id e (a n d
d r o p e r i d o l ) h a s s ig n if ic a n t a n tid o p a m in e r g ic p r o p e r tie s a n d s h o u ld b e a v o id e d in th e s e p a tie n ts
in th e tr e a tm e n t o f n a u s e a a n d v o m itin g .

85. B. T h e N e w Y o rk H e a r t A s s o c ia tio n c la s s if ic a tio n f o r h e a r t f a ilu r e is b a s e d o n b o th a


f u n c tio n a l a n d o b je c tiv e a s s e s s m e n t o f th e p a tie n t’s c a p a b ilitie s a n d s y m p to m s . T h is p a tie n t is
a s y m p to m a tic a t r e s t a n d c a n g o a b o u t h is a c tiv itie s o f d a ily liv in g w ith o u t is s u e s . H o w e v e r,
w ith m o r e s tr e n u o u s a c tiv ity , h e b e c o m e s d y s p n e ic . H is c la s s if ic a tio n w o u ld , th e r e f o r e , b e 2
(T a b le s 1-1 a n d 1 -2 ).

Table 1-1 Functional capacity: How a patient with cardiac disease feels during physical activity

Class I: Patients with cardiac disease but resulting in no limitation of physical Ordinary physical activity does not cause undue fatigue, palpitation, <
activity. or anginal pain.
Class II: Patients with cardiac disease resulting in slight limitation of physical They are comfortable at rest. Ordinary physical activity results in fati
activity. palpitation, dyspnea, or anginal pain.
Class III: Patients with cardiac disease resulting in marked limitation of They are comfortable at rest. Less-than-ordinary activity causes fatig
physical activity. palpitation, dyspnea, or anginal pain.
Class IV: Patients with cardiac disease resulting in inability to carry on any Symptoms of heart failure or the anginal syndrome may be present ev<
physical activity without discomfort. rest. If any physical activity is undertaken, discomfort increases.

Table 1-2 Objective assessment


Class A: No objective evidence of cardiovascular disease. No symptoms and no limitation in ordinary physical activity.
Class B: Objective evidence of minimal cardiovascular Mild symptoms and slight limitation during ordinary activity. Comfortable at rest.
disease.
Class C: Objective evidence of moderately severe Marked limitation in activity due to symptoms, even during less-than-ordinary activity
cardiovascular disease. Comfortable only at rest.
Class D: Objective evidence of severe cardiovascular Severe limitations. Experiences symptoms even while at rest.
disease.
86. D. In th e e v e n t o f a p o s td u r a l p u n c tu r e h e a d a c h e (P D P H ), 5 3 % o f h e a d a c h e s r e s o lv e in 4 d a y s,
7 2 % in 7 d a y s , a n d 8 5 % w ith in 6 w e e k s . M ild - m o d e r a te P D P H is u s u a lly tr e a te d c o n s e r v a tiv e ly
(f lu id s , c a f f e in e d r in k s , a n a lg e s ic s ) . S e v e re P D P H m a y r e q u i r e a n e p id u r a l b l o o d p a tc h .

87. B. F o llo w in g a r a s h o f s u d d e n d e a th s in p a tie n ts ta k in g m e th a d o n e , th e F D A in 2 0 0 6 is s u e d a


b la c k b o x w a r n in g f o r a ll p r a c titio n e r s , s p e c if ic a lly d e ta ilin g th e h ig h r i s k o f p r o l o n g e d Q T
s y n d r o m e a n d s u d d e n d e a th in p a tie n ts p r e s c r i b e d th is m e d ic a tio n .

88. A. S u r g e r y f o r c a r c in o id tu m o r d e b u lk in g o r r e s e c tio n m a y p r e c ip ita te a c a r c in o id c r is is in


th e p a tie n t c o n s is tin g o f f lu s h in g , h y p o te n s io n , b r o n c h o s p a s m , a c id o s is , a n d v e n tr ic u la r
ta c h y c a r d ia . P a tie n ts w h o r e c e iv e d o c tr e o tid e e x p e r ie n c e d n o s ig n if ic a n t in tr a o p e r a tiv e
c o m p lic a tio n s .

89. C. I n je c tio n o f l a r g e a m o u n t o f lo c a l a n e s th e tic in to th e v e r te b r a l a r te r y o r in to th e


s u b a r a c h n o id o r s u b d u r a l s p a c e r e s u ltin g in a s e iz u r e is a w e ll- k n o w n c o m p lic a tio n o f th e
in te r s c a le n e b lo c k . T r e a tm e n t f o r th is p a tie n t is to f i r s t e s ta b lis h a n a ir w a y (A B C s) a n d th e n tr e a t
th e s e iz u r e .

90. B. P a tie n ts w ith d ila te d c a r d io m y o p a th y a r e e x tr e m e ly s e n s itiv e to c h a n g e s in a f te r lo a d .


T h e r e f o r e , a f te r lo a d s h o u ld b e m in im iz e d to m a in ta in s tr o k e v o lu m e .

91. C. In s e p tic s h o c k , b o th d o p a m in e a n d n o r e p in e p h r in e c a n b e u s e d to tr e a t p e r s is te n t
h y p o te n s io n . H o w e v e r, d o p a m in e m a y p r o m o t e f u r th e r tis s u e a c id o s is in th e s p la n c h n ic
c ir c u la tio n , w h e r e a s n o r e p in e p h r in e d o e s n o t, th u s m a k in g it th e d r u g o f c h o ic e f o r th is
s c e n a r io .

92. B. T h e h e a r t, b e c a u s e o f its h ig h o x y g e n r e q u ir e m e n ts , is th e le a s t to le r a n t o f is c h e m ia .
H y p e r k a le m ic c r y s ta ll o id c a r d io p le g i a a t 4 ° C f o r a m a x im u m o f 4 h o u r s is u s e d to p r e s e r v e th e
h e a r t. T h u s , r e d u c in g th e is c h e m ic tim e o f d o n o r h e a r ts w ill d e c r e a s e m o r b i d ity a n d c o s ts o f
c a r d ia c tr a n s p la n ta tio n s .

93. D. S u g a m m a d e x r e v e r s e s n e u r o m u s c u la r b lo c k a d e b y n o n d e p o la r iz i n g m u s c le r e la x a n ts b y
d ir e c tly b in d in g to r o c u r o n iu m , v e c u r o n iu m , a n d p a n c u r o n iu m , w ith o u t a n y s id e e ffe c ts .
R e v e r s a l o f n e u r o m u s c u la r b lo c k a d e is a c h ie v e d in a d o s e - d e p e n d e n t m a n n e r a n d c a n b e u s e d in
th e e v e n t o f f a ile d in tu b a tio n . F o r n o r m a l r e v e r s a l, th a t is , w ith tw o tw itc h e s , th e d o s e is 2
m g /k g . W h e n th e b lo c k a d e is d e e p e r, th e d o s e m u s t b e in c r e a s e d . W h e n r e v e r s i n g f o ll o w i n g a
f a ile d in tu b a tio n , a d o s e o f 8 m g /k g o f s u g a m m a d e x w ill e f f e c tiv e ly r e v e r s e r o c u r o n i u m g iv e n
a t 0 .6 m g /k g . If th e d o s e o f r o c u r o n i u m g iv e n is 1.2 m g /k g , r e v e r s a l w ith s u g a m m a d e x r e q u ir e s
a d o s e o f 16 m g /k g .

94. B. W ith p r o l o n g e d p e r io d s o f s ta r v a tio n f o ll o w e d b y r e in tr o d u c tio n o f e n te r a l o r p a r e n te r a l


n u tr itio n , th e in c r e a s e d r e le a s e o f p a n c r e a tic in s u lin le a d s to a n a n a b o lic s ta te a n d a n
in tr a c e llu la r s h if t o f p h o s p h a te , m a g n e s iu m , a n d p o ta s s iu m . O f th e s e d e r a n g e m e n ts ,
h y p o p h o s p h a te m ia le a d s to th e m o s t s e v e r e c o n d itio n s , in c lu d in g c a r d ia c f a ilu r e .

95. C. F e ta l h e a r t r a te a n d u te r in e m o n ito r in g s h o u ld b e p e r f o r m e d d u r in g in d u c tio n , e m e r g e n c e ,


r e c o v e r y , a n d , if p o s s ib le , d u r in g th e s u r g e r y in a n y p r e g n a n c y o f m o r e th a n 2 4 w e e k s ’
g e s ta tio n . T h e fe tu s b e c o m e s v ia b le a t th is g e s ta tio n a g e .

96. A. A ir w a y m a n a g e m e n t in o b e s e p a tie n ts b e g in s f i r s t w ith a n a d e q u a te p h y s ic a l e x a m a s th e s e


p a tie n ts a r e m o r e lik e ly to b e b o th m o r e d if f ic u lt to v e n tila te a n d to in tu b a te . T h e b e s t p r e d ic to r
o f d if f ic u lty is a s h o r t, th ic k n e c k ( p r e tr a c h e a l tis s u e v o lu m e ) a n d a h is to r y o f o b s tr u c tiv e s le e p
apnea.

97. C. In p u lm o n a r y r e s e c tio n s , p r e o p e r a t iv e im p a ir m e n t is d ir e c tly r e la te d to o p e r a tiv e r is k .


U s in g r o u tin e p u lm o n a r y f u n c tio n te sts, c r i t e r i a h a v e b e e n e s ta b lis h e d f o r h ig h - r i s k p a tie n ts .
• P a C O 2 > 4 5 m m H g o r P a O 2 < 5 0 m m H g o n r o o m a ir
• FE V <25%
• F E V i < 2 L p r e o p e r a t iv e ly o r < 0 .8 L o r < 4 0 % o f p r e d ic te d p o s to p e r a tiv e ly
• F E V i/F V C < 5 0 % p r e d ic te d
• M a x im u m b r e a th in g c a p a c ity < 5 0 % o f p r e d ic te d
• M a x im u m V O 2 < 1 0 m L /k g /m in

98. A. B is p h o s p h o n a te s a r e u s e d in th e tr e a tm e n t o f o s t e o p o r o s i s a s th e y in h ib it o s te o c la s tic
r e s o r p t i o n o f b o n e . B ip h o s p h o n a te s d o n o t a f f e c t th e e x c r e tio n o f c a lc iu m .

99. B. E n a la p r il e x p o s u r e d u r in g th e f i r s t tr im e s te r o f p r e g n a n c y h a s b e e n a s s o c ia te d w ith
m u ltip le fe ta l d e fe c ts , a f f e c tin g th e c a r d ia c , p u lm o n a r y , r e n a l, a n d m u s c u lo s k e le ta l s y s te m s .

100. C. S o m a to s e n s o r y - e v o k e d p o te n tia ls a r e u s u a lly m o n ito r e d o n th e p o s t e r i o r tib ia l n e r v e s o f


th e le g s d u r in g s p in a l s u r g e r y a n d a r e u s e d to a s s e s s th e in te g r ity o f th e d o r s a l c o lu m n s o f th e
s p in a l c o r d .

101. A. T h e m a in is s u e w ith d e s f lu r a n e is th a t it h a s a h ig h s a tu r a te d v a p o r p r e s s u r e a t r o o m
te m p e r a tu r e (6 6 9 m m H g a t 2 0 ° C ). It b o ils a t ju s t 2 2 .8 ° C c o m p a r e d w ith s e v o f lu r a n e a t 5 8 .5 °C
o r is o f lu r a n e a t 4 8 .5 °C . T h e r e f o r e , th e d e s f lu r a n e v a p o r iz e r is h e a te d to 3 9 ° C a n d p r e s s u r i z e d a t
2 a tm .

102. D. T h e in c id e n c e o f r e n a l f a ilu r e a fte r a b d o m in a l a o r tic a n e u r y s m s u r g e r y is 5 .4 % , o f w h ic h


0 .6 % r e q u ir e s h e m o d ia ly s is . L o o p d iu r e tic s ( f u r o s e m id e ) , d o p a m in e , m a n n ito l, f e n o ld o p a m ,
a n d A -a c e ty lc y s te in e a r e p r o p o s e d r e n a l p r o te c tiv e a g e n ts ; h o w e v e r, th e r e is n o c o n c r e te
e v id e n c e to s u p p o r t th e ir u s e . T h e m a in s ta y o f r e n a l p r e s e r v a tio n is b y r e d u c in g a o r tic c r o s s ­
c la m p in g tim e , a d e q u a te f lu id r e s u s c ita tio n , a n d a v o id a n c e o f n e p h r o to x in s ( n o n s te r o id a l a n ti­
in f l a m m a to r y d r u g s , a n g io te n s in - c o n v e r tin g - e n z y m e in h ib ito r s , a m in o g ly c o s id e a n tib io tic s ) .
Airway Management
Yuriy Bronshteyn

1. A major difference between the adult and neonatal airway is that the
A. Neonate’s larynx is located more superiorly in the neck
B. Neonate’s epiglottis is angled more superiorly
C. Narrowest segment of a neonate’s upper airway occurs at the level of the vocal cords
D. Neonate is at lower risk of postextubation stridor compared to the adult

2. The narrowest segment of a 14-day-old child’s upper airway is located at the


A. Hyoid bone
B. Thyroid cartilage
C. Vocal cords
D. Subglottic region

3. Airway obstruction in Pierre Robin syndrome most likely occurs


A. Between the tongue and pharyngeal wall
B. At the level of the glottis
C. In the subglottic trachea
D. At the bronchial level

4. Airway management in Klippel-Feil syndrome is most likely to be challenging because of


A. Micrognathia
B. Macroglossia
C. Subglottic stenosis
D. Cervical spine fusion

5. One of the following statements regarding airway management in patients with congenital
syndromes is most accurate:
A. Laryngoscopy is often challenging in Turner syndrome because of a high frequency of
laryngeal distortion
B. Airway management in Treacher Collins syndrome is complicated by a high incidence of
cervical spine instability
C. Intubation in patients with Goldenhar syndrome is often challenging due to a high rate of
subglottic stenosis
D. Airway management of patients with trisomy 21 is complicated by a high incidence of
cervical spine instability

6. A healthy 2-year-old male is scheduled to undergo a laparoscopic inguinal hernia repair. His
airway was managed uneventfully with mask ventilation followed by direct laryngoscopy and
intubation with a 4.5-mm uncuffed endotracheal tube (ETT). Manual ventilation produces an air
leak in the oropharynx beginning at a peak pressure of 20 cm H2O. The best next step in the
anesthetic management is to
A. Continue current management
B. Replace the ETT with a smaller-sized uncuffed tube
C. Replace the ETT with a larger-sized uncuffed tube
D. Replace the ETT with a 4.0-mm cuffed ETT

7. A 4-year-old patient scheduled for laparoscopic gastrostomy tube placement undergoes


induction of general anesthesia and endotracheal intubation with a 4.5-mm cuffed endotracheal
tube. The tube is taped 14 cm at the gumline, and the patient is placed on volume-control
ventilation. The most likely first sign of a right main stem intubation is
A. Arterial desaturation
B. Hypercapnia
C. Increased peak inspiratory pressures
D. Hypotension

8. A 6-year-old patient scheduled for laparoscopic bilateral inguinal hernia repair undergoes
inhalational induction and intubation with a 5.0-mm cuffed endotracheal tube. The tube is
secured with the 15-cm mark at the patient’s gumline. Auscultation reveals equal breath sounds
bilaterally. Inflation of the pilot balloon results in palpation of the inflated tube cuff just above
the cricoid cartilage. A leak test reveals leak of air into the oropharynx at a positive pressure of
20 cm H2O. The next best step in management is
A. No change in anesthetic care is indicated
B. The tube cuff should be deflated until a leak is present starting at 15 cm H2O of positive
pressure
C. The tube cuff should be deflated and the tube advanced until the cuff, when inflated, is
palpable below the cricoid cartilage
D. The tube cuff should be deflated and the tube withdrawn until ventilator peak pressures
decrease

9. A 4-year-old boy with autism and failure-to-thrive undergoes a gastrostomy tube placement. At
the completion of the operation, the patient remains unresponsive but is breathing
spontaneously and has a mild gag response to oral suctioning. The anesthesiologist extubates
the patient and immediately shuts off the volatile agent. The anesthesiologist then inserts an
appropriately sized oropharyngeal airway and places a face mask connected to the ventilator
circuit over the patient’s face, allowing the patient to breathe 100% oxygen. Despite providing a
chin lift, jaw thrust, and positive-pressure breaths, the anesthesiologist notes that the ventilator
shows no end-tidal carbon dioxide. Auscultation over the sternal notch reveals no air
movement. The pulse oximeter reading then rapidly drops to 70% from 100%. The next best
step in management is
A. Administration of albuterol
B. Insertion of a nasal trumpet
C. Endotracheal reintubation
D. Administration of succinylcholine

10. In the scenario above, if the patient’s postextubation condition is left untreated, the patient will
most likely experience
A. Aspiration
B. Bronchospasm
C. Pulmonary edema
D. Croup

11. A 2-year-old child weighing 13 kg is scheduled for inguinal hernia repair. She is at the 55th
percentile for height for her age. An appropriately-sized cuffed endotracheal tube for this
patient will have an internal diameter of
A. 3.0 mm
B. 4.0 mm
C. 5.0 mm
D. 6.0 mm

12. The superior surface of the epiglottis is innervated by the


A. Hypoglossal nerve
B. Recurrent laryngeal nerve
C. Internal branch of the superior laryngeal nerve
D. External branch of the superior laryngeal nerve

13. Tactile sensation from the anterior third of the tongue is carried by fibers of the
A. Trigeminal nerve
B. Facial nerve
C. Glossopharyngeal nerve
D. Hypoglossal nerve

14. A 48-year-old female patient with temporomandibular joint dysfunction and associated limited
mouth opening is scheduled for a thyroidectomy for goiter. Due to concern for challenging
laryngoscopy, the anesthesiologist elects to perform an awake fiberoptic intubation. In order to
anesthetize the posterior third of the tongue, the anesthesiologist should perform a nerve block
of the
A. Cranial nerve V
B. Cranial nerve VII
C. Cranial nerve IX
D. Cranial nerve XII

15. A patient who suffers acute, bilateral denervation of the external branch of the superior
laryngeal nerve will most likely present with
A. No symptoms
B. Hoarseness
C. Stridor
D. Aspiration

16. To anesthetize the supraglottic laryngeal mucosa, the local anesthetic should be injected into
one of the following areas:
A. The base of the anterior tonsillar pillar
B. Medial to the lesser cornu of the hyoid bone
C. Superior to the superior cornu of the thyroid cartilage
D. Through the cricothyroid membrane

17. The efferent limb of the glottic closure reflex, which is involved in laryngospasm, primarily
involves the
A. Internal branch of the superior laryngeal nerve
B. Hypoglossal nerve
C. Recurrent laryngeal nerve
D. Glossopharyngeal nerve

18. A 65-year-old woman undergoes a thyroidectomy for papillary thyroid cancer. Immediately
after emergence and extubation, she is aphonic and has minimal chest movement, despite
spontaneously moving her limbs and head. Auscultation reveals lack of breath sounds over the
chest. There is no evidence of a surgical site hematoma. The anesthesiologist provides a jaw
thrust and positive-pressure breaths, which slightly improve the patient’s oxygenation and
ventilation. The surgeon suggests a bilateral block of both the internal and external branches of
the patient’s superior laryngeal nerve. If performed this block would likely result in
A. Worsening of the patient’s respiratory distress and no change in her aphonia
B. Improvement of the patient’s respiratory distress and no change in her aphonia
C. No change in the patient’s respiratory distress and improvement of her aphonia
D. No change in the patient’s respiratory distress and no change in her aphonia

19. A 48-year-old woman with temporomandibular joint dysfunction and limited mouth opening is
scheduled for thyroidectomy for goiter. Due to concern for a difficult laryngoscopy, the
anesthesiologist elects to perform an awake oral fiberoptic intubation. To reliably blunt the
afferent limb of the cough reflex, the anesthesiologist should perform a bilateral block of the
A. Superior laryngeal nerve and the recurrent laryngeal nerve
B. Glossopharyngeal nerve and internal branch of the superior laryngeal nerve
C. Glossopharyngeal nerve and external branch of the superior laryngeal nerve
D. Internal and external branches of the superior laryngeal nerve

20. If an adult patient were to suffer an acute, bilateral transection of cranial nerve X, awake
laryngoscopy would most likely reveal
A. Fully adducted vocal cords
B. Fully abducted vocal cords
C. Vocal cords in a partially adducted position with 2 to 3 mm of space between them
D. Vocal cords oscillating between adducted and abducted position

21. Several hours after undergoing repair of an ascending aortic dissection, a 65-year-old male
patient is extubated in the intensive care unit. All of the arch vessels were preserved during the
operation. After extubation, the patient’s voice is noted to be hoarse. Awake fiberoptic
laryngoscopy would most likely show the following during inspiration:
A. Vocal cords in a fully abducted position
B. Vocal cords in a fully adducted position
C. Left vocal cord in an adducted position and right vocal cord fully abducted
D. Left vocal cord in an abducted position and right vocal cord fully adducted

22. An awake tracheostomy would be facilitated by a regional block of the


A. Trigeminal nerve
B. Glossopharyngeal nerve
C. Superior laryngeal nerve
D. Recurrent laryngeal nerve

23. One of the following statements regarding the innervation of airway structures is most correct:
A. The afferent limb of the gag reflex is primarily carried by fibers of the recurrent laryngeal
nerve
B. Trigeminal nerve block would facilitate awake nasotracheal intubation
C. The superior surface of the epiglottis is primarily innervated by the glossopharyngeal
nerve
D. Tactile sensation from the posterior one-third of the tongue is carried by the hypoglossal
nerve

24. A nasal trumpet would be most appropriate for management of anesthetic-induced upper airway
obstruction in one of the following patients:
A. A 25-year-old passenger ejected out of a motorcycle now with Glasgow Coma Scale of 13
and some periorbital bruising
B. A 32-year-old term parturient, otherwise healthy except for gestational thrombocytopenia,
who requires emergent cesarean section under general anesthesia
C. A 45-year-old female with temporomandibular joint syndrome and breast cancer scheduled
for bilateral mastectomy
D. A 65-year-old male with a mechanical mitral valve on therapeutic anticoagulation
undergoing emergent coronary catheterization for unstable angina

25. A 55-year-old woman with severe anxiety and rheumatoid arthritis is scheduled for
thyroidectomy for medullary thyroid cancer. Her airway exam in the upright position is notable
for a nonvisible uvula with the tongue protruded, a 2 fingerbreadth mouth opening, a
thyromental distance of 2.5 fingerbreadths, and neck range-of-motion at the atlanto-occipital
joint of about 70 degrees. Examination of her neck reveals an enlarged, fixed, and nonmobile
mass that appears to be contiguous with the thyroid gland when the patient swallows. The
trachea cannot be palpated. The patient is highly anxious and tells you that under no
circumstance will she let you insert a “breathing tube inside my airway while I’m awake.” The
next best step in anesthetic management is
A. Induction of general anesthesia followed by fiberoptic bronchoscopy
B. Induction of general anesthesia followed by rigid bronchoscopy
C. Induction of general anesthesia followed by laryngeal mask airway placement
D. Cancel the case

26. After rapid sequence induction of general anesthesia, a patient is unable to be intubated.
Subsequent attempts at ventilation by face mask and a supraglottic airway device are also
unsuccessful. One of the following statements regarding transtracheal jet ventilation and
surgical cricothyrotomy in this situation is most correct:
A. Transtracheal jet ventilation does not require a patent natural airway
B. Ventilation through a surgical cricothyrotomy allows both inhalation and exhalation to
occur
C. The development of laryngospasm during ventilation through a cricothyrotomy would
rapidly cause pulmonary overinflation and barotrauma
D. Transtracheal jet ventilation can be continued for a longer period of time than can
ventilation via a cricothyrotomy

27. Use of a laryngeal mask airway would be most appropriate for airway management in the
following patient:
A. An obese patient with acute appendicitis who, after rapid sequence induction, cannot be
intubated
B. An elderly patient with restrictive lung disease scheduled for inguinal hernia repair
C. An obese male patient with a hiatal hernia and GERD scheduled for umbilical hernia repair
D. A full-term parturient brought to the OR for emergent cesarean section because of fetal
bradycardia

28. After undergoing an uneventful operation, one of the following patients would be the best
candidate for “deep extubation”:
A. A 2 3 - y e a r - o ld w o m a n w ith a s th m a w h o h a s j u s t u n d e r g o n e a n e x p l o r a t o r y l a p a r o to m y f o r
s m a ll b o w e l o b s tr u c tio n
B. A 6 5 - y e a r - o ld m a n w ith g a s tr o e s o p h a g e a l r e f lu x w h o h a s j u s t u n d e r g o n e a n in g u in a l h e r n ia
r e p a ir
C. A n 1 8 - y e a r - o ld p a tie n t w ith s c o li o s is w h o h a s j u s t u n d e r g o n e a 6 - h o u r p o s te r io r
th o r a c o lu m b a r s p in a l in s tr u m e n ta tio n a n d f u s io n
D. A 6 4 - y e a r - o ld f e m a le w ith c o r o n a r y a r te r y d is e a s e w h o h a s ju s t u n d e r g o n e a to ta l h ip
a r th r o p la s ty u n d e r g e n e r a l a n e s th e s ia

29. O n e o f th e f o ll o w i n g is a p r i m a r y r i s k f a c to r f o r d if f ic u lt m a s k v e n tila tio n :

A. L im ite d m o u th o p e n in g
B. T h y r o m e n ta l d is ta n c e le s s th a n 3 f in g e r b r e a d th s
C. H ig h a r c h e d p a la te
D. In a b ility to b r in g m a n d ib u la r in c is o r s a n te r io r to th e m a x illa r y in c is o r s

30. A n o th e r w is e h e a lth y p a tie n t w ith a h is to r y o f d a y tim e s le e p in e s s a n d s n o r in g f r o m la r y n g e a l


p a p illo m a to s is u n d e r g o e s p o ly s o m n o g r a p h y a n d s p ir o m e tr y , w h ic h s h o w s d y n a m ic in s p i r a t o r y
o b s tr u c tio n . T h e f l o w - v o l u m e l o o p th a t w o u ld b e m o s t c o n s is te n t w ith th is p a tie n t’s c o n d itio n is

F igu re 2-1

A. F ig u r e 2 -1 A
B. F ig u r e 2 -1 B
C. F ig u r e 2 -1 C
D. F ig u r e 2 -1 D
CHAPTER 2 ANSWERS

1. A. T h e n e o n a te ’s la r y n x is lo c a te d m o r e s u p e r i o r l y in th e n e c k th a n th e a d u lt’s. T h e lo c a t io n o f
th e a d u lt’s la r y n x is a t C 4 - C 5 le v e l o f th e s p in e , w h ile th e n e o n a te ’s is a t C 3 - C 4 le v e l. T h e
n e o n a te ’s e p ig lo ttis is r e la tiv e ly lo n g e r , s tiffe r, a n d a n g le d m o r e posteriorly c o m p a r e d to th e
a d u lt’s, w h ic h is o n e o f th e r e a s o n s w h y s tr a ig h t b la d e s a r e m o r e p o p u la r a m o n g p e d ia tr ic
a n e s th e s io lo g is ts . T h e n a r r o w e s t p a r t o f th e u p p e r a ir w a y is a t th e le v e l o f th e c r i c o i d c a r tila g e
in n e o n a te s , a n d a t th e le v e l o f th e v o c a l c o r d s in a d u lts . T h e c h i l d ’s a ir w a y ta k e s o n a d u lt
c h a r a c te r is tic s b e tw e e n th e a g e s o f 5 a n d 10 y e a r s . T h e n e o n a te is a t g r e a te r r i s k o f
p o s te x tu b a tio n s tr id o r c o m p a r e d to th e a d u lt. R e s is ta n c e th r o u g h a c y lin d r ic a l tu b e (s u c h a s th e
tr a c h e a ) is in v e r s e ly p r o p o r t i o n a l to th e r a d iu s r a is e d to th e f o u r th p o w e r ( P o is e u ille la w ).
T h u s , a 1 -m m r e d u c tio n in tr a c h e a l d ia m e te r d u e to e d e m a r e s u lts in a m a r k e d r i s e in a ir w a y
r e s is ta n c e in s m a ll c h ild r e n , w h ic h m a y b e in c o n s e q u e n tia l in a d u lts .

2. D. A c c o r d in g to c la s s ic a l te a c h in g , th e n a r r o w e s t p o r t i o n o f a c h i l d ’s u p p e r a ir w a y is a t th e
le v e l o f th e c r i c o i d c a r tila g e , w h e r e a s th e n a r r o w e s t p o r t i o n o f a n a d u lt’s u p p e r a ir w a y is a t th e
le v e l o f th e v o c a l c o r d s . H o w e v e r, a m o r e r e c e n t b r o n c h o s c o p ic s tu d y o f a ir w a y d im e n s io n s in
c h ild r e n f o u n d th a t b e tw e e n th e a g e s o f 6 m o n th s a n d 13 y e a r s , th e g lo ttis , n o t th e c r ic o id
c a r tila g e , is th e n a r r o w e s t p o r t i o n o f th e c h i l d ’s a irw a y . T h is s tu d y d id n o t m e a s u r e a ir w a y
d im e n s io n s in c h ild r e n y o u n g e r th a n 6 m o n th s .

3. A. P ie r r e R o b in is a c o n g e n ita l s y n d r o m e a s s o c ia te d w ith e n la r g e d to n g u e , s m a ll m o u th , a n d
m a n d ib u la r a n o m a lie s ty p ic a lly m a n if e s te d a s m ic r o g n a th ia . A ll o f th e s e l im it th e
o r o p h a r y n g e a l s p a c e , c o n tr ib u tin g to a ir w a y o b s tr u c tio n b e tw e e n th e to n g u e a n d p o s te r io r
p h a r y n g e a l w a ll. W h e n y o u s e e P ie r r e R o b in , th in k P R : P o s te r io r R e s tr ic tio n b e h in d th e to n g u e .

4. D. K lip p e l- F e il is a c o n g e n ita l s y n d r o m e a s s o c ia te d w ith th e p h e n o ty p ic a l tr ia d o f s h o r t n e c k ,


l o w p o s t e r i o r h a i r lin e , a n d c o n g e n ita l s p in a l f u s io n c a u s in g lim ite d n e c k m o b ility . F u s e d
s e g m e n ts o f th e c e r v ic a l s p in e in p a tie n ts w ith th is s y n d r o m e p r o m o t e h y p e r m o b ilit y a t u n f u s e d
s p in e s e g m e n ts , in c r e a s in g th e r i s k o f n e u r o l o g i c c o m p r o m is e d u r in g n e c k m a n ip u la tio n . W h e n
y o u s e e K lip p e l- F e il, th in k K F : C e r v ic a l F u s io n .

5. D. A ir w a y m a n a g e m e n t o f p a tie n ts w ith tr i s o m y 21 (D o w n s y n d r o m e ) is c o m p lic a te d b y


s e v e r a l f a c to r s . T h e s e p a tie n ts te n d to h a v e s m a ll m o u th s a n d l a r g e to n g u e , r e s u ltin g in lim ite d
o r o p h a r y n g e a l s p a c e . T h e y a r e p r o n e to la r y n g o s p a s m . T h e y a ls o h a v e a h ig h in c id e n c e o f
s u b g lo ttic s te n o s is , s u c h th a t e n d o tr a c h e a l tu b e s s h o u ld b e d o w n s iz e d b y 0 .5 m m f r o m th e
c a lib e r e x p e c te d f o r a p a tie n t o f th e s a m e s iz e w ith o u t D o w n s y n d r o m e . F in a lly , th e y h a v e a
h ig h in c id e n c e o f c e r v ic a l s p in e in s ta b ility . T h e o th e r th r e e s y n d r o m e s s h a r e a c o m m o n f e a tu r e
o f m ic r o g n a th ia ( s m a ll ja w ) , w h ic h r e n d e r s th e s e p a tie n ts a c h a lle n g e f o r d ir e c t la r y n g o s c o p y .
T u r n e r s y n d r o m e o c c u r s in f e m a le s w h o la c k a c o m p le te s e c o n d X c h r o m o s o m e ( m o n o s o m y ) .
T h e s e w o m e n te n d to h a v e s h o r t n e c k s a n d s m a ll ja w s . L a r y n g e a l d is to r tio n ( c h o ic e A ) h a s n o t
b e e n d e s c r ib e d in th is p o p u la tio n . T r e a c h e r C o llin s is a r a r e s y n d r o m e c h a r a c te r iz e d b y
a b n o r m a l d e v e lo p m e n t o f f a c ia l b o n e s (e .g ., m a x illa r y a n d m a n d ib u la r ) . A h ig h in c id e n c e o f
c e r v ic a l s p in e in s ta b ility ( c h o ic e B ) h a s n o t b e e n d e s c r ib e d in th is p o p u la tio n . G o ld e n h a r
s y n d r o m e is m a n if e s te d b y d y s p la s tic g r o w th o f th e fa c e ( e s p e c ia lly th e e a r s , e y e s , a n d m o u th )
a n d v e r te b r a l a n o m a lie s (e .g ., s c o li o s is ) . A h ig h in c id e n c e o f s u b g lo ttic s te n o s is ( c h o ic e D ),
a lth o u g h a c o m m o n f in d in g in p a tie n ts w ith t r i s o m y 2 1 , h a s n o t b e e n d e s c r ib e d in th e
G o ld e n h a r p o p u la tio n .

6. A. A p o s itiv e - p r e s s u r e le a k te s t p r o v id e s i n f o r m a t io n a b o u t th e tig h tn e s s o f th e s e a l f o r m e d
b e tw e e n a n E T T a n d its s u r r o u n d in g m u c o s a . A le a k a t p r e s s u r e s b e lo w 2 5 c m H 2O p la c e s th e
tr a c h e a l m u c o s a a t a v e r y l o w r i s k o f is c h e m ic in ju r y . P r e s s u r e s a b o v e 3 0 c m H 2O , th e
a r t e r i o l a r - c a p i l l a r y p e r f u s io n p r e s s u r e , c a n c a u s e m u c o s a l is c h e m ia , w ith r e s u ltin g
in f la m m a tio n , u lc e r a tio n , s tr id o r , a n d la te r s c a r r i n g a n d s te n o s is .

7. C. W h e n a n e n d o tr a c h e a l tu b e m ig r a te s f r o m a n in tr a tr a c h e a l to a n e n d o b r o n c h ia l p o s itio n
w h ile o n v o lu m e - c o n tr o l v e n tila tio n , th e f i r s t s ig n o f m i g r a t i o n is g e n e r a lly a n in c r e a s e in p e a k
i n s p i r a t o r y p r e s s u r e s . P e a k i n s p i r a t o r y p r e s s u r e r e s u lts f r o m th e r e s is ta n c e to f lo w o f th e l a r g e
a ir w a y s a n d th e s ta tic c o m p lia n c e o f th e lu n g . A f ix e d v o lu m e o f a ir m o v in g o u t o f a n
e n d o b r o n c h ia l tu b e w o u ld e n c o u n te r s ig n if ic a n tly m o r e l a r g e a ir w a y r e s is ta n c e c o m p a r e d to
th e s a m e v o lu m e m o v in g o u t o f a n e n d o tr a c h e a l tu b e ( r e m e m b e r : r e s is ta n c e is in v e r s e ly
p r o p o r t i o n a l to r a d iu s r a is e d to th e f o u r th p o w e r ) . T h u s , th e f i r s t s ig n o f a n e n d o b r o n c h ia l
in tu b a tio n w o u ld b e a n e le v a tio n in p e a k i n s p i r a t o r y p r e s s u r e s . B e c a u s e th e n o n v e n tila te d lu n g
h a s s o m e r e s e r v e o f o x y g e n , p a s s iv e o x y g e n a tio n w o u ld d e la y o n s e t o f h y p o x e m ia b r i e f l y
( c h o ic e A ). H y p e r c a p n ia ( c h o ic e B ) w o u ld e v e n tu a lly d e v e lo p in a n e n d o b r o n c h ia ll y in tu b a te d
p a tie n t o n c o n tr o lle d v e n tila tio n if th e m in u te v e n tila tio n w e r e k e p t c o n s ta n t. H y p o te n s io n
( c h o ic e D ) m ig h t o c c u r if th e r i g h t lu n g is a llo w e d to h y p e r in f la te , r e s tr ic tin g v e n o u s r e tu r n .
H o w e v e r, th is w o u ld n o t b e a s im m e d ia te a s a r i s e in p e a k i n s p i r a t o r y p r e s s u r e s .

8. C. P a lp a tio n o f th e e n d o tr a c h e a l tu b e c u f f p a lp a b le a b o v e th e c r i c o i d c a r tila g e im p lie s th a t th e


c u f f ’s p o s itio n is in tr a la r y n g e a l . T h is is p r o b le m a tic f o r tw o r e a s o n s : (1 ) a n in f la te d c u f f in th e
la r y n x m a y c a u s e la r y n g e a l in ju r y a n d p o s to p e r a tiv e r e s p i r a t o r y c o m p r o m is e , a n d (2 ) s u c h a
h ig h tu b e p o s itio n m a y in c r e a s e th e r i s k o f in a d v e r te n t e x tu b a tio n . T h e c u f f s h o u ld b e d e fla te d
a n d th e tu b e a d v a n c e d u n til th e c u f f (w h e n in fla te d ) is p a lp a b le b e lo w th e c r i c o i d c a r tila g e .
C h o ic e B is in c o r r e c t: a c u f f in f la te d to e n a b le a ir le a k a t 2 0 to 2 5 c m H 2O o f p o s itiv e p r e s s u r e
s h o u ld n o t c a u s e a ir w a y in ju r y a n d e d e m a in r o u tin e c ir c u m s ta n c e s . C h o ic e D w o u ld lik e ly
r e s u lt in th e tip o f th e e n d o tr a c h e a l tu b e m o v in g f r o m a n in tr a la r y n g e a l to a s u p r a la r y n g e a l
p o s itio n .

9. D. T h e s c e n a r io d e s c r ib e s e x tu b a tio n o f a c h ild d u r in g a “ l i g h t ” p la n e o f a n e s th e s ia w h e n
la r y n g e a l r e f le x e s a r e h y p e r s e n s itiv e . R e tu rn o f a g a g r e f le x is a c h a r a c te r is tic o f th is lig h te r ,
h y p e r e x c ita b le s ta g e . If a p a tie n t is e x tu b a te d w h ile lig h tly a n e s th e tiz e d , th e r e is a n in c r e a s e d
r i s k o f la r y n g o s p a s m . S tim u la tio n o f th e la r y n g e a l m u c o s a b y s e c r e tio n s o r a f o r e i g n b o d y
(e .g ., th e e n d o tr a c h e a l tu b e o r a n o r a l a ir w a y ) c a n r e s u lt in la r y n g o s p a s m d u r in g th e e x c ita tio n
s ta g e o f a n e s th e s ia o r s o m e tim e s e v e n d u r in g a w a k e sta te s.
L a r y n g o s p a s m a n d o th e r c a u s e s o f u p p e r a ir w a y o b s tr u c tio n (e .g ., to n g u e c o lla p s e d a g a in s t
th e p o s t e r i o r p h a r y n g e a l w a ll) m a y n o t b e im m e d ia te ly d is tin g u is h a b le . H o w e v e r, th e in itia l
tr e a tm e n t is id e n tic a l: a n te r io r d is p la c e m e n t o f th e m a n d ib le u s in g a c h in lif t o r j a w th r u s t
c o m b in e d w ith p o s itiv e - p r e s s u r e v e n tila tio n . If th e s e m e a s u r e s f a il to r e lie v e th e la r y n g o s p a s m
a n d h y p o x e m ia d e v e lo p s , p h a r m a c o lo g i c th e r a p y s h o u ld b e in itia te d e m e r g e n tly . In a p a tie n t
w ith n o c o n tr a in d ic a tio n s , a s m a ll d o s e o f s u c c in y lc h o lin e (0 .2 5 - 0 .5 m g /k g ) o r d e e p e n in g o f th e
a n e s th e tic (e .g ., w ith p r o p o f o l o r a n o th e r g e n e r a l a n e s th e tic ) s h o u ld b r e a k th e la r y n g o s p a s m .

10. C. L e ft u n tre a te d , u p p e r a ir w a y o b s tr u c tio n in a s p o n ta n e o u s ly b r e a th in g p a tie n t c a n r e s u lt in


th e d e v e lo p m e n t o f n e g a ti v e - p r e s s u r e p u lm o n a r y e d e m a (a ls o c a lle d p o s to b s tr u c tiv e p u lm o n a r y
e d e m a ). F o r c e f u l in s p ir a tio n a g a in s t a c lo s e d u p p e r a ir w a y g e n e r a te s a l a r g e n e g a tiv e
in tr a th o r a c ic p r e s s u r e w h ic h c a n r e s u lt in p u lm o n a r y e d e m a b y in c r e a s in g c a p il la r y tr a n s m u r a l
p r e s s u r e a n d /o r b y a c u te ly e le v a tin g le f t v e n tr ic u la r e n d - d ia s to lic p r e s s u r e . A s p ir a tio n ( c h o ic e
A ) w o u ld b e im p o s s i b le d u r in g la r y n g o s p a s m . B r o n c h o s p a s m ( c h o ic e B ) w o u ld n o t b e e x p e c te d
a s a d ir e c t c o n s e q u e n c e o f p r o l o n g e d la r y n g o s p a s m . C r o u p o r l a r y n g o tr a c h e a l b r o n c h itis
( c h o ic e D ) is a f o r m o f u p p e r a ir w a y o b s tr u c tio n th a t ty p ic a lly o c c u r s in r e s p o n s e to a v i r a l o r
b a c te r ia l u p p e r r e s p i r a t o r y tr a c t in f e c tio n in c h ild r e n b e tw e e n th e a g e s o f 6 m o n th s a n d 6 y e a r s .
T h is c lin ic a l s c e n a r io is n o t s u g g e s tiv e o f a n in f e c tio u s e t i o l o g y f o r th e u p p e r a ir w a y
o b s tr u c tio n .

11. B. A c o m m o n ly u s e d f o r m u l a f o r e s tim a tin g th e in te r n a l d ia m e te r o f a n uncuffed e n d o tr a c h e a l


tu b e in c h ild r e n is

In te rn a l d ia m e te r (in m m ) = (A g e + 1 6 )/4

T h e r e s u ltin g v a lu e s h o u ld b e r e d u c e d b y 0 .5 m m w h e n u s in g a cuffed e n d o tr a c h e a l tu b e to
a llo w s p a c e in th e tr a c h e a l lu m e n f o r c u f f in f la tio n . S in c e th is c h ild a p p e a r s to h a v e a h e ig h t a n d
w e ig h t a p p r o p r ia te f o r h e r a g e , th e f o r m u l a w o u ld b e r e a s o n a b le to u s e . F o r th e p a tie n t in th is
q u e s tio n , th e in te r n a l d ia m e te r is (2 + 1 6 )/4 = 4 .5 m m f o r a n u n c u f f e d tu b e , w h ic h is r e d u c e d to
4 .0 m m f o r a c u f f e d tu b e .

12. C. T h e v a g u s n e r v e p r o v id e s s e n s o r y in n e r v a tio n to th e s tr u c tu r e s o f th e a ir w a y b e g in n in g
w ith th e e p ig lo ttis a n d m o v i n g c a u d a lly . It h a s tw o m a j o r b r a n c h e s th a t in n e r v a te d is tin c t p a rts
o f th e a irw a y : th e s u p e r io r la r y n g e a l n e r v e (S L N ) a n d r e c u r r e n t la r y n g e a l n e r v e (R L N ). A b o v e
th e v o c a l c o r d s , th e s e n s o r y in n e r v a tio n o f th e la r y n x is v ia th e S L N . B e lo w th e v o c a l fo ld s ,
s e n s o r y in n e r v a tio n o f th e a ir w a y is p r o v id e d b y b r a n c h e s o f th e r e c u r r e n t R L N . T h e v o c a l
c o r d s th e m s e lv e s r e c e iv e d u a l in n e r v a tio n f r o m b o th n e r v e s . T h e S L N h a s tw o b r a n c h e s :
in te r n a l a n d e x te r n a l. T h e in te r n a l S L N b r a n c h ( c h o ic e C ) is e x c lu s iv e ly a s e n s o r y n e r v e th a t
in n e r v a te s b o th th e s u p e r io r a n d i n f e r i o r s u r f a c e s o f th e e p ig lo ttis . T h e e x te r n a l b r a n c h o f th e
S L N is a m o to r n e r v e th a t in n e r v a te s th e c r i c o t h y r o i d m u s c le ( F ig . 2 - 2 ). T h e R L N ( c h o ic e B ) is
a m ix e d m o to r a n d s e n s o r y n e r v e . T h e m o to r b r a n c h in n e r v a te s a ll o f th e la r y n g e a l m u s c le s ,
e x c e p t th e c r i c o t h y r o i d m u s c le , w h ile th e s e n s o r y b r a n c h in n e r v a te s th e s u b g lo ttic m u c o s a o f
th e a irw a y . T h e h y p o g lo s s a l n e r v e ( c h o ic e A ) is a p u r e ly m o t o r n e r v e th a t in n e r v a te s th e
m u s c l e s o f th e to n g u e .
Thyrohyoid Superior
Internal branch
larynqeal n.
of SLN
(SLN)
Epiglottis External
branch of
Thyroid cartilage
SLN
Transverse
Vocal
arytenoid m.
cords
Cricoid Cricothyroid m.
cartilage

Trachea

F igu re 2-2. Subdivisions of the superior laryngeal nerve in the sagittal view.

13. A. T h e to n g u e h a s in n e r v a tio n f o r b o th g u s ta to r y (a k a “ ta s te ” ) a n d ta c tile ( g e n e r a l s e n s o r y )


in p u t. G u s ta to r y (ta s te ) s e n s a tio n f o r th e a n te r io r tw o - th ir d s o f th e to n g u e is p r o v id e d b y th e
f a c ia l n e r v e (C N V II), a n d f o r th e p o s te r io r th ir d o f th e to n g u e b y th e g lo s s o p h a r y n g e a l n e r v e
(C N IX ). T a c tile s e n s a tio n f o r th e a n te r io r tw o - th ir d s o f th e to n g u e is p r o v id e d b y th e tr i g e m in a l
n e r v e (C N V ), a n d f o r th e p o s te r io r o n e - th ir d o f th e to n g u e b y th e g lo s s o p h a r y n g e a l n e r v e (C N
IX ). In a d d itio n , a s m a ll p o r t i o n o f s e n s o r y in n e r v a tio n o f th e p o s te r io r to n g u e is p r o v id e d b y
f ib e r s o f th e s u p e r io r la r y n g e a l n e r v e ’s in te r n a l b r a n c h ( “ s p illo v e r f i b e r s ” f r o m th a t n e r v e ’s
in n e r v a tio n o f th e e p ig lo ttis ) ( F ig . 2 - 3 ). T h e h y p o g lo s s a l n e r v e ( c h o ic e D ) is a p u r e ly m o to r
n e r v e th a t in n e r v a te s th e m u s c le s o f th e to n g u e .
14. C. T h e g l o s s o p h a r y n g e a l n e r v e (C N IX ) is a m ix e d m o to r a n d s e n s o r y n e r v e . Its s e n s o r y
f ib e r s c a r r y i n f o r m a t io n a b o u t g e n e r a l s e n s a tio n a n d ta ste f r o m th e p o s te r io r th ir d o f th e to n g u e
( F ig . 2 - 3 ). O f n o te , th e g l o s s o p h a r y n g e a l n e r v e d o e s n o t p r o v id e s e n s o r y in n e r v a tio n to th e
e p ig lo ttis ; it is p r o v id e d b y th e s u p e r io r la r y n g e a l n e r v e . T h e tr i g e m in a l n e r v e (C N V ) ( c h o ic e
A ) c a r r ie s g e n e r a l s e n s o r y i n f o r m a t io n f r o m th e a n te r io r tw o - th ir d s o f th e to n g u e . T h e fa c ia l
n e r v e (C N V II) ( c h o ic e B ) is a m ix e d m o to r a n d s e n s o r y n e r v e . It c a r r ie s ta ste s e n s a tio n f r o m
th e a n te r io r tw o - th ir d s o f th e to n g u e a n d o r a l c a v ity . T h e h y p o g lo s s a l n e r v e (C N X II) ( c h o ic e
D ) is a p u r e ly m o to r n e r v e th a t in n e r v a te s th e m u s c le s o f th e to n g u e .

15. B. T h e s u p e r io r la r y n g e a l n e r v e (S L N ) is a m ix e d m o to r a n d s e n s o r y n e r v e th a t r e c e iv e s
s e n s o r y i n f o r m a t io n f r o m th e s u p r a g lo ttic la r y n x a n d p r o v id e s m o to r in n e r v a tio n to th e
c r i c o t h y r o i d m u s c le . T h e c r i c o t h y r o i d m u s c le te n s e s a n d a d d u c ts th e v o c a l c o r d s . T h is a c tio n
r a is e s th e p itc h o f s p e e c h a n d e n a b le s s in g in g . A c u te , b ila te r a l d e n e r v a tio n o f th e e x te r n a l
b r a n c h o f th e S L N m a y c a u s e h o a r s e n e s s a n d o th e r s u b tle v o ic e f in d in g s . H o w e v e r, th e a b ility to
a d d u c t a n d a b d u c t th e v o c a l c o r d s w o u ld r e m a in in ta c t.

16. C. S e n s o r y in n e r v a tio n o f th e la r y n x a b o v e th e v o c a l c o r d s is c a r r i e d b y f ib e r s o f th e s u p e r io r
la r y n g e a l n e r v e (S L N ). T h e in te r n a l b r a n c h o f th e S L N p r o v id e s s e n s o r y in n e r v a tio n to th e
s u p r a g lo ttic p o r t i o n o f th e la r y n x , in c lu d in g a ll o f th e e p ig lo ttis a n d th e s u p r a g lo ttic m u c o s a .
T h e e x te r n a l b r a n c h o f th e S L N is p r i m a r i l y a m o to r n e r v e th a t in n e r v a te s th e c r ic o th y r o id
m u s c le . T h e S L N c a n b e b lo c k e d a s it d e s c e n d s b e tw e e n th e g r e a te r c o r n u o f th e h y o id b o n e a n d
th e s u p e r io r c o r n u o f th e th y r o i d c a r tila g e . A s s h o w n in F ig u r e 2 - 4 , “ S L N b l o c k ” is lik e ly to
b lo c k th e in te r n a l b r a n c h o f th e S L N , b u t n o t th e e x te r n a l “ m o t o r ” b r a n c h . C h o ic e A d e s c r ib e s a
g lo s s o p h a r y n g e a l b lo c k . C h o ic e B d o e s n o t d e s c r ib e a c lin ic a ll y r e le v a n t p r o c e d u r e (i.e., th e
in je c tio n w o u ld b e to o m e d ia l to r e l i a b l y b lo c k th e S L N ). C h o ic e D d e s c r ib e s a tr a n s tr a c h e a l
to p ic a liz a tio n o f R L N f ib e r s .

17. C. T h e e f f e r e n t lim b o f th e g lo ttic c lo s u r e r e f le x in v o lv e d in la r y n g o s p a s m is p r i m a r i l y


m e d ia te d b y th e r e c u r r e n t la r y n g e a l n e r v e (R L N ), w h ile th e a f f e r e n t lim b is m e d ia te d b y th e
s u p e r io r la r y n g e a l n e r v e (S L N ). T h e R L N in n e r v a te s a ll o f th e m u s c le s o f th e la r y n x e x c e p t th e
c r i c o t h y r o i d m u s c le . T h e e x te r n a l b r a n c h o f th e S L N ( n o t o n e o f th e lis te d o p tio n s ) is a m o to r
n e r v e th a t in n e r v a te s th e c r i c o t h y r o i d m u s c le . T h e c r i c o t h y r o i d m u s c le c o n tr ib u te s to
la r y n g o s p a s m b y le n g th e n in g , a n d th u s te n s in g th e v o c a l c o r d s .

18. B. T h e p r e s e n ta tio n o f a c u te a p h o n ia a n d r e s p i r a t o r y d is tr e s s im m e d ia te ly a fte r th y r o id e c to m y


a r e s u g g e s tiv e o f b ila te r a l in ju r y to th e r e c u r r e n t la r y n g e a l n e r v e (R L N ), a r e c o g n iz e d
c o m p lic a tio n o f th is s u r g e r y . B ila te r a l R L N in ju r y le a v e s th e v o c a l c o r d s te n s e d a n d c lo s e d d u e
to th e u n o p p o s e d a c tio n o f th e c r i c o t h y r o i d m u s c le s . T h e c r i c o t h y r o i d m u s c le is in n e r v a te d b y
th e e x te r n a l ( m o t o r ) b r a n c h o f th e s u p e r io r la r y n g e a l n e r v e (S L N ). B lo c k a d e o f th e m o to r
b r a n c h o f th e S L N s h o u ld im p r o v e th e p a tie n t’s r e s p i r a t o r y d is tr e s s b y r e la x in g th e v o c a l c o r d s
b u t w o u ld h a v e n o im p a c t o n th e a p h o n ia . P r a c tic a lly s p e a k in g , a ty p ic a l “ S L N b l o c k ” (i.e.,
in je c tio n o f —2 m L o f lo c a l a n e s th e tic b e tw e e n th e g r e a te r c o r n u o f th e h y o id c a r tila g e a n d th e
s u p e r io r c o r n u o f th e th y r o i d c a r tila g e ) is lik e ly to o n ly b lo c k th e in te r n a l ( s e n s o r y ) b r a n c h o f
th is n e r v e a s o p p o s e d to th e m o to r b r a n c h ( F ig . 2 - 4 ).
POSTERIOR ANTERIOR
Hyoid bone
Thyrohyoid Superior
Internal branch laryngeal n.
of SLN (SLN)
Epiglottis Externa!
branch of
Thyroid cartilage SLN
Transverse
Vocal
arytenoid m.
cords
Cricoid Cricothyroid m.
cartilage

Trachea
Recuurent
laryngeal n.

F igu re 2-4. Gross anatomic distribution of the SLN and RLN.

19. A. A c o u g h o c c u r s th r o u g h th e s tim u la tio n o f a c o m p le x r e f le x a rc . T h is is in itia te d b y th e


ir r i t a t i o n o f c o u g h r e c e p to r s , w h ic h a r e f o u n d in th e p h a r y n x , la r y n x , tra c h e a , c a r in a , b r a n c h in g
p o in ts o f l a r g e a ir w a y s , a n d m o r e d is ta l s m a lle r a ir w a y s . W h e n t r i g g e r e d , im p u ls e s tr a v e l v ia
th e in te r n a l b r a n c h o f th e s u p e r io r la r y n g e a l n e r v e a n d th e r e c u r r e n t la r y n g e a l n e r v e , w h ic h
s te m f r o m th e v a g u s n e r v e , to th e m e d u lla o f th e b r a in . T h is is th e a f f e r e n t n e u r a l p a th w a y . T h e
e f f e r e n t n e u r a l p a th w a y th e n f o llo w s , w ith r e le v a n t s ig n a ls tr a n s m itte d b a c k f r o m th e c e r e b r a l
c o r te x a n d m e d u lla v ia th e v a g u s a n d s u p e r io r la r y n g e a l n e r v e s to th e g lo ttis , e x te r n a l
in te r c o s ta ls , d ia p h r a g m , a n d o th e r m a jo r i n s p i r a t o r y a n d e x p ir a to r y m u s c le s .

20. B. A c u te , b ila te r a l in ju r y to th e v a g u s n e r v e (C N X ) te r m in a te s a ll o f th e m o to r in n e r v a tio n to


th e la r y n x . T h is le a v e s th e v o c a l c o r d s in a f u lly o p e n o r a b d u c te d p o s itio n . In c o n tr a s t, b ila te r a l
in ju r y to th e r e c u r r e n t la r y n g e a l n e r v e (a b r a n c h o f th e v a g u s ) w o u ld le a v e th e c o r d s p a r a ly z e d
in a p a r tia lly a d d u c te d p o s itio n b e c a u s e o f u n o p p o s e d a c tio n o f th e c r i c o t h y r o i d m u s c le . T h is
a d d u c te d p o s itio n m a y c a u s e s tr id o r a n d r e s p i r a t o r y d is tr e s s , e s p e c ia lly if th e p a tie n t h a s a n y
c o n c u r r e n t la r y n g e a l e d e m a . C h o ic e A w o u ld b e o b s e r v e d d u r in g la r y n g o s p a s m . C h o ic e D
w o u ld b e o b s e r v e d in a p a tie n t w ith a n o r m a l la r y n x w h o is a lte r n a tin g b e tw e e n b r e a th in g a n d
p h o n a tin g .

21. C. P o s to p e r a tiv e h o a r s e n e s s c a n r e s u lt f r o m in ju r y to th e m o to r n e r v e s w h ic h in n e r v a te th e
la r y n x . T h e le f t r e c u r r e n t la r y n g e a l n e r v e (R L N ) is p a r tic u la r ly v u ln e r a b le to in ju r y d u r in g
c a r d io th o r a c ic s u r g e r i e s a n d m a n y n e c k s u r g e r i e s d u e to its a n a to m ic lo c a tio n . A fte r b r a n c h in g
o f f th e le f t v a g u s n e r v e in th e c h e s t, th e le f t R L N p a s s e s b e tw e e n th e le f t p u lm o n a r y a r te r y a n d
th e a r c h o f th e a o r ta a b o v e b e f o r e a s c e n d in g a lo n g s i d e th e tr a c h e a to th e la r y n x . T h e r i g h t R L N ,
in c o n tr a s t, b r a n c h e s o f f th e r i g h t v a g u s n e r v e in th e lo w e r n e c k w h e r e it p a s s e s u n d e r th e r o o t
o f th e r i g h t s u b c la v ia n a r te r y b e f o r e a s c e n d in g a lo n g s i d e th e tr a c h e a to th e la r y n x . A n a o r tic
a r c h r e p a ir th a t s p a r e s th e a r c h v e s s e ls w o u ld b e m o r e lik e ly to d a m a g e th e le f t R L N th a n th e
rig h t R LN .
A c u te in ju r y to th e le f t R L N w o u ld le a v e th e le f t v o c a l c o r d s u b je c t to th e u n o p p o s e d a c tio n
o f th e c r i c o t h y r o i d m u s c le (th e o n ly la r y n g e a l m u s c le N O T in n e r v a te d b y th e R L N ). T h is
m u s c le s tre tc h e s a n d te n s e s th e v o c a l c o r d s , a n a c tio n th a t s h ifts th e v o c a l c o r d s to w a r d m id lin e
(a d d u c tio n ). D u r in g in s p ir a tio n , b o th v o c a l c o r d s n o r m a l l y a b d u c t, m a x im iz in g th e g lo ttic
o p e n in g f o r a ir m o v e m e n t. D u r in g in s p ir a tio n , a p a tie n t w ith a c u te le f t R L N p a ls y w o u ld b e
e x p e c te d to h a v e a n a d d u c te d le f t v o c a l c o r d a n d a n a b d u c te d r i g h t v o c a l c o r d .

22. D. A b o v e th e v o c a l c o r d s , th e s e n s o r y in n e r v a tio n o f th e la r y n x is v ia th e s u p e r io r la r y n g e a l
n e r v e . B e lo w th e v o c a l c o r d s , s e n s o r y in n e r v a tio n is v ia b r a n c h e s o f th e r e c u r r e n t la r y n g e a l
n e r v e (R L N ). T h e v o c a l c o r d s th e m s e lv e s r e c e iv e d u a l in n e r v a tio n f r o m b o th n e r v e s . T h e
tr i g e m in a l n e r v e ( c h o ic e A ) p r o v id e s ta c tile s e n s a tio n , a m o n g o th e r th in g s , to th e a n te r io r tw o -
th ir d s o f th e to n g u e a n d th e n a s a l p a s s a g e s . T h e g l o s s o p h a r y n g e a l n e r v e ( c h o ic e B ) p r o v id e s
ta c tile a n d g u s ta to r y s e n s a tio n to th e p o s t e r i o r o n e - th ir d o f th e to n g u e . N o n e o f th e c h o ic e s
e x c e p t f o r th e R L N w o u ld b e s tim u la te d d u r in g a n a w a k e tr a c h e o s to m y .

23. B. T h e o p h th a lm ic (V 1 ) a n d m a x illa r y (V 2 ) d iv is io n s o f th e tr i g e m in a l n e r v e (C N V ) c o n v e y
s e n s o r y i n f o r m a t io n f r o m th e n a s a l m u c o s a . B lo c k a d e o f th e s e n e r v e s w o u ld f a c ilita te a w a k e
n a s o tr a c h e a l in tu b a tio n . T h e g a g r e f le x is e lic ite d p r i m a r i l y b y ta c tile s tim u la tio n o f th e
p o s te r io r o n e - th ir d o f th e to n g u e . T h e a f f e r e n t lim b o f th is r e f le x is c a r r i e d b y th e
g lo s s o p h a r y n g e a l n e r v e (C N IX ), n o t th e r e c u r r e n t la r y n g e a l n e r v e ( c h o ic e A ) o r th e
h y p o g l o s s a l n e r v e ( c h o ic e D ). T h e s u p e r io r s u r f a c e o f th e e p ig lo ttis is in n e r v a te d b y th e
s u p e r io r la r y n g e a l n e r v e (S L N ), n o t th e g lo s s o p h a r y n g e a l ( c h o ic e C ). In g e n e r a l, th e S L N
p r o v id e s s e n s o r y in n e r v a tio n to a ll s tr u c tu r e s o f th e la r y n x a b o v e th e v o c a l c o r d s , in c lu d in g th e
e p ig lo ttis .

24. C. B o th n a s o tr a c h e a l in tu b a tio n a n d n a s a l tr u m p e t in s e r tio n a r e c o n tr a in d ic a te d in p a tie n ts w ith


f a c ia l o r s k u ll in ju r ie s ( c h o ic e A ), w ith c o a g u lo p a th y ( c h o ic e B ), a n d th o s e o n a n tic o a g u la tio n
( c h o ic e D ). In c h o ic e A , th e p a tie n t’s m e c h a n is m o f in ju r y a n d f in d in g s o f p e r io r b it a l b r u is in g
s u g g e s t a n u n d e r ly in g s k u ll f r a c tu r e . In a d d itio n to p e r io r b it a l e c c h y m o s e s , o th e r c la s s ic s ig n s
o f a b a s ila r s k u ll f r a c tu r e in c lu d e le a k a g e o f b l o o d o r c e r e b r o s p in a l f lu id f r o m th e n a r e s ,
e c c h y m o s e s o n th e s k in o v e r ly in g th e m a s to id p r o c e s s , a n d h e m o ty m p a n u m o r b le e d in g f r o m
th e e a r s . F o r a p a tie n t w ith te m p o r o m a n d ib u la r j o i n t d y s f u n c tio n w h o h a s n o n e o f th e a b o v e
c o n tr a in d ic a tio n s ( c h o ic e C ), th e n a s a l tr u m p e t w o u ld b e a r e a s o n a b le w a y to b y p a s s th e
p a tie n t’s lim ite d m o u th o p e n in g a n d r e lie v e u p p e r a ir w a y o b s tr u c tio n .

25. D. T h is p a tie n t h a s m u ltip le r i s k f a c to r s f o r d if f ic u lt in tu b a tio n , in c lu d in g M a lla m p a ti c la s s >


2, th y r o m e n ta l d is ta n c e < 3 f in g e r b r e a d th s , m o u th o p e n in g < 3 f in g e r b r e a d th s , a n d to ta l a tla n to -
o c c ip ita l r a n g e - o f - m o t i o n < 8 0 d e g r e e s . P a tie n ts w ith in f l a m m a to r y r h e u m a to id a r th r itis (R A )
h a v e a n in c r e a s e d in c id e n c e o f te m p o r o m a n d ib u la r j o i n t d is e a s e (a n d a s s o c ia te d lim ite d m o u th
o p e n in g ) a n d im m o b il e c e r v ic a l v e r te b r a ( a s s o c ia te d w ith lim ite d n e c k r a n g e - o f - m o t io n ) .
A d d itio n a lly , p a tie n ts w ith R A c a n h a v e o c c u lt a ir w a y a b n o r m a litie s n o t a p p a r e n t o n p h y s ic a l
e x a m , s u c h a s la r y n g e a l r o ta tio n , c r ic o a r y te n o id a r th r itis , a n d c e r v ic a l s p in e in s ta b ility . T h e
p a tie n t’s th y r o i d m a lig n a n c y m a y r e s u lt in o th e r a ir w a y a b n o r m a litie s in c lu d in g tr a c h e a l
d e v ia tio n a n d /o r c o m p r e s s io n . W e re s u c h a p a tie n t to b e in d u c e d a n d m a s k v e n tila tio n tu r n o u t
to u n s u c c e s s f u l, th e r e w o u ld b e n o r e lia b le b a c k u p m e th o d o f a ir w a y m a n a g e m e n t. T h e s a f e s t
w a y to s e c u r e th is p a tie n t’s a ir w a y w o u ld b e a n a w a k e f ib e r o p tic in tu b a tio n . S in c e th e p a tie n t h a s
r e f u s e d th is o p tio n a n d th e c a s e is n o t u r g e n t, th e a n e s th e s io l o g is t s h o u ld c a n c e l th e o p e r a tio n
a n d d is c u s s th e o p tio n s f o r a ir w a y m a n a g e m e n t w ith th e p a tie n t so th a t a m u tu a lly a c c e p ta b le
p la n c a n b e re a c h e d .

26. B. T h e “ c a n n o t in tu b a te , c a n n o t v e n tila te ” s c e n a r io is a n e m e r g e n c y a n d n e c e s s ita te s


im m e d ia te in v a s iv e a ir w a y a c c e s s to p r e v e n t a n o x ic in ju r y . T w o o p tio n s in c lu d e tr a n s tr a c h e a l
j e t v e n tila tio n a n d s u r g ic a l c r ic o th y r o to m y . T r a n s tr a c h e a l j e t v e n tila tio n r e q u ir e s th a t th e a ir w a y
b e c a n n u la te d in s o m e w a y . In e m e r g e n t c ir c u m s ta n c e s , th is m a y b e a c c o m p lis h e d b y
c a n n u la tin g th e c r i c o t h y r o i d m e m b r a n e w ith a n in tr a v e n o u s c a th e te r (e .g ., 1 4 /1 6 G ) a n d th e n
a tta c h in g th e e n d o f th e c a th e te r to a j e t v e n tila to r . J e t v e n tila tio n r e q u ir e s a p a th w a y f o r e x p ir e d
a ir to e g r e s s o u t o f th e lu n g s . T h u s , w h e n u s in g tr a n s tr a c h e a l v e n tila tio n , la r y n g o s p a s m ( c h o ic e
C ), o r a n o th e r c a u s e o f u p p e r a ir w a y o b s tr u c tio n ( c h o ic e A ), w o u ld r a p id ly c a u s e p u lm o n a r y
o v e r in f la tio n a n d b a r o tr a u m a . In c o n tr a s t, a s u r g ic a l c r ic o th y r o to m y p e r m its b o th in h a la tio n
a n d e x h a la tio n th r o u g h th e lu m e n o f in s e r te d tu b e ( c h o ic e B ) a n d so is n o t d e p e n d e n t o n u p p e r
a ir w a y p a te n c y in o r d e r to f u n c tio n s a fe ly . T r a n s tr a c h e a l je t v e n tila tio n is a te m p o r a r y w a y to
p r o v id e o x y g e n a tio n u n til a d e f in itiv e a ir w a y c a n b e e s ta b lis h e d . W ith p r o l o n g e d j e t v e n tila tio n ,
th e d e liv e r e d h ig h p r e s s u r e s c a n e x p e l th e c a th e te r o u t o f th e tr a c h e a . W h e n th e c a th e te r
m ig r a te s in to th e a n te r io r c e r v ic a l s o ft tis s u e s , c a ta s tr o p h ic s u b c u ta n e o u s e m p h y s e m a c a n
r a p id ly d e v e lo p r e n d e r in g o th e r a tte m p ts at in v a s iv e a ir w a y a c c e s s im p o s s ib le . S u r g ic a l
c r ic o th y r o to m y , o n th e o th e r h a n d , is a d e f in itiv e m e th o d o f s e c u r in g th e a ir w a y th a t c a n b e
u s e d f o r u p to 72 h o u r s .

27. A. T h e A S A D if f ic u lt A ir w a y A lg o r it h m r e c o m m e n d s u s e o f s u p r a g lo ttic d e v ic e s s u c h a s th e
la r y n g e a l m a s k a ir w a y (L M A ) a s r e s c u e to o ls w h e n l a r y n g o s c o p y a n d m a s k v e n tila tio n a r e
u n s u c c e s s f u l. A lth o u g h th e p a tie n t in c h o ic e A id e a l ly w o u ld b e tr e a te d w ith “ fu ll s to m a c h ”
p r e c a u tio n s , if a r a p id s e q u e n c e in d u c tio n a n d in tu b a tio n a r e u n s u c c e s s f u l, a n L M A m a y b e a
lif e - s a v in g to o l to o x y g e n a te a n d v e n tila te th e p a tie n t. A s id e f r o m its u s e a s a r e s c u e d e v ic e , th e
L M A c a n b e u s e d a s a s u p r a g lo ttic a ir w a y f o r e le c tiv e s u r g e r y . R e la tiv e c o n tr a in d ic a tio n s to th e
e le c tiv e u s e o f th e L M A in c lu d e lo w a ir w a y c o m p lia n c e ( c h o ic e s B a n d C ), in c o m p e te n c e o f th e
g a s tr o e s o p h a g e a l s p h in c te r ( c h o ic e C ), a n d in p a tie n ts w ith a fu ll s to m a c h ( c h o ic e D ).

28. D. “ D e e p e x tu b a tio n ” r e f e r s to th e te c h n iq u e o f r e m o v in g th e e n d o tr a c h e a l tu b e in a p a tie n t


b r e a th in g s p o n ta n e o u s ly w h o r e m a in s a n e s th e tiz e d s u c h th a t h is o r h e r p r o te c tiv e a ir w a y
r e f le x e s a r e s till a b o lis h e d . T h is te c h n iq u e d e c r e a s e s th e c h a n c e o f a p a tie n t c o u g h in g d u r in g
e m e r g e n c e in r e s p o n s e to th e p r e s e n c e o f a n e n d o tr a c h e a l tu b e . D e e p e x tu b a tio n m a y b e
p e r f o r m e d b e c a u s e o f p o te n tia l b e n e f it r e la te d to a p a tie n t’s m e d ic a l c o m o r b id iti e s o r f o r
s u r g ic a l r e a s o n s . F o r e x a m p le , a p a tie n t w ith c o r o n a r y a r te r y d is e a s e o r h e a r t f a ilu r e m a y
b e n e f it f r o m d e e p e x tu b a tio n to a v o id th e s y m p a th e tic s u r g e a s s o c ia te d w ith a w a k e e x tu b a tio n
a n d a p a tie n t u n d e r g o in g a b d o m in a l h e r n i a r e p a ir m a y b e n e f it f r o m d e e p e x tu b a tio n to a v o id
th e in c r e a s e d in tr a - a b d o m in a l p r e s s u r e a s s o c ia te d w ith c o u g h in g . H o w e v e r, d e e p e x tu b a tio n
s h o u ld n o t b e a tte m p te d in p a tie n ts w ith c o n tr a in d ic a tio n s to th is te c h n iq u e . T h e s e in c lu d e
p a tie n ts w ith a fu ll s to m a c h ( c h o ic e s A a n d B ) a n d in p a tie n ts w h o m a y b e c h a lle n g in g to m a s k
v e n tila te o r re in tu b a te . C h o ic e C w o u ld f a ll in to th is la tte r c a te g o r y b e c a u s e o f th e p o te n tia l f o r
a ir w a y e d e m a f r o m p r o l o n g e d p r o n e p o s itio n in g .

29. D. M a s k v e n tila tio n c a n b e m a d e d if f ic u lt b y a n y th in g th a t p re v e n ts th e fa c e m a s k f r o m


f o r m in g a n a d e q u a te s e a l w ith th e p a tie n t’s fa c e (e .g ., a b e a r d ) o r in c r e a s e s th e r e s is ta n c e to
a i r f l o w b e tw e e n th e m o u th a n d la r y n x . E d e n tu lo u s n e s s , a h is to r y o f s n o r in g , h is to r y o f n e c k
r a d ia tio n , m u ltip le a tte m p ts a t la r y n g o s c o p y , m a le g e n d e r , o b e s ity , a n d M a lla m p a ti sta tu s > 3 a r e
a ll f a c to r s a s s o c ia te d w ith d if f ic u lt m a s k v e n tila tio n . C h o ic e s A , B , a n d C r e p r e s e n t r i s k f a c to r s
f o r d if f ic u lt in tu b a tio n . In g e n e r a l, f a c to r s th a t m a k e it d if f ic u lt to a lig n th e o r a l a x is w ith th e
la r y n g e a l a x is r e s u lt in d if f ic u lt in tu b a tio n . T h e s e f a c to r s in c lu d e p r o m i n e n t m a x illa r y te e th , a
h ig h ly a r c h e d o r v e r y n a r r o w p a la te , a n d a n a c u te a n g le b e tw e e n th e m o u th a n d la r y n x .

30. B. F l o w - v o l u m e l o o p s c a n h e lp d if f e r e n tia te f ix e d v s . d y n a m ic c a u s e s o f a ir w a y o b s tr u c tio n .


T h e y c a n a ls o h e lp to d is tin g u is h e x tr a th o r a c ic v s . in tr a th o r a c ic s o u r c e s o f th e o b s tr u c tio n .
D u r in g th e i n s p i r a t o r y p h a s e o f s p o n ta n e o u s v e n tila tio n , a n e x tr a th o r a c ic o b s tr u c tio n is d r a w n
in to th e p a th w a y o f a ir m o v e m e n t b y s u b a tm o s p h e r ic in tr a lu m in a l p r e s s u r e s . In c o n tr a s t, a n
in tr a th o r a c ic o b s tr u c tio n is s te n te d o p e n d u r in g in s p ir a tio n b y th e n e g a tiv e e x tr a lu m in a l
in tr a th o r a c ic p r e s s u r e . D u r in g e x p ir a tio n in a s p o n ta n e o u s ly b r e a th in g p a tie n t, a n e x tr a th o r a c ic
o b s tr u c tio n is s te n te d o p e n b y s u p r a - a t m o s p h e r ic in tr a lu m in a l p r e s s u r e . In c o n tr a s t, a n
in tr a th o r a c ic o b s tr u c tio n is e x a c e r b a te d d u r in g e x p ir a tio n , s in c e th e e x tr a lu m in a l in tr a th o r a c ic
p r e s s u r e e x c e e d s th e in tr a lu m in a l p r e s s u r e . C h o ic e A r e p r e s e n ts a n o r m a l f l o w - v o l u m e lo o p .
C h o ic e C r e p r e s e n ts a f ix e d o b s tr u c tio n , th a t is , o n e p r e s e n t d u r in g b o th in s p ir a tio n a n d
e x p ir a tio n . C h o ic e D r e p r e s e n ts a d y n a m ic in tr a th o r a c ic o b s tr u c tio n , w h ic h w o u ld b e e x p e c te d
in a p a tie n t w ith a s th m a o r c h r o n ic o b s tr u c tiv e p u lm o n a r y d is e a s e .
Anesthesia Machine
Paul Sikka

1. Pipeline gases are supplied at pressures of about______psi:


A. 25
B. 40
C. 50
D. 75

2. Which of the following prevents delivery of hypoxic gas mixture once the oxygen pressure
falls below 25 psi?
A. Diameter index safety system
B. Pin index safety system
C. Inspiratory check valve
D. Fail-safe valve

3. The oxygen-flush valve provides which of the following oxygen flows (L/min) to the common
gas outlet?
A. 10
B. 25
C. 50
D. 90

4. Gas flowmeters
A. Are gas-specific
B. Have a gas flow rate which depends on viscosity at high turbulent flows
C. Have a gas flow rate which depends on density at low laminar flows
D. Are cylindrical in shape

5. Which of the following flowmeters is situated nearest to the gas outlet?


A. Nitrous oxide
B. Oxygen
C. Air
D. None of the above
6. Modern vaporizers are
A. Agent-specific
B. Temperature-compensated
C. Pressure-compensated
D. Both A and B

7. The Tec 6 desflurane vaporizer


A. Is electrically heated to 39°C
B. Is pressurized to 3 atm
C. Is pressure-compensated
D. All of the above

8. Variable bypass vaporizers should be located


A. Between the common gas outlet (upstream) and the flowmeters (downstream)
B. Between the flowmeters (upstream) and the common gas outlet (downstream)
C. Between the gas pipeline and the flowmeters
D. Inside the circle system

9. A standing or ascending bellow is preferred for anesthesia ventilators, as disconnection is


indicated by
A. Collapse
B. Filling by gravity
C. Disconnection alarm
D. Stoppage of flowmeter gas

10. The National Institute for Occupational Safety and Health (NIOSH) recommends limiting
operating-room concentration of nitrous oxide t o ______ppm:
A. 10
B. 25
C. 50
D. 100

11. The National Institute for Occupational Safety and Health (NIOSH) recommends limiting
operating-room concentration of volatile inhalational agents t o ______ppm:
A. 0.2
B. 0.5
C. 1
D. 2

12. Capacity of an oxygen “E” cylinder is approximately______L:


A. 500
B. 600
C. 650
D. 750

13. If pressure in a full nitrous oxide “E” cylinder is 745 psi at 20°C, the pressure in a half-full
cylinder will be about______psi:
A. 186
B. 248
C. 372
D. 745

14. Which of the following system prevents the wrong gas cylinder being attached to the anesthesia
machine?
A. Diameter index safety system
B. Pin index safety system
C. Hanger yoke assembly system
D. Gauge-safety system

15. A line-isolation monitor


A. Warns that an electrical shock is imminent
B. Warns of a fault between the power line and the ground
C. Warns of the presence of two faults
D. Trips the ground leakage circuit breaker

16. The highest content of soda lime is


A. Calcium hydroxide
B. Potassium hydroxide
C. Sodium hydroxide
D. Silica

17. End products of the reaction in a soda lime CO2 canister are
A. Carbonates, water, heat
B. Carbonates, heat, sodium hydroxide
C. Sodium hydroxide, water, heat
D. Carbonates, sodium hydroxide, water, heat
1. CO2 + H2O - H2CO3
2. H2CO3 + 2 NaOH (or KOH) - Na2CO3 (or K2CO3) + 2 H2O + Energy
3. Na2CO3 (or K2CO3) + Ca(OH)2 - CaCO3 + 2 NaOH (or KOH)
18. If you notice that the CO2 absorbent is exhausted during the surgical procedure, which of the
following minimal fresh gas flows (L/min) will make the CO2 absorbent unnecessary?
A. 3
B. 5
C. 7
D. 10

19. Compared to the Mapleson A system, the circle system


A. Is less bulky
B. Has a decreased risk of disconnection
C. Has decreased resistance to patient breathing
D. Better conserves humidity

20. Incorrect statement regarding the mechanisms of an Ambu bag is


A. It contains a nonrebreathing valve, same as the circle system
B. It is capable of delivery of nearly a 100% O2 concentration
C. It allows for positive-pressure ventilation
D. Patient valve has low resistance to both inspiration and expiration

21. You are preparing to set up for anesthesia in an off-floor location in the interventional
radiology suite. The radiography equipment is consuming the limited space that is available in
the suite, and therefore, the decision is made to double the extension tube length from the
ventilator to the patient table. What is the impact on the dead-space ventilation that would have
occurred secondary to doubling the extension tubing length?
A. It would double as well
B. It has been decreased to half the original volume
C. It would have increased by 4-fold
D. It would have not changed

22. Malfunction of which of the following valves within a circle system may cause rebreathing of
carbon dioxide and could potentially result in hypercapnia?
A. Inspiratory valve
B. Expiratory valve
C. Both A and B
D. None of the above

23. Since fresh gas flow equal to minute ventilation is sufficient to prevent rebreathing, which of
the following Mapleson circuit breathing/ventilation systems is the most efficient for
spontaneous ventilation of the patient?
A. Mapleson A
B. Mapleson B
C. Mapleson C
D. Mapleson D

24. Different semi closed anesthetic ventilation/breathing systems (classically referred to as


Mapleson systems and designated A to F) are pictured below. While setting up for anesthesia
delivery in an “off-floor” location and planning for controlled ventilation of an asthmatic
patient, which of the Mapleson systems provides for the best efficacy?

A. D>B>C>A
B. A>B>C>D
C. D>C>B>A
D. C>A>D>B

25. Degradation of sevoflurane by soda lime results in the production of


A. Compound A
B. Compound B
C. Compound C
D. Compound D

26. In a CO2-absorbent canister, the greatest amount of carbon monoxide is produced by which of
the following volatile agents?
A. Sevoflurane
B. Halothane
C. Isoflurane
D. Desflurane
CHAPTER 3 ANSWERS

1. C. P ip e lin e g a s e s a r e s u p p lie d a t p r e s s u r e s b e tw e e n 4 5 a n d 5 5 p s i. T h is is in c o n tr a s t to
c y lin d e r g a s p r e s s u r e s , w h ic h a r e m u c h h ig h e r , a n d a r e r e d u c e d b y p r e s s u r e r e g u l a t o r s to le s s
th a n 50 p s i.

2. D. T h e f a il- s a f e v a lv e a u to m a tic a lly c lo s e s n itr o u s o x id e (a n d o th e r g a s e s ) to p r e v e n t d e liv e r y


o f h y p o x ic g a s m ix tu r e to th e p a tie n t. T h e f a il- s a f e v a lv e is d e s ig n e d to b e a c tiv a te d w h e n
o x y g e n p r e s s u r e f a lls b e lo w 2 5 p s i.

3. C. T h e o x y g e n - f lu s h v a lv e p r o v id e s g a s f lo w a t p ip e lin e p r e s s u r e s o f a b o u t 4 5 to 5 5 p s i a t 3 5
to 7 5 L /m in . T h e h ig h f lo w o f o x y g e n is p r o v id e d d ir e c tly to th e c o m m o n g a s o u tle t, b y p a s s in g
th e f lo w m e te r s a n d v a p o r iz e r s . O n e s h o u ld b e c a r e f u l w h e n u s in g th e o x y g e n - f lu s h v a lv e , as
h ig h g a s f lo w s a t h ig h p r e s s u r e s c a n c a u s e lu n g b a r o tr a u m a in th e p a tie n t.

4. A. G a s f lo w m e te r s a r e c a lib r a te d f o r a p a r tic u la r g a s . G a s f lo w r a te d e p e n d s o n its v is c o s ity a t


l o w la m in a r flo w s , a n d its d e n s ity a t h ig h tu r b u le n t flo w s . F lo w m e te r s a r e ta p e r e d in s h a p e , w ith
th e d ia m e te r th e s m a lle s t n e a r th e b o tto m o f th e tu b e .

5. B. T h e o x y g e n f lo w m e te r is s itu a te d n e a r e s t to th e g a s o u tle t. T h is is b e c a u s e , if a le a k
d e v e lo p s in th e f lo w m e te r tu b e s , a h y p o x ic g a s m ix tu r e c a n b e d e liv e r e d to th e p a tie n t. T o
m in im iz e th is , th e o x y g e n f lo w m e te r is p o s itio n e d d o w n s tr e a m a n d n e a r e s t to th e g a s o u tle t.

6. D. M o d e r n v a p o r i z e r s a r e a g e n t- s p e c if ic a n d te m p e r a tu r e - c o m p e n s a te d . A ls o , s p e c if ic f il le r s
a r e a v a ila b le f o r e a c h v o la tile a g e n t, w h ic h p r e v e n t f il lin g o n th e w r o n g a g e n t. A c o n s ta n t
c o n c e n tr a tio n o f a g e n t is d e liv e r e d , u n a f f e c te d b y te m p e r a tu r e o r f lo w ra te s . T e m p e r a tu r e
c o m p e n s a tio n is a c h ie v e d b y a m e ta llic s tr ip c o m p o s e d o f tw o d if f e r e n t m e ta ls , w h ic h
e x p a n d s /c o n tr a c ts to d e liv e r a c o n s ta n t c o n c e n tr a tio n o f v a p o r .

7. A. T h e T e c 6 d e s f lu r a n e v a p o r iz e r is e le c tr i c a lly h e a te d to 3 9 ° C a n d p r e s s u r i z e d to 2 a tm . T h is
is d o n e b e c a u s e d e s f lu r a n e b o ils a t r o o m te m p e r a tu r e a t s e a le v e l (1 a tm ). T h e h e a tin g a n d
p r e s s u r i z a tio n o p tim iz e s th e d e liv e r y o f d e s f lu r a n e .

8. B. V a p o r iz e r s a r e lo c a te d b e tw e e n th e f lo w m e te r s (u p s tr e a m ) a n d th e c o m m o n g a s o u tle t
( d o w n s tr e a m ) . In o th e r w o r d s , v a p o r i z e r s a r e lo c a te d o u ts id e th e c ir c le s y s te m . T h is d e c r e a s e s
th e l i k e lih o o d o f d e liv e r y o f h ig h v a p o r c o n c e n tr a tio n s w h e n u s in g th e o x y g e n - f lu s h v a lv e .

9. A. A n a s c e n d in g b e llo w c o lla p s e s w h e n d is c o n n e c tio n o c c u r s . A d e s c e n d in g b e llo w , h o w e v e r,


c o n tin u e s to f ill b y g r a v ity w h e n d is c o n n e c tio n o c c u r s . T h e r e f o r e , a s c e n d in g b e llo w s a r e
p r e f e r r e d f o r a n e s th e s ia v e n tila to r s .
10. B. N IO S H r e c o m m e n d s lim itin g o p e r a t i n g - r o o m c o n c e n tr a tio n o f n itr o u s o x id e to 2 5 p p m .
M in im iz in g o p e r a t i n g - r o o m p o llu tio n is im p o r ta n t to p r e v e n t h e a lth - r e la te d e ffe c ts in h e a lth ­
c a r e p r o v id e r s . W a s te - s c a v e n g in g s y s te m s a r e u tiliz e d to d e c r e a s e o p e r a t i n g - r o o m p o llu tio n .

11. D. N IO S H r e c o m m e n d s lim itin g o p e r a t i n g - r o o m c o n c e n tr a tio n o f v o la tile a g e n ts to 2 p p m .


M in im iz in g o p e r a t i n g - r o o m p o llu tio n is im p o r ta n t to p r e v e n t h e a lth - r e la te d e ffe c ts in h e a lt h ­
c a r e p r o v id e r s . W a s te - s c a v e n g in g s y s te m s a r e u tiliz e d to d e c r e a s e o p e r a t i n g - r o o m p o llu tio n .

12. C. T h e c a p a c ity o f a n “ E ” c y lin d e r o f o x y g e n is a b o u t 6 2 5 to 7 0 0 L . T h e p r e s s u r e in a fu ll


c y lin d e r is a b o u t 1 ,8 0 0 p s i a t 2 0 °C . C y lin d e r s a r e c o lo r - c o d e d , w ith o x y g e n b e in g g r e e n ,
n itr o u s o x id e b e in g b lu e , a n d a ir b e in g y e llo w .

13. D. P r e s s u r e in a h a lf - f u ll “ E ” c y lin d e r o f n itr o u s o x id e w ill s till b e 7 4 5 p s i. N itr o u s o x id e is


p r e s e n t in th e c y lin d e r a s a liq u id , a n d th e r e f o r e , th e v o lu m e r e m a in in g in th e c y lin d e r d o e s n o t
r e f le c t th e p r e s s u r e in th e c y lin d e r. C a p a c ity o f a n “ E c y lin d e r ” o f n itr o u s is a b o u t 1 5 9 0 L . It is
n o t u n til th r e e - f o u r t h o f th e g a s is c o n s u m e d ( a b o u t 4 0 0 L r e m a in in g ) th a t th e p r e s s u r e in th e
c y lin d e r b e g in s to fa ll. T h e r e f o r e , th e r e lia b le w a y to d e te r m in e th e r e m a in in g n itr o u s o x id e in
th e c y lin d e r is to w e ig h th e c y lin d e r. T h e e m p ty w e ig h t o f th e c y lin d e r is s ta m p e d o n th e
c y lin d e r.

14. B. C y lin d e r m a n u f a c tu r e s h a v e a d o p te d th e p in in d e x s a f e ty s y s te m , w h ic h p r e v e n ts a tta c h m e n t


o f w r o n g g a s c y lin d e r to th e a n e s th e s ia m a c h in e . T h e d ia m e te r in d e x s a f e ty s y s te m p re v e n ts
a tta c h m e n t o f th e w r o n g g a s h o s e f r o m th e w a ll s u p p ly . H a n g e r y o k e a s s e m b ly is th e m e th o d o f
a tta c h m e n t o f g a s c y lin d e r s to th e a n e s th e s ia m a c h in e .

15. B. A lin e - is o la tio n m o n ito r , w h e n a la r m in g , in d ic a te s th a t a s in g le f a u lt h a s o c c u r r e d b e tw e e n


th e p o w e r lin e a n d th e g r o u n d . A s s o o n a s th e a la r m is tr i g g e r e d , th e e q u ip m e n t s h o u ld b e
c h e c k e d , e s p e c ia lly th e la s t e q u ip m e n t th a t w a s p lu g g e d in . A s in g le f a u lt d o e s n o t c a u s e a n
e le c tr ic a l s h o c k , a s tw o fa u lts a r e r e q u ir e d to p r o d u c e a s h o c k .

16. A. T h e h ig h e s t c o n te n t o f s o d a lim e is c a lc iu m h y d r o x id e (7 5 % ). O th e r c o n s titu e n ts in c lu d e


s o d iu m (3 % ) a n d p o ta s s iu m h y d r o x id e (1 % ), w a te r (2 0 % ), a n d s ilic a , w h ic h is a d d e d to p r o d u c e
h a r d n e s s . A n in d ic a to r d y e , s u c h a s e th y l v io le t, is a d d e d to in d ic a te th e d e g r e e o f e x h a u s tio n .

17. D. E n d p r o d u c ts o f th e r e a c tio n o c c u r r i n g in a s o d a lim e C O 2 c a n is te r a r e c a r b o n a te s , s o d iu m


h y d r o x id e ( r e g e n e r a ti o n ) , w a te r, a n d h e a t. F o llo w in g a r e th e r e a c tio n s :

18. B. A d v a n ta g e s o f a c ir c le s y s te m in c lu d e th e u s e o f lo w f r e s h g a s f lo w r a te s b e c a u s e o f th e
p r e s e n c e o f a C O 2- a b s o r b e n t c a n is te r. H o w e v e r, if th e C O 2 a b s o r b e n t is e x h a u s te d d u r in g a
s u r g ic a l p r o c e d u r e , th e f r e s h g a s f lo w r a te h a s to b e in c r e a s e d . A m in im u m f r e s h g a s f lo w r a te
o f 5 L /m in w ill m a k e th e u s e o f th e a b s o r b e n t u n n e c e s s a ry . N e w e r a n e s th e s ia m a c h in e s a llo w
c h a n g in g th e C O 2- a b s o r b e n t c a n is te r d u r in g th e s u r g ic a l p r o c e d u r e , if n e c e s s a ry .
19. D. A d v a n ta g e s o f th e c ir c le s y s te m in c lu d e e c o n o m y ( lo w f r e s h g a s f lo w ra te s , d e c r e a s e d u s e
o f v o la tile a g e n ts ), c o n s e r v a tio n o f h e a t a n d h u m id ity , a n d d e c r e a s e d o p e r a t i n g - r o o m p o llu tio n .
D is a d v a n ta g e s o f c ir c le s y s te m in c lu d e g r e a te r s iz e , d e c r e a s e d p o r ta b ility , in c r e a s e d r i s k o f
d is c o n n e c tio n , a n d in c r e a s e d r e s is ta n c e to p a tie n t b r e a th in g .

20. A. W h ile r e s u s c ita tio n d e v ic e s s u c h a s A m b u b a g s o r b a g - m a s k u n its h a v e n o n r e b r e a th in g


v a lv e s , n e ith e r th e M a p le s o n ( o n ly h a s a d ju s ta b le p r e s s u r e - li m itin g v a lv e ) n o r th e c ir c le s y s te m
( o n ly h a s u n id ir e c tio n a l v a lv e s a n d d o e s a llo w r e b r e a t h in g ) h a s th is c o m p o n e n t. A m b u
r e s u s c ita tio n b a g s d o a llo w f o r p o s itiv e - p r e s s u r e v e n tila tio n a s th e in ta k e v a lv e c lo s e s d u r in g
b a g c o m p r e s s io n . T h e p a tie n t v a lv e h a s l o w r e s is ta n c e , b u t c a n b e c o m e o b s tr u c te d b y e x h a le d
m o is tu r e . A m b u b a g s h a v e a r e s e r v o i r s y s te m to p r e v e n t r o o m a ir e n tr a p m e n t a n d a r e a b le to
d e liv e r n e a r ly 1 0 0 % o x y g e n .

21. D. O n e a d v a n ta g e o f th e c ir c le s y s te m v e n tila tio n w h e n c o m p a r e d to th e M a p le s o n s y s te m is


th e p r e s e n c e o f u n id ir e c tio n a l v a lv e s ( in s p ir a to r y a n d e x p ir a to r y v a lv e s ) . W ith th e u s e o f s u c h
v a lv e s , th e v o lu m e o f d e a d - s p a c e v e n tila tio n is lim ite d only to th a t v o lu m e d is ta l to Y -p ie c e
( in c lu d in g th e e n d o tr a c h e a l tu b e ), w h e r e i n s p i r a t o r y a n d e x p ir a to r y g a s e s m i x a n d c o n v e r g e ,
r e g a r d l e s s o f th e le n g th o f tu b in g p r o x im a l to th e Y -p ie c e (to th e a n e s th e s ia m a c h in e ).

22. C. M a lf u n c tio n in e ith e r o f th e u n id ir e c tio n a l v a lv e s w ith in a c ir c le s y s te m c o u ld r e s u lt in th e


a c c u m u la tio n a n d e v e n tu a l C O 2 r e b r e a th in g th a t m a y r e s u lt in h y p e r c a p n ia .

23. A. D u r in g s p o n ta n e o u s v e n tila tio n /b r e a th in g o f th e p a tie n t, th e M a p le s o n c ir c u it p r o v id i n g f o r


th e m o s t e f f ic a c y r a n g e s f r o m A > D > C > B (in th e o r d e r o f d e c r e a s in g e f f ic ie n c y ) .

24. A. T h e e f f ic ie n c y o f M a p le s o n s y s te m s d r o p s f r o m D > B > C > A f o r c o n tr o lle d v e n tila tio n .


T h e M a p le s o n D c ir c u it is m o s t e f f ic ie n t d u r in g c o n tr o lle d v e n tila tio n , a s its f r e s h g a s f lo w
d r iv e s e x p ir e d a ir a w a y f r o m th e p a tie n t a n d to w a r d th e e x p ir a to r y /e x h a u s t v a lv e .

25. A. S e v o f lu r a n e is d e g r a d e d b y s o d a lim e , r e s u ltin g in th e p r o d u c tio n o f a p o te n tia lly


n e p h r o to x ic c o m p o u n d A . C o m p o u n d A p r o d u c tio n is in c r e a s e d b y u s in g l o w f r e s h g a s f lo w
ra te s , u s in g h ig h c o n c e n tr a tio n s o f s e v o f lu r a n e , a n d f o r lo n g h o u r s (> 6 h o u r s ) .

26. D. D e s f lu r a n e p r o d u c e s th e h ig h e s t a m o u n t o f c a r b o n m o n o x id e in th e C O 2- a b s o r b e n t
c a n is te r, w h ic h c a n in c r e a s e c a r b o x y h e m o g lo b i n b l o o d c o n c e n tr a tio n . P r o d u c ti o n o f c a r b o n
m o n o x id e is in c r e a s e d b y u s in g l o w f r e s h g a s f lo w r a te s , h ig h c o n c e n tr a tio n s o f v o la tile a g e n t,
a n d a d r y a b s o r b e n t.
Patient Monitoring
Darren Hyatt, Ala Nozari, and Edward Bittner

1. To help encourage universal quality and safety practices, the ASA has adopted and mandates the
use of all the following monitors during general anesthesia, except
A. An oxygen analyzer
B. Capnography
C. Continuous visual display of an ECG
D. A peripheral nerve stimulator

2. Current ASA standards require that during anesthesia, systemic blood pressure and heart rate be
evaluated at least every
A. 3 minutes
B. 5 minutes
C. 7 minutes
D. 10 minutes

3. Lead II of an ECG is represented by placing the


A. Positive electrode on the right arm and the negative electrode on the left leg
B. Negative electrode on the right arm and the positive electrode on the left leg
C. Positive electrode on the right arm and the negative electrode on the left arm
D. Negative electrode on the right arm and the positive electrode on the left arm

4. During the course of a complicated cardiac case, the surgeon informs you that he is worried
about damage to the right coronary artery in a patient with a right-dominant coronary system.
During reperfusion, you are looking for signs of ischemia, and are most interested in leads
A. V1-V3
B. V4—V6
C. II, III, and AvF
D. I and AvL

5. Use of lead V5 alone on ECG results in the detection o f _____(%) of ischemic episodes:
A. 35
B. 55
C. 75
D. 95

6. You are taking over a case from another anesthesia provider with a patient in the beach chair
position and a history of moderate carotid artery disease. You are told during pass-off that the
patient’s blood pressures have consistently been 90/50 mm Hg. You notice the blood pressure
cuff on the left arm is one or two sizes small and barely stays on the patient. A blood pressure
cuff that is too small will
A. Incorrectly underestimate the true blood pressure
B. Incorrectly overestimate the true blood pressure
C. Randomly both over- and underestimate the true blood pressure
D. Not give an incorrect blood pressure, but will be uncomfortable in an awake patient

7. When performing the oscillometric method to measure blood pressure, for example, when you
do not have a stethoscope or automated blood pressure cuff, it is important to remember that
you will not be able to measure the
A. Systolic blood pressure
B. Diastolic blood pressure
C. Mean arterial pressure
D. Diastolic or mean arterial blood pressure

8. The diastolic blood pressure recorded with an automated blood pressure cuff using the
oscillometric method will be
A. Approximately 10 mm Hg higher when compared to direct arterial measurement
B. Approximately 10 mm Hg lower when compared to direct arterial measurement
C. Equal to direct arterial measurement
D. Random and unreliable

9. When measuring blood pressure manually and listening for Korotkoff sounds, the diastolic
blood pressure is measured at the onset of
A. Phase 1
B. Phase 2
C. Phase 3
D. Phase 5

10. You are preparing for an emergent mitral valve repair that will need to be done on
cardiopulmonary bypass (CPB). While on CPB
A. A pulse oximeter can be used to monitor oxygen saturation
B. A noninvasive blood pressure cuff can be used to monitor perfusion pressures
C. An arterial line can be used to measure perfusion pressures
D. None of the above
11. The incidence of distal ischemia resulting from arterial cannulation is less than
A. 10%
B. 1%
C. 0.1%
D. 0.01%

12. When considering the advantages and disadvantages of different sites for arterial cannulation
such as radial, ulnar, femoral, brachial, and dorsalis pedis, the
A. Radial artery provides the principal source of blood to the hand
B. Cannulation of ulnar artery is commonly associated with damage to the median nerve
C. Dorsalis pedis artery is commonly used during emergencies and low-flow states
D. Cannulation of the femoral artery risks local and retroperitoneal hematoma

13. Systolic blood pressures are generally higher and diastolic blood pressures are generally
lower in which of the following conditions?
A. The further you are from the heart when using a direct arterial measurement
B. The closer you are to the heart when using a direct arterial measurement
C. When using an automated noninvasive blood pressure cuff compared to a direct arterial
measurement
D. When recording from an over dampened arterial tracing

14. While taking care of a patient, you notice that the arterial monitor transducer has slipped off its
stand and is hanging approximately 30 cm lower than where it was originally leveled. This
would correspond to a blood pressure reading that is
A. 30 mm Hg lower than the actual pressure
B. 30 mm Hg higher than the actual pressure
C. 22 mm Hg lower than the actual pressure
D. 22 mm Hg higher than the actual pressure

15. An important consideration in using the subclavian approach for central venous access includes
the
A. Ease of compressibility if a hematoma or laceration develops
B. Lower risk of pneumothorax when compared to internal jugular approach
C. Ability of the vessel to remain patent in the setting of hypovolemia
D. Increased risk of damaging the brachial plexus when compared to internal jugular
approach

16. When interpreting a CVP waveform, the end of systole best coincides with the
A. A wave
B. C wave
C. V wave
D. X decent

17. When interpreting a CVP waveform, the beginning of systole is best represented by the
A. A wave
B. C wave
C. V wave
D. X decent

18. After placing a central line in an unstable patient in the ICU, you notice the initial CVP tracing
shows very prominent C-V waves. If an echocardiogram was then obtained, you might expect
to find
A. Cardiac tamponade
B. Significant tricuspid regurgitation
C. Atrial fibrillation
D. AV dissociation

19. You receive a patient from the emergency department with multiple stab wounds to the upper
abdomen. The patient is unstable, and needs to emergently come to the operating room with
minimal to no time for fluid resuscitation. After placing a central line, you notice loss of the Y
descent on the CVP tracing, as well as universally elevated filling pressures. If you were to then
do an echocardiogram, you might expect to find which of the following?
A. Cardiac tamponade
B. Significant tricuspid regurgitation
C. Descending thoracic aortic dissection
D. AV dissociation

20. The risk of complication from pulmonary artery catheter placement is less than
A. 0.05%
B. 0.5%
C. 5%
D. 15%

21. Insertion of a pulmonary artery catheter can be beneficial in the management of all of the
following cases, except
A. Helping to determine cardiogenic versus noncardiogenic pulmonary edema
B. Following cardiac output in an unstable patient with acute-onset tricuspid regurgitation
C. Following the response to therapy in a patient with severe pulmonary hypertension
D. Following response to therapy in an unstable septic patient using mixed venous oxygen
tension

22. During placement of a pulmonary artery catheter, you are watching the pressure tracing, as
s h o w n . A t th e p o in t in d ic a te d b y th e a r r o w , th e c a th e te r tip is lo c a te d in th e

A. Right atrium
B. Right ventricle
C. Pulmonary artery
D. Wedge position

23. The tip of a pulmonary artery catheter typically enters the pulmonary artery at approximately
A. 15 to 25 cm
B. 25 to 35 cm
C. 35 to 45 cm
D. 45 to 55 cm

24. Typical mixed venous oxygen tension in a healthy adult is


A. 25 mm Hg
B. 40 mm Hg
C. 55 mm Hg
D. 75 mm Hg

25. A pulmonary artery catheter is placed to help guide management of hypotension. Cardiac output
is found to be markedly decreased with low central venous, pulmonary artery, and pulmonary
artery occlusion pressures. Systemic vascular resistance is moderately elevated. Of the options
listed below, the most beneficial intervention at this time would be to
A. Administer volume
B. Begin diuresis
C. Start an infusion of milrinone
D. Start an infusion of epinephrine

26. A pulmonary artery catheter is placed to help guide management of an obese patient with a
known history of poorly controlled obstructive sleep apnea who is admitted with refractory
hypotension. Cardiac output and pulmonary artery occlusion pressures are markedly decreased,
while central venous and pulmonary artery pressures are markedly increased. Of the options
listed below, the most beneficial intervention at this time would be to
A. A d m in is te r v o lu m e
B. B e g in d iu r e s is
C. S ta rt a n in f u s io n o f m ilr in o n e
D. S ta rt a n in f u s io n o f e p in e p h r in e

27. Normal systemic vascular resistance ranges between______ (dynes)(s)/cm5:


A. 50 and 150
B. 300 and 600
C. 900 and 1500
D. 1800 and 2100

28. Normal pulmonary vascular resistance ranges between______(dynes)(s)/cm5:


A. 50 and 150
B. 300 and 600
C. 900 and 1500
D. 1800 and 2100

29. The cardiac index in a healthy adult ranges between______L/min/m2:


A. 0.8 and 1.2
B. 1.4 and 2.0
C. 2.2 and 4.2
D. 4.4 and 6.0

30. Serious complications with transesophageal echocardiography (TEE), such as oral or


pharyngeal injury or esophageal rupture, have an incidence as high as
A. 0.01%
B. 0.1%
C. 1%
D. 10%

31. When evaluating regurgitant lesions with transesophageal echocardiography, the Nyquist limit
should be set between______cm/s:
A. 30 and 40
B. 40 and 50
C. 50 and 60
D. 60 and 70

32. When evaluating flow at a specific point during echocardiography, you would use
A. Continuous-wave Doppler
B. Pulse-wave Doppler
C. Color Doppler
D. P u ls e -w a v e o r c o n tin u o u s - w a v e D o p p le r

33. Pulse oximetry illuminates tissue samples with two wavelengths of light in order to calculate
oxygen saturation. These wavelengths a re ______nm:
A. 540 and 780
B. 660 and 940
C. 720 and 960
D. 480 and 720

34. The accuracy of pulse oximetry can be significantly reduced by all of the following, except
A. Intravenous bolus of methylene blue
B. Intravenous bolus of heparin
C. Severe acidosis
D. Low blood flow

35. A patient with carboxyhemoglobin will have a pulse oximetry reading that
A. Converges around a saturation of 85%
B. Converges around a saturation of 65%
C. Converges around a saturation of 45%
D. Varies widely

36. A patient with methemoglobinemia will have a pulse oximetry reading that
A. Converges around a saturation of 85%
B. Converges around a saturation of 65%
C. Converges around a saturation of 45%
D. Varies widely

37. For the removal of a complex spinal cord tumor, the surgeon expresses concern of damage to
the anterior spinal artery. The monitoring that would be helpful to determine viability of the
anterior spinal cord intraoperatively would include
A. Electroencephalography
B. Motor-evoked potentials
C. Somatosensory-evoked potentials
D. Bispectral index or Sedline monitoring

38. A sudden drop in somatosensory-evoked potentials (SSEPs) would cause you to be worried
about
A. Damage to the anterior spinal artery
B. Damage to the posterior spinal arteries
C. An insufficient depth of anesthesia
D. The inadvertent administration of a neuromuscular blocking agent
39. During cervical spine surgery for the resection of an intradural mass, the patient begins to
cough. The concentration of isoflurane is subsequently increased. With respect to
somatosensory-evoked potential (SSEP) monitoring, you would expect
A. Amplitude and latency to decrease
B. Amplitude and latency to increase
C. Amplitude to decrease and latency to increase
D. Amplitude to increase and latency to decrease

40. While monitoring somatosensory-evoked potentials, an increase in amplitude is noted. Of the


options listed below, the most likely medication to have caused this increase in amplitude would
be
A. Etomidate
B. Propofol
C. Midazolam
D. Sevoflurane

41. If somatosensory-evoked potentials change significantly, the anesthesia provider should


consider
A. Increasing blood pressure
B. Hyperventilating the patient
C. Cooling the patient
D. Hemodilution

42. In the capnogram below, the segment that correlates with the exhalation of anatomic dead space

F igu re 4-2.

A. A to B
B. A to C
C. C to D
D. D to E

43. In the capnogram (Fig. 4-2), the segment correlating with inspiration is represented by points
A. A to B
B. A to C
C. C to D
D. D to E

44. Capnography can help detect all of the following, except


A. Endobronchial intubation
B. Esophageal intubation
C. Bronchospasm
D. Pulmonary embolism

45. The capnograph depicted in Figure 4-3 is most likely a result of

A. Pulmonary embolism
B. Bronchospasm or airway obstruction
C. Esophageal intubation
D. Elimination of neuromuscular blockers

46. Approximately 30 minutes after the induction of general anesthesia in a healthy adult patient,
you notice that core body temperature has dropped by a full degree Celsius. This is most likely
due to
A. Conduction
B. Convection
C. Redistribution
D. Radiation

47. According to the American Society of Anesthesiologists, temperature monitoring is


A. Always required
B. Never required, but recommended
C. Required for all general anesthetics, however not required for sedation
D. Up to the discretion of the anesthesia provider
48. D e tr im e n ta l e ffe c ts o f h y p o th e r m ia in c lu d e a ll o f th e f o ll o w i n g , e x c e p t

A. I n c r e a s in g c e r e b r a l o x y g e n c o n s u m p tio n
B. I n c r e a s in g s u r g ic a l s ite in f e c tio n s
C. I m p a ir m e n t o f p la te le t f u n c tio n
D. I n c r e a s in g th e d u r a tio n o f a c tio n o f m u s c le r e la x a n ts

49. D u r in g a c o m p le x m itr a l v a lv e r e p la c e m e n t, it is d e te r m in e d th a t th e p a tie n t w ill b e n e f it f r o m


b r i e f p r o te c tiv e h y p o th e r m ia . O f th e o p tio n s lis te d b e lo w , c o r e te m p e r a tu r e is b e s t m e a s u r e d v ia
th e

A. T y m p a n ic m e m b r a n e
B. B la d d e r
C. N a so p h a rn y x
D. R e c tu m

50. W h ile m o n ito r in g a p a tie n t f o r r e tu r n o f n e u r o m u s c u la r f u n c tio n a fte r u s in g r o c u r o n iu m , y o u


n o tic e th e p a tie n t h a s r e g a in e d f o u r tw itc h e s u s in g tr a in o f f o u r s tim u la tio n s . W ith f o u r tw itc h e s
o n tr a in o f f o u r s tim u la tio n s , th e p a tie n t m a y s till h a v e b lo c k a g e o f a c e ty lc h o lin e r e c e p to r s o f
u p to

A. 25%
B. 50%
C. 75%
D. 90%
CHAPTER 4 ANSWERS

1. D. A S A s ta n d a r d s m a n d a te th e u s e o f p u ls e o x im e tr y , c a p n o g r a p h y , a n o x y g e n a n a ly z e r in th e
b r e a th in g s y s te m , d is c o n n e c t a la r m s , a v is u a l d is p la y o f a n E C G , s y s te m ic b l o o d p r e s s u r e a n d
h e a r t r a te m o n ito r in g , a n d te m p e r a tu r e m o n ito r in g (w h e n c lin ic a ll y in d ic a te d ) f o r a ll c a s e s . T h e
u s e o f a p e r ip h e r a l n e r v e s tim u la to r is n o t a m a n d a te d m o n ito r .

2. B. D u r in g th e d e liv e r y o f a n e s th e s ia , th e c u r r e n t s ta n d a r d o f c a r e is to m e a s u r e s y s te m ic b lo o d
p r e s s u r e a n d h e a r t r a te e v e r y 5 m in u te s a t a m in im u m . T h e c lin ic a l s c e n a r io a n d p h a s e o f th e
o p e r a tio n m a y m a n d a te m o r e f r e q u e n t m o n ito r in g , w h ic h is u p to th e j u d g m e n t o f th e a n e s th e s ia
p r o v id e r .

3. B. L e a d I c o r r e la te s w ith th e p la c e m e n t o f th e n e g a tiv e e le c tr o d e o n th e r i g h t a r m a n d th e
p o s itiv e e le c tr o d e o n th e le f t a r m . L e a d II c o r r e la te s w ith th e p la c e m e n t o f th e n e g a tiv e e le c tr o d e
o n th e r i g h t a r m a n d th e p o s itiv e e le c tr o d e o n th e le f t le g . L e a d III c o r r e la te s w ith th e p la c e m e n t
o f th e n e g a tiv e e le c tr o d e o n th e le f t a r m a n d th e p o s itiv e e le c tr o d e o n th e le f t le g .

4. C. T h e u n d e r s ta n d in g o f c o r o n a r y a n a to m y a n d r e g i o n s o f is c h e m ia o n a n E C G is
f u n d a m e n ta l. T h e r i g h t c o r o n a r y a r te r y p r o v id e s p e r f u s io n to th e i n f e r i o r o f th e h e a r t in
a p p r o x im a te ly 8 0 % o f p a tie n ts w h o a r e c o n s id e r e d to b e r ig h t - d o m in a n t (th e p o s te r io r
d e s c e n d in g a r te r y is s u p p lie d b y th e r i g h t c o r o n a r y a r te r y in a r ig h t - d o m in a n t s y s te m ). T h is
i n f e r i o r d is tr ib u tio n is r e p r e s e n te d b y le a d s II, III, a n d A vF. T h e a n te r io r w a ll is s u p p lie d b y th e
le f t a n te r io r d e s c e n d in g a rte ry , a n d is r e p r e s e n te d r o u g h l y b y le a d s V 1 - V 4 . T h e la te r a l w a ll o f
th e h e a r t is s u p p lie d p r i m a r i l y b y th e le f t c ir c u m f le x a rte ry , a n d is r e p r e s e n te d b y I, A v L , V 5 ,
and V 6.

5. C. T h e u s e o f th e V 5 le a d r e s u lts in th e d e te c tio n o f 7 5 % o f is c h e m ic e p is o d e s . T h is c a n b e
in c r e a s e d to 9 0 % w ith th e a d d itio n o f th e V 4 le a d , a n d u p to 9 6 % w ith th e a d d itio n o f le a d s II
and V 4.

6. B. A p r o p e r ly - s iz e d n o n in v a s iv e b l o o d p r e s s u r e c u f f s h o u ld e n c o m p a s s 4 0 % o f th e
c ir c u m f e r e n c e o f th e a r m . A c u f f th a t is to o s m a ll w ill r e s u lt in a r e a d in g th a t is in c o r r e c t l y
h ig h , w h e r e a s a c u f f th a t is to o l a r g e w ill r e s u lt in a lo w e r - th a n - a c c u r a te p r e s s u r e . T h is is
p a r tic u la r ly w o r r i s o m e in th is p a tie n t w h e n c o n s id e r in g h e r c e r e b r a l p e r f u s io n p r e s s u r e , s in c e
s h e a lr e a d y h a s a h is to r y o f c a r o tid a r te r y d is e a s e a n d is in th e b e a c h c h a ir p o s itio n .

7. B. W h e n u s in g th e o s c il lo m e t r ic m e th o d to m e a s u r e b l o o d p r e s s u r e , th e c u f f is in f la te d u n til
n o o s c illa tio n s o n th e s p h y g m o m a n o m e te r a r e s e e n . T h e c u f f is th e n s lo w ly d e f la te d u n til
o s c illa tio n s a r e s e e n , w h ic h r e p r e s e n ts th e s y s to lic b l o o d p r e s s u r e . A s th e c u f f c o n tin u e s to b e
d e fla te d , y o u n o te th e p o in t w h e r e m a x im a l o s c illa tio n s o c c u r . T h is p o in t o f m a x im a l
o s c il la ti o n r e p r e s e n ts th e m e a n a r te r ia l p r e s s u r e . It is n o t p o s s ib le to m e a s u r e a d ia s to lic b l o o d
p r e s s u r e w ith th e o s c il lo m e t r ic m e th o d .
8. A. T h e D IN A M A P (d e v ic e f o r in d ir e c t n o n in v a s iv e a u to m a tic m e a n a r te r ia l p r e s s u r e ) m e th o d
f o r m e a s u r in g b l o o d p r e s s u r e u s e s a n a u to m a te d c u f f th a t m e a s u r e s o s c il lo m e t r ic v a r ia tio n s
w ith r e d u c tio n in c u f f p r e s s u r e to c a lc u la te s y s to lic , m e a n , a n d d ia s to lic p r e s s u r e s . In g e n e r a l,
d ia s to lic m e a s u r e m e n ts w ith D IN A M A P a r e a b o u t 10 m m H g h ig h e r w ith a u to m a te d a s o p p o s e d
to d ir e c t a r te r ia l m e a s u r e m e n t, w h e r e a s s y s to lic a n d m e a n p r e s s u r e s te n d to c o r r e la te w e ll.

9. D. K o r o tk o f f s o u n d s a r e u s e d to in te r p r e t b l o o d p r e s s u r e w h e n u s in g a s te th o s c o p e a n d a
n o n in v a s iv e b l o o d p r e s s u r e c u ff, a n d is d e s c r ib e d in 5 p h a s e s o f s o u n d . P h a s e 1 h e r a ld s th e
o n s e t o f th e f i r s t s o u n d h e a r d a n d c o r r e la te s w ith th e s y s to lic b l o o d p r e s s u r e . P h a s e 5 o c c u r s a t
th e c u f f p r e s s u r e a t w h ic h th e s o u n d f i r s t d is a p p e a r s , a n d is th e p h a s e r e c o m m e n d e d b y th e
A m e r ic a n H e a r t A s s o c ia tio n to c o r r e s p o n d m o s t r e l i a b l y w ith th e d ia s to lic h e a r t s o u n d . In c a s e s
w h e r e P h a s e 5 d o e s n o t o c c u r (th e s o u n d n e v e r f u lly d is a p p e a r s ) , P h a s e 4 is th e n u s e d to
r e p r e s e n t th e d ia s to lic b l o o d p r e s s u r e , a n d is d e s c r ib e d a s a th u m p in g o r m u tin g o f th e s o u n d
j u s t b e f o r e d ia s to le . P h a s e s 2 a n d 3 h a v e n o c lin ic a l s ig n if ic a n c e .

10. C. B o th p u ls e o x im e tr y a n d n o n in v a s iv e b l o o d p r e s s u r e c u ffs r e q u i r e p u ls a tile b l o o d f lo w in


o r d e r to o b ta in m e a s u r e m e n ts . T h e s e m o n ito r s w ill n o t b e e f f e c tiv e d u r in g C P B w h e n b l o o d
f lo w is a r tif i c ia lly s u s ta in e d w ith a m o r e c o n tin u o u s flo w . T h is c a n a ls o b e th e c a s e w ith s o m e
p a tie n ts o n le f t v e n tr ic u la r a s s is t d e v ic e s , a n d v e n o u s to a r te r ia l e x tr a c o r p o r e a l m e m b r a n e
o x y g e n a tio n d e v ic e s , w h e r e p u ls a tile f lo w is m in im a l.

11. C. C o m p lic a tio n s f r o m a r te r ia l c a n n u la tio n in c lu d e d is ta l is c h e m ia (< 0 .1 % ), in f e c tio n , a n d


h e m o r r h a g e . C o m m o n s ite s f o r c a n n u la tio n in c lu d e r a d ia l, b r a c h ia l, a x illa r y , d o r s a lis p e d is ,
a n d f e m o r a l a r te r ie s . C o m m o n in d ic a tio n s f o r d ir e c t b l o o d p r e s s u r e m o n ito r in g in c lu d e
c a r d io p u lm o n a r y b y p a s s , w h e n w id e s w in g s in B P a r e e x p e c te d , w h e n r i g o r o u s c o n tr o l o f B P is
n e c e s s a ry , a n d w h e n th e r e is n e e d f o r m u ltip le a r te r ia l b l o o d g a s m e a s u r e m e n ts .

12. D. T h e u ln a r a r te r y is th e p r in c ip a l s o u r c e o f b l o o d f lo w to th e h a n d . H e n c e r a d ia l a r te r y
c a n n u la tio n is m u c h m o r e c o m m o n ly u s e d f o r in v a s iv e b l o o d p r e s s u r e m o n ito r in g . C a n n u la tio n
o f th e b r a c h ia l a r te r y r is k s d a m a g e to th e m e d ia n n e r v e . T h e f e m o r a l a r te r y is o f te n u s e d in
e m e r g e n c ie s , s in c e it is a l a r g e v e s s e l a n d c a n s till b e id e n tif ie d in lo w f lo w s ta te s . C a n n u la tio n
o f th e f e m o r a l a r te r y r is k s b o th lo c a l a n d r e t r o p e r ito n e a l h e m a to m a . D o r s a l is p e d is a r te r y
c a n n u la tio n , w h ile n o t id e a l s in c e it is fa r f r o m th e c e n tr a l c ir c u la tio n , c a n r e l i a b l y m e a s u r e
m e a n a r te r ia l p r e s s u r e .

13. A. S y s to lic b l o o d p r e s s u r e s a r e g e n e r a lly h ig h e r a n d d ia s to lic b l o o d p r e s s u r e s a r e g e n e r a lly


lo w e r th e f u r th e r y o u a r e f r o m th e h e a r t w h e n u s in g d ir e c t in v a s iv e a r te r ia l m e a s u r e m e n t. F o r
e x a m p le , w h e n c o m p a r in g a d o r s a lis p e d is a r te r ia l m e a s u r e m e n t to a f e m o r a l a r te r ia l
m e a s u r e m e n t, th e d o r s a lis p e d is w ill r e c o r d h ig h e r s y s to lic a n d lo w e r d ia s to lic p r e s s u r e s
c o m p a r e d to th e f e m o r a l lin e . H o w e v e r, th e m e a n a r te r ia l p r e s s u r e s w ill b e a p p r o x im a te ly th e
s a m e . A n o n in v a s iv e a u to m a te d b l o o d p r e s s u r e c u f f w ill te n d to c o r r e la te w ith s y s to lic a r te r ia l
b l o o d p r e s s u r e s , b u t th e d ia s to lic p r e s s u r e w ill b e a p p r o x im a te ly 10 m m H g lo w e r w h e n
m e a s u r e d v ia th e d ir e c t in v a s iv e a r te r ia l m o n ito r . A n o v e r d a m p e n e d a r te r ia l lin e tr a c in g w ill
te n d to r e d u c e s y s to lic p r e s s u r e s a n d in c r e a s e d ia s to lic p r e s s u r e s .
14. D. F o r e v e r y 3 0 c m in h e ig h t th a t a tr a n s d u c e r is m o v e d u p a n d d o w n , th e r e is a c o r r e s p o n d i n g
c h a n g e o f 22 m m H g in th e b l o o d p r e s s u r e r e a d in g (1 c m H 2O = 0 .7 4 m m H g ).

15. C. R is k s a n d b e n e f its o f d if f e r e n t c e n tr a l c a n n u la tio n s ite s a r e im p o r ta n t f o r a n a n e s th e s ia


p r o v id e r to u n d e rs ta n d . T h e in te r n a l ju g u l a r a p p r o a c h h a s g o o d la n d m a r k s , p r e d ic ta b le
a n a to m y , a n d th e c o n v e n ie n c e o f b e in g e a s ily a c c e s s ib le a t th e h e a d o f th e b e d . D is a d v a n ta g e s
in c lu d e r i s k o f c a r o tid a r te r y p u n c tu re , tr a u m a to th e b r a c h ia l p le x u s , a n d r i s k o f p n e u m o th o r a x
w ith lo w e r p la c e m e n ts . T h e le f t in te r n a l j u g u l a r v e in c a r r ie s th e a d d e d r i s k o f d a m a g e to th e
th o r a c ic d u c t, a n d c a n b e m o r e d if f ic u lt to p a s s a p u lm o n a r y a r te r y c a th e te r w h e n n e e d e d . T h e
e x te r n a l ju g u l a r v e in c a n a ls o b e c a n n u la te d , a n d its s u p e r f ic ia l lo c a t io n m a k e s it a n e a s y ta rg e t,
b u t it c a n b e m o r e d if f ic u lt to th r e a d a c a th e te r c e n tr a lly . T h e s u b c la v ia n a p p r o a c h h a s th e
b e n e f it o f a ls o h a v in g g o o d la n d m a r k s , a s w e ll a s r e m a in in g r e la tiv e ly p a te n t in a h y p o v o le m ic
p a tie n t. T h e s u b c la v ia n h o w e v e r d o e s c a r r y th e h ig h e s t r i s k o f p n e u m o th o r a x , a n d c a n b e
d if f ic u lt to c o m p r e s s if a h e m a to m a o r la c e r a tio n o c c u r s .

16. C. In th e C V P w a v e f o r m d e p ic te d b e lo w , th e A w a v e r e p r e s e n ts a tr ia l c o n tr a c tio n , th e C w a v e
r e p r e s e n ts b u lg in g o f th e tr ic u s p id v a lv e in to th e a tr iu m d u r in g th e b e g in n in g o f s y s to le , th e X
d e c e n t o c c u r s d u r in g s y s to le a n d c o r r e s p o n d s to a tr ia l r e la x a tio n , th e V w a v e r e p r e s e n ts f illin g
o f th e a tr iu m w h ile th e tr ic u s p id v a lv e is c lo s e d , a n d th e Y d e s c e n t o c c u r s w h e n th e tr ic u s p id
v a lv e o p e n s a n d th e a tr iu m s ta rts to e m p ty .

17. B. In th e C V P w a v e f o r m d e p ic te d in F ig u r e 4 - 4 , th e A w a v e r e p r e s e n ts a tr ia l c o n tr a c tio n , th e C
w a v e r e p r e s e n ts b u lg in g o f th e tr ic u s p id v a lv e in to th e a tr iu m d u r in g th e b e g in n in g o f s y s to le ,
th e X d e c e n t o c c u r s d u r in g s y s to le a n d c o r r e s p o n d s to a tr ia l r e la x a tio n , th e V w a v e r e p r e s e n ts
f illin g o f th e a tr iu m w h ile th e tr ic u s p id v a lv e is c lo s e d , a n d th e Y d e s c e n t o c c u r s w h e n th e
tr ic u s p id v a lv e o p e n s a n d th e a tr iu m s ta rts to e m p ty .

18. B. D u r in g s y s to le in a p a tie n t w ith tr ic u s p id r e g u r g ita tio n , p a r t o f th e e je c te d v o lu m e flo w s


b a c k w a r d in to th e a tr iu m . In s te a d o f s e e in g a s m a ll C w a v e th a t n o r m a l l y r e p r e s e n ts th e b u lg in g
o f th e tr ic u s p id v a lv e , a m u c h l a r g e r C w a v e w o u ld b e s e e n a s b l o o d f lo w s r e t r o g r a d e in to th e
r i g h t a tr iu m a n d to w a r d th e tr a n s d u c e r . T h is r e t r o g r a d e b l o o d f lo w w o u ld c o n tin u e t h r o u g h o u t
th e s y s to le , a n d w o u ld , th e r e f o r e , a ls o in c r e a s e th e V w a v e s iz e , s in c e th is is a s y s to lic
c o m p o n e n t o f th e C V P tra c e . D u r in g c a r d ia c ta m p o n a d e , th e r e w ill b e e le v a te d p r e s s u r e s
th r o u g h o u t th e e n tir e w a v e f o r m , a s w e ll a s lo s s o f th e Y d e s c e n t. In p a tie n ts w ith a tr ia l
f ib r illa tio n , th e r e w ill b e a lo s s o f th e A w a v e , s in c e th e r e is n o l o n g e r a u n if o r m a tr ia l
c o n tr a c tio n , a n d a n o v e r a ll in c r e a s e in th e C w a v e s iz e , s in c e f il lin g p r e s s u r e s e le v a te to
c o m p e n s a te a n d im p r o v e v e n tr ic u la r f illin g . W ith A V d is s o c ia tio n , th e r e a r e l a r g e a n d
e x a g g e r a te d A w a v e s ( o f te n c a lle d “ c a n n o n ” A w a v e s ), w h ic h r e p r e s e n t a tr ia l c o n tr a c tio n
a g a in s t a c lo s e d tr ic u s p id v a lv e .

19. A. S e e th e a n s w e r e x p la n a tio n o f Q u e s tio n 18. It w o u ld b e h ig h ly u n lik e ly to h a v e e le v a te d


f illin g p r e s s u r e s in a b le e d in g tr a u m a p a tie n t w h o h a s n o t y e t b e e n r e s u s c ita te d . A o r tic
d is s e c tio n s c a n c a u s e c a r d ia c ta m p o n a d e , b u t o n ly if th e y in v o lv e th e a o r tic r o o t a n d th e n e x te n d
in to th e p e r ic a r d iu m .

20. B. W h ile th e in c id e n c e o f c o m p lic a tio n s is in f r e q u e n t, s o m e o f th e c o m p lic a tio n s c a n c a r r y


s e v e r e m o r b i d ity a n d m o r t a lit y r is k s . In a d d itio n to u n iv e r s a l c o m p lic a tio n s a s s o c ia te d w ith
c e n tr a l lin e p la c e m e n t, s o m e a d d itio n a l p u lm o n a r y a r te r y c a th e te r c o m p lic a tio n s in c lu d e
d y s r h y th m ia s ( m o s t c o m m o n ) , c a th e te r k n o ttin g , c a r d ia c v a lv e in ju r y , p u lm o n a r y a r te r y
r u p tu r e , d e v e lo p m e n t o f c o m p le te h e a r t b lo c k in a p a tie n t w ith p r e e x is tin g le f t b u n d le b r a n c h
b lo c k , p u lm o n a r y th r o m b o e m b o li s m o r a ir e m b o lis m , b a c te r e m ia , e n d o c a r d itis , a n d s e p s is .

21. B. A s w e ll a s k n o w in g s o m e v a lu a b le in d ic a tio n s , it is im p o r ta n t to k n o w s o m e o f th e
lim ita tio n s o f a p u lm o n a r y a r te r y c a th e te r b e f o r e s u b je c tin g a p a tie n t to r is k s . F o r e x a m p le , th e
m e a s u r e m e n t o f c a r d ia c o u tp u t in p a tie n ts w ith tr ic u s p id r e g u r g i t a t i o n o r v e n tr ic u la r s e p ta l
d e fe c ts is in a c c u r a te d u e to d ilu tio n o f th e in je c ta te . P u lm o n a r y a r te r y o c c lu s io n p r e s s u r e c a n
a ls o in a c c u r a te ly r e p r e s e n t le f t v e n tr ic u la r e n d d ia s to lic p r e s s u r e in p a tie n ts w ith m itr a l
s te n o s is , le f t a tr ia l m y x o m a s , p u lm o n a r y v e n o u s o b s tr u c tio n , e le v a te d a lv e o la r p r e s s u r e s , a n d
d e c r e a s e d le f t v e n tr ic u la r c o m p lia n c e . O th e r c o m m o n e r r o r s in m e a s u r e m e n t th a t a r e n o t
p a tie n t d e p e n d e n t c a n in c lu d e a n in a c c u r a te v o lu m e o r te m p e r a tu r e o f th e in je c ta te s o lu tio n .

22. B. A p u lm o n a r y a r te r y c a th e te r is p la c e d w h ile m o n ito r in g th e p r e s s u r e c h a n g e s m e a s u r e d a t


th e tip o f th e c a th e te r. T h e f i r s t s e c tio n s h o w s a tr a d itio n a l C V P w a v e f o r m m e a s u r e d in th e r i g h t
a tr iu m . A s th e c a th e te r is a d v a n c e d , a s y s to lic s te p -u p is s e e n w h e n e n te r in g th e r i g h t v e n tr ic le , a
d ia s to lic s te p -u p w h e n e n te r in g th e p u lm o n a r y a rte ry , a n d a r e tu r n to a tr a d itio n a l C V P
w a v e f o r m w h e n e n te r in g th e w e d g e p o s itio n .
23. C. T h e tip o f th e p u lm o n a r y a r te r y c a th e te r ty p ic a lly e n te r s th e p u lm o n a r y a r te r y a t a r o u n d 3 5
to 4 5 c m . T h is c a n v a r y f r o m p a tie n t to p a tie n t, e s p e c ia lly w ith p a tie n ts a t th e e x tr e m e s o f h e ig h t.

24. B. M ix e d v e n o u s o x y g e n te n s io n c a n p r o v id e v a lu a b le i n f o r m a t io n o n th e b a la n c e b e tw e e n
o x y g e n c o n s u m p tio n a n d d e liv e ry . T y p ic a l m ix e d v e n o u s o x y g e n te n s io n in a h e a lth y a d u lt is 4 0
m m H g , y ie ld in g a s a tu r a tio n o f a p p r o x im a te ly 7 5 % . R e d u c tio n in o x y g e n d e liv e r y c a n b e d u e
to a r e d u c tio n in o x y g e n c o n te n t p e r d e c ilite r le a v in g th e le f t v e n tr ic le , o r a r e d u c tio n in o v e r a ll
c a r d ia c o u tp u t. I n c r e a s e d o x y g e n c o n s u m p tio n ( lo w m ix e d v e n o u s o x y g e n ) o c c u r s d u r in g
p e r io d s o f e le v a te d m e ta b o lic s ta te s , s u c h a s d u r in g v i g o r o u s e x e r c is e o r s e p s is .

25. A. In th e c lin ic a l s c e n a r io , l o w c e n tr a l v e n o u s , p u lm o n a r y a rte ry , a n d p u lm o n a r y a r te r y


o c c lu s io n p r e s s u r e s s u p p o r t th e d ia g n o s is o f h y p o v o le m ia . I n c r e a s in g in tr a v a s c u la r v o lu m e
w o u ld b e th e m o s t b e n e f ic ia l in te r v e n tio n a t th is tim e .

26. C. T h e p a tie n t’s h is to r y a n d c lin ic a l s c e n a r io s u g g e s t r i g h t h e a r t f a ilu r e d u e to p u lm o n a r y


h y p e r te n s io n . M ilr in o n e m a y b e b e n e f ic ia l in d e c r e a s in g p u lm o n a r y v a s c u la r r e s is ta n c e a s w e ll
a s in c r e a s in g c a r d ia c o u tp u t.

27. C. I n te r p r e tin g p h y s io lo g ic d a ta f r o m a p u lm o n a r y a r te r y c a th e te r a n d g u id in g th e r a p y
r e q u ir e s h a v in g a n in tim a te k n o w le d g e o f b a s e lin e v a lu e s . O n a v e r a g e , n o r m a l p h y s io lo g ic
v a s c u la r r e s is ta n c e f a lls b e tw e e n 9 0 0 a n d 1 5 0 0 ( d y n e s ) ( s ) /c m 5.

28. A. N o r m a l p u lm o n a r y v a s c u la r r e s is ta n c e r a n g e s b e tw e e n a p p r o x im a te ly 5 0 a n d 1 5 0 (d y n e s )
( s ) /c m 5.

29. C. N o r m a l c a r d ia c in d e x in a h e a lth y a d u lt r a n g e s b e tw e e n 2.2 a n d 4 .2 L / m in /m 2. C a r d ia c


in d e x is o f te n u s e d o v e r c a r d ia c o u tp u t in e s tim a tin g c a r d ia c fu n c tio n , s in c e it is m o r e r e lia b le
w ith e x tr e m e s o f h e ig h t.

30. B. S e r io u s c o m p lic a tio n s w ith T E E h a v e b e e n r e p o r te d in a p p r o x im a te ly 0 .1 % o f c a s e s , o r


a p p r o x im a te ly 1 in 1 ,0 0 0 p a tie n ts . S tr ic t c o n tr a in d ic a tio n s to T E E in c lu d e b u t a r e n o t lim ite d to
e s o p h a g e a l s p a s m , e s o p h a g e a l s tr ic tu r e , e s o p h a g e a l la c e r a tio n , e s o p h a g e a l p e r f o r a tio n , a n d
e s o p h a g e a l d iv e r tic u lu m . R e la tiv e c o n tr a in d ic a tio n s in c lu d e b u t a r e n o t lim ite d to u p p e r G I
b le e d , d y s p h a g ia o r o d y n o p h a g ia , m e d ia s tin a l r a d ia tio n , l a r g e d ia p h r a g m a tic h e r n ia s ,
a tla n to a x ia l d is e a s e , a n d d if f ic u lt in tu b a tio n d u e to p o s s ib il ity o f u n in te n tio n a l e x tu b a tio n w ith
p r o b e m a n ip u la tio n .

31. C. T h e c u r r e n t g u id e lin e s r e c o m m e n d a N y q u is t l im it o f 5 0 to 6 0 c m /s w h e n e v a lu a tin g


r e g u r g i t a n t le s io n s . S e ttin g th e l im it to l o w c o u ld r e s u lt in o v e r e s tim a tin g th e r e g u r g i t a n t le s io n ,
a n d s e ttin g th e l im it to h ig h c o u ld r e s u lt in u n d e r e s tim a tin g th e r e g u r g i t a n t le s io n .

32. B. P u ls e -w a v e D o p p le r is u s e d to c a p tu r e f lo w a t a s p e c if ic p o in t. D u r in g p u ls e - w a v e D o p p le r,
a s in g le c r y s ta l is u s e d to b o th e m it a n d r e c e iv e u ltr a s o u n d e n e r g y , a n d th e lo c a t io n o f th e s ig n a l
c a n b e c a lc u la te d . C o n tin u o u s - w a v e D o p p le r, o n th e o th e r h a n d , u s e s tw o s e p a r a te c r y s ta ls to
s e n d a n d r e c e iv e u ltr a s o u n d e n e r g y . T h is a llo w s th e e c h o m a c h in e to d e te c t h ig h e r v e lo c itie s
a n d e n e r g y s h ifts ; h o w e v e r, th e e x a c t lo c a t io n o f th e s ig n a l c a n n o t b e d e te r m in e d . C o lo r - w a v e
D o p p le r is u s e d to e x a m in e r e g u r g i t a n t le s io n s .

33. B. P u ls e o x im e tr y u s e s tw o w a v e le n g th s o f lig h t to c a lc u la te o x y g e n s a tu r a tio n . T h e s e


w a v e le n g th s a r e 6 6 0 n m o f r e d lig h t (w e ll a b s o r b e d b y o x y g e n a te d h e m o g lo b in ) a n d 9 4 0 n m o f
in f r a r e d lig h t (w e ll a b s o r b e d b y d e o x y g e n a te d h e m o g lo b in ) .

34. B. T h e a c c u r a c y o f p u ls e o x im e tr y c a n b e a ffe c te d b y m a n y f a c to r s . T h e s e in c lu d e b u t a r e n o t
lim ite d to lo w b l o o d f lo w c o n d itio n s , p a tie n t m o v e m e n t, a m b ie n t lig h t, d y s f u n c tio n a l
h e m o g lo b in m o le c u le s , d y e s s u c h a s m e th y le n e b lu e a n d in d ig o c a r m in e , a n d a lte r e d
r e la tio n s h ip s in th e h e m o g lo b in d is s o c ia tio n c u r v e ( s e v e r e a c id o s is ) . I n tr a v e n o u s h e p a r in b o lu s
is n o t k n o w n to d is to r t th e a c c u r a c y o f p u ls e o x im e try .

35. D. M a n y d if f e r e n t c lin ic a l s itu a tio n s w ill c a u s e p u ls e o x im e tr y to r e a d in c h a r a c te r is tic


p a tte rn s . M e th e m o g lo b in e m ia a b s o r b s b o th w a v e le n g th s o f lig h t a n d te n d s to c o n v e r g e a r o u n d a
s a tu r a tio n o f 8 5 % . C a r b o x y h e m o g l o b in o n ly a b s o r b s r e d lig h t, b u t n o t in f r a r e d lig h t, a n d c a n
v a r y w id e ly in s a tu r a tio n r e a d in g s . M e th y le n e b lu e , a c o m m o n d y e u s e d d u r in g s u r g e r y , te n d s to
c a u s e s a tu r a tio n s to c o n v e r g e a r o u n d 6 5 % .

36. A. M a n y d if f e r e n t c lin ic a l s itu a tio n s w ill c a u s e p u ls e o x im e tr y to r e a d in c h a r a c te r is tic


p a tte rn s . M e th e m o g lo b in e m ia a b s o r b s b o th w a v e le n g th s o f lig h t a n d te n d s to c o n v e r g e a r o u n d a
s a tu r a tio n o f 8 5 % . C a r b o x y h e m o g l o b in o n ly a b s o r b s r e d lig h t, b u t n o t in f r a r e d lig h t, a n d c a n
v a r y w id e ly in s a tu r a tio n r e a d in g s . M e th y le n e b lu e , a c o m m o n d y e u s e d d u r in g s u r g e r y , te n d s to
c a u s e s a tu r a tio n s to c o n v e r g e a r o u n d 6 5 % .

37. B. T h e c o r tic o s p i n a l tra c ts r e s p o n s ib le f o r m o to r f u n c tio n tr a v e l a lo n g th e a n te r io r s p in a l


c o r d , a n d c a n b e m o n ito r e d u s in g m o to r - e v o k e d p o te n tia ls . S e n s o r y tra c ts , o n th e o th e r h a n d ,
tr a v e l a lo n g th e p o s te r io r s p in a l c o r d , a n d c a n b e m o n ito r e d u s in g s o m a t o s e n s o r y - e v o k e d
p o te n tia ls . E l e c tr o e n c e p h a lo g r a p h y is c o m m o n ly u s e d to m e a s u r e c e r e b r a l a c tiv ity d u r in g
n e u r o v a s c u la r s u r g e r ie s , s u c h a s c a r o tid e n d a r te r e c to m ie s , lo o k in g f o r d e c r e a s e d c e r e b r a l
b l o o d flo w . B is p e c tr a l in d e x o r S e d lin e m o n ito r in g is s o m e w h a t c o n tr o v e r s ia l, b u t is u s e d to
m o n ito r th e a d e q u a c y o f d e p th o f a n e s th e s ia .

38. B. S S E P s m o n ito r th e p o s t e r i o r s p in a l c o lu m n , w h ic h w o u ld b e a ffe c te d b y d a m a g e to th e


p o s te r io r s p in a l a r te r ie s o r c o m p r e s s io n o f th e p o s t e r i o r s p in a l c o r d . A lig h t p la n e o f a n e s th e s ia
w o u ld n o t c a u s e a d r o p in S S E P s, n o r w o u ld th e a d m in is tr a tio n o f a n e u r o m u s c u la r b lo c k in g
a g e n t (th e la tte r w o u ld h in d e r th e u s e o f m o to r - e v o k e d p o te n tia ls ) .

39. C. H a lo g e n a te d a n e s th e tic s a s w e ll a s n itr o u s o x id e ( e s p e c ia lly w h e n c o m b in e d to g e th e r ) c a n


d e c r e a s e a m p litu d e a n d in c r e a s e la te n c y . F o r th is r e a s o n , it is r e c o m m e n d e d to m in im iz e th e u s e
o f v o la tile a n e s th e tic s to b e lo w 1 M A C , o r to u s e a to ta l in tr a v e n o u s te c h n iq u e w h e n m o n ito r in g
S S E P s.

40. A. E to m id a te is k n o w n to in c r e a s e th e a m p litu d e o f s o m a t o s e n s o r y - e v o k e d p o te n tia ls (S S E P s ),


a n d c a n s o m e tim e s b e d r a m a tic . P r o p o f o l is c o n s id e r e d to h a v e m in im a l to n o e f f e c t o n
a m p litu d e , a n d is c o m m o n ly u s e d a s a n in f u s io n f o r th e m a in te n a n c e o f a n e s th e s ia w h e n
m o n ito r in g S S E P s. M id a z o la m h a s b e e n s h o w n to d e c r e a s e a m p litu d e , a n d th is s h o u ld b e k e p t in
m in d w h e n u s e d f o r p r e m e d ic a tio n . A s d is c u s s e d in th e p r e v io u s q u e s tio n , s e v o f lu r a n e w o u ld b e
e x p e c te d to d e c r e a s e a m p litu d e a n d in c r e a s e la te n c y o f S S E P s.

41. A. M e d ic a tio n s a r e n o t th e o n ly v a r ia b le s th a t a f f e c t s o m a t o s e n s o r y - e v o k e d p o te n tia ls , as


c h a n g e s in p h y s i o l o g y c a n a ls o a lte r la te n c y a n d a m p litu d e . A m p litu d e d e c r e a s e s d u r in g
e p is o d e s o f h y p o te n s io n , h y p o x ia , a n d h y p e r th e r m ia . L a te n c y c a n b e in c r e a s e d d u r in g
h y p o th e r m ia , h y p o c a r b ia , a n d h e m o d ilu tio n /a n e m ia .

42. A. A to B o c c u r s d u r in g e x h a la tio n o f a n a to m ic d e a d s p a c e , B to C o c c u r s d u r in g m ix in g o f
e x h a le d d e a d s p a c e a n d a lv e o la r g a s , C to D r e f le c ts th e e x h a la tio n o f a lv e o la r g a s , w ith p o in t D
c o r r e la tin g w ith e n d -tid a l c a r b o n d io x id e , a n d D to E r e p r e s e n ts th e b e g in n in g o f in s p ir a tio n .

43. D. A to B o c c u r s d u r in g e x h a la tio n o f a n a to m ic d e a d s p a c e , B to C o c c u r s d u r in g m ix in g o f
e x h a le d d e a d s p a c e a n d a lv e o la r g a s , C to D r e f le c ts th e e x h a la tio n o f a lv e o la r g a s , w ith p o in t D
c o r r e la tin g w ith e n d -tid a l c a r b o n d io x id e , a n d D to E r e p r e s e n ts th e b e g in n in g o f in s p ir a tio n .

44. A. It is im p o r ta n t to r e m e m b e r th a t c a p n o g r a p h y w ill s h o w a n o r m a l c a p n o g r a p h a n d e n d -tid a l


C O 2 im m e d ia te ly f o ll o w i n g e n d o b r o n c h ia l in tu b a tio n . A n e s th e s ia p r o v i d e r s m u s t b e v ig ila n t to
a lw a y s lis te n f o r b ila te r a l b r e a th s o u n d s a n d o b s e r v e b ila te r a l c h e s t r i s e to c o n f ir m tr a c h e a l
in tu b a tio n .

45. D. T h e c la s s ic im a g e a b o v e is c o m m o n ly r e f e r r e d to a s a c u r a r e c le ft, a n d o c c u r s w h e n a
p a tie n t b e g in s to a tte m p t in s p ir a tio n d u r in g th e e x p ir a to r y p h a s e o f m e c h a n ic a l v e n tila tio n . T h is
is o n e o f th e in d ic a tio n s th a t n e u r o m u s c u la r f u n c tio n is r e tu r n in g .
46. C. O n a v e r a g e , c o r e te m p e r a tu r e d e c lin e s b y a p p r o x im a te ly 1 to 1 .5 °C a fte r th e in d u c tio n o f
g e n e r a l a n e s th e s ia . T h is in itia l d r o p in c o r e b o d y te m p e r a tu r e is p r i m a r i l y d u e to r e d is tr ib u tio n
( c o r e to p e r ip h e r y ) f r o m th e v a s o d ila tin g p r o p e r tie s o f m a n y a n e s th e tic s . T e m p e r a tu r e m a y
c o n tin u e to d r o p a s p r o c e s s e s o f h e a t lo s s , s u c h a s c o n d u c tio n , c o n v e c tio n , r a d ia tio n , a n d
e v a p o r a tio n , o c c u r (a s o p p o s e d to r e d is tr ib u tio n ) .

47. D. T h e c u r r e n t r e c o m m e n d a tio n s f r o m th e A m e r ic a n S o c ie ty o f A n e s th e s io lo g is ts s ta te th a t
te m p e r a tu r e m o n ito r in g is r e q u ir e d “w h e n c lin ic a ll y s ig n if ic a n t c h a n g e s in b o d y te m p e r a tu r e
a r e in te n d e d , a n tic ip a te d , o r s u s p e c te d .” In a d d itio n to c o n s id e r in g th e s u r g ic a l p r o c e d u r e , it is
a ls o im p o r ta n t to c o n s id e r a t r i s k p o p u la tio n s s u c h a s th e e ld e r ly , in fa n ts , b u r n p a tie n ts , a n d
p a tie n ts w ith a u to n o m ic d y s fu n c tio n .

48. A. U n c o n tr o lle d h y p o th e r m ia h a s m a n y d e tr im e n ta l e ffe c ts , in c lu d in g in c r e a s e d o x y g e n


u tiliz a tio n th r o u g h s h iv e r in g , im p a ir e d p la te le t f u n c tio n a n d c o a g u la tio n , d e la y e d w o u n d
h e a lin g a n d in c r e a s in g s u r g ic a l s ite in f e c tio n s , a s w e ll a s p o te n tia l f o r s e r io u s d y s r h y th m ia s .
C e r e b r a l o x y g e n c o n s u m p tio n , h o w e v e r, d e c r e a s e s b y a p p r o x im a te ly 7 % p e r d e g r e e C e ls iu s
d e c r e a s e in te m p e r a tu r e .

49. A. N u m e r o u s s ite s c a n b e u s e d to m o n ito r te m p e r a tu r e in th e o p e r a tin g r o o m . O f th e m o s t


c o m m o n , ty m p a n ic m e m b r a n e ( p e r f u s e d b y c a r o tid a r te r y ) a n d p u lm o n a r y a r te r y m e a s u r e m e n ts
te n d to b e th e b e s t r e f le c to r s o f c o r e te m p e r a tu r e , f o ll o w e d b y b la d d e r te m p e r a tu r e s . R e c ta l
te m p e r a tu r e s o v e r a ll te n d to b e a p o o r s u b s titu te , w h ile a x illa r y a n d s k in te m p e r a tu r e s a r e
h ig h ly p r o n e to e r r o r .

50. C. U n d e r s ta n d in g th e lim ita tio n s o f n e u r o m u s c u la r tw itc h m o n ito r in g d e v ic e s is fu n d a m e n ta l


f o r a n a n e s th e s ia p r o v id e r . A t th e p o in t th e f o u r th tw itc h r e a p p e a r s , s till u p to 7 5 % to 8 0 % o f
a c e ty lc h o lin e r e c e p to r s m a y b e b lo c k e d . A d e q u a te r e v e r s a l ( n e o s t i g m i n e - g l y c o p y r r o l a t e )
s h o u ld b e g iv e n , a n d c lin ic a l s ig n s f o r r e tu r n o f n e u r o m u s c u la r f u n c tio n s h o u ld b e u s e d to
g a u g e r e a d in e s s f o r e x tu b a tio n .
Fluid Management and Blood Transfusion
Rebecca Kalman and Edward Bittner

1. All of the following are signs of dehydration, except


A. Progressive metabolic acidosis
B. Urinary specific gravity > 1.010
C. Urine osmolality < 300 mOsm/kg
D. Urine sodium < 10 mEq/L

2. Regarding central venous pressure (CVP) monitoring


A. Low values of <5 mm Hg may be considered normal in the absence of other signs of
hypovolemia
B. CVP readings can be interpreted independently of the clinical setting
C. CVP monitoring is never indicated in patients with normal cardiac and pulmonary function
D. In a patient with right ventricular dysfunction, a CVP of 10 mm Hg should be considered
elevated

3. In healthy patients, the lactate in lactated Ringer solution


A. Causes a lactic acidosis
B. Is converted to bicarbonate by the liver
C. Is rapidly bound by albumin
D. Causes a hyperchloremic metabolic acidosis

4. All of the following fluids are generally considered to be isotonic, except


A. Lactated Ringer
B. Normal saline
C. D5 normal saline
D. D5^ normal saline

5. All of the following statements regarding dextran solutions are true, except
A. Dextran 40 may improve blood flow through the microcirculation
B. Dextrans may have antiplatelet effects
C. Large-volume infusions of dextrans have been associated with renal failure
D. Dextran 40 is a better volume expander than dextran 70
6. Which of the following statements is true regarding fluid loss?
A. Substantial evaporative losses can be associated with large wounds and are directly
proportionate to the surface area exposed
B. Internal redistribution of fluids, “third spacing,” cannot cause massive fluid shifts
C. Traumatized, inflamed, or infected tissues can only sequester minimal amounts of fluid in
the interstitial space
D. Cellular dysfunction as a result of hypoxia usually produces a decrease in intracellular
fluid volume

7. The probability of developing anti-D antibodies after a single exposure to the Rh antigen is
A. <1%
B. 5% to 10%
C. 50% to 70%
D. >80%

8. In a conventional crossmatch
A. Donor cells are mixed with recipient serum
B. Recipient cells are mixed with donor serum
C. Donor serum is tested against red cells of known antigenic composition
D. None of the above

9. A leftward shift of the oxyhemoglobin dissociation curve may be related to


A. Low levels of 2,3-DPG in packed red blood cells
B. Hypothermia resulting from transfusion of blood
C. Both A and B
D. None of the above

10. Which of the following statements regarding fresh-frozen plasma (FFP) is correct?
A. Contains all of the clotting factors except factor VIII
B. Should not be used in patients with antithrombin III deficiency
C. Carries the same infection risk as a unit of whole blood
D. Is contraindicated in the case of isolated-factor deficiencies

11. The most common cause of an acute hemolytic transfusion reaction is


A. An error during type and screen
B. An error during type and crossmatch
C. Misidentification of the patient, blood specimen, or transfusion unit
D. Defective blood filter

12. Evidence for the fact that leukocyte-containing blood products appear to be immunosuppressive
includes all of the following, except
A. Preoperative blood transfusions appear to improve graft survival in renal transplant
patients
B. Recurrence of malignant growths may be more likely in patients who receive a blood
transfusion during surgery
C. Transfusion of allogeneic leukocytes can activate latent viruses in a recipient
D. Blood transfusion may decrease the incidence of serious infection following surgery or
trauma

13. Bacterial infection due to a contaminated blood product is most likely with transfusion of
A. Packed red blood cells
B. Fresh-frozen plasma
C. Platelets
D. Cryoprecipitate

14. All of the following qualities are advantages of crystalloid solutions, except
A. Nontoxic
B. Reaction-free
C. Relatively inexpensive
D. Have the ability to remain in the intravascular space for a relatively long amount of time

15. Administration of large volumes of normal saline can lead to


A. A metabolic alkalosis
B. A hyperchloremic-induced nongap metabolic acidosis
C. An anion gap lactic acidosis
D. None of the above

16. All of the following solutions contain potassium, except


A. Lactated Ringer solution
B. PlasmaLyte
C. Hespan
D. Packed red blood cells

17. The storage time for packed red blood cells at temperatures of 1 to 6°C is
A. 7 to 10 days
B. 21 to 35 days
C. 60 to 80 days
D. 120 days

18. Which of the following statements regarding transfusion of packed red blood cells is most
correct?
A. The hematocrit of 1 unit is usually 30% to 40%
B. T r a n s f u s io n o f a s in g le u n it w ill in c r e a s e a n a d u lt’s h e m o g lo b in c o n c e n tr a tio n a b o u t 4 g /d L
C. M a y c a u s e c lo ttin g if th e tr a n s f u s e d p a c k e d r e d b l o o d c e lls a r e m ix e d w ith la c ta te d R in g e r
s o lu tio n
D. T h e ir p r in c ip le u s e a s th a t o f a v o lu m e e x p a n d e r

19. B lo o d p r o d u c ts a r e te s te d f o r a ll o f th e f o ll o w i n g , e x c e p t

A. H e p a titis C
B. H IV
C. W e s t N ile v ir u s
D. H e r p e s v ir u s

20. R e g a r d in g a s s e s s m e n t o f s u r g ic a l b l o o d lo s s

A. B o th s u r g e o n s a n d a n e s th e s io lo g is ts te n d to u n d e r e s tim a te b l o o d lo s s
B. M e a s u r e m e n t o f b l o o d in th e s u r g ic a l s u c tio n c o n ta in e r is a ll th a t is n e c e s s a r y to e s tim a te
b l o o d lo s s
C. T h e u s e o f i r r i g a t i n g s o lu tio n s d o e s n o t c o m p lic a te a s s e s s m e n t o f b l o o d lo s s
D. A s o a k e d “ l a p ” p a d c a n h o ld 10 to 15 m L o f b lo o d

21. T h e m o s t c o m m o n n o n h e m o ly tic r e a c tio n to tr a n s f u s io n o f b l o o d p r o d u c ts is

A. A lle r g ic
B. F e b r ile
C. A n a p h y la c to id
D. U r tic a r ia l

22. T y p e s o f a u to lo g o u s b l o o d tr a n s f u s io n in c lu d e a ll o f th e f o ll o w i n g , e x c e p t

A. P r e d e p o s ite d d o n a tio n
B. I n tr a o p e r a tiv e b l o o d s a lv a g e
C. N o r m o v o le m ic h e m o d ilu tio n
D. D o n o r - d i r e c te d tr a n s f u s io n

23. A p a tie n t w ith ty p e O b l o o d w ill h a v e w h ic h o f th e f o ll o w i n g p la s m a a n tib o d ie s ?

A. A n ti-A
B. A n ti-B
C. B o th a n ti-A a n d a n ti-B
D. N one

24. A fte r b l o o d is c o lle c te d , th e p r e s e r v a tiv e C P D A -1 is c o m m o n ly a d d e d . T h is c o n ta in s a ll o f th e


f o ll o w i n g , e x c e p t

A. C itra te
B. P h o s p h a te
C. D e x tr o s e
D. Potassium

25. A 51-year-old patient was an unrestrained driver in a motor vehicle crash in which he sustained
multiple traumatic injuries. He is on mechanical ventilation, and has received 8 units of packed
red blood cells, 4 units of fresh-frozen plasma, and 6 units of platelets. His arterial blood gas
reveals a metabolic alkalosis. The most likely explanation for this finding is
A. Metabolism of citrate to bicarbonate
B. Under-resuscitation
C. Continued bleeding
D. Hypoventilation

26. A 70-year-old patient with chronic renal failure is in the operating room undergoing a kidney
transplant. There has been more blood loss than expected, and he has received 6 units of packed
red blood cells and 3 units of fresh-frozen plasma. The surgeons still complain that the patient
“won’t clot.” All of the following are potential contributors to his coagulopathy, except
A. Temperature of 34.9°C
B. Uremia
C. Dilutional thrombocytopenia
D. Fibrinogen level of 250 mg/dL

27. The estimated maintenance fluid requirement for a 9-year-old, 35-kg patient is
A. 50 mL/h
B. 75 mL/h
C. 100 mL/h
D. 20 mL/h

28. Which of the following patients is least likely to need calcium supplementation due to citrate-
induced hypocalcemia related to blood transfusion?
A. A 30-year-old trauma patient receiving massive blood transfusion through a rapid
transfuser at a rate of 75 mL/min
B. A patient with end-stage liver disease undergoing a complicated open shunt procedure, who
is hypothermic and has received greater than 2 blood volumes of transfusion
C. A neonate undergoing congenital diaphragmatic hernia repair
D. A 50-year-old patient with coronary artery disease undergoing an open femoral popliteal
bypass procedure, who has received 3 units of packed red blood cells

29. A medical student asks you if “young” blood is better for critically ill patients. Which of the
following statements regarding “young” blood is most correct?
A. Fresher blood has better ability to deliver oxygen to tissues
B. Blood from younger donors has lower risk of immunosuppression than blood donated by
the elderly
C. Older blood has a lower potassium content
D. F r e s h e r b l o o d c a n b e tr a n s f u s e d m o r e r a p id ly th a n o ld e r b lo o d

30. Y o u a r e c a r in g f o r a n 1 8 - y e a r - o ld f e m a le tr a u m a p a tie n t w h o w a s e m e r g e n tl y tr a n s p o r te d to th e
o p e r a tin g r o o m f o r c o n tr o l o f m a s s iv e b le e d in g . D u e to th e a c u te n e s s o f th e p a tie n t’s b le e d in g ,
th e r e w a s n o tim e f o r b l o o d ty p in g a n d s h e h a s r e c e iv e d 3 u n its o f O - n e g a tiv e p a c k e d r e d b lo o d
c e lls . T h e b l o o d b a n k n o tif ie s y o u th a t th e p a tie n t’s b l o o d ty p e is A - p o s itiv e . If th e p a tie n t
r e q u ir e s f u r th e r tr a n s f u s io n , w h ic h o f th e f o ll o w i n g s h o u ld b e a d m in is te r e d ?

A. A - p o s itiv e R B C s
B. A -n e g a tiv e R B C s
C. O - n e g a tiv e R B C s
D. R hoG A M
CHAPTER 5 ANSWERS

1. C. W h e n d e h y d ra te d , p a tie n ts w ith n o r m a l r e n a l f u n c tio n w ill r e ta in s o d iu m a n d p r o d u c e a


c o n c e n tr a te d u r in e . U r in e o s m o la lity is ty p ic a lly g r e a te r th a n 4 5 0 m O s m /k g in th is s e ttin g .
U r in e s o d iu m w ill b e lo w , a n d s p e c if ic g r a v ity w ill b e h ig h .

2. A. C V P m e a s u r e m e n ts m u s t b e e v a lu a te d in c o n te x t o f th e c lin ic a l s e ttin g . F a c to r s s u c h as
u n d e r ly in g c a r d io p u lm o n a r y d is e a s e , p a tie n t p o s itio n , a n d a n a to m y c a n a f f e c t th e v a lu e s . A C V P
o f < 5 m m H g c a n b e n o r m a l in a h e a lth y p a tie n t w ith o u t s ig n s o f h y p o v o le m ia . F o r s u r g ic a l
c a s e s d u r in g w h ic h l a r g e f lu id s h ifts a r e e x p e c te d , p la c e m e n t o f a C V P m o n ito r m a y b e
in d ic a te d . P a tie n ts w ith c o m p r o m is e d r i g h t v e n tr ic u la r f u n c tio n g e n e r a lly h a v e h ig h C V P s, a n d
th u s , a C V P o f 10 m m H g s h o u ld b e c o n s id e r e d n o r m a l to lo w d e p e n d in g o n th e d e g r e e o f
d y s fu n c tio n .

3. B. In h e a lth y p a tie n ts th e la c ta te in la c ta te d R in g e r s s o lu tio n is r a p id ly c o n v e r te d to


b ic a r b o n a te b y th e liv e r a n d d o e s n o t c a u s e a la c tic a c id o s is . A d m in is tr a tio n o f a l a r g e v o lu m e
o f n o r m a l s a lin e c a n c a u s e a h y p e r c h lo r e m ic m e ta b o lic a c id o s is . L a c ta te is n o t b o u n d b y
a lb u m in .

4. C. A n in tr a v e n o u s s o l u t i o n ’s e f f e c t o n f lu id m o v e m e n t d e p e n d s in p a r t o n its to n ic ity . T h is
te r m is s o m e tim e s u s e d in te r c h a n g e a b ly w ith o s m o la r ity , a lth o u g h th e y a r e s u b tly d if f e r e n t.
O s m o la r ity is th e n u m b e r o f o s m o le s o r m o le s o f s o lu te p e r lite r o f s o lu tio n . T o n ic ity is th e
e f f e c tiv e o s m o la lity a n d is e q u a l to th e s u m o f th e c o n c e n tr a tio n s o f th e s o lu te s w h ic h h a v e th e
c a p a c ity to e x e r t a n o s m o tic f o r c e a c r o s s th e m e m b r a n e . A s o lu tio n is is o to n ic if its to n ic ity
f a lls w ith in ( o r n e a r ) th e n o r m a l r a n g e f o r b l o o d s e r u m — f r o m 2 7 5 to 2 9 5 m O s m /k g . A
h y p o to n ic s o lu tio n h a s lo w e r o s m o l a r i t y (< 2 5 0 ), a n d a h y p e r to n ic s o lu tio n h a s h ig h e r
o s m o l a r i t y (> 3 5 0 ) (T a b le 5 -1 ).

Table 5-1 Osmolarity and tonicity of commonly used crystalloid solutions


F lu id M o s m /L T o n ic it y

Lactated Ringers 273 Isotonic


Normal saline 305 Isotonic
D5 normal saline 586 Hypertonic
D5M normal saline 355 Isotonic

5. D. W h ile d e x tr a n 4 0 h a s a m o le c u la r w e ig h t o f 4 0 ,0 0 0 , d e x tr a n 7 0 h a s a m o le c u la r w e ig h t o f
7 0 ,0 0 0 , a n d th e r e f o r e , th e la tte r is b r o k e n d o w n m o r e s lo w ly , la s ts lo n g e r , a n d is a b e tte r
v o lu m e e x p a n d e r. D e x tra n 4 0 a p p e a r s to im p r o v e b l o o d f lo w th r o u g h th e m i c r o c ir c u la tio n , a n d
a ll d e x tra n s m a y h a v e a n tip la te le t e ffe c ts . I n f u s io n o f l a r g e v o lu m e o f d e x tr a n (> 2 0 m L /k g /d a y )
h a s b e e n a s s o c ia te d w ith r e n a l f a ilu r e .

6. A. S u b s ta n tia l e v a p o r a tiv e lo s s e s c a n b e a s s o c ia te d w ith l a r g e w o u n d s a n d a r e d ir e c tly


p r o p o r tio n a t e to th e s u r f a c e a r e a e x p o s e d . T h i r d s p a c in g c a n c a u s e m a s s iv e f lu id s h ifts , a n d
tr a u m a tiz e d , in f la m e d , o r in f e c te d tis s u e c a n s e q u e s te r l a r g e a m o u n ts o f flu id . C e llu la r
d y s f u n c tio n a s a r e s u lt o f h y p o x ia u s u a lly p r o d u c e s a n in c r e a s e in in tr a c e l lu la r f lu id v o lu m e .

7. C. T h e R h b l o o d g r o u p is s e c o n d in im p o r ta n c e o n ly to th e A B O b l o o d g r o u p in th e f ie ld o f
tr a n s f u s io n m e d ic in e . It h a s r e m a in e d o f p r i m a r y im p o r ta n c e in o b s te tr ic s , b e in g th e m a in c a u s e
o f h e m o ly tic d is e a s e o f th e n e w b o r n . T h e s ig n if ic a n c e o f th e R h b l o o d g r o u p is r e la te d to th e
f a c t th a t th e R h a n tig e n (D a n tig e n ) is h ig h ly im m u n o g e n ic . In th e c a s e o f th e D a n tig e n ,
in d iv id u a ls w h o d o n o t p r o d u c e th e D a n tig e n w ill p r o d u c e a n ti-D if th e y e n c o u n te r th e D
a n tig e n w h e n tr a n s f u s e d w ith R B C s (c a u s in g a h e m o ly tic tr a n s f u s io n r e a c tio n ) . F o r th is r e a s o n ,
th e R h sta tu s is r o u ti n e ly d e te r m in e d in b l o o d d o n o r s , tr a n s f u s io n r e c ip ie n ts , a n d m o th e r s - to - b e .

8. A. A c r o s s m a tc h m im ic s a tr a n s f u s io n , w h e r e d o n o r c e lls a r e m ix e d w ith th e r e c i p i e n t ’s
s e r u m . T h is h a s th r e e p u r p o s e s : (1 ) c o n f ir m s A B O /R h ty p in g , (2 ) d e te c ts r e c ip ie n t a n tib o d ie s to
o th e r b l o o d g r o u p s y s te m s , a n d (3 ) d e te c ts a n tib o d ie s in l o w tite r s o r th o s e th a t d o n o t
a g g lu tin a te e a s ily . C h o ic e C d e s c r ib e s a n a n tib o d y s c r e e n .

9. C. T h e le v e l o f 2 ,3 -D P G in s to r e d b l o o d is re d u c e d , c a u s in g d e c r e a s e d o x y g e n u n lo a d in g to
th e tis s u e s . H y p o th e r m ia a ls o c a u s e s a le f tw a r d s h if t o f th e o x y h e m o g lo b in d is s o c ia tio n c u r v e
( F ig . 5 - 1 ) .
10. C. F F P is th e f lu id p o r t i o n o b ta in e d f r o m a s in g le u n it o f w h o le b l o o d th a t is f r o z e n w ith in 6
h o u r s o f c o lle c tio n . A ll c o a g u la tio n f a c to r s , e x c e p t p la te le ts , a r e p r e s e n t in FFP, w h ic h e x p la in s
th e u s e o f th is c o m p o n e n t in th e tr e a tm e n t o f h e m o r r h a g e . F F P is a ls o in d ic a te d in a n tith r o m b in
III d e f ic ie n c y a n d is o la te d - f a c to r d e f ic ie n c ie s . A tr a n s f u s io n o f F F P c a r r ie s th e s a m e r i s k o f
in f e c tio n a s tr a n s f u s in g a w h o le b lo o d .

11. C. H e m o ly tic r e a c tio n s o c c u r w h e n th e w r o n g b l o o d ty p e is a d m in is te r e d to a p a tie n t. T h e


im m e d ia te s ig n s o f a c u te h e m o ly tic tr a n s f u s io n r e a c tio n s in c lu d e lu m b a r a n d s u b s te r n a l p a in ,
fe v e r, c h ills , d y s p n e a , f lu s h in g o f th e s k in , a n d h y p o te n s io n . T h e a p p e a r a n c e o f f r e e
h e m o g lo b in in p la s m a o r u r in e is p r e s u m p tiv e e v id e n c e o f a h e m o ly tic r e a c tio n . A c u te r e n a l
f a ilu r e r e f le c ts p r e c ip ita tio n o f s tr o m a l a n d lip id c o n te n ts ( n o t f r e e h e m o g lo b in ) o f h e m o ly z e d
e r y th r o c y te s in d is ta l r e n a l tu b u le s . A c u te h e m o ly tic tr a n s f u s io n r e a c tio n s a r e u s u a lly d u e to
A B O b l o o d in c o m p a tib ility , a n d th e m o s t c o m m o n c a u s e is m is id e n tif ic a tio n o f th e p a tie n t,
b l o o d s p e c im e n , o r tr a n s f u s io n u n it ( c le r ic a l e r r o r ) .

12. D. B lo o d tr a n s f u s io n s u p p r e s s e s c e ll- m e d ia te d im m u n ity , w h ic h m a y p la c e s u r g ic a l p a tie n ts a t


r i s k f o r p o s to p e r a tiv e in f e c tio n . T h e a s s o c ia tio n w ith lo n g - t e r m p r o g n o s i s in c a n c e r s u r g e r y is
u n c le a r, b u t th e r e is a s u g g e s tio n o f a c o r r e l a t i o n b e tw e e n tu m o r r e c u r r e n c e a n d b l o o d
tr a n s f u s io n s . R e m o v in g m o s t o f th e w h ite b l o o d c e lls f r o m b l o o d a n d p la te le ts ( le u k o r e d u c tio n )
r e d u c e s th e in c id e n c e o f n o n h e m o ly tic f e b r ile tr a n s f u s io n r e a c tio n s a n d th e tr a n s m is s io n o f
le u k o c y te - a s s o c ia te d v ir u s e s . P r e o p e r a tiv e b l o o d tr a n s f u s io n s a p p e a r to im p r o v e g r a f t s u r v iv a l
in r e n a l tr a n s p la n t p a tie n ts .

13. C. O n e o f th e le a d in g c a u s e s o f tr a n s f u s io n - r e la te d fa ta litie s in th e U n ite d S ta te s is b a c te r ia l


c o n ta m in a tio n , w h ic h is m o s t lik e ly to o c c u r in p la te le t c o n c e n tr a te s . P la te le t- r e la te d s e p s is c a n
b e fa ta l a n d o c c u r s a s f r e q u e n tly a s 1 in 5 ,0 0 0 tr a n s f u s io n s . P la te le ts a r e s to r e d a t 2 0 to 2 4 ° C
in s te a d o f 4 °C , w h ic h p r o b a b ly a c c o u n ts f o r th e g r e a te r r i s k o f b a c te r ia l g r o w th th a n w ith o th e r
b l o o d p r o d u c ts . A n y p a tie n t in w h o m a f e v e r d e v e lo p s w ith in 6 h o u r s o f r e c e iv in g p la te le t
c o n c e n tr a te s s h o u ld b e c o n s id e r e d to b e p o s s ib ly m a n if e s tin g p la te le t- in d u c e d s e p s is , a n d
e m p ir ic a l a n tib io tic th e r a p y s h o u ld b e in s titu te d .

14. D. A d v a n ta g e s o f c r y s ta ll o id s o lu tio n s a r e th a t th e y a r e n o n to x ic , r e a c tio n - f r e e , a n d


in e x p e n s iv e . C o llo id s o lu tio n s a r e c o m p o s e d o f l a r g e - m o l e c u l a r - w e i g h t s u b s ta n c e s th a t r e m a in
in th e in tr a v a s c u la r s p a c e l o n g e r th a n c r y s ta llo id s , a n d ty p ic a lly , th e in itia l v o lu m e o f
d is tr ib u tio n is e q u iv a le n t to th e p la s m a v o lu m e . T h e s y n th e tic c o llo id s a n d p r o c e s s e d a lb u m in
h a v e m in im a l o r n o r is k s o f in f e c tio n . C o llo id s a r e m o r e e x p e n s iv e th a n c r y s ta llo id s , b u t h a v e
fe w e r r is k s th a n b l o o d p r o d u c ts .

15. B. N o r m a l s a lin e (0 .9 % N a C l) is s lig h tly h y p e r to n ic a n d c o n ta in s m o r e c h lo r id e th a n


e x tr a c e llu la r f lu id . A d m in is tr a tio n o f l a r g e v o lu m e s o f n o r m a l s a lin e s o lu tio n c a n le a d to a
h y p e r c h lo r e m ic n o n - a n i o n g a p m e ta b o lic a c id o s is . A d m in is tr a tio n o f l a r g e a m o u n ts o f la c ta te d
R in g e r s o lu tio n m a y r e s u lt in a m e ta b o lic a lk a lo s is b e c a u s e o f in c r e a s e d b ic a r b o n a te
p r o d u c tio n f r o m th e m e ta b o lis m o f la c ta te .
16. C. H e s p a n is c o l l o i d c o n ta in in g s ta r c h a n d s a lin e . A ll o f th e o th e r o p tio n s c o n ta in p o ta s s iu m .
M a n y p a tie n ts w ith h y p e r k a le m ia , in c lu d in g p a tie n ts w ith r e n a l f a ilu r e , r o u ti n e ly r e c e iv e n o r m a l
s a lin e b e c a u s e it c o n ta in s n o p o ta s s iu m .

17. B. T h e s to r a g e tim e (7 0 % v ia b ility o f tr a n s f u s e d e r y th r o c y te s 2 4 h o u r s a fte r tr a n s f u s io n ) is 21


to 3 5 d a y s , d e p e n d in g o n th e s to r a g e m e d iu m . C h a n g e s th a t o c c u r in b l o o d d u r in g s to r a g e
r e f le c t th e le n g th o f s to r a g e a n d th e ty p e o f p r e s e r v a tiv e u s e d .

18. C. M ix in g o f p a c k e d r e d b l o o d c e lls w ith la c ta te d R in g e r s o lu tio n c a n c a u s e c lo ttin g a s th e


c itr a te in th e b l o o d p r o d u c t c a n b in d w ith c a lc iu m in th e la c ta te d R in g e r. T h e o th e r o p tio n s a r e
a ll fa ls e . T h e h e m a t o c r i t o f 1 u n it o f p a c k e d r e d b l o o d c e lls is 7 0 % to 8 0 % . T r a n s f u s io n o f a
s in g le u n it w ill in c r e a s e a n a d u lt’s h e m o g lo b in c o n c e n tr a tio n b y a b o u t 1 g /d L . T h e o b je c tiv e in
tr a n s f u s io n o f p a c k e d r e d b l o o d c e lls is to in c r e a s e th e b l o o d ’s o x y g e n - c a r r y in g c a p a c ity .
A lth o u g h tr a n s f u s io n o f p a c k e d r e d b l o o d c e lls in c r e a s e s in tr a v a s c u la r f lu id v o lu m e , th e y
s h o u ld n o t b e u s e d r o u ti n e ly f o r th is p u r p o s e g iv e n th e r is k s a s s o c ia te d w ith tr a n s f u s io n .

19. D. T h e in c id e n c e o f in f e c tio n f r o m b l o o d tr a n s f u s io n s h a s m a r k e d ly d e c r e a s e d . A lth o u g h


m a n y f a c to r s a c c o u n t f o r th e m a r k e d d e c r e a s e d in c id e n c e o f tr a n s m is s io n o f in f e c tio u s a g e n ts
v ia b l o o d tr a n s f u s io n , th e m o s t im p o r ta n t o n e is im p r o v e d m e th o d s f o r te s tin g o f d o n o r b lo o d .
C u rre n tly , h e p a titis C , HIV, a n d W e s t N ile v ir u s a r e te s te d b y n u c le ic a c id te c h n o lo g y .

20. A. B o th s u r g e o n s a n d a n e s th e s io lo g is ts te n d to u n d e r e s tim a te b l o o d lo s s . M e a s u r e m e n t o f
b l o o d in th e s u r g ic a l s u c tio n c o n ta in e r is o n ly o n e c o m p o n e n t o f e s tim a tin g b l o o d l o s s . B lo o d
l o s t in s p o n g e s , “ l a p ” p a d s , a n d o c c u lt b le e d in g u n d e r th e d r a p e s m u s t b e a c c o u n te d fo r . T h e u s e
o f i r r i g a t i n g s o lu tio n s o f te n c o m p lic a te s th e a s s e s s m e n t o f b l o o d lo s s . A s o a k e d “ l a p ” p a d c a n
h o ld u p to 1 0 0 to 1 5 0 m L o f b lo o d .

21. B. F e b r ile r e a c tio n s a r e th e m o s t c o m m o n a d v e r s e n o n h e m o ly tic r e a c tio n a n d o c c u r w ith 0 .5 %


to 1 % o f tr a n s f u s io n s . T h e m o s t lik e ly c a u s e is a n in te r a c tio n b e tw e e n th e r e c i p i e n t ’s a n tib o d ie s
a n d th e a n tig e n p r e s e n t o n th e le u k o c y te s o f p la te le ts o f th e d o n o r . T h e p a tie n t’s te m p e r a tu r e
r a r e l y in c r e a s e s a b o v e 3 8 °C , a n d th e c o n d itio n is tr e a te d b y s lo w in g th e in f u s io n a n d
a d m in is te r in g a n tip y r e tic s . S e v e re f e b r ile r e a c tio n s a c c o m p a n ie d b y c h ills a n d s h iv e r in g m a y
r e q u i r e d is c o n tin u a tio n o f th e b l o o d tr a n s f u s io n .

22. D. A d ir e c te d ( o r d e s ig n a te d ) b l o o d d o n a tio n is o n e in w h ic h a p a tie n t s e le c ts h is /h e r o w n


b l o o d d o n o r ( s ) f o r a n a n tic ip a te d , n o n e m e r g e n c y tr a n s f u s io n . T h e d o n o r is ty p ic a lly a f r ie n d o r
r e la tiv e to th e p a tie n t. P a tie n ts u n d e r g o in g e le c tiv e p r o c e d u r e s w ith a h ig h p r o b a b ility o f b lo o d
tr a n s f u s io n c a n d o n a te th e ir o w n b l o o d 4 to 5 w e e k s p r i o r to s u r g e r y , a n d th is is r e f e r r e d to a s a
p r e d e p o s ite d d o n a tio n . B lo o d s a lv a g e r e f e r s to th e c o lle c tio n o f s h e d b l o o d in tr a o p e r a tiv e ly ,
w h ic h is th e n c o n c e n tr a te d , w a s h e d , a n d tr a n s f u s e d b a c k to th e p a tie n t. F o r n o r m o v o l e m i c
h e m o d ilu tio n , b l o o d is r e m o v e d j u s t p r i o r to s u r g e r y a n d r e p la c e d w ith c r y s ta ll o id o r c o llo id .
T h e b l o o d is s to r e d f o r u p to 6 h o u r s , a n d th e n b e g iv e n b a c k to th e p a tie n t a fte r b l o o d lo s s .
23. C. R o u tin e ty p in g o f b l o o d is p e r f o r m e d to id e n tif y th e a n tig e n s (A , B , R h ) o n th e m e m b r a n e s
o f e r y th r o c y te s . N a tu r a lly - o c c u r r in g a n tib o d ie s (a n ti-B , a n ti-A ) a r e f o r m e d w h e n e v e r
e r y th r o c y te m e m b r a n e s la c k A o r B a n tig e n s ( o r b o th ). T h e s e a n tib o d ie s a r e c a p a b le o f c a u s in g
r a p id in tr a v a s c u la r d e s tr u c tio n o f e r y th r o c y te s th a t c o n ta in th e c o r r e s p o n d i n g a n tig e n s .

24. D. C P D A -1 is th e m o s t c o m m o n ly a d d e d p r e s e r v a tiv e a d d e d to b l o o d p r o d u c ts . It c o n ta in s
c itr a te a s a n a n tic o a g u la n t, p h o s p h a te a s a b u f f e r , d e x tr o s e a s a r e d b l o o d c e ll e n e r g y s o u r c e ,
a n d a d e n in e n e e d e d f o r th e m a in te n a n c e o f r e d c e ll A T P le v e ls . T h e p o ta s s iu m f o u n d in b lo o d
c o m e s f r o m th e b r e a k d o w n o f r e d b l o o d c e lls .

25. A. T h e c itr a te in th e b l o o d p r e s e r v a tiv e is m e ta b o liz e d to b ic a r b o n a te b y th e liv e r a n d c a n


c a u s e a m e ta b o lic a lk a lo s is f o ll o w i n g a l a r g e - v o lu m e tr a n s f u s io n . U n d e r - r e s u s c ita tio n a n d
b le e d in g a r e lik e ly to c a u s e a m e ta b o lic a c id o s is , w h e r e a s h y p o v e n tila tio n c a u s e s a r e s p i r a t o r y
a c id o s is .

26. D. H y p o th e r m ia , u r e m ia , a n d d ilu tio n f r o m m a s s iv e tr a n s f u s io n a r e a ll p o te n tia l r e a s o n s f o r


c o a g u lo p a th y in th is p a tie n t. A f i b r i n o g e n g r e a te r th a n 1 5 0 m g /d L s h o u ld b e a d e q u a te f o r
c lo ttin g .

27. B. A c c o r d in g to th e “ 4 -2 -1 r u l e ,” 75 m L /h w o u ld b e th e m a in te n a n c e ra te . T h is is c a lc u la te d as
4 0 + 2 0 + 15 = 75 m L /h (T a b le 5 -2 ).

Table 5-2 Formula for calculation of maintenance fluid requirement

W e ig h t u p to 10 kg 4 m L /k g /h

11-20 kg Add 2 mL/kg/h


21 kg and above Add 1 mL/kg/h

28. D. H y p o c a lc e m ia a s a r e s u lt o f c itr a te b in d in g o f c a lc iu m is r a r e b e c a u s e o f m o b iliz a tio n o f


c a lc iu m s to r e s f r o m th e b o n e , a n d th e a b ility o f th e liv e r to r a p id ly m e ta b o liz e c itr a te to
b ic a r b o n a te . T h e r e f o r e , a r b i t r a r y a d m in is tr a tio n o f c a lc iu m in th e a b s e n c e o f o b je c tiv e e v id e n c e
o f h y p o c a lc e m ia is n o t in d ic a te d . S u p p le m e n ta l c a lc iu m m a y b e n e e d e d w h e n (1 ) th e r a te o f
b l o o d in f u s io n is m o r e r a p id th a n 50 m L /m in , (2 ) h y p o th e r m ia o r liv e r d is e a s e in te r f e r e s w ith
th e m e ta b o lis m o f c itra te , o r (3 ) th e p a tie n t is a n e o n a te .

29. A. F r e s h e r b l o o d (< 5 d a y s o f s to r a g e ) h a s b e e n r e c o m m e n d e d f o r c r itic a lly ill p a tie n ts in a n


e f f o r t to im p r o v e th e d e liv e r y o f o x y g e n ( 2 ,3 - d ip h o s p h o g ly c e r a te c o n c e n tr a tio n s a r e b e tte r
m a in ta in e d w ith f r e s h e r b lo o d ) . M o r e re c e n tly , s o m e e v id e n c e s u g g e s ts th a t a d m in is tr a tio n o f
y o u n g e r b l o o d (i.e., s to r e d < 1 4 d a y s ) is a s s o c ia te d w ith b e tte r o u tc o m e s in c lu d in g d e c r e a s e d
m o r t a lit y r a te a n d fe w e r p o s to p e r a tiv e c o m p lic a tio n s , e s p e c ia lly w ith m a jo r s u r g e r y .

30. C. In a n e m e r g e n c y s itu a tio n th a t r e q u ir e s tr a n s f u s io n b e f o r e ty p e a n d c o m p a tib ility te s tin g


c a n b e p e r f o r m e d , O - n e g a tiv e p a c k e d r e d b l o o d c e lls m a y b e a d m in is te r e d . E v e n if th e p a tie n t’s
b l o o d ty p e b e c o m e s k n o w n a n d a v a ila b le , a fte r 2 u n its o f ty p e O - n e g a tiv e p a c k e d r e d b l o o d
c e lls h a v e b e e n tr a n s f u s e d , s u b s e q u e n t tr a n s f u s io n s s h o u ld c o n tin u e w ith O - n e g a tiv e b lo o d .
R h o G A M is n o t in d ic a te d s in c e th e p a tie n t’s b l o o d ty p e is R h + .
Anesthetic Pharmacology
Mian Ahmad and Ashish Sinha

1. Correct statement about metabolism of drugs by the liver is


A. For drugs with low extraction ratio, liver blood flow is the rate-limiting step in their
metabolism
B. For drugs with high extraction ratio, the capacity of the liver to metabolize the drug is the
rate-limiting step
C. Cytochrome P450 system is highly drug-specific
D. Removal of the drug from the blood by hepatic clearance is directly proportional to hepatic
blood flow and intrinsic clearance

2. When asked to describe the symptoms of her allergy to a local anesthetic that a 26-year-old
female had at the dentist’s office, the patient describes a feeling of light-headedness,
palpitations, and flushing. This reaction is most likely caused by
A. Methylparaben reaction
B. Vasovagal reaction
C. Para-aminobenzoic acid allergy
D. Epinephrine in the local anesthetic

3. All of the following drugs increase the mean arterial blood pressure, except
A. Dopamine
B. Norepinephrine
C. Epinephrine
D. Isoproterenol

4. All of the following drugs increase cardiac output, except


A. Dopamine
B. Epinephrine
C. Dobutamine
D. Norepinephrine

5. In general, norepinephrine causes increase in all of the following, except


A. Mean arterial blood pressure
B. Heart rate
C. Cardiac dysrhythmias
D. Systemic vascular resistance

6. Stimulation of a 2 receptors causes


A. Hypertension
B. Bradycardia
C. Salivation
D. Anxiety

7. Labetalol is relatively contraindicated for


A. Treatment of hypertension in aortic dissection
B. Treatment of hypertension in preeclampsia
C. Hypertensive emergencies after cardiac surgery involving second-degree heat block
D. Hypertension secondary to clonidine withdrawal

8. The best initial treatment for anaphylaxis during general anesthesia is


A. Methylprednisolone
B. Famotidine
C. Diphenhydramine
D. Epinephrine

9. Compared with thiopental, etomidate causes


A. Less nausea
B. Increased seizure threshold
C. Greater myoclonic activity
D. Greater histamine release

10. Compared with propofol, ketamine causes


A. More depression of respiratory drive
B. More depression of airway reflexes
C. More bronchodilation
D. Less analgesia

11. A 65-year-old African American patient is undergoing laparoscopic repair of inguinal hernia
under general anesthesia. He has a history of hypertension, diabetes, and depression. His
medication list includes lisinopril, hydrochlorothiazide, metformin, and phenelzine.
Intraoperative hypotension develops secondary to injury to inferior epigastric artery. Which of
the following medications is relatively contraindicated to treat this hypotension?
A. Epinephrine
B. Norepinephrine
C. Ephedrine
D. Phenylephrine

12. True statement regarding flumazenil is


A. It binds irreversibly with benzodiazepine receptor
B. It causes hypertension and tachycardia
C. It has a shorter duration of action than midazolam
D. It reverses opioid-induced respiratory depression

13. Midazolam can be administered through all of the following routes, except
A. Oral
B. Sublingual
C. Transcutaneous
D. Transnasal

14. When sodium bicarbonate is added to lidocaine, more rapid onset of action of lidocaine occurs
because of
A. Increased nonionized lidocaine concentration
B. Increased ionized lidocaine concentration
C. Decreased extracellular pH
D. Increased intracellular pH

15. Which of the following findings suggests current use of cocaine in a patient undergoing
preoperative evaluation?
A. Bradycardia
B. Hypertension
C. Pinpoint pupils
D. Hypothermia

16. Which of the following local anesthetics is an ester?


A. Lidocaine
B. Prilocaine
C. Mepivacaine
D. Cocaine

17. Which of the statements among the following is true?


A. Ropivacaine is more potent than bupivacaine
B. Ropivacaine causes more motor than sensory block
C. Bupivacaine causes more vasoconstriction than ropivacaine
D. Ropivacaine is an S-enantiomer of bupivacaine
18. A 7 5 - y e a r - o ld p a tie n t is s h iv e r in g a n d h a s c h e s t p a in in th e r e c o v e r y r o o m f o llo w in g
e x p l o r a t o r y l a p a r o to m y f o r a r u p tu r e - o b s tr u c te d h e r n ia . H is h e a r t r a te is 1 2 3 /m in , b lo o d
p r e s s u r e is 2 0 0 /1 0 0 m m H g , a n d S p o 2 is 9 7 % o n 2 L o f o x y g e n v ia n a s a l c a n n u la . A n E K G
s h o w s S T -T w a v e c h a n g e s , w h ic h a r e tr e a te d w ith n itr o g l y c e r i n e w ith n o e ffe c t. W h ic h o f th e
f o ll o w i n g is th e m o s t a p p r o p r ia te n e x t s te p ?

A. A d m in is tr a tio n o f h y d r a la z in e
B. A d m in is tr a tio n o f n itr o p r u s s id e
C. A d m in is tr a tio n o f e s m o l o l
D. A p p lic a tio n o f a w a r m in g b la n k e t

19. W h ic h o f th e f o ll o w i n g s ta te m e n ts a b o u t th e lo c a l a n e s th e tic s is f a ls e ?

A. T h e y a re w eak b ases
B. T h e y c o n ta in e ith e r e s te r o r a m id e lin k a g e
C. It is th e ir c h a r g e d f o r m th a t in te r a c ts w ith th e r e c e p to r
D. T h e y b in d th e r e c e p to r in s id e th e c e ll

20. L o c a l a n e s th e tic s c a u s e th e ir e ffe c ts b y

A. I n c r e a s in g th e th r e s h o ld p o te n tia l
B. A lte r in g th e r e s tin g m e m b r a n e p o te n tia l
C. I n c r e a s in g th e r a te o f d e p o la r iz a tio n
D. D e c r e a s in g th e r a te o f d e p o la r iz a tio n

21. L ip id s o lu b ility o f lo c a l a n e s th e tic s

A. G e n e r a lly c o r r e la te s d ir e c tly w ith th e tim e to o n s e t o f a c tio n


B. In c r e a s e s a s th e f r a c tio n o f io n iz e d f o r m o f th e lo c a l a n e s th e tic in c r e a s e s
C. In c r e a s e s a s th e f r a c tio n o f u n io n iz e d f o r m o f th e lo c a l a n e s th e tic in c r e a s e s
D. M a y b e d if f e r e n t in in v iv o o r in v itr o s y s te m s

22. W h ic h is th e c o r r e c t e x p e c te d d u r a tio n o f a n e s th e s ia a fte r in f i ltr a tio n w ith th e f o ll o w i n g lo c a l


a n e s th e tic s ?
A. Lidocaine 60 to 120 minutes
B. Mepivacaine 120 to 240 minutes
C. Ropivacaine 120 to 180 minutes
D. Bupivacaine 120 to 180 minutes

23. U s e o f w h ic h o f th e f o ll o w i n g lo c a l a n e s th e tic s f o r s p in a l a n e s th e s ia is c o n tr o v e r s ia l?

A. R o p iv a c a in e
B. B u p iv a c a in e
C. T e tr a c a in e
D. L id o c a in e
The following three questions belong to this clinical situation:
During placement of an interscalene block utilizing 0.5% bupivacaine, a 62-year-old patient suddenly
starts experiencing seizures and loses consciousness.

24. Which of the following statements regarding local anesthetic toxicity is correct?
A. Seizure is a sign of neurotoxicity from high dose of local anesthetic
B. Loss of consciousness is a sign of low-dose local anesthetic neurotoxicity
C. The seizure threshold is increased by the administration of thiopental
D. Seizure may have been caused by injection of the local anesthetic into cervical nerve root

25. Which of the following statements is false?


A. Seizure may have happened secondary to the injection of local anesthetic into vertebral
artery
B. Loss of consciousness may be secondary to high epidural anesthesia
C. Loss of consciousness may be secondary to high spinal anesthesia
D. In general, decreased local anesthetic protein-binding decreases potential CNS toxicity

26. Which of the following statements is false?


A. Repeated attempts at aspiration would have prevented this complication
B. Addition of epinephrine to the local anesthetic may have helped to prevent this
complication
C. Loss of consciousness means that patient has developed cardiac arrest
D. Amiodarone is the first line of treatment for cardiovascular toxicity caused by bupivacaine

27. During induction of anesthesia for cesarean delivery in a 22-year-old female, rocuronium is
inadvertently substituted for succinylcholine. The neonate does not show any sign of muscle
relaxation because rocuronium is
A. Highly protein bound
B. “Unaffected by ion trapping”
C. Lipid soluble
D. Highly ionized

28. All of the following can lead to hyperkalemic response to the administration of
succinylcholine, except
A. Burn injury
B. Spinal cord injury
C. Prolonged ICU stay
D. Cerebral palsy

29. The dibucaine number in a patient having heterozygous type of plasma cholinesterase will be
A. 20% to 30%
B. 30% to 40%
C. 60% to 80%
D. 50% to 60%

30. Which of the following muscle relaxants is eliminated mostly by the kidneys?
A. Rocuronium
B. Succinylcholine
C. Vecuronium
D. Pancuronium

31. The correct recommended intubating dose among the following muscle relaxants is
A. Vecuronium 0.08 to 0.1 mg/kg
B. Pancuronium 0.05 to 0.07 mg/kg
C. Succinylcholine 0.5 to 0.07 mg/kg
D. Cisatracurium 0.5 to 0.8 mg/kg

32. Which of the following drugs is able to cross the blood-brain barrier?
A. Physostigmine
B. Neostigmine
C. Pyridostigmine
D. Glycopyrrolate

33. All of the following are side effects of anticholinesterase drugs, except
A. Excessive salivation
B. Increased bowel motility
C. Bradycardia
D. Bronchodilation

34. Which of the following characteristics of electrical stimulation is the correct representation of
the stimulus generated by the nerve stimulator used for monitoring the neuromuscular
blockade?
A. Tetany: A sustained stimulus of 50 to100 Hz, usually lasting 2 seconds
B. Twitch: A single pulse 0.5 second in duration
C. Train of four: A series of four twitches in 2 seconds (2-Hz frequency), each 0.2 ms long
D. Double-burst stimulation: Three short (0.2 ms) high-frequency stimulations separated by a
30-ms interval and followed 1 second later by two or three additional impulses

35. Which of the following antibiotics augments the action of nondepolarizing muscle relaxants?
A. Penicillin
B. Cephalosporin
C. Erythromycin
D. Streptomycin
36. Immediately after induction of general anesthesia for hip replacement surgery, a 56-year-old
patient with severe mitral stenosis and a normal ejection fraction develops a blood pressure of
70/35 mm Hg with a heart rate of 90 bpm. Which of the following is the most appropriate initial
treatment?
A. Dobutamine
B. Epinephrine
C. Phenylephrine
D. Milrinone

37. Mechanism of action of droperidol involves antagonism at all of the following receptors,
except
A. Serotonin
B. Dopamine
C. a-Adrenergic
D. Glutamate

38. Which of the following is not seen in acute cyanide poisoning?


A. Metabolic acidosis
B. Cardiac arrhythmias
C. Tolerance to the antihypertensive effect of nitroprusside
D. Decreased mixed venous oxygen saturation

39. Which of the following medications is associated with extrapyramidal effects?


A. Midazolam
B. Glycopyrrolate
C. Metoclopramide
D. Famotidine

40. Which of the following medications should be discontinued before the elective surgery?
A. Metoprolol
B. Monoamine oxidase inhibitors
C. Atorvastatin
D. Ranitidine

41. Administration of magnesium sulfate for preeclampsia results in a decreased dose requirement
for each of the following, except
A. Succinylcholine
B. Rocuronium
C. Desflurane
D. Lidocaine
42. B e n e fits o f e p in e p h r in e 1 :2 0 0 ,0 0 0 a d d e d to lid o c a in e f o r a n e p id u r a l in je c tio n in c lu d e a ll o f th e
f o ll o w i n g , e x c e p t

A. P r o l o n g a t i o n o f d u r a tio n o f a c tio n o f lid o c a in e


B. B e tte r q u a lity o f b lo c k
C. P r o p h y la c tic tr e a tm e n t o f h y p o te n s io n a s s o c ia te d w ith th e b o lu s a d m in is tr a tio n o f lid o c a in e
D. D e la y e d a b s o r p tio n in to s y s te m ic c ir c u la tio n , th e r e b y d e c r e a s in g p r o b a b ility o f lo c a l
a n e s th e tic to x ic ity

43. W h ic h o f th e f o ll o w i n g c h o ic e s is c o r r e c t r e g a r d i n g th e b l o o d g a s p a r titio n c o e f f ic ie n t?

A. N itr o u s o x id e 0 .4 7
B. D e s f lu r a n e 0 .6 2
C. I s o f lu r a n e 2.4
D. S e v o f lu r a n e 0 .8 5

44. T h e u s e o f n e o s tig m in e to r e v e r s e r e s id u a l n e u r o m u s c u la r b lo c k m a y s lo w th e m e ta b o lis m o f


w h ic h o f th e f o ll o w i n g d r u g s a d m in is te r e d s u b s e q u e n tly ?

A. R o c u r o n iu m
B. C is a tr a c u r iu m
C. P a n c u r o n iu m
D. S u c c in y lc h o lin e

45. A 4 5 - y e a r - o ld p a tie n t w ith h is to r y o f h y p e r tr o p h ic s u b a o r tic c a r d io m y o p a th y b e c o m e s


h y p o te n s iv e . W h ic h o f th e f o ll o w i n g d r u g s is most a p p r o p r ia te f o r tr e a tm e n t o f h y p o te n s io n ?

A. E p h e d r in e
B. A m r in o n e
C. P h e n y le p h r in e
D. N itr o g l y c e r in e

46. F a c to r s th a t c o n tr a in d ic a te k e to r o la c a d m in is tr a tio n in c lu d e a ll o f th e f o ll o w i n g e x c e p t

A. R e n a l in s u f f ic ie n c y
B. F a c to r V III d e f ic ie n c y
C. A c tiv e p e p tic u lc e r d is e a s e
D. D a ily in g e s t io n o f a s p ir in

47. A fte r r e c e iv in g m a s s iv e b l o o d tr a n s f u s io n , a p a tie n t a n e s th e tiz e d w ith is o f lu r a n e , fe n ta n y l, a n d


n itr o u s o x id e d e v e lo p s a c u te p u lm o n a r y e d e m a . T h e d r u g m o s t lik e ly to h e lp h im a c u te ly is

A. I s o f lu r a n e
B. N itr o g l y c e r in e
C. D ig o x in
D. M o r p h in e
48. A 22-year-old college athlete with a history of prolonged QT syndrome presents for an
inguinal hernia repair. Which of the following agents would be least likely to further lengthen
the QT interval?
A. Ondansetron
B. Metoclopramide
C. Succinylcholine
D. Propofol

49. Which of the following statements concerning naloxone is true?


A. Elimination half-life is longer than most of the ^-receptor opioids
B. It has mixed agonist-antagonist activity
C. It relieves opioid-induced spasm of the sphincter of Oddi
D. It does not cross the placenta

50. Which of the following drugs is most likely to cause tachycardia?


A. Fentanyl
B. Meperidine
C. Morphine
D. Sufentanil

51. Addition of fentanyl to epidural bupivacaine will cause


A. No change in duration of analgesia
B. More rapid onset of analgesia
C. Increased vagal activity
D. Increased sensory block

52. Compared with sufentanil, alfentanil is characterized by


A. Higher pKa
B. Larger unionized fraction at physiologic pH
C. Less protein-binding
D. Greater lipid solubility

53. An inhaled anesthetic has blood/gas partition coefficient of 14.8. Recovery time primarily
depends on
A. Oil/gas solubility of the agent
B. Cardiac output
C. Duration of administration
D. MAC of the drug

54. Nitroprusside therapy for hypertension should be discontinued in the presence of


A. Acute myocardial infarction
B. Increasing metabolic acidosis
C. Mitral regurgitation
D. Renal failure

55. A 24-year-old man is apprehensive of general anesthesia and prefers a regional anesthetic.
Decision is made to conduct spinal anesthesia for the repair of inguinal hernia along with
midazolam and fentanyl to allay anxiety. During the procedure, he suddenly loses
consciousness. There is profound hypotension with systolic blood pressure of 44 mm Hg and a
heart rate of 28 bpm. Cardiopulmonary resuscitation is started. The next most appropriate
intervention is administration of
A. Atropine
B. Ephedrine
C. Epinephrine
D. Flumazenil

56. The effect of gentamycin at the neuromuscular junction is


A. Prevented by pretreatment with magnesium
B. Potentiated by anticholinesterases
C. Decreased by depolarizing relaxants
D. Partially reversed by calcium

57. Compared with lorazepam (Ativan), midazolam (Versed)


A. Has a shorter elimination half-life
B. Has more rapid clearance
C. Has a larger volume of distribution
D. Undergoes slower hepatic metabolism

58. The drug that causes dose-dependent EEG evidence of both central nervous system excitation
and depression is
A. Thiopental
B. Lidocaine
C. Isoflurane
D. Midazolam

59. Normal pseudocholinesterase


A. Is produced primarily at nerve terminals
B. Is antagonized by acetyl cholinesterase
C. Resists dibucaine inhibition more than atypical pseudocholinesterase
D. Metabolizes succinylcholine by Hofmann elimination
60. Succinylcholine has prolonged action in patients carrying homozygous pseudocholinesterase.
Which of the following best explains this phenomenon?
A. Diffusion away from the neuromuscular junction is slow
B. Hepatic clearance of succinylcholine is reduced
C. Succinylmonocholine induces neuromuscular block
D. An increased proportion of succinylcholine reaches the neuromuscular junction

61. Opioid analgesics cause all of the following effects except


A. Contraction of smooth muscle of the gallbladder
B. Contraction of detrusor muscle of the urinary bladder
C. Depress cellular immunity
D. Delayed gastric emptying

62. Opioids may have more pronounced action in all of the following except
A. In men compared to women
B. In older than in younger patients
C. During liver transplant surgery
D. In kidney failure

63. Which of the following drugs decreases lower esophageal sphincter tone?
A. Succinylcholine
B. Glycopyrrolate
C. Metoclopramide
D. Neostigmine

64. A 28-year-old burn patient needs daily wound debridement. Which of the following agents is
not appropriate to provide a short duration of anesthesia?
A. Nitrous oxide
B. Ketamine
C. Etomidate
D. Midazolam

65. Eutectic mixture of local anesthetics (EMLA cream) is sometimes used to numb the skin before
attempting an intravenous access in pediatric patients. Which of the following local anesthetics
is combined with prilocaine to produce this cream?
A. Bupivacaine
B. Lidocaine
C. Mepivacaine
D. Ropivacaine

66. A 76-year-old man with history of hypertension and cancer of the colon had colectomy under
general anesthesia 24 hours ago. He is receiving an epidural infusion of fentanyl at the rate of
100 micro symbol g/h. Which of the following is least likely?
A. Nausea
B. Pruritus
C. Respiratory depression
D. Hypotension

67. Which of the following may help in mapping of a seizure focus under general anesthesia by
enhancing the EEG activity or inducing the seizure?
A. Thiopental
B. Ketamine
C. Diazepam
D. Isoflurane

68. Which of the following anesthetic agents is contraindicated for use in patients with intermittent
porphyria?
A. Ketamine
B. Etomidate
C. Isoflurane
D. Thiopental

69. Replacing 10 mg of morphine with 30 mg of ketorolac can increase the risk of


A. Respiratory depression
B. Analgesia
C. Nausea
D. Bleeding

70. The minimum anesthesia concentration (MAC) of desflurane is decreased by


A. Chronic alcohol use
B. Respiratory alkalosis
C. Chronic anemia with hemoglobin of 7.5 gm/dL
D. Hypothermia to 34°C

71. A 45-year-old woman has been using heroin for last 20 years. Use of which of the following
drugs will cause acute withdrawal symptoms?
A. Butorphanol
B. Nalbuphine
C. Buprenorphine
D. Naltrexone

72. Ketamine administered in anesthetic doses


A. Decreases intracranial pressure
B. Causes respiratory depression
C. Is metabolized by the liver
D. Increases bronchomotor tone

73. Which of the following drugs is the most appropriate agent for acute treatment of hypertension
in a preeclamptic patient?
A. Magnesium
B. Labetalol
C. Lisinopril
D. Nitroglycerine

74. Which of the following provides the best estimate of complete reversal of neuromuscular
blockade?
A. Double-burst ratio of 1
B. Train-of-four-ratio of 1
C. Absence of fade on tetanic stimulation at 50 Hz
D. Absence of fade of single twitch

75. Which of the following is contraindicated in a patient with Guillain-Barre syndrome?


A. Intrathecal opioids
B. Nondepolarizing muscle relaxant
C. Epidural local anesthetics
D. Succinylcholine

76. Which of the following drugs is the most appropriate for management of anesthesia in a patient
who needs emergency surgery and admits to using cocaine in last 3 hours?
A. Labetalol before induction
B. Ketamine for induction
C. Propofol for induction
D. Ephedrine for treatment of hypotension

77. During general anesthesia, which of the following agents is most appropriate to treat an acute
episode of cyanosis in a child with tetralogy of Fallot?
A. Atropine
B. Epinephrine
C. Phenylephrine
D. 100% oxygen

78. Rebound hypertension is most likely after sudden discontinuation of which of the following
classes of antihypertensive drugs?
A. Thiazide diuretics
B. Calcium channel blockers
C. a-Agonist
D. Angiotensin-converting enzyme inhibitors

79. A 65-year-old man has nausea and vomiting in the post-anesthesia care unit, needing antiemetic
therapy. He develops involuntary facial movements, difficulty swallowing, and torticollis.
Which of the following drugs is most likely to be the cause of these symptoms?
A. Promethazine (Phenergan)
B. Diphenhydramine (Benadryl)
C. Metoclopramide (Reglan)
D. Granisetron (Kytril)

80. Which of the following statements about ketamine is true?


A. Tolerance may develop after repeated administration
B. It is extensively bound to plasma protein
C. Primary site of action is GABA receptor
D. Kidney is the primary route of elimination

81. Which of the following statements about etomidate is most likely true?
A. It causes significant dose-dependent respiratory depression
B. It causes cerebral vasodilatation
C. It increases frequency of excitatory spikes on the EEG more than thiopental
D. Most of the administered dose is excreted unchanged by the kidney

82. The MOST likely analgesic mechanism of action of gabapentin for neuropathic pain is
A. Antagonism at the GABA receptor
B. NMDA receptor inhibition
C. Sodium channel blockade
D. Calcium channel modulation

83. Which of the following properties of local anesthetics is most likely a primary determinant of
potency?
A. Vasodilation
B. pKa
C. Protein-binding
D. Lipid solubility

84. Which of the following statements about etomidate is most likely true?
A. It is water soluble at an acidic pH and lipid soluble at physiologic pH
B. It may be used as an infusion for sedation in the ICU
C. It is related to propofol in its chemical structure
D. Awakening from induction dose is secondary to very rapid liver metabolism

85. A 64-year-old man is scheduled for an open abdominal aortic aneurysm surgery. Anesthetic
plan includes placement of an epidural catheter for postoperative pain relief. On review of his
medication list, it is noted that he has been taking clopidogrel for a coronary artery stent that
was inserted 2 years ago. Which of the following statements about clopidogrel is most likely
true?
A. The American Society for Regional Anesthesia recommends that clopidogrel be stopped 3
days before neuraxial anesthesia
B. A single dose of clopidogrel may have a clinically significant effect on platelet function
C. Clopidogrel is associated with pancytopenia
D. Inhibition of platelet function by clopidogrel is reversible

86. Which of the following statements about ketamine is most likely true?
A. Analgesic efficacy of epidural ketamine is equivalent to epidural morphine
B. Ketamine decreases the duration of action of nondepolarizing neuromuscular-blocking
drugs
C. Ketamine is a direct myocardial depressant
D. Ketamine decreases the cortical amplitude of somatosensory-evoked potentials

87. A patient has a history of an allergic reaction to a local anesthetic but does not recall the name.
Which of the following local anesthetics will most likely be the cause of a true allergic
reaction?
A. Procaine
B. Lidocaine
C. Mepivacaine
D. Bupivacaine

88. Which of the following is most likely the (analgesic) mechanism of action of lidocaine when
used for neuropathic pain?
A. Inhibition of G-protein-coupled receptors
B. Antagonism of NMDA receptors
C. Calcium channel blockade
D. Sodium channel blockade

89. A 45-year-old farmer is brought into the emergency room. He is agitated and confused. On
examination, he has dry skin with fever and rapid heart rate. Anticholinergic poisoning is
suspected. Which of the following medications is most appropriate to treat his condition?
A. Neostigmine
B. Pyridostigmine
C. Edrophonium
D. Physostigmine

90. Which of the following medications will prolong the neuromuscular blockade produced by
vecuronium?
A. Carbamazepine
B. Clindamycin
C. Quinidine
D. Verapamil

91. The shorter duration of action of remifentanil compared with fentanyl is primarily due to its
A. Rapid redistribution
B. Renal elimination
C. Metabolism by esterases
D. Hepatic extraction ratio

92. Which of the following statements about dexmedetomidine is most likely true?
A. It has more a 2 selectivity than clonidine
B. It can increase opioid-induced rigidity
C. Context-sensitive half time increases markedly after prolonged infusion of
dexmedeto midine
D. It has no effect on systemic vascular resistance

93. Which of the following can precipitate an episode of myotonia in a patient with myotonic
dystrophy?
A. Lidocaine administration
B. Neostigmine administration
C. Nondepolarizing neuromuscular-blocker administration
D. Hypothermia

94. A 50-year-old woman had cholecystectomy done under general anesthesia. Rocuronium was
used as muscle relaxant, and a combination of anticholinergic and anticholinesterase was used
for reversal of muscle-relaxant action. The patient is now bradycardic. The combination of
reversal agents most likely to cause the bradycardia is
A. Atropine and edrophonium
B. Glycopyrrolate and edrophonium
C. Atropine and neostigmine
D. Glycopyrrolate and neostigmine

95. A 68-year-old man is undergoing exploratory laparotomy for intestinal obstruction. Cause of
obstruction is found to be an ileal carcinoid tumor. Suddenly, the patient develops
bronchospasm, and the peak airway pressure increases from 24 to 45 cm of H2O. Which of the
f o ll o w i n g is th e b e s t tr e a tm e n t f o r th e b r o n c h o s p a s m in th is s itu a tio n ?

A. D e x a m e th a s o n e
B. S e v o f lu r a n e
C. K e ta m in e
D. S o m a to s ta tin

96. A 1 5 - y e a r - o ld b o y h a s s e v e r e g a s tr o e n te r itis w ith n a u s e a , v o m itin g , a n d d ia r r h e a f o r la s t 3


d a y s . A C T s c a n o f th e a b d o m e n s h o w s in tu s s u s c e p tio n s , w h ic h n e e d s e x p l o r a t o r y la p a r o to m y
f o r r e l i e f o f in te s tin a l o b s tr u c tio n . T h e p a tie n t’s s y s to lic b l o o d p r e s s u r e is 7 8 m m H g a n d h e a r t
r a te is 112 b p m . T h io p e n ta l is s e le c te d a s th e in d u c tio n a g e n t f o r g e n e r a l a n e s th e s ia . A
d e c r e a s e d d o s e o f th is a g e n t is r e c o m m e n d e d in p a tie n ts w ith h y p o v o le m ic s h o c k p r i m a r i l y
because

A. D e liv e r y o f th e d r u g to th e b r a i n is in c r e a s e d
B. H e p a tic c le a r a n c e is d e c r e a s e d
C. T h io p e n ta l is a m y o c a r d ia l d e p r e s s a n t
D. T h io p e n ta l is a v a s o d ila to r

97. A 7 5 - y e a r - o ld w o m a n is s c h e d u le d f o r m itr a l v a lv e r e p a ir . H ig h - d o s e fe n ta n y l is u s e d to in d u c e
a n e s th e s ia . In o r d e r to c o u n te r a c t th e b r a d y c a r d i a c a u s e d b y fe n ta n y l, p a n c u r o n iu m is
a d m in is te r e d . P a n c u r o n iu m b lo c k s th e b r a d y c a r d i a c a u s e d b y fe n ta n y l b y a c tin g o n w h ic h o f th e
f o ll o w i n g ?

A. ^ - A d r e n e r g ic r e c e p to r s
B. C a r d ia c m u s c a r in i c r e c e p to r s
C. C a r o tid b a r o r e c e p t o r s
D. C e n tra l v a g a l n u c le i

98. W h ic h o f th e f o ll o w i n g m e d ic a tio n s w o u ld b e most a p p r o p r ia te to tr e a t s y m p to m a tic


b r a d y c a r d i a 1 m o n th a fte r c a r d ia c tra n s p la n t?

A. G l y c o p y r r o la te
B. P h e n y le p h r in e
C. A tr o p in e
D. I s o p r o te r e n o l

99. W h ic h o f th e f o ll o w i n g s ta te m e n ts a b o u t p r o p o f o l in f u s io n s y n d r o m e is most lik e ly fa ls e ?

A. M o r ta lity r a te in a n e s ta b lis h e d c a s e is v e r y h ig h
B. R h a b d o m y o ly s is is o n e o f th e d ia g n o s tic c r ite r ia
C. T a c h y c a r d ia is a n e a r ly s ig n o f th is s y n d r o m e
D. C a r d ia c d y s f u n c tio n is v e r y c o m m o n in th is c o n d itio n

100. A p a tie n t is u n d e r g o in g r e s e c tio n o f a s u p r a te n to r ia l b r a i n tu m o r . H e is n o r m o c a r b ic , a n d h is


m e a n b l o o d p r e s s u r e is 70 m m H g . A d m in is tr a tio n o f w h ic h o f th e f o ll o w i n g is m o s t lik e ly to
d e c r e a s e c e r e b r a l b l o o d v o lu m e ?
A. N itr o u s o x id e a t 0 .5 m in im u m a lv e o la r c o n c e n tr a tio n (M A C )
B. D e s f lu r a n e a t 1 M A C
C. T h io p e n ta l 2 m g /k g
D. P h e n y to in 15 m g /k g

101. W h ic h o f th e f o ll o w i n g c la s s e s o f d r u g s is m o s t lik e ly to b e r e s p o n s ib le f o r a n a n a p h y la c tic


r e a c tio n d u r in g g e n e r a l a n e s th e s ia ?

A. N e u r o m u s c u la r - b l o c k in g d r u g s
B. O p io id s
C. A n tib io tic s
D. R a d io c o n tr a s t d y e s
CHAPTER 6 ANSWERS

1. D. C le a r a n c e o f d r u g s th a t a r e m a in ly m e ta b o liz e d in th e liv e r is a f u n c tio n o f th e a m o u n t o f


d r u g b r o u g h t in to th e liv e r b y its b l o o d f lo w m u ltip lie d b y a b ility o f th e h e p a to c y te s to c le a r th e
b l o o d o f th a t d r u g . F o r d r u g s th a t h a v e a h ig h e x tr a c tio n r a tio , th e liv e r r e m o v e s th e e n tir e d r u g
e n te r in g th e liv e r in o n e p a s s . L id o c a in e a n d p r o p r a n o l o l a r e e x a m p le s o f th is k in d o f c le a r a n c e .
A lfe n ta n il, o n th e o th e r h a n d , h a s a l o w e x tr a c tio n r a tio , a n d liv e r b l o o d f lo w d o e s n o t r e a l l y
a f f e c t its c le a r a n c e . In s te a d , it is th e in tr in s ic a b ility o f th e liv e r to c le a r th e b l o o d o f th is d r u g
th a t d e te r m in e s a lf e n ta n i l’s c le a r a n c e . C y to c h r o m e P 4 5 0 s y s te m c a n m e ta b o liz e a w id e v a r ie ty
o f d r u g s b y a s in g le g r o u p o f e n z y m e s .

2. D. M a n y p a tie n ts b e lie v e th a t th e y a r e a l l e r g i c to lo c a l a n e s th e tic . Q u e s tio n in g th e m a b o u t it is


im p o r ta n t; o th e r w is e , o p tio n s f o r s a f e d e liv e r y o f a n e s th e s ia c a n g e t c h a lle n g in g . D e n tists
u s u a lly a d d e p in e p h r in e to th e lo c a l a n e s th e tic to d e c r e a s e th e b le e d in g a s s o c ia te d w ith th e
d e n ta l p r o c e d u r e . If a n y e p in e p h r in e g e ts a c c e s s to th e v a s c u la r s y s te m , it c a n c a u s e tr a n s ie n t
ta c h y c a r d ia a n d h y p e r te n s io n th a t p a tie n t m a y d e s c r ib e a s p a lp ita tio n s , f lu s h in g , a n d d iz z in e s s . If
la b e le d a s a n a l l e r g i c r e a c tio n , it m a y l im it a n e s th e tic o p tio n s f o r th e p a tie n t in th e fu tu r e . In a n
e m e r g e n t s itu a tio n w h e r e a s p in a l a n e s th e tic c o u ld h a v e b e e n p o s s ib le , o n e m a y h a v e to u tiliz e
g e n e r a l a n e s th e s ia a n d r i s k a ir w a y c o m p lic a tio n s . It is im p o r ta n t to e lu c id a te th e r e a l a lle r g ic
r e a c tio n f o r a n e le c tiv e c a s e b y s u b je c tin g th e p a tie n t to a l l e r g y te s tin g . S e r u m te s tin g is
a v a ila b le . S k in te s tin g is n o t in d ic a te d b e c a u s e o f th e r is k s in v o lv e d . M o s t o f th e a lle r g ic
r e a c tio n s o b s e r v e d w ith lo c a l a n e s th e tic s a r e n o t d u e to th e lo c a l a n e s th e tic m o le c u le b u t e ith e r
to p a r a - a m in o b e n z o ic a c id , a m e ta b o lite o f e s te r lo c a l a n e s th e tic s , o r to m e th y lp a r a b e n a n d
m e ta b is u lp h ite , w h ic h a r e b o th p r e s e r v a tiv e s . T r u e ty p e 1 a l l e r g i c r e a c tio n w ith lo c a l a n e s th e tic
is e x tr e m e ly r a r e b u t w ill p r e s e n t a s a n a p h y la x is w ith h y p o te n s io n a n d r e s p i r a t o r y s y m p to m s .
V a s o v a g a l r e s p o n s e u s u a lly m a n if e s ts a s p a le s k in w ith v e r y l o w h e a r t r a te a n d b l o o d p r e s s u r e .
A lth o u g h th is p a tie n t d id h a v e lig h t- h e a d e d n e s s , s h e a ls o h a d f lu s h in g a n d p a lp ita tio n w h ic h is
n o t c o n s is te n t w ith v a s o v a g a l r e a c tio n .

3. D. E f f e c t o f s y m p a th o m im e tic d r u g s o n th e m e a n a r te r ia l b l o o d p r e s s u r e is m e d ia te d th r o u g h
th e ir e f f e c t o n th e a d r e n e r g i c r e c e p to r s th e y s tim u la te . U ltim a te e f f e c t is g e n e r a te d th r o u g h
c o m p le x in te r a c tio n o f d if f e r e n t f a c to r s b a s e d o n b a s e lin e s y m p a th e tic to n e , p a tie n t’s v o lu m e
s ta tu s, a n d c o n d itio n o f th e h e a r t. A lth o u g h i s o p r o t e r e n o l in c r e a s e s c o n tr a c tility th r o u g h its
a c tio n o n P2 r e c e p to r s , it a ls o s tim u la te s P! r e c e p to r s in b ig v a s c u la r b e d s lik e m u s c le , c a u s in g
v a s o d ila ta tio n a n d d e c r e a s in g th e s y s te m ic v a s c u la r r e s is ta n c e . S o d e s p ite in c r e a s in g
c o n tr a c tility o f th e h e a r t a n d in c r e a s in g th e c a r d ia c o u tp u t, th e m e a n a r te r ia l b l o o d p r e s s u r e c a n
d e c r e a s e w ith a d m in is tr a tio n o f th is d r u g . O th e r m e d ic a tio n s in c r e a s e th e m e a n a r te r ia l b lo o d
p r e s s u r e b y th e ir e f f e c t o n a ! a n d P! r e c e p to r s .

4. D. A ll o f th e s e c a te c h o la m in e s c a u s e s tim u la tio n o f P! r e c e p to r s , th u s in c r e a s in g c a r d ia c
c o n tra c tility , b u t n o r e p in e p h r in e a ls o h a s a v e r y s tr o n g e f f e c t o n a ! r e c e p to r s , th u s in c r e a s in g
a f te r lo a d to s u c h a d e g r e e th a t c a r d ia c o u tp u t m a y a c tu a lly d e c r e a s e a fte r a d m in is tr a tio n o f th is
d ru g .

C a r d ia c o u tp u t = S y s te m ic b l o o d p r e s s u r e /S y s te m ic v a s c u la r r e s is ta n c e

A c c o r d in g to th is f o r m u la , s y s te m ic v a s c u la r r e s is ta n c e (S V R ) is i n v e r s e ly r e la te d to c a r d ia c
o u tp u t; th u s , a n in c r e a s e in th e S V R m a y d e c r e a s e th e c a r d ia c o u tp u t. O th e r d r u g s h a v e m o r e
e f f e c t o n c a r d ia c c o n tr a c tility th a n o n S V R , a n d th u s , c a r d ia c o u tp u t in c r e a s e s w ith th e ir
a d m in is tr a tio n .

5. B. E f f e c t o f n o r e p in e p h r in e is o n a ! r e c e p to r s a n d s y s te m ic v a s c u la r r e s is ta n c e , a n d th u s o n
s y s te m ic m e a n a r te r ia l b l o o d p r e s s u r e m a y b e so p r o n o u n c e d th a t th e r e m a y b e a d e c r e a s e in
h e a r t r a te s e c o n d a r y to b a r o r e c e p t o r r e s p o n s e . It d o e s in c r e a s e th e l i k e lih o o d o f c a r d ia c
d y s r h y th m ia s in th e p r e s e n c e o f s o m e o ld e r a n e s th e tic s lik e h a lo th a n e a s w e ll a s in h y p o x ia a n d
h y p e r c a r b ia .

6. B. S tim u la tio n o f a 2 r e c e p to r s c a u s e s in h ib itio n o f r e le a s e o f n o r e p in e p h r in e , th u s d e c r e a s in g


th e a c tiv ity o f th e s y m p a th e tic n e r v o u s s y s te m . H y p o te n s io n a n d b r a d y c a r d i a a r e s id e e ffe c ts th a t
s o m e tim e s l im it th e u s e o f m e d ic a tio n s lik e c lo n id in e a n d d e x m e d e to m id in e . In th e p a st, th e s e
d r u g s w e r e u s e d m a in ly a s a n tih y p e r te n s iv e s , b u t a p p lic a tio n s b a s e d o n th e ir s e d a tiv e ,
a n x io ly tic , a n d a n a lg e s ic p r o p e r tie s a r e b e c o m in g i n c r e a s in g ly c o m m o n . D r y m o u th m a y a ls o
r e s u lt f r o m th e u s e o f d e x m e d e to m id in e .

7. C. L a b e ta lo l is a c o m p e titiv e a n ta g o n is t a t th e a ! a n d P a d r e n e r g i c r e c e p to r s . It is a u s e f u l
a g e n t in th e p e r io p e r a tiv e p e r i o d b e c a u s e v a s o d ila ta tio n c a u s e d b y a ! b lo c k a d e is n o t
a c c o m p a n ie d w ith ta c h y c a r d ia w ith its a tte n d a n t r is k s . It is a p a r tic u la r ly u s e f u l d r u g in
h y p e r te n s iv e p a tie n t w ith d ia g n o s is o f a o r tic d is s e c tio n a s it d e c r e a s e s th e s h e e r f o r c e a c r o s s th e
d is s e c tio n . It d o e s n o t c r o s s th e p la c e n ta a n d d o e s n o t d e c r e a s e th e u te r in e b l o o d f lo w e v e n w h e n
p a tie n t is h y p o te n s iv e , so it is u s e d in o b s te tr ic p a tie n ts w ith p r e e c la m p s ia to c o n tr o l th e ir b lo o d
p r e s s u r e . C lo n id in e is a s tim u la n t o f a 2 r e c e p to r s , th u s d e c r e a s in g s y m p a th e tic a c tiv ity , a n d its
lo n g - t e r m a d m in is tr a tio n le a d s to u p r e g u la tio n o f a d r e n e r g i c r e c e p to r s . S u d d e n w ith d r a w a l o f
th is m e d ic a tio n le a d s to o v e r a c tiv ity o f th e s y m p a th e tic s y s te m a n d a P - b lo c k e r a n ta g o n is t is
v e r y h e lp f u l in c o n tr o llin g th e m a n if e s ta tio n s o f th is o v e ra c tiv ity . A b n o r m a litie s o f c a r d ia c
c o n d u c tio n s y s te m a r e a r e la tiv e c o n tr a in d ic a tio n to th e a d m in is tr a tio n o f la b e ta lo l a s it m a y
w o r s e n th e d e g r e e o f c o n d u c tio n b lo c k a d e .

8. D. A lth o u g h a ll o f th e s e m e d ic a tio n s m a y b e u s e f u l in th e e v e n t o f a n a p h y la x is d u r in g a
g e n e r a l a n e s th e tic , e p in e p h r in e is c o n s id e r e d to b e th e d r u g o f c h o ic e a n d is in d ic a te d a s th e
f i r s t lin e o f tre a tm e n t. D o s e d e p e n d s o n th e s e v e r ity o f th e r e a c tio n a n d m a y b e a n y w h e r e f r o m
10 |ig to 1 m g if c a r d ia c a r r e s t d e v e lo p s . Its a ! a c tio n c o u n te r a c ts th e s e v e r e v a s o d ila ta tio n ,
w h ic h is th e h a llm a r k o f th is c o n d itio n . Its P2 a c tio n h e lp s tr e a t b r o n c h o c o n s tr ic ti o n w h ile P1
a c tio n h e lp s s u p p o r t th e c a r d ia c o u tp u t. H 1- a n d H 2- b lo c k e r s a r e a ls o in d ic a te d in th e tr e a tm e n t
o f a n a p h y la x is a n d h e lp m itig a te th e e ffe c ts o f ty p e 1 a n tig e n a n tib o d y r e a c tio n o n m a s t c e lls
a n d o th e r m e d ia to r s , c a u s in g v a s c u la r d ila ta tio n a n d in c r e a s e d c a p il la r y p e r m e a b ility , b u t th e ir
a c tio n s a r e n e ith e r a s r a p id n o r a s p r o f o u n d a s e p in e p h r in e . S te r o id s s ta b iliz e th e c e ll
m e m b r a n e s o f m a s t c e lls a n d e o s in o p h ils , th u s d e c r e a s in g th e in te n s ity o f th e im m u n o lo g i c
r e s p o n s e , b u t th e ir a c tio n ta k e s 4 to 6 h o u r s to d e v e lo p , a n d th e r e is little e v id e n c e to s u p p o r t th e
u s e o f s te r o id s f o r th e a c u te tr e a tm e n t o f a n a p h y la x is .

9. C. E to m id a te a c tu a lly c a u s e s a d e c r e a s e in th e s e iz u r e th r e s h o ld , a n d is th u s u s e f u l in c a s e s o f
e le c tr o c o n v u ls iv e th e r a p y f o r s e v e r e d e p r e s s io n . S p o n ta n e o u s m o v e m e n ts c h a r a c te r iz e d as
m y o c lo n u s o c c u r in m o r e th a n 5 0 % o f p a tie n ts r e c e iv in g e to m id a te a n d m a y b e a s s o c ia te d w ith
s e iz u r e - lik e a c tiv ity o n th e E E G . A n o th e r s id e e f f e c t is a n in c r e a s e d in c id e n c e o f n a u s e a a n d
v o m itin g in th e p o s to p e r a tiv e p e r io d . It d o e s n o t c a u s e h is ta m in e r e le a s e in c o n tr a s t to
th io p e n ta l, w h ic h h a s b e e n s h o w n to r e le a s e h is ta m in e in v itr o .

10. C. K e ta m in e d if f e r s f r o m o th e r in d u c tio n a g e n ts u s e d in c o n te m p o r a r y p r a c tic e o f a n e s th e s ia


in m a n y im p o r ta n t w a y s . In s te a d o f a c tin g d ir e c tly o n th e r e tic u la r - a c tiv a tin g s y s te m (R E S ), it
c a u s e s d is s o c ia tio n o f th a la m u s (w h ic h r e la y s s e n s o r y i n f o r m a t io n f r o m th e R E S to c e r e b r a l
c o r te x ) f r o m th e lim b ic c o r te x (w h ic h is in v o lv e d w ith th e a w a r e n e s s o f s e n s a tio n ) . In s h a r p
c o n tr a s t to o th e r a n e s th e tic a g e n ts , it in c r e a s e s b l o o d p r e s s u r e , h e a r t ra te , a n d c a r d ia c o u tp u t b y
c e n tr a l s tim u la tio n o f s y m p a th e tic s y s te m a n d in h ib itio n o f u p ta k e o f n o r e p in e p h r in e . In th e
s a m e v e in , its e f f e c t o n th e v e n tila to r d r iv e a n d a ir w a y r e f le x e s is m in im a l, if an y . It is a n
e x c e lle n t b r o n c h o d ila to r a n d a n a lg e s ic . T h io p e n ta l, p r o p o f o l , a n d e to m id a te , o n th e o th e r h a n d ,
m a y b e a n ti- a n a lg e s ic .

11. B. P h e n e lz in e in h ib its m o n o a m in e o x id a s e , a n e n z y m e th a t m e ta b o liz e s c a te c h o la m in e s ,


a llo w in g th e ir le v e ls to b u ild u p in th e a d r e n e r g i c n e u r o n s . E p h e d r in e h a s b o th d ir e c t a n d
in d ir e c t a c tio n s o n a d r e n e r g i c s y s te m . I n d ir e c t a c tio n in v o lv e s r e le a s e o f e x a g g e r a te d a m o u n ts
o f n o r e p in e p h r in e f r o m th e a d r e n e r g i c n e u r o n s le a d in g to c a ta s tr o p h ic in c r e a s e in b lo o d
p r e s s u r e . M o r e d ir e c t- a c tin g m e d ic a tio n lik e p h e n y le p h r in e is a b e tte r c h o ic e in a p a tie n t u s in g
m o n o a m in e o x id a s e in h ib ito r s .

12. A. F lu m a z e n il is u s e f u l a s a s p e c if ic r e v e r s a l a g e n t f o r b e n z o d ia z e p in e o v e r d o s e . It h a s
m in im a l in tr in s ic a c tiv ity o n th is r e c e p to r , b u t b e c a u s e o f s im ila r ity in th e c h e m ic a l s tr u c tu r e , it
a c ts a s a c o m p e titiv e a n ta g o n is t in th e p r e s e n c e o f a g o n is t a t th e r e c e p to r s ite . D u r a tio n o f a c tio n
is s h o r t ( 6 0 - 9 0 m in u te s ), so r e p e a te d d o s e s o r in f u s io n is r e q u i r e d if r e c u r r e n c e o f s e d a tio n is
d e s ir e d . R e v e r s a l o f b e n z o d ia z e p in e a c tio n d o e s n o t le a d to c a r d io v a s c u la r s id e e ffe c ts o r
e v id e n c e o f a c u te s tr e s s r e s p o n s e . It d o e s n o t h a v e a n y e f f e c t o n o p io id r e c e p to r , so is n o t u s e f u l
to r e v e r s e r e s p i r a t o r y d e p r e s s io n c a u s e d b y n a r c o tic o v e r d o s e .

13. C. A lth o u g h m o s t c o m m o n r o u te o f a d m in is tr a tio n o f m id a z o la m in a n e s th e s ia is in tr a v e n o u s ,


th is d r u g c a n b e g iv e n v ia m a n y r o u te s . M id a z o la m o r a l s u s p e n s io n is u s e d r o u ti n e ly in
p e d ia tr ic a n e s th e s ia . I n tr a m u s c u la r in je c tio n c a n b e p a in f u l, b u t in tr a n a s a l r o u te u tiliz in g
m u c o s a l a to m iz a tio n d e v ic e m a y b e u s e f u l to tr e a t s e iz u r e a c tiv ity . B io a v a ila b ility o f s u b lin g u a l
m id a z o la m is m u c h b e tte r th a n o r a l l y a d m in is te r e d d r u g . E x p o s u r e o f th e a c id ic m id a z o la m
p r e p a r a t io n to th e p h y s io lo g ic p H o f b l o o d c a u s e s a c h a n g e in th e r i n g s tr u c tu r e th a t r e n d e r s th e
d r u g m o r e lip id s o lu b le , th u s s p e e d in g its p a s s a g e a c r o s s th e b l o o d - b r a i n b a r r i e r . T h e r e is n o
p r e p a r a t io n a v a ila b le f o r tr a n s c u ta n e o u s d e liv e r y o f th is d r u g in c o m p a r is o n to fe n ta n y l.

14. A. S in c e lo c a l a n e s th e tic s a r e w e a k b a s e s , th e y e x is t l a r g e l y in th e io n ic f o r m , m a k in g it
d if f ic u lt f o r th e m to c r o s s th e c e ll m e m b r a n e , th u s d e la y in g th e ir lo c a l a n e s th e tic a c tio n .
A d d itio n o f s o d iu m b ic a r b o n a te p r o m o t e s n o n io n iz e d f r a c tio n o f lo c a l a n e s th e tic , p r o m o t in g
m o r e r a p id o n s e t o f its a c tio n . D e p e n d in g o n th e a m o u n t in je c te d , th e lo c a l p H m a y in c r e a s e , b u t
e ffe c t o n th e in tr a c e l lu la r p H is m in im a l, if an y .

15. B. C o c a in e in h ib its r e u p ta k e o f c a te c h o la m in e s in to p r e g a n g l i o n i c n e r v e te r m in a l; it h a s a
s y m p a th o m im e tic e ffe c t, c a u s in g ta c h y c a r d ia , h y p e r te n s io n , p u p illa r y d ila ta tio n , a n d in c r e a s e d
s k in te m p e r a tu r e . C h r o n ic u s e m a y le a d to c a r d io m y o p a th y a n d d e p le tio n o f c a te c h o la m in e
s to r e s w ith u n p r e d ic ta b le m a n if e s ta tio n s d u r in g a n e s th e s ia .

16. D. T h e r e a r e im p o r ta n t d if f e r e n c e s b e tw e e n th e tw o c la s s e s o f lo c a l a n e s th e tic s . E s te r lo c a l
a n e s th e tic s a r e m e ta b o liz e d b y p la s m a c h o lin e s te r a s e , a n d so p a tie n ts w ith a ty p ic a l v a r ie ty o f
e n z y m e is lia b le to d e v e lo p lo c a l a n e s th e tic to x ic ity b e c a u s e o f s lo w m e ta b o lis m . T h e y a r e
m e ta b o liz e d to p a r a - a m in o b e n z o ic a c id (P A B A ), a n d in d iv id u a ls k n o w n to h a v e a l l e r g y to th is
s u b s ta n c e s h o u ld n o t b e g iv e n e s te r k in d o f lo c a l a n e s th e tic . C o m m e r c ia l m u ltid o s e
p r e p a r a t io n s o f a m id e s o f te n c o n ta in m e th y lp a r a b e n , w h ic h h a s a c h e m ic a l s tr u c tu r e s im ila r to
th a t o f P A B A . T h is p r e s e r v a tiv e m a y b e r e s p o n s ib le f o r m o s t o f th e r a r e a l l e r g i c r e s p o n s e s to
a m id e a g e n ts . A m id e lo c a l a n e s th e tic s a r e m e ta b o liz e d b y liv e r , a n d d e c r e a s e d liv e r b l o o d f lo w
o r h is to r y o f liv e r f a ilu r e m a y le a d to to x ic ity e v e n w h e n le s s - th a n - m a x im u m - r e c o m m e n d e d
d o s e is u s e d . A ll o f th e lo c a l a n e s th e tic s in c lu d e d in th is q u e s tio n h a v e a m id e s tr u c tu r e in th e ir
m o le c u le , e x c e p t c o c a in e .

17. D. R o p iv a c a in e is le s s lip id s o lu b le , a n d th u s le s s p o te n t th a n b u p iv a c a in e . F o r a g iv e n d o s e ,
th e s e n s o r y b lo c k is m o r e th a n th e m o to r b lo c k . P a r t o f th e r e a s o n r o p iv a c a in e m a y b e le s s
c a r d io to x ic th a n b u p iv a c a in e is th a t it c a u s e s v a s o c o n s tr ic tio n in th e tis s u e s , th u s d e c r e a s in g th e
r a te o f a b s o r p tio n in to s y s te m ic c ir c u la tio n .
E n a n tio m e r s a r e s t e r e o is o m e r s th a t e x is t a s m i r r o r im a g e s . E n a n tio m e r s h a v e id e n tic a l
p h y s ic a l p r o p e r tie s e x c e p t f o r th e d ir e c tio n o f th e r o ta tio n o f th e p la n e o f th e p o la r iz e d lig h t.
R o p iv a c a in e is a n S - e n a n tio m e r o f m e p iv a c a in e a n d b u p iv a c a in e .

18. C. O x y g e n d e m a n d o f th e m y o c a r d iu m in c r e a s e s w ith in c r e a s e in h e a r t r a te a n d b l o o d
p r e s s u r e . E s m o lo l w ill d e c r e a s e b o th th e h e a r t r a te a n d b l o o d p r e s s u r e r a p id ly , th u s d e c r e a s in g
th e o x y g e n d e m a n d . H y d r a la z in e w ill d e c r e a s e th e b l o o d p r e s s u r e b u t m a y m a k e th e ta c h y c a r d ia
w o r s e . S a m e is tr u e f o r n itr o p r u s s id e . S h iv e r in g in c r e a s e s o x y g e n d e m a n d a n d n e e d s to b e
tr e a te d p r o m p tly . A p p lic a tio n o f w a r m in g b la n k e t m a y b e e f f e c tiv e if p a tie n t is h y p o th e r m ic , b u t
it m a y ta k e a w h ile f o r th e b o d y te m p e r a tu r e to im p r o v e . It m a y n o t b e e f f e c tiv e in th is p a tie n t if
h is te m p e r a tu r e is n o r m a l a n d if th e m e c h a n is m o f s h iv e r in g in v o lv e s in h ib itio n o f
t h e r m o r e g u l a t o r y m e c h a n is m s o f th e b o d y b y th e r e s id u a l a n e s th e tic .
E s m o lo l is a s h o r t- a c tin g P - b lo c k e r th a t w ill d e c r e a s e th e h e a r t r a te a n d m y o c a r d ia l
c o n tr a c tility th r o u g h its in h ib itio n o f ^ r e c e p to r s , th u s d e c r e a s in g th e m y o c a r d ia l o x y g e n
d e m a n d . I m p r o v e d h e a r t r a te w ill a ls o h e lp w ith im p r o v e d s u p p ly o f o x y g e n to th e m y o c a r d iu m
b y in c r e a s in g th e d ia s to lic tim e d u r in g w h ic h m o s t o f th e le f t- v e n tr ic u la r m y o c a r d iu m g e ts its
oxygen.

19. C. L o c a l a n e s th e tic s e x e r t th e ir e le c tr o p h y s io lo g ic e ffe c ts b y b lo c k in g s o d iu m io n


c o n d u c ta n c e . T h is e f f e c t is p r i m a r i l y m e d ia te d b y in te r a c tio n w ith s p e c if ic r e c e p to r s th a t a r e
w ith in th e in n e r v e s tib u le o f th e s o d iu m i o n c h a n n e l.

20. D. L o c a l a n e s th e tic s d e c r e a s e th e r a te o f d e p o la r iz a tio n o f th e c e ll m e m b r a n e w h e n a n e r v e


im p u ls e a r r iv e s a n d c h a n g e s th e r e s tin g m e m b r a n e p o te n tia l o f th e n e u r o n . N o r m a lly , th is
p o te n tia l is - 9 0 m V in s id e th e c e ll. W h e n a n im p u ls e r e a c h e s th e c e ll, it in c r e a s e s th e in f l o w o f
s o d iu m in to th e c e ll, c a u s in g th is p o te n tia l to m o v e to w a r d a p o s itiv e v a lu e . If th is c h a n g e is
e n o u g h to r e a c h a c r itic a l le v e l c a lle d th e th r e s h o ld p o te n tia l, a n a c tio n p o te n tia l is g e n e r a te d ;
o th e r w is e , th e im p u ls e d ie s d o w n . L o c a l a n e s th e tic s d e c r e a s e th e r a te o f c h a n g e o f th is p o te n tia l
so th a t it d o e s n o t r e a c h th e th r e s h o ld le v e l. T h e y d o n o t c h a n g e th e th r e s h o ld p o te n tia l o r
r e s tin g m e m b r a n e p o te n tia l p e r se.

21. D. L ip id s o lu b ility o f a lo c a l a n e s th e tic is d ir e c tly p r o p o r t i o n a l to its p o te n c y . It is o r d i n a r i l y


e x p r e s s e d a s a p a r titio n c o e f f ic ie n t, w h ic h is d e te r m in e d b y c o m p a r in g th e s o lu b ility in a q u e o u s
p h a s e , g e n e r a lly w a te r o r b u f f e r e d s o lu tio n . S in c e th e s ite o f a c tio n o f th e a n e s th e tic m o le c u le is
in s id e th e n e r v e c e ll, h ig h e r th e p r o p o r t i o n o f n e u tr a l b a s e o r th e u n io n iz e d f o r m , h ig h e r th e
lip id s o lu b ility a n d h ig h e r th e p o te n c y . I o n iz a tio n m a k e s it m o r e r e s is ta n t to e n te r th e c e ll a n d
d e c r e a s e s its p o te n c y .
It is im p o r ta n t to a p p r e c ia te th a t m e a s u r e s o f a n e s th e tic a c tiv ity m a y b e a ffe c te d b y th e in
v itr o a n d in v iv o s y s te m in w h ic h th e s e e ffe c ts a r e d e te r m in e d . F o r e x a m p le , te tr a c a in e is 2 0
tim e s m o r e p o te n t th a n b u p iv a c a in e w h e n s tu d ie d in is o la te d n e r v e tis s u e b u t h a s e q u iv a le n t
p o te n c y c o m p a r e d w ith b u p iv a c a in e w h e n te s te d in in ta c t in v iv o s y s te m s . T h e e f f e c t a c tu a lly
m a y v a r y e v e n in d if f e r e n t a r e a s o f th e b o d y lik e e p id u r a l s p a c e v s . p e r ip h e r a l n e r v e b lo c k
b e c a u s e o f s e c o n d a r y e ffe c ts s u c h a s th e in h e r e n t v a s o a c tiv e p r o p e r tie s o f th e a n e s th e tic .

22. A. I n f iltr a tio n o f lo c a l a n e s th e tic in p e r ip h e r a l tis s u e s is d if f e r e n t f r o m in je c tio n in to a n e r v e


s h e a th f o r a p e r ip h e r a l n e r v e b lo c k . D u r a tio n o f a n e s th e s ia a fte r in f i ltr a tio n is m u c h le s s th a n
a fte r a n e r v e b lo c k . A n e s th e s ia a fte r r o p iv a c a in e a n d b u p iv a c a in e la s ts 4 to 8 h o u r s , w h ile
lid o c a in e is e f f e c tiv e o n ly f r o m 1 to 2 h o u r s . M e p iv a c a in e is in b e tw e e n w ith d u r a tio n o f a c tio n
f r o m 9 0 to 1 8 0 m in u te s .

23. D. U s e o f m a n y lo c a l a n e s th e tic s f o r s p in a l a n e s th e s ia is s till e v o lv in g . L id o c a in e w a s


in tr o d u c e d in to c lin ic a l p r a c tic e in 1 9 4 6 . It w a s c o m m o n ly u s e d f o r s p in a l a n e s th e s ia u n til
r e p o r ts o f tr a n s ie n t n e u r o l o g i c s y m p to m s (T N S ) s ta r te d a p p e a r in g in th e lite r a tu r e . In c id e n c e o f
T N S is r e la tiv e ly h ig h w ith u p to o n e - th ir d o f p a tie n ts c o m p la in in g o f p a in a n d d y s e s th e s ia 12 to
2 4 h o u r s a fte r th e s u r g e r y . A lth o u g h s y m p to m s a r e tra n s ie n t, p a in s o m e tim e s is so s e v e r e th a t it
m a y e x c e e d th e p a in o f s u r g e r y a n d m a y n e c e s s ita te r e a d m is s io n in to th e h o s p ita l. T h is is s u e
h a s r a is e d q u e s tio n s r e g a r d i n g th e a d v is a b ility o f c o n tin u e d u s e o f lid o c a in e f o r s p in a l
a n e s th e s ia .

24. A. If c e n tr a l n e r v o u s s y s te m to x ic ity w a s to h a p p e n f r o m s lo w a b s o r p tio n o f lo c a l a n e s th e tic


o r in je c tio n in to a v e in , s y m p to m s a r e u s u a lly m ild e r to b e g in a n d e s c a la te f in a lly to th e s e iz u r e
le v e l. E a r ly s y m p to m s o f lo c a l a n e s th e tic to x ic ity m a y m a n if e s t a s c i r c u m o r a l n u m b n e s s ,
m e ta llic ta ste in th e m o u th , a n d tr e m o r s . A s th e s e r u m le v e ls o f th e lo c a l a n e s th e tic in c r e a s e
fu r th e r , s e iz u r e m a y r e s u lt d u e to e x c ita tio n o f s o m e f o c u s in th e c e n tr a l n e r v o u s s y s te m .
F u r th e r in c r e a s e in th e s e le v e ls le a d s to d e p r e s s io n o f th e n e r v o u s s y s te m m a n if e s tin g as
l e th a r g y a n d c o m a .
D ir e c t in je c tio n in to a n e r v e u s u a lly c a u s e s a s h o o tin g p a in a lo n g th e le n g th o f th e n e r v e
w ith in v o lu n ta r y w ith d r a w a l o f th e lim b in a n a w a k e p a tie n t, n o t a s e iz u r e . It is r e c o m m e n d e d to
a v o id p e r f o r m a n c e o f a n e r v e b lo c k u n d e r g e n e r a l a n e s th e s ia so th a t p a tie n t c o u ld p o in t to th is,
a n d th e r e b y a v o id a n c e o f n e r v e in ju r y . T r e a tm e n t is s u p p o r tiv e . S p e c ific tr e a tm e n t f o r th e
s e iz u r e is e ith e r b e n z o d ia z e p in e o r b a r b itu r a te s th a t in c r e a s e th e s e iz u r e th r e s h o ld . It is
im p o r ta n t to c o n tr o l th e s e iz u r e b e c a u s e in te n s e m u s c u la r a c tiv ity in c r e a s e s o x y g e n u tiliz a tio n
a n d c a r b o n d io x id e p r o d u c tio n , c a u s in g b o th m e ta b o lic a s w e ll a s r e s p i r a t o r y a c id o s is . If th e
d o s e o f b u p iv a c a in e th a t g a in e d a c c e s s to v a s c u la r s y s te m is h ig h , th e m a jo r d a n g e r is c a r d io
to x ic ity th a t is m a d e m u c h w o r s e a n d is e x tr e m e ly d if f ic u lt to tr e a t in th e p r e s e n c e o f a c id o s is .

25. D. It is r e c o m m e n d e d th a t th e n e e d le u s e d f o r in te r s c a le n e b lo c k s h o u ld b e o f a p p r o p r ia te
le n g th f o r th e g iv e n p a tie n t. N e r v e r o o t s in th is r e g i o n a r e v e r y s u p e r f ic ia l a n d if a l o n g e r
n e e d le is u s e d , a m e d ia lly d ir e c te d n e e d le m a y e n d u p in e ith e r e p id u r a l o r in tr a th e c a l s p a c e
le a d in g to h ig h e p id u r a l o r in tr a th e c a l b lo c k r e s p e c tiv e ly . T h is u s u a lly p r e s e n ts a s c a ta s tr o p h ic
h y p o te n s io n , r e s p i r a t o r y d is tr e s s f o ll o w e d b y c a r d io v a s c u la r c o lla p s e d e p e n d in g o n th e d o s e
in je c te d . S e iz u r e is n o t a p r e s e n ta tio n o f h ig h e p id u r a l o r in tr a th e c a l in je c tio n . H ig h e r p r o t e i n ­
b in d in g d e c r e a s e s th e c h a n c e o f n e u r o to x ic ity b y d e c r e a s in g th e f r e e f r a c tio n o f th e d r u g
a v a ila b le f o r a b s o r p tio n in to th e c ir c u la tio n .

26. D. F r e q u e n t a tte m p ts a t a s p ir a ti o n a r e r e c o m m e n d e d w h e n e v e r l a r g e d o s e a n d v o lu m e o f lo c a l
a n e s th e tic is in je c te d f o r e p id u r a l o r p e r ip h e r a l n e r v e b lo c k to a v o id in je c tin g th e lo c a l
a n e s th e tic in th e v a s c u la r o r in tr a th e c a l s p a c e . C a s e s o f lo c a l a n e s th e tic to x ic ity h a v e b e e n
r e p o r te d e v e n w h e n a s p ir a ti o n w a s n e g a tiv e f o r b l o o d o r C SF.
A d d itio n o f a d ilu te c o n c e n tr a tio n o f e p in e p h r in e to lo c a l a n e s th e tic is h e lp f u l a s it m a y a le r t
th e p r a c titio n e r to in a d v e r te n t v a s c u la r in je c tio n o f th e lo c a l a n e s th e tic b y in c r e a s e in th e h e a r t
ra te . It a ls o h e lp s d e la y th e s y s te m ic a b s o r p tio n o f lo c a l a n e s th e tic , th u s k e e p in g th e m a x im u m
s e r u m c o n c e n tr a tio n lo w e r.
It is p o s s ib le th a t lo s s o f c o n s c io u s n e s s is s e c o n d a r y to c a r d ia c a r r e s t, b u t it m a y a ls o b e
s e c o n d a r y to h ig h - d o s e C N S to x ic ity , r e s p i r a t o r y a r r e s t, o r h ig h n e u r a x ia l a n e s th e s ia .
A m io d a r o n e w a s th e d r u g o f c h o ic e to tr e a t c a r d io v a s c u la r to x ic ity o f b u p iv a c a in e in th e
p a st, b u t n o w 2 0 % in tr a lip i d h a s r e p la c e d th is d r u g . R e c o m m e n d e d d o s a g e f o r c a r d io v a s c u la r
c o lla p s e s e c o n d a r y to b u p iv a c a in e to x ic ity is 1.5 m L /k g b o lu s , f o ll o w e d b y 0 .2 5 m L /k g /m in f o r
th e n e x t 10 m in u te s . O x y g e n a tio n , v e n tila tio n , a n d g o o d b a s ic a n d a d v a n c e d lif e s u p p o r t a r e
e x tr e m e ly e s s e n tia l a s b u p iv a c a in e to x ic ity is a d v e r s e ly a f f e c te d b y h y p e r c a r b ia , a c id o s is , a n d
h y p o x ia .

27. D. N e u r o m u s c u la r - b l o c k in g d r u g s (N M B D s ) a r e h ig h ly c h a r g e d m o le c u le s b e c a u s e o f th e
p r e s e n c e o f a q u a te r n a r y a m m o n iu m g r o u p in th e ir s tr u c tu r e . T h is m a k e s th e m p o o r l y lip id
s o lu b le so th a t th e y d o n o t c r o s s b i o l o g i c m e m b r a n e s lik e b l o o d - b r a i n b a r r i e r , r e n a l tu b u la r
e p ith e liu m , a n d p la c e n ta . A d m in is tr a tio n o f th e s e d r u g s th u s d o e s n o t p r o d u c e c e n tr a l n e r v o u s
s y s te m e ffe c ts ; r e n a l tu b u la r r e a b s o r p t i o n is m in im a l, a n d m a te r n a l a d m in is tr a tio n d o e s n o t
a d v e r s e ly a f f e c t th e fe tu s . Is s u e o f i o n tr a p p in g c a n o n ly d e v e lo p if a d r u g g e ts tr a p p e d in th e
a c id ic e n v ir o n m e n t o f fe ta l b l o o d a fte r it h a s c r o s s e d th e p la c e n ta .

28. D. A n y tim e p r o l o n g e d s k e le ta l m u s c le in a c tiv ity o r e x te n s iv e m u s c le d a m a g e e x is ts , p a tie n t


m a y b e s u s c e p tib le to h y p e r k a le m i a a fte r th e a d m in is tr a tio n o f s u c c in y lc h o lin e a n d is d e p e n d e n t
o n d e v e lo p m e n t o f e x tr a ju n c tio n a l a ty p ic a l r e c e p to r s . In s o m e p a tie n ts , p o ta s s iu m le v e ls m a y
e x c e e d 10 m E q /L . T h is c a n le a d to s e r io u s c a r d ia c a r r h y th m ia s a n d e v e n c a r d ia c a r r e s t. T h e
d u r a tio n o f s u s c e p tib ility to th e h y p e r k a le m ic r e s p o n s e to s u c c in y lc h o lin e is u n k n o w n b u t
p r o b a b ly d e c r e a s e s a fte r 3 to 6 m o n th s o f d e n e r v a tio n in ju r y .
A lth o u g h c e r e b r a l p a ls y s e e m s to b e a m u s c u la r p r o b le m , c o n s e n s u s is th a t it is s a f e to u s e
s u c c in y lc h o lin e f o r th e s e p a tie n ts if a ir w a y m a n a g e m e n t w ill b e f a c ilita te d b y its u s e .

29. B. A ty p ic a l p la s m a c h o lin e s te r a s e la c k s th e a b ility to h y d r o ly z e e s te r b o n d s in d r u g s s u c h as


s u c c in y lc h o lin e a n d r e m if e n ta n il. D ia g n o s is o f p r e s e n c e o f a ty p ic a l e n z y m e c a n b e m a d e b y
m e a s u r in g th e a b ility o f d ib u c a in e , a lo c a l a n e s th e tic , to in h ib it th e a c tiv ity o f th is e n z y m e . A
n o r m a l e n z y m e g e ts in h ib ite d th e m o s t (8 0 % ), w h ile a ty p ic a l h o m o z y g o u s ty p e is m in im a lly
in h ib ite d (2 0 % ). H e te r o z y g o u s v a r ie ty h a s le s s e r in h ib itio n .
In c lin ic a l te r m s , th is le a d s to p r o l o n g e d p a r a ly s is f o ll o w i n g a d m in is tr a tio n o f
s u c c in y lc h o lin e , w ith d u r a tio n r a n g in g f r o m 5 to 10 m in u te s w ith n o r m a l e n z y m e to 6 0 to 1 8 0
m in u te s f o r p a tie n ts h a v in g h o m o z y g o u s a ty p ic a l v a r ie ty o f e n z y m e .

30. D. C h o ic e o f th e m u s c le r e la x a n t f o r a n y g iv e n a n e s th e tic d e p e n d s o n m a n y f a c to r s lik e


d u r a tio n o f s u r g ic a l p r o c e d u r e , c o m o r b id iti e s o f th e p a tie n t, r e q u i r e d s p e e d o f o n s e t, r o u te o f
e lim in a tio n , d u r a tio n o f a c tio n , a n d a s s o c ia te d s id e e ffe c ts lik e ta c h y c a r d ia o r h y p e r k a le m ia .
O u t o f th e s e f a c to r s , th e d u r a tio n o f a c tio n d e p e n d s o n th e d o s e , p r e s e n c e o f a s s o c ia te d
c o n d itio n s lik e h y p o th e r m ia , c o n c o m ita n t u s e o f d r u g s th a t in f lu e n c e th e m u s c le r e la x a ti o n lik e
m a g n e s iu m o r c a lc iu m c h a n n e ls b lo c k e r s , a n d th e p r e s e n c e o f h e p a tic o r r e n a l d is e a s e .
In te rm e d ia te a c tin g n e u r o m u s c u la r b lo c k e r s lik e r o c u r o n i u m a n d v e c u r o n iu m u n d e r g o
p r i m a r i l y h e p a tic m e ta b o lis m a n d b i l i a r y e x c r e tio n w ith m in im a l r e n a l e x c r e tio n ( 1 0 % - 2 5 % ) .
M a n y o f th e lo n g - a c tin g n e u r o m u s c u la r b lo c k e r s a r e e x c r e te d b y th e k id n e y , a n d th e ir u s e in
p a tie n ts w ith r e n a l f a ilu r e m a y le a d to p r o l o n g e d n e u r o m u s c u la r b lo c k a d e . P a n c u r o n iu m is o n e
o f th e s e lo n g - a c tin g a g e n ts , a n d 8 0 % o f its a d m in is te r e d d o s e is e x c r e te d b y th e k id n e y . A s
d is c u s s e d in th e p r e v io u s q u e s tio n , s u c c in y lc h o lin e is m e ta b o liz e d b y th e p la s m a c h o lin e s te r a s e
a n d o n ly a f r a c tio n o f th e a d m in is te r e d d o s e r e a c h e s th e n e u r o m u s c u la r ju n c tio n .

31. A. O n s e t a n d d u r a tio n o f a c tio n a r e l a r g e l y d e p e n d e n t o n th e d o s e a d m in is te r e d . A lth o u g h


s m a lle r d o s e s c a n b e e ffe c tiv e , th e r e c o m m e n d e d in tu b a tin g d o s e is u s u a lly tw o to f o u r tim e s
h ig h e r th a n E D 95. T h is p r o v id e s a h ig h e r in c id e n c e o f b e tte r a n d e a r lie r in tu b a tin g c o n d itio n s
th a n w o u ld b e p o s s ib le w ith E D 95. H ig h e r d o s e s d o le a d to l o n g e r d u r a tio n o f a c tio n so th a t th e
r e tu r n o f th e f i r s t tw itc h o n tr a in o f fo u r , a p r e r e q u is ite b e f o r e a r e v e r s a l a g e n t c a n b e
a d m in is te r e d , is d e la y e d .

32. A. A n tic h o lin e s te r a s e s a r e u s e d to r e v e r s e th e e ffe c ts o f n o n d e p o la r iz i n g n e u r o m u s c u la r -


b lo c k in g a g e n ts (N M B D s ). S e le c tio n o f th e s e d r u g s d e p e n d s o n m a n y f a c to r s . O n e o f th e
c o n s id e r a tio n s is th e a b ility o f th e s e le c te d d r u g to c r o s s th e b l o o d - b r a i n b a r r i e r . A s N M B D s d o
n o t c r o s s th e b l o o d - b r a i n b a r r i e r , th e r e is n o e f f e c t o n th e c e n tr a l n e r v o u s s y s te m . U s in g a n
a n tic h o lin e s te r a s e w ith o n ly p e r ip h e r a l a c tio n th u s m a k e s s e n s e b e c a u s e c e n tr a l n e r v o u s s y s te m
s id e e ffe c ts o f a d r u g lik e p h y s o s tig m in e c a n b e a v o id e d . T h e s e e ffe c ts c a n b e v e r y p r o n o u n c e d
in e ld e r ly p a tie n ts , le a d in g to c o n f u s io n a n d a g ita tio n in th e r e c o v e r y r o o m . N e o s tig m in e a n d
p y r id o s tig m in e , w ith th e ir q u a te r n a r y a m m o n iu m s tr u c tu r e a n d c o n s e q u e n t in a b ility to c r o s s th e
b l o o d - b r a i n b a r r i e r , a r e th u s p r e f e r r e d a g e n ts to r e v e r s e th e a c tio n s o f N M B D s th a n
p h y s o s tig m in e w h ic h c r o s s e s th a t b a r r i e r r e a d ily . S a m e p r in c ip le a p p lie s to a n tic h o lin e r g ic
a g e n ts th a t a r e a d m in is te r e d a lo n g w ith a n tic h o lin e s te r a s e d r u g to c o u n te r a c t th e s u r g e o f
a c e ty lc h o lin e c a u s in g b r a d y c a r d ia . A tr o p in e a n d s c o p o la m in e , w ith th e ir t e r tia r y c h a r a c te r , m a y
c r o s s th e b l o o d - b r a i n b a r r i e r a n d m a y c a u s e c e n tr a l n e r v o u s s y s te m e ffe c ts . G ly c o p y r r o la te , o n
th e o th e r h a n d , h a s q u a te r n a r y s tr u c tu r e a n d la c k s c e n tr a l n e r v o u s s y s te m e ffe c ts a n d h a s l a r g e l y
r e p la c e d a tr o p in e f o r b lo c k in g th e a d v e r s e m u s c a r in i c e ffe c ts .

33. D. A d m in is tr a tio n o f a n tic h o lin e s te r a s e a g e n t le a d s to a c c u m u la tio n o f a c e ty lc h o lin e ,


m a n if e s tin g in c r e a s e d c h o lin e r g ic a c tio n s in th e b o d y . V a g a l s tim u la tio n c a u s e s b r a d y c a r d i a
a n d , if n o t a n ta g o n iz e d b y c o n c o m ita n t a d m in is tr a tio n o f a c h o lin e r g ic a g e n t, m a y le a d to
c a r d ia c s ta n d s till. A n tis ia la g o g u e a c tio n s o f th e s e a g e n ts a r e a ls o h e lp f u l in r e d u c in g th e
e x c e s s iv e s a liv a tio n in d u c e d b y p a r a s y m p a th e tic o v e r a c tiv ity o f a c e ty lc h o lin e g e n e r a te d b y
in h ib itio n o f c h o lin e s te r a s e . D is r u p tio n o f g a s tr o in te s tin a l a n a s to m o s is is a n o th e r
c o n s id e r a tio n , s e c o n d a r y to in c r e a s e d p e r is ta ls is o f th e b o w e l. C h o lin e r g ic a c tiv ity m a y a ls o
le a d to b r o n c h o s p a s m a n d n o t b r o n c h o d ila tio n .

34. C. Im p u ls e s g e n e r a te d b y th e n e r v e s tim u la to r a r e s ta n d a r d iz e d to e n s u r e u n if o r m ity o f


m o n ito r in g . O u t o f th e d if f e r e n t p a tte rn s d e s c r ib e d in th e q u e s tio n , o n ly th e c h a r a c te r is tic o f
tr a in o f f o u r is c o r r e c t . A lth o u g h th is m o d e is u s e d m o r e o f te n in m o d e r n c lin ic a l p r a c tic e o f
a n e s th e s ia , in f a c t th e a b s e n c e o f fa d e — a h a llm a r k o f n o n d e p o la r iz i n g b lo c k — is a m o r e
r e lia b le in d ic a to r o f r e v e r s a l o f n e u r o m u s c u la r b lo c k a d e w ith te ta n ic o r d o u b le - b u r s t
s tim u la tio n . A d e p o la r iz i n g b lo c k is c h a r a c te r iz e d b y a b s e n c e o f fa d e b u t ta k e s o n
c h a r a c te r is tic s o f a n o n d e p o la r iz i n g b lo c k if e n o u g h d e p o la r iz i n g a g e n t is a d m in is te r e d .

35. D. M a n y p h y s io lo g ic f a c to r s a n d a n e s th e tic a n d n o n a n e s th e tic d r u g s in te r a c t w ith


n o n d e p o la r iz i n g m u s c le r e la x a n ts (N D M R s ). V o la tile a n d lo c a l a n e s th e tic s p o te n tia te , w h ile
d e p o la r iz i n g m u s c le r e la x a n ts a n ta g o n iz e th e e ffe c ts o f th e s e d r u g s . I n tr a v e n o u s a n e s th e tic
a g e n ts d o n o t h a v e a n y a p p r e c ia b le m u s c le - r e la x a n t e f f e c t in n o r m a l d o s e s . A m in o g ly c o s id e
a n tib io tic s p o te n tia te th e e ffe c ts o f N D M R s, w h ile p e n ic illin , c e p h a lo s p o r in s , e r y th r o m y c in , a n d
te tr a c y c lin e a r e d e v o id o f n e u r o m u s c u la r e ffe c ts . S tr e p to m y c in b e lo n g s to a m in o g ly c o s id e
fa m ily , a n d th u s a u g m e n ts th e e ffe c ts o f N D M R s.

36. C. I n d u c tio n o f a n e s th e s ia in a p a tie n t w ith s e v e r e v a lv u la r s te n o s is c a n le a d to d e c r e a s e d


v e n tr ic u la r f illin g s e c o n d a r y to v a s o d ila ta tio n a n d d e c r e a s e d v e n o u s r e tu r n . I n c r e a s e in th e h e a r t
r a te is v e r y p o o r l y to le r a te d a s th e v e n tr ic u la r f il lin g is im p a ir e d b e c a u s e o f d e c r e a s e d d ia s to lic
tim e . In th e p r e s e n c e o f n o r m a l e je c tio n f r a c tio n , a u g m e n ta tio n o f c a r d ia c c o n tr a c tility w ith
a d m in is tr a tio n o f m ilr in o n e , e p in e p h r in e , o r d o b u ta m in e m a y n o t b e n e e d e d . A d m in is tr a tio n o f
p h e n y le p h r in e w ill c a u s e v e n o c o n s tr ic tio n th r o u g h s tim u la tio n o f a ! r e c e p to r s a n d in c r e a s e th e
le f t- v e n tr ic u la r f il lin g p r e s s u r e , th u s in c r e a s in g th e c a r d ia c o u tp u t a n d b l o o d p r e s s u r e . I n c r e a s e
in b l o o d p r e s s u r e w ill c a u s e b a r o r e c e p to r - in d u c e d d e c r e a s e in h e a r t ra te , a llo w in g m o r e tim e in
d ia s to le , im p r o v in g th e le f t- v e n tr ic u la r f illin g . I n c r e a s in g a f te r lo a d w ith p h e n y le p h r in e w ill
d e c r e a s e a b n o r m a l tr a n s m itr a l v a lv u la r p r e s s u r e g r a d ie n t th a t w ill h e lp w ith r e s t o r a t i o n o f
p e r f u s io n p r e s s u r e .

37. D. D r o p e r id o l is a b u ty r o p h e n o n e a n d is s tr u c tu r a lly r e la te d to h a lo p e r id o l. It a ffe c ts m a n y


r e c e p to r s in th e c e n tr a l n e r v o u s s y s te m , in c lu d in g d o p a m in e r e c e p to r s in th e c a u d a te n u c le u s
a n d th e m e d u lla r y c h e m o r e c e p to r t r i g g e r z o n e . T h e la tte r e f f e c t e x p la in s its a b ility to c o u n te r a c t
n a u s e a a n d v o m itin g . A p a r t f r o m th a t it a ls o in te r f e r e s w ith tr a n s m is s io n m e d ia te d b y s e r o to n in ,
n o r e p in e p h r in e , a n d G A B A . T h e n e t e f f e c t is a p p e a r a n c e o f tr a n q u ility a n d s e d a tio n in p a tie n ts
p r e m e d ic a te d w ith th is d r u g , b u t th e y a r e o f te n e x tr e m e ly a p p r e h e n s iv e a n d f e a r f u l. F o r th is
r e a s o n , d r o p e r i d o l h a s f a lle n in to d is f a v o r a s a s o le p r e m e d ic a tio n . P e r ip h e r a lly , d r o p e r i d o l
c a u s e s a b lo c k a d e . A d m in is tr a tio n o f d r o p e r i d o l m a y le a d to h y p o te n s io n in a h y p o v o le m ic
p a tie n t. It m a y a ls o c a u s e p r o l o n g a t i o n o f Q T in te r v a l a n d to r s a d e s d e p o in te s , a n d b e c a u s e o f
th is , th e U S F o o d a n d D r u g A d m in is tr a tio n h a s a s s o c ia te d a b la c k b o x w a r n in g w ith d r o p e r id o l.
P r i o r to u s e o f d r o p e r id o l, a 12 le a d s h o u ld b e r e c o r d e d , a n d in th e p r e s e n c e o f Q T in te r v a l
b e in g m o r e th a n 4 4 0 m s in m e n a n d m o r e th a n 4 5 0 m s in w o m e n , d r o p e r i d o l s h o u ld n o t b e
g iv e n .

38. D. A h e a lth y a d u lt c a n e lim in a te c y a n id e v ia th e liv e r a t a r a te e q u iv a le n t to c y a n id e p r o d u c tio n


d u r in g s o d iu m n i tr o p r u s s id e (S N P ) in f u s io n a t th e r a te o f 2 |ig /k g /m in . W h e n th e r a te o f S N P
in f u s io n e x c e e d s th a t o r w h e n s u lf u r d o n o r s a n d m e th e m o g lo b in a r e e x h a u s te d , c y a n id e to x ic ity
m a y d e v e lo p . F r e e c y a n id e r a d ic a l b in d s w ith in a c tiv e tis s u e c y to c h r o m e o x id a s e a n d p r e v e n t
o x id a tiv e p h o s p h o r y la tio n . T h is m a y c a u s e tis s u e a n o x ia , m e ta b o lic a c id o s is , a n d in c r e a s e d
o x y g e n s a tu r a tio n o f v e n o u s b l o o d b e c a u s e o f in a b ility o f th e c e lls to e x tr a c t o x y g e n f r o m
a r te r ia l b lo o d . U ltim a te ly , c a r d ia c a r r h y th m ia s m a y d e v e lo p . C y a n id e to x ic ity m u s t b e s u s p e c te d
e a r lie r th a n th a t s ta g e in a n y p a tie n t w h o d e v e lo p s r e s is ta n c e to th e h y p o te n s iv e a c tio n o f a
m a x im u m d o s e o f SN P.

39. C. D o p a m in e is a m a jo r n e u r o tr a n s m itte r in e x tr a p y r a m id a l s y s te m . D r u g s th a t a n ta g o n iz e
d o p a m in e a n d a r e a b le to c r o s s th e b l o o d - b r a i n b a r r i e r m a y le a d to e x tr a p y r a m id a l s y m p to m s ,
w h ic h m a y m a n if e s t a s to r t ic o ll is , o c u l o g y r i c c r is is , a n d a g ita tio n . L is t o f d r u g s th a t c a n
p r e c ip ita te th e s e s y m p to m s is lo n g , b u t im p o r ta n t o n e s in th e p e r io p e r a tiv e p e r i o d a r e
d r o p e r id o l, m e to c lo p r a m id e , h a lo p e r id o l, a n d p r o m e th a z in e . F o r tu n a te ly , it is r e a d ily tr e a te d b y
a d m in is tr a tio n o f d ip h e n h y d r a m in e . M id a z o la m is a ls o h e lp f u l in tr e a tin g th is c o n d itio n .
F a m o tid in e a n d g ly c o p y r r o la te a r e n o t a s s o c ia te d w ith a n y e x tr a p y r a m id a l e ffe c ts .

40. B. A ll o f th e m e d ic a tio n s m e n tio n e d in th e q u e s tio n s h o u ld b e c o n tin u e d in th e p e r io p e r a tiv e


p e r io d , except m o n o a m in e o x id a s e in h ib ito r s . B y d e c r e a s in g th e m e ta b o lis m o f c a te c h o la m in e s ,
th e s e m e d ic a tio n s c a u s e a n in c r e a s e in th e a m o u n t o f n o r e p in e p h r in e a v a ila b le at th e p r e s y n a p tic
a d r e n e r g i c n e r v e e n d in g . U s e o f in d ir e c t- a c tin g s y m p a th o m im e tic d r u g lik e e p h e d r in e to tr e a t
h y p o te n s io n w ill le a d to e x a g g e r a te d r e s p o n s e w ith s e v e r e d e g r e e o f h y p e r te n s io n a n d c a r d ia c
a r r h y th m ia s . R e c o m m e n d a tio n is to s to p th e s e a g e n ts a t le a s t 2 w e e k s b e f o r e th e p la n n e d
s u r g e r y . S in c e th is c a n c a u s e p r o b le m in a p a tie n t w h o is d e p e n d e n t o n th is m e d ic a tio n , th is
g r o u p o f m e d ic a tio n is f a llin g o u t o f f a v o r.
T h e r e is s tr o n g e v id e n c e to c o n tin u e th e u s e o f P - b lo c k e r s , c h o le s te r o l- l o w e r in g a g e n ts , a n d
H 2- b lo c k e r s . M o s t h o s p ita ls h a v e p o lic ie s to e n s u r e th a t p a tie n ts u s in g lo n g - t e r m P - b lo c k e r s
r e c e iv e th e m in th e p e r io p e r a tiv e p e r io d . S im ila r ly , th e r e is e v id e n c e th a t p e r io p e r a tiv e
c o n tin u e d u s e o f s ta tin s le a d s to b e tte r o u tc o m e s .

41. D. D o s e o f m a g n e s iu m s u lfa te u s e d to tr e a t p r e e c la m p s ia is h ig h a n d c a n i n te r f e r e w ith th e


e ffe c ts o f m a n y m e d ic a tio n s u s e d in a n e s th e s ia . It d e c r e a s e s th e M A C o f v o la tile a n e s th e tic s a n d
p o te n tia te s th e m u s c le r e la x a ti o n c a u s e d b y b o th d e p o la r iz i n g a s w e ll a s th e n o n d e p o la r iz i n g
m u s c le r e la x a n ts . T h e d o s e s o f th e s e a g e n ts n e e d to b e r e d u c e d , a n d th e a b ility o f th e p a tie n t to
b r e a th e s p o n ta n e o u s ly th e e n d o f g e n e r a l a n e s th e tic w h e r e m u s c le r e la x a n t w a s u s e d n e e d s to b e
a s s e s s e d v e r y c a r e f u lly . M a g n e s iu m d o e s n o t a f f e c t th e d o s e o f lo c a l a n e s th e tic .

42. C. T h e a b s o r p tio n o f th e e p in e p h r in e f r o m th e e p id u r a l s p a c e in to s y s te m ic c ir c u la t io n is to o
s lo w f o r it to c o u n te r a c t th e h y p o te n s io n w h ic h is c a u s e d b y a b o lu s o f lid o c a in e . L o c a l
v a s o c o n s tr ic tio n b y e p in e p h r in e s lo w s d o w n th e s y s te m ic a b s o r p tio n o f lid o c a in e , le a d in g to
lo w e r s e r u m le v e ls , d e c r e a s in g th e c h a n c e o f lo c a l a n e s th e tic to x ic ity a s w e ll a s p r o l o n g a t i o n o f
th e b lo c k b y a llo w in g th e lid o c a in e to w o r k l o n g e r o n th e n e u r o n a l tis s u e . E p in e p h r in e
im p r o v e s th e q u a lity o f b lo c k b y a c tin g o n th e a n a lg e s ic a d r e n e r g i c r e c e p to r s in th e s p in a l c o r d .

43. A. T h e r e la tiv e s o lu b ility o f a n a n e s th e tic in a ir, b lo o d , a n d tis s u e s is e x p r e s s e d a s p a r titio n


c o e f f ic ie n ts . E a c h e f f ic ie n t is th e r a tio o f th e c o n c e n tr a tio n o f th e a n e s th e tic g a s in e a c h o f th e
tw o p h a s e s at e q u ilib r iu m . L o w e r th e p a r titio n c o e f f ic ie n t, h ig h e r th e r a te o f e q u ilib r a tio n . In
o th e r w o r d s , f o r a n a n e s th e tic w ith a lo w e r a lv e o la r to b l o o d p a r titio n c o e f f ic ie n t, th e r a te o f
r i s e o f a lv e o la r c o n c e n tr a tio n , a n d th u s a lv e o la r p a r tia l p r e s s u r e , w ill b e h ig h e r th a n a n
a n e s th e tic w ith h ig h e r p a r titio n c o e f f ic ie n t. S in c e it is th e a lv e o la r p a r tia l p r e s s u r e th a t
d e te r m in e s th e p a r tia l p r e s s u r e in th e b r a in , m o r e r a p id r i s e o f a lv e o la r p r e s s u r e is tr a n s la te d
in to fa s te r a n e s th e tic in d u c tio n . A n o th e r f a c to r th a t p la y s a r o l e in th is r e g a r d is th e
c o n c e n tr a tio n e ffe c t. N itr o u s o x id e , b e in g a le s s p o te n t a n e s th e tic w ith a M A C o f 1 0 4 , is
a d m in is te r e d in m u c h l a r g e r q u a n titie s to in d u c e a n e s th e s ia th a n a p o te n t a g e n t lik e s e v o f lu r a n e .
T h e m a s s iv e in f l o w ( h ig h e r c o n c e n tr a tio n ) o f n itr o u s o x id e le a d s to h ig h e r r a te o f r i s e o f
a lv e o la r c o n c e n tr a tio n (F A ) o f w ith a b l o o d g a s p a r titio n c o e f f ic ie n t o f 0 .4 6 c o m p a r e d w ith
d e s f lu r a n e w ith p a r titio n c o e f f ic ie n t o f 0 .4 2 .
44. D. N e o s tig m in e c a u s e s in h ib itio n o f p la s m a c h o lin e s te r a s e . A s s u c c in y lc h o lin e is m e ta b o liz e d
b y th is e n z y m e , a d m in is tr a tio n o f s u c c in y lc h o lin e a fte r th e u s e o f n e o s tig m in e f o r r e v e r s a l o f
n e u r o m u s c u la r b lo c k a d e m a y le a d to lo n g e r - th a n - e x p e c te d d u r a tio n o f a c tio n o f
s u c c in y lc h o lin e . In th is s itu a tio n , c o n tin u e to m e c h a n ic a lly v e n tila te th e p a tie n t u n til th e p a tie n t
m e e ts e x tu b a tio n c r ite r ia . R o c u r o n iu m is m a in ly m e ta b o liz e d b y liv e r a n d e x c r e te d in to b ile ,
c is a tr a c u r iu m v ia H o f m a n n e lim in a tio n a n d p a n c u r o n iu m v ia k id n e y . N e o s tig m in e d o e s n o t
i n te r f e r e w ith a n y o f th e s e p r o c e s s e s .

45. C. P a tie n ts w ith h y p e r tr o p h ic c a r d io m y o p a th y b e h a v e a s if th e y h a v e a o r tic s te n o s is e x c e p t th a t


th e le f t- v e n tr ic u la r o u tf lo w o b s tr u c tio n is d y n a m ic in s te a d o f b e in g fix e d . D e c r e a s e d a f te r lo a d
u n d e r g e n e r a l a n e s th e s ia c a u s e s th e g r a d ie n t b e tw e e n th e le f t- v e n tr ic u la r p r e s s u r e a n d th e a o r tic
p r e s s u r e to in c r e a s e , le a d in g to c o lla p s e o f th e le f t- v e n tr ic u la r o u tf lo w tra c t, in c r e a s in g th e
o b s tr u c tio n , a n d d e c r e a s in g th e c a r d ia c o u tp u t. R e s to r a tio n o f th e a f te r lo a d w ith a d m in is tr a tio n
o f p h e n y le p h r in e r e v e r s e s th is e ffe c t. It a ls o d e c r e a s e s th e h e a r t ra te , a llo w in g m o r e tim e f o r
le f t- v e n tr ic u la r p e r f u s io n to ta k e p la c e d u r in g th e d ia s to le . D e c r e a s in g th e c a r d ia c c o n tr a c tility
m a y a ls o b e h e lp f u l a s th a t w ill p r e v e n t th e o p p o s in g w a lls o f th e o u tf lo w tra c t to c o m e to g e th e r
r e lie v in g th e o b s tr u c tio n . A m r in o n e w ill a c tu a lly in c r e a s e th e c o n tr a c tility w h ile r e d u c in g th e
a f te r lo a d : b o th e ffe c ts b e in g u n d e s ir a b le in th is c lin ic a l s itu a tio n . E p h e d r in e w ill in c r e a s e th e
h e a r t r a te a s w e ll a s c a r d ia c c o n tra c tility , th u s m a k in g th e s itu a tio n w o r s e a s d e s c r ib e d a b o v e .
N itr o g l y c e r in e m a y w o r s e n th e h y p o te n s i o n a n d m a y n o t b e a g o o d c h o ic e f o r a h y p o te n s iv e
p a tie n t.

46. D. K e to r o la c is a v a lu a b le n o n s te r o id a l a n a lg e s ic w ith m o d e s t a n ti- in f la m m a to r y a c tio n . It w a s


th e s o le n o n s te r o id a l a n ti- in f la m m a to r y d r u g a v a ila b le in in tr a v e n o u s f o r m p r i o r to th e
a v a ila b ility o f IV ib u p r o f e n . T h i r ty m i l l i g r a m s o f k e to r o la c is e q u iv a le n t in p o te n c y to 1 0 0 m g
o f m e p e r id in e o r 10 m g o f m o r p h in e . U n fo rtu n a te ly , it h a s m a n y s id e e ffe c ts th a t l im it its u s e in
th e p e r io p e r a tiv e p e r io d . I n h ib itio n o f p r o s ta g la n d i n w h ic h is p a r t o f its a n a lg e s ic m e c h a n is m o f
a c tio n le a d s to a f f e r e n t a r t e r i o l a r c o n s tr ic tio n .

47. B. A s th e c lin ic a l s itu a tio n s e e m s to in d ic a te th e n e e d f o r a n a g e n t th a t is p o te n t a n d e x tr e m e ly


fa s t in its o n s e t o f a c tio n , n itr o g ly c e r in e w ill b e m o r e h e lp f u l in th is s itu a tio n . N itr o g l y c e r in e is
c o n v e r te d in to n itr ic o x id e , w h ic h is a v e r y p o te n t v a s o d il a to r in c r e a s in g th e v e n o u s
c a p a c ita n c e . T h is a c tio n o f n itr o g l y c e r i n e h e lp s r e lo c a te th e in tr a v a s c u la r v o lu m e in to
p e r ip h e r a l c o m p a r tm e n t, th u s u n lo a d in g th e c e n tr a l c o m p a r tm e n t a n d a llo w in g th e p u lm o n a r y
e d e m a f lu id to b e r e a b s o r b e d in to th e c ir c u la tio n .

48. D. O n d a n s e tr o n h a s b e e n s h o w n to in c r e a s e th e Q T in te r v a l. T h is r e s p o n s e is c o m p a r a b le to
th a t o c c u r r i n g w ith d r o p e r i d o l . A lth o u g h th e r e is n o c le a r a s s o c ia tio n b e tw e e n to r s a d e s d e
p o in te s a n d th is d r u g , it is r e c o m m e n d e d th a t th is d r u g b e a v o id e d in p a tie n ts w ith c o n g e n ita l
p r o l o n g e d Q T s y n d r o m e . M e to c lo p r a m id e h a s a s im ila r e ffe c t. S u c c in y lc h o lin e a d m in is tr a tio n
c a n p r o l o n g Q T in te r v a l p o s s ib ly f r o m p o ta s s iu m e f f lu x a n d b y its e ffe c t o n th e a u to n o m ic
n e r v o u s s y s te m . P r o p o f o l, o n th e o th e r h a n d , h a s b e e n s h o w n to b e s a f e in p a tie n ts w ith th is
c o n d itio n a n d m a y a c tu a lly d e c r e a s e th e Q T in te r v a l in c r e a s e in d u c e d b y s e v o f lu r a n e .
49. C. N a lo x o n e is a n o n s e le c tiv e o p io id a n ta g o n is t a t a ll th r e e ^ - r e c e p to r s . It d o e s n o t s e e m to
h a v e a n y a g o n is t a c tiv ity a t th e o p io id r e c e p to r s . U n fo rtu n a te ly , h a lf - li f e is s h o r te r ( 3 0 - 4 5
m in u te s ) th a n m o s t c o m m o n ly u s e d o p io id s . S o r e n a r c o t iz a tio n is a p o s s ib ility . It is u s e f u l in th e
tr e a tm e n t o f o p io id - in d u c e d s p a s m o f th e s p h in c te r o f O d d i. N a lo x o n e e a s ily c r o s s e s th e
p la c e n ta . F o r th is r e a s o n , a d m in is tr a tio n o f n a lo x o n e to a n o p io id - d e p e n d e n t p a r tu r ie n t m a y
p r o d u c e a c u te w ith d r a w a l in th e n e o n a te .

50. B. O p io id s u s u a lly c a u s e b r a d y c a r d ia . T h is e f f e c t is m e d ia te d th r o u g h c e n tr a l n e r v o u s s y s te m .
T h e y a ls o h a v e d ir e c t e f f e c t o n th e c a r d ia c p a c e m a k e r c e lls . M o r p h in e c a u s e s v a s o d ila ta tio n ,
a n d in th e p r e s e n c e o f p r e e x is tin g h y p o v o le m ia , it m a y le a d to d e c r e a s e d b l o o d p r e s s u r e a n d
b a r o r e c e p to r - in d u c e d ta c h y c a r d ia . M e p e r id in e is a n e x c e p tio n ; it h a s in tr in s ic a tr o p in e lik e
a c tiv ity th a t m a y c a u s e ta c h y c a r d ia a fte r its a d m in is tr a tio n .

51. B. O p io id r e c e p to r s a r e f o u n d in s id e s u b s ta n tia g e la tin o s a in th e s p in a l c o r d . A d d itio n o f


fe n ta n y l to lo c a l a n e s th e tic in je c te d in th e e p id u r a l s p a c e d e c r e a s e s th e o n s e t o f a n a lg e s ia tim e .
S in c e e p id u r a l b u p iv a c a in e h a s a l o n g e r d u r a tio n o f a c tio n th a n e p id u r a l fe n ta n y l, th e d u r a tio n
o f b lo c k m a y n o t b e p r o lo n g e d . E p id u r a l fe n ta n y l h a s n o e f f e c t o n th e v a g u s n e r v e . D e g r e e o f
a n a lg e s ia is e n h a n c e d b y a d d itio n o f fe n ta n y l to e p id u r a l lo c a l a n e s th e tic , b u t th e e f f e c t o n th e
s e n s o r y a n d m o to r b lo c k is n o t a u g m e n te d .

52. B. A lfe n ta n il h a s a fa s t o n s e t o f a c tio n c o m p a r e d w ith s u fe n ta n il b e c a u s e o f a v e r y h ig h


p r o p o r t i o n o f it b e in g u n io n iz e d a t p h y s io lo g ic p H : 9 0 % v s . 2 0 % . T h is is e x p la in e d b y th e
lo w e r p K a o f a lf e n ta n il (6 .5 ) v s . s u fe n ta n il (8 .0 ). S o its p e n e tr a tio n in to b r a i n is m u c h fa s te r th a n
s u fe n ta n il. Its p r o te in - b in d in g is c o m p a r a b le to s u fe n ta n il, w h ile lip id s o lu b ility is m u c h le s s ,
le a d in g to lo w e r to ta l v o lu m e o f d is tr ib u tio n .

53. C. H ig h e r o i l/g a s p a r titio n c o e f f ic ie n t m e a n s h ig h e r p r o p o r t i o n o f in h a le d a g e n t is in s o lu b le


f o r m in b l o o d b e f o r e e n o u g h p a r tia l p r e s s u r e is a c h ie v e d a t th e a lv e o la r , a n d f in a lly in th e
b r a in , to a n e s th e tiz e th e p a tie n t. S a m e p r o c e s s is r e v e r s e d a t th e tim e o f a w a k e n in g . W ith
in c r e a s e d tim e o f a d m in is tr a tio n , so m u c h a n e s th e tic is f o u n d in th e tis s u e s in a s o lu b le f o r m
th a t a ll o th e r f a c to r s b e c o m e m u c h le s s im p o r ta n t a s d e te r m in a n ts o f r e c o v e r y tim e . H ig h e r
c a r d ia c o u tp u t m a y s lo w d o w n th e r e c o v e r y tim e , b u t its e f f e c t w ill b e s m a lle r th a n th e e f f e c t o f
d u r a tio n o f a d m in is tr a tio n . M A C o f th e d r u g in its e lf d o e s n o t d e te r m in e th e tim e o f in d u c tio n
o r re c o v e ry .

54. D. H a llm a r k o f n i tr o p r u s s id e p o is o n in g is in c r e a s in g m e ta b o lic a c id o s is s e c o n d a r y to


im p a ir e d o x id a tiv e p h o s p h o r y la tio n in th e c e ll b e c a u s e o f a c c u m u la tio n o f c y a n id e io n s . A c u te
m y o c a r d i a l i n f a r c tio n is n o t a c o n tr a in d ic a tio n in its e lf o f n i tr o p r u s s id e th e r a p y a s lo n g a s it is
n e e d e d to tr e a t h ig h b l o o d p r e s s u r e . S a m e is tr u e f o r m itr a l r e g u r g ita tio n , a n d in fa c t,
n i tr o p r u s s id e m a y b e h e lp f u l a s it m a y in c r e a s e th e c a r d ia c o u tp u t in th is c o n d itio n b y
d e c r e a s in g th e a f te r lo a d .
R e n a l f a ilu r e m a y in c r e a s e th e a v a ila b ility o f s u lfa te io n , w h ic h a llo w s p r o d u c tio n o f m o r e
th io s u lf a te to a c t a s a d o n o r a n d th u s c o n v e r t c y a n id e to th io c y a n a te . P r o l o n g e d a d m in is tr a tio n
o f h ig h d o s e s o f n i tr o p r u s s id e m a y le a d to th io c y a n a te a c c u m u la tio n a n d to x ic ity .

55. C. S p in a l a n e s th e s ia is r a r e l y a s s o c ia te d w ith d r a m a tic d r o p o f h e a r t r a te a n d b l o o d p r e s s u r e in


y o u n g in d iv id u a ls . T h e m e c h a n is m is p o o r l y u n d e r s to o d . P r o p o s e d m e c h a n is m in c lu d e s
p r e e x is tin g h y p o v o le m ia , u n r e c o g n iz e d h y p o x e m ia s e c o n d a r y to s e d a tio n , o r a h ig h s p in a l w ith
in h ib itio n o f c a r d i o a c c e l e r a t o r y s y m p a th e tic n e r v e s a r is in g f r o m T 1 to T 4 s e g m e n ts o f th e
s p in a l c o r d .
In th e c lin ic a l s c e n a r io d e s c r ib e d , a tr o p in e in its e lf m a y n o t b e a b le to c o r r e c t th e
h e m o d y n a m ic s , a n d th e s itu a tio n s c a ll f o r in itia tio n o f m e a s u r e s r e q u ir e d in a d v a n c e d c a r d ia c
lif e s u p p o r t. If th e r e is n o p u ls e , c h e s t c o m p r e s s io n s a lo n g w ith a d m in is tr a tio n o f e p in e p h r in e
m a y b e th e b e s t c o u r s e o f a c tio n .

56. D. G e n ta m y c in is a n a m in o g ly c o s id e a n tib io tic th a t e n h a n c e s n e u r o m u s c u la r b lo c k a d e a c tio n


o f m u s c le r e la x a n ts u s e d in a n e s th e s ia . M a g n e s iu m in its e lf p o te n tia te s n e u r o m u s c u la r - b lo c k in g
a g e n ts ’ a c tio n a n d so a c ts s y n e r g is ti c a lly to p r o l o n g th e n e u r o m u s c u la r b lo c k a d e .
A n tic h o lin e s te r a s e s in c r e a s e th e a m o u n t o f a c e ty lc h o lin e a v a ila b le a t th e n e u r o m u s c u la r
ju n c tio n b y in h ib itin g th e e n z y m e th a t m e ta b o liz e s it. S u c c in y lc h o lin e - in d u c e d n e u r o m u s c u la r
b lo c k a d e e n h a n c e s th e w e a k n e s s p r o d u c e d b y a m in o g ly c o s id e a n tib io tic s .
P r o p o s e d m e c h a n is m o f a c tio n o f th e s e a n tib io tic s in c a u s in g th e p o te n tia tio n o f a c tio n o f
n e u r o m u s c u la r - b lo c k in g a g e n ts is th e in h ib itio n o f r e le a s e o f a c e ty lc h o lin e a t th e p r e ju n c tio n a l
s ite . C a lc iu m a n ta g o n iz e s th is a c tio n o f a n tib io tic s , a n d a t le a s t t e m p o r a r i l y r e v e r s e s th e ir e f f e c t
o n e n h a n c e m e n t o f n e u r o m u s c u la r - b lo c k in g a c tio n o f th e s e a n tib io tic s . B u t s in c e c a lc iu m a ls o
s ta b iliz e s th e p o s tju n c tio n a l m e m b r a n e to th e e f f e c t o f a c e ty lc h o lin e , s o m e tim e s th e e f f e c t o f
c a lc iu m o n a n ta g o n is m o f a n tib io tic - in d u c e d e n h a n c e m e n t o f n e u r o m u s c u la r b lo c k a d e
p r o d u c e d b y n o n d e p o la r iz i n g n e u r o m u s c u la r - b lo c k in g a g e n ts is u n p r e d ic ta b le .

57. A. L o r a z e p a m is c o n ju g a te d in th e liv e r w ith g lu c u r o n ic a c id to p r o d u c e in a c tiv e m e ta b o lite s ,


b u t th is p r o c e s s is m u c h s lo w e r th a n th e m e ta b o lis m o f m id a z o la m . A s a r e s u lt, th e e lim in a tio n
h a lf - li f e o f lo r a z e p a m is m u c h l o n g e r ( 1 0 - 2 0 h o u r s ) c o m p a r e d w ith m id a z o la m ( 1 - 4 h o u r s ) .
S im ila r ly , th e c le a r a n c e o f m id a z o la m is s ix to e ig h t tim e s th a t o f lo r a z e p a m . V o lu m e o f
d is tr ib u tio n o f lo r a z e p a m is c o m p a r a b le to m id a z o la m .

58. B. V o la tile a n e s th e tic s c a u s e c h a r a c te r is tic d o s e - d e p e n d e n t c h a n g e s in th e E E G . I n c r e a s in g


d e p th o f a n e s th e s ia w ith is o f lu r a n e f r o m th e a w a k e s ta te is c h a r a c te r iz e d b y in c r e a s e d a m p litu d e
a n d s y n c h r o n y . P e r io d s o f e le c tr ic a l s ile n c e b e g in to o c c u p y a g r e a te r p o r t i o n o f th e tim e as
d e p th in c r e a s e s ( b u r s t s u p p r e s s io n ) . M id a z o la m a n d th io p e n ta l b o th in c r e a s e th e in h ib ito r y
a c tio n o f G A B A r e c e p to r a n d s lo w d o w n th e E E G . L id o c a in e , o n th e o th e r h a n d , h a s a b ip h a s ic
a c tio n . A t a lo w e r s e r u m le v e l, it c a u s e s r e s tle s s n e s s , tr e m o r , tin n itu s , a n d v e r tig o c u lm in a tin g
in t o n i c - c l o n i c s e iz u r e , w h ic h r e f le c ts in h ib itio n o f c o r tic a l in h ib ito r y n e u r o n s . L a r g e r d o s e s
in h ib it b o th in h ib ito r y a n d e x c ita to r y n e u r o n s , le a d in g to c e n tr a l n e r v o u s s y s te m d e p r e s s io n a n d
com a.

59. B. P la s m a p s e u d o c h o lin e s te r a s e o r n o n s p e c if ic c h o lin e s te r a s e is a n e n z y m e w ith m o le c u la r


w e ig h t o f 3 2 0 ,0 0 0 . It is f o u n d in p la s m a a n d m o s t tis s u e s b u t n o t in r e d b l o o d c e lls . It d e g r a d e s
a c e ty lc h o lin e r e le a s e d a t th e n e u r o m u s c u la r ju n c tio n . It is p r i m a r i l y p r o d u c e d in th e liv e r , so
e n d - s ta g e liv e r d is e a s e m a y d e c r e a s e p la s m a c h o lin e s te r a s e a c tiv ity . N o r m a l p la s m a
p s e u d o c h o lin e s te r a s e d o e s n o t r e s i s t d ib u c a in e in h ib itio n , w h ile th e a b n o r m a l o n e d o e s . S o th e
d ib u c a in e n u m b e r is a g o o d e s tim a tio n o f th e d e g r e e o f q u a lita tiv e a b n o r m a lity o f th e e n z y m e .
A c e ty lc h o lin e s te r a s e s a n ta g o n iz e th is e n z y m e . M e ta b o lis m o f s u c c in y lc h o lin e b y
p s e u d o c h o lin e s te r a s e is a tw o -s te p p r o c e s s o f h y d r o ly s is . F ir s t s te p c o n v e r ts s u c c in y lc h o lin e to
s u c c in y lm o n o c h o lin e , a n d th e s e c o n d s te p to s u c c in ic a c id .

60. D. A s m e n tio n e d in th e p r e v io u s d is c u s s io n p s e u d o c h o lin e s te r a s e m e ta b o liz e s th e in je c te d


s u c c in y lc h o lin e b e f o r e it r e a c h e s n e u r o m u s c u la r ju n c tio n . T h is p r o c e s s is so f a s t th a t o n ly 5 %
o f in je c te d s u c c in y lc h o lin e r e a c h e s th e n e u r o m u s c u la r ju n c tio n . In th e p r e s e n c e if a ty p ic a l
p s e u d o c h o lin e s te r a s e , th is m e ta b o lis m is s lo w , a n d g r e a te r q u a n tity o f s u c c in y lc h o lin e r e a c h e s
n e u r o m u s c u la r ju n c tio n , le a d in g to p r o l o n g e d a p n e a , f o ll o w i n g th e s ta n d a r d d o s e o f
s u c c in y lc h o lin e . D if f u s io n a w a y f r o m th e n e u r o m u s c u la r ju n c tio n s ta y s th e s a m e w h e th e r th e
p a tie n t h a s n o r m a l o r a ty p ic a l e n z y m e a n d d o e s n o t c o n tr ib u te m u c h to th e c e s s a tio n o f a c tio n
o f s u c c in y lc h o lin e . S u c c in y lc h o lin e is n o t m e ta b o liz e d in th e liv e r , a lth o u g h
p s e u d o c h o lin e s te r a s e is p r o d u c e d in th e liv e r . L iv e r d is e a s e h a s to b e s e v e r e b e f o r e d e c r e a s e s in
p la s m a p s e u d o c h o lin e s te r a s e p r o d u c tio n s u f f ic ie n t to p r o l o n g s u c c in y lc h o lin e - in d u c e d
n e u r o m u s c u la r b lo c k w ill o c c u r b e c a u s e a n in c r e a s e d p r o p o r t i o n o f s u c c in y lc h o lin e r e a c h e s th e
n e u r o m u s c u la r ju n c tio n .

61. B. E ffe c ts o f n a r c o tic s o n s m o o th m u s c le s a r e v a r ia b le in d if f e r e n t a r e a s o f th e b o d y . It c a u s e s


c o n tr a c tio n o f th e s m o o th m u s c le o f th e g a s tr o in te s tin a l tra c t, c a u s in g v a r ie ty o f s id e e ffe c ts
lik e c o n s tip a tio n , b i l i a r y c o lic , a n d d e la y e d g a s tr ic e m p ty in g . I n c r e a s e d b i l i a r y p r e s s u r e o c c u r s
w h e n th e g a llb la d d e r c o n tr a c ts a g a in s t a c lo s e d o r n a r r o w e d s p h in c te r o f O d d i. U r in a r y u r g e n c y
is p r o d u c e d b y o p io id - in d u c e d a u g m e n ta tio n o f d e tr u s o r to n e , b u t, a t th e s a m e tim e , th e to n e o f
th e b la d d e r s p h in c te r is e n h a n c e d , c a u s in g u r i n a r y r e te n tio n . O p io id s a lte r th e d e v e lo p m e n t,
d if f e r e n tia tio n , a n d f u n c tio n o f im m u n e c e lls . C h r o n ic r a th e r th a n a c u te u s e o f o p io id s is
a s s o c ia te d w ith im m u n o s u p p r e s s io n , a n d w ith d r a w a l f r o m o p io id s c a n a ls o in c r e a s e th e d e g r e e
o f im m u n o s u p p r e s s io n .

62. A. M o r p h in e e x h ib its g r e a te r a n a lg e s ic p o te n c y a n d s lo w e r o n s e t o f a c tio n in w o m e n th a n


m e n . O ld e r in d iv id u a ls a r e a ls o m o r e p r o n e to th e s e d a tiv e e ffe c t o f o p io id d r u g s c o m p a r e d
w ith y o u n g e r in d iv id u a ls . L iv e r d is e a s e d o e s n o t s e e m to a f f e c t th e s e n s itiv ity o f th e in d iv id u a l
to o p io id a d m in is tr a tio n e x c e p t d u r in g liv e r tr a n s p la n t s u r g e r y ; w h e n in a n h e p a tic p h a s e , th e
e f f e c t o f o p io id s m a y b e e n h a n c e d . M o r p h in e - 6 - s u lf a te m a y a c c u m u la te in c a s e s o f r e n a l
f a ilu r e , c a u s in g u n e x p e c te d v e n tila to r y d e p r e s s a n t e ffe c ts f r o m e v e n a s m a ll d o s e o f m o r p h in e .

63. B. L o w e r e s o p h a g e a l s p h in c te r m e c h a n is m c o n s is ts o f th e in tr in s ic to n e o f th e in tr in s ic
s m o o th m u s c le o f th e d is ta l e s o p h a g u s a n d th e s k e le ta l m u s c le o f th e d ia p h r a g m . U n d e r n o r m a l
c ir c u m s ta n c e s , th e lo w e r e s o p h a g e a l s p h in c te r is a p p r o x im a te ly 4 c m lo n g . M u s c le to n e in th e
lo w e r e s o p h a g e a l s p h in c te r is th e r e s u lt o f n e u r o g e n ic a n d m y o g e n ic m e c h a n is m s . A s u b s ta n tia l
p o r t i o n o f th e n e u r o g e n ic to n e in th e h u m a n s is d u e to c h o lin e r g ic in n e r v a tio n v ia th e v a g u s
n e r v e . T h e p r e s y n a p tic n e u r o tr a n s m itte r is a c e ty lc h o lin e , a n d p o s ts y n a p tic n e u r o tr a n s m itte r is
n itr ic o x id e . T h e n o r m a l lo w e r e s o p h a g e a l s p h in c te r p r e s s u r e is 10 to 3 0 m m H g a t e n d
e x h a la tio n . S u c c in y lc h o lin e in c r e a s e s i n tr a g a s t r ic a n d lo w e r s e s o p h a g e a l p r e s s u r e s .
N e o s tig m in e a ls o in c r e a s e s th is s p h i n c t e r ’s to n e b y in c r e a s in g th e c o n c e n tr a tio n o f
a c e ty lc h o lin e . M e to c lo p r a m id e a ls o in c r e a s e s lo w e r e s o p h a g e a l s p h in c te r to n e a n d is h e lp f u l in
tr e a tin g th e s y m p to m s o f g a s tr o e s o p h a g e a l r e f lu x a n d a s s o c ia te d e s o p h a g itis . G ly c o p y r r o la te ,
o n th e o th e r h a n d , r e la x e s th e s m o o th m u s c le o f th is s p h in c te r.

64. C. A ll o f th e a g e n ts m e n tio n e d in th is q u e s tio n c a n b e u s e d to a n e s th e tiz e a p a tie n t f o r a s h o r t


d u r a tio n o f tim e o n f r e q u e n t b a s is e x c e p t f o r e to m id a te a s its a d r e n a l s u p p r e s s iv e a c tio n w ill
im p a ir th e a b ility o f th e p a tie n t to m o u n t a s tr e s s r e s p o n s e , w h ic h th is p a tie n t w ill n e e d o n a n
o n g o in g b a s is .

65. B. E u te c tic m ix tu r e is a c o m b in a tio n o f tw o s u b s ta n c e s w h o s e m e ltin g p o in t is lo w e r th a n th a t


o f e ith e r o f th e c o n s titu e n ts . E M L A c r e a m is a e u te c tic m ix tu r e o f lid o c a in e (2 .5 % ) a n d
p r il o c a in e (2 .5 % ) w ith a m e ltin g p o in t o f 1 8 0 ° C so th a t th e m ix tu r e is a n o i l y liq u id a t b o d y
te m p e r a tu r e .
F iv e p e r c e n t E M L A c r e a m is a p p lie d to d r y in ta c t s k in a n d c o v e r e d w ith a n o c c lu s iv e
d r e s s in g f o r a t le a s t a n h o u r . It p r o v id e s to p ic a l a n e s th e s ia f o r 1 to 2 h o u r s . T h e a m o u n t o f d r u g
a b s o r b e d d e p e n d s o n a p p lic a tio n tim e , d e r m a l b l o o d flo w , s k in th ic k n e s s , a n d to ta l d o s e
a d m in is te r e d . S o m e p a tie n ts m a y d is lik e th e tin g lin g f e e lin g th a t is p r o d u c e d b y th is d r u g . It
s h o u ld n o t b e a p p lie d to b r o k e n s k in o r m u c o u s m e m b r a n e s . S id e e ffe c ts in c lu d e s k in
b la n c h in g , e ry th e m a , e d e m a , a n d m e th e m o g lo b in e m ia . T h e la s t s id e e f f e c t is s e c o n d a r y to
m e ta b o lis m o f p r il o c a in e O - to lu id in e a n d m a y b e m o r e c o m m o n if th e p a tie n t is c o n c u r r e n t ly
ta k in g s u lf o n a m id e s a n d o th e r m e th e m o g lo b in - in d u c in g d r u g s .

66. D. E p id u r a l o p io id s c a n c a u s e n a u s e a , p r u r itu s , a n d r e s p i r a t o r y d e p r e s s io n . B ig g e s t a d v a n ta g e
o f th e s e a g e n ts o v e r e p id u r a l a d m in is tr a tio n o f lo c a l a n e s th e tic s is th e h e m o d y n a m ic s ta b ility , as
th e r e is n o in h ib itio n o f s y m p a th e tic s y s te m . H y p o te n s io n is h ig h ly u n lik e ly w ith e p id u r a l
fe n ta n y l a d m in is tr a tio n .

67. B. I n tr a c ta b le s e iz u r e s a r e s o m e tim e s tr e a te d w ith e x c is io n o f th e s e iz u r e f o c u s in th e b r a in .


A n e s th e s io lo g is t is s o m e tim e s a s k e d in th e s e c a s e s to h e lp lo c a te th e f o c u s th r o u g h e n h a n c in g
th e E E G a c tiv ity o r a c tu a lly in d u c in g th e s e iz u r e d u r in g th e a n e s th e tic . S o m e a n e s th e tic a g e n ts
a r e k n o w n to in c r e a s e th e s e iz u r e a c tiv ity a n d c a n b e u tiliz e d f o r th a t p u r p o s e . E to m id a te ,
m e th o h e x ita l, o ld e r in h a le d a n e s th e tic e n f lu r a n e , a n d , to s o m e d e g r e e , k e ta m in e c a n b e h e lp f u l
in th is r e g a r d . O th e r a n e s th e tic s a c tu a lly in c r e a s e th e s e iz u r e th r e s h o ld a n d m a k e it d if f ic u lt f o r
th e s u r g e o n to f in d th e a r e a o f in te re s t.

68. D. A ll th e p o r p h y r ia s r e s u lt f r o m a d e f e c t in h e m e s y n th e s is . H e m e is a n e s s e n tia l c o m p o n e n t
o f h e m o g lo b in , m y o g lo b in , a n d c y to c h r o m e s , th a t is , c o m p o u n d s in v o lv e d in th e tr a n s p o r t a n d
a c tiv a tio n o f o x y g e n a n d th e e le c tr o n tr a n s p o r t c h a in . F o r a n e s th e s io lo g is ts , p o r p h y r i a c a n b e
d iv id e d in to in d u c ib le a n d n o n in d u c ib le . I n d u c ib le o n e s a r e th o s e th a t a r e t r i g g e r e d b y a n
e x o g e n o u s f a c to r lik e a d m in is tr a tio n o f a d r u g . D r u g s th a t in d u c e c y to c h r o m e e n z y m e s lik e
b a r b itu r a te s a n d p h e n y to in c a n p r e c ip ita te a n e p is o d e o f p o r p h y r ia . S ig n s a n d s y m p to m s d e p e n d
o n th e ty p e o f p o r p h y r ia , b u t a n e s th e s io l o g is t is u s u a lly in v o lv e d in a c a s e w h e r e p a tie n t is
b r o u g h t to th e o p e r a tin g r o o m f o r e x p l o r a t o r y l a p a r o to m y s e c o n d a r y to n a u s e a , v o m itin g , a n d
p a in in th e a b d o m e n . N o o r g a n i c c a u s e o f th e s e s y m p to m s is fo u n d , a n d p a tie n t m a y th e n
d e v e lo p o th e r s ig n s o f p o r p h y r i a p o s to p e r a ti v e ly lik e n e u r o l o g i c s ig n s o f h e m ip le g ia ,
q u a d r ip le g ia , p s y c h ia tr ic d is tu r b a n c e s , a n d a lte r a tio n o f c o n s c io u s n e s s o r p a in .
In h a le d a n e s th e tic s , n itr o u s o x id e , in d u c tio n a g e n ts o th e r th a n b a r b itu r a te s , a n d o p io id s a r e
a ll s a f e to u s e in th e s e p a tie n ts . E lic ita tio n o f f a m ily h is to r y a n d p a s t h is to r y o f s im ila r e p is o d e s
c a n h e lp w ith th e d ia g n o s is . P e r io p e r a tiv e ly , d is tu r b a n c e s o f a u to n o m ic s y s te m a n d e le c tr o ly te
im b a la n c e a r e c o m m o n a n d n e e d to b e a d d r e s s e d .

69. D. K e to r o la c is a n o n s te r o id a l a n ti- in f la m m a to r y a n a lg e s ic th a t is a v a ila b le in p a r e n te r a l f o r m .


A d m in is tr a tio n o f th is m e d ic a tio n w ill h e lp a v o id s id e e ffe c ts th a t a r e a s s o c ia te d w ith th e u s e o f
m o r p h in e , s u c h a s n a u s e a a n d r e s p i r a t o r y d e p r e s s io n . K e to r o la c 3 0 m g p r o d u c e s e q u iv a le n t
a n a lg e s ia c o m p a r e d w ith 10 m g o f m o r p h in e . S in c e it is d e v o id o f a c tio n o n th e s p h in c te r o f
O d d i, it is a u s e f u l d r u g in p a tie n ts w h o h a v e p a in s e c o n d a r y to b i l i a r y s p a s m . L ik e a n y o th e r
n o n s te r o id a l a n ti- in f la m m a to r y d r u g , it d o e s c a r r y th e s id e e f f e c t o f in h ib itio n o f p la te le t
f u n c tio n a n d in c r e a s in g th e c h a n c e o f b le e d in g p o s to p e r a tiv e ly .

70. D. M A C is d e f in e d a s th e d o s e o f a n a n e s th e tic a t w h ic h 5 0 % o f p a tie n ts d o n o t m o v e in


r e s p o n s e to a s u r g ic a l in c is io n . D if f e r e n t d r u g s a n d p h y s io lo g ic a n d p a th o lo g ic sta te s c a n a f f e c t
th e M A C o f a n a n e s th e tic . C h r o n ic a lc o h o l u s e in c r e a s e s th e M A C , w h ile a c u te in to x ic a tio n
d e c r e a s e s it. R e s p ir a to r y a lk a lo s is d o e s n o t s e e m to h a v e a n y e ffe c t. C h r o n ic a n e m ia d e c r e a s e s
M A C , b u t it s e e m s to d o so o n ly if h e m o g lo b in le v e l is b e lo w 5 g m /d L . H y p o th e r m ia d e c r e a s e s
th e M A C , w h ile h y p e r th e r m ia in c r e a s e s it.

71. D. A ll o f th e d r u g s m e n tio n e d in th e q u e s tio n a r e a g o n i s t - a n t a g o n i s t a t d if f e r e n t o p io id


r e c e p to r s e x c e p t n a ltr e x o n e , w h ic h is a p u r e a n ta g o n is t. U s e o f n a ltr e x o n e in th is p a tie n t w h o
h a s b e e n u s in g h e r o i n f o r s u c h a lo n g tim e w ill p re c ip ita te w ith d r a w a l s y m p to m s , w h ic h in c lu d e
b o d y a c h e s , r u n n y n o s e , e x c e s s iv e te a r in g a n d s a liv a tio n , d ia r r h e a , m o o d s w in g s , a n d , in s o m e
c a s e s , h ig h b l o o d p r e s s u r e , ta c h y c a r d ia , a n d in c r e a s e d te m p e r a tu r e . S e v e r ity a n d d u r a tio n o f
th e s e s y m p to m s v a ry .

72. C. K e ta m in e c a u s e s m in im a l to n o r e s p i r a t o r y d e p r e s s io n w h e n u s e d to in d u c e g e n e r a l
a n e s th e s ia . T h e v e n tila to r y r e s p o n s e to c a r b o n d io x id e is m a in ta in e d , a n d th e PaCO2 is u n lik e ly
to in c r e a s e m o r e th a n 3 m m H g . It is a p o te n t v a s o d il a to r o f c e r e b r a l v e s s e ls , a n d p a tie n ts p r o n e
to h a v e in c r e a s e d in tr a c r a n ia l p r e s s u r e (IC P ) m a y s h o w a s u s ta in e d r i s e in IC P a fte r in d u c tio n o f
a n e s th e s ia w ith k e ta m in e d e s p ite n o r m o c a p n ia .
K e ta m in e h a s b r o n c h o d ila to r a c tiv ity a n d is a t le a s t a s e f f e c tiv e a s h a lo th a n e in p r e v e n tin g
e x p e r im e n ta lly in d u c e d b r o n c h o s p a s m in d o g s . It h a s b e e n u s e d in s u b a n e s th e tic d o s e s to tr e a t
b r o n c h o s p a s m in th e o p e r a tin g r o o m a n d IC U . It is r e a d ily m e ta b o liz e d in th e liv e r b y th e
c y to c h r o m e P 4 5 0 s y s te m o f e n z y m e s to f o r m n o r k e ta m in e , w h ic h is o n e - f if th to o n e - th ir d as
p o te n t a s k e ta m in e .
73. B. T r e a tm e n t o f h y p e r te n s io n in a p r e e c la m p tic p a tie n t a im s a t d e c r e a s in g th e r i s k o f c e r e b r a l
h e m o r r h a g e w h ile m a in ta in in g a n d e v e n im p r o v in g tis s u e p e r f u s io n . N itr o p r u s s id e , a p o te n t
v a s o d il a to r o f r e s is ta n c e a n d c a p a c ita n c e v e s s e ls w ith a n im m e d ia te b u t e v a n e s c e n t a c tio n , is
u s e f u l in p r e v e n tin g d a n g e r o u s e le v a tio n s in s y s te m ic a n d p u lm o n a r y a r te r y b l o o d p r e s s u r e
d u r in g la r y n g o s c o p y , a n d is id e a l f o r tr e a tm e n t o f h y p e r te n s iv e e m e r g e n c ie s . Its in f u s io n c a n b e
titr a te d to e ffe c t. L a b e ta lo l a n d h y d r a la z in e c a n b e u s e d to p r o v id e a l o n g e r la s tin g c o n tr o l o f
b l o o d p r e s s u r e b u t m a y n o t b e fa s t e n o u g h in th e ir a c tio n to c o n tr o l a s u d d e n a c u te r i s e o f b lo o d
p r e s s u r e th a t is a s s o c ia te d w ith th is c o n d itio n . M a g n e s iu m is p r i m a r y th e r a p y to p r e v e n t
s e iz u r e s in th is c o n d itio n . It is a s m o o th - m u s c le r e la x a n t a n d h e lp s w ith c o n tr o l o f h ig h b lo o d
p r e s s u r e b u t in its e lf is n o t g o o d e n o u g h to c o n tr o l th e e le v a tio n o f b l o o d p r e s s u r e in
p r e e c la m p s ia . L i s in o p r il is a n a n g io te n s in - c o n v e r tin g e n z y m e in h ib ito r , w h ic h is
c o n tr a in d ic a te d d u r in g p r e g n a n c y b e c a u s e o f th e r i s k o f fe ta l a b n o r m a litie s .

74. B. T h e r e s p o n s e o f th e n e r v e to e le c tr ic a l s tim u la tio n d e p e n d s o n th r e e f a c to r s : th e c u r r e n t


a p p lie d , th e d u r a tio n o f th e c u r r e n t, a n d th e p o s itio n o f th e e le c tr o d e s . T h e s e f a c to r s c a n b e
m o d if ie d in d if f e r e n t w a y s to ta k e a d v a n ta g e o f th e c h a r a c te r is tic f e a tu r e s o f th e
n o n d e p o la r iz i n g n e u r o m u s c u la r b lo c k a d e : fa d e a n d p o s t- te ta n ic f a c ilita tio n w ith h ig h - f r e q u e n c y
s tim u la tio n . W h e n s tim u la tio n is a p p lie d a t a f r e q u e n c y o f > 3 0 H z, th e m e c h a n ic a l r e s p o n s e o f
th e m u s c le is th e f u s io n o f in d iv id u a l tw itc h r e s p o n s e s . In th e a b s e n c e o f n e u r o m u s c u la r -
b lo c k in g d r u g s , n o fa d e is p r e s e n t a n d th e r e s p o n s e is s u s ta in e d . D u r in g n o n d e p o la r iz i n g b lo c k ,
th e r e s p o n s e a c h ie v e s a p e a k a n d th e n fa d e s . H ig h e r th e f r e q u e n c y , m o r e u s e f u l it is to d e te c t
r e s id u a l b lo c k a d e , a lth o u g h s o m e tim e s th e r e m a y b e fa d e a fte r s tim u la tio n w ith 1 0 0 H z in th e
a b s e n c e o f a n e u r o m u s c u la r b lo c k . O n th e o th e r h a n d , w ith tr a in o f f o u r w h e n 2 -H z s tim u la tio n
is u s e d , th e m e c h a n ic a l o r e le c tr ic a l r e s p o n s e d e c r e a s e s little a fte r th e f o u r th s tim u lu s , a n d th e
d e g r e e o f fa d e is s im ila r to th a t f o u n d a t 5 0 H z. P r o b le m w ith tr a in o f f o u r is th e d if f ic u lty to
e v a lu a te b y v is u a l o r ta c tile m e a n s th e d if f e r e n c e b e tw e e n th e h e ig h t o f f i r s t a n d th e f o u r th
tw itc h . I r r e s p e c tiv e o f th e e x p e r ie n c e , it is d if f ic u lt f o r a n e s th e s io lo g is ts to d e te c t tr a in - o f - f o u r
fa d e w h e n a c tu a l t r a in - o f - f o u r r a tio is 0 .4 o r g r e a te r , m e a n in g th e r e b y th a t r e s id u a l p a r a ly s is
m a y g o u n d e te c te d . T h is s h o r tc o m in g m a y b e o v e r c o m e , to s o m e e x te n t b y a p p ly in g tw o s h o r t
te ta n ic s tim u la tio n s ( th r e e im p u ls e s a t 5 0 H z, s e p a r a te d b y 7 5 0 m s ), a n d b y e v a lu a tin g th e r a tio
o f th e s e c o n d to th e f i r s t r e s p o n s e .

75. D. G u i l l a i n - B a r r e s y n d r o m e is th e m o s t c o m m o n c a u s e o f a c u te f la c c id p a r a ly s is . It is a n
a u to im m u n e d is e a s e t r i g g e r e d b y b a c te r ia l o r v i r a l in f e c tio n . P a r a ly s is le a d s to p r o l i f e r a t i o n o f
e x tr a ju n c tio n a l a c e ty lc h o lin e r e c e p to r s w ith r i s k o f h y p e r k a le m ia if s u c c in y lc h o lin e is u s e d .
T h e s e p a tie n ts m a y s h o w a r a n g e o f s e n s itiv ity to n o n d e p o la r iz i n g m u s c le r e la x a n ts f r o m
e x tr e m e s e n s itiv ity to r e s is ta n c e . U s e o f th e s e a g e n ts a lth o u g h is n o t c o n tr a in d ic a te d a s lo n g as
c a u tio n is p r a c tic e d in a s s u r in g r e tu r n o f n o r m a l m u s c u la r p o w e r a t th e e n d o f th e p r o c e d u r e
b e f o r e e x tu b a tio n is p e r f o r m e d . If th e c ir c u m s ta n c e s a llo w , in tr a th e c a l o p io id s a n d e p id u r a l
lo c a l a n e s th e tic s m a y a c tu a lly b e a b e tte r w a y to p r o v id e a n e s th e s ia f o r th e s e p a tie n ts .

76. C. S y m p a th e tic s tim u la tio n a t th e tim e o f in tu b a tio n m a y c a u s e a n e x a g g e r a te d s y m p a th e tic


r e s p o n s e a n d n e e d s to b e c o n s id e r e d in th is s itu a tio n . B lo c k a d e o f a r e c e p to r s b y la b e ta lo l m a y
b e h e lp f u l u n d e r th is c o n d itio n , b u t its c o n c o m ita n t b lo c k a d e o f P r e c e p to r s m a y p o s e p r o b le m .
In te n s e in c r e a s e in th e a f te r lo a d s e c o n d a r y to e x a g g e r a te d r e le a s e o f n o r e p in e p h r in e m a y n o t b e
to le r a te d b y th e h e a r t th a t is in h ib ite d b y la b e ta lo l. D ir e c t v a s o d il a to r s lik e n itr o g ly c e r in e a n d
n i tr o p r u s s id e a r e m o r e s u ita b le f o r th is s itu a tio n th a n a n o n s e le c tiv e P - b lo c k e r a g e n t. A s
k e ta m in e c a n s tim u la te s y m p a th e tic d is c h a r g e , it is n o t a n a p p r o p r ia te in d u c tio n a g e n t f o r th is
p a tie n t. E p h e d r in e m a y a ls o in c r e a s e th e r e le a s e o f n o r e p in e p h r in e f r o m th e s to r e d s ite a n d
c a u s e p r o f o u n d h y p e r te n s io n a n d c a r d ia c d y s r h y th m ia s . P r o p o f o l d o e s n o t h a v e a n y s u c h
a c tio n , a n d it is a p p r o p r ia te f o r u s e in th is c a s e . A s m e n tio n e d e a r lie r , c o c a in e in h ib its r e u p ta k e
o f n o r e p in e p h r in e in to th e p r e s y n a p tic n e r v e , th u s m a k in g it a c c u m u la te in th e s e n e r v e e n d in g s .

77. C. A n a c u te o f e p is o d e o f c y a n o s is in a c h ild w ith h is to r y o f te tr a lo g y o f F a llo t s ig n if ie s m o r e


r ig h t - to - le f t s h u n t b e c a u s e o f d e c r e a s e d s y s te m ic v a s c u la r r e s is ta n c e . P h e n y le p h r in e m a y b e a n
a p p r o p r ia te a g e n t f o r th is s itu a tio n , a s it w ill in c r e a s e th e s y s te m ic v a s c u la r r e s is ta n c e th r o u g h
its a c tio n o n a ! r e c e p to r s , th u s d e c r e a s in g th e s h u n t. A s m o r e b l o o d f lo w s th r o u g h th e lu n g s
b e f o r e r e tu r n in g to th e h e a r t, m o r e o x y g e n w ill b e a v a ila b le to th e p e r ip h e r a l tis s u e s im p r o v in g
th e c y a n o s is .
O x y g e n in th is s itu a tio n m a y n o t b e h e lp f u l a s n o t e n o u g h o f th e b l o o d is f lo w in g th r o u g h
th e lu n g s . O th e r o p tio n s a ls o m a y h e lp tr e a t th e p r i m a r y p r o b le m . C h ild r e n w ith th is c o n d itio n
le a r n to tr e a t th e s e e p is o d e s , c a lle d “ te t s p e ll s ,” b y s q u a ttin g d o w n , th u s in c r e a s in g th e ir
s y s te m ic v a s c u la r r e s is ta n c e a n d v e n o u s r e tu r n to th e h e a rt.

78. C. C lo n id in e h a s b e e n in c lin ic a l u s e f o r o v e r tw o d e c a d e s n o w . It is a v e r y p o te n t
a n tih y p e r te n s iv e a n d is v e r y u s e f u l to c o n tr o l h ig h b l o o d p r e s s u r e in s o m e p a tie n ts w ith a v e r y
r e s is ta n t k in d o f d is e a s e . U n fo rtu n a te ly , o r a l d o s e r e q u ir e s r e p e a te d d o s e s , a n d o m is s i o n o f a
d o s e , a s w ill h a p p e n if a lo n g s u r g ic a l p r o c e d u r e w a s s c h e d u le d , m a y le a d to s e v e r e r e b o u n d
h y p e r te n s io n . A v a ila b ility o f tr a n s c u ta n e o u s p a tc h h a s h e lp e d d e c r e a s e th is p r o b le m . A
d if f e r e n tia l d ia g n o s is in a p a tie n t w ith h y p e r te n s io n in th e r e c o v e r y r o o m s h o u ld a lw a y s in c lu d e
p o s s ib il ity o f r e b o u n d h y p e r te n s io n s e c o n d a r y to o m is s i o n o f a d o s e o f a n a a g o n is t lik e
c lo n id in e .
O th e r a n tih y p e r te n s iv e a g e n ts m e n tio n e d in th e q u e s tio n d o n o t le a d to r e b o u n d
h y p e r te n s io n , a lth o u g h h y p o te n s io n d u r in g a n e s th e s ia a s s o c ia te d w ith p e r io p e r a tiv e u s e o f
d r u g s c a u s in g a n g io te n s in - c o n v e r tin g e n z y m e b lo c k a d e c a n b e d if f ic u lt to m a n a g e .

79. C. P o s to p e r a tiv e n a u s e a a n d v o m itin g (P O N V ) is o n e o f th e m o s t d is lik e d s id e e ffe c ts o f


a n e s th e s ia a n d s u r g e r y . E f f o r ts to tr e a t it s o m e tim e s le a d to tr o u b l e s o m e s id e e ffe c ts w h ic h l o o k
a la r m in g a t th e tim e o f p r e s e n ta tio n b u t a r e e a s y to tre a t. D y s to n ic r e a c tio n s , in c lu d in g ta r d iv e
d y s k in e s ia , t o r t ic o ll is ( c o m m o n ly c a lle d o c u l o g y r i c c r is is ) , d y s p h a g ia , a n d e x c e s s iv e
s a liv a tio n , a r e s o m e o f th e m a n if e s ta tio n s o f th is p s e u d o p a r k in s o n ia n s y n d r o m e .
B u ty r o p h e n o n e s , p h e n o th ia z in e s , g a s tr o in te s tin a l p r o k in e tic s , a n d lith iu m a r e s o m e o f th e
e tio lo g ic a g e n ts .
A lth o u g h p r o m e th a z in e c a n c a u s e th e s e r e a c tio n s , th e in c id e n c e is m u c h lo w e r th a n w ith
m e to c lo p r a m id e . G r a n is e tr o n is a 5 - H T 3- r e c e p to r a n ta g o n is t s im ila r to o n d a n s e tr o n . T h e 5-
H T 3- r e c e p to r a n ta g o n is ts h a v e b e c o m e th e m o s t f r e q u e n tly a d m in is te r e d p r o p h y la x is a n d
tr e a tm e n t f o r P O N V d u e to th e ir e ffic a c y . C e n tra l n e r v o u s s y m p to m s o c c u r in le s s th a n 8 % o f
p a tie n ts tr e a te d w ith th is g r o u p o f d r u g s . I n tr a v e n o u s d ip h e n h y d r a m in e p r o v id e s e x c e lle n t r e l i e f
o f th e s e n e u r o l o g i c s y m p to m s .

80. A. R e p e a te d a d m in is tr a tio n o f k e ta m in e m a y n e c e s s ita te in c r e a s e d d o s e to a c h ie v e th e s a m e


e ffe c t. T h is p h e n o m e n o n c o u ld p a r tly b e e x p la in e d b y th e f a c t th a t c h r o n ic a d m in is tr a tio n o f
k e ta m in e s tim u la te s th e liv e r e n z y m e s th a t m e ta b o liz e it. It e x e r ts its p r i m a r y a n e s th e tic a n d
a n a lg e s ic a c tio n th r o u g h N M D A r e c e p to r , a lth o u g h it is f o u n d to in te r a c t w ith m a n y o th e r
r e c e p to r s in th e c e n tr a l n e r v o u s s y s te m . P r i m a r y s ite o f m e ta b o lis m o f k e ta m in e is liv e r . It is n o t
s ig n if ic a n tly b o u n d to p la s m a p r o te in s a n d th u s is r e a d ily a v a ila b le f o r d is tr ib u tio n in to th e
tis s u e s .

81. C. E to m id a te c a n n o t b e u s e d a s a n in f u s io n b e c a u s e o f its s u p p r e s s iv e a c tio n o n th e a d r e n a l


g la n d ; o th e r w is e , it w o u ld b e a n id e a l s e d a tiv e a g e n t f o r p r o c e d u r e th a t n e e d s m o d e r a te s e d a tio n
a s it d o e s n o t d e p r e s s r e s p i r a t i o n e v e n w h e n u s e d in in d u c tio n d o s e s . It is a p o te n t c e r e b r a l
v a s o c o n s tr ic to r a n d c a u s e s d e c r e a s e in c e r e b r a l m e ta b o lis m a s w e ll a s c e r e b r a l b l o o d flo w . In
c o m p a r is o n to th io p e n ta l it in c r e a s e s th e e x c ita to r y s p ik e s o n th e E E G a n d is a g o o d a g e n t f o r
a n e s th e s ia f o r e le c tr o c o n v u ls iv e th e r a p y a n d m a p p in g f o r s e iz u r e f o c u s if E E G a c tiv ity n e e d s to
b e f a c ilita te d . A n a d m in is te r e d d o s e o f e to m id a te is a lm o s t c o m p le te ly m e ta b o liz e d b y th e liv e r,
a n d v e r y little o f th e p a r e n t m o le c u le is f o u n d in th e u r in e .

82. D. G a b a p e n tin h a s b e c o m e a f ir s t- lin e tr e a tm e n t f o r n e u r o p a th ic p a in . T h is a c tio n is n o t


m e d ia te d th r o u g h G A B A r e c e p to r s a s w o u ld b e e x p e c te d f r o m th e n a m e . In s te a d , it m o d u la te s
v o lta g e - g a t e d c a lc iu m c h a n n e ls w h ic h g e t a c tiv a te d in a n in ju r e d n e r v e . In h ib itio n o f th is
c h a n n e l le a d s to d e c r e a s e d in f lu x o f c a lc iu m in to th e n e r v e c e ll, d e c r e a s in g n e u r o n a l
tr a n s m is s io n r e s p o n s ib le f o r c a u s in g p a in .
G a b a p e n tin h a s n o t b e e n f o u n d to in te r a c t d ir e c tly w ith N M D A r e c e p to r . G a b a p e n tin ’s r o l e
in tr e a tm e n t o f n e u r o p a th ic p a in h a s n o t b e e n lin k e d w ith a n y in te r a c tio n w ith s o d iu m c h a n n e l.

83. D. L ip id s o lu b ility o f a lo c a l a n e s th e tic is m o s t c lo s e ly r e la te d to its p o te n c y , w h ile p K a


d e te r m in e s th e o n s e t o f a c tio n . A s a r e s u lt, m o r e lip id - s o lu b le lo c a l a n e s th e tic s lik e te tr a c a in e
a n d b u p iv a c a in e a r e m o r e p o te n t ( n e e d in g le s s d o s e ) th a n le s s lip id - s o lu b le lo c a l a n e s th e tic s
lik e lid o c a in e . T h e p H a t w h ic h th e c h a r g e d a n d u n c h a r g e d f o r m s o f th e d r u g e x is t in e q u a l
c o n c e n tr a tio n is p K a . A s it is th e u n c h a r g e d f o r m th a t c r o s s e s th e n e u r o n a l m e m b r a n e , lo c a l
a n e s th e tic s w ith a p K a f a r th e r f r o m th e p h y s io lo g ic p H h a v e m o r e o f th e d r u g in io n iz e d
( c h a r g e d ) f o r m d e la y in g th e ir o n s e t o f a c tio n . T h is is n o t th e o n ly d e te r m in a n t o f th e o n s e t o f
a c tio n , th o u g h c h l o r o p r o c a i n e h a s a v e r y s h o r t o n s e t o f a c tio n d e s p ite h a v in g a p K a o f 8.7.
R e a s o n is th a t th e q u a n tity o f th e d r u g in je c te d is so h ig h w ith c h l o r o p r o c a i n e th a t m o r e o f its
m o le c u le s a r e a v a ila b le in its u n c h a r g e d f o r m d e s p ite th e l o w p e r c e n ta g e o f th e d r u g f o u n d in
u n c h a r g e d f o r m . P r o te in - b in d in g o f th e lo c a l a n e s th e tic d e te r m in e s its d u r a tio n o f a c tio n a s th e
r e c e p to r s a r e p r o te in s , a n d a d r u g w ith a h ig h e r a f f in ity f o r p r o te in w ill la tc h o n to th e s e
r e c e p to r s l o n g e r th a n a d r u g w ith le s s a f f in ity f o r p r o te in - b in d in g . In tr in s ic p r o p e r t y o f a d r u g
to c a u s e v a s o d ila ta tio n c a u s e s th e d r u g to g e t a b s o r b e d in th e s y s te m ic c ir c u la tio n , r e d u c in g its
d u r a tio n o f a c tio n a t th e s ite o f in je c tio n c o m p a r e d to a lo c a l a n e s th e tic w h ic h c a u s e s
v a s o c o n s tr ic tio n .
84. A. E to m id a te is a c a r b o x y la te d im id a z o le . It r e s e m b le s m id a z o la m in its p h a r m a c o k in e tic s , in
th a t it is w a te r s o lu b le in a c id ic f o r m a n d w o u ld b e u s e le s s a s a n in d u c tio n a g e n t, e x c e p t th a t it
c h a n g e s its c h a r a c te r is tic s in th e b o d y a n d b e c o m e s lip id s o lu b le o n e x p o s u r e to p h y s io lo g ic
p H . U n lik e p r o p o f o l , it is n o t u s e f u l f o r s e d a tio n o f th e p a tie n ts in th e IC U b e c a u s e o f its a c tio n
o n a d r e n a l g la n d s . It in h ib its 1 1 - ^ - h y d r o x y la s e , th u s c a u s in g in h ib itio n o f c o n v e r s io n o f
c h o le s te r o l to c o r tis o l. P a tie n ts e x p e r ie n c in g s e p s is o r h e m o r r h a g e , a n d w h o m ig h t r e q u i r e a n
in ta c t c o r tis o l r e s p o n s e , w o u ld b e a t r i s k if e to m id a te is a d m in is te r e d to th e m . E v e n a s in g le
d o s e m a y le a d to a p r o l o n g e d d e p r e s s a n t e f f e c t ( 4 - 8 h o u r s ) o n th e a d r e n a l g la n d . A s p r o p o f o l is
a s u b s titu te d is o p r o p y l p h e n o l, it is v e r y d is s im ila r in its c h e m ic a l s tr u c tu r e to e to m id a te . L ik e
o th e r in d u c tio n a g e n ts , a w a k e n in g f r o m a n in d u c tio n d o s e o f e to m id a te is s e c o n d a r y to
r e d is tr ib u tio n o f th e d r u g . L iv e r m e ta b o lis m o f e to m id a te is v e r y c o m p le te b u t n o t fa s t e n o u g h
th a t th a t a c tio n w ill le a d to a w a k e n in g f r o m a n in d u c tio n d o s e o f th is a g e n t.

85. A. C lo p id o g r e l b in d s to A D P r e c e p to r s o n th e s u r f a c e o f p la te le ts . T h is a c tio n le a d s to
in h ib itio n o f a c tiv a tio n , a g g r e g a tio n , a n d d e g r a n u la ti o n o f p la te le ts . C lo p id o g r e l m o d if ie s th e
A D P r e c e p to r ir r e v e r s ib ly , r e s u ltin g in its in h ib itio n f o r th e lif e tim e o f th e p la te le t, w h ic h is u p
to 7 d a y s . S o r e c o m m e n d a tio n f r o m A S R A is to a v o id p e r f o r m i n g a n e u r a x ia l b lo c k f o r 7 d a y s
e v e n if o n ly a s in g le th e r a p e u tic d o s e o f c l o p i d o g r e l w a s u s e d b y th e p a tie n t. C lo p id o g r e l m a y
c a u s e n e u tr o p e n ia , th r o m b o tic th r o m b o c y to p e n ic p u r p u r a , a n d h e p a tic d y s f u n c tio n b u t h a s n o t
b e e n a s s o c ia te d w ith p a n c y to p e n ia .

86. C. N M D A r e c e p to r s h a v e b e e n d o c u m e n te d in th e s p in a l c o r d , a n d e p id u r a l k e ta m in e d o e s
h a v e s o m e a n a lg e s ic a c tio n b u t m u c h le s s th a n m o r p h in e . O f a ll th e a n e s th e tic s a v a ila b le ,
k e ta m in e m a in ta in s th e m u s c le to n e th e m o s t, a n d so it d o e s n o t d e c r e a s e th e d u r a tio n o f a c tio n
o f n o n d e p o la r iz i n g n e u r o m u s c u la r - b lo c k in g a g e n ts . A lth o u g h in v iv o k e ta m in e h a s b e e n s h o w n
to m a in ta in th e s tr o k e v o lu m e , in v itr o s tu d ie s w ith is o la te d m y o c a r d ia l c e ll s h o w th a t k e ta m in e
le a d s to d e c r e a s e in th e f o r c e o f c o n tr a c tio n o f m y o c a r d ia l c e ll. In v iv o f in d in g is e x p la in e d b y
th e a b ility o f k e ta m in e to s tim u la te s y m p a th e tic s y s te m , w h ic h c o u n te r a c ts its m y o c a r d ia l
d e p r e s s a n t a c tio n . T h is a c tio n o f k e ta m in e c a n g e t u n m a s k e d in a p a tie n t w ith c h r o n ic
c o n g e s tiv e h e a r t f a ilu r e , th a t is , a lr e a d y u s in g m a x im u m s y m p a th e tic a c tiv ity to m a in ta in h is o r
h e r c a r d ia c o u tp u t. K e ta m in e d o e s n o t d e c r e a s e th e a m p litu d e o f th e c o r tic a l s e n s o r y - e v o k e d
p o te n tia ls .

87. A. T r u e a l l e r g i c r e a c tio n to a lo c a l a n e s th e tic is v e r y r a r e , b u t is m o r e c o m m o n w ith e s te r


lo c a l a n e s th e tic s c o m p a r e d w ith a m id e lo c a l a n e s th e tic s . P r o c a in e is th e o n ly e s te r lo c a l
a n e s th e tic a m o n g th e c h o ic e s g iv e n in th is q u e s tio n . A ll o th e r s a r e a m id e s . S o it is p r o b a b ly
f r o m p r o c a in e .

88. A. L id o c a in e h a s b e e n in u s e f o r tr e a tin g a m y r i a d o f p a in c o n d itio n s in v o lv in g n e u r o p a th ic


p a in . A lth o u g h p r i m a r y m e c h a n is m o f its a n a lg e s ic a c tio n m a y b e th r o u g h b lo c k a d e o f th e
s o d iu m c h a n n e l, it a c ts o n m a n y o th e r c h a n n e ls in c lu d in g c a lc iu m a n d G - p r o t e i n - c o u p l e d
r e c e p to r s . S o th e m e c h a n is m o f its a n a lg e s ic a c tio n m a y b e m o r e c o m p lic a te d th a n s im p le
b lo c k a d e o f s o d iu m c h a n n e l. A lth o u g h lid o c a in e h a s b e e n f o u n d to in h ib it N M D A r e c e p to r in
s u p r a c lin ic a l le v e ls , a c tiv a tio n o f th is r e c e p to r w o u ld n o t le a d to a n a lg e s ia .
89. D. B e lla d o n n a a lk a lo id s f o u n d in th e n a tu r e s o m e tim e s le a d to m a n if e s ta tio n s o f
a n tic h o lin e r g ic s y n d r o m e . F a c to r s th a t n e e d to b e ta k e n in to c o n s id e r a tio n f o r tr e a tm e n t o f th is
c o n d itio n in c lu d e w h e th e r th e r e is C N S in v o lv e m e n t o r n o t. In th is c a s e , it s e e m s th a t
m a n if e s ta tio n s o f C N S in v o lv e m e n t a r e o b v io u s . N e o s tig m in e , p y r id o s tig m in e , a n d
e d r o p h o n iu m h a v e q u a te r n a r y a m m o n iu m i o n in th e ir c h e m ic a l s tr u c tu r e , m a k in g th e m u n a b le
to c r o s s th e b l o o d - b r a i n b a r r i e r . P h y s o s tig m in e b e in g a t e r tia r y c o m p o u n d is a b le to c r o s s th is
b a r r i e r a n d is a b le to a n ta g o n iz e th e c e n tr a l a c tio n s o f th e s e a n tic h o lin e r g ic a g e n ts .

90. D. M a n y p h y s io lo g ic f a c to r s a n d m e d ic a tio n s a r e a s s o c ia te d w ith p o te n tia tio n o f a c tio n o f


n e u r o m u s c u la r b lo c k e r s . C a lc iu m c h a n n e l b lo c k e r s lik e v e r a p a m il le a d to d e c r e a s e d
s a r c o p la s m ic c o n c e n tr a tio n o f c a lc iu m , w h ic h m a y p o te n tia te th e m u s c le w e a k n e s s a s w e ll as
p r o l o n g th e d u r a tio n o f a c tio n o f n e u r o m u s c u la r - b lo c k in g a g e n ts .
C a r b a m a z e p in e w a s o r i g i n a l l y u s e d a s a n tic o n v u ls a n t b u t h a s b e e n f o u n d to b e u s e f u l in
m a n y o th e r c o n d itio n s . It is n o t a s s o c ia te d w ith p r o l o n g e d a c tio n o f n e u r o m u s c u la r - b lo c k in g
a g e n ts .

M a n y a n tib io tic s , n o ta b ly a m in o g ly c o s id e s , p r o l o n g th e a c tio n s o f th e s e d r u g s , b u t


c lin d a m y c in is s a f e to u s e a n d h a s n o t b e e n s h o w n to h a v e a n y d e le te r io u s e ffe c t w h e n u s e d
c o n c o m ita n tly w ith d r u g s lik e v e c u r o n iu m .

91. C. R e m if e n ta n il is m e ta b o liz e d b y n o n s p e c if ic e s te r a s e s f o u n d in b l o o d a n d tis s u e s . T h is


m e ta b o lis m is so r a p id th a t a c tio n o f r e m if e n ta n il is te r m in a te d w ith o u t th e n e e d f o r
r e d is tr ib u tio n o r h e p a tic e x tr a c tio n . T h e r e is n o a c c u m u la tio n o f th e d r u g in p a tie n ts w ith r e n a l
f a ilu r e . E lim in a tio n h a lf - li f e (6 m in u te s ) is n o t p r o l o n g e d e v e n a fte r a p r o l o n g e d in f u s io n .
H y p o th e r m ia d u r in g c a r d ia c b y p a s s d o e s p r o l o n g its e lim in a tio n tim e u p to 2 0 % .

92. C. D e x m e d e to m id in e is a n a 2- a g o n is t, w h ic h a c ts c e n tr a lly to in h ib it s y m p a th e tic n e r v o u s


s y s te m a c tiv ity . C lo n id in e b e lo n g s to th e s a m e g r o u p o f m e d ic in e s . D e x m e d e to m id in e a c tu a lly
s o m e tim e s is u s e d to tre a t s p a s m o b s e r v e d in c e r e b r a l p a ls y p a tie n ts . Its e ffe c t in o p io id - in d u c e d
r i g i d i t y is n o t w e ll s tu d ie d . D e c a y tim e o f th e s e r u m le v e ls o f d e x m e d e to m id in e a fte r a s h o r t
d u r a tio n o f in f u s io n is v e r y fa st, b u t if it u s e d f o r l o n g e r tim e , f o r e x a m p le , 10 h o u r s , a s it is
s o m e tim e s u s e d in in te n s iv e c a r e u n it s e ttin g , th e s e d a tiv e a c tio n m a y ta k e a lo n g tim e to
d is s ip a te . D e x m e d e to m id in e d e c r e a s e s s y s te m ic v a s c u la r r e s is ta n c e , a n d th a t is o n e o f its
m e c h a n is m b y w h ic h it m a y c a u s e a d e c r e a s e in b l o o d p r e s s u r e .

93. D. M y o to n ia is c h a r a c te r iz e d b y c o n tin u e d in v o lu n ta r y c o n tr a c tio n o f a g r o u p o f m u s c le s . So


o n c e tr i g g e r e d , m u s c le s f a il to r e la x . M y o to n ic d y s tr o p h y is th e m o s t c o m m o n f o r m .
S u c c in y lc h o lin e c a n c a u s e s e v e r e h y p e r k a le m ia a n d is c o n tr a in d ic a te d in th is c o n d itio n .
N o n d e p o la r iz in g m u s c le r e la x a n ts m a y n o t b e a b le to r e v e r s e th is s p a s m . A s n e o s tig m in e c a n
a ls o t r i g g e r m y o to n ic e p is o d e , its u s e f o r r e v e r s a l o f m u s c le r e la x a n ts is n o t in d ic a te d . F o r th a t
r e a s o n , it is p r u d e n t to a v o id u s in g l o n g e r - a c tin g n o n d e p o la r iz i n g a g e n ts th a t m a y r e q u ir e
r e v e r s a l a g e n ts .
H y p o th e r m ia c a n t r i g g e r th is c o n d itio n , so p r e c a u tio n s n e e d to b e ta k e n to a v o id it d u r in g
a n y a n e s th e tic . B e s t tr e a tm e n t to r e lie v e th e s p a s m is to in je c t lo c a l a n e s th e tic in to th e m u s c le .

94. C. A d m in is tr a tio n o f a n tic h o lin e s te r a s e a g e n ts to r e v e r s e th e a c tio n o f n o n d e p o la r iz i n g a g e n t


a llo w s th e a c e ty lc h o lin e le v e ls to b u ild u p n o t o n ly in n e u r o m u s c u la r ju n c tio n o f th e s k e le ta l
m u s c le b u t a ls o a t th e le v e l o f m u s c a r in i c r e c e p to r s . T h a t le a d s to th e p r o b le m o f b r a d y c a r d i a if
n o t e f f e c tiv e ly c o u n te r b a la n c e d b y a d m in is tr a tio n o f a n a n tic h o lin e r g ic a g e n t. F in d in g th e b e s t
c o m b in a tio n o f th e s e a g e n ts is o f g r e a t im p o r ta n c e to a n a n e s th e s io lo g is t. O ld e r
a n tic h o lin e s te r a s e a g e n t e d r o p h o n iu m c a u s e s th e c h o lin e s te r a s e a c tiv ity to in c r e a s e in c a r d ia c
m u s c a r in i c r e c e p to r s , w h ile n e o s tig m in e w a s f o u n d to h a v e a d d itio n a l d ir e c t a c tio n o n th e s e
c a r d ia c r e c e p to r s . D e s p ite th a t f in d in g , n e o s tig m in e - in d u c e d b r a d y c a r d i a is e f f e c tiv e ly
p r e v e n te d b y a d m in is tr a tio n o f a n tic h o lin e r g ic a g e n t c o m p a r e d w ith e d r o p h o n iu m , w h e r e th is
e f f e c t is n o t a s p r e d ic ta b le . A n o th e r f a c to r to c o n s id e r is th e o n s e t o f a c tio n o f th e s e d r u g s .
A tr o p in e a n d e d r o p h o n iu m a r e v e r y fa s t- a c tin g , w h ile n e o s tig m in e a n d g ly c o p y r r o la te a r e
s lo w e r in o n s e t. A s a r e s u lt, if g ly c o p y r r o la te is in je c te d to c o u n te r a c t th e c h o lin e r g ic r e s p o n s e
in d u c e d b y e d r o p h o n iu m , th e c h a n c e o f d e v e lo p m e n t o f b r a d y c a r d i a is g r e a te s t w ith th is
c o m b in a tio n , a s g ly c o p y r r o la te w ill ta k e m u c h l o n g e r a n d m a y n o t r e v e r s e a ll o f th e
c h o lin e r g ic e f f e c t p r o d u c e d b y e d r o p h o n iu m .

95. D. C a r c in o i d s y n d r o m e c a n b e p r e c ip ita te d in th e o p e r a tin g r o o m w h e n th e c a r c in o id tu m o r is


m a n ip u la te d b y th e s u r g e o n . C a u s e is th e r e le a s e o f m a s s iv e a m o u n t o f h o r m o n e s m a d e b y th e
c a r c in o id tu m o r in to th e s y s te m ic c ir c u la tio n . S ig n s a n d s y m p to m s d u r in g a n e s th e s ia m a y
in c lu d e f lu s h in g o f h e a d , n e c k , a n d u p p e r th o r a x , b r o n c h o s p a s m , a n d h y p o o r h y p e r te n s io n .
M a n y c a r c in o id tu m o r s c o n ta in s o m a to s ta tin r e c e p to r s : a g a s tr o in te s tin a l r e g u l a t o r y p e p tid e th a t
r e d u c e s th e p r o d u c tio n a n d r e le a s e o f g a s tr o p a n c r e a tic h o r m o n e . S o m a to s ta tin in f u s io n c a n
a v o id a n d tr e a t th e m a n if e s ta tio n o f th is s y n d r o m e in th e p e r io p e r a tiv e p e r io d . A s y n th e tic
a n a lo g u e : o c tr e o tid e is u s e d m o r e o f te n in c o n te m p o r a r y p r a c tic e . It is r e c o m m e n d e d to b e
s ta r te d 2 w e e k s p r i o r to th e s c h e d u le d s u r g e r y a n d c o n tin u e d in th e p o s to p e r a tiv e p e r io d .
K e ta m in e is n o t in d ic a te d in th is c o n d itio n , a s it m a y in c r e a s e th e s y m p a th e tic n e r v o u s s y s te m
d is c h a r g e a n d w o r s e n th e s itu a tio n . S e v o f lu r a n e is h e lp f u l to tr e a t b r o n c h o s p a s m a c u te ly b u t is
n o t s p e c if ic f o r th is c o n d itio n a n d m a y n o t b e p o te n t e n o u g h to tr e a t b r o n c h o s p a s m in d u c e d b y
th e s e h o r m o n e s . D e x a m e th a s o n e is n o t in d ic a te d f o r th e a c u te tr e a tm e n t o f b r o n c h o s p a s m , a s its
o n s e t o f a c tio n m e a s u r e s in h o u r s a n d n o t in m in u te s .

96. D. T h is p a tie n t s e e m s to b e in h y p o v o le m ic s h o c k . B o d y ’s r e s p o n s e to s u c h a s ta te is to
r e d i r e c t th e in tr a v a s c u la r v o lu m e to v ita l o r g a n s lik e b r a i n a n d h e a r t b y c a u s in g p e r ip h e r a l
v a s o c o n s tr ic tio n th r o u g h a c tiv a tio n o f th e s y m p a th e tic s y s te m . T h io p e n ta l is a k n o w n
v a s o d ila to r . M e c h a n is m o f th is a c tio n in c lu d e s b a r b itu r a te - in d u c e d d e p r e s s io n o f th e m e d u lla r y
v a s o m o t o r c e n te r a n d d e c r e a s e d C N S o u tf lo w f r o m th e C N S . T h is v a s o d ila ta tio n w ill c a u s e
p e r ip h e r a l p o o lin g o f b lo o d . T h is m a y c a u s e c a ta s tr o p h ic d r o p in b l o o d p r e s s u r e , w h ic h th is
p a tie n t m a y n o t b e a b le to to le r a te . M e ta b o lis m o f th io p e n ta l b y th e liv e r is s lo w a n d d o e s n o t
c o n tr ib u te a p p r e c ia b ly to th e te r m in a tio n o f e f f e c t o f th is d r u g . T h io p e n ta l d o e s h a v e n e g a tiv e
i n o tr o p ic e ffe c ts o n th e h e a r t, a lth o u g h th is e f f e c t is n o r m a l l y m a s k e d b y b a r o r e c e p to r - m e d ia te d
re sp o n se s.
97. D. P a tie n ts w ith v a lv u la r h e a r t d is e a s e a r e s o m e tim e s e x tr e m e ly s e n s itiv e to a b r u p t c h a n g e s in
th e h e a r t ra te . H ig h - d o s e fe n ta n y l h a s b e e n s h o w n to c a u s e b r a d y c a r d i a th r o u g h its e f f e c t o n th e
v a g u s n e r v e . T o s o m e e x te n t, th is m a y b e d e s ir a b le in a p a tie n t w ith s te n o tic v a lv u la r le s io n . In
c a s e o f r e g u r g i t a n t le s io n s , th is m a y le a d to c r itic a l d e c r e a s e in c a r d ia c o u tp u t. In o r d e r to
c o u n te r a c t th is b r a d y c a r d i a a c tio n o f th e fe n ta n y l, s o m e a n e s th e s io lo g is ts lik e to u s e a m u s c le
r e la x a n t th a t c a u s e s ta c h y c a r d ia . P a n c u r o n iu m a n d a n o ld e r n o n d e p o la r iz i n g m u s c le - r e la x a n t
g a lla m in e w e r e u s e d f o r th is p u r p o s e . E f f e c t o f p a n c u r o n iu m o n th e h e a r t r a te is e lic ite d a t th e
le v e l o f s in o a t r ia l n o d e b y b lo c k a d e o f th e m u s c a r in ic r e c e p to r s . O n th e o th e r h a n d , e f f e c t o f
fe n ta n y l th a t c a u s e d b r a d y c a r d i a w a s m o r e th r o u g h c e n tr a l n u c le i o f th e v a g u s n e r v e . C a r o tid
b a r o r e c e p t o r s a n d [3 a d r e n e r g i c r e c e p to r a r e n o t in v o lv e d in in d u c tio n o f ta c h y c a r d ia n o tic e d
a fte r a d m in is tr a tio n o f p a n c u r o n iu m .

98. D. D if f e r e n t s u r g ic a l te c h n iq u e s a r e u s e d to s u tu r e th e d o n o r h e a r t to th e c u f f o f th e r e c ip ie n t
h e a r t. In a n y c a s e , d o n o r s in o a t r ia l (S A ) n o d e is s e v e r e d o f its a u to n o m ic n e r v o u s s y s te m
c o n n e c tio n s . A s a r e s u lt, a ll o f th e d r u g s th a t u s e S A n o d e a n d n o r m a l c o n d u c tio n s y s te m o f th e
h e a r t to p r o d u c e th e ir c a r d ia c e ffe c ts f a il to p r o d u c e th e ir e ffe c ts in a tr a n s p la n te d h e a rt.
A lth o u g h oq r e c e p to r s a r e f o u n d in th e h e a r t, th e ir s tim u la tio n w ith p h e n y le p h r in e is n o t
a s s o c ia te d w ith c h r o n o t r o p i c e ffe c ts .
O n ly d r u g s th a t a r e a b le to d ir e c d y s tim u la te th e a d r e n e r g i c r e c e p to r s f o u n d in th e
m y o c a r d iu m a r e u s e f u l in tr e a tin g b r a d y c a r d i a in a tr a n s p la n te d h e a r t. I s o p r o te r e n o l b e in g a
d ir e c t P! s tim u la n t is th e d r u g o f c h o ic e to tr e a t a n e p is o d e o f b r a d y c a r d i a u n d e r th e s e
c ir c u m s ta n c e s .

99. C. P r o p o f o l in f u s io n s y n d r o m e w a s f i r s t d e s c r ib e d in p e d ia tr ic in te n s iv e c a r e u n its , b u t q u ite a


f e w a d u lt c a s e s h a v e b e e n r e p o r t e d n o w . E a r li e s t s ig n o f s y n d r o m e is m e ta b o lic a c id o s is
b e c a u s e o f im p a ir m e n t o x id a tiv e p h o s p h o r y la tio n in th e m ito c h o n d r ia , a n a e r o b ic m e ta b o lis m ,
a n d a c c u m u la tio n o f la c tic a c id . C a r d ia c d y s f u n c tio n is a ls o a n e a r ly s ig n , w h ic h m a n if e s ts e a r ly
o n a s b r a d y c a r d ia , a n d r i g h t - h e a r t b lo c k is v e r y c o m m o n . R h a b d o m y o ly s is is a h a llm a r k o f th is
c o n d itio n , a n d f r e e m y o g l o b i n p r e c ip ita te s in th e r e n a l tu b u le s , le a d in g to k id n e y f a ilu r e . In a n
e s ta b lis h e d c a s e , th e m o r t a lit y r a te is e x tr e m e ly h ig h , in e x c e s s o f 8 0 % . H ig h e r c u m u la tiv e d o s e
o f p r o p o f o l o v e r a r e la tiv e ly l o n g e r p e r i o d o f tim e s e e m s to b e a s s o c ia te d w ith h ig h e r
in c id e n c e o f th is c o n d itio n . T h is h a s le d to a d r o p in th e p o p u la r ity o f th is d r u g in th e p e d ia tr ic
in te n s iv e c a r e s e ttin g .

100. C. G e n e r a lly , c e r e b r a l b l o o d f lo w is r e la te d to c e r e b r a l m e ta b o lic r a te w ith f a c to r s th a t


d e c r e a s e c e r e b r a l m e ta b o lic r a te d e c r e a s e th e c e r e b r a l b l o o d flo w . T h is c o u p lin g e f f e c t is
p r e s e r v e d d u r in g a n e s th e s ia , b u t th e d e g r e e o f th e c o u p lin g m a y b e a lte r e d b y a n e s th e tic a g e n ts
w h e r e in h a la tio n a l a g e n ts h a v e r e la tiv e ly h ig h e r c e r e b r a l b l o o d f lo w f o r a n y d e g r e e o f c e r e b r a l
m e ta b o lic ra te . T h is v a r ia tio n is d if f e r e n t f o r d if f e r e n t in h a la tio n a l a g e n ts a n d a ls o a t d if f e r e n t
d o s e s o f th e s a m e a g e n t. A t 1.5 M A C , th e o v e r a ll e f f e c t o f d e s f lu r a n e is c e r e b r a l v a s o d ila ta tio n
a n d in c r e a s e d b l o o d flo w .
A lth o u g h n itr o u s o x id e h a s b e e n s h o w n to h a v e s p e c ie s - s p e c if ic a c tio n o n c e r e b r a l
v a s c u la tu r e , it is c le a r th a t in h u m a n s , it is a c e r e b r a l v a s o d il a to r w ith p o te n tia l o f in c r e a s in g th e
c e r e b r a l b l o o d f lo w a n d in tr a c r a n ia l p r e s s u r e . P h e n y to in is a n a n tic o n v u ls a n t, w h ic h h a s m a n y
c e n tr a l n e r v o u s s y s te m e ffe c ts b u t h a s n o t b e e n s h o w n to b e a c e r e b r a l v a s o d ila to r . P e n to th a l
d e c r e a s e s c e r e b r a l m e ta b o lic r a te a n d is k n o w n to p r e s e r v e th e n o r m a l c o u p lin g b e tw e e n th e
m e ta b o lis m a n d b l o o d flo w . S o a d m in is tr a tio n o f th is a g e n t w ill d e c r e a s e th e c e r e b r a l b l o o d
flo w .

101. A. A n a p h y la c tic r e a c tio n d u r in g a n e s th e s ia is e s tim a te d to o c c u r b e tw e e n 1 in 5 ,0 0 0 a n d 1 in


2 5 ,0 0 0 c a s e s . N e u r o m u s c u la r - b l o c k in g d r u g s s e e m to b e th e m o s t c o m m o n a g e n ts c a u s in g th is
r e a c tio n . In c id e n c e is fa r to o lo w to d e te r m in e th e r e la tiv e f r e q u e n c y o f o c c u r r e n c e w ith
in d iv id u a l n e u r o m u s c u la r b lo c k e r . L a te x , a n tib io tic s , a n d in d u c tio n a g e n ts a r e m u c h le s s
c o m m o n e tio lo g ic a g e n ts in a n e s th e s ia . O p io id s e x tr e m e ly r a r e l y c a u s e a n a p h y la c tic r e a c tio n s .
Spinal and Epidural Anesthesia
Thomas Halaszynski

1. A spinal neuraxial anesthetic was given 20 minutes earlier to a 28-year-old G3P2 parturient
scheduled for repeat cesarean section. Alcohol swab exam revealed that she has lost
temperature sensation up to T2 level. At what level do you anticipate the block will reach to
provide adequate pain control?
A. T2
B. T3
C. T4
D. T5

2. You have just administered a bolus of 2% lidocaine (25 mL) through an epidural catheter that
has been working well for labor analgesia in preparation for emergency cesarean section for
fetal distress in an otherwise-healthy 35-year-old woman. Shortly after administration of
lidocaine, the patient complains of nausea, and you notice that her heart rate has decreased from
99 to 38 bpm. The most likely cause is
A. Anaphylactic reaction to lidocaine
B. Pneumothorax
C. Epidural level is higher than T4
D. Amniotic fluid embolus

3. Contraindication(s) for neuraxial blockade include(s)


A. Severe aortic stenosis
B. Severe bleeding tendency
C. Existing severe hypotension
D. All of the above

4. During epidural placement using a midline approach, the epidural needle penetrates all the
following anatomical layers, except
A. Ligamentum flavum
B. Subarachnoid membrane
C. Supraspinous ligament
D. Intraspinous ligament
5. Major benefits of a neuraxial block in a Whipple procedure include all the following, except
A. Decreases the incidence of atelectasis
B. Leads to earlier return of GI function
C. Decreases the risk of urinary retention
D. Reduces the risk of pulmonary embolism or deep-vein thrombosis

6. The correct statement for human neuraxial anatomy is


A. Adult spinal cord ends at L2
B. Spinal cord in children ends at L3
C. The dural sac and subarachnoid space in adults end at S1
D. The dural sac and subarachnoid space in children end at S2

7. Blood supply to the human spinal cord includes all of the following, except
A. Blood supply to the spinal cord is from a single anterior spinal artery and two posterior
spinal arteries
B. The anterior spinal artery supplies the anterior two-thirds of the spinal cord, and the
posterior spinal arteries supply the posterior one-third
C. Anterior spinal artery originates from the vertebral artery
D. Posterior spinal artery originates from the posterior cerebral artery

8. The principal site of action of local anesthetics placed into the epidural space is the
A. Spinal cord
B. Nerve roots
C. Epidural space
D. Subarachnoid space

9. Lidocaine and epinephrine are commonly used together when testing epidural anesthesia
because
A. Lidocaine injection (3 mL of 1.5%) intravascularly will induce local anesthetic toxicity
such as perioral numbness
B. Intrathecal injection of epinephrine will result in a high spinal
C. Intrathecal injection of lidocaine can cause a low-level spinal anesthesia with some degree
of motor block
D. Intravascular injection of epinephrine (typically 15 |ig/3 mL) can cause hypertension more
than tachycardia

10. As an adjuvant in epidural anesthesia, epinephrine can


A. Prolong duration of blockade
B. Improve the quality of blockade
C. Decrease the peak plasma levels of local anesthetic concentration
D. All of above
11. Factors that can affect the level of an epidural anesthetic include
A. Patient weight, amount of local anesthetic injected, patient position
B. Patient height, amount of local anesthetic injected, patient position
C. Patient age, amount of local anesthetic injected, patient position
D. B and C

12. Addition of sodium bicarbonate to epidural local anesthetics may accelerate the onset of
blockade with all of the local anesthetics, except
A. Lidocaine
B. Chloroprocaine
C. Mepivacaine
D. Bupivacaine

13. Factors influencing the level of spinal anesthesia achieved include all of the following, except
A. Baricity of anesthetic solution
B. Patient age
C. Volume of anesthetic solution injected
D. Patient gender

14. All of the following factors may influence the spinal level achieved during spinal anesthesia,
except
A. Drug dose
B. Needle direction
C. Patient position at the time and immediately following injection
D. Patient weight

15. Complications from neuraxial blockade may include all of the following, except
A. Radiculopathy
B. Anterior spinal artery syndrome
C. Arachnoiditis
D. Constipation

16. Neuraxial block complications using local anesthetics alone include all of the following, except
A. Post-dural puncture headache
B. Urinary retention
C. Postoperative cognitive dysfunction
D. High spinal anesthesia

17. Spinal anesthesia was performed on a 25-year-old healthy male for ureter stent placement. A
total of 1.5 mL of 5% preservative-free lidocaine in 7.5% dextrose was injected intrathecally
after being mixed with CSF. There was evidence of free CSF flow before and after injection.
T h e s u r g e r y w a s p e r f o r m e d in th e lith o to m y p o s itio n a n d w a s u n e v e n tf u l, b u t th e p a tie n t
c o m p la in e d o f s e v e r e b u tto c k p a in in th e p o s t- a n e s th e s ia c a r e u n it. A n e u r o e x a m w a s n e g a tiv e
f o r s e n s o r y a n d m o to r d e fic its . T h e m o s t lik e ly d ia g n o s is is

A. S p in a l h e m a to m a
B. S p in a l a b s c e s s
C. T r a n s ie n t n e u r o l o g i c a l s y m p to m s
D. R a d ic u lo p a th y

18. Y o u a r e c o n s u lte d a s to w h e n a p a tie n t w o u ld b e a n a p p r o p r ia te c a n d id a te f o r a n e u r a x ia l b lo c k


f o ll o w i n g a d m in is tr a tio n o f th e f o ll o w i n g a n tic o a g u la n t m e d ic a tio n s ( p a tie n t d o e s n o t h a v e a n y
o th e r c o a g u lo p a th ie s , a n d d o e s n o t ta k e o th e r m e d ic a tio n s th a t c o u ld in f lu e n c e c o a g u la tio n ) .
T h e m o s t c o r r e c t s ta te m e n t is

A. L a s t d o s e o f tic lo p id in e (T ic lid ) 7 d a y s a g o
B. L a s t d o s e o f c l o p i d o g r e l (P la v ix ) th is m o r n i n g
C. L a s t d o s e o f a b c ix im a b ( R e o P r o ) 2 4 h o u r s a g o
D. L a s t d o s e o f e p tifib a tid e ( I n te g r ilin ) 12 h o u r s a g o

19. Y o u p e r f o r m e d a n e p id u r a l a n e s th e tic f o r a n e le c tiv e o p e n - a b d o m in a l a n e u r y s m r e p a ir . Y o u a r e


a s k e d to a d v is e th e s u r g e o n w h e n it w o u ld b e c o n s id e r e d s a f e to a d m in is te r in tr a o p e r a tiv e
in tr a v e n o u s h e p a r in :

A. N o t a t a ll
B. O n e h o u r a fte r e p id u r a l p la c e m e n t
C. T w o h o u r s a fte r e p id u r a l p la c e m e n t
D. F o u r h o u r s a fte r e p id u r a l p la c e m e n t

20. A h ig h e r - th a n - e x p e c te d s p in a l le v e l a c h ie v e d o r g r e a te r d e r m a to m a l s p r e a d o f lo c a l a n e s th e tic
c a n b e a s s o c ia te d w ith a ll o f th e f o ll o w i n g c lin ic a l s itu a tio n s , e x c e p t

A. P reg n an cy
B. A s c ite s
C. E ld e r ly
D. F e m a le g e n d e r

21. C o m m o n ly u s e d s p in a l a n e s th e s ia a d ju v a n ts in c lu d e a ll th e f o ll o w i n g , e x c e p t

A. M o r p h in e
B. C lo n id in e
C. D e x m e d e to m id in e
D. E p h e d r in e

22. A 7 5 - y e a r - o ld f e m a le w ith o v a r ia n c a n c e r is s c h e d u le d f o r to ta l a b d o m in a l
h y s te r e c to m y /b ila te r a l s a lp in g o o o p h o r e c to m y a n d tu m o r d e b u lk in g . A th o r a c ic e p id u r a l
a n e s th e s ia w a s p e r f o r m e d u s in g a te s t d o s e o f 1 .5 % lid o c a in e w ith 1 :2 0 0 ,0 0 0 e p in e p h r in e
in je c te d th r o u g h th e e p id u r a l T u o h y n e e d le th a t r e s u lte d in n o e v id e n c e o f a d v e r s e s e q u e la e . A n
epidural catheter was then threaded through the needle followed by evidence of negative
aspiration through the catheter. A total of 10 mL 0.5% bupivacaine was administered through
the epidural catheter. Thirty seconds later, the patient became agitated and complained of
lightheadedness, tinnitus, and feeling faint, but still able to move all of her extremities. Her BP
decreased from 150/70 to 100/45 mm Hg and her HR decreased from 85 to 55 bpm. The patient
maintained spontaneous breathing throughout with an oxygen saturation (Spo2) of 95%. The
most likely diagnosis is
A. Local anesthetic systemic toxicity (LAST)
B. High epidural anesthesia
C. Total spinal anesthesia
D. Anaphylactic reaction

23. All of the following local anesthetic systemic toxicity (LAST) treatment measures should be
performed when caring for a patient who may be experiencing toxicity, except
A. Stop epidural medication administration
B. Support the airway with 100% oxygen
C. Administer intravenous epinephrine according to ACLS protocols
D. Administer an intralipid bolus and continuous infusion

24. You have just placed a lumbar epidural for labor analgesia at L3-L4 interspace in a 34-year-old
G2P1 woman of 39 weeks’ gestation. The patient is 6-feet tall and weighs 300 pounds. Two
hours later, you are called for an emergency cesarean section on the woman. The minimum
amount of 2% lidocaine you would need to administer through the epidural catheter in order to
achieve a T4 level is
A. 5 mL
B. 10 mL
C. 20 mL
D. 30 mL

25. The correct statement regarding caudal anesthesia is


A. Caudal anesthesia is essentially sacral epidural anesthesia
B. Caudal anesthesia can only be performed in pediatric population
C. A caudal anesthesia catheter should be positioned without penetrating the sacrococcygeal
ligament
D. The younger the child, the less likely you are to experience an intrathecal injection

26. You just placed a thoracic epidural in a morbidly obese female (5’1” and 350 lb). You quickly
administer a total of 20 mL 0.5% bupivacaine through the epidural catheter. As you reposition
the patient from the sitting to the supine position, the patient complains of shortness of breath,
bilateral arm weakness, and nausea. Her HR has decreased from a baseline of 98 to 41 bpm, and
her systolic blood pressure has decreased from a baseline of 140s to 70s mm Hg. The most
likely cause is
A. Accidental intravascular injection of local anesthetic
B. Local anesthetic systemic toxicity
C. A high epidural block
D. Anaphylactic reaction to local anesthetic or latex

27. The most likely reason for dyspnea in a patient experiencing the effects of a high neuraxial
blockade is
A. Phrenic nerve palsy when the neuraxial level reaches T3-T5
B. Patient is experiencing an anxiety attack
C. Medullary hypoperfusion
D. Congestive heart failure

28. In the situation of a high spinal anesthetic, which of the following drug is
pharmacodynamically considered the least useful in controlling hypotension
A. Ephedrine
B. Atropine
C. Phenylephrine
D. Epinephrine

29. During performance of lumbar epidural anesthesia for labor analgesia, you experience free-
flowing cerebrospinal fluid (CSF) from the advancing 17G Tuohy epidural needle. The
epidural needle is removed and a second attempt is successfully performed with an epidural
catheter placed at a different level. Which of the following you would not recommend for the
patient to practice in the next 72 hours?
A. Bed rest
B. Fluid restriction
C. Increase caffeine intake
D. Continue with daily stool softener

30. You are consulted by an emergency room (ER) physician to evaluate a patient experiencing a
severe and bilateral retro-orbital headache, described as constant, along with diplopia. The ER
physician also indicated that the patient presented to the ER 2 days prior with fever, chills, and
photophobia when a diagnostic lumbar puncture was performed with a 20G needle. The CSF
study proved negative for meningitis, but now the patient has returned to the ER with
complaints of a severe headache that has failed therapies of bed rest, caffeine, nonsteroidal anti­
inflammatory drugs, and increased fluid intake. The next method of treatment you would
suggest is
A. Repeat the CSF study as one set of negative results is not definitive
B. Recommend opioids for treatment of the headache
C. Recommend performing an epidural blood patch
D. Continue with conservative therapy as it will eventually prevail
31. I n c o r r e c t s ta te m e n t r e g a r d i n g n e u r a x ia l b lo c k a d e is

A. D e r m a to m e le v e l o f a n e s th e s ia a c h ie v e d w ith a s p in a l a n e s th e tic is o f te n m o r e p r e d ic ta b le
th a n f o ll o w i n g a n e p id u r a l b lo c k a d e
B. S p in a l a n e s th e s ia c a n m o r e r a p id ly a n d c o n s is te n tly p r o d u c e d e n s e r m o to r b lo c k a d e th a n
e p id u r a l a n e s th e s ia
C. L o c a l a n e s th e tic s a d m in is te r e d d u r in g e p id u r a l a n e s th e s ia a r e ty p ic a lly m o r e v o lu m e -
d e p e n d e n t, a n d d u r in g s p in a l a n e s th e s ia a r e m o r e c o n c e n tr a tio n - d e p e n d e n t
D. T h o r a c i c e p id u r a l a n e s th e s ia h a s a n in c r e a s e d r i s k o f u r i n a r y r e te n tio n c o m p a r e d to lu m b a r
e p id u r a l a n e s th e s ia w h e n th e s a m e v o lu m e o f lo c a l a n e s th e tic is a d m in is te r e d

32. F o llo w in g p e r f o r m a n c e o f s p in a l a n e s th e s ia a t th e L 4 - L 5 le v e l w ith 3 m L o f 5 % lid o c a in e , y o u


s u s p e c t a p o te n tia l in ju r y to th e c o n u s m e d u lla r is . W h ic h o f th e f o ll o w i n g s y m p to m s is least
lik e ly to b e a s s o c ia te d w ith c a u d a e q u in a s y n d r o m e ?

A. U r in a r y in c o n tin e n c e
B. S a d d le a n e s th e s ia
C. Q u a d r ic e p s w e a k n e s s
D. B ic e p s f e m o r is w e a k n e s s

33. Y o u a r e c a lle d to s e e a 7 6 - y e a r - o ld f e m a le w h o h a d a L 3 - L 4 lu m b a r e p id u r a l p la c e d 3 d a y s
p r i o r f o r p o s to p e r a tiv e a n a lg e s ia f o r a c o le c to m y . T h e e p id u r a l p la c e m e n t w a s tr a u m a tic o n th e
f i r s t a tte m p t (a t L 4 - L 5 le v e l) w ith e v id e n c e o f p o s itiv e b l o o d a s p ir a tio n . T h e p a tie n t is n o w
c o m p la in in g o f n e w o n s e t b a c k p a in w ith r a d ia tio n to th e r i g h t lo w e r e x tr e m ity a n d r i g h t k n e e
w e a k n e s s th a t w a s c o n f ir m e d b y p h y s ic a l e x a m . T h e m o s t lik e ly d ia g n o s is a n d o p tim a l
m a n a g e m e n t is

A. B r e a k th r o u g h p a in in a p a tie n t w h o is c o n f u s e d , tr e a t w ith a d d itio n a l p a in m e d ic a tio n s


B. S ta t M R I o f th e b a c k to r u le o u t n e u r a x ia l h e m a to m a
C. S u r g ic a l c o m p lic a tio n , c o n s u lt o r th o p e d ic s
D. S y m p to m a tic s p in a l s te n o s is , c o n s u lt n e u r o l o g y f o r s u g g e s tio n s
CHAPTER 7 ANSWERS

1. C. In s p in a l a n d e p id u r a l a n e s th e s ia , d if f e r e n tia l b lo c k a d e is f r e q u e n tly r e p o r te d to o b s e r v e th e
“ tw o s e g m e n ts r u l e ,” n a m e ly , s y m p a th e tic b lo c k is tw o s e g m e n ts h ig h e r th a n s e n s o r y b lo c k , a n d
s e n s o r y b lo c k is tw o s e g m e n ts h ig h e r th a n m o to r b lo c k . In th is s p in a l b lo c k , a lc o h o l s w a b te s te d
th e le v e l o f s e n s o r y /s y m p a th e tic b lo c k a d e .

2. C. A l a r g e lo c a l a n e s th e tic b o lu s to a p a r tu r ie n t w ith a n a n tic ip a te d e p id u r a l s p a c e r e d u c e d in


s iz e s e c o n d a r y to e n g o r g e d e p id u r a l v e in s a n d e n la r g e d u te r u s c a n c a u s e a h ig h e r le v e l o f
e p id u r a l b lo c k a d e th a n a n tic ip a te d . If th e b lo c k le v e l r e a c h e s h ig h e r th a n T 4 a n d in f lu e n c e s T 1 -
T 4 ( c a r d ia c a c c e le r a to r f ib e r s ) , p a tie n ts m a y h a v e b r a d y c a r d ia , h y p o te n s io n , a n x ie ty o n p h y s ic a l
e x a m a n d r e p o r t s y m p to m s s u c h a s n a u s e a , v o m itin g , a n d h e a d a c h e , a n d e v e n p a r e s th e s ia in th e
u p p e r e x tr e m itie s .

3. D. N e u r a x ia l b lo c k is a g r e a t a lte r n a tiv e to g e n e r a l a n e s th e s ia f o r m a n y s u r g ic a l p r o c e d u r e s
b e lo w th e d ia p h r a g m a n d a n e x c e lle n t c h o ic e f o r p o s to p e r a tiv e p a in c o n tr o l. H o w e v e r, th e r e a r e
c o n d itio n s w h e r e n e u r a x ia l b lo c k n e e d s to b e u s e d w ith c a u tio n . N e u r a x ia l b lo c k s a r e a s s o c ia te d
w ith a s y m p a th e c to m y a n d c a n th e r e f o r e w o r s e n e x is tin g h y p o te n s io n a n d h y p o v o le m ia .
H y p o te n s io n in c o m b in a tio n w ith a o r tic a n d /o r m itr a l v a lv e s te n o s is m a y n o t b e v e r y w e ll
to le r a te d . A lth o u g h s p in a l/e p id u r a l h e m a to m a is r a r e y e t p o s s ib le , th e r i s k o f b le e d in g is
s ig n if ic a n tly h ig h e r in p a tie n ts w ith a k n o w n c o a g u lo p a th y .

4. B. T o p e r f o r m a n e p id u r a l b lo c k , th e n e e d le p a s s e s th r o u g h s e v e r a l la y e r s , in c lu d in g s k in ,
s u b c u ta n e o u s tis s u e , s u p r a s p in o u s lig a m e n t, in tr a s p in o u s lig a m e n t, a n d lig a m e n t fla v u m . T o
p e r f o r m a s p in a l a n e s th e s ia , th e n e e d le g o e s d e e p e r to p e n e tr a te th e d u r a a n d f r e q u e n tly th e
s u b a r a c h n o id m e m b r a n e .

5. C. N e u r a x ia l b lo c k s in u p p e r a b d o m in a l a n d th o r a c ic p r o c e d u r e s o f f e r a d v a n ta g e s o f
d e c r e a s e d p u lm o n a r y a n d c a r d ia c c o m p lic a tio n s in h ig h - r i s k p a tie n t p o p u la tio n s , p r o m o t e
p e r is ta ls is , a n d r e d u c e c o n d itio n s f o r a h y p e r c o a g u la tio n s ta te p e r io p e r a tiv e ly . H o w e v e r,
u r i n a r y r e te n tio n is o n e o f th e p o te n tia l m a jo r s id e e ffe c ts a s s o c ia te d w ith n e u r a x ia l b lo c k a d e .

6. B. T h e s p in a l c o r d ty p ic a lly e n d s a r o u n d L 1 in a d u lts , a n d a r o u n d L 3 in c h ild r e n . T h is is th e


r e a s o n w h y n e u r a x ia l b lo c k s a r e p e r f o r m e d b e lo w th e s e le v e ls a n d c a r r y a lo w e r r i s k o f d ir e c t
s p in a l c o r d in ju r y . T h e d u r a l s a c a n d s u b a r a c h n o id s p a c e s e n d a t S2 in a d u lts a n d S 3 in c h ild r e n .

7. D. B lo o d s u p p ly to th e s p in a l c o r d is b y o n e a n te r io r s p in a l a r te r y a n d tw o p o s te r io r s p in a l
a r te r ie s . T h e a n te r io r s p in a l a r te r y s u p p lie s th e a n te r io r tw o - th ir d s o f th e s p in a l c o r d , a n d th e
p o s te r io r s p in a l a r te r ie s s u p p ly th e p o s te r io r o n e - th ir d . T h e a n te r io r s p in a l a r te r y is b r a n c h e d
f r o m th e v e r te b r a l a rte ry , a n d th e p o s t e r i o r s p in a l a r te r y a r is e s f r o m th e p o s t e r i o r i n f e r i o r
c e r e b e lla r a rte ry .
8. B. M a jo r s ite o f a c tio n o f n e u r a x ia l b lo c k a d e ta k e s p la c e o n th e n e r v e r o o ts . L o c a l a n e s th e tic s
a c t o n n e r v e r o o t s in th e s u b a r a c h n o id s p a c e in th e c a s e o f a s p in a l b lo c k a d e a n d o n th e n e r v e
r o o t s in th e e p id u r a l s p a c e in th e c a s e o f e p id u r a l a n e s th e s ia .

9. C. A to ta l o f 3 m L o f 1 .5 % lid o c a in e w ith 1 :2 0 0 ,0 0 0 e p in e p h r in e is c o m m o n ly u s e d w h e n
te s tin g f o r e p id u r a l a n e s th e s ia to r u le o u t in tr a th e c a l ( lid o c a in e c a n r e s u lt in s p in a l b lo c k a d e )
a n d /o r in tr a v a s c u la r in je c tio n . I n tr a v a s c u la r in je c tio n o f e p in e p h r in e (1 5 |ig ) c a n r e s u lt in a
tr a n s ie n t in c r e a s e in h e a r t r a te o f 2 0 % o r h ig h e r , w ith in 3 0 s e c o n d s o f in je c tio n a n d w ith o u t
e v id e n c e o f a B P c h a n g e .

10. D. D u r in g e p id u r a l a n e s th e s ia , e p in e p h r in e in th e d o s e o f 5 |ig /m L w ill im p r o v e th e q u a lity o f


a n e p id u r a l a n e s th e tic . A d d itio n a lly , e p in e p h r in e c a n a ls o p r o l o n g b lo c k a d e d u r a tio n , d e la y s
lo c a l a n e s th e tic in tr a v a s c u la r a b s o r p tio n , a n d d e c r e a s e s p e a k p la s m a lo c a l a n e s th e tic
c o n c e n tr a tio n ( s ) .

11. D. It is c u r r e n tly b e lie v e d th a t b o d y w e ig h t a lo n e d o e s n o t in f lu e n c e th e le v e l o f a n e p id u r a l


b lo c k ( a lth o u g h e x tr e m e o b e s ity m a y ). P a tie n t h e ig h t ( v e r te b r a l le v e ls c o v e r e d d e c r e a s e w ith
h e ig h t) a n d a g e ( v e r te b r a l le v e ls c o v e r e d in c r e a s e w ith a g e ) a lo n g w ith lo c a l a n e s th e tic v o lu m e
(a b o u t 1 to 2 m L lo c a l a n e s th e tic m e d ic a tio n p e r s e g m e n t) a n d p a tie n t p o s itio n ( th e o r y o f
g r a v ity ) c a n p la y s ig n if ic a n t r o le s .

12. D. A d d itio n o f a b a s e w ith a c id ic lo c a l a n e s th e tic m e d ic a tio n s w ill in c r e a s e th e a m o u n t o f


u n c h a r g e d lo c a l a n e s th e tic m o le c u le s in je c te d a n d c a n th e r e f o r e in c r e a s e d if f u s io n o f lo c a l
a n e s th e tic m o le c u le s th r o u g h th e lip id la y e r o f th e c e ll m e m b r a n e . H o w e v e r, s o d iu m
b ic a r b o n a te is n o t u s e d w ith b u p iv a c a in e a s it c a n p re c ip ita te in s o lu tio n s o f a p H a b o v e 6.8.

13. D. M a jo r f a c to r s in f lu e n c in g th e le v e l o f s p in a l a n e s th e s ia in c lu d e s b a r ic ity o f lo c a l a n e s th e tic


s o lu tio n , p a tie n t p o s itio n im m e d ia te ly f o ll o w i n g s p in a l b lo c k p la c e m e n t, d r u g d o s e u s e d , s ite o f
in je c tio n , p a tie n t a g e a n d s p in e a n a to m y , p H o f th e C SF, d r u g v o lu m e u s e d , n e e d le o r i f i c e
d ir e c tio n , p a tie n t h e ig h t, a n d p a tie n ts b e in g p r e g n a n t.

14. D. M a jo r f a c to r s in f lu e n c in g th e le v e l o f s p in a l a n e s th e s ia in c lu d e s b a r ic ity o f lo c a l a n e s th e tic


s o lu tio n , p a tie n t p o s itio n im m e d ia te ly f o ll o w i n g s p in a l b lo c k p la c e m e n t, d r u g d o s e u s e d , s ite o f
in je c tio n , p a tie n t a g e a n d s p in e a n a to m y , p H o f th e C SF, d r u g v o lu m e u s e d , n e e d le o r i f i c e
d ir e c tio n , p a tie n t h e ig h t, a n d p a tie n ts b e in g p r e g n a n t.

15. D. C o m p lic a tio n s f r o m n e u r a x ia l b lo c k a d e c a n b e d iv e r s e a n d r a n g e f r o m d e a th , c a r d ia c


a r r e s t, s e iz u r e s , p a r a p le g ia , r a d ic u lo p a th y , a n te r io r s p in a l a r te r y s y n d r o m e , h ig h /to ta l s p in a l
a n e s th e s ia , a r a c h n o id itis , p o s t - d u r a l p u n c tu r e h e a d a c h e , b a c k p a in , e p id u r a l h e m a to m a , e p id u r a l
a b s c e s s , a n d u r i n a r y r e te n tio n . H o w e v e r, th e c o m p lic a tio n r a te s a r e ty p ic a lly l o w a n d m a y e v e n
im p r o v e b o w e l f u n c tio n a n d d e c r e a s e c o n s tip a tio n .

16. C. P o te n tia l c o m p lic a tio n s o f n e u r a x ia l b lo c k a d e c a n b e d iv e r s e a n d r a n g e f r o m d e a th , c a r d ia c


a r r e s t, s e iz u r e s , p a r a p le g ia , r a d ic u lo p a th y , a n te r io r s p in a l a r te r y s y n d r o m e , h ig h /to ta l s p in a l
a n e s th e s ia , a r a c h n o id itis , p o s t - d u r a l p u n c tu r e h e a d a c h e , b a c k p a in , e p id u r a l h e m a to m a , a n d
e p id u r a l a b s c e s s . H o w e v e r, c o m p lic a tio n r a te s a r e l o w a n d p a tie n ts d o n o t ty p ic a lly e x p e r ie n c e
d e lir iu m u n le s s s y s te m ic o p io id a n a lg e s ic s h a v e b e e n u s e d .

17. C. A lth o u g h tr a n s ie n t n e u r o l o g i c a l s y m p to m s a r e u s u a lly s e lf - lim itin g , it c a n b e b o th e r s o m e


to p a tie n ts . T h e e t i o l o g y is m o s tly lik e ly d u e to th e h ig h c o n c e n tr a tio n o f lid o c a in e ; th e r e f o r e ,
5 % lid o c a in e is n o w a v o id e d in s p in a l a n e s th e s ia w h e n p o s s ib le .

18. D. A c c o r d in g to th e A S R A g u id e lin e s , w a itin g p e r i o d f o r th e c o m m o n ly u s e d a n tip la te le t


a g e n ts a r e a s f o llo w s : tic lo p id in e (T ic lid ) 1 4 d a y s , c l o p i d o g r e l (P la v ix ) 7 d a y s , a b c ix im a b
( R e o P r o ) 4 8 h o u r s , a n d e p tifib a tid e ( I n te g r ilin ) 8 h o u r s .

19. B. S u b c u ta n e o u s h e p a r in p r o p h y la x is a t o n c e o r tw ic e d a ily is n o t a c o n tr a in d ic a tio n to


n e u r a x ia l a n e s th e s ia p la c e m e n t o r p r i o r to e p id u r a l c a th e te r r e m o v a l. S y s te m ic h e p a r in
a d m in is tr a tio n c a n b e c o n s id e r e d s a f e if g iv e n 1 h o u r o r l o n g e r f o ll o w i n g n e u r a x ia l b lo c k a d e
a c c o r d in g to th e A S R A g u id e lin e s .

20. D. F a c to r s a s s o c ia te d w ith a d e c r e a s e d C S F v o lu m e in c lu d e p re g n a n c y , l a r g e a b d o m in a l
tu m o r , a s c ite s , a n d th e e ld e r ly , a n d c a n b e a s s o c ia te d w ith a n e x a g g e r a te d s p r e a d o f n e u r a x ia l
lo c a l a n e s th e tic ( v o lu m e a n d a m o u n t o f lo c a l a n e s th e tic in je c te d r e m a in c o n s ta n t).

21. D. A d ju v a n ts a d d e d to n e u r a x ia l lo c a l a n e s th e tic s m a y im p r o v e q u a lity a n d /o r p r o l o n g th e


d u r a tio n o f s p in a l a n e s th e s ia . S o m e c o m m o n ly u s e d a g e n ts in c lu d e th e f o llo w in g : o p io id s s u c h
a s m o r p h i n e a n d fe n ta n y l, a ! a g o n is t s u c h a s e p in e p h r in e a n d a 2 a g o n is ts s u c h as
c lo n id in e /d e x m e d e to m id in e . I n d ir e c t- a c tin g v a s o p r e s s o r s a d d e d to lo c a l a n e s th e tic m ix tu r e s
h a v e n o t b e e n s h o w n to b e e ffe c tiv e .

22. A. A c c o r d in g to A S A c lo s e d - c la im s d a ta b a s e , L A S T is m o r e c o m m o n th a n w h a t is b e in g
f o r m a l l y r e p o r te d . P e r f o r m i n g a te s t d o s e w ith e p in e p h r in e a n d a s p ir a ti o n is n o t a lw a y s 1 0 0 %
e ffe c tiv e . S m a ll a n d in c r e m e n ta l d o s in g o f e p id u r a l m e d ic a tio n s s h o u ld a lw a y s b e c o n s id e r e d as
a n o th e r s a f e ty m e a s u r e to d e c r e a s e th e r is k .

23. C. In L A S T m a n a g e m e n t, s te p s ta k e n to w a r d a d v a n c e d lif e s u p p o r t s till n e e d to b e f o llo w e d


d e s p ite e v id e n c e th a t in tr a lip i d a d m in is tr a tio n is th e d e f in itiv e tre a tm e n t. A d m in is tr a tio n o f
e p in e p h r in e a s w e ll v a s o p r e s s i n in th e tr e a tm e n t o f L A S T s h o u ld b e a v o id e d a s it h a s n o t b e e n
s h o w n to b e a s s o c ia te d w ith im p r o v e d p a tie n t o u tc o m e s .

24. B. In itia l v e r te b r a l le v e l a c h ie v e d w ith e p id u r a l a n e s th e s ia c a n b e v a r ia b le a n d is n o t as


p r e d ic ta b le a s s p in a l a n e s th e s ia . T h e g e n e r a lly a c c e p te d r u le is th a t 1 to 2 m L o f a n
a p p r o p r ia te ly s e le c te d lo c a l a n e s th e tic s h o u ld b e a d m in is te r e d f o r e a c h v e r te b r a l le v e l o f
a n e s th e s ia d e s ir e d in a d u lts .
25. A. C a u d a l a n e s th e s ia is a ty p e o f e p id u r a l a n e s th e s ia p e r f o r m e d in th e s a c r a l r e g i o n j u s t as
lu m b a r e p id u r a l a n e s th e s ia is p e r f o r m e d in th e lu m b a r r e g i o n . C a u d a l a n e s th e s ia c a n a ls o b e
u s e d in a d u lts , b u t m a y b e m o r e d if f ic u lt to p e r f o r m d u e to c a lc if ic a tio n o f th e s a c r o c o c c y g e a l
lig a m e n t. C a u d a l a n e s th e s ia n e e d le /c a th e te r p la c e m e n t m u s t p e n e tr a te th e s a c r o c o c c y g e a l
lig a m e n t in o r d e r to e n te r th e c a u d a l s p a c e . W ith in th e s a c r a l c a n a l, th e d u r a l s a c s to p s a t th e
f i r s t s a c r a l v e r te b r a in a d u lts a n d a p p r o x im a te ly a r o u n d th e th ir d s a c r a l v e r te b r a in in fa n ts ;
th e r e f o r e , th e r i s k o f s p in a l a n e s th e s ia is h ig h e r in y o u n g e r c h ild r e n .

26. C. R a p id in je c tio n o f l a r g e v o lu m e s o f lo c a l a n e s th e tic s e ith e r e p id u r a lly o r in tr a th e c a lly ,


e s p e c ia lly in s h o r t a n d o b e s e p a tie n ts c a n p r e d is p o s e th e m to h ig h e r - th a n - a n tic ip a te d le v e ls o f
n e u r a x ia l a n e s th e s ia . In th is p a r tic u la r s itu a tio n , th e c a r d ia c a c c e le r a to r f ib e r s w e r e a ffe c te d , a n d
th e r e f o r e , th e p a tie n t e x p e r ie n c e d b r a d y c a r d i a a n d h y p o te n s io n .

27. C. A lth o u g h p h r e n ic n e r v e p a ls y m a y c o n tr ib u te to p a tie n t’s e x p e r ie n c e s o f s h o r tn e s s o f b r e a th


a n d a p n e a , th e m o s t lik e ly r e a s o n f o r d y s p n e a f o ll o w i n g a h ig h n e u r a x ia l b lo c k a d e is p e r s is te n t
h y p o te n s io n - in d u c e d b r a in - s te m h y p o p e r f u s io n . T h e r e f o r e , a ir w a y s u p p o r t is n e e d e d a n d
a g g r e s s iv e c o n tr o l o f h y p o te n s io n is im p o r ta n t in th e m a n a g e m e n t o f h ig h n e u r a x ia l b lo c k a d e
e ffe c ts .

28. C. H y p o te n s io n a s s o c ia te d w ith a h ig h s p in a l m a y b e w o r s e n e d a s a r e s u lt o f e ffe c ts o n th e


c a r d ia c a c c e le r a to r f ib e r s a t th e T 1 - T 4 le v e ls . T h e r e f o r e , a v a s o p r e s s o r th a t c a n s im u lta n e o u s ly
in c r e a s e b o th H R a n d B P w o u ld b e th e m o s t id e a l m e d ic a tio n to a d m in is te r . A ll o f th e a b o v e
d r u g s , e x c e p t p h e n y le p h r in e , c a n b e u s e d to tr e a t s e v e r e b r a d y c a r d i a in th e m a n a g e m e n t o f a
h ig h n e u r a x ia l b lo c k a s s o c ia te d w ith a d e c r e a s in g h e a r t ra te .

29. B. In p a tie n ts w h o m a y e x p e r ie n c e a “w e t ta p ” d u r in g p la c e m e n t o f a n e p id u r a l, c o n s e r v a tiv e


th e r a p y s h o u ld in c lu d e b e d r e s t a n d p le n ty o f f lu id in ta k e , in c lu d in g c a f fe in e ; f o o d d ie t lo w in
f ib e r a n d s to o l s o f te n e r s a r e e n c o u r a g e d to p r e v e n t s tr a in in g .

30. C. In itia lly , a p o s t - d u r a l p u n c tu r e h e a d a c h e is ty p ic a lly tr e a te d c o n s e r v a tiv e ly . If th e r e is


in s u f f ic ie n t o r n o e v id e n c e o f s y m p to m a tic im p r o v e m e n t a fte r 2 4 to 4 8 h o u r s , m o s t c lin ic ia n s
m a y c h o o s e to p e r f o r m a n e p id u r a l b l o o d p a tc h ( if n o c o n tr a in d ic a tio n s ) w ith 15 to 2 0 m L o f
a u to lo g o u s b lo o d .

31. D. D u r in g n e u r a x ia l b lo c k a d e , u r i n a r y r e te n tio n is m o s t o f te n d u e to th e lo c a l a n e s th e tic


e ffe c ts o n th e S 2 - S 4 n e r v e r o o ts . O p io id s c a n a ls o a d v e r s e ly a f f e c t b la d d e r fu n c tio n . T h e r e f o r e ,
a lu m b a r e p id u r a l a n e s th e tic h a s a h ig h e r r i s k o f b la d d e r r e f le x in h ib itio n a n d u r i n a r y r e te n tio n
th a n a th o r a c ic e p id u r a l.

32. C. C a u d a e q u in a s y n d r o m e is u s u a lly s e c o n d a r y to n e u r o to x ic e ffe c ts f r o m lo c a l a n e s th e tic s


o n th e s a c r a l n e r v e r o o ts . A ll o f a b o v e s y m p to m s , w ith th e e x c e p tio n o f th e q u a d r ic e p s m u s c le s ,
c o u ld b e e x p la in e d b y th e c a u d a e q u in a s y n d r o m e ( in n e r v a te d b y th e s a c r a l p le x u s ) . Q u a d r ic e p s
m u s c le s a r e in n e r v a te d b y lu m b a r p le x u s a n d lu m b a r n e r v e r o o t s a n d a r e r a r e l y in v o lv e d in th e
cauda equine syndrome.

33. B. Epidural hematoma may present with back pain, focal neurological deficits, and bowel and
bladder dysfunction. If a neuraxial hematoma is suspected, emergent intervention needs to be
taken to confirm diagnosis and then to perform an emergency decompression as soon as
possible to avoid permanent spinal cord/nerve roots injury.
Peripheral Nerve Blocks
Thomas Halaszynski

1. An 85-year-old male is scheduled for a right distal radius and ulnar open reduction interior
fixation at the wrist. Medical history is significant for chronic obstructive pulmonary disease
dependent on 2 L of oxygen, hypertension, diabetes mellitus, and coronary artery disease with a
stent inserted one year ago. Given that the surgeon plans to use a forearm tourniquet, the
regional anesthesia technique that would be most appropriate for this patient is
A. An inter scalene brachial plexus block plus an intercostal brachial nerve block
B. A supraclavicular approach to the brachial plexus plus an intercostal brachial nerve block
C. An infraclavicular block of the brachial plexus at the cords plus an intercostal brachial
nerve block
D. Superficial cervical plexus blockade plus an intercostal brachial nerve block

2. While performing an axillary brachial plexus block, all of the following nerves are spared,
except
A. Musculocutaneous nerve
B. Ulnar nerve
C. Lateral brachial cutaneous nerve
D. Medial brachial cutaneous nerve

3. Contraindications to safely perform peripheral regional anesthesia include all of the following,
except
A. Patients who may not provide absolute cooperation during nerve block placement (mental
retardation) without administration of sedation
B. Patient refusal
C. Severe coagulopathy while anticipating a deep nerve plexus blockade
D. Evidence of infection at injection site

4. While performing a peripheral nerve block in an awake patient, access and/or use of all of the
following should be considered mandatory, except
A. Administer supplemental oxygen
B. Apply standard ASA monitors
C. Access to resuscitation medications and equipment
D. Immediate access to a mechanical ventilator

5. The most correct statement regarding the appropriate use of ultrasound equipment during
performance of regional anesthesia is
A. Higher frequency ultrasound probes are used for deeper penetration
B. High-frequency ultrasound probes provide for higher image resolution
C. Liner array probes are typically used for imaging deeper anatomical structures
D. The curvilinear probe is designed to best image superficial structures

6. Which of the following nerves is typically spared during performance of an interscalene


brachial plexus block?
A. Median
B. Axillary
C. Musculocutaneous
D. Ulnar

7. Following successful performance of a right interscalene block for surgical rotator cuff repair
in a 27-year-old patient with no other medical issues, you are called to the recovery room
(post-anesthesia care unit) 3 hours later to evaluate the patient. The patient’s symptoms include
drooping of the right eyelid, redness of the conjunctiva, and pupillary constriction. The most
likely diagnosis is
A. Spinal anesthesia
B. Subdural injection of local anesthetic
C. Horner syndrome
D. Cerebrovascular accident (CVA)

8. A supraclavicular block of the brachial plexus does not provide consistent surgical anesthesia
for shoulder surgery secondary to potential sparing of which of the following nerve branches
of the brachial plexus?
A. Musculocutaneous and axillary nerve branches
B. Axillary and suprascapular nerve branches
C. Ulnar and axillary nerve branches
D. Suprascapular and supraclavicular nerve branches

9. Performing an infraclavicular approach for brachial plexus blockade would deposit local
anesthetics at which of the following anatomical levels of the plexus?
A. Trunks
B. Divisions
C. Cords
D. Roots
10. A supraclavicular approach for brachial plexus blockade would deposit local anesthetics at
which of the following anatomical levels of the plexus?
A. Branches
B. Trunks/Divisions
C. Cords
D. Roots

11. When performing an axillary block of the brachial plexus for distal upper extremity surgery,
which of the following nerves most often needs to be targeted separately?
A. Ulnar
B. Radial
C. Musculocutaneous
D. Median

12. Anatomical location of the musculocutaneous nerve in the upper forearm is most frequently
found within which of the following muscles?
A. Triceps brachii
B. Biceps brachii
C. Coracobrachialis
D. Brachialis

13. While performing an ultrasound-guided axillary nerve block along with a nerve stimulator,
your needle tip is imaged inferior to the pulsating axillary artery, and you see evidence of
flexion of fourth and fifth digits. The stimulating needle tip is in closest proximity to which of
the following peripheral nerve branches of the brachial plexus?
A. Median
B. Ulnar
C. Musculocutaneous
D. Radial

14. During placement of an ultrasound-guided and nerve stimulator-assisted axillary nerve block,
your needle tip is imaged superiorly to the axillary artery. You also see pronation of the
patient’s forearm. The needle tip is in closest proximity to which of the following branches of
the brachial plexus?
A. Median nerve
B. Axillary nerve
C. Musculocutaneous nerve
D. Interscalene nerve

15. While performing an axillary nerve block by both ultrasound guidance and nerve-stimulator
assistance, the image of your needle tip is seen posterior to axillary artery, and you observe
supination of the forearm. The needle tip is closest to which of the following brachial plexus
n e rv e b ra n c h e s?

A. Infraclavicular nerve
B. Ulnar
C. Intercostal brachial nerve
D. Radial nerve

16. After performing an axillary peripheral nerve block, your ultrasound probe moves to scan
laterally and you see what appears to be an oval and hyperechoic nerve structure within the
belly of the coracobrachialis muscle. When the needle tip is advanced closer to this structure
and the nerve stimulator is activated, you notice that the elbow begins to flex. The most likely
nerve branch that is being stimulated is
A. Median nerve
B. Triceps brachii nerve
C. Musculocutaneous nerve
D. Radial nerve

17. You successfully perform a right supraclavicular nerve block for a right wrist open reduction
interior fixation. You are called to the post-anesthesia care unit 2 hours later because the patient
is complaining of pain on the back of the wrist, which extends distal to the index, middle, and
ring fingers on the dorsal surface of the hand. You consent the patient to perform a terminal
branch nerve block to supplement the initial block. The nerve that would be needed to be
blocked is
A. Median nerve
B. Radial nerve
C. Infraclavicular nerve
D. Interscalene nerve

18. You have just successfully performed a Bier block using 50 mL 0.5% lidocaine for carpal
tunnel release surgery in a 45-year-old male (height, 6 ft; weight, 200 lb). The patient was
sedated with 2 mg of midazolam upon arrival to the OR. Ten minutes following the local
anesthetic placement, the surgeon indicates that the surgery is finished. At the surgeon’s request,
the nurse releases the tourniquet that was placed on the upper arm. The patient soon becomes
agitated, and you notice twitching of the patient’s arms and legs. The most likely diagnosis is
A. Anaphylaxis to midazolam
B. New-onset seizure disorder
C. Allergic reaction to the local anesthetic
D. Local anesthetic systemic toxicity (LAST)

19. A properly performed lumbar plexus block will result in blockade of all the following nerve
branches, except
A. Femoral nerve
B. Lateral femoral cutaneous nerve
C. Obturator nerve
D. Sciatic nerve

20. Electrical nerve stimulation of which of the following nerves will produce quadriceps muscle
contraction?
A. Femoral nerve
B. Sciatic nerve
C. Lateral femoral cutaneous nerve
D. Obturator nerve

21. You have just performed a femoral nerve block in preparation for a tibial plateau fracture
repair using 20 mL 0.5% ropivacaine. Three hours postsurgery in the recovery room, the
patient complains of lateral thigh pain. Was the femoral nerve block a failure and what would
be the most appropriate action?
A. Yes, repeat the femoral nerve block due to a failed block
B. No, repeat the femoral nerve block as the effectiveness of the local anesthetic has worn off
after 4 hours
C. No, the pain expressed is not located within the distribution of the femoral nerve,
supplement with a lateral femoral cutaneous nerve block
D. Yes, the pain is due to a failed femoral block, but do not repeat the block as there exists a
high risk of nerve injury

22. A properly placed psoas compartment block or posterior lumbar plexus block can be
associated with any of the following complications, except
A. Retroperitoneal hematoma
B. Spinal anesthesia
C. Local anesthetic systemic toxicity
D. Sciatic nerve injury

23. You are consulted on an ASA IV patient for a right-ankle surgery. The patient has a known
history of difficult intubation and status post-spinal fusion surgery. The surgeon is requesting
for a peripheral nerve block that will provide for surgical anesthesia. Which of the following
nerves will need to be blocked in order to provide for complete anesthesia during performance
of foot and ankle surgery?
A. Both sciatic and femoral nerve blockade
B. Sciatic nerve block alone
C. Femoral nerve block alone
D. Sciatic, femoral, and obturator nerve blocks

24. All of the following nerves provide sensory innervation to the foot, except
A. Lateral femoral cutaneous nerve
B. Sural nerve
C. D eep p e ro n e a l n e rv e
D. S u p e r f ic ia l p e r o n e a l n e r v e

25. The most correct statement concerning a unilateral paravertebral block is


A. Such a block is always associated with a similar degree of sympathectomy as with an
epidural block
B. Such a block is often associated with a higher serum level of local anesthetic than that
achieved with an intercostal nerve block due to high vascularity
C. It is not likely to be associated with a pneumothorax
D. Such a block may be associated with epidural spread of local anesthetic

26. The most incorrect statement regarding transversus abdominis plane (TAP) block is
A. TAP blocks can provide analgesia following hernia repair surgeries
B. TAP blocks can often alleviate both somatic and visceral pain
C. One potential complication includes liver injury
D. Unilateral TAP blocks never cross over the midline

27. When performing a transversus abdominis plane (TAP) block, the goal is to deposit/inject local
anesthetic between which of the following two muscle layers?
A. External oblique and internal oblique muscles
B. Internal oblique and transversus abdominis muscles
C. Transversus abdominis and external oblique muscles
D. Rectus abdominis and external oblique muscles

28. While performing the popliteal approach for a sciatic nerve block under ultrasound guidance,
you are able to identify the popliteal artery adjacent to two hyperechoic nerve structures that
appear to become one nerve structure upon proximal movement of the ultrasound probe placed
within the popliteal fossa. The correct identity of the two nerve branches is
A. The nerve on the lateral side is the common peroneal nerve, and the nerve on the medial
side is the tibial nerve (combined nerve is the sciatic nerve)
B. The nerve on the lateral side is the sciatic nerve, and nerve on the medial side is the deep
peroneal nerve (combined nerve is the femoral nerve)
C. The nerve on the lateral side is the common tibial nerve, and nerve on the medial is the
superficial peroneal nerve (combined nerve is the sciatic nerve)
D. The nerve on the lateral side is the common posterior tibial nerve, and the nerve on the
medial side is the superficial peroneal nerve (combined nerve is the femoral nerve)

29. The most appropriate statement regarding the function of the saphenous nerve is
A. It serves as both a motor nerve and a sensory nerve
B. It is the motor terminal branch of the femoral nerve
C. It is the sensory terminal branch of the femoral nerve
D. It is a sensory terminal branch of the sciatic nerve
30. An interscalene block will typically deposit the local anesthetic between which of the following
two muscles?
A. Anterior and middle scalene muscles
B. Middle and posterior scalene muscles
C. Anterior and posterior scalene muscles
D. Sternocleidomastoid and anterior scalene muscles

31. A 45-year-old healthy male is scheduled for bilateral elbow open reduction interior fixation
secondary to a motor vehicle accident. Successful bilateral supraclavicular blocks were planned
and performed under ultrasound guidance, with 20 mL 0.5% ropivacaine injected for each
block on each side. In the operating room, the patient is receiving 25 |ig/kg/min of a propofol
infusion and oxygen via a non-rebreather bag. The patient also received 2 mg of midazolam,
but no opioids. Thirty minutes after incision, the patient is experiencing progressive respiratory
depression, and the oxygen saturation decreases from 100% to 85%. The most likely diagnosis
is
A. Local anesthetic systemic toxicity (LAST)
B. Dysfunction of the diaphragm (diaphragm palsy)
C. Methemoglobinemia
D. Aspiration pneumonia

32. The most appropriate treatment for the patient in the above scenario is
A. Methylene blue due to local anesthetic systemic toxicity
B. Flumazenil to antagonize midazolam (oversedation)
C. Endotracheal intubation to provide respiratory support
D. Antibiotics to treat aspiration pneumonia

33. A 56-year-old woman is scheduled for a right total knee replacement. She has a medical history
of hypertension, diabetes mellitus, obesity, and is status post L1-L5 vertebral fusion. The
regional anesthetic technique that will provide her the most optimal perioperative pain
management is
A. A femoral nerve block and an epidural
B. A femoral and proximal sciatic nerve block
C. Both a femoral and popliteal sciatic nerve block
D. A sciatic nerve block and a spinal

34. A 65-year-old female is scheduled for a right total shoulder replacement. Under ultrasound
guidance, you perform a right interscalene nerve block and place a catheter for continuous
local anesthetic infiltration planned for 3 days. One week later, the patient complains of
persistent parasthesia of the entire right arm, including the wrist, hand, and all fingers (from the
shoulder to the fingers). An MRI shows a diffuse swelling of the brachial plexus at the level of
the cords. The most likely diagnosis is
A. Direct nerve injury/trauma from the block needle used
B. Irritation of the brachial plexus at the level of the branches from the continuous peripheral
nerve catheter
C. Surgical trauma/manipulation of the brachial plexus at the level of the cords
D. Local anesthetic toxicity of the brachial plexus at the level of the roots/trunks

35. The foot is supplied mainly by which of the following nerve(s)?


A. Sciatic nerve
B. Obturator and tibial nerves
C. Femoral and lateral femoral cutaneous nerves
D. Saphenous and common peroneal nerves

36. The following local anesthetic medication is associated with the highest risk for cardiovascular
collapse in the event of local anesthetic systemic toxicity (LAST)
A. Lidocaine
B. Bupivacaine
C. Ropivacaine
D. Mepivacaine

37. The most appropriate nerve block for pain management in a patient scheduled for a total hip
replacement is
A. Femoral nerve block
B. Lumbar plexus block
C. Femoral and obturator nerve block
D. Femoral and lateral femoral cutaneous nerve block

38. The femoral nerve provides sensory innervation to the


A. Lower extremity below the knee
B. Anterior and medial thigh
C. Posterior and medial thigh
D. Almost the entire ankle

39. Sciatic nerve blockade provides sensory loss of the


A. Anterior and lateral thigh
B. Posterior thigh and majority of the leg below the knee
C. Medial and posterior thigh
D. Medial leg below the knee

40. You perform a right-side T3-T5 paravertebral blockade for a patient who is to undergo a right
mastectomy with axillary lymph node dissection. Medical history of the patient includes alcohol
abuse and panic attacks. After the surgery in the post-anesthesia care unit, the patient complains
o f a n e w - o n s e t r i g h t - a r m p a r e s th e s ia . V ita l s ig n s r e m a in s ta b le a lo n g w ith s tr o n g a n d e q u a l
u p p e r e x tr e m ity b ila te r a l p u ls e s . T h e m o s t lik e ly d ia g n o s is is

A. S u r g e r y - r e la t e d b r a c h ia l p le x u s n e r v e in ju r y a n d /o r p o s itio n a l in ju r y
B. T h e p a tie n t is e x p e r ie n c in g w ith d r a w a l f r o m a lc o h o l
C. S id e e f f e c ts /c o m p lic a tio n s o f th e p a r a v e r t e b r a l b lo c k o n th e b r a c h ia l p le x u s
D. P a tie n t is h a v in g a p a n ic a tta c k

41. Y o u s u c c e s s f u lly p e r f o r m a n d p la c e a b ila te r a l T 8 c o n tin u o u s p a r a v e r t e b r a l b lo c k c a th e te rs f o r


a n o p e n - p a r t i a l h e p a te c to m y . E ig h te e n h o u r s p o s to p e r a tiv e ly , th e p a tie n t c o m p la in s o f 7 /1 0
p a in . T o im p r o v e p o s to p e r a tiv e a n a lg e s ia , 10 m L o f 0 .2 % r o p iv a c a in e is a d m in is te r e d th r o u g h
e a c h c a th e te r. T w e n ty m in u te s la te r, th e p a tie n t in d ic a te s th a t th e p a in h a s d e c r e a s e d to 4 /1 0 . T h e
m o s t lik e ly a s p e c t o f p a r a v e r t e b r a l b lo c k a d e th a t c a n a c c o u n t f o r th e r e a s o n w h y th e p a tie n t d id
n o t a c h ie v e a p a in - f r e e c o n d itio n is

A. T h e b lo c k le v e l w a s to o h ig h ; it s h o u ld h a v e b e e n p la c e d a t th e T 1 0 le v e l
B. T h e b lo c k le v e l is to o lo w ; it s h o u ld h a v e b e e n p la c e d a t th e T 6 le v e l
C. P a r a v e r te b r a l b lo c k a d e a n a lg e s ia p r o v id e s f o r m o s tly s o m a tic b lo c k a d e a n d d o e s n o t
p r o v id e f o r c o m p le te c o v e r a g e o f v is c e r a l p a in
D. T h e lo c a l a n e s th e tic v o lu m e a d m in is te r e d is to o s m a ll

42. A p a tie n t is to u n d e r g o s u r g e r y to c r e a te a n a r te r io v e n o u s f is tu la f o r h e m o d ia ly s is o n th e
a n te c u b ita l a r e a o f th e r i g h t u p p e r e x tre m ity . Y o u p e r f o r m a r i g h t s u p r a c la v ic u la r b lo c k
u n e v e n tf u lly u s in g 2 0 m L 0 .5 % r o p iv a c a in e . T h e p a tie n t h a s a m e d ic a l h is to r y s ig n if ic a n t f o r
h y p e r te n s io n a n d e n d - s ta g e r e n a l d is e a s e . T h r e e d a y s f o ll o w i n g th e s u r g e r y , th e p a tie n t
c o m p la in s th a t s h e h a s n o s e n s a tio n f r o m th e r i g h t e lb o w to th e tip s o f a ll h e r f in g e r s , b u t s h e
c a n m o v e a ll o f h e r f in g e r s n o r m a lly . T h e m o s t lik e ly e t i o l o g y is

A. N e u r o to x ic ity o f th e tr u n k s /d iv is io n s o f th e b r a c h ia l p le x u s s e c o n d a r y to th e r o p iv a c a in e
B. N e r v e in ju r y s e c o n d a r y to th e r e g i o n a l b lo c k n e e d le u s e d
C. P r o l o n g e d e f f e c t o f th e lo c a l a n e s th e tic s e c o n d a r y to th e p a tie n t’s r e n a l f a ilu r e
D. P o s s ib le s u r g e r y - r e la te d in ju r y a t th e e lb o w th a t m a y w a r r a n t a n e l e c tr o p h y s io lo g y s tu d y

43. W h ile p e r f o r m i n g a n a x illa r y b r a c h ia l p le x u s b lo c k a d e , th e g o a l is to d e p o s it lo c a l a n e s th e tic


m e d ic a tio n s a t w h a t lo c a t io n o f th e b r a c h ia l p le x u s a n d to ta r g e t w h ic h s p e c if ic n e r v e
s tr u c tu r e s ?

A. L e v e l o f th e b r a n c h e s a n d ta r g e tin g th e r a d ia l, m e d ia n , a n d u ln a r p e r ip h e r a l n e r v e s
B. L e v e l o f th e tr u n k s a n d ta r g e tin g th e in te r s c a le n e , r a d ia l, a n d u ln a r p e r ip h e r a l n e r v e s
C. L e v e l o f th e d iv is io n s a n d ta r g e tin g th e s u p r a c la v ic u la r , m e d ia n , a n d r a d ia l p e r ip h e r a l
n e rv e s
D. L e v e l o f th e c o r d s a n d ta r g e tin g th e in f r a c la v ic u la r , u ln a r, a n d r a d ia l p e r ip h e r a l n e r v e s

44. W h ic h o f th e f o ll o w i n g a p p r o a c h e s to b lo c k a d e o f th e b r a c h ia l p le x u s is a s s o c ia te d w ith th e
h ig h e s t in c id e n c e o f a p n e u m o th o r a x ?

A. In te r s c a le n e a n d a x illa r y a p p r o a c h e s
B. Supraclavicular and interscalene approaches
C. Infraclavicular and axillary approaches
D. Axillary and interscalene approaches

45. All of the following medication adjuvants can be used in combination with local anesthetic
solutions during performance of a peripheral nerve blockade to extend the
duration/effectiveness of nerve blockade, except
A. Epinephrine
B. Ketamine
C. Dexamethasone
D. Clonidine

46. While performing a femoral nerve block guided with a nerve stimulator, you observe a strong
sartorius muscle twitch that disappears at 0.2 mA. What does this mean and how should you
proceed further?
A. The stimulating block needle tip is in the correct position, and the local anesthetic can be
injected
B. The needle tip is likely superficial to the femoral nerve, and the block needle needs to be
readjusted (twitch may not be from stimulation of the femoral nerve) prior to local
anesthetic injection
C. Sartorius muscle twitch indicates that the needle tip is in the correct location, but you need
to get closer to the nerve as 0.2 mA stimulus is too high
D. The block needle needs to be repositioned more medially, and a paresthesia must be elicited
prior to local anesthetic injection

47. The trauma team in the ICU did not want a thoracic epidural placed on a trauma patient with
bilateral rib fractures secondary to concerns about the potential hemodynamic instability that
may result. Therefore, both right T7 and left T5 continuous paravertebral catheters were
successfully placed for this patient under ultrasound guidance. Twenty minutes following the
administration of 10 mL of 0.2% ropivacaine administered through each catheter (following
evidence of negative aspiration), the systolic blood pressure dropped by 50 mm Hg. The most
likely diagnosis is
A. Performance of paravertebral blockade creates identical concerns about potential
hemodynamic compromise as do thoracic epidural blocks
B. Local anesthetic toxicity as the paravertebral space is very vascular
C. Possible epidural spread of local anesthetics from either one or both the paravertebral
catheters
D. Venous bleeding into the paravertebral space resulting in large volumes of local anesthetic
absorption from the paravertebral blocks
CHAPTER 8 ANSWERS

1. C. In a p a tie n t w ith s e v e r e p u lm o n a r y c o m p r o m is e , p e r f o r m i n g e ith e r a n in te r s c a le n e o r


s u p r a c la v ic u la r b lo c k o f th e b r a c h ia l p le x u s s h o u ld b e a p p r o a c h e d w ith c a u tio n s e c o n d a r y to th e
in c r e a s e d r i s k o f a n ip s ila te r a l p h r e n ic n e r v e p a ls y . P la c e m e n t o f a n in te r s c a le n e b lo c k f o r w r is t
s u r g e r y m a y a ls o n o t b e o p tim a l a s it m a y n o t e f f e c tiv e ly b lo c k th e u ln a r n e r v e d is tr ib u tio n to
th e w r is t. A s u p e r f ic ia l c e r v ic a l p le x u s b lo c k ( C 1 - C 4 ) w ill n o t e f f e c tiv e ly p r o v id e
a n e s th e s ia /a n a lg e s ia to th e w r is t. B o th in f r a c la v ic u l a r a n d a x illa r y a p p r o a c h e s to th e b r a c h ia l
p le x u s w o u ld b e a p p r o p r ia te f o r w r is t s u r g e r y , a lo n g w ith a r e d u c e d in c id e n c e o f a d v e r s e
e ffe c ts o n th e p h r e n ic n e r v e . I n te r c o s to b r a c h ia l n e r v e b lo c k a d e is a d d e d to c o v e r th e T 2
d e r m a to m e d is tr ib u tio n th a t is n o t in c lu d e d in a p r o p e r l y p e r f o r m e d b r a c h ia l p le x u s b lo c k a n d
w ill c o n tr ib u te to a lle v ia tin g to u r n iq u e t d i s c o m f o r t in th e m e d ia l p o r t i o n o f th e u p p e r a rm .

2. B. T h e m u s c u lo c u ta n e o u s a n d m e d ia l b r a c h ia l c u ta n e o u s n e r v e s b r a n c h f r o m th e b r a c h ia l
p le x u s a t a m o r e p r o x im a l lo c a t io n th a n c a n b e c o n s is te n tly a n e s th e tiz e d w ith a n a x illa r y n e r v e
b lo c k a p p r o a c h o f th e b r a c h ia l p le x u s . T h e r e f o r e , th e s e n e r v e b r a n c h e s n e e d to b e b lo c k e d
s e p a r a te ly if th e y in n e r v a te th e p la n n e d s u r g ic a l a r e a . T h e la te r a l b r a c h ia l c u ta n e o u s n e r v e is a
b r a n c h o f m u s c u lo c u ta n e o u s n e rv e .

3. A. P a tie n t r e f u s a l is a n a b s o lu te c o n tr a in d ic a tio n f o ll o w i n g in f o r m e d c o n s e n t. E v id e n c e o f
a n tic ip a te d in je c tio n - s ite in f e c tio n a n d s e v e r e c o a g u lo p a th y a r e c o n s id e r e d r e la tiv e
c o n tr a in d ic a tio n s , a n d r is k - to - b e n e f it a n a ly s is n e e d s to b e c a r e f u l ly c o n s id e r e d . N o n ­
c o o p e r a tiv e p a tie n ts c a n o f te n p o s e a n in c r e a s e d r i s k to p a tie n t/o p e r a to r sa fe ty , b u t it is n o t a n
a b s o lu te c o n tr a in d ic a tio n to p e r f o r m i n g r e g i o n a l a n e s th e s ia .

4. D. R e g io n a l a n e s th e s ia s h o u ld b e a d m in is te r e d in a m o n ito r e d lo c a t io n w h e r e s ta n d a r d A S A
m o n ito r s . S u p p le m e n ta l o x y g e n a lo n g w ith r e s u s c ita tiv e m e d ic a tio n s a n d e q u ip m e n t s h o u ld b e
r e a d ily a c c e s s ib le a n d im m e d ia te ly a v a ila b le . H o w e v e r, im m e d ia te a c c e s s to a f u n c tio n in g
a n e s th e s ia v e n tila to r is n o t a lw a y s n e c e s s a ry .

5. B. H ig h - f r e q u e n c y u ltr a s o u n d p r o b e s a r e ty p ic a lly m a n u f a c tu r e d w ith a lin e r p r o b e d e s ig n a n d


p r o v id e h ig h im a g e r e s o lu tio n u s e d f o r s u p e r f ic ia l a n a to m ic a l s tr u c tu r e s . L o w - f r e q u e n c y
u ltr a s o u n d p r o b e e q u ip m e n t is ty p ic a lly p r o d u c e d w ith a c u r v ilin e a r p r o b e d e s ig n a n d r e v e a ls a
lo w e r im a g e r e s o lu tio n , b u t is u s e d f o r v is u a liz in g d e e p e r a n a to m ic a l s tr u c tu r e s s e c o n d a r y to
b e tte r p e n e tr a tio n .

6. D. T h e u ln a r n e r v e b r a n c h o r ig i n a te s f r o m th e C 8 - T 1 n e r v e r o o ts . P r o p e r l y p e r f o r m e d
in te r s c a le n e a p p r o a c h to b r a c h ia l p le x u s b lo c k a d e c a n p r o v id e f o r a d e n s e b lo c k a d e o f th e C 5 -
C 7 n e r v e r o o ts /tr u n k s a n d le s s c o n s is te n t b lo c k a d e o f th e C 8 - T 1 n e r v e r o o ts /tr u n k s . T h e r e f o r e ,
a n in te r s c a le n e a p p r o a c h to b lo c k a d e o f th e b r a c h ia l p le x u s f o r d is ta l u p p e r e x tr e m ity s u r g ic a l
p r o c e d u r e s m a y n o t b e th e m o s t id e a l a p p r o a c h .
7. C. A H o r n e r s y n d r o m e ( m io s is , p to s is , a n d a n h id r o s is ) c a n b e c o m m o n ly s e e n f o ll o w i n g a n
in te r s c a le n e b lo c k . T h is s y n d r o m e is o f te n d u e to p r o x im a l tr a c k in g o f lo c a l a n e s th e tic a n d
b lo c k a d e o f th e s y m p a th e tic f ib e r s to th e c e r v ic o th o r a c ic g a n g lio n . In p a tie n ts w h e r e a C V A m a y
a ls o b e w ith in th e d if f e r e n tia l d ia g n o s is , a t h o r o u g h h is to r y a n d n e u r a l e x a m s h o u ld a lw a y s b e
in c lu d e d .

8. B. A s u p r a c la v ic u la r a p p r o a c h to b r a c h ia l p le x u s b lo c k a d e d o e s n o t c o n s is te n tly a n d r e lia b ly
p r o v id e a n e s th e s ia /a n a lg e s ia to th e a x illa r y a n d s u p r a s c a p u la r n e r v e b r a n c h e s . T h e r e f o r e , a
s u p r a c la v ic u la r b lo c k c a n b e u s e d f o r p o s to p e r a tiv e a n a lg e s ia , b u t m a y n o t b e id e a l f o r s u r g ic a l
a n e s th e s ia d u r in g in v a s iv e s h o u ld e r p r o c e d u r e s . S p a r in g o f u ln a r n e r v e d u r in g a
s u p r a c la v ic u la r b lo c k m a y a ls o o c c u r th a t w o u ld n o t p r o v id e e f f e c tiv e a n e s th e s ia f o r
p r o c e d u r e s d is ta l to th e m id - h u m e r u s .

9. C. A t th e in f r a c la v ic u l a r le v e l, th e b r a c h ia l p le x u s f o r m s th r e e c o r d s in r e la tio n to a x illa r y
a r te r y a n d n a m e d a c c o r d in g to th e ir p o s itio n a r o u n d th e a r te r y : m e d ia l, la te r a l, a n d p o s te r io r
c o rd s.

10. B. S u p r a c la v ic u la r b lo c k a d e o f th e b r a c h ia l p le x u s is o f te n r e f e r r e d to a s th e “ s p in a l
a n e s th e s ia ” o f th e u p p e r e x tre m ity . It p r o v id e s a n e s th e s ia o f th e b r a c h ia l p le x u s d is ta l to th e
r o o t s a n d p r o x im a l to th e c o r d s o f th e p le x u s . T h e r e h a s b e e n a n in c r e a s e d p r a c tic e o f
p e r f o r m i n g th e s u p r a c la v ic u la r a p p r o a c h to b lo c k a d e o f th e b r a c h ia l p le x u s s e c o n d a r y to th e
in tr o d u c tio n o f u ltr a s o u n d in to c lin ic a l p r a c tic e a s a n e s th e s io lo g is ts c a n n o w a p p r e c ia te a
d e c r e a s e d in c id e n c e o f p n e u m o th o r a x u n d e r r e a l- tim e u ltr a s o u n d g u id a n c e .

11. C. T h e m u s c u lo c u ta n e o u s n e r v e ty p ic a lly b r a n c h e s o f f m o r e p r o x im a l to th e a x illa r y


a p p r o a c h o f b r a c h ia l p le x u s b lo c k a d e a n d is f r e q u e n tly n o t a d e q u a te ly a n e s th e tiz e d w ith a
tr a d itio n a l a x illa r y b lo c k o f th e p le x u s ( lo c a l a n e s th e tic s a r e d e p o s ite d a r o u n d th e a x illa r y
a r te r y ) . T h e r e f o r e , th e m u s c u lo c u ta n e o u s n e r v e m u s t b e ta r g e te d s e p a r a te ly w h e n p e r f o r m i n g
a n a x illa r y b lo c k o f th e b r a c h ia l p le x u s f o r d is ta l u p p e r e x tr e m ity s u r g e r y .

12. C. A lth o u g h s o m e a n a to m ic a l v a r ia tio n c a n b e f o u n d w ith th e b r a c h ia l p le x u s a t th e le v e l o f


th e a x illa , th e m u s c u lo c u ta n e o u s n e r v e is m o s t c o m m o n ly p o s itio n e d w ith in th e
c o r a c o b r a c h ia lis m u s c le o r b e tw e e n th e b e llie s o f th e b ic e p s a n d c o r a c o b r a c h ia lis m u s c le s .

13. B. S o m e a n a to m ic a l v a r ia tio n c a n e x ist, b u t th e u ln a r n e r v e is f r e q u e n tly p o s itio n e d in f e r i o r


to th e a x illa r y a rte ry . S tim u la tio n o f th e u ln a r n e r v e w ill c a u s e w r is t f le x io n , f le x io n o f th e
f o u r th a n d fif th d ig its , a n d th u m b a d d u c tio n .

14. A. T h e m e d ia n n e r v e is m o s t f r e q u e n tly p o s itio n e d s u p e r io r to th e a x illa r y a r te r y (w ith s o m e


a n a to m ic a l v a r ia tio n s ) . S tim u la tio n o f th e m e d ia n n e r v e w ill c a u s e m u s c le s tim u la tio n , c r e a tin g
w r is t f le x io n , th u m b o p p o s itio n , a n d f o r e a r m p r o n a tio n .

15. D. D e s p ite s o m e a n a to m ic a l v a r ia tio n s w ith in th e n e r v e - b r a n c h d is tr ib u tio n o f th e b r a c h ia l


p le x u s a r o u n d th e a x illa r y a rte ry , th e r a d ia l n e r v e is m o s t f r e q u e n tly p o s itio n e d p o s t e r i o r to
a x illa r y a rte ry . S tim u la to r o f r a d ia l n e r v e w ill in d u c e d ig it/w r is t/ e lb o w e x te n s io n a n d f o r e a r m
s u p in a tio n .

16. C. M u s c u lo c u ta n e o u s n e r v e is f r e q u e n tly f o u n d w ith in c o r a c o b r a c h ia lis m u s c le a n d /o r


b e tw e e n th e b ic e p s a n d c o r a c o b r a c h ia lis m u s c le s . S tim u la tio n o f th e m u s c u lo c u ta n e o u s n e r v e
w ill c h a r a c te r is tic a lly c a u s e e lb o w f le x io n .

17. B. T h e s e n s o r y d is tr ib u tio n o n th e d o r s a l s u r f a c e o f th e h a n d d e s c r ib e d in th e q u e s tio n


m a tc h e s th e in n e r v a tio n p r o v id e d b y th e r a d ia l n e r v e . T h e r e f o r e , a te r m in a l n e r v e b lo c k
a n y w h e r e a lo n g th e d is tr ib u tio n o f th e r a d ia l n e r v e p r o x im a l to th e w r is t w o u ld b e a n
a p p r o p r ia te p la c e to s u p p le m e n t th e in itia l b r a c h ia l p le x u s b lo c k .

18. D. L A S T c a n o c c u r w h e n a l a r g e v o lu m e o f lo c a l a n e s th e tic is a b s o r b e d in to o r d ir e c tly


in je c te d in to th e s y s te m ic c ir c u la tio n . A B ie r b lo c k c a n p r o v id e s u r g ic a l a n e s th e s ia f o r s h o r t
p r o c e d u r e s o f th e e x tre m ity , la s tin g 6 0 m in u te s o r le s s . H o w e v e r, p a tie n ts m a y c o m p la in o f
to u r n iq u e t p a in th a t c a n b e c o m e e v id e n t a s e a r ly a s 2 0 m in u te s f o ll o w i n g b lo c k p e r f o r m a n c e . In
o r d e r to p r e v e n t o r r e d u c e th e in c id e n c e o f L A S T , th e to u r n iq u e t n e e d s to r e m a in in f la te d a n d in
p o s itio n f o r a m in im u m o f 15 to 2 0 m in u te s e v e n if th e s u r g ic a l p r o c e d u r e f in is h e s e a rly . E v e n
a fte r 15 to 2 0 m in u te s h a s e la p s e d , c a u tio u s , in te rm itte n t, a n d s lo w r e le a s e o f to u r n iq u e t is
re c o m m e n d e d .

19. D. T h e th r e e m a jo r n e r v e b r a n c h e s o f th e lu m b a r p le x u s th a t a r e a f f e c te d b y s u c h a b lo c k
in c lu d e f e m o r a l, la te r a l f e m o r a l c u ta n e o u s , a n d o b tu r a to r n e r v e s . S c ia tic n e r v e o r ig i n a te s f r o m
th e s a c r a l p le x u s a n d is n o t p a r t o f th e lu m b e r p le x u s .

20. A. T h e f e m o r a l n e r v e p r o v id e s m o to r s u p p ly to th e q u a d r ic e p s m u s c le s a n d s e n s o r y s u p p ly to
p o r t i o n o f th e m e d ia l th ig h . T h e f e m o r a l n e r v e d o e s n o t h a v e a n y m o to r c o m p o n e n ts b e lo w th e
k n e e ( o n ly a s e n s o r y b r a n c h , s a p h e n o u s n e r v e , b e lo w th e k n e e ).

21. C. T h e la te r a l f e m o r a l c u ta n e o u s n e r v e s u p p lie s th e la te r a l p o r t i o n o f th e th ig h . B lo c k a d e o f
th e la te r a l f e m o r a l c u ta n e o u s n e r v e is n o t a lw a y s c o n s is te n tly b lo c k e d w ith f e m o r a l n e r v e b lo c k
a p p r o a c h , b u t c a n b e b lo c k e d s e p a r a te ly if /w h e n n e e d e d .

22. D. A lu m b a r p le x u s b lo c k is c o n s id e r e d a d e e p b lo c k a n d h a s b e e n d e s c r ib e d a s a n a d v a n c e d
b lo c k in r e g i o n a l a n e s th e s ia . S o m e p o te n tia l c o m p lic a tio n s in c lu d e r e t r o p e r ito n e a l h e m a to m a ,
lo c a l a n e s th e tic s y s te m ic to x ic ity , in tr a th e c a l a n d /o r e p id u r a l in je c tio n s o f lo c a l a n e s th e tic s , a n d
r e n a l in ju r y (w ith p o te n tia l f o r s u b s e q u e n t h e m a to m a ) . T h e ty p ic a l a p p r o a c h f o r lu m b a r p le x u s
b lo c k a d e s h o u ld n o t c a u s e in ju r y to th e s c ia tic n e r v e u n le s s a n i m p r o p e r l y p la c e d o r
m is d ir e c te d r e g i o n a l b lo c k n e e d le is p o s itio n e d to o c a u d a d th a t c o u ld th e n r e s u lt in in ju r y to
s a c r a l p le x u s a n d th e s c ia tic n e rv e .

23. A. F o r c o m p le te s u r g ic a l a n e s th e s ia o f th e f o o t a n d a n k le , b o th s c ia tic a n d f e m o r a l/s a p h e n o u s


n e r v e s n e e d to b e a n e s th e tiz e d /b lo c k e d . T h e o b tu r a to r n e r v e d o e s n o t p r o v id e s e n s o r y o r m o to r
n e r v e d is tr ib u tio n to f o o t o r a n k le .

24. A. A n a n k le b lo c k c a n b e p e r f o r m e d b y p r o v id i n g a n e s th e s ia a n d b lo c k in g th e fiv e n e r v e s th a t
in n e r v a te th e f o o t, n a m e ly , th e s u p e r f ic ia l a n d d e e p p e r o n e a l n e r v e , s a p h e n o u s n e r v e , s u r a l
n e r v e , a n d p o s te r io r tib ia l n e rv e .

25. D. A d v a n ta g e s o f p r o p e r l y p la c e d p a r a v e r t e b r a l n e r v e b lo c k s in c lu d e r e d u c e d d e g r e e s o f
lo c a l a n e s th e tic - in d u c e d s y m p a th e c to m y c o m p a r e d to e p id u r a l o r s p in a l a n e s th e s ia a n d a lo w e r
r i s k o f lo c a l a n e s th e tic s y s te m ic to x ic ity a s c o m p a r e d w ith in te r c o s ta l n e r v e b lo c k s . H o w e v e r,
o n e o f th e m a jo r c o n c e r n s f o r p o te n tia l c o m p lic a tio n s is d e v e lo p m e n t o f a p n e u m o th o r a x , a n d
p a r a v e r t e b r a l b lo c k s c a n b e a s s o c ia te d w ith v a r ia b le d e g r e e s o f lo c a l a n e s th e tic e p id u r a l s p re a d ,
e s p e c ia lly w h e n p la c in g b ila te r a l p a r a v e r t e b r a l b lo c k s .

26. B. T A P b lo c k s c a n p r o v id e a n a lg e s ia f o r p e r ip h e r a l s o m a tic p a in o f th e a b d o m e n a n d c a n b e
a s s o c ia te d w ith a l o w y e t p o te n tia l r i s k o f b o w e l p e r f o r a t i o n a n d liv e r in ju r y . F o r m id lin e
v e n tr a l h e r n ia s u r g e r y , p e r f o r m i n g b ila te r a l T A P b lo c k s a r e o f te n n e e d e d . T A P b lo c k s d o n o t
c o v e r c ra p p y , v is c e r a l p a in .

27. B. T h e s u b c o s ta l (T 1 2 ), ilio in g u i n a l (L 1 ), a n d i l i o h y p o g a s tr ic (L 1 ), a n d g e n ito f e m o r a l n e r v e s


a r e ta r g e te d w h e n p e r f o r m i n g a T A P b lo c k . T h e s e n e r v e s h a v e a ty p ic a l d is tr ib u tio n b e tw e e n th e
in te r n a l o b liq u e a n d tr a n s v e r s u s a b d o m in is m u s c le s .

28. A. P o p lite a l a p p r o a c h to th e s c ia tic n e r v e b lo c k is ty p ic a lly p e r f o r m e d a t th e s ite o f


b if u r c a tio n o f th e tib ia l ( m e d ia l p o s itio n ) a n d c o m m o n p e r o n e a l ( la te r a l p o s itio n ) n e r v e s . T h e
s c ia tic n e r v e is m o s t o p tim a lly b lo c k e d w ith lo c a l a n e s th e tic a t th e u n io n ( b if u r c a tio n ) o f th e s e
tw o n e r v e s th a t f r e q u e n tly b e c o m e o n e n e r v e s tr u c tu r e a p p r o x im a te ly 7 to 10 c m p r o x im a l to
th e p o p lite a l c r e a s e .

29. C. T h e s a p h e n o u s n e r v e is a te r m in a l s e n s o r y n e r v e b r a n c h o f th e f e m o r a l n e r v e w ith N O
m o to r c o m p o n e n ts . U n d e r c e r ta in c lin ic a l s itu a tio n s , th e s a p h e n o u s n e r v e is p r e f e r e n tia lly
b lo c k e d to a v o id m o to r b lo c k a d e o f th e a n te r io r q u a d m u s c le s th a t c a n r e s u lt f r o m p e r f o r m a n c e
o f a f e m o r a l n e r v e b lo c k ( in c r e a s e d r i s k o f f a ll).

30. A. T h e b r a c h ia l p le x u s n e r v e r o o t / t r u n k is u s u a lly p o s itio n e d b e tw e e n th e a n te r io r a n d m id d le


s c a le n e m u s c le s . W h e n lo c a l a n e s th e tic s a r e p la c e d b e tw e e n th e s e tw o m u s c le b u n d le s , it is
c o m m o n ly r e f e r r e d to a s a n in te r s c a le n e b lo c k .

31. B. B ila te r a l s u p r a c la v ic u la r b lo c k a d e c a n s ig n if ic a n tly in c r e a s e th e r i s k o f s y m p to m a tic


p h r e n ic n e r v e p a ls y . M e th e m o g lo b in e m ia c a n h a p p e n in p a tie n ts w ith c e r ta in lo c a l a n e s th e tic s ,
b u t u s u a lly n o t f r o m r o p iv a c a in e a d m in is tr a tio n . L A S T c a n o c c u r f r o m a d m in is tr a tio n o f to x ic
d o s e s o f a n y lo c a l a n e s th e tic , b u t is m o s t o f te n a n a c u te e v e n t f r o m s y s te m ic a d m in is tr a tio n .
32. C. T h e w itn e s s e d r e s p i r a t o r y d e p r e s s io n is m o s t lik e ly d u e to d ia p h r a g m p a ls y a n d th e u r g e n t
n e e d f o r v e n tila tio n a s s is ta n c e u n til r e s o l u t i o n o f p h r e n ic n e r v e d y s fu n c tio n . A n a p p r o p r ia te
o p tio n w o u ld b e to in tu b a te th e p a tie n t a n d p r o v id e a n y n e c e s s a r y s e d a tio n a n d th e n e x tu b a tio n
u p o n e v id e n c e o f r e c o v e r y o f d ia p h r a g m fu n c tio n .

33. B. F e m o r a l a n d p r o x im a l s c ia tic n e r v e b lo c k to g e th e r c a n o f te n p r o v id e f o r e x c e lle n t


p e r io p e r a tiv e p a in c o n tr o l a n d c a n f a c ilita te p h y s ic a l th e r a p y w ith a r e d u c e d in c id e n c e o f
in te r f e r e n c e w ith a m b u la tio n . T h e s e p e r ip h e r a l r e g i o n a l te c h n iq u e s c a n b e p a r tic u la r ly u s e f u l in
p a tie n ts w ith d if f ic u lty o r c o n tr a in d ic a tio n s to n e u r a x ia l b lo c k a d e .

34. C. S h o u ld e r s u r g e r y is o n e o f th e u p p e r e x tr e m ity p r o c e d u r e s th a t c a n o f te n b e a s s o c ia te d
w ith n e r v e in ju r ie s s e c o n d a r y to p a tie n t p a th o lo g y , s u r g ic a l m a n ip u la tio n ( s ) , s u r g ic a l tra u m a ,
b r a c h ia l p le x u s n e r v e s tr e tc h in g o r c o m p r e s s io n , etc. If s u c h a n in ju r y w a s d u e to p e r f o r m a n c e
o f th e p e r ip h e r a l n e r v e b lo c k a n d /o r c a th e te r p la c e m e n t, it o f te n te n d s to in v o lv e m o r e is o la te d
n e r v e r o o ts /tr u n k s o f th e b r a c h ia l p le x u s f r o m th e in te r s c a le n e a p p r o a c h r a th e r th a n d if f u s e
in f lu e n c e s a t m o r e d is ta l le v e ls o f th e p le x u s . S u r g ic a l c o m p lic a tio n s o f th e b r a c h ia l p le x u s
o f te n te n d to b e m o r e d if f u s e a n d le s s s e le c tiv e . N e r v e - c o n d u c tio n s tu d ie s a n d E M G s h o u ld b e
c o n s id e r e d r a th e r th a n m e r e ly d e lin e a tin g a n e t i o l o g y o f th e in ju r y .

35. A. T h e s c ia tic n e r v e s u p p lie s a ll o f th e m o to r in n e r v a tio n a n d th e m a j o r i t y o f th e s e n s o r y


in n e r v a tio n to th e lo w e r e x tr e m itie s b e lo w k n e e e x c e p t th e m e d ia l s id e o f th e lo w e r e x tr e m ity
th a t is in n e r v a te d b y th e s a p h e n o u s n e rv e .

36. B. B u p iv a c a in e is b e s t k n o w n f o r its h ig h c a r d io v a s c u la r to x ic ity , a lth o u g h a n y o f th e lo c a l


a n e s th e tic m e d ic a tio n s lis te d a b o v e c a n r e s u lt in L A S T . O n e o f th e r e p o r te d a d v a n ta g e s o f
r o p iv a c a in e o v e r b u p iv a c a in e is its r e la tiv e ly lo w e r e d in c id e n c e o f c a r d io v a s c u la r to x ic ity . T h e
o th e r lis te d lo c a l a n e s th e tic m e d ic a tio n s te n d to h a v e n e u r o l o g i c a l to x ic ity p r i o r to p r o g r e s s i n g
to w a r d c a r d io v a s c u la r c o lla p s e .

37. B. A f e m o r a l b lo c k f o r h ip s u r g ic a l p r o c e d u r e s h a v e in tr in s ic lim ita tio n s a s d o e s n o t


c o m p le te ly c o v e r A L L d e r m a to m e d is tr ib u tio n s o f th e h ip . A p r o p e r l y p la c e d a n d f u n c tio n in g
lu m b a r p le x u s b lo c k a d e /c a th e te r w ill c o v e r th e f e m o r a l, o b tu r a to r , a n d la te r a l f e m o r a l
c u ta n e o u s n e r v e a n d o f te n p r o v id e s f o r b e tte r p a in c o n tr o l o f th e h ip in c o n ju n c tio n w ith a
s c ia tic /s a c r a l n e r v e p le x u s b lo c k .

38. B. T h e f e m o r a l n e r v e p r o v id e s s e n s o r y in n e r v a tio n to th e a n te r io r a n d m e d ia l th ig h a b o v e th e
k n e e , a n d m e d ia l s id e o f th e lo w e r e x tr e m ity b e lo w th e k n e e . T h e f e m o r a l n e r v e in n e r v a te s a n d
s u p p lie s m o to r c o n tr o l o f th e a n te r io r q u a d r ic e p s m u s c le s a b o v e th e k n e e a n d n o m o to r
in n e r v a tio n b e lo w th e k n e e .

39. B. S c ia tic n e r v e b lo c k a d e p r o v id e s s e n s o r y lo s s to th e p o s t e r i o r th ig h b y b lo c k in g th e
p o s te r io r c u ta n e o u s n e r v e a lo n g w ith e v e r y th in g b e lo w th e k n e e , e x c e p t f o r th e m e d ic a l lo w e r
le g , w h ic h is in n e r v a te d b y th e s a p h e n o u s n e rv e .
40. A. T h e m o s t lik e ly c a u s e is s e c o n d a r y to a x illa r y ly m p h n o d e d i s s e c tio n - r e la te d b r a c h ia l
p le x u s in ju r y . T h e le v e l o f p a r a v e r t e b r a l b lo c k s w a s a t T 3 - T 5 ; th e r e f o r e , th e b r a c h ia l p le x u s
s h o u ld n o t b e a ffe c te d ( C 4 - T 1 ) b y th e p a r a v e r t e b r a l- in je c te d lo c a l a n e s th e tic .

41. C. P a r a v e r te b r a l b lo c k a d e p r o v id e s m o s tly f o r s o m a tic - in d u c e d p a in w ith little v is c e r a l p a in


c o v e r a g e ; th e r e f o r e , h e p a te c to m y p a tie n ts n e e d a d d itio n a l p a in - m a n a g e m e n t m o d a litie s s u c h as
o p io id s .

42. D. N e u r o lo g ic in ju r ie s s e c o n d a r y to p o s itio n a l, c o m p r e s s io n a l, is c h e m ic in ju r y o f te n c re a te s
a m o r e d if f u s e ty p e o f a n in ju r y p a tte r n s im ila r to th e o n e d e s c r ib e d in th e q u e s tio n . If th e
n e u r o l o g i c in ju r y w e r e d u e to c o m p lic a tio n s f r o m p la c e m e n t o f a s i n g le - s h o t s u p r a c la v ic u la r
b lo c k a d e o r lo c a l a n e s th e tic u s e d d u r in g b lo c k p la c e m e n t, th e n th e s e ty p e s o f in ju r ie s w o u ld
te n d to h a v e a m o r e is o la te d p a tte rn . P e r ip h e r a l n e r v e b lo c k in ju r ie s f r o m a s u p r a c la v ic u la r
b lo c k w o u ld b e m o r e lik e ly to r e s u lt in e v id e n c e o f a n in ju r y p a tte r n is o la te d to th e tr u n k s o r
d iv is io n s o f th e b r a c h ia l p le x u s , a n d th e p a tie n t w o u ld ty p ic a lly r e v e a l s y m p to m s a b o v e e lb o w
a s w e ll. W ith o u t a n y a d ju v a n t, r o p iv a c a in e b lo c k w ill n o t la s t a s lo n g a s 72 h o u r s .

43. A. A x illa r y b lo c k is ty p ic a lly p e r f o r m e d a t th e le v e l o f th e in d iv id u a l p e r ip h e r a l n e r v e


b r a n c h e s o f th e b r a c h ia l p le x u s , s p e c if ic a lly th e r a d ia l, m e d ia n , a n d u ln a r n e r v e s .

44. B. S u p r a c la v ic u la r a p p r o a c h to b lo c k a d e o f th e b r a c h ia l p le x u s c a r r ie s a h ig h r i s k o f
p n e u m o th o r a x f o ll o w e d b y th e in te r s c a le n e a p p r o a c h . T h is p n e u m o th o r a x r i s k h a s d e c r e a s e d
a n d is b e lie v e d to b e s e c o n d a r y to th e m o r e f r e q u e n t u s e o f u ltr a s o u n d - g u id e d r e g i o n a l
a n e s th e s ia . N o w th e s u p r a c la v ic u la r a p p r o a c h to b lo c k a d e o f th e b r a c h ia l p le x u s is c o m m o n ly
p e r f o r m e d w ith u ltr a s o u n d g u id a n c e .

45. B. A ll o f th e a b o v e a d ju v a n t m e d ic a tio n s , e x c e p t k e ta m in e , a r e c o m m o n ly u s e d in p e r ip h e r a l
n e r v e b lo c k s to im p r o v e th e d e n s ity a n d p r o l o n g th e d u r a tio n o f n e r v e b lo c k a d e e ffic a c y .
K e ta m in e , a lo n g w ith e p h e d r in e , w h e n m ix e d w ith lo c a l a n e s th e tic s d u r in g a p e r ip h e r a l n e r v e
b lo c k h a s b e e n s tu d ie d in a n im a l m o d e ls a n d w a s d e e m e d to o f f e r little to n o a d d itio n a l b e n e f its
o r s y n e r g is tic e ffe c ts .

46. B. S a r to r iu s m u s c le tw itc h c o u ld b e s e c o n d a r y to s tim u la tio n o f a s m a ll b r a n c h f r o m th e


f e m o r a l n e r v e th a t in n e r v a te s th e s a r to r iu s m u s c le o r s e c o n d a r y to d ir e c t m u s c le s tim u la tio n .
T h e f e m o r a l n e r v e is u s u a lly p o s itio n e d m o r e la te r a l a n d d e e p e r to th is s m a ll b r a n c h th a t
o r ig i n a te s f r o m th e f e m o r a l n e r v e w h ic h in n e r v a te s th e s a r to r iu s m u s c le .

47. C. A p o te n tia l a d v a n ta g e o f p a r a v e r t e b r a l b lo c k a d e c o m p a r e d to n e u r a x ia l b lo c k a d e is a
r e d u c e d in c id e n c e o f c r e a tin g a n in te n s e s y m p a th e c to m y r e s u ltin g in h e m o d y n a m ic
c o m p r o m is e . H o w e v e r, w h e n b ila te r a l p a r a v e r t e b r a l b lo c k s a r e p e r f o r m e d , th e p o te n tia l e x is ts
th a t e p id u r a l s p r e a d c o u ld b e s ig n if ic a n t, r e s u ltin g in a n o b s e r v a tio n o f a m o d e r a te B P d e c r e a s e .
Pain Management
Thomas Halaszynski

1. At what time frame following the postsurgical period does persistent postsurgical pain become
defined as being “chronic pain”?
A. 1 to 2 weeks
B. 3 to 4 weeks
C. 1 to 2 months
D. 6 to 12 months

2. Both surgical trauma and anesthetic administration techniques can modulate which of the
following human stress responses?
A. Neuroendocrine
B. Metabolic
C. Inflammatory
D. All of the above

3. Nonsteroidal anti-inflammatory drugs (NSAIDs) are often used as part of “multimodal”


analgesic therapy; some of the potential advantages include all of the following, except
A. Decreases opioid requirements
B. Can decrease postoperative pain intensity
C. Indirect effect of decreasing opioid-related side effects
D. Can improve wound healing

4. Type(s) of symptomatic pain conditions that best describes “chronic” pain often includes
A. Neuropathic pain alone
B. Nociceptive pain alone
C. Neuropathic or nociceptive pain
D. Somatic or visceral pain

5. At what levels does the modulation of pain by electrical stimulation result in the activation of
inhibitory fibers?
A. Nociceptor level alone
B. Spinal cord level alone
C. Only within the brain
D. All of the above

6. Activation of which of the following mechanisms and/or pathways best describes “central
sensitization” at the level of the spinal cord?
A. Second-order wide dynamic range neurons
B. Dorsal horn neuron
C. Spinal cord reflexes
D. All of the above

7. A 26-year-old female undergoes a left stellate ganglion block for treatment of complex
regional pain syndrome of the left hand. Twenty minutes after the block is placed, skin
temperature in the left arm rises from 33 to 36.5°C. Venous engorgement of the left arm and
hand, left eye papillary constriction, and drooping of the eyelid are observed. The pain is not
relieved. Which of the following can best explain the block failure?
A. Pain-carrying fibers originated from right stellate ganglion
B. Pain-carrying fibers originated from middle cervical ganglion
C. Pain-carrying fibers originated from inferior cervical ganglion
D. Pain-carrying fibers originated from second thoracic ganglion

8. Chronic pain indications for insertion of a spinal cord stimulator include all of the following,
except
A. Phantom pain
B. Complex regional pain syndrome
C. Chronic visceral pelvic pain
D. Compartment syndrome pain

9. The term used to best describe the PAIN condition “perception toward ordinary non-noxious
stimulus as being painful” is
A. Hyperalgesia
B. Anesthesia dolorosa
C. Hypalgesia
D. Allodynia

10. Incorrect statement related to the definition of an abnormal sensation is


A. Dysesthesia is an abnormal sensation with or without a stimulus
B. Paresthesia is abnormal sensation without a stimulus
C. Neuralgia is due to abnormality in nerve roots
D. Hyperesthesia is an abnormal sensation of exaggerated response to mild stimulation

11. Which of the following clinical diagnoses best describes deafferentation pain?
A. Herniated disk
B. Amputation
C. Neuropathic pain
D. Diabetic neuropathy

12. Gasserian ganglion block is most commonly used for neuropathic pain located in which of the
following nerve distributions?
A. Facial nerve
B. Trigeminal nerve
C. Glossopharyngeal nerve
D. Vagal nerve

13. Major excitatory neurotransmitters responsible for pain modulation include all the following,
except
A. Substance P
B. Glutamate
C. Somatostatin
D. Aspartate

14. All the following are inhibitory neurotransmitters in the pain pathway, except
A. Norepinephrine
B. Adenosine
C. Serotonin
D. Calcitonin gene-related peptide

15. Incorrect statement regarding secondary hyperalgesia is


A. It is caused by neurogenic inflammation
B. It is associated with Lewis’ triple response
C. It is increased by injection of local anesthetics
D. It is increased by application of capsaicin

16. Types of pain disorders that are commonly treated using “sympathetic blockade” include all of
the following, except
A. Complex regional pain syndrome
B. Phantom limb pain
C. Postherpetic neuralgia
D. Acute pain due to pelvic exenteration

17. Systemic responses of the human body that can develop secondary to symptoms of acute pain
include all of the following, except
A. Hypertension and tachycardia
B. Increased work of breathing
C. Urinary retention
D. Increased peristalsis

18. A 56-year-old man presented to his primary care physician with a complaint of right buttock
and right leg pain along with numbness and tingling sensations. He was subsequently diagnosed
with a piriformis syndrome (trapped nerve). The nerve(s) responsible for this diagnosis is/are
A. Femoral and saphenous nerves
B. Ilioinguinal nerve
C. Sciatic nerve
D. Obturator and femoral nerves

19. A 56-year-old patient with a past medical history of hypertension, diabetes, and alcohol abuse
presents to the operating room for a right-elbow open reduction internal fixation, secondary to
a motor vehicle accident that occurred 24 hours ago. On postoperative day 1, the patient
complains of right fourth and fifth digit numbness and minor pain. A diagnosis of cubital tunnel
syndrome has been made. The nerve most likely to be involved is
A. Median nerve
B. Ulnar nerve
C. Radial nerve
D. Musculocutaneous nerve

20. Incorrect statement regarding myofascial pain is


A. Myofascial pain is associated with muscle discomfort (pain, stiffness, weakness, spasm)
B. Patient may have several trigger points producing pain upon stimulation
C. Systemic diseases such as connective tissue disease may cause myofascial pain
D. Myofascial pain is never associated with autonomic dysfunctions

21. The diagnosis of fibromyalgia includes all of the following, except


A. Minor pain
B. Pain lasts more than 3 months
C. No other pathologies can explain or contribute to the pain
D. Frequent association with psychiatric diagnosis

22. Common causes for lower back pain include all of the following, except
A. Lumbosacral strain
B. Degenerative disk disease
C. Myofascial syndromes
D. Fibromyalgia syndrome

23. A 68-year-old male presents to his primary care physician’s office with a major complain of
back pain radiating into the gluteal region and pain in the distribution of the plantar surface of
the foot on the same side. The patient’s physical examination reveals decreased plantar flexion
of the foot. An MRI will most commonly show a herniated disk at
A. L2-L3
B. L3-L4
C. L4-L5
D. L5-S1

24. Disk herniation at L4-L5 of the vertebral column often presents with all of the following
clinical symptoms, except
A. Diminished dorsiflexion of the foot
B. Quadriceps femoris muscle weakness
C. Posterior-lateral thigh pain
D. Dorsal foot pain between first and second toes

25. Facet syndrome is characterized by all the following, except


A. Pain relieved by local anesthetic injection of the medial branches of the posterior rami of
spinal nerves
B. Pain relieved by an intra-articular injection of the zygapophyseal joints
C. Pain can be exacerbated by overextension and lateral rotation of back
D. Pain is sympathetically mediated

26. Incorrect statement regarding neuropathic pain is


A. It includes pain associated with stroke, spinal cord injury, and diabetic neuropathy
B. It is not associated with low back pain or multiple sclerosis
C. Neuropathic pain can be paroxysmal
D. Neuropathic pain can be associated with hyperpathia

27. Regarding the treatment of neuropathic pain, the correct statement is


A. Narcotics is the most effective and “first-line” treatment option
B. It is most optimally treated with multimodal therapies
C. Sympathetic blockade will eliminate all neuropathic pain
D. Spinal cord stimulator is not an effective therapy

28. Pathological features of complex regional pain syndrome include all the following, except
A. It is sympathetically mediated
B. It is often associated with documented nerve injury
C. It is only associated with major injuries (never from minor procedures)
D. It is not associated with evidence of skin color, hair, and temperature changes

29. Incorrect statement regarding treatment of complex regional pain syndrome (CRPS) is
A. E f f ic a c io u s tr e a tm e n t w ith m u ltim o d a l th e r a p y e a r ly in th e d ia g n o s is (w ith in 1 m o n th o f
s y m p to m ) is m o s t e ffe c tiv e
B. It r e s p o n d s w e ll to s y m p a th e tic b lo c k a d e
C. If n o t tr e a te d p r o p e r l y a n d in a tim e ly f a s h io n , C R P S c a n r e s u lt in f u n c tio n a l d is a b ility
D. P a tie n ts n e e d to r e f r a i n f r o m p h y s ic a l th e r a p y u n til th e p a in s y n d r o m e is r e s o lv e d

30. P o s s ib le c o m p lic a tio n s to d is c lo s e w h e n o b ta in in g a n a n e s th e s ia c o n s e n t f r o m a p a tie n t p r i o r to


p e r f o r m a n c e o f a c e lia c p le x u s b lo c k in c lu d e a ll o f th e f o ll o w i n g , e x c e p t

A. P o s tu r a l h y p o te n s io n a n d lig h th e a d e d n e s s
B. C o n s tip a tio n a n d u r i n a r y r e te n tio n
C. V e n a c a v a a n d a o r tic v a s c u la r in ju r y
D. R e tr o p e r ito n e a l h e m o r r h a g e
CHAPTER 9 ANSWERS

1. C. P e r s is te n t p o s ts u r g ic a l p a in is d e f in e d a s c h r o n ic p a in th a t c o n tin u e s b e y o n d th e u s u a l
r e c o v e r y p e r i o d o f 1 to 2 m o n th s f o ll o w i n g s u r g e r y (w e ll p a s t th e n o r m a l c o n v a le s c e n c e p e r io d
e x p e c te d f o r a p a r tic u la r /s p e c if ic s u r g ic a l p r o c e d u r e ) . C h r o n ic p a in is d e f in e d a s p a in th a t h a s
la s te d l o n g e r th a n 3 to 6 m o n th s , th o u g h s o m e o th e r in v e s tig a to r s h a v e p la c e d th e tr a n s itio n
f r o m a c u te to c h r o n ic p a in a t 12 m o n th s . T h e in c id e n c e o f p e r s is te n t p o s ts u r g ic a l p a in c a n o fte n
e x c e e d a n in c id e n c e o f 3 0 % a fte r c e r ta in h i g h - r i s k / s u r g i c a l l y in v a s iv e p r o c e d u r e s s u c h as
a m p u ta tio n s , th o r a c o to m y , m a s te c to m y , a n d in g u in a l h e r n ia r e p a ir . A c u te p a in w ill ty p ic a lly la s t
le s s th a n 3 0 d a y s , c h r o n ic p a in to m o r e th a n 6 m o n th s d u r a tio n , a n d s u b a c u te p a in la s ts f r o m 1
to 6 m o n th s . A p o p u la r a lte r n a tiv e d e f in itio n o f c h r o n ic p a in in v o lv in g n o a r b i t r a r i l y fix e d
d u r a tio n s is “ p a in th a t e x te n d s b e y o n d th e e x p e c te d p e r i o d o f h e a lin g .”

2. D. M a n y p e r io p e r a tiv e f a c to r s c a n p r o d u c e s ig n if ic a n t in f lu e n c e to w a r d a m p lif y in g o r
d e c r e a s in g th e s u r g ic a l s tr e s s r e s p o n s e ( s ) s u c h a s n e u r o e n d o c r i n e , m e ta b o lic , a n d in f l a m m a to r y
c h a n g e s . T h e s e f a c to r s c a n b e f u r th e r m o d if ie d b y p a tie n t- s p e c if ic c o n tr ib u tio n s s u c h as
a n x ie ty /d e p r e s s io n , s u r g ic a l h is to r y , s u r g ic a l te c h n iq u e ( o p e n v s . la p a r o s c o p y ) , a n d a n e s th e tic
te c h n iq u e s ( g e n e r a l v s . r e g io n a l) .

3. D. N S A ID s h a v e n o t o n ly m a n y o f th e a b o v e - id e n tif ie d a d v a n ta g e s , b u t a ls o s e v e r a l p o te n tia l
s id e e ffe c ts th a t th e p r a c titio n e r m u s t r e m a in c o g n iz a n t o f s u c h a s r i s k o f g a s tr o in te s tin a l
b le e d in g , r e n a l in ju r y , a n d th e p o te n tia l to im p a ir w o u n d h e a lin g .

4. C. C h r o n ic p a in is m o s t o f te n d e f in e d a s n e u r o p a th ic a n d /o r n o c ic e p tiv e in n a tu re . C h r o n ic
p a in m a y b e d iv id e d in to n o c ic e p tiv e p a in — c a u s e d b y a c tiv a tio n o f n o c ic e p to r s — a n d
n e u r o p a th ic p a in — c a u s e d b y d a m a g e to o r m a lf u n c tio n o f th e n e r v o u s s y s te m . N e u r o p a th ic
p a in is d iv id e d in to p e r ip h e r a l (w ith in th e p e r ip h e r a l n e r v o u s s y s te m ) a n d c e n tr a l ( o r ig in a t in g
f r o m th e b r a in /s p in a l c o r d ) . P e r ip h e r a l n e u r o p a th ic p a in is o f te n d e s c r ib e d a s b u r n in g , tin g lin g ,
e le c tr ic a l, s ta b b in g , a n d /o r p in s a n d n e e d le s s e n s a tio n ( s ) . N o c ic e p tiv e p a in is d iv id e d in to
s u p e r f ic ia l o r d e e p , a n d d e e p p a in in to d e e p s o m a tic a n d v is c e r a l p a in . S u p e r f ic ia l p a in is
in itia te d b y a c tiv a tio n o f n o c ic e p to r s in th e s k in o r s u p e r f ic ia l tis s u e s . D e e p s o m a tic p a in is
in itia te d b y s tim u la tio n o f n o c ic e p to r s in lig a m e n ts , te n d o n s , b o n e s , b l o o d v e s s e ls , a n d m u s c le s ,
a n d is d e s c r ib e d a s d u ll, a c h in g , p o o r l y - l o c a l i z e d p a in . V is c e r a l p a in o r ig i n a te s in th e in te r n a l
o r g a n s y s te m (s ) o f th e b o d y . V is c e r a l p a in m a y b e w e ll- lo c a liz e d , b u t o f te n is d if f ic u lt to lo c a te ,
a n d s e v e r a l v is c e r a l r e g i o n s c a n p r o d u c e “ r e f e r r e d ” p a in w h e n d a m a g e d o r in f la m e d , w h e r e th e
s e n s a tio n is lo c a te d in a n a r e a d is ta n t f r o m th e s ite o f p a th o lo g y o r in ju r y .

5. D. M o d u la tio n o f p a in c a n h a p p e n c e n tr a lly o r p e r ip h e r a lly . It c a n o c c u r a t th e n o c ic e p to r le v e l


p e r ip h e r a lly o r c e n tr a lly e ith e r in th e s p in a l c o r d o r in s u p r a s p in a l s tr u c tu r e s . T h e s e m o d u la tio n
e ffe c ts c a n b e e ith e r in h ib itiv e o r fa c ilita tiv e . In th e b r a i n a n d th e s p in a l c o r d , m u c h o f th e
i n f o r m a t io n f r o m th e n o c ic e p tiv e a f f e r e n t f ib e r s r e s u lts f r o m e x c ita to r y d is c h a r g e s o f
m u ltir e c e p tiv e n e u r o n s . P a in i n f o r m a t io n in th e c e n tr a l n e r v o u s s y s te m is c o n tr o lle d b y
a s c e n d in g a n d d e s c e n d in g in h ib ito r y p a th w a y s (u s in g e n d o g e n o u s o p io id s o r o th e r e n d o g e n o u s
s u b s ta n c e s ). In a d d itio n , a p o w e r f u l in h ib itio n o f p a in - r e la te d i n f o r m a t io n o c c u r s in th e s p in a l
c o r d . T h e s e in h ib ito r y s y s te m s c a n b e a c tiv a te d b y b r a i n s tim u la tio n a n d p e r ip h e r a l n e r v e
s tim u la tio n . H o w e v e r, p a in is a c o m p le x p e r c e p tio n th a t is in f lu e n c e d a ls o b y p r i o r e x p e r ie n c e
a n d b y th e c o n te x t w ith in w h ic h th e n o x io u s s tim u lu s o c c u r s . T h is s e n s a tio n is a ls o in f lu e n c e d
b y e m o tio n a l state.

6. D. C e n tra l s e n s itiz a tio n is a n e n h a n c e m e n t in th e f u n c tio n o f n e u r o n s a n d c ir c u its in


n o c ic e p tiv e p a th w a y s , c a u s e d b y in c r e a s e s in m e m b r a n e e x c ita b ility a n d s y n a p tic e f f ic a c y as
w e ll a s r e d u c e d in h ib itio n a n d is a m a n if e s ta tio n o f th e p la s tic ity o f th e s o m a t o s e n s o r y n e r v o u s
s y s te m in r e s p o n s e to a c tiv ity , in f la m m a tio n , a n d n e u r a l in ju r y . C e n tra l s e n s itiz a tio n is
r e s p o n s ib le f o r h y p e r a lg e s ia a n d th e r e a r e th r e e m e c h a n is m s th a t h a v e b e e n id e n tif ie d a t th e
le v e l o f s p in a l c o r d : (1 ) w in d u p o f s e c o n d - o r d e r w id e d y n a m ic r a n g e n e u r o n s , (2 ) d o r s a l h o r n
n e u r o n r e c e p to r f ie ld e x p a n s io n , a n d (3 ) h y p e r e x c ita b ility o f f le x io n r e f le x e s .

7. D. T h e s te lla te g a n g lio n ( c e r v ic o th o r a c ic g a n g lio n o r i n f e r i o r c e r v ic a l g a n g lio n ) is a


s y m p a th e tic g a n g lio n f o r m e d b y th e f u s io n o f th e i n f e r i o r c e r v ic a l a n d f ir s t th o r a c ic g a n g lio n .
S te lla te g a n g lio n is lo c a te d a t th e le v e l o f C 7 ( s e v e n th c e r v ic a l v e r te b r a ) , a n te r io r to th e
tr a n s v e r s e p r o c e s s o f C 7 , s u p e r io r to th e n e c k o f th e f i r s t r ib , a n d j u s t b e lo w th e s u b c la v ia n
a rte ry . C o m p lic a tio n s o f s te lla te b lo c k in c lu d e in tr a v a s c u la r in je c tio n , in tr a th e c a l/e p id u r a l
in je c tio n , b le e d in g , p n e u m o th o r a x , b r a c h ia l p le x u s in v o lv e m e n t, lo c a l a n e s th e tic s s p r e a d to
r e c u r r e n t la r y n g e a l n e r v e , a n d o s te o m y e litis o r m e d ia s tin itis ( r a r e ly ) .

8. D. A s p in a l c o r d s tim u la to r is a d e v ic e u s e d to e x e r t p u ls e d e le c tr ic a l s ig n a ls to th e s p in a l c o r d
to c o n tr o l c h r o n ic p a in , a n d a d d itio n a l a p p lic a tio n s in c lu d e u s e in s o m e m o to r d is o r d e r s . S p in a l
c o r d s tim u la tio n is m o s t e f f e c tiv e f o r n e u r o p a th ic p a in , o f w h ic h s o m e c o m m o n in d ic a tio n s
in c lu d e s y m p a th e tic a lly m e d ia te d p a in , p h a n to m lim b p a in , is c h e m ic p a in d u e to p e r ip h e r a l
v a s c u la r d is e a s e , p e r ip h e r a l n e u r o p a th ie s , a n d v is c e r a l p a in . C o m p a r tm e n t s y n d r o m e p a in o fte n
r e q u ir e s u r g e n t e v a lu a tio n a n d p o s s ib le n e e d f o r e m e r g e n c y f a s c io to m y .

9. D. H y p e r a lg e s ia is a n e x a g g e r a te d r e s p o n s e to n o x io u s s tim u li, a n e x tr e m e a n d e x a g g e r a te d
r e a c tio n to a s tim u lu s w h ic h is n o r m a l l y p a in f u l. A n e s th e s ia d o l o r o s a is p a in in a r e a th a t h a s n o
s e n s a tio n , is p a in fe lt in a n a r e a ( u s u a lly o f th e fa c e ) th a t is c o m p le te ly n u m b to to u c h w ith th e
p a in d e s c r ib e d a s c o n s ta n t, b u r n in g , a c h in g , o r s e v e r e . H y p a lg e s ia e q u a ls r e d u c e d s e n s itiv ity to
p a in , th e o p p o s ite o f h y p e r a lg e s ia . A llo d y n ia is d e f in e d a s p a in d u e to a s tim u lu s th a t d o e s n o t
n o r m a l l y p r o v o k e p a in . T e m p e r a tu r e o r p h y s ic a l s tim u li c a n p r o v o k e a llo d y n ia (w h ic h m a y
fe e l lik e a b u r n in g s e n s a tio n ) a n d c a n o f te n o c c u r a fte r in ju r y .

10. C. D y s e s th e s ia is a n a b n o r m a l s e n s a tio n w ith o r w ith o u t a s tim u lu s a n d is d e f in e d a s a n


u n p le a s a n t, a b n o r m a l s e n s e o f to u c h a n d o f te n p r e s e n ts a s p a in ( m a y a ls o p r e s e n t a s a n
in a p p r o p r ia te , b u t n o t d is c o m f o r tin g , s e n s a tio n ) . D y s e s th e s ia is c a u s e d b y le s io n s o f th e n e r v o u s
s y s te m ( p e r ip h e r a l o r c e n tr a l) a n d in v o lv e s s e n s a tio n s ( s p o n ta n e o u s o r e v o k e d ) s u c h as
b u r n in g , w e tn e s s , itc h in g , e le c tr ic s h o c k , a n d p in s a n d n e e d le s . D y s e s th e s ia c a n in c lu d e
s e n s a tio n s in a n y b o d ily tis s u e , in c lu d in g m o s t o f te n th e m o u th , s c a lp , s k in , o r le g s . P a r e s th e s ia
is a b n o r m a l s e n s a tio n w ith o u t a s tim u lu s w ith a s e n s a tio n o f tin g lin g , tic k lin g , p r ic k lin g ,
p r ic k in g , o r b u r n in g o f a p e r s o n ’s s k in w ith n o a p p a r e n t l o n g - t e r m p h y s ic a l e ffe c t. T h e
m a n if e s ta tio n o f a p a r e s th e s ia m a y b e tr a n s ie n t o r c h r o n ic . T h e m o s t f a m ilia r k in d o f
p a r e s th e s ia is th e s e n s a tio n k n o w n a s “ p in s a n d n e e d le s ” o r o f a lim b “ f a llin g a s le e p .” N e u r a lg ia
is p a in s e n s a tio n in th e d is tr ib u tio n o f a n e r v e o r a g r o u p o f n e r v e s ( r a d ic u lo p a th y is p a in
s e c o n d a r y to n e r v e r o o t s p a th o lo g ie s ) . N e u r a l g ia is p a in in o n e o r m o r e n e r v e s c a u s e d b y a
c h a n g e in n e u r o l o g i c a l s tr u c tu r e o r f u n c tio n o f th e n e r v e s r a th e r th a n b y e x c ita tio n o f h e a lth y
p a in r e c e p to r s . N e u r a l g ia f a lls in to tw o c a te g o r ie s : c e n tr a l n e u r a lg ia (th e c a u s e o f th e p a in is
lo c a te d in th e s p in a l c o r d o r b r a in ) a n d p e r ip h e r a l n e u r a lg ia . H y p e r e s th e s ia is e x a g g e r a te d
r e s p o n s e to m ild s tim u la tio n o r a c o n d itio n th a t in v o lv e s a n a b n o r m a l in c r e a s e in s e n s itiv ity to
s tim u li o f th e s e n s e .

11. B. D e a f f e r e n ta tio n p a in is a ty p e o f n e u r o p a th ic p a in th a t is a s s o c ia te d w ith lo s s o f s e n s o r y


in p u t f r o m th e p e r ip h e r y to th e c e n tr a l n e r v o u s s y s te m , s u c h a s p h a n to m lim b p a in . It is th e
in te r r u p t io n o r d e s tr u c tio n o f th e a f f e r e n t c o n n e c tio n s o f n e r v e c e lls (e .g ., in a n im a l
e x p e r im e n ts , d e a f f e r e n ta tio n d e m o n s tr a te s th e s p o n ta n e ity o f l o c o m o t o r m o v e m e n t b y th e
f r e e in g o f a m o to r n e r v e f r o m s e n s o r y c o m p o n e n ts ) .

12. B. T h e g a s s e r ia n g a n g lio n is f o r m e d f r o m tw o r o o t s th a t e x it th e v e n tr a l s u r f a c e o f th e
b r a in s te m a t th e m id p o n tin e le v e l, a n d th e s e r o o t s p a s s in a f o r w a r d a n d la te r a l d ir e c tio n in th e
p o s te r io r c r a n ia l f o s s a a c r o s s th e b o r d e r o f th e p e tr o u s b o n e . T h e y e n te r a r e c e s s c a lle d M e c k e l
c a v e , w h ic h is f o r m e d b y a n in v a g in a tio n o f th e s u r r o u n d in g d u r a m a te r in to th e m id d le c r a n ia l
f o s s a . T h e d u r a l p o u c h th a t lie s j u s t b e h in d th e g a n g lio n is c a lle d th e tr i g e m in a l c is te r n a n d
c o n ta in s c e r e b r o s p in a l f lu id . T h e g a s s e r ia n g a n g lio n is c a n o e - s h a p e d , w ith th e th r e e s e n s o r y
d iv is io n s — th e o p h th a lm ic (V 1 ), th e m a x illa r y (V 2 ), a n d th e m a n d ib u la r (V 3 )— e x itin g th e
a n te r io r c o n v e x a s p e c t o f th e g a n g lio n . A s m a ll m o to r r o o t jo i n s th e m a n d ib u la r d iv is io n a s it
e x its th e c r a n ia l c a v ity v ia th e f o r a m e n o v a le . T h e g a s s e r ia n g a n g lio n c o n ta in s th e c e ll b o d ie s
o f s e n s o r y f ib e r s o f tr i g e m in a l n e r v e . T h is p r o c e d u r e c a lle d a g a s s e r ia n g a n g lio n b lo c k to tr e a t
f a c ia l p a in is w h e r e a s m a ll a m o u n t o f lo c a l a n e s th e tic (w ith o r w ith o u t s te r o id ) is in je c te d o n to
th e p a r t o f th e n e r v e s u p p ly to th e fa c e c a lle d th e g a s s e r ia n g a n g lio n ( lo c a te d to th e b a c k o f th e
fa c e b e tw e e n th e e a r a n d e y e s o c k e t).

13. C. S u b s ta n c e P, g lu ta m a te , a s p a rta te , a n d A T P a r e a m o n g th e m a jo r e x c ita to r y m o le c u le s


r e s p o n s ib le f o r p a in m o d u la tio n . S o m a to s ta tin , a c e ty lc h o lin e , a n d e n d o r p h in a r e a m o n g th e
m a jo r in h ib ito r y m e d ia to r s o f p a in .

14. D. N o r e p in e p h r in e , a d e n o s in e , a n d s e r o to n in a r e a m o n g th e m a jo r in h ib ito r y
n e u r o tr a n s m itte r s in th e p a in c a s c a d e . H o w e v e r, c a lc ito n in g e n e - r e la te d p e p tid e is a n e x c ita to r y
n e u r o tr a n s m itte r .

15. A. S e c o n d a r y h y p e r a lg e s ia is d e f in e d a s a n in c r e a s e in p a in s e n s itiv ity w h e n a n o x io u s


s tim u lu s is d e liv e r e d to a r e g i o n s u r r o u n d in g , b u t n o t in c lu d in g , th e z o n e o f in ju r y ( in c r e a s e d
p a in s e n s itiv ity o u ts id e o f th e a r e a o f in ju r y o r in f la m m a tio n ) . S e c o n d a r y h y p e r a lg e s ia , a ls o
k n o w n a s n e u r o g e n ic in f la m m a tio n , is a s s o c ia te d w ith lo c a l r e d n e s s , tis s u e e d e m a , a n d
s e n s itiz a tio n to n o x io u s s tim u li. L o c a l a n e s th e tic s in je c tio n o r c a p s a ic in to p ic a l a p p lic a tio n c a n
d im in is h th e s e r e a c tio n s . S e c o n d a r y h y p e r a lg e s ia is a c e n tr a lly m e d ia te d c o n d itio n th a t m a y
o c c u r d u e to in ju r y o r d is e a s e in a n a r e a o f th e b o d y . S e c o n d a r y h y p e r a lg e s ia is d u e to c e n tr a l
n e u r o n s e n s itiz a tio n a n d r e q u ir e s c o n tin u o u s n o c ic e p to r in p u t f r o m th e z o n e o f p r i m a r y
h y p e r a lg e s ia f o r its m a in te n a n c e . S e c o n d a r y h y p e r a lg e s ia im p lie s o n ly m e c h a n ic a l
h y p e r a lg e s ia (e .g ., a llo d y n ia a n d p in p r ic k ) .

16. D. C e r ta in c h r o n ic p a in c o n d itio n s a r e s y m p a th e tic a lly m a in ta in e d a n d w ill r e s p o n d to


s y m p a th e tic b lo c k a d e , s u c h a s c o m p le x r e g i o n a l p a in s y n d r o m e , p h a n to m lim b p a in ,
p o s th e r p e tic n e u r a lg ia , a n d tr i g e m in a l n e u r a lg ia . H o w e v e r, a c u te p a in s e c o n d a r y to p e lv ic
e x e n te r a tio n s u r g e r y , a lth o u g h v e r y d if f ic u lt to tre a t, is ty p ic a lly n o t m e d ia te d s y m p a th e tic a lly
a n d d o e s n o t u s u a lly r e s p o n d w e ll to a s y m p a th e c to m y .

17. D. O n e o f th e m a n y r e a s o n s a c u te p a in n e e d s to b e m a n a g e d p r o p e r l y is its s y s te m ic e ffe c ts ,


w h ic h in c lu d e h y p e r te n s io n , ta c h y c a r d ia , a n d in c r e a s e d m in u te v e n tila tio n , c a n p r o m o t e ile u s
a n d u r i n a r y r e te n tio n , a lo n g w ith th e r e le a s e o f c a ta b o lic h o r m o n e s .

18. C. P i r i f o r m i s s y n d r o m e is a n e u r o m u s c u la r d is o r d e r th a t o c c u r s w h e n th e s c ia tic n e r v e is
c o m p r e s s e d o r o th e r w is e ir r ita te d b y th e p i r i f o r m i s m u s c le , c a u s in g p a in , tin g lin g , a n d
n u m b n e s s in th e b u tto c k s a n d a lo n g th e p a th o f th e s c ia tic n e r v e d e s c e n d in g d o w n th e p o s te r io r
lo w e r th ig h a n d in to th e le g . T h e s c ia tic n e r v e c a n b e tr a p p e d a t th e s c ia tic n o tc h a n d c a u s e
im p in g e m e n t s y n d r o m e s (b u tto c k s a n d le g p a in ).

19. B. T h e c u b ita l tu n n e l is a c h a n n e l th a t a llo w s th e u ln a r n e r v e to tr a v e l o v e r th e e lb o w a n d is


b o r d e r e d b y th e m e d ia l e p ic o n d y le o f th e h u m e r u s , th e o l e c r a n o n p r o c e s s o f th e u ln a , a n d th e
te n d in o u s a r c h j o i n i n g th e h u m e r a l a n d u ln a r h e a d s o f th e f le x o r c a r p i u ln a r is . C u b ita l tu n n e l
s y n d r o m e is a c o n d itio n b r o u g h t o n b y in c r e a s e d p r e s s u r e o n th e u ln a r n e r v e at th e e lb o w ,
ty p ic a lly a g a in s t m e d ia l e p ic o n d y le w h e r e th e u ln a r n e r v e p a s s e s . T h is c a n o c c u r d u e to c h r o n ic
c o m p r e s s io n o f th is n e r v e , p o s itio n a l o r d u e to in a p p r o p r ia te c a s t/s p lin t p la c e m e n t.

20. D. M y o f a s c ia l p a in s y n d r o m e s a r e a s s o c ia te d w ith m u s c le s y m p to m s s u c h a s s p a s m , p a in ,
w e a k n e s s , a n d s tiffn e s s , a n d a s s o c ia te d w ith a u to n o m ic d y s f u n c tio n (e .g ., v a s o c o n s tr ic tio n ) . T h e
t r i g g e r p o in ts c a n s p o n ta n e o u s ly r e s o lv e , b u t m a y c o n tin u e o n a n d b e c o m e la te n t a n d a c tiv a te d
a t a la te r tim e . M y o f a s c ia l p a in n e e d s to b e r u le d o u t in p a tie n ts w ith c h r o n ic lo w e r b a c k p a in as
t r i g g e r p o in ts in q u a d ra tu s lu m b o r u m , a n d g lu te u s m e d iu s m u s c le s c a n b e th e c a u s e f o r it. S o m e
s y s te m ic d is e a s e s s u c h a s c o n n e c tiv e tis s u e d is e a s e c a n c a u s e m y o f a s c ia l p a in . P o o r p o s tu r e a n d
e m o tio n a l d is tu r b a n c e s m ig h t a ls o in s tig a te o r c o n tr ib u te to m y o f a s c ia l p a in . T h e d ia g n o s is o f
m y o f a s c ia l p a in is b y th e p a in a n d e x is te n c e o f t r i g g e r p o in ts .

21. A. F ib r o m y a lg ia is c h a r a c te r iz e d b y c h r o n ic w id e s p r e a d p a in a n d a llo d y n ia (a h e ig h te n e d a n d
p a in f u l r e s p o n s e to p r e s s u r e ) . Its e x a c t c a u s e is u n k n o w n , b u t b e lie v e d to in v o lv e p s y c h o lo g ic a l,
g e n e tic , n e u r o b i o l o g i c a l , a n d e n v ir o n m e n ta l f a c to r s . F ib r o m y a lg ia s y m p to m s a r e n o t r e s tr ic te d
to p a in . O th e r s y m p to m s c a n in c lu d e d e b ilita tin g fa tig u e , s le e p d is tu r b a n c e s , a n d j o i n t s tiffn e s s .
T h e A m e r ic a n C o lle g e o f R h e u m a to lo g y d ia g n o s is c r i t e r i o n in d ic a te s th a t th e p a in b e a t le a s t
m o d e r a te to s e v e r e in s c a le : W id e s p r e a d P a in In d e x (W P I) s c o r e o f 7 o r h ig h e r a n d th e
S y m p to m S e v e r ity (S S ) s c a le s c o r e o f 5 o r h ig h e r . A n o th e r c a te g o r y o f c r i t e r i a to d ia g n o s e
f i b r o m y a l g i a in c lu d e s a W P I o f 3 to 6 a lo n g w ith a n SS s c a le s c o r e o f 9 o r h ig h e r . T h e o th e r
tw o c r i t e r i a f o r d ia g n o s is in c lu d e c h r o n ic c o n d itio n s a n d a b s e n c e o f o th e r c o e x is tin g c h r o n ic
p a in d is o r d e r s . T r e a tm e n t in c lu d e s p r e g a b a lin ( L y ric a ) , d u lo x e tin e (C y m b a lta ), a n d m iln a c ip r a n
(S a v e lla ) to id e n tif y a f e w o p tio n s .

22. D. C h r o n ic lo w e r b a c k p a in is o n e o f th e to p r e a s o n s f o r p h y s ic ia n o f f ic e v is its a n d a ls o o n e
o f th e g r e a te s t r e a s o n s f o r w o r k a b s e n c e . L u m b o s a c r a l s tra in , d e g e n e r a tiv e d is k d is e a s e , a n d
m y o f a s c ia l s y n d r o m e s a r e th e m o s t c o m m o n c a u s e s , a n d f i b r o m y a l g i a is n o t ty p ic a lly
a s s o c ia te d w ith a d ia g n o s is o f lo w e r b a c k p a in .

23. D. D is k h e r n ia tio n a t L 5 - S 1 is th e m o s t c o m m o n lo c a t io n o f v e r te b r a l d is k p a th o lo g y
p r e s e n tin g a s b a c k p a in (a ffe c ts th e S1 n e r v e r o o t) . P a tie n ts o f te n h a v e a s s o c ia te d g lu te a l p a in
a n d n u m b n e s s a lo n g w ith p a in /p a r e s th e s ia in th e p o s t e r i o r th ig h , p o s te r o la t e r a l c a lf, la te r a l
d o r s u m , a n d u n d e r s u r f a c e o f th e f o o t. P h y s ic a l e x a m in a tio n w ill a ls o id e n tif y a d im in is h e d
p la n ta r f le x io n o f th e a n k le o n th e a f f e c te d s id e .

24. B. D is k h e r n ia tio n a t L 4 - L 5 is a v e r y c o m m o n lo c a t io n f o r s u c h p a th o lo g y a n d a ffe c ts th e L 5


n e r v e r o o t. P a tie n ts m a y p r e s e n t w ith p a in a n d p a r e s th e s ia a n y w h e r e a lo n g th e d e r m a to m e
d is tr ib u tio n o f th e L 5 n e r v e r o o t ( la te r a l th ig h , a n te r o la te r a l c a lf, m e d ia l d o r s u m o f th e f o o t,
p a r tic u la r ly b e tw e e n th e f i r s t a n d s e c o n d to e s ) . T h e s y m p to m s o f q u a d r ic e p s f e m o r is m u s c le
w e a k n e s s w o u ld b e s e c o n d a r y to p a th o lo g y o f n e r v e r o o t s L 2 - L 4 .

25. D. F a c e t jo in ts a r e f o r m e d b y th e s u p e r io r a n d i n f e r i o r p r o c e s s e s o f e a c h v e r te b r a . F a c e t
s y n d r o m e is a s y n d r o m e in w h ic h th e z y g a p o p h y s e a l jo in ts ( s y n o v ia l d ia r th r o s e s , f r o m C 2 to
S 1 ) c a u s e b a c k p a in . F if ty - f iv e p e r c e n t o f f a c e t s y n d r o m e c a s e s o c c u r in c e r v ic a l v e r te b r a e , a n d
3 1 % in th e lu m b a r a r e a . F a c e t s y n d r o m e c a n p r o g r e s s to s p in a l o s te o a r th r itis , w h ic h is k n o w n
a s s p o n d y lo s is . B a c k p a in s e c o n d a r y to d e g e n e r a tiv e c h a n g e s in th e f a c e t ( z y g a p o p h y s e a l) jo in ts
is a ls o c a lle d f a c e t s y n d r o m e . It is c h a r a c te r iz e d b y n e a r m id lin e p a in th a t m a y r a d ia te to th e
g lu te a l r e g i o n , th ig h , a n d k n e e . F a c e t s y n d r o m e s y m p to m s m a y w o r s e n b y h y p e r e x te n s io n o r
la te r a l r o ta tio n o f th e b a c k . C o n f ir m a tiv e te s t is p a in r e l i e f o f f e r e d b y in tr a - a r ti c u la r in je c tio n o f
lo c a l a n e s th e tic s o r b lo c k a d e o f th e p o s t e r i o r r a m u s m e d ia l n e r v e b r a n c h .

26. B. N e u r o p a th ic p a in is p a in c a u s e d b y d a m a g e o r d is e a s e th a t a ffe c ts th e s o m a t o s e n s o r y
s y s te m . N e u r o p a th ic p a in a lo n g w ith c o m p o n e n ts o f n e u r o p a th ic p a in c a n b e a s s o c ia te d w ith
s e v e r a l c h r o n ic d is e a s e s s u c h a s d ia b e te s , s tr o k e , s p in a l c o r d p a th o lo g y , p o s th e r p e tic n e u r a lg ia ,
m u ltip le s c le r o s is , c a n c e r p a in , o r lo w b a c k p a in . N e u r o p a th ic p a in is o f te n d e s c r ib e d a s “w a x
a n d w a n e ” ty p e s o f p a in s y m p to m s (e .g ., c o m e s a n d g o e s ) , b u r n in g , a n d e le c tr ic a l, a s d e s c r ib e d
b y p a tie n ts . A llo d y n ia o r h y p e r a lg e s ia c a n o f te n b e a s s o c ia te d w ith n e u r o p a th ic p a in .

27. B. N e u r o p a th ic p a in c a n b e v e r y d if f ic u lt to tr e a t e f f e c tiv e ly a n d o f te n r e q u ir e s m u ltip le


th e r a p e u tic m o d a litie s f o r tre a tm e n t. T h e s e in c lu d e a n tic o n v u ls a n ts , a n tid e p re s s a n ts ,
a n tia r r h y th m ic s , a 2- a d r e n e r g ic a g o n is ts , to p ic a l a g e n ts , a n d a n a lg e s ic s ( n o n s te r o id a l a n ti­
in f l a m m a to r y d r u g s a n d o p io id s ) . S y m p a th e tic b lo c k s a s w e ll a s s p in a l c o r d s tim u la tio n w o r k
f o r c e r ta in p a tie n ts r e s is ta n t to p h a r m a c o lo g i c a l in te r v e n tio n s .

28. D. C o m p le x r e g i o n a l p a in s y n d r o m e (C R P S ), f o r m e r l y c a lle d r e f le x s y m p a th e tic d y s tr o p h y


o r c a u s a lg ia , o r r e f le x n e u r o v a s c u la r d y s tr o p h y o r a m p lif ie d m u s c u lo s k e le ta l p a in s y n d r o m e ,
is a c h r o n ic s y s te m ic d is e a s e c h a r a c te r iz e d b y s e v e r e p a in , s w e llin g , a n d c h a n g e s in th e s k in .
C R P S is e x p e c te d to w o r s e n o v e r tim e . S o m e f o r m s o f C R P S a r e s y m p a th e tic a lly m a in ta in e d
a n d a r e th e r e f o r e r e s p o n s iv e to s y m p a th e tic b lo c k a d e . C R P S ty p e 2 is a s s o c ia te d w ith
d o c u m e n te d n e r v e d a m a g e /in ju r y , b u t n o t C R P S ty p e 1. C R P S c a n b e a s s o c ia te d w ith e ith e r
m in o r o r m a jo r s u r g ic a l p r o c e d u r e s o r in ju r ie s . W h e n th e a u to n o m ic n e r v o u s s y s te m is
in v o lv e d , a d d itio n a l s ig n s a n d s y m p to m s c a n in c lu d e s w e a tin g ( s u d o m o to r c h a n g e s ) , c o lo r , a n d
s k in te m p e r a tu r e c h a n g e s , a lo n g w ith tr o p h ic c h a n g e s o f th e s k in , h a ir, a n d n a ils . M o to r s tr e n g th
a n d r a n g e o f m o tio n o f th e e x tr e m ity m a y a ls o b e a ffe c te d .

29. D. T h e g e n e r a l s tr a te g y in C R P S tr e a tm e n t is o f te n m u ltid is c ip lin a r y , w ith th e u s e o f d if f e r e n t


ty p e s o f m e d ic a tio n s c o m b in e d w ith d is tin c t p h y s ic a l th e r a p ie s . P h y s ic a l th e r a p y p la y s a c e n tr a l
r o l e in th e m u ltim o d a l tr e a tm e n t o f C R P S . T h e r a p y is f a c ilita te d w ith s y m p a th e tic b lo c k a d e o r
in tr a v e n o u s r e g i o n a l b lo c k s . P h y s ic a l th e r a p y ty p ic a lly c o n s is ts o f a c tiv e m o v e m e n t w ith o u t
w e ig h ts a n d d e s e n s itiz a tio n th e ra p y . If n o t tr e a te d in tim e ly f a s h io n , C R P S c a n r e s u lt in
f u n c tio n a l d is a b ility . T h e in c id e n c e o f a c u r e is a b o u t 9 0 % w ith e f f e c tiv e m u ltim o d a l th e r a p y
in itia te d w ith in 1 m o n th o f s y m p to m s .

30. B. P o te n tia l c o m p lic a tio n s o f a c e lia c p le x u s b lo c k in c lu d e p o s tu r a l h y p o te n s io n f r o m th e


v is c e r a l s y m p a th e c to m y a n d v a s o d il a tio n d u e to th e lo c a l a n e s th e tic in je c tio n . B o th th e v e n a
c a v a a n d th e a o r ta a r e in c lo s e p r o x im ity a n d s u s c e p tib le to in tr a v a s c u la r in ju r y /in je c tio n . O th e r
p o te n tia l c o m p lic a tio n s in c lu d e a p n e u m o th o r a x , r e t r o p e r ito n e a l h e m o r r h a g e , in ju r y to th e
k id n e y s o r p a n c r e a s , a n d s e x u a l d y s fu n c tio n . T h e v is c e r a l s y m p a th e tic c h a in is in c lo s e
p r o x im ity , a n d b lo c k a d e m a y r e s u lt in u n o p p o s e d p a r a s y m p a th e tic a c tiv ity th a t m a y le a d to
in c r e a s e d g a s tr o in te s tin a l m o tility a n d d ia r r h e a .
Orthopedic Anesthesia
Thomas Halaszynski

1. The surgeon is performing a right total knee arthroplasty under a combined spinal-epidural
anesthetic. The surgical team is providing you with information that within the next 15 minutes
they plan to place bone cement (polymethylmethacrylate) to anchor the prosthesis. The most
likely clinical side effect that may occur is
A. Hypertension
B. Increased work of breathing and hypercapnia
C. Cardiac arrhythmias
D. Decreased pulmonary shunt

2. Potential complications of use of a pneumatic tourniquet include all of the following, except
A. Tourniquet pain that is relieved by performing a peripheral nerve block
B. A compression nerve injury
C. Development of arterial thromboembolism
D. Pulmonary embolism

3. A 20-year-old male (status post car accident) sustained a right femur and pelvic fracture 2 days
prior. In the last 24 hours, he has become progressively more short of breath, requiring 100%
Fi o 2 to maintain an oxygen saturation in the high 80s and is now becoming more confused and
disoriented. Physical exam reveals petechiae on the anterior chest wall, arms, and conjunctiva
along with decreased breath sounds to auscultation. The most likely diagnosis is
A. Cognitive dysfunction
B. Pulmonary fat embolism
C. Undiagnosed pneumothorax
D. Congestive heart failure

4. Incorrect statement regarding neuraxial anesthesia and deep-vein thrombosis/pulmonary


embolism (DVT/PE) in orthopedic surgical procedures is
A. Neuraxial anesthesia may reduce thromboembolic complications
B. Neuraxial anesthesia may reduce blood loss
C. Neuraxial anesthesia may decrease platelet reactivity
D. Neuraxial anesthesia may increase activity of both factor VIII and von Willebrand factor
5. On postoperative day 1, an orthopedic surgeon has consulted you about his total knee
arthroplasty patient who is in severe pain and has failed a regimen of patient-controlled
analgesia using morphine. He is now consulting you for an epidural catheter placement for
postoperative pain control, and would like to know for what time interval once-daily
prophylactic low-molecular-weight heparin (LMWH) should be held prior to performing the
epidural procedure:
A. 4 hours and no absolute contraindication to placement of a catheter
B. 6 hours and a relative contraindication to place a catheter
C. 12 hours and no absolute contraindication to placement of a catheter
D. 24 hours and absolute contraindication to place a catheter

6. In the anesthesia preadmission testing clinic, you are assessing a 58-year-old female with a
medical history significant for hypertension, diabetes, fibromyalgia, and rheumatoid arthritis
(RA). The RA is affecting the upper extremities bilaterally and the cervical spine, but her RA
symptoms are well-controlled with methotrexate. She is now presenting for an elective total hip
arthroplasty. The radiographs that should be ordered to rule out atlantoaxial instability are
A. Lateral view: flexion of the cervical spine
B. Lateral view: extension of the cervical spine
C. No radiographs are indicated since the patient is asymptomatic
D. Lateral view: both flexion and extension of the cervical spine

7. You were involved in a complicated left lower leg procedure (open reduced internal fixation of
proximal tibia-fibula fracture repair), where the final total tourniquet time was 3 hours 15
minutes. In the postanesthesia care unit, the patient showed no signs of any peripheral nerve
injury of the left lower extremity. However, on postoperative day 2, you discovered that the
patient required hemodialysis secondary to rhabdomyolysis. Which of the following could be
responsible for the rhabdomyolysis?
A. Compartment syndrome
B. Prolonged tourniquet inflation time
C. Statin medication use that patient started 2 weeks prior
D. All of the above

8. Concurrent administration of all of the following anticoagulants and thrombolytic therapy


should be avoided when planning for neuraxial blockade, except for
A. Fibrinolytic and thrombolytic therapy
B. Thrombin inhibitors (desirudin, lepirudin, bivalirudin, and Argatroban)
C. Therapeutic dosing of low-molecular-weight heparin (LMWH)
D. Subcutaneous heparin daily dose of 10,000 U or less

9. The most correct statement regarding blood loss that may occur in a patient with a hip fracture
is
A. Intertrochanteric > base of femoral neck > subcapital
B. Transcervical > base of femoral neck > subcapital
C. Subtrochanteric > subcapital > transcervical
D. Subcapital > base of femoral neck > transcervical

10. A 76-year-old female is to undergo a right femoral neck fracture repair. You perform a spinal
anesthetic using 1.5 mL 0.5% bupivacaine mixed with 100 |ig of preservative-free morphine.
How long should the patient be monitored for postoperative apnea/hypoventilation secondary
to the intrathecal morphine administration?
A. 3 days
B. 48 hours
C. 12 hours
D. 24 hours

11. A 56-year-old female with medical history significant for obesity (BMI 50), hypertension,
diabetes (IDDM), tobacco abuse, and asthma is scheduled for bilateral hip replacement surgery.
Preoperative laboratory results show a hematocrit (Hct) of 45%, blood urea nitrogen of 25
mg/dL, and creatinine of 1.0 mg/dL. Immediately following application of cement for the
second hip, the patient became hypotension with sinus tachycardia. Arterial blood gas results
reveal an Hct of 23% that responds to a crystalloid fluid bolus and blood transfusion (2 L
crystalloids, 1 L albumin, and 2 U packed red blood cells). The possible cause(s) for the
hypotension is/are
A. Hypovolemia and/or low Hct
B. Pulmonary embolism
C. Vasodilation caused by monomer of the bone cement
D. All of the above

12. A 68-year-old female (5’1” and 250 lb) with a medical history of chronic lower back pain and
radiculopathy in the lumbar 4 to sacral 1 vertebral levels presents for anterior and posterior
fusion. Her home medications include methadone 75 mg daily, oxycodone 10 mg every 3 hours
as needed, a fentanyl patch (50 |ig/h), and lisinopril 10 mg daily. The patient stated she has 7/10
pain daily. All of the following should be considered in the perioperative pain management
regimen for this patient, except
A. Continue with daily methadone
B. Consider a perioperative ketamine infusion
C. Consider transversus abdominis plane (TAP) block for the anterior abdomen
D. Add ketorolac 30 mg every 6 hours as needed for 14 days

13. You are administering anesthesia for a cervical spine procedure, and the surgeon has indicated
that she plans to monitor somatosensory-evoked potentials (SSEPs) and motor-evoked
potentials (MEPs). Your anesthetic plan includes avoidance of long-acting muscle relaxants in
addition to avoiding the use of
A. 1 MAC or higher of sevoflurane as needed for maintenance anesthesia
B. H a lf M A C o f n itr o u s o x id e to s u p p le m e n t th e in h a la tio n a g e n t
C. C o n tin u o u s p r o p o f o l in f u s io n a s a n e s th e s ia m a in te n a n c e
D. D e x m e d e to m id in e to s m o o th o u t th e a n e s th e tic d e liv e r y

14. AH of the following can be used to assist in reducing the amount of perioperative surgical
blood loss in an orthopedic procedures, except
A. Hemodilution
B. Controlled hypotension
C. Tranexamic acid
D. Aprotinin

15. All of the following statements when positioning patients for spine surgery in the prone
position are true, except
A. The neck should be in neutral position (without hyperextension or hyperflexion)
B. The eyes must be free of pressure and checked periodically
C. The abdomen must always be supported (never permitted to hang freely)
D. The arms are kept at less than 90 degrees of extension and flexion

16. The most incorrect statement regarding postoperative vision loss (POVL) that may occur
during prone positioning in spine surgery patients is
A. Ischemic optic neuropathy accounts for the highest incidence of POVL
B. Ischemic optic neuropathy is associated with decreased ocular perfusion pressure
C. Prone positioning, greater than 1 L intraoperative blood loss, and surgery lasting greater
than 6 hours represent the highest risk
D. POVL due to central retinal artery occlusion (CRAO) tends to be bilateral

17. After 180 minutes of tourniquet time during a difficult right total knee arthroplasty in a patient
under sedation and intraoperative anesthesia provided by a combined spinal-epidural, the
tourniquet is released and surgical closure is started. The patient may experience all the
following subsequent to tourniquet release, except
A. Hypotension and tachycardia
B. Transient increase of end-tidal carbon dioxide
C. Arrhythmia secondary to increased serum potassium
D. Arrhythmia secondary to increased total serum calcium

18. The most incorrect statement regarding placement of a femoral perineural catheter for pain
management during unilateral knee replacement surgery is that a femoral nerve block when
compared to neuraxial blockade
A. Provides equipotent analgesia
B. Is associated with reduced incidence of pruritus, nausea, and vomiting
C. Is associated with reduced incidence of urinary retention
D. Femoral nerve block when combined with a sciatic nerve block can provide adequate
a n a lg e s ia f o r k n e e s u r g e r y

19. A 5 6 - y e a r - o ld f e m a le is s c h e d u le d f o r a r i g h t to ta l s h o u ld e r r e p la c e m e n t in th e b e a c h c h a ir
p o s itio n . M e d ic a l h is to r y is s ig n if ic a n t f o r h y p e r te n s io n , d ia b e te s , a n d a r e c e n t tr a n s ie n t
is c h e m ic a tta c k . T h e s u r g e o n is r e q u e s tin g a h y p o te n s iv e te c h n iq u e to r e d u c e in tr a o p e r a tiv e
b l o o d lo s s . W h e r e is th e m o s t o p tim a l lo c a t io n to p la c e th e a r te r ia l lin e tr a n s d u c e r ?

A. T h e le v e l o f th e h e a r t a s th is is th e c la s s ic w a y o f m e a s u r in g
B. T h e le v e l o f th e s te r n u m to m e a s u r e a d e q u a te p e r f u s io n to th e b r a in
C. L e v e l o f th e e x te r n a l m e a tu s to m o n ito r b r a i n s te m p e r f u s io n
D. L e v e l o f s h o u ld e r to m e a s u r e a d e q u a te s h o u ld e r p e r f u s io n

20. T h e a n e s th e tic a g e n t( s ) th a t c a n c a u s e a d v e r s e c h a n g e s o n th e w a v e f o r m s w h e n m o n ito r in g


s o m a t o s e n s o r y - e v o k e d p o te n tia ls (S S E P s ) is /a r e

A. H ig h c o n c e n tr a tio n s o f in h a la tio n a l a g e n ts ( r e d u c e s w a v e f o r m a m p litu d e )


B. 1 M A C o f n itr o u s o x id e ( r e d u c e s w a v e f o r m a m p litu d e )
C. I n tr a v e n o u s a n e s th e s ia w ith k e ta m in e ( e x a g g e r a te s w a v e f o r m s )
D. A L L o f th e a b o v e

21. W h ic h o f th e f o ll o w i n g s u r g ic a l c o n d itio n s m a y n e g a tiv e ly in f lu e n c e c h a n g e s o n


s o m a t o s e n s o r y - e v o k e d p o te n tia ls (S S E P s ) w a v e f o r m s ?

A. S p in a l c o r d in ju r y
B. I s c h e m ia in d u c e d b y h y p o p e r f u s io n
C. I n tr a o p e r a tiv e b le e d in g
D. A ll o f th e a b o v e
CHAPTER 10 ANSWERS

1. C. P la c e m e n t o f b o n e c e m e n t ( b o n e c e m e n t im p la n ta tio n s y n d r o m e ) c a n r e s u lt in a n y
c o m b in a tio n o f a d v e r s e e v e n ts in c lu d in g h y p o x ia , h y p o te n s io n , c a r d ia c a r r h y th m ia s ( p o s s ib ly
h e a r t b lo c k o r e v e n s in u s a r r e s t) , p u lm o n a r y h y p e r te n s io n , a n d d e c r e a s e d c a r d ia c o u tp u t.

2. A. U s e o f a c o m p r e s s io n to u r n iq u e t o n u p p e r a n d lo w e r e x tr e m itie s c a n f a c ilita te s u r g e r y a n d
d e c r e a s e b l o o d lo s s , b u t it c a n r e s u lt in c o m p lic a tio n s a n d c a n n o t b e a p p lie d f o r p r o l o n g e d
p e r io d s . U s e o f s u c h d e v ic e s c a n b e a s s o c ia te d w ith is c h e m ic p a in th a t is n o t ty p ic a lly o r
c o m p le te ly r e lie v e d b y p e r f o r m i n g p e r ip h e r a l n e r v e b lo c k s o f th e e x tre m ity . M e ta b o lic
a lte r a tio n s u p o n to u r n iq u e t r e le a s e , a r te r ia l th r o m b o e m b o li s m , a n d p u lm o n a r y e m b o lis m a r e
o th e r p o te n tia l c o m p lic a tio n s .

3. B. A v e n o u s fa t e m b o lis m w ill ty p ic a lly p r e s e n t its e lf w ith in 72 h o u r s f o ll o w i n g lo n g - b o n e


a n d /o r p e lv ic f r a c tu r e in ju r ie s . S u c h a c o n d itio n m a y a ls o o c c u r f o ll o w i n g c a r d io p u lm o n a r y
r e s u s c ita tio n , p a r e n ta l fe e d in g w ith lip id in f u s io n , a n d lip o s u c tio n s u r g e r y . T h e c la s s ic a l tr ia d
in c lu d e s d y s p n e a , c o n f u s io n , a n d p e te c h ia e .

4. D. A d v a n ta g e s o f n e u r a x ia l a n e s th e s ia in o r th o p e d ic s u r g e r y m a y in c lu d e r e d u c e d in c id e n c e
o f D V T a n d P E f o r m a tio n , d e c r e a s e d p la te le t a c tiv ity , d e c r e a s e d f a c to r V III a n d v o n W ille b r a n d
f a c to r a c tiv ity , a n d a tte n u a tio n o f s tr e s s h o r m o n e r e s p o n s e s .

5. C. F o r p a tie n ts r e c e iv in g o n c e - d a ily d o s a g e o f L M W H f o r p r o p h y la x is , b o th e p id u r a l a n d
s p in a l n e u r a x ia l te c h n iq u e s m a y b e p e r f o r m e d ( o r n e u r a x ia l c a th e te rs p la c e d o r r e m o v e d ) 10 to
12 h o u r s f o ll o w i n g th e p r e v io u s d o s e o f L M W H . In a d d itio n , a 4 - h o u r tim e d e la y s h o u ld o c c u r
b e f o r e a d m in is te r in g th e n e x t d o s e o f d a ily p r o p h y la c tic L M W H .

6. D. A d v a n c e d R A c a n a f f e c t th e c e r v ic a l s p in e s u c h th a t p a tie n ts m a y r e q u i r e tr e a tm e n t
in c lu d in g s te r o id s , im m u n e th e ra p y , a n d /o r m e th o tr e x a te . R a d io g r a p h s o f b o th f le x io n a n d
e x te n s io n w ith la te r a l v ie w s o f th e c e r v ic a l s p in e a r e n e c e s s a r y to r u le o u t a tla n to a x ia l
in s ta b ility . If a tla n to a x ia l in s ta b ility is p re s e n t, in tu b a tio n s h o u ld b e p e r f o r m e d w ith in lin e
s ta b iliz a tio n u tiliz in g v id e o o r f ib e r o p tic l a r y n g o s c o p y to m in im iz e e x c e s s iv e h e a d a n d n e c k
m o v e m e n t in o r d e r to r e d u c e th e in c id e n c e o f c e r v ic a l s p in a l c o r d /n e r v e r o o t in ju r y .

7. D. A n y f o r m o f m u s c le d a m a g e o f s u f f ic ie n t s e v e r ity c a n c a u s e r h a b d o m y o ly s is . M u ltip le
c a u s e s c a n b e p r e s e n t s im u lta n e o u s ly in o n e in d iv id u a l. S o m e p a tie n ts m a y h a v e a n u n d e r ly in g
m u s c le c o n d itio n , u s u a lly h e r e d ita r y in n a tu re , w h ic h m a y m a k e th e m m o r e p r o n e to
r h a b d o m y o ly s is . R h a b d o m y o ly s is c a n b e in d u c e d b y s e v e r a l c o n d itio n s in c lu d in g c o m p a r tm e n t
s y n d r o m e , p r o l o n g e d to u r n iq u e t in f la tio n tim e , m e d ic a tio n s s u c h a s s ta tin s , a n d m a lig n a n t
h y p e r th e r m ia . It is s u g g e s te d th a t to u r n iq u e t tim e s u s u a lly b e k e p t to 2 h o u r s o r le s s to d e c r e a s e
th e r i s k o f n e r v e in ju r y , is c h e m ia , a n d r h a b d o m y o ly s is , w h ic h c o u ld le a d to r e n a l f a ilu r e .
8. D. A c c o r d in g to th e A m e r ic a n S o c ie ty o f R e g io n a l A n e s th e s ia a n d P a in M e d ic in e
a n tic o a g u la tio n g u id e lin e s , m e d ic a tio n s s u c h a s a n tip la te le t a g e n ts (P la v ix , a n d in tr a v e n o u s
g ly c o p r o te in Ilb /IIIa in h ib ito r s ) , th r o m b o ly tic s , f o n d a p a r in u x , d ir e c t th r o m b in in h ib ito r s , o r
th e r a p e u tic L M W H p r e s e n t a n u n a c c e p ta b le r i s k f o r s p in a l a n d /o r e p id u r a l h e m a to m a
d e v e lo p m e n t w ith o u t s u f f ic ie n t tim e la p s e b e tw e e n a d m in is tr a tio n o f s u c h m e d ic a tio n s a n d
n e u r a x ia l te c h n iq u e s . M a x im u m a d m in is tr a tio n o f s u b c u ta n e o u s h e p a r in o f 5 ,0 0 0 U b id is
e s tim a te d to b e s a f e w ith e p id u r a l a n d s p in a l a n e s th e s ia . H e p a r in a d m in is tr a tio n o f 5 ,0 0 0 U tid is
n o t k n o w n to b e a c c e p te d in c lin ic a l p r a c tic e in c o n ju n c tio n w ith n e u r a x ia l b lo c k a d e .

9. A. B lo o d lo s s in a p a tie n t s e c o n d a r y to a h ip f r a c tu r e c a n b e s ig n if ic a n t, a n d s o m e
a n e s th e s io lo g is ts p la n to u tiliz e c e ll s a v e r s a n d /o r p e r f o r m h y p o te n s iv e te c h n iq u e s to m in im iz e
f u r th e r b l o o d lo s s . B lo o d lo s s f r o m a h ip f r a c tu r e d e p e n d s o n th e a c tu a l lo c a t io n o f th e f r a c tu r e .
A s a g e n e r a l r u le , in tr a c a p s u la r (s u b c a p ita l, tr a n s c e r v ic a l) f r a c tu r e s h a v e b e e n a s s o c ia te d w ith
le s s b l o o d lo s s th a n e x tr a c a p s u la r (b a s e o f th e f e m o r a l n e c k , in te r tr o c h a n te r ic , s u b tr o c h a n te r ic )
f r a c tu r e s , a s th e c a p s u le d e c r e a s e s b l o o d lo s s b y a c tin g lik e a to u r n iq u e t. In g e n e r a l, b l o o d lo s s
f r o m a s u b tr o c h a n te r ic a n d in te r tr o c h a n te r ic > b a s e o f f e m o r a l n e c k > tr a n s c e r v ic a l a n d
s u b c a p ita l.

10. D. In tr a th e c a l m o r p h i n e c a n d e p r e s s v e n tila tio n a n d C O 2 r e s p o n s iv e n e s s th a t c a n la s t f o r u p to


2 4 h o u r s . T h e f i r s t p e a k e f f e c t o c c u r s a b o u t 6 to 8 h o u r s p o s t in je c tio n , a n d th e s e c o n d p e a k
c o u ld h a p p e n a s la te a s 2 4 h o u r s la te r. T h e p h y s io lo g ic a n d p h a r m a c o lo g i c m e c h a n is m s o f th is
in c lu d e v a s c u la r o p io id u p ta k e b y th e e p id u r a l o r s u b a r a c h n o id v e n o u s p le x u s e s , r o s t r a l s p r e a d
o f th e a q u e o u s c e r e b r o s p in a l f lu id to th e b r a in s te m , a n d /o r d ir e c t p e r im e d u lla r y v a s c u la r
c h a n n e ls .

11. D. T o ta l h ip a r th r o p la s ty s u r g e r y c a n b e c h a r a c te r iz e d b y s ig n if ic a n t b l o o d lo s s , e s p e c ia lly in
th e s itu a tio n o f b ila te r a l h ip r e p la c e m e n t. In a c u te b le e d in g , m e a s u r in g a n H c t m a y n o t
a c c u r a te ly r e f le c t th e tr u e v a lu e a s e q u ilib r iu m ta k e s s o m e tim e to s h o w th e tr u e H c t. In a d d itio n ,
b o n e c e m e n t c a n c a u s e v a s o d ila tio n , w h ic h c a n f u r th e r c o n tr ib u te to th e lo w b l o o d p r e s s u r e .
C e m e n t p la c e m e n t h a s b e e n a s s o c ia te d w ith p u lm o n a r y e m b o lis m a n d p u lm o n a r y h y p e r te n s io n .

12. D. C h r o n ic p a in is o f te n a c o m m o n o c c u r r e n c e in p a tie n ts p r e s e n te d f o r s p in e /b a c k s u r g e r ie s .
A m u ltim o d a l th e r a p e u tic p a in m a n a g e m e n t s tr a te g y a im e d a t d if f e r e n t p a in c a s c a d e p a th w a y s is
f r e q u e n tly u tiliz e d . It is a c o m m o n p r a c tic e to c o n tin u e m e th a d o n e if p a tie n ts a r e a lr e a d y ta k in g
s u c h m e d ic a tio n s a n d to c o n s id e r s ta r tin g m e th a d o n e in p a tie n ts w ith u n c o n tr o lle d p o s to p e r a tiv e
p a in . K e ta m in e (G A B A a g o n is t) is e f f e c tiv e in c h r o n ic p a in p a tie n ts . T A P b lo c k a d e w ith lo c a l
a n e s th e tic s c a n p r o v id e e f f e c tiv e s o m a tic p a in r e l i e f o f th e a n te r io r a b d o m e n th a t w ill h e lp in
th e tr e a tm e n t o f in c is io n a l p a in . E v id e n c e s u p p o r ts th e u s e o f n o n s te r o id a l a n ti- in f la m m a to r y
d r u g s a t l o w d o s e s in s p in e s u r g e r ie s , b u t h ig h e r c o n c e n tr a tio n s h a v e b e e n a s s o c ia te d w ith a r a te
o f n o n u n io n , so a r e th e r e f o r e d is c o u r a g e d .

13. A. H ig h c o n c e n tr a tio n s o f p o te n t in h a la tio n a l a g e n ts (s u c h a s d e s f lu r a n e a n d s e v o f lu r a n e )


m a y in c r e a s e n e u r o m o n i t o r i n g la te n c y a n d d e c r e a s e a m p litu d e o f th e S S E P a n d M EP.
T h e r e f o r e , in h a la tio n a g e n ts c a n b e u s e d f o r in tr a o p e r a tiv e m a in te n a n c e a n e s th e s ia , b u t a r e u s e d
a t le s s th a n o n e fu ll M A C c o n c e n tr a tio n . I n tr a v e n o u s (IV ) a n e s th e tic s a r e m o r e c o m m o n ly u s e d
f o r m a in te n a n c e o f a n e s th e s ia , a s th e y a r e m o r e c o m p a tib le w ith S S E P a n d M E P
n e u r o m o n i t o r i n g ( s o m e e x p e c te d , b u t to le r a b le c h a n g e s o n e ith e r la te n c y a n d /o r a m p litu d e ).
T h e s e IV a n e s th e tic s in c lu d e p r o p o f o l , k e ta m in e , e to m id a te , d e x m e d e to m id in e ,
b e n z o d ia z e p in e s , a n d o p io id s in d e p e n d e n tly a n d in v a r io u s c o m b in a tio n s . O p io id s h a v e th e le a s t
p o te n tia l to i n te r f e r e S S E P a n d M E P n e u r o m o n ito r in g .

14. D. S p in e s u r g e r y c a n b e a s s o c ia te d w ith s ig n if ic a n t b l o o d lo s s . S u r g ic a l a n d a n e s th e tic


te c h n iq u e s th a t h a v e b e e n d e v e lo p e d to c o n tr o l p e r io p e r a tiv e b l o o d lo s s in c lu d e h e m o d ilu tio n ,
a u to lo g o u s b l o o d d o n a tio n p r e o p e r a tiv e ly , u s e o f c e ll s a v e r, a n d e p in e p h r in e a t w o u n d site .
P h a r m a c o lo g ic a lly , a n tif ib r in o ly tic s s u c h a s tr a n e x a m ic a c id a n d e - a m in o c a p r o ic a c id h a v e
b e e n u s e d w ith s o m e e ffic a c y . T r a n e x a m ic a c id is a s y n th e tic d e r iv a tiv e o f th e a m in o a c id
ly s in e , a n d it is u s e d to tr e a t o r p r e v e n t e x c e s s iv e b l o o d lo s s d u r in g s u r g e r y a n d in v a r io u s
o th e r m e d ic a l c o n d itio n s . A p r o tin in h a s b e e n a s s o c ia te d w ith a 5 0 % in c r e a s e o f c a r d ia c s id e
e ffe c ts ( m y o c a r d ia l in f a r c tio n /c o n g e s tiv e h e a r t f a ilu r e ) , in c r e a s e (d o u b le ) o f th e r i s k o f s tr o k e ,
a n d h ig h e r d e a th r a te s ( in c r e a s e d m o r ta lity ) .

15. C. P r o n e p o s itio n in g o f p a tie n ts n e e d s to b e c a r e f u l ly e x e c u te d , e s p e c ia lly d u r in g s p in e


s u r g e r i e s ( p r o l o n g e d p r o c e d u r e s ) a n d in p a tie n ts w h o h a v e o th e r a s s o c ia te d c o m o r b id iti e s s u c h
a s r h e u m a to id a r th r itis a n d a n k y lo s in g s p o n d y litis . T h e e n d o tr a c h e a l tu b e n e e d s to b e p r o p e r l y
s e c u r e d , a n d e y e s a n d n o s e s h o u ld b e p a d d e d a n d c h e c k e d p e r i o d i c a l l y to e n s u r e th a t th e y a r e
p r e s s u r e - f r e e . T h e n e c k a n d a r m s s h o u ld b e k e p t p o s itio n e d in a n a n a to m ic a lly n e u tr a l p o s itio n .
T h e a b d o m e n n e e d s to r e m a in f r e e to a v o id in c r e a s e d v e n o u s p r e s s u r e (a s s is ts in r e d u c in g
in c r e a s e d v e n o u s b le e d in g ) a n d to r e d u c e th e in c id e n c e o f a b d o m in a l c o m p a r tm e n t s y n d r o m e
th a t c a n d e v e lo p d u r in g p r o l o n g e d d u r a tio n o f s u r g ic a l in te r v e n tio n a n d a g g r e s s iv e flu id
a d m in is tr a tio n .

16. D. Is c h e m ic o p tic n e u r o p a th y is a m a jo r c a u s e o f p e r io p e r a tiv e P O V L a c c o r d i n g l y to th e


v is io n lo s s r e g i s t r y c o lle c te d b y th e A S A . A n y in c r e a s e o f in tr a o c u la r p r e s s u r e (IO P ) o r
d e c r e a s e o n m e a n a r te r ia l p r e s s u r e (M A P ) w ill a f f e c t o c u la r p e r f u s io n p r e s s u r e (O P P ),
p a r tic u la r ly w ith p a tie n ts in a h e a d - d o w n p o s itio n w h e r e e d e m a c a n d e v e lo p in th e o r b i t th a t w ill
in c r e a s e v e n o u s p r e s s u r e . O P P = M A P - IO P. C R A O a c c o u n ts f o r a s m a ll p e r c e n ta g e o f p a tie n ts
w h o e x p e r ie n c e v is io n d y s f u n c tio n a c c o r d in g to th e v is io n lo s s r e g is tr y . C R A O m a y b e e m b o lic
in n a tu r e o r th e r e s u lt o f d ir e c t p r e s s u r e o n th e e y e b a ll; th e r e f o r e , it te n d s to b e m o s tly
u n ila te r a l.

17. D. R e le a s e o f a to u r n iq u e t u s e d o n a n e x tr e m ity d u r in g s u r g e r y is o f te n a s s o c ia te d w ith th e


r e le a s e o f m e ta b o lic ( a c id o tic ) b y - p r o d u c ts f r o m th e is c h e m ic lim b th a t a r e d u m p e d in to th e
s y s te m ic c ir c u la tio n . In p a tie n ts w ith p o o r p r e o p e r a t iv e f u n c tio n a l sta tu s o r th o s e th a t m a y
e x p e r ie n c e s ig n if ic a n t in tr a o p e r a tiv e b l o o d lo s s , th e in c r e a s e d s y s te m ic m e ta b o lic b y - p r o d u c ts
m a y b e e n o u g h to r e s u lt in h y p o te n s io n a n d c a r d ia c a r r h y th m i a th a t m a y r e q u i r e v o lu m e
r e s u s c ita tio n a n d /o r p h a r m a c o lo g i c s u p p o r t. In r a r e in s ta n c e s , th e h y p e r k a le m ia m a y n e e d to b e
tr e a te d ( s o d iu m b ic a r b o n a te o r c a lc iu m ) .
18. D. D u r in g u n ila te r a l k n e e r e p la c e m e n t s u r g e r y , p r o p e r l y f u n c tio n in g lu m b a r e p id u r a l a n d
f e m o r a l p e r in e u r a l c a th e te rs c a n p r o v id e e q u iv a le n t p e r io p e r a tiv e a n a lg e s ia . H o w e v e r, a
p e r ip h e r a l n e r v e b lo c k u s in g a f e m o r a l p e r in e u r a l c a th e te r d o e s n o t h a v e s e v e r a l o f th e ty p ic a l
s id e e ffe c ts th a t c a n b e a s s o c ia te d w ith n e u r a x ia l b lo c k a d e , s u c h a s m o r e in te n s e
s y m p a th e c to m y , p r u r itu s (w h e n o p io id s a r e m ix e d w ith lo c a l a n e s th e tic s ), n a u s e a a n d v o m itin g ,
u r i n a r y r e te n tio n , o r o r th o s ta tic h y p o te n s io n a n d lig h th e a d e d n e s s . S e v e r a l s tu d ie s h a v e s h o w n
th a t p a tie n ts w ith r e g i o n a l a n e s th e s ia /a n a lg e s ia ( f e m o r a l c a th e te r p a tie n ts m a y m e e t d is c h a r g e
c r i t e r i o n e a r lie r ) m a y s h o w e a r lie r im p r o v e d o u tc o m e s . C o n s id e r in g v a r ia tio n s o f s u r g ic a l
te c h n iq u e , th e p o s to p e r a tiv e p a in d u r in g to ta l k n e e a r th r o p la s ty is lo c a te d o n th e a n te r io r k n e e
th a t c a n b e e q u a lly c o n tr o lle d b y e ith e r lu m b a r e p id u r a l o r f e m o r a l n e r v e b lo c k a lo n e . F o r
b ila te r a l k n e e r e p la c e m e n t s u r g e r y , e ith e r b ila te r a l f e m o r a l c a th e te rs o r lu m b a r e p id u r a l
c a th e te r m a y b e a r e a s o n a b le o p tio n .

19. C. S h o u ld e r o p e r a tio n s m a y b e p e r f o r m e d in e ith e r a s ittin g ( “b e a c h c h a i r ” ) o r th e la te r a l


d e c u b itu s p o s itio n . T h e b e a c h c h a ir p o s itio n m a y b e a s s o c ia te d w ith d e c r e a s e s in c e r e b r a l
p e r f u s io n le a d in g to th e p o te n tia l f o r in c r e a s e d r i s k o f b lin d n e s s , s tr o k e , a n d b r a i n is c h e m ia . If
a c o n tr o lle d h y p o te n s io n te c h n iq u e is c h o s e n , a n a r te r ia l tr a n s d u c e r s h o u ld b e p o s itio n e d m o s t
p r e f e r a b l y a t th e le v e l o f th e b r a i n s te m (i.e., e x te r n a l m e a tu s o f th e e a r ).

20. D. M o s t o f th e c u r r e n tly u s e d a n e s th e tic a g e n ts m a y h a v e s o m e e f f e c ts /n e g a tiv e in f lu e n c e o n


S S E P ( d if f e r e n c e s m a y b e m in o r o r m a jo r c h a n g e s ) . S e v e r a l o th e r p e r io p e r a tiv e v a r ia b le s s u c h
a s h e m o g lo b in c o n c e n tr a tio n , te m p e r a tu r e , C O 2, a n d a r te r ia l b l o o d p r e s s u r e m a y a ls o in f lu e n c e
th e S S E P tr a c in g .

21. D. T h e r e a r e a h o s t o f r e a s o n s c a u s in g n e g a tiv e S S E P - tra c in g c h a n g e s . In a d d itio n to s e v e r a l


a n e s th e tic c o n s id e r a tio n s ( f r o m a n e s th e tic a g e n t c h o ic e to te c h n iq u e s u s e d ) , th e r e a r e s u r g ic a l
te c h n iq u e s a n d c o n s id e r a tio n s th a t c a n in f lu e n c e SS E P. D ir e c t tr a u m a , is c h e m ia , a n d p r e s s u r e to
th e s p in a l c o r d a r e c a p a b le o f in d u c in g a c u te c h a n g e s o n SS E P. In a d d itio n , s p in a l c o r d is c h e m ia
c h a n g e s s e c o n d a r y to d e c r e a s e d b l o o d s u p p ly , a n d /o r v e s s e l in ju r y ( s tr e tc h in g /p r e s s u r e ) m a y
ta k e a s m u c h a s a h a lf a n h o u r to m a n if e s t its e lf.
Cardiovascular Anesthesia
Deppu Ushakumari and Ashish Sinha

1. W h ic h o f th e f o ll o w i n g is r e s p o n s ib le f o r th e p la te a u p h a s e o f c a r d ia c a c tio n p o te n tia l?

A. S lo w m o v e m e n t o f p o ta s s iu m o u t o f th e c e ll
B. S lo w m o v e m e n t o f c a lc iu m in to th e c e ll
C. S lo w m o v e m e n t o f c a lc iu m o u t o f th e c e ll
D. B o th A a n d C

2. A 2 - y e a r - o ld b o y is in d u c e d w ith h a lo th a n e - in h a la tio n in d u c tio n . T h e p a tie n t s u d d e n ly g e ts


b r a d y c a r d ic , a n d y o u d e c id e to a d m in is te r a tr o p in e 0 .4 m g in tr a v e n o u s ly . I m m e d ia te ly
th e r e a f te r , y o u n o tic e th a t th e p a tie n t is h a v in g a ju n c tio n a l ta c h y c a r d ia . W h ic h o f th e f o ll o w i n g
m o s t a c c u r a te ly d e s c r ib e s th e s e q u e n c e o f e v e n ts ?

A. S in o a tr ia l (S A ) n o d e s u p p r e s s io n b y h a lo th a n e f o ll o w e d b y a n tic h o lin e r g ic a c tio n o f


a tr o p in e
B. A tr io v e n tr i c u la r (A V ) n o d e s u p p r e s s io n b y h a lo th a n e f o ll o w e d b y a n tic h o lin e r g ic a c tio n o f
a tr o p in e
C. S A n o d e a n d AV n o d e s u p p r e s s io n b y h a lo th a n e f o ll o w e d b y a n tic h o lin e r g ic a c tio n o f
a tr o p in e
D. S A n o d e a n d AV n o d e s u p p r e s s io n b y h a lo th a n e f o ll o w e d b y p a r o x y s m a l ta c h y c a r d ic a c tio n
o f a tr o p in e

3. S ig n if ic a n t in tr a v e n o u s a b s o r p tio n /in a d v e r te n t in tr a v e n o u s in je c tio n o f b u p iv a c a in e c a n c a u s e


p r o f o u n d b r a d y c a r d i a a n d s in u s n o d e a r r e s t. W h ic h o f th e f o ll o w i n g b e s t d e s c r ib e s th e
m e c h a n is m o f c a r d ia c to x ic ity o f b u p iv a c a in e ?

A. B u p iv a c a in e b in d s in a c tiv a te d f a s t s o d iu m c h a n n e ls a n d d is s o c ia te s f r o m th e m s lo w ly
B. B u p iv a c a in e b in d s a c tiv a te d f a s t s o d iu m c h a n n e ls a n d d is s o c ia te s f r o m th e m s lo w ly
C. B u p iv a c a in e b in d s in a c tiv a te d s lo w s o d iu m c h a n n e ls a n d d is s o c ia te s f r o m th e m s lo w ly
D. B u p iv a c a in e b in d s a c tiv a te d s lo w s o d iu m c h a n n e ls a n d d is s o c ia te s f r o m th e m s lo w ly

4. T h e m e c h a n is m s o f d e p r e s s io n o f c a r d ia c c o n tr a c tility b y v o la tile a n e s th e tic s in c lu d e a ll th e


f o ll o w i n g , e x c e p t

A. T h e y d e c r e a s e th e e n tr y o f c a lc iu m in to c e lls d u r in g d e p o la r iz a tio n
B. T h e y a f f e c t o n ly L -ty p e c a lc iu m c h a n n e ls
C. T h e y a lte r k in e tic s o f c a lc iu m r e le a s e
D. T h e y d e c r e a s e th e s e n s itiv ity o f c o n tr a c tile p r o te in s to c a lc iu m

5. T h e m e c h a n is m o f “ x ” d e s c e n t ( d e s c e n t b e tw e e n C a n d V w a v e s ) in th e f o ll o w i n g r i g h t a tr ia l
tr a c in g ( F ig 1 1 -1 ) is

A. D o w n w a r d m o v e m e n t o f th e a tr io v e n tr ic u la r (A V ) v a lv e c u s p s a fte r v e n tr ic u la r c o n tr a c tio n
B. P u llin g d o w n o f th e a tr iu m b y v e n tr ic u la r c o n tr a c tio n
C. R e la x a tio n o f a tr iu m a fte r a tr ia l s y s to le
D. D e c lin e in a tr ia l p r e s s u r e a s th e A V v a lv e s o p e n

6. A 3 8 - y e a r - o ld h e a lth y m a le v o lu n te e r is u n d e r g o in g c a r d ia c f u n c tio n te sts a s p a r t o f a


p h y s i o l o g y e x p e r im e n t. H is v ita l s ig n s a r e H R = 6 2 b p m , B P = 1 2 4 /7 4 m m H g , r e s p i r a t o r y r a te
= 12 b r e a th s /m in , S p o 2 = 1 0 0 % o n r o o m a ir, a n d H b = 1 4 g /d L . W h ic h o f th e f o ll o w i n g is th e
b e s t d e te r m in a tio n o f th e a d e q u a c y o f h is c a r d ia c o u tp u t?

A. C a r d ia c in d e x 4 .0 L / m in /m 2
B. C a r d ia c o u tp u t 8.1 L /m in b y th e r m o d ilu t io n te c h n iq u e
C. C a r d ia c o u tp u t 8.1 L /m in b y F ic k m e th o d
D. S v o 2 o f 7 5 % f r o m a p u lm o n a r y a r te r y (P A ) c a th e te r

7. W h ic h o f th e f o ll o w i n g p a tie n ts w ill b e a ffe c te d th e m o s t f r o m lo s s o f a tr ia l c o n tr ib u tio n to


p r e lo a d ?

A. A 6 5 - y e a r - o ld p a tie n t w ith s e v e r e a o r tic r e g u r g i t a t i o n w h o w e n t in to r e c e n t o n s e t a tr ia l


f ib r illa tio n
B. A 3 5 - y e a r - o ld p a tie n t w ith m itr a l- v a lv e a r e a o f 1.0 c m 2 w h o w e n t in to r e c e n t o n s e t a tr ia l
f ib r illa tio n
C. A n 8 0 - y e a r - o ld p a tie n t w ith s e v e r e a o r tic s te n o s is w h o w e n t in to r e c e n t o n s e t a tr ia l
f ib r illa tio n
D. A 5 5 - y e a r - o ld p a tie n t w ith a c u te r ig h t - v e n t r ic u la r m y o c a r d ia l in f a r c tio n
8. W h ic h o f th e f o ll o w i n g f o r m u la e e x p la in s th e h y p e r tr o p h y o f h e a r t in r e s p o n s e to p r e s s u r e o r
v o lu m e lo a d s (P, in tr a v e n tr ic u la r p r e s s u r e ; R , v e n tr ic u la r r a d iu s ; t, w a ll th ic k n e s s ; T ,
c ir c u m f e r e n tia l s tr e s s ) ?

A. P = 2 T t/R
B. T = 2 P /R t
C. T = 2 R /P t
D. PT = Rt

9. D o s e o f h e p a r in ( U /k g ) a d m in is te r e d f o r c a r d io p u lm o n a r y b y p a s s is ( a p p r o x im a te ly )

A. 1 0 0 to 2 0 0
B. 2 0 0 to 3 0 0
C. 3 0 0 to 4 0 0
D. 4 0 0 to 5 0 0

10. T h e s in o a t r ia l a n d th e a tr io v e n tr ic u la r (A V ) n o d e s a r e s u p p lie d in m a j o r i t y o f th e in d iv id u a ls b y

A. L e ft a n te r io r d e s c e n d in g a r te r y
B. R ig h t c o r o n a r y a r te r y
C. C ir c u m f le x a r te r y
D. P o s te r io r d e s c e n d in g a r te r y

11. B a r o r e c e p to r r e f le x is in e f f e c tiv e f o r lo n g - t e r m b l o o d p r e s s u r e (B P ) c o n tr o l b e c a u s e

A. R e n in a n g io te n s in a ld o s t e r o n e s y s te m ta k e s o v e r th e c o n tr o l
B. R e n a l r e g u la tio n o f B P is m o r e p o w e r f u l
C. O f a d a p ta tio n to c h a n g e s in B P o v e r 1 to 2 d a y s
D. A ll o f th e a b o v e

12. W h ic h o f th e f o ll o w i n g p o r ti o n s o f m y o c a r d iu m h a s a d u a l b l o o d s u p p ly ?

A. B u n d le o f H is
B. A tr io v e n tr ic u la r n o d e
C. P o s te r io r p a p illa r y m u s c le
D. S in o a tr ia l n o d e

13. W h ic h o f th e f o ll o w i n g ty p e s o f m y o c a r d ia l w o r k n e e d s th e h ig h e s t o x y g e n r e q u ir e m e n t?

A. E le c tr ic a l a c tiv ity
B. V o lu m e w o r k
C. P ressu re w o rk
D. B a s a l r e q u ir e m e n t

14. W h ic h o f th e f o ll o w i n g in h a la tio n a l a g e n ts c a u s e s th e le a s t c o r o n a r y v a s o d ila tio n ?

A. H a lo th a n e
B. I s o f lu r a n e
C. D e s f lu r a n e
D. S e v o f lu r a n e

15. W h ic h o f th e f o ll o w i n g s u r g e r i e s c a r r ie s th e h ig h e s t c a r d io v a s c u la r r is k ?

A. E m e r g e n c y a p p e n d e c to m y
B. C a r o tid e n d a r te r e c to m y
C. F e m o ra l-p o p lite a l b y p a ss s u rg e ry
D. I n g u in a l h e r n ia r e p a ir

16. A 6 7 - y e a r - o ld p a tie n t w ith u n c o n tr o lle d h y p e r te n s io n p r e s e n ts f o r a n e le c tiv e d ia ly s is a c c e s s


c r e a tio n . W h ic h o f th e f o ll o w i n g te c h n iq u e s is n o t s u ite d f o r a tte n u a tin g th e h y p e r te n s iv e
r e s p o n s e to in tu b a tio n ?

A. A d m in is te r in g 3 |ig /k g o f fe n ta n y l in tr a v e n o u s ly
B. A d m in is te r in g to p ic a l a ir w a y a n e s th e s ia
C. A d m in is te r in g lid o c a in e 0 .5 m g /k g in tr a v e n o u s ly
D. A d m in is te r in g e s m o l o l 1 m g /k g in tr a v e n o u s ly

17. T h e p a tie n t m e n tio n e d a b o v e d e v e lo p s s e v e r e h y p o te n s io n im m e d ia te ly a fte r in tu b a tio n . W h ic h


o f th e f o ll o w i n g a g e n ts is m o s t s u ite d to b r in g th e b l o o d p r e s s u r e b a c k to n o r m a l v a lu e s ?

A. E p h e d r in e
B. P h e n y le p h r in e
C. E p in e p h r in e
D. D o p a m in e

18. W h ic h o f th e f o ll o w i n g a n tia n g in a l a g e n ts h a s th e h ig h e s t c o r o n a r y v a s o d ila tin g p o te n tia l?

A. N itra te s
B. V e ra p a m il
C. D ih y d r o p y r id in e s
D. 0 - B lo c k e r s

19. W h ic h o f th e f o ll o w i n g s ta te m e n ts a b o u t c a lc iu m c h a n n e l b lo c k e r s (C C B s ) is n o t tru e ?

A. C C B s p o te n tia te b o th d e p o la r iz i n g a n d n o n d e p o la r iz i n g n e u r o m u s c u la r b lo c k e r s
B. C C B s p o te n tia te th e c ir c u l a t o r y e ffe c ts o f v o la tile a n e s th e tic a g e n ts
C. V e ra p a m il m a y d e c r e a s e a n e s th e tic r e q u ir e m e n ts
D. V e ra p a m il h a s n o e f f e c t o n c a r d ia c c o n tr a c tility ; it a c ts o n ly o n th e a tr io v e n tr ic u la r (A V )
node

20. W h ic h o f th e f o ll o w i n g 0 - b lo c k e r s is m o s t s u ite d f o r a p a tie n t w ith b r o n c h o s p a s tic d is e a s e ?

A. P ro p ra n o lo l
B. M e to p r o lo l
C. A c e b u to lo l
D. B is o p r o l o l

21. A 2 4 - y e a r - o ld f e m a le p a tie n t w ith a p r e o p e r a t iv e Q T c in te r v a l o f 5 5 0 m s is u n d e r g o in g b r e a s t


s u r g e r y u n d e r g e n e r a l a n e s th e s ia . D r o p e r id o l is a d m in is te r e d to th e p a tie n t f o r p r e v e n tio n o f
p o s to p e r a tiv e n a u s e a , f o ll o w i n g w h ic h th e p a tie n t g o e s in to p o ly m o r p h ic - v e n tr ic u la r
ta c h y c a r d ia . W h ic h o f th e f o ll o w i n g d r u g s /th e r a p ie s is b e s t f o r th e p a tie n t a t th is p o in t?

A. A m io d a r o n e
B. L id o c a in e
C. P a c in g
D. D iltia z e m

22. W h ic h o f th e f o ll o w i n g f a c to r s is n o t a s s o c ia te d w ith s e v e r e m u ltiv e s s e l d is e a s e d u r in g


e x e r c is e e le c tr o c a r d io g r a p h y ?

A. S u s ta in e d d e c r e a s e (> 1 0 m m H g ) in s y s to lic b l o o d p r e s s u r e d u r in g e x e r c is e
B. F a ilu r e to r e a c h a m a x im u m h e a r t r a te g r e a te r th a n 7 0 % o f p r e d ic te d
C. P e r s is te n c e o f S T -s e g m e n t d e p r e s s io n a fte r e x e r c is in g f o r 5 m in u te s o r l o n g e r
D. A 1 -m m u p s lo p in g o f S T s e g m e n t

23. S u r g ic a l e le c tr o c a u te r y m a y c a u s e a p r o b le m w ith a n a u to m a te d im p la n ta b le c a r d io v e r te r
d e f ib r il la to r (A IC D ) b y a ll th e f o ll o w i n g m e c h a n is m s , e x c e p t

A. A IC D in te r p r e tin g a c a u te r y c u r r e n t a s v e n tr ic u la r f ib r illa tio n


B. In h ib itio n o f p a c e m a k e r f u n c tio n d u e to c a u te r y a r tif a c t
C. I n c r e a s e d p a c in g r a te d u e to a c tiv a tio n o f a r a te - r e s p o n s iv e s e n s o r
D. C a u te r y c u r r e n t g e n e r a tin g to o m u c h h e a t a t th e lo c a t io n o f A IC D a n d c a u s in g b u r n s

24. W h ic h o f th e f o ll o w i n g E C G le a d s is m o s t s e n s itiv e to d e te c t a n a n te r io r - w a ll m y o c a r d ia l
is c h e m ia ?

A. V5
B. V4
C. II
D. V2

25. W h ic h o f th e f o ll o w i n g is n o t tr u e a b o u t s y s te m ic h y p o th e r m ia d u r in g c a r d io p u lm o n a r y b y p a s s
(C P B )?

A. I n te n tio n a l h y p o th e r m ia is a lw a y s u s e d f o ll o w i n g th e in itia tio n o f C P B


B. C o r e b o d y te m p e r a tu r e is u s u a lly r e d u c e d to 2 0 to 3 2 ° C
C. M e ta b o lic o x y g e n r e q u ir e m e n ts a r e u s u a lly h a lv e d f o r e v e r y o f 1 0 °C r e d u c tio n in
te m p e r a tu r e
D. P r o f o u n d h y p o th e r m ia to te m p e r a tu r e s o f 15 to 1 8 °C a llo w s to ta l c ir c u l a t o r y a r r e s t f o r u p
to 6 0 m in u te s
26. A d v e r s e e ffe c ts o f h y p o th e r m ia in c lu d e a ll th e f o ll o w i n g , e x c e p t

A. P la te le t d y s f u n c tio n
B. I r r e v e r s ib le c o a g u lo p a th y
C. P o te n tia tio n o f c itr a te to x ic ity
D. D e p r e s s io n o f m y o c a r d ia l c o n tr a c tility

27. C o r o n a r y p e r f u s io n p r e s s u r e is
A. Arterial diastolic pressure left-ventricular end diastolic pressure
B. Arterial diastolic pressure left-ventricular end systolic pressure
C. Arterial systolic pressure left-ventricular end diastolic pressure
D. Arterial systolic pressure left-ventricular end systolic pressure

28. W h ic h o f th e f o ll o w i n g v ie w s o f tr a n s e s o p h a g e a l e c h o c a r d i o g r a p h (T E E ) is m o s t s u ite d to
v is u a liz e b l o o d s u p p ly o f a ll th e s e g m e n ts o f th e h e a r t?

A. M id e s o p h a g e a l f o u r th - c h a m b e r v ie w
B. M id e s o p h a g e a l s e c o n d - c h a m b e r v ie w
C. T r a n s g a s tr ic m id s h o r t a x is v ie w
D. M id e s o p h a g e a l th ir d - c h a m b e r v ie w

29. D is a d v a n ta g e s o f h ig h - d o s e o p io id in d u c tio n in c lu d e a ll th e f o ll o w i n g , e x c e p t

A. P r o l o n g e d p o s to p e r a tiv e r e s p i r a t o r y d e p r e s s io n
B. H ig h in c id e n c e o f r e c a ll d u r in g s u r g e r y
C. P o s s ib le im p a ir m e n t o f im m u n e r e s p o n s e
D. M y o c a r d ia l d e p r e s s io n

30. A 6 6 - y e a r - o ld m a le is u n d e r g o in g c o r o n a r y a r te r y b y p a s s g r a f tin g (C A B G ). A fte r th e c h e s t is


o p e n e d , a p r o g r e s s i v e d e c lin e in c a r d ia c o u tp u t is n o tic e d . T h e m o s t a c c u r a te s ta te m e n t
r e g a r d i n g th e c h a n g e is

A. It is n o r m a l in d e e p ly a n e s th e tiz e d p a tie n ts
B. I n tr a v e n o u s f lu id a d m in is tr a tio n w ill n o t h e lp c o r r e c t th is c h a n g e
C. It im p lie s im m in e n t r i s k o f d e a th , a n d y o u s h o u ld a s k f o r b l o o d to b e tr a n s f u s e d
D. It is c a u s e d b y s u r g e o n lif tin g th e h e a r t, e s p e c ia lly if it is n o t a c c o m p a n ie d b y a d r o p in
b lo o d p re s su re

31. A p r o tin in th e r a p y s h o u ld b e c o n s id e r e d f o r a ll o f th e f o ll o w i n g p a tie n ts , e x c e p t

A. J e h o v a h w itn e s s e s
B. R e d o s u r g e r ie s
C. P a tie n ts w h o h a d p r i o r e x p o s u r e to a p r o tin in
D. P a tie n ts o n c o m b in e d c l o p i d o g r e l (P la v ix ) a n d a s p ir in th e r a p y

32. W h ic h o f th e f o ll o w i n g s ta te m e n ts is f a ls e r e g a r d i n g p la c e m e n t o f v e n o u s c a n n u la s f o r
c a r d io p u lm o n a r y b y p a s s (C P B )?
A. V e n o u s c a n n u la s a r e in s e r te d b e f o r e a o r tic c a n n u la p la c e m e n t
B. V e n o u s c a n n u la in s e r tio n f r e q u e n tly p r e c ip ita te s a tr ia l o r v e n tr ic u la r a r r h y th m ia s
C. V e n o u s c a n n u la s c a n im p e d e v e n o u s r e tu r n to th e h e a r t
D. V e n o u s c a n n u la s c a n c a u s e s u p e r io r v e n a c a v a s y n d r o m e

33. F o llo w in g in itia tio n o f c a r d io p u lm o n a r y b y p a s s (C P B ) f o r a o r tic v a lv e r e p la c e m e n t, y o u n o tic e


th e m e a n a r te r ia l p r e s s u r e (M A P ) c o n s is te n tly a b o v e 1 0 0 m m H g . T h e m o s t a p p r o p r ia te n e x t
s te p is

A. It is n o r m a l, a n d n o a c tio n is n e e d e d
B. P u m p f lo w s h o u ld b e d e c r e a s e d to d e c r e a s e th e b l o o d p r e s s u r e
C. It is u s u a lly c a u s e d b y a n a ir l o c k in th e a r te r ia l c a n n u la
D. A d m in is te r m id a z o la m to p r e v e n t a w a r e n e s s

34. W h ic h o f th e f o ll o w i n g is n o t a n in d ic a tio n o f lo w f lo w r a te s u n d e r c a r d io p u lm o n a r y b y p a s s
(C P B )?

A. S vo2 >80%
B. P r o g r e s s i v e m e ta b o lic a lk a lo s is
C. L o w u r in e o u tp u t
D. H y p o x e m ia n o tic e d o n a n in - lin e v e n o u s o x y g e n s a tu r a tio n m o n ito r

35. D is c o n tin u in g v e n tila tio n p r e m a tu r e ly b e f o r e fu ll f lo w is a c h ie v e d o n c a r d io p u lm o n a r y b y p a s s


(C P B ) c a u s e s

A. A r ig h t - to - le f t s h u n t le a d in g to h y p o x e m ia
B. In c re a se d d ead sp ace
C. H e lp s to in c r e a s e v e n o u s r e tu r n v ia th e v e n o u s o u tf lo w c a n n u la
D. A id s th e s u r g e o n to v is u a liz e a n d c a n n u la te th e c o r o n a r y s in u s

36. W h ic h o f th e f o ll o w i n g is th e m o s t s e n s itiv e to d e te c t a ir b u b b le s a t th e te r m in a tio n o f


c a r d io p u lm o n a r y b y p a s s (C P B )?

A. T r a n s e s o p h a g e a l e c h o c a r d io g r a p h y (T E E )
B. D o p p le r u l tr a s o n o g r a p h y
C. M a n u a l v is u a liz a tio n
D. E p ia o r tic e c h o c a r d io g r a p h y

37. S w e a tin g d u r in g th e r e w a r m in g p h a s e o f te r m in a tio n o f c a r d io p u lm o n a r y b y p a s s (C P B )

A. Im p lie s lig h t a n e s th e s ia
B. Is a h y p o th a la m ic r e s p o n s e to p e r f u s io n w ith b l o o d th a t is o f te n a t 3 9 °C
C. N e c e s s ita te s c o o lin g th e o p e r a tin g r o o m
D. C a n b e p r e v e n te d b y u s in g a f o r c e d a ir - w a r m in g d e v ic e d u r in g th e s u r g e r y

38. U s e o f c o r r e c t e d g a s te n s io n s d u r in g h y p o th e r m ia
A. Is c a lle d p H -s ta t m a n a g e m e n t
B. P r e s e r v e s c e r e b r a l a u to r e g u la t io n
C. I m p r o v e s m y o c a r d ia l p r e s e r v a tio n
D. Is d o n e b y a d d in g s o d iu m b ic a r b o n a te to th e v e n o u s r e s e r v o i r

39. I n f u s io n o f n itr o g l y c e r i n a t th e te r m in a tio n o f c a r d io p u lm o n a r y b y p a s s (C P B )

A. D ila te s th e c o r o n a r y v e s s e ls a n d h e lp s im p r o v e c o r o n a r y f lo w
B. S p e e d s th e r e w a r m in g p r o c e s s a n d d e c r e a s e s l a r g e te m p e r a tu r e g r a d ie n ts
C. Is a n o ld te c h n iq u e th a t p r o d u c e s u n n e c e s s a r y h e m o d y n a m ic c h a n g e s
D. I m p r o v e s r e n a l b l o o d f lo w

40. G e n e r a l g u id e lin e s f o r s e p a r a tio n f r o m c a r d io p u lm o n a r y b y p a s s (C P B ) in c lu d e a ll th e


f o ll o w i n g , e x c e p t

A. C o r e b o d y te m p e r a tu r e o f a t le a s t 3 4 ° C
B. S ta b le h e a r t r h y th m o r p a c e r r h y th m
C. H e a r t r a te a r o u n d 8 0 to 1 0 0 b p m
D. A d e q u a te v e n tila tio n w ith 1 0 0 % O 2

41. T im in g o f in f la tio n o f a n in tr a - a o r ti c b a l l o o n p u m p (IA B P ) s h o u ld b e

A. J u s t b e f o r e th e d ic r o tic n o tc h
B. J u s t a fte r th e d ic r o tic n o tc h
C. A s s o o n a s th e d o w n w a r d s lo p e o f a o r tic p u ls e b e g in s
D. S y n c h r o n iz e d w ith th e r i s e o f a o r tic p u ls e

42. A 6 8 - y e a r - o ld p a tie n t w ith a n in f e c te d p r o s th e tic a o r tic v a lv e u n d e r w e n t a v a lv e r e p la c e m e n t.


P o s t - c a r d i o p u l m o n a r y b y p a s s (C P B ), h is c e n tr a l v e n o u s p r e s s u r e (C V P ), p u lm o n a r y c a p il la r y
w e d g e p r e s s u r e (P C W P ), a n d s y s te m ic v a s c u la r r e s is ta n c e (S V R ) a r e lo w , w h ile th e c a r d ia c
o u tp u t (C O ) is h ig h . T h e n e x t s te p in m a n a g e m e n t o f th is p a tie n t is

A. A d d in g a n in o tr o p e
B. A d d in g in tr a - a o r ti c b a l l o o n p u m p (IA B P )
C. A d d in g a p u lm o n a r y v a s o d il a to r
D. I n c r e a s in g th e h e m a to c r it

43. A fte r n e u tr a liz in g h e p a r in , w h ic h o f th e f o ll o w i n g is th e fa te o f th e h e p a r i n - p r o t a m i n e r e a c tio n


p r o d u c t?

A. T h e o n ly p r o d u c t r e m a in in g w ill b e w a te r s in c e it is a n a c id - b a s e r e a c tio n
B. It is r e m o v e d b y th e r e tic u lo e n d o th e lia l s y s te m
C. It is r e m o v e d b y th e k id n e y s
D. It is e x c r e te d u n c h a n g e d v ia g a s tr o in te s tin a l (G I) tr a c t

44. H e p a r in r e b o u n d a fte r te r m in a tio n o f c a r d io p u lm o n a r y b y p a s s (C P B ) is d u e to


A. R e d is tr ib u tio n o f p r o ta m in e to p e r ip h e r a l c o m p a r tm e n ts
B. R e d is tr ib u tio n o f h e p a r in to c e n tr a l c o m p a r tm e n t
C. B o th A a n d B a r e tru e
D. B o th A a n d B a r e fa ls e ; it is d u e to in a d e q u a te p r o ta m in e d o s in g

45. D D A V P ( d e s m o p r e s s in ) a d m in is tr a tio n c a n in c r e a s e th e a c tiv ity o f a ll th e f o ll o w i n g f a c to r s ,


except

A. F a c to r V II
B. F a c to r V III
C. F a c to r X II
D. v o n W ille b r a n d f a c to r

46. In th e f i r s t f e w p o s to p e r a tiv e h o u r s a fte r a n o p e n h e a r t s u r g e r y , th e e m p h a s is is o n

A. M o n ito r in g f o r e x c e s s iv e p o s to p e r a tiv e b le e d in g
B. M a in ta in in g a d e q u a te u r in e o u tp u t
C. T r y in g f o r a n e a r ly e x tu b a tio n
D. M a in ta in in g e u th e r m ia

47. In h a le d n itr ic o x id e (N O ) a t 6 0 p p m h a s a ll o f th e f o ll o w i n g e ffe c ts , e x c e p t

A. D r o p in s y s te m ic v a s c u la r r e s is ta n c e (S V R )
B. D r o p in p u lm o n a r y v a s c u la r r e s is ta n c e (P V R )
C. I m p r o v e m e n t in c a r d ia c o u tp u t
D. B e tte r r i g h t c o r o n a r y p e r f u s io n

48. D o n o r - r e c i p i e n t c o m p a tib ility in c a r d ia c tr a n s p la n ta tio n is b a s e d o n a ll, e x c e p t

A. H e a r t s iz e
B. A B O b l o o d - g r o u p ty p in g
C. C y to m e g a lo v ir u s s e r o l o g y
D. T is s u e c r o s s m a tc h in g

49. T h e c e n tr a l v e n o u s p r e s s u r e (C V P ) w a v e f o r m in c a r d ia c ta m p o n a d e is c h a r a c te r iz e d b y

A. A b o litio n o f X d e s c e n t
B. A b o litio n o f Y d e s c e n t
C. C V w a v e fo rm
D. T a ll C w a v e s

50. In c o n s tr ic tiv e p e r ic a r d itis ,

A. I n c r e a s e d d ia s to lic f il lin g d o e s n o t o c c u r , in c o n tr a s t to a c u te ta m p o n a d e
B. T h e Y d e s c e n t is a b s e n t in C V P w a v e f o r m
C. P u ls u s p a r a d o x u s is u n c o m m o n
D. D if f u s e T -w a v e a b n o r m a litie s a r e a r a r e s ig n
51. A 2 5 - y e a r - o ld m a le w ith a f a m ily h is to r y o f s u d d e n c a r d ia c d e a th s is u n d e r g o in g a
la p a r o s c o p ic a p p e n d e c to m y . I m m e d ia te ly a fte r in d u c tio n a n d in tu b a tio n , y o u n o tic e a h e a r t r a te
o f 1 2 0 b p m a n d b l o o d p r e s s u r e o f 6 0 /4 0 m m H g , w ith a n o r m a l c a p n o g r a m . Y o u s u s p e c t th e
p a tie n t h a s id io p a th ic h y p e r tr o p h ic s u b a o r tic s te n o s is . W h ic h o f th e f o ll o w i n g m a n e u v e r s is
m o s t lik e ly to h e lp th is p a tie n t’s h e m o d y n a m ic s ?

A. L o w e r in g th e h e a d e n d o f th e b e d a n d a d m in is te r in g 10 m g o f e p h e d r in e IV
B. A d m in is te r in g a b o lu s o f 1 L o f n o r m a l s a lin e a n d e s m o l o l 10 m g IV
C. A d m in is te r in g v e r a p a m il 5 m g IV im m e d ia te ly
D. A d m in is te r in g a b o lu s o f n o r m a l s a lin e a n d p h e n y le p h r in e 1 0 0 |ig IV

52. P u lm o n a r y c a p il la r y w e d g e p r e s s u r e (P C W P ) d o e s n o t c o r r e s p o n d to th e le f t- v e n tr ic u la r e n d
d ia s to lic p r e s s u r e (L V E D P ) in a ll o f th e f o ll o w i n g s itu a tio n s , e x c e p t

A. M itra l s te n o s is
B. T r ic u s p id r e g u r g i t a t i o n
C. V e ry h ig h p o s itiv e e n d - e x p i r a t o r y p r e s s u r e (P E E P )
D. L e f t- a tr ia l m y x o m a

53. N o r m a l m ix e d v e n o u s o x y g e n te n s io n i s _______ ( m m H g ):

A. 75
B. 40
C. 45
D. 560

54. T h e o n ly c lin ic a ll y p r o v e n m e th o d to r e d u c e th e r i s k o f p e r io p e r a tiv e m y o c a r d ia l in f a r c tio n


(M I) a n d a s s o c ia te d d e a th is

A. P e r io p e r a tiv e P - b lo c k e r th e r a p y
B. P e r io p e r a tiv e c lo n id in e th e r a p y
C. B o th A a n d B
D. U s e o f e s m o l o l b o lu s e s in tr a o p e r a tiv e ly to k e e p th e h e a r t r a te < 8 0 b p m

55. W h ic h o f th e f o ll o w i n g s ta te m e n ts is f a ls e r e g a r d i n g p e r io p e r a tiv e m y o c a r d ia l i n f a r c tio n (M I)?

A. M o s t p e r io p e r a tiv e M Is o c c u r in th e f i r s t 4 8 to 72 h o u r s p o s to p e r a tiv e ly
B. A 1 -m in u te e p is o d e o f 1 -m m S T -s e g m e n t e le v a tio n o r d e p r e s s io n o n th e E C G in c r e a s e s th e
r i s k f o r c a r d ia c e v e n ts b y 1 0 - f o ld
C. P e r io p e r a tiv e r i s k r e d u c tio n w ith P - b lo c k e r s a n d c lo n id in e is i n f e r i o r to r i s k s tr a tif ic a tio n
w ith in v a s iv e te s tin g , a n g io p la s ty , a n d c o r o n a r y a r te r y b y p a s s g r a f tin g (C A B G )
D. T a c h y c a r d ia (> 1 0 5 b p m ) f o r 5 m in u te s in th e p o s to p e r a tiv e p e r i o d c a n in c r e a s e th e r i s k o f
d e a th b y 1 0 - f o ld

56. W h ic h o f th e f o ll o w i n g is th e m o s t e f f e c tiv e m e a n s o f p r e d ic tin g a p e r io p e r a tiv e c a r d ia c e v e n t?

A. E c h o c a r d io g r a p h y w a ll- m o tio n a b n o r m a litie s


B. E c h o c a r d io g r a p h y e je c tio n f r a c tio n
C. D ip y r id a m o le - th a lliu m s c in tig r a p h y
D. C a r e f u l p r e o p e r a t iv e e v a lu a tio n

57. W h ic h o f th e f o ll o w i n g is m o s t e f f e c tiv e m e th o d o f p r e v e n tin g th e h e m o d y n a m ic c h a n g e s


a s s o c ia te d w ith in tu b a tio n ?

A. B r ie f l a r y n g o s c o p y (< 1 5 s e c o n d s )
B. E s m o lo l 1 m g /k g IV b e f o r e in tu b a tio n
C. L id o c a in e 2 m g /k g b e f o r e in tu b a tio n
D. D e e p e n th e a n e s th e s ia w ith p r o p o f o l 1 m g /k g

58. W h ic h o f th e f o ll o w i n g e v e n ts is n o t lik e ly to a d v e r s e ly a f f e c t h e m o d y n a m ic s in a p a tie n t w ith


m itr a l- v a lv e p r o la p s e ?

A. S y m p a th e tic s tim u la tio n


B. D e c r e a s e d s y s te m ic v a s c u la r r e s is ta n c e
C. H e a d -u p p o s itio n o f th e p a tie n t
D. I n c r e a s e d p u lm o n a r y v a s c u la r r e s is ta n c e

59. A n e s th e tic c o n s id e r a tio n s in a p a tie n t w ith m itr a l r e g u r g i t a t i o n in c lu d e a ll th e f o ll o w i n g , e x c e p t

A. A v o id s u d d e n d e c r e a s e s in h e a r t r a te
B. A v o id s u d d e n d e c r e a s e s in s y s te m ic v a s c u la r r e s is ta n c e (S V R )
C. M in im iz e d r u g - in d u c e d m y o c a r d ia l d e p r e s s io n
D. M o n ito r th e m a g n itu d e o f th e C w a v e o f C V P a s a r e f le c tio n o f m i t r a l - r e g u r g i t a n t f lo w

60. T r e a tm e n t o f p a tie n ts w ith p r o l o n g e d Q T in te r v a l in c lu d e a ll, e x c e p t

A. 0 - B lo c k e r s
B. R ig h t s te lla te g a n g lio n b lo c k
C. A v o id a n c e o f d r u g s th a t p r o l o n g th e Q T in te r v a l
D. A v a ila b ility o f e le c tr ic a l c a r d i o v e r s i o n w h ile th e p a tie n ts a r e u n d e r g o in g s u r g ic a l
p ro c e d u re s

61. A n e s th e tic c o n s id e r a tio n s in p a tie n ts w ith a o r tic s te n o s is in c lu d e a ll, e x c e p t

A. I n tr a - a r te r ia l b l o o d p r e s s u r e m o n ito r in g
B. P r o p h y la c tic a d m in is tr a tio n o f in tr a v e n o u s v a s o c o n s tr ic to r p h e n y le p h r in e
C. A v o id a n c e o f e x tr e m e b r a d y c a r d i a o r ta c h y c a r d ia
D. A v o id a n c e o f s u d d e n in c r e a s e s in s y s te m ic v a s c u la r r e s is ta n c e (S V R )

62. V e n tric u la r p r e m a tu r e b e a ts (V P C s ) c a n b e tr e a te d w ith lid o c a in e ( 1 - 2 m g /k g IV ) w h e n th e y

A. A r e f r e q u e n t ( m o r e th a n s ix p r e m a tu r e b e a ts /m in )
B. A r e m u ltif o c a l
C. T a k e p la c e d u r in g th e a s c e n d in g lim b o f th e T w a v e ( R - o n - T p h e n o m e n o n )
D. A ll o f th e a b o v e

63. W h ic h o f th e f o ll o w i n g d r u g s n e e d s n o t b e a v o id e d in th e a n e s th e tic m a n a g e m e n t o f a p a tie n t


w ith W o l f f - P a r k i n s o n - W h i t e (W P W ) s y n d r o m e ?

A. K e ta m in e
B. P a n c u r o n iu m
C. S u c c in y lc h o lin e
D. D ig ita lis

64. W h ic h o f th e f o ll o w i n g s ta te m e n ts is f a ls e r e g a r d i n g m a n a g e m e n t o f a p a tie n t w ith a n


a u to m a te d im p la n ta b le c a r d io v e r te r d e f ib r il la to r (A IC D )?

A. T h e “ m a g n e t m o d e ” is a lw a y s s a fe
B. T h e g r o u n d p la te s h o u ld b e p la c e d a s fa r a s p o s s ib le f r o m th e p u ls e g e n e r a to r
C. B ip o la r e le c tr o c a u te r y m a y b e u s e d o v e r u n ip o la r e le c tr o c a u te r y to r e d u c e in te r f e r e n c e
b e tw e e n e le c tr o s u r g ic a l c a u te r y a n d th e p a c e m a k e r
D. T h e m a g n e t m o d e m a y p r o d u c e a s y n c h r o n o u s p a c in g a t 9 9 b p m

65. C a r d ia c ta m p o n a d e is c h a r a c te r iz e d b y

A. I n c r e a s e in d ia s to lic f illin g o f th e v e n tr ic le s
B. D e c r e a s e in s tr o k e v o lu m e
C. I n c r e a s e in s y s te m ic b l o o d p r e s s u r e d u e to in c r e a s e d in tr a p e r ic a r d ia l p r e s s u r e f r o m
a c c u m u la tio n o f f lu id in th e p e r ic a r d ia l s p a c e
D. S y s to lic d y s fu n c tio n , a n d n o t d ia s to lic d y s fu n c tio n , is th e p r i m a r y p r o b le m

66. A n 8 1 - y e a r - o ld p a tie n t w ith a h is to r y o f m o d e r a te a o r tic r e g u r g i t a t i o n is u n d e r g o in g a


c o r o n a r y a r te r y b y p a s s g r a f tin g (C A B G ). T h e s u r g e o n d e c id e s n o t to v e n t th e le f t v e n tr ic le . Y o u
th in k th is is a w r o n g d e c is io n , a n d y o u r a r g u m e n ts in c lu d e a ll th e f o ll o w i n g , e x c e p t

A. V e n tin g c a n b e d o n e th r o u g h a d r a in p la c e d f r o m th e r i g h t s u p e r io r p u lm o n a r y v e in in to th e
le f t v e n tr ic le
B. V e n tin g c a n b e d o n e th r o u g h a p u lm o n a r y v e n o u s d r a in
C. R e t r o g r a d e f lo w th r o u g h th e a o r tic v a lv e c o u ld c a u s e le f t- v e n tr ic u la r d is te n s io n
D. V e n tin g d o n e b y a s p ir a tin g f r o m th e a n te g r a d e c a r d io p le g i a lin e p la c e d in th e p r o x im a l
a s c e n d in g a o r ta w ill n o t b e h e lp f u l

67. C e n tr if u g a l p u m p s a r e s u p e r io r to r o l l e r p u m p s b e c a u s e o f a ll, e x c e p t

A. T h e y a r e le s s tr a u m a tic to b l o o d c e lls
B. T h e y d o n o t p u m p a ir b u b b le s s e c o n d a r y to a ir b e in g le s s d e n s e th a n b l o o d
C. T h e y a r e a f te r lo a d - d e p e n d e n t, a n d a v o id th e r i s k o f lin e r u p tu r e w ith c la m p in g o f th e
a r te r ia l in f l o w c ir c u it
D. R o lle r p u m p s c o m p r e s s th e f lu i d - f ille d tu b in g b e tw e e n th e r o l l e r a n d c u r v e d m e ta l b a c k
p la te a n d h e n c e a v o id a ir
68. During cardiopulmonary bypass (CPB), the nasopharyngeal temperature is 28°C, the
hematocrit is 20%, the temperature corrected Paco2 is 50 mm Hg, and the uncorrected Paco2 is
60 mm Hg. The most appropriate management is to
A. Administer additional opioid
B. Administer packed red blood cells to increase hematocrit to 25%
C. Further decrease the patient’s temperature
D. Increase fresh-gas flo w to the oxygenator

69. Two days after coronary artery bypass grafting, a 62-year-old man remains sedated,
endotracheally intubated, and mechanically ventilated. Over the next 3 hours, Pao2 decreases
from 90 to 70 mm Hg at an Fio 2 of 0.7, peak inspiratory pressure measured proxim ally in the
ventilator circuit increases from 40 to 66 cm H2O, and plateau pressure remains unchanged at
30 cm H2O. Which of the follow ing is the most likely case of these changes?

A. Adult respiratory distress syndrome (ARDS)


B. Bronchial mucus plugging
C. Left-ventricular failure
D. Tension pneumothorax

70. Regarding the maintenance of blood pressure during cardiopulmonary bypass (CPB), which of
the follow ing is false?
A. Lower blood pressures may reduce cerebral blood flo w and reduce emboli load to the
brain, while higher pressures may improve cerebral blood flo w but cause more emboli
B. Pressures less than 40 mm Hg are avoided if possible in adults
C. Pressures higher than 90 mm Hg are used during rewarming
D. Pressures up to 90 mm Hg may be used in patients with cerebral vascular disease

71. During total cardiopulmonary bypass, metabolic acidosis and decreasing mixed venous oxygen
saturation are noted. The most likely cause is
A. Hypothermia
B. Hypoperfusion
C. Rewarming
D. Light anesthesia

72. While monitoring coronary sinus pressure during retrograde cardioplegia,


A. If the pressure at the distal tip of the coronary sinus catheter during cardioplegia
administration at 200 mL/min is equal to central venous pressure, the catheter is not in the
coronary sinus but is most likely in the pulmonary artery
B. If the pressure is very high (>100 mm Hg), the coronary sinus catheter is in the left
ventricle
C. If the pressure in the coronary sinus catheter is 40 to 60 mm Hg during a 200-mL/min
infusion, the catheter is correctly positioned
D. If th e c a th e te r is p la c e d to o d is ta lly , d e liv e r y o f c a r d io p le g i a to th e le f t v e n tr ic le w ill b e
c o m p r o m is e d a n d r e s u lt in le f t- v e n tr ic u la r d y s f u n c tio n

73. T h e e le c tr o m e c h a n ic a lly q u ie t h e a r t a t 2 2 ° C c o n s u m e s o x y g e n a t a r a te o f

A. 2 m L /1 0 0 g /m in
B. 8 m L /1 0 0 g /m in
C. 0 .3 m L /1 0 0 g /m in
D. 0.1 m L /1 0 0 g /m in

74. A d d itio n a l s u p p le m e n ta l a n e s th e tic s a n d m u s c le r e la x a n ts s h o u ld b e a d m in is te r e d

A. A t in s titu tio n o f c a r d io p u lm o n a r y b y p a s s (C P B )
B. A t r e w a r m in g
C. B o th A a n d B
D. In th e e a r ly p e r i o d a fte r c o n c lu s io n o f C P B

75. T h e m o s t c o m m o n h e m o d y n a m ic a b n o r m a lity a fte r c a r d io p u lm o n a r y b y p a s s (C P B ) is

A. L o w c a r d ia c o u tp u t
B. L o w s y s te m ic v a s c u la r r e s is ta n c e (S V R )
C. H ig h p u lm o n a r y v a s c u la r r e s is ta n c e
D. L o w h e a r t r a te

76. A 5 7 - y e a r - o ld m a le is u n d e r g o in g c o r o n a r y a r te r y b y p a s s g r a f tin g ( le f t in te r n a l m a m m a r y
a r te r y to le f t a n te r io r d e s c e n d in g a r te r y ) . A fte r te r m in a tio n o f c a r d io p u lm o n a r y b y p a s s (C P B ),
y o u n o tic e a p r o m i n e n t V w a v e in th e p u lm o n a r y a r te r y o c c lu s io n p r e s s u r e (P A O P ) tr a c in g .
T h e m o s t lik e ly r e a s o n f o r th e f in d in g is

A. L e f t- v e n tr ic u la r d y s f u n c tio n
B. R ig h t- v e n tr ic u la r d y s f u n c tio n
C. C a r d ia c ta m p o n a d e
D. P o s te r io r p a p illa r y m u s c le d y s f u n c tio n
CHAPTER 11 ANSWERS

1. B. T h e n o r m a l v e n tr ic u la r c e l l - r e s t i n g m e m b r a n e p o te n tia l is - 8 0 to - 9 0 m V . N a - K A T P a s e
b o u n d to th e m e m b r a n e is r e s p o n s ib le f o r c o n c e n tr a tin g K+ i n t r a c e l lu la r ly a n d in e x c h a n g e f o r
N a a n d m a in ta in in g th is r e s tin g - m e m b r a n e p o te n tia l. A c tio n p o te n tia l ( d e p o la r iz a tio n ) o c c u r s
w h e n c e ll m e m b r a n e b e c o m e s le s s n e g a tiv e a n d c r o s s e s a th r e s h o ld v a lu e . T h is d e p o la r iz a tio n
r a is e s th e m e m b r a n e p o te n tia l o f th e m y o c a r d ia l c e ll, s o m e tim e s a s h ig h a s + 2 0 m v . T h e c a r d ia c
a c tio n p o te n tia l is s lig h tly d if f e r e n t f r o m n e u r o n a l a c tio n p o te n tia l in th a t it h a s a c h a r a c te r is tic
s p ik e a n d p la te a u a p p e a r a n c e . T h e s p ik e p o r t i o n o f th is a c tio n p o te n tia l is p r o d u c e d b y o p e n in g
o f fa s t s o d iu m c h a n n e ls a lo n g w ith a d e c r e a s e d p e r m e a b il ity to p o ta s s iu m a n d th e p la te a u
p o r t i o n ( 0 .2 - 0 .3 s e c o n d s ) is d u e to o p e n in g o f s lo w e r c a lc iu m c h a n n e ls . A fte r d e p o la r iz a tio n ,
th e s o d iu m a n d c a lc iu m c h a n n e ls c lo s e a n d th e m e m b r a n e p e r m e a b il ity to p o ta s s iu m is r e s to r e d .
T h is r e s to r e s th e r e s tin g - m e m b r a n e p o te n tia l to its b a s e lin e . S p o n ta n e o u s ly d e p o la r iz i n g c e lls ,
r e s p o n s ib le f o r th e m y o c a r d ia l rh y th m , d o so p r i m a r i l y b y in tr in s ic s lo w le a k a g e o f c a lc iu m
in to c e lls a id e d b y le a k y N a c h a n n e ls m o v in g N a+ in (T a b le 1 1 -1 ).

Table 11-1

A C T I O N P O T E N T IA L P H A S E NAM E N E T IO N M O V E M E N T

0 Rapid upstroke Na+ in (relative impermeability to K+)


1 Early rapid repolarization K+ out (increased permeability to K+ transien
2 Plateau (a part of repolarization) Ca++ in
3 Final repolarization K+ out of cells
4 Resting-membrane potential Na+ in and K+ out

2. A. H a lo th a n e a n d is o f lu r a n e d e p r e s s S A n o d e a u to m a tic ity a n d m a k e A V n o d e r e f r a c to r y . B y
g iv in g a n a n tic h o lin e r g ic , w e s tim u la te d th e c o n d u c tio n s y s te m o f th e h e a r t, b u t S A a n d AV
n o d e s h a v e b e e n s u p p r e s s e d b y th e in h a la tio n a l a g e n t. S o th e n e x t tis s u e in th e c o n d u c tin g
p a th w a y ( ju n c tio n a l p a c e m a k e r s ) ta k e s o v e r a n d p r o d u c e s ju n c tio n a l rh y th m . W h ile th e
d e p r e s s io n o f S A a n d A V n o d e s b y in h a la tio n a l a g e n ts is w e ll k n o w n , th e e f f e c t o f in h a la tio n a l
a g e n ts o n P u r k in je f ib e r s a n d v e n tr ic u la r m y o c a r d iu m is u n p r e d ic ta b le w ith r e p o r ts o f b o th
a r r h y th m ia - in d u c in g a n d a n tia r r h y th m ic e ffe c ts . A r r h y t h m o g e n ic ity b y in h a la tio n a l a g e n ts is
d u e to p o te n tia tio n o f a c tio n o f c a te c h o la m in e s , a n d th e d ir e c t d e p r e s s io n o f c a lc iu m c h a n n e ls
r e n d e r s s o m e a n tia r r h y th m ic e ffe c t. O p io id s d e p r e s s c a r d ia c c o n d u c tio n , in c r e a s e A V n o d e
r e f r a c to r in e s s , a n d p r o l o n g th e d u r a tio n o f P u r k in je f i b e r - a c t i o n p o te n tia l.

3. A. T h e th e r a p e u tic e ffe c ts o f l o w c o n c e n tr a tio n s o f lid o c a in e tu r n to x ic a t h ig h e r


c o n c e n tr a tio n s — th e y b in d to f a s t N a c h a n n e ls a n d d e p r e s s c o n d u c tio n . If w e in c r e a s e th e
c o n c e n tr a tio n fu r th e r , th e y d e p r e s s th e a u to m a tic ity o f h e a r t b y its e f f e c t o n s in o a t r ia l n o d e . T h is
is v e r y d if f e r e n t f r o m th e m o r e p o te n t lo c a l a n e s th e tic s lik e b u p iv a c a in e a n d r o p iv a c a in e , w h ic h
c a u s e to x ic ity b y its e f f e c t o n P u r k in je f ib e r s a n d v e n tr ic u la r m u s c le . B u p iv a c a in e b in d s
in a c tiv a te d fa s t s o d iu m c h a n n e ls a n d d is s o c ia te s f r o m th e m s lo w ly . Its e ffe c ts c a n b e s in u s
b r a d y c a r d ia , s in u s n o d e a r r e s t, o r m a lig n a n t v e n tr ic u la r a r r h y th m ia .
4. B. A ll a n e s th e tic a g e n ts c a n d e p r e s s cardiac contractility. T h is o c c u r s b y a lte r a tio n s in th e
in tr a c e llu la r c o n c e n tr a tio n o f c a lc iu m a s f o llo w s :
I n h a la tio n a l a g e n ts : d e c r e a s in g th e e n tr y o f c a lc iu m in to c e lls b y a f f e c tin g b o th T - a n d L -ty p e
c a lc iu m c h a n n e ls , a lte r in g th e k in e tic s o f c a lc iu m r e le a s e a n d u p ta k e in to th e s a r c o p la s m ic
r e tic u lu m , a n d d e c r e a s in g th e s e n s itiv ity o f c o n tr a c tile p r o te in s to c a lc iu m . T h e s e e ffe c ts a r e
m o r e a p p a r e n t w ith h a lo th a n e th a n w ith m o d e r n in h a la tio n a l a g e n ts lik e is o f lu r a n e , s e v o f lu r a n e ,
a n d d e s f lu r a n e . F a c to r s th a t c a n w o r s e n th is c a r d ia c d e p r e s s io n in c lu d e h y p o c a lc e m ia , a -
a d r e n e r g i c b lo c k a d e , a n d c a lc iu m c h a n n e l b lo c k e r s .
N itr o u s o x id e : r e d u c e s th e in tr a c e llu la r c a lc iu m c o n c e n tr a tio n ( d o s e -d e p e n d e n t).
I n tr a v e n o u s - in d u c tio n a g e n t k e ta m in e : a g e n t w ith n o s ig n if ic a n t m y o c a r d ia l d e p r e s s io n , e x c e p t
in c r itic a lly ill p a tie n ts w ith d e p le te d c a te c h o la m in e s , w h e r e it a c ts a s a d ir e c t m y o c a r d ia l
d e p re s s a n t.
L o c a l a n e s th e tic a g e n ts : r e d u c e c a lc iu m in f lu x a n d r e le a s e in a d o s e - d e p e n d e n t f a s h io n .
B u p iv a c a in e , te tr a c a in e , a n d r o p iv a c a in e c a u s e g r e a te r d e p r e s s io n th a n lid o c a in e a n d
c h lo r o p r o c a in e .

5. C. T h e C V P w a v e f o r m c o n s is ts o f th r e e p o s itiv e w a v e f o r m s c a lle d a, c, a n d v a n d tw o
n e g a tiv e s lo p e s c a lle d th e x a n d y d e p r e s s io n s .
a wave atrial contraction
c wave cusps bulging into the right atrium
x descent atrial relaxation during ventricular systole
v wave venous filling of the right atrium
y descent atrial emptying when tricuspid valve opens

6. D. V e n tric u la r s y s to lic f u n c tio n is d o c u m e n te d m o s t c o m m o n ly a s c a r d ia c o u tp u t o r e je c tio n


f r a c tio n . C a r d ia c o u tp u t is d e f in e d a s th e v o lu m e o f b l o o d p u m p e d b y th e h e a r t p e r m in u te .
N o r m a lly , th e r i g h t a n d le f t v e n tr ic le s h a v e th e s a m e o u tp u t. C O = S V x H R , w h e r e C O is th e
c a r d ia c o u tp u t, S V is th e s tr o k e v o lu m e (th e v o lu m e p u m p e d p e r c o n tr a c tio n ) , a n d H R is h e a r t
ra te . V a ria tio n s in b o d y s iz e c a n le a d to a m b ig u ity if w e j u s t u s e c a r d ia c o u tp u t a s a m e a s u r e .
T h is c a n b e a v o id e d b y u s in g c a r d ia c in d e x : C I = C O /B S A , w h e r e C I is th e c a r d ia c in d e x a n d
B S A is th e to ta l b o d y s u r f a c e a r e a . B S A is u s u a lly o b ta in e d f r o m n o m o g r a m s b a s e d o n h e ig h t
a n d w e ig h t. N o r m a l C I is 2.5 to 4 .2 L / m in /m 2. A s y o u c a n s e e , th e r e is a w id e r a n g e f o r C I a n d
th e p a tie n t s h o u ld h a v e a g r o s s v e n tr ic u la r im p a ir m e n t p r i o r to it b e in g e v id e n t o n C I. M ix e d
v e n o u s o x y g e n s a tu r a tio n is id e a l ly o b ta in e d f r o m a PA c a th e te r. A b e tte r e s tim a te o f v e n tr ic u la r
p e r f o r m a n c e c a n b e o b ta in e d if w e s u b je c t th e v e n tr ic le s to s o m e s tr e s s lik e e x e r c is e . T h is w ill
r e v e a l u n d e r ly in g in a b ility o f th e h e a r t to d e liv e r a d e q u a te o x y g e n to th e tis s u e s a n d c a n b e
n o te d a s a f a llin g m ix e d v e n o u s o x y g e n s a tu r a tio n . In a d e q u a te tis s u e p e r f u s io n r e la tiv e to
d e m a n d is c a u s in g th e d r o p in m ix e d v e n o u s s a tu r a tio n . T h u s , in th e a b s e n c e o f h y p o x ia o r
s e v e r e a n e m ia , m e a s u r e m e n t o f m ix e d v e n o u s o x y g e n te n s io n ( o r s a tu r a tio n ) is th e b e s t
d e te r m in a tio n o f th e a d e q u a c y o f c a r d ia c o u tp u t.

7. C. V e n tric u la r f il lin g is in f lu e n c e d b y b o th h e a r t r a te a n d rh y th m . S in c e th e tim e s p e n t in


d ia s to le is h ig h e r th a n th e tim e s p e n t in s y s to le , a n y in c r e a s e in h e a r t r a te h a s m o r e e f f e c t o n th e
d ia s to lic f illin g tim e m o r e th a n th e s y s to lic e je c tio n tim e . A t v e r y h ig h h e a r t r a te s (> 1 2 0 b p m ) in
a d u lts , th e le f t- v e n tr ic u la r f il lin g is s ig n if ic a n tly im p a ir e d b y th e s h e e r d e c r e a s e in d u r a tio n o f
d ia s to le . In a d d itio n , a tr ia l c o n tr a c tio n (k ic k ) c o n tr ib u te s a b o u t 2 0 % to 3 0 % o f th e v e n tr ic u la r
f illin g in a n o r m a l h e a r t. A n y c o n d itio n th a t a ffe c ts th e a tr ia l c o n tr a c tio n , lik e a tr ia l
f ib r illa tio n /f lu tte r , o r a lte r s th e tim in g o f a tr ia l k ic k , w ill n e g a te th is c o n tr ib u tio n a n d c a n h a v e
s ig n if ic a n t h e m o d y n a m ic c o n s e q u e n c e s in s o m e p a tie n ts . T h e a tr ia l c o n tr ib u tio n to v e n tr ic u la r
f illin g is m o r e im p o r ta n t in p a tie n ts w ith r e d u c e d v e n tr ic u la r c o m p lia n c e w h o d e p e n d o n a c tiv e
f illin g w ith a tr ia l c o n tr a c tio n th a n p a s s iv e f il lin g o f th e v e n tr ic le f o r a d e q u a te p r e lo a d .

8. A. A f te r lo a d is th e f o r c e a g a in s t w h ic h v e n tr ic le is p u s h in g th e b l o o d o u t. It c a n b e d e n o te d b y
th e v e n tr ic u la r - w a ll te n s io n d u r in g s y s to le o r im p e d a n c e o f th e a r te r ia l tre e . V e n tr ic u la r - w a ll
te n s io n c a n b e c a lc u la te d b y L a p la c e la w :

C ir c u m f e r e n tia l s tr e s s = in tr a v e n tr ic u la r p r e s s u r e x v e n tr ic u la r r a d iu s /2 x w a ll th ic k n e s s

T h is r e la tio n s h ip is a p p lic a b le to s p h e r ic a l s tr u c tu r e s , b u t c a n b e a p p lie d to le f t v e n tr ic le as


w e ll, w h ic h is a p r o la p s e d e llip s o id . A n y in c r e a s e in v e n tr ic u la r r a d iu s a s in a d ila tio n in c r e a s e s
th e w a ll te n s io n . H o w e v e r, a n y in c r e a s e in th ic k n e s s ( h y p e r tr o p h y ) d e c r e a s e s th e w a ll te n s io n .
T h is is a p r o te c tiv e m e c h a n is m s e e n in p a tie n ts w ith lo n g - s ta n d in g h y p e r te n s io n o r a o r tic
s te n o s is in a n a tte m p t to d e c r e a s e th e w a ll te n s io n .

9. C. R e c o m m e n d e d d o s e o f h e p a r in b e f o r e in itia tio n o f c a r d io p u lm o n a r y b y p a s s is 3 0 0 to 4 0 0
U /k g . T h e d o s e is g iv e n to a c h ie v e a n a c tiv a te d c lo ttin g tim e o f 4 0 0 to 4 5 0 s e c o n d s .

10. B. T h e S A n o d e is s u p p lie d b y th e r i g h t c o r o n a r y a r te r y in 6 0 % o f in d iv id u a ls , a n d b y th e le f t
a n te r io r d e s c e n d in g a r te r y in 4 0 % o f th e in d iv id u a ls . T h e A V n o d e is s u p p lie d b y th e r i g h t
c o r o n a r y a r te r y in 8 5 % o f in d iv id u a ls , a n d b y th e c ir c u m f le x a r te r y in 1 5 % o f in d iv id u a ls .

11. C. B a r o r e c e p to r s h a v e a n im p o r ta n t r o l e in a c u te r e g u la tio n o f b l o o d p r e s s u r e . T h e y a r e
lo c a te d at th e b if u r c a tio n o f th e c o m m o n c a r o tid a n d in th e a o r tic a rc h . T h e s e r e c e p to r s s e n s e a n
in c r e a s e in b l o o d p r e s s u r e a n d e n h a n c e th e v a g a l to n e , th e r e b y in h ib itin g s y s te m ic
v a s o c o n s tr ic tio n . T h is is c a lle d th e b a r o r e c e p t o r r e f le x . T h e a f f e r e n t p a th w a y f o r th e
b a r o r e c e p t o r r e f le x is v ia a b r a n c h o f th e g lo s s o p h a r y n g e a l n e r v e , s o m e tim e s c a lle d th e H e r in g
n e r v e . T h e a f f e r e n t p a th w a y f o r b a r o r e c e p t o r r e f le x f r o m th e a o r tic r e c e p to r s tr a v e ls a lo n g th e
v a g u s n e r v e . C h a n g e s in b l o o d p r e s s u r e c a u s e d b y a c u te e v e n ts lik e c h a n g e in p o s tu r e a r e
m in im iz e d p r i m a r i l y b y th e c a r o tid b a r o r e c e p t o r b e tw e e n m e a n a r te r ia l p r e s s u r e s o f 8 0 a n d 1 6 0
m m H g . H o w e v e r, r e a d a p ta tio n to c h a n g e s in a c u te b l o o d p r e s s u r e o c c u r s o v e r th e c o u r s e o f 1
to 2 d a y s , m a k in g th is r e f le x in e f f e c tiv e f o r l o n g - t e r m b l o o d p r e s s u r e c o n tr o l. A ll v o la tile
a n e s th e tic s d e p r e s s th e n o r m a l b a r o r e c e p t o r r e s p o n s e , le s s so w ith is o f lu r a n e a n d d e s f lu r a n e .

12. A. T h e b u n d le o f H is is th e o n ly p a r t o f th e c a r d ia c c o n d u c tin g s y s te m , w h ic h h a s a d u a l b lo o d
s u p p ly d e r iv e d f r o m th e p o s te r io r d e s c e n d in g a r te r y (P D A ) a n d th e le f t a n te r io r d e s c e n d in g
(L A D ) a rte ry . B lo o d s u p p ly to th e h e a r t is f r o m th e r i g h t a n d le f t c o r o n a r y a r te r ie s . T h e r i g h t
c o r o n a r y a r te r y (R C A ) n o r m a l l y s u p p lie s th e r ig h t a tr iu m , m o s t o f th e r ig h t v e n tr ic le , a n d th e
i n f e r i o r w a ll o f th e le ft v e n tr ic le . In 8 5 % o f p e r s o n s , th e P D A , w h ic h s u p p lie s p a rt o f th e
in te r v e n tr ic u la r s e p tu m a n d i n f e r i o r w a ll, a r is e s f r o m th e R C A , a n d th e s e p e o p le a r e s a id to
h a v e a r ig h t - d o m in a n t c ir c u la tio n . In th e r e m a in in g 1 5 % o f p e r s o n s , th e P D A a r is e s f r o m th e
le f t c o r o n a r y a r te r y a n d is a p p r o p r ia te ly la b e le d le f t- d o m in a n t c ir c u la tio n .
T h e le f t c o r o n a r y a r te r y n o r m a l l y s u p p lie s th e le f t a tr iu m a n d m o s t o f th e in te r v e n tr ic u la r
s e p tu m a n d le f t v e n tr ic le . T h e le f t m a in c o r o n a r y a r te r y d iv id e s in to th e L A D a r te r y a n d th e
c ir c u m f le x (C X ) a rte ry . T h e L A D a r te r y s u p p lie s th e s e p tu m a n d a n te r io r le f t- v e n tr ic u la r w a ll,
a n d th e C X a r te r y s u p p lie s th e la te r a l w a ll.

13. C. A u to r e g u la to r y n a tu r e o f th e m y o c a r d iu m m a k e s th e m y o c a r d ia l o x y g e n d e m a n d a n
im p o r ta n t d e te r m in a n t o f m y o c a r d ia l b l o o d flo w . P r e s s u r e w o r k u s e s m o s t o f th e o x y g e n , 6 5 % ,
f o ll o w e d b y b a s a l r e q u ir e m e n ts = 2 0 5 , v o lu m e w o r k = 1 5 % , w ith o n ly 1 % o f th e s u p p lie d
o x y g e n b e in g u s e d f o r e le c tr ic a l a c tiv ity . T h e m y o c a r d iu m a ls o h a s a v e r y h ig h e x tr a c tio n r a tio .
It e x tra c ts 6 5 % o f th e o x y g e n in a r te r ia l b lo o d , c o m p a r e d w ith 2 5 % in m o s t o th e r tis s u e s .
C o r o n a r y s in u s o x y g e n s a tu r a tio n is u s u a lly 3 0 % . H e n c e , a n y d r o p in m y o c a r d ia l o x y g e n
s u p p ly is d e le te r io u s , a s it c a n n o t c o m p e n s a te f o r r e d u c tio n in f lo w b y in c r e a s in g o x y g e n
e x tr a c tio n . F a c to r s in f lu e n c in g th e s u p p ly a n d d e m a n d a r e lis te d in T a b le 1 1-2.

Table 11-2

M Y O C A R D IA L O X Y G E N S U P P L Y M Y O C A R D IA L O X Y G E N D E M A N D

Heart rate
Diastolic time Basal requirements
Aortic diastolic blood pressure Heart rate
Coronary perfusion pressure Wall tension
Ventricular end diastolic pressure Preload
Arterial oxygen content and tension Afterload
Hemoglobin concentration Contractility
Coronary vessel diameter

14. D. H a lo g e n a te d a n e s th e tic a g e n ts a r e in h e r e n t v a s o d il a to r s . T h e ir e f f e c t o n c o r o n a r y b l o o d
f lo w is v a r ia b le a n d d e p e n d s o n a n in te r p la y b e tw e e n th e ir e f f e c t o n b l o o d p r e s s u r e , m e ta b o lic
o x y g e n r e q u ir e m e n ts o f th e m y o c a r d iu m , a n d th e ir d ir e c t v a s o d ila tin g p r o p e r tie s . A lth o u g h th e
m e c h a n is m is n o t c le a r, it m a y in v o lv e a c tiv a tio n o f A T P -s e n s itiv e K+ c h a n n e ls a n d s tim u la tio n
o f a d e n o s in e ( A 1) r e c e p to r s . H a lo th a n e a n d is o f lu r a n e s ta n d a p a rt, a s h a lo th a n e p r i m a r i l y
a ffe c ts l a r g e c o r o n a r y v e s s e ls a n d is o f lu r a n e a ffe c ts m o s tly s m a lle r v e s s e ls . D o s e - d e p e n d e n t
a b o litio n o f a u to r e g u la t io n m a y b e g r e a te s t w ith is o f lu r a n e . A u to n o m ic a lly m e d ia te d
v a s o d il a tio n is s ig n if ic a n t f o r d e s f lu r a n e . S e v o f lu r a n e a p p e a r s to la c k c o r o n a r y v a s o d ila tin g
p r o p e r tie s .

15. C. A c c o r d in g to A C C /A H A g u id e lin e s f o r n o n c a r d ia c s u r g e r y in c a r d ia c p a tie n ts , S u r g e r ie s


c a n b e c la s s if ie d in to h ig h , in te r m e d ia te , a n d l o w r i s k w ith h ig h - r i s k s u r g e r i e s h a v in g > 5 % r i s k
a n d l o w - r i s k s u r g e r i e s h a v in g < 1 % r i s k (T a b le 1 1 -3 ).

Table 11-3 Cardiac Risk Stratification for Noncardiac Surgical Procedures.


H ig h (reported cardiac risk often greater than 5%)
Emergent major operations, particularly in the elderly
Aortic and other major vascular surgery
Peripheral vascular surgery
Anticipated prolonged surgical procedures associated with large fluid shifts and/or blood loss
I n t e r m e d ia t e (reported cardiac risk generally less than 5%)
Carotid endarterectomy
Head and neck surgery
Intraperitoneal and intrathoracic surgery
Orthopedic surgery
Prostate surgery
L ow (reported cardiac risk generally less than 1%)
Endoscopic procedures
Superficial procedure
Cataract surgery
Breast surgery

16. C. C h r o n ic h y p e r te n s iv e p a tie n ts s h o w w id e f lu c tu a tio n s in b l o o d p r e s s u r e o n in d u c tio n


( h y p o te n s io n ) a n d in tu b a tio n ( h y p e r te n s io n ) . D u r a tio n o f l a r y n g o s c o p y < 1 5 s e c o n d s h a s b e e n
s h o w n to p r e v e n t th is h y p e r te n s iv e r e s p o n s e to in tu b a tio n . I n tu b a tio n p e r f o r m e d u n d e r d e e p
a n e s th e s ia is a ls o s h o w n n o t to p r o d u c e s ig n if ic a n t r i s e in b l o o d p r e s s u r e . B u t th is c o m e s a t th e
p r ic e o f h y p o te n s io n . T h e r e a r e s e v e r a l te c h n iq u e s th a t c a n b e u s e d to p r e v e n t s u d d e n s p ik e s in
b l o o d p r e s s u r e o n in tu b a tio n . T o p ic a l a ir w a y a n e s th e s ia , P - b lo c k e r s lik e e s m o l o l 0 .3 to 1.5
m g /k g , s h o r t- a c tin g o p io id s lik e fe n ta n y l 2.5 to 5 |ig /k g , in tr a v e n o u s p r e s e r v a tiv e - f r e e
lid o c a in e a t 1.5 m g /k g h a v e a ll b e e n s h o w n to b e e f f e c tiv e in a tte n u a tin g th e h y p e r te n s iv e
re s p o n se .

17. B. D ir e c t a 1 a g o n is ts lik e p h e n y le p h r in e a r e p r e f e r a b le to in d ir e c t s y m p a th o m im e tic s lik e


e p h e d r in e to tr e a t h y p o te n s io n , f o ll o w i n g in d u c tio n in p a tie n ts w ith u n c o n tr o lle d h y p e r te n s io n
p r e o p e r a tiv e ly . C a te c h o la m in e s — b o th e n d o g e n o u s a n d e x o g e n o u s — c a n p r o d u c e e x a g g e r a te d
h y p e r te n s iv e r e s p o n s e in th e s e p a tie n ts . W e c a n s ta r t w ith s m a ll d o s e s o f p h e n y le p h r in e , f o r
e x a m p le , 2 5 to 50 |ig , p r o v id e d th e h e a r t r a te is n o t to o lo w . If th e h e a r t r a te is lo w , s m a ll d o s e s
o f e p h e d r in e ( 5 - 1 0 m g ) o r e v e n e p in e p h r in e ( 2 - 5 |ig ) m a y b e u s e d . In p a tie n ts w h o a r e o n
a n g io te n s in - r e c e p to r b lo c k e r p r e o p e r a tiv e ly , th e r e f r a c t o r y h y p o te n s io n m a y r e s p o n d o n ly to
v a s o p r e s s in . A v o id in g h ig h h e a r t r a te s a n d p r o l o n g e d h y p e r te n s io n h a s b e e n s h o w n to d e c r e a s e
c a r d io v a s c u la r m o r b id ity .

18. C. C o r o n a r y v a s o d il a tio n p o te n tia l o f d ih y d r o p y r id in e s ( n if e d ip in e , n ic a r d ip in e , n im o d ip in e )


is m u c h g r e a te r th a n th o s e b y v e r a p a m il a n d d iltia z e m . T h e y e v e n e x c e e d n itr a te s in th e ir
v a s o d il a to r y p o te n tia l. P - B lo c k e r s h o w e v e r h a v e n o v a s o d il a to r y a c tio n o n c o r o n a r y b lo o d
v e s s e ls .

19. D. C C B s h a v e s ig n if ic a n t a n e s th e tic im p lic a tio n s . B o th d e p o la r iz i n g a n d n o n d e p o la r iz i n g


n e u r o m u s c u la r - b lo c k in g a g e n ts a r e p o te n tia te d b y C C B s. C C B s a ls o p o te n tia te th e c ir c u la t o r y
e ffe c ts o f v o la tile a g e n ts a n d m a y c a u s e m o r e h y p o te n s io n . B o th v e r a p a m il a n d d iltia z e m c a n
p o te n tia te c a r d ia c d e p r e s s io n a n d in h ib it c o n d u c tio n in th e A V n o d e c a u s e d b y v o la tile
a n e s th e tic s . V e ra p a m il m a y a ls o m o d e s tly d e c r e a s e a n e s th e tic r e q u ir e m e n ts . D ih y d r o p y r id in e
d e r iv a tiv e s p o te n tia te s y s te m ic v a s o d il a tio n u n d e r a n e s th e s ia .

20. C. C a r d io s e le c tiv ity o f a g e n ts lik e m e t o p r o l o l is d o s e - d e p e n d e n t ^ - r e c e p t o r - s p e c i f i c ) . E v e n


th e ^ - r e c e p t o r - s p e c i f i c a g e n ts c a n h a v e s o m e P2- b lo c k in g a c tio n a t h ig h e r d o s e s . P - B lo c k e rs
w ith in tr in s ic s y m p a th o m im e tic a c tiv ity , lik e a c e b u to lo l, p r o v id e a u n iq u e a d v a n ta g e in p a tie n ts
w ith b r o n c h o s p a s tic a ir w a y d is e a s e .

21. C. P r o l o n g e d Q T in te r v a l (Q T c > 0 .4 4 s e c o n d ) c a n b e c a u s e d b y m y o c a r d ia l is c h e m ia , d r u g
to x ic ity ( a n tia r r h y th m ic a g e n ts , a n tid e p r e s s a n ts , o r p h e n o th ia z in e s ) , e le c tr o ly te a b n o r m a litie s
( h y p o k a le m ia o r h y p o m a g n e s e m ia ) , a u to n o m ic d y s fu n c tio n , m itr a l- v a lv e p r o la p s e , o r, le s s
c o m m o n ly , a c o n g e n ita l a b n o r m a lity . P r o l o n g e d Q T in te r v a l p r e d is p o s e s p a tie n ts to v e n tr ic u la r
a r r h y th m ia s , p a r tic u la r ly p o ly m o r p h ic - v e n tr ic u la r ta c h y c a r d ia , a ls o k n o w n a s to r s a d e d e
p o in te s o r tw is tin g p o in ts , w h ic h c a n le a d to v e n tr ic u la r f ib r illa tio n . P r o l o n g e d Q T in te r v a l is
d u e to n o n u n if o r m p r o l o n g a t i o n o f v e n tr ic u la r r e p o la r iz a ti o n . T h is p r e d is p o s e s p a tie n ts to
r e e n tr y p h e n o m e n a a n d r e s u lts in v e n tr ic u la r ta c h y c a r d ia o r f ib r illa tio n . E le c tiv e s u r g e r y
s h o u ld b e p o s tp o n e d u n til d r u g to x ic ity a n d e le c tr o ly te im b a la n c e s a r e e x c lu d e d . P o ly m o r p h ic
ta c h y a r r h y th m ia s w ith a lo n g Q T in te r v a l a r e u s u a lly tr e a te d w ith in tr a v e n o u s m a g n e s iu m o r b y
p a c in g . T h is is b e c a u s e th e y d o n o t r e s p o n d to c o n v e n tio n a l a n tia r r h y th m ic s . P a tie n ts w ith
c o n g e n ita l p r o l o n g a t i o n g e n e r a lly r e s p o n d to P - a d r e n e r g ic b lo c k in g a g e n ts . L e ft- s te lla te -
g a n g lio n b lo c k a d e h a s a ls o b e e n tr ie d a n d h a s s o m e s u c c e s s in th e s e p a tie n ts s u g g e s tin g th a t th is
m a y b e d u e to a n a u to n o m ic im b a la n c e .

22. D. S e v e re m u ltiv e s s e l d is e a s e c a n b e d e te c te d u s in g e x e r c is e E K G if th e p a tie n t ( d e v e lo p s )

• C a n n o t a tta in a m a x im u m H R > 7 0 % o f p r e d ic te d
• D y s r h y th m ia s a t a lo w e r H R
• S u s ta in e d f a ll in s y s to lic b l o o d p r e s s u r e d u r in g e x e r c is e (> 1 0 m m H g )
• S T d e p r e s s io n > 2 m m , e ith e r h o r iz o n ta l o r d o w n s lo p in g
• S T d e p r e s s io n a t a v e r y l o w w o r k lo a d
• S T d e p r e s s io n s u s ta in e d e v e n a fte r th e e x e r c is e is > 5 m in

23. D. S u r g ic a l e le c tr o c a u te r y in te r f e r e n c e w ith A IC D s a n d p a c e m a k e r d e v ic e s a r e w e ll k n o w n .
T h e o ld a d a g e o f “ p u t a m a g n e t o n i t ” is b a s e d o n th e f a c t th a t a n tita c h y c a r d ia f u n c tio n in s o m e
( o ld e r ) p a c e m a k e r s w a s tu r n e d o f f b y th e a p p lic a tio n o f a m a g n e t. H o w e v e r, th is is n o t tr u e f o r
m o s t o f th e n e w e r A IC D s. Id e a lly , th e m a n u f a c t u r e r ’s r e p r e s e n ta tiv e o r c a r d i o l o g y s h o u ld b e
c o n ta c te d to f in d o u t if th e d e v ic e c o u ld b e r e p r o g r a m m e d to h a v e th e a n tita c h y c a r d ia f u n c tio n
o f f p r i o r to th e s u r g e r y . T h is is in a d d itio n to c o n f ir m i n g th a t th e p a c e m a k e r w a s i n te r r o g a te d
f o r f u n c tio n a lity w ith in th e la s t y e a r a n d A IC D w a s in te r r o g a te d in th e la s t 6 m o n th s .
E l e c tr o s u r g ic a l in te r f e r e n c e c a n b e c a u s e d b y th e d e v ic e in te r p r e tin g th e c u r r e n t a s v e n tr ic u la r
f i b r illa tio n a n d f ir in g , in te r f e r in g w ith its p a c e m a k e r c a p a b ility , r e s e ttin g o f th e d e v ic e to
b a c k u p m o d e . S o m e A IC D s a r e p r o g r a m m e d w ith a r a te - r e s p o n s iv e fu n c tio n , a n d th is m a y b e
a c tiv a te d b y a c a u te r y d e v ic e .
If th e r e is n o tim e to r e p r o g r a m th e d e v ic e p r i o r to s u r g e r y , u s e o f a b ip o la r c a u te ry , p la c e m e n t
o f e le c tr ic a l r e tu r n p a d fa r a w a y f r o m th e d e v ic e , u s in g e le c tr o c a u te r y in s m a ll b u r s ts a r e s o m e
m e th o d s to d e c r e a s e s u c h a n in te r f e r e n c e . In a d d itio n , a ll s u c h p a tie n ts s h o u ld h a v e
tr a n s c u ta n e o u s p a d s o n a n d a d e f ib r il la to r /p a c e r s h o u ld b e a v a ila b le in th e r o o m . E v e r y e f f o r t
s h o u ld b e m a d e to r e p r o g r a m th e d e v ic e to its o r i g i n a l s e ttin g p r i o r to d is c h a r g e o f th e p a tie n t
f r o m th e p o s ta n e s th e s ia c a r e u n it.

24. A. T h e s e n s itiv ity o f th e i n tr a o p e r a tiv e /p e r io p e r a ti v e E C G in d e te c tin g is c h e m ia is d ir e c tly


p r o p o r t i o n a l to th e n u m b e r o f le a d s m o n ito r e d . V 5 is th e m o s t u s e f u l le a d . In o r d e r o f
d e c r e a s in g s e n s itiv ity , V 5 is f o ll o w e d b y V 4 , II, V 2 , a n d V 3 le a d s . U s u a lly tw o le a d s a r e
m o n ito r e d s im u lta n e o u s ly in p e r io p e r a tiv e p e r io d . L e a d s II a n d V 5 a r e th e tw o m o s t c o m m o n ly
u s e d le a d s . L e a d II h e lp s to d e te c t a r r h y th m ia s a n d i n f e r i o r - w a l l is c h e m ia , w h ile le a d V 5 is
u s e f u l f o r d e te c tin g la te r a l- w a ll is c h e m ia . M o d if ie d V 5 le a d is v e r y u s e f u l w h e n o n ly o n e
c h a n n e l c a n b e m o n ito r e d ( th r e e le a d s a p p lie d w ith le f t- a r m le a d a t V 5 p o s itio n a n d m o n ito r in g
le a d I). P o s te r io r w a ll c a n b e m o n ito r e d u s in g a n e s o p h a g e a l le a d .

25. A. It is a c o m m o n p r a c tic e to c o o l th e b o d y to a c o r e b o d y te m p e r a tu r e o f 2 0 to 3 2 °C
f o ll o w i n g C P B s ta rt. H o w e v e r, it is n o t a lw a y s r e q u ir e d . T h is is b a s e d o n th e p r in c ip le th a t
m e ta b o lic o x y g e n r e q u ir e m e n ts c a n b e h a lv e d w ith e a c h r e d u c tio n o f 1 0 °C in b o d y te m p e r a tu r e .
T h is te m p e r a tu r e is b r o u g h t b a c k to a c c e p ta b le le v e ls ( w h e r e a r r h y th m ia s a r e lo w e r ) a t th e e n d
o f C P B — a p h a s e c a lle d r e w a r m in g . S o m e p r o c e d u r e s n e e d a c o m p le te c ir c u l a t o r y s ta n d s till—
c a lle d c i r c u l a t o r y a r r e s t — a n d d e e p h y p o th e r m ia is e m p lo y e d f o r s u c h p r o c e d u r e s — c o o lin g to
15 to 1 8 °C a llo w s a n a r r e s t tim e o f a r o u n d 6 0 m in u te s .

26. B. T h e a d v e r s e e ffe c ts o f h y p o th e r m ia a r e a r r h y th m ia s , p la te le t d y s fu n c tio n , c o a g u lo p a th y ,


d e c r e a s e d s y s to lic f u n c tio n o f m y o c a r d iu m , a n d r e d u c tio n in s e r u m - io n iz e d c a lc iu m d u e to
c itr a te to x ic ity .

27. A. C o r o n a r y p e r f u s io n p r e s s u r e is d e te r m in e d b y th e d if f e r e n c e b e tw e e n th e a r te r ia l d ia s to lic
p r e s s u r e a n d th e le f t- v e n tr ic u la r e n d d ia s to lic p r e s s u r e . T h e le f t v e n tr ic le is p e r f u s e d d u r in g
d ia s to le , w h ile th e r i g h t v e n tr ic le is p e r f u s e d b o th d u r in g d ia s to le a n d s y s to le . A n in c r e a s e in
h e a r t r a te r e d u c e s c o r o n a r y p e r f u s io n b e c a u s e o f a s h o r te r d ia s to le . N o r m a l c o r o n a r y b l o o d
f lo w a t r e s t is a b o u t 2 5 0 m L /m in .

28. C. T r a n s g a s tr ic m id - p a p ill a r y ( m i d s h o r t a x is ) v ie w p r o v id e s a s n a p s h o t o f a ll th e d if f e r e n t
b l o o d v e s s e ls s u p p ly in g th e h e a r t ( F ig 1 1 -2 ).
F igu re 11-2.Reused with permission from Shanewise JS, Shin JJ, Vezina DP, et al. Comprehensive and abbreviated intraoperative TEE
examination. In: Savage RM, Aronson S, Shernan SK, eds. Comprehensive Textbook of Perioperative Transesophageal
Echocardiography. 2nd ed. Philadelphia, PA: Lippincott Williams & Wilkins; 2011: 86.

29. D. P u r e h ig h - d o s e o p i o i d a n e s th e s ia (e .g ., fe n ta n y l 5 0 - 1 0 0 ^ g /k g o r s u f e n ta n il 1 5 - 2 5 ^ g /k g )
h a s f a lle n o u t o f v o g u e in c a r d ia c a n e s th e s ia p r a c tic e . It w a s u s e f u l a t a tim e in a n e s th e s ia w h e n
th e o n ly in h a le d a g e n ts a v a ila b le p r o d u c e d u n a c c e p ta b le m y o c a r d ia l d e p r e s s io n . T h e m a in
d is a d v a n ta g e s o f h ig h - d o s e o p i o i d te c h n iq u e in c lu d e p r o l o n g e d p o s to p e r a tiv e r e s p i r a t o r y
d e p r e s s io n ( e a r ly e x tu b a tio n is b e c o m in g a v e r y c o m m o n tr e n d in c o r o n a r y a r te r y b y p a s s
g r a f tin g s u r g e r ie s ) , h ig h in c id e n c e o f p a tie n t a w a r e n e s s /r e c a ll, e x a g g e r a te d h y p e r te n s iv e
r e s p o n s e to s tim u la tio n lik e s te r n o to m y in a p a tie n t w ith g o o d le f t- v e n tr ic u la r fu n c tio n ,
b r a d y c a r d ia , c h e s t-w a ll r ig id ity , p o s to p e r a tiv e ile u s , a n d i m p a ir e d im m u n ity .

30. A. A p r o g r e s s i v e d e c lin e in c a r d ia c o u tp u t is s o m e tim e s s e e n a fte r th e c h e s t is o p e n e d . T h is is


a ttrib u te d to th e lo s s o f n e g a tiv e in tr a th o r a c ic p r e s s u r e a n d d e c r e a s e d p r e lo a d . H e n c e a IV flu id
b o lu s m a y h e lp . F a c to r s p o te n tia tin g s u c h a r e s p o n s e in c lu d e d e e p a n e s th e s ia a n d p r e o p e r a t iv e
a n g io te n s in - r e c e p to r - b l o c k a d e u s e . A n o th e r c o m m o n r e s p o n s e s e e n d u r in g s te r n a l r e tr a c tio n
a n d p e r ic a r d ie c to m y is b r a d y c a r d i a a n d h y p o te n s io n d u e to e x a g g e r a te d v a g a l r e s p o n s e . T h is is
p o te n tia te d b y h y p o x ia , P - b lo c k e r s , a n d c a lc iu m c h a n n e l b lo c k e r s .

31. C. A p r o tin in , a n in h ib ito r o f s e r in e p r o te a s e s , s u c h as p la s m in , k a llik r e in , a n d tr y p s in , a ls o


h e lp s to p r e s e r v e p la te le t a g g r e g a t i o n a n d a d h e s iv e n e s s . It h a s b e e n s h o w n to d e c r e a s e b l o o d
lo s s a n d tr a n s f u s io n r e q u ir e m e n ts a n d s h o u ld b e c o n s id e r e d in r e d o s u r g e r ie s , J e h o v a h ’s
w itn e s s e s , r e c e n t a d m in is tr a tio n o f g ly c o p r o te in Ilb /IIIa in h ib ito r s ( a b c ix im a b [R e o P r o ] ,
e p tifib a tid e [I n te g r ilin ], o r tir o f ib a n [ A g g ra s ta t], p a tie n ts w ith c o a g u lo p a th ie s , a n d p a tie n ts w ith
lo n g p u m p r u n s . H o w e v e r, r e p e a t e x p o s u r e to a p r o tin in h a s b e e n s h o w n to c a u s e a lle r g ic
r e a c tio n s , w h ic h m a y in c lu d e a n a p h y la x is . P a tie n ts o n a c o m b in a tio n o f a s p ir in a n d A D P -
r e c e p to r a n ta g o n is t a r e a t h ig h r i s k o f b le e d in g a n d m a y b e n e f it f r o m a p r o tin in .

32. A. T h e e v e n ts o c c u r r i n g in s e q u e n c e a fte r h e p a r in iz a tio n a r e a o r tic c a n n u la tio n f o ll o w e d b y


v e n o u s c a n n u la tio n . V e n o u s c a n n u la tio n u s u a lly c a u s e s h e m o d y n a m ic c h a n g e s , a n d w e h a v e a n
a c c e s s to p r o v id e r a p id in f u s io n th r o u g h th e a o r tic c a n n u la if n e c e s s a r y . V e n o u s c a n n u la tio n
a ls o f r e q u e n tly p r e c ip ita te s a r r h y th m ia s . P r e m a tu r e a tr ia l c o n tr a c tio n s a n d tr a n s ie n t b u r s ts o f a
s u p r a v e n tr ic u la r ta c h y c a r d ia a r e c o m m o n . S u s ta in e d a r r h y th m ia s m u s t b e tre a te d
p h a r m a c o lo g i c a lly , e le c tr ic a lly , o r b y im m e d ia te a n tic o a g u la tio n a n d in itia tio n o f b y p a s s
d e p e n d in g o n th e a m o u n t o f h e m o d y n a m ic c o m p r o m is e . S o m e tim e s , s to p p in g th e s u r g ic a l
s tim u lu s is a ll th a t is n e e d e d . S u p e r io r v e n a c a v a s y n d r o m e c a n b e c a u s e d b y a m a lp o s itio n e d
v e n o u s c a n n u la s c a n b e i n te r f e r in g w ith v e n o u s d r a in a g e f r o m th e h e a d a n d n e c k .

33. B. A fte r in itia tio n o f C P B , p u m p f lo w is g r a d u a lly in c r e a s e d to 2 to 2.5 L / m in /m 2 a n d M A P s


a r e m o n ito r e d . It is c o m m o n to s e e a n in itia l f a ll in BP. In itia l m e a n s y s te m ic a r te r ia l ( r a d ia l)
p r e s s u r e s o f 3 0 to 4 0 m m H g a r e n o t u n u s u a l. A b r u p t h e m o d ilu tio n , w h ic h r e d u c e s b lo o d
v is c o s ity a n d e f f e c tiv e ly lo w e r s s y s te m ic v a s c u la r r e s is ta n c e (S V R ), m a y b e r e s p o n s ib le f o r th is
d r o p . T h e e f f e c t is p a r tia lly c o m p e n s a te d b y s u b s e q u e n t h y p o th e r m ia , w h ic h te n d s to r a is e b lo o d
v is c o s ity a g a in .
A d is a s tr o u s s c e n a r io is a p e r s is te n t a n d e x c e s s iv e d e c r e a s e in M A P (< 3 0 m m H g );
tr a n s e s o p h a g e a l e c h o c a r d i o g r a p h e v a lu a tio n is v e r y u s e f u l in s u c h a s itu a tio n to l o o k f o r
u n r e c o g n iz e d a o r tic d is s e c tio n . If d is s e c tio n is p re s e n t, C P B m u s t b e t e m p o r a r i l y s to p p e d u n til
th e a o r ta is r e c a n n u la te d d is ta lly to p r e v e n t f u r th e r e x te n s io n o f a d is s e c tio n fla p w ith g r a v e
c o n s e q u e n c e s . P o o r v e n o u s r e tu r n , p u m p m a lf u n c tio n , o r p r e s s u r e - tr a n s d u c e r e r r o r m a y a ll
c a u s e h y p o te n s io n . A o r tic c a n n u la m is d ir e c te d to w a r d th e in n o m in a te a r te r y m a y b e a c a u s e f o r
f a ls e h y p e r te n s io n w h e n r i g h t r a d ia l a r te r y is u s e d f o r m o n ito r in g .
T h e r e la tio n s h ip b e tw e e n p u m p flo w , S V R , a n d m e a n s y s te m ic a r te r ia l b l o o d p r e s s u r e m a y b e
c o n c e p tu a liz e d a s f o llo w s :

M A P = P u m p f lo w x S V R

W ith a c o n s ta n t S V R , M A P is p r o p o r t i o n a l to p u m p flo w . S im ila r ly , a t a n y g iv e n p u m p flo w ,


M A P is p r o p o r t i o n a l to S V R . P u m p f lo w s o f 2 to 2.5 L / m in /m 2 ( 5 0 - 6 0 m L /k g /m in ) a n d m e a n
a r te r ia l p r e s s u r e s b e tw e e n 50 a n d 8 0 m m H g a r e c o m m o n ly u s e d . F lo w r e q u ir e m e n ts a r e
g e n e r a lly lo w e r d u r in g d e e p h y p o th e r m ia ( 2 0 - 2 5 ° C ) , a s m e a n b l o o d p r e s s u r e s a s l o w a s 3 0 m m
H g m a y s till p r o v id e a d e q u a te c e r e b r a l b l o o d flo w . S V R c a n b e in c r e a s e d w ith a a g o n is ts lik e
p h e n y le p h r in e .
H ig h s y s te m ic a r te r ia l p r e s s u r e s (> 1 5 0 m m H g ) a r e a ls o d e le te r io u s b e c a u s e th e y p r o m o t e
a o r tic d is s e c tio n o r a c e r e b r o v a s c u la r a c c id e n t in a d d itio n to in c r e a s in g th e s u r g ic a l b le e d in g .
H y p e r te n s io n is s a id to e x is t o n p u m p w h e n M A P s e x c e e d 1 0 0 m m H g , a n d th is is tr e a te d b y
d e c r e a s in g p u m p f lo w o r d e e p e n in g a n e s th e s ia u s in g is o f lu r a n e a t th e o x y g e n a to r in f l o w g a s .
S o m e tim e s , a h y p e r te n s io n is r e f r a c t o r y to th e s e m a n e u v e r s o r, if p u m p f lo w is a lr e a d y lo w ,
m a y n e c e s s ita te a v a s o d il a to r lik e n itr o p r u s s id e .

34. B. M o n ito r in g d u r in g C P B is u s u a lly d o n e b y th e p e r f u s io n is ts . T h e y m o n ito r th e p u m p f lo w


ra te , v e n o u s r e s e r v o i r le v e l, a r te r ia l in f l o w lin e p r e s s u r e , b l o o d ( p e r f u s a te a n d v e n o u s ) a n d
m y o c a r d ia l te m p e r a tu r e s , a n d in - lin e ( a r te r ia l a n d v e n o u s ) o x y g e n s a tu r a tio n s . I n - lin e p H , C O 2
te n s io n , a n d o x y g e n - te n s io n s e n s o r s a r e a ls o a v a ila b le in n e w e r b y p a s s m a c h in e s . B u t m o s t
m a c h in e s d o n o t p r o v id e a g lu c o s e m o n ito r , a n d h y p o g ly c e m ia is s till a th re a t. B lo o d g a s
te n s io n s a n d p H a r e c o n f ir m e d b y d ir e c t m e a s u r e m e n ts p e r io d ic a l ly — 3 0 m in u te - in te r v a ls .
In a d e q u a te tis s u e p e r f u s io n c a u s e d b y in a d e q u a te f lo w r a te s is e v id e n c e d b y l o w v e n o u s o x y g e n
s a tu r a tio n s (< 7 0 % ), p r o g r e s s i v e m e ta b o lic a c id o s is , o r l o w u r i n a r y o u tp u t, p r o v id e d th e r e is n o
h y p o x e m ia .
D u r in g b y p a s s , a r te r ia l in f l o w lin e p r e s s u r e is a lm o s t a lw a y s h ig h e r th a n th e s y s te m ic a r te r ia l
p r e s s u r e r e c o r d e d f r o m a r a d ia l a r te r y o r e v e n a n a o r tic c a th e te r, c a u s e d b y th e p r e s s u r e d r o p
a c r o s s th e a r te r ia l filte r , th e a r te r ia l tu b in g , a n d th e n a r r o w o p e n in g o f th e a o r tic c a n n u la .

35. A. B e f o r e d is c o n tin u in g v e n tila tio n a fte r in itia tio n o f C P B , it is a g o o d p r a c tic e to c o n f ir m


w h e th e r fu ll f lo w h a s b e e n a tta in e d w ith th e p e r f u s io n is t. D is c o n tin u in g v e n tila tio n p r e m a tu r e ly
c a u s e s a n y r e m a in in g p u lm o n a r y b l o o d f lo w to a c t a s a r ig h t - to - le f t s h u n t, w h ic h c a n p r o m o t e
h y p o x e m ia . T h e e x te n t o f h y p o x e m ia d e p e n d s o n th e r e la tiv e r a tio o f r e m a in in g p u lm o n a r y
b l o o d f lo w to p u m p flo w . O n c e th e h e a r t s to p s e je c tin g b lo o d , v e n tila tio n c a n b e d is c o n tin u e d .
F o llo w in g in s titu tio n o f fu ll C P B , v e n tr ic u la r e je c tio n m a y c o n tin u e f o r a b r i e f p e r i o d o f tim e .

36. D. E p ia o r tic e c h o c a r d io g r a p h y is th e m o s t s e n s itiv e a n d s p e c if ic te c h n iq u e to d e te c t a ir


b u b b le s a t th e te r m in a tio n o f C P B . D e - a ir in g is f a c ilita te d b y h e a d - d o w n p o s itio n , a n d v e n tin g
b e f o r e a n d d u r in g in itia l c a r d ia c e je c tio n , in a d d itio n to f illin g u p th e h e a r t w ith v e n t in p la c e .
T E E is v e r y u s e f u l in d e te c tin g p o c k e ts o f a ir, e s p e c ia lly w ith in th e le f t v e n tr ic le . B u t th e r i s k o f
a th e r o m a to u s e m b o li s till p e r s is ts a n d is w o r s e in c a s e s w h e r e a o r ta w a s m a n ip u la te d
e x te n s iv e ly , c r o s s - c la m p e d n u m e r o u s tim e s a n d in p e r c u ta n e o u s tr a n s c a th e te r a o r tic v a lv e
r e p la c e m e n ts . N e w e r d e v ic e s w ith b a s k e ts to c a tc h s u c h e m b o li h a v e p r o v e n to b e v e r y u s e f u l.

37. B. S w e a tin g d u r in g r e w a r m in g is a h y p o th a la m ic r e s p o n s e to p e r f u s io n w ith b lo o d , w h ic h is


o f te n a t 3 9 °C . It is im p o r ta n t to r e m e m b e r to a d m in is te r a n e s th e tic a g e n ts , a n d s o m e tim e s
a d d itio n a l m u s c le r e la x a n ts , d u r in g th e r e w a r m in g p h a s e . T h e in c id e n c e o f a w a r e n e s s /r e c a ll is
h ig h d u r in g r e w a r m in g b e c a u s e th e in h a la tio n a l a g e n t d e liv e r e d v ia th e o x y g e n a to r is tu r n e d o f f
j u s t p r i o r to te r m in a tio n o f C P B to a v o id r e s id u a l m y o c a r d ia l d e p r e s s io n .

38. A. p H -s ta t m a n a g e m e n t r e f e r s to th e p r a c tic e o f t e m p e r a t u r e - c o r r e c tin g g a s te n s io n s b y


a d d in g C O 2 a n d m a in ta in in g a “ n o r m a l ” C O 2 te n s io n o f 4 0 m m H g a n d a p H o f 7 .4 0 d u r in g
h y p o th e r m ia . a - S ta t m a n a g e m e n t, o n th e o th e r h a n d , r e f e r s to th e u s e o f u n c o r r e c t e d g a s
te n s io n s d u r in g h y p o th e r m ia . T h is d o e s n o t r e q u i r e a d d itio n o f C O 2 a n d h a s b e e n s h o w n to
p r e s e r v e c e r e b r a l a u to r e g u la t io n a n d im p r o v e m y o c a r d ia l p r e s e r v a tio n . A t p h y s io lo g ic p H , th e
h is tid in e r e s id u e s o f in tr a c e l lu la r p r o te in s p la y a m a jo r r o l e in m a in ta in in g e le c tr ic a l n e u tra lity .
p H -s ta t m a n a g e m e n t is c o m m o n ly u s e d in p e d ia tr ic c a r d ia c s u r g e r y , b u t a - s ta t is m o r e
c o m m o n ly u s e d in a d u lt c a r d ia c s u r g e r y .

39. B. R a p id r e w a r m in g c a n r e le a s e g a s b u b b le s th a t w e r e d is s o lv e d r a p id ly b a c k in to th e b l o o d
s tr e a m . It a ls o r e s u lts in l a r g e te m p e r a tu r e g r a d ie n ts b e tw e e n w e ll- p e r f u s e d o r g a n s a n d
p e r ip h e r a l v a s o c o n s tr ic te d tis s u e s . T h e b o d y e q u ilib r a te s th is g r a d ie n t f o ll o w i n g s e p a r a tio n
f r o m C P B , a n d p a tie n t m a y b e c o m e h y p o th e r m ic a g a in . M e th o d s u s e d to s p e e d th e r e w a r m in g
p r o c e s s in c lu d e in f u s io n o f a v a s o d il a to r d r u g ( n it r o p r u s s id e o r n i tr o g ly c e r in ) a n d a llo w in g
s o m e p u ls a tile f lo w ( v e n tr ic u la r e je c tio n ) .

40. A. S e p a r a tio n f r o m C P B c a n b e g u id e d b y a m n e m o n ic :

A = A ir w a y — o x y g e n a tio n a n d v e n tila tio n w ith 1 0 0 % o x y g e n


B = B lo o d g a s — c o r r e c t e le c tr o ly te a b n o r m a litie s / h e m o g lo b i n
C = C o a g u la tio n — r e v e r s e h e p a r in w ith p r o ta m in e
D = D y s r h y th m ia s — s in u s r h y th m is g o o d ; p a c in g n e e d e d s o m e tim e s ( 8 0 - 1 0 0 b p m )
E = E p in e p h r in e — in o t r o p e s / v a s o p r e s s o r s u s e d a s n e e d e d . E p in e p h r in e m a y in c r e a s e
m y o c a r d ia l O 2 n e e d
F = F lu id s — f o r r a p id v o lu m e r e s u s c ita tio n
G = G o o d c o n tr a c tility b y d ir e c t v is u a l iz a tio n /tr a n s e s o p h a g e a l e c h o c a r d io g r a m
H = H y p o th e r m ia is a v o id e d ; > 3 7 ° C is a im e d
I = In v a s iv e m o n ito r s r e c a lib r a te d

41. B. IA B P is s o m e tim e s u s e d to f a c ilita te w e a n in g th e p a tie n t o f f c a r d io p u lm o n a r y b y p a s s . T h is


p r o v id e s a s y s to lic a u g m e n ta tio n o f b l o o d p r e s s u r e in a d d itio n to im p r o v in g m y o c a r d ia l
o x y g e n s u p p ly d u r in g d ia s to le . T im in g a n d lo c a t io n o f a n IA B P a r e c r itic a l f o r o p tim a l
f u n c tio n in g . Id e a l in f l a tio n o f th e b a l l o o n s h o u ld b e ju s t a fte r th e d ic r o tic n o tc h ( c lo s u r e o f
a o r tic v a lv e ) . I n f la tio n w h ile th e a o r tic v a lv e is s till o p e n c a n in c r e a s e a f te r lo a d , w o r s e n a o r tic
r e g u r g i t a t i o n a n d le f t- v e n tr ic u la r (L V ) v o lu m e . I n f la tio n to o la te in th e d ia s to lic p h a s e w ill
r e d u c e d ia s to lic a u g m e n ta tio n a n d m y o c a r d ia l s u p p ly . S im ila r ly , th e d e f la tio n s h o u ld b e tim e d
j u s t p r i o r to L V e je c tio n to p r o d u c e a n o p tim a l r e d u c tio n in a f te r lo a d . T im in g is u s u a lly
s y n c h r o n iz e d w ith E K G /a r te r ia l p u ls e . T h e lo c a t io n o f th e tip o f th e IA B P s h o u ld b e j u s t d is ta l
to th e ta k e o f f o f th e le f t- s u b c la v ia n a rte ry , u s u a lly c o n f ir m e d w ith tr a n s e s o p h a g e a l
e c h o c a r d io g r a p h /f lu o r o s c o p y .

42. D. T h is p a tie n t h a s a l o w C V P, P C W P s u g g e s tiv e o f lo w - f illin g p r e s s u r e s , in d ic a tin g th a t h e is


h y p o v o le m ic . B u t th e r e s t o f th e c lin ic a l p ic tu r e o f l o w S V R a n d h ig h C O is s t r o n g l y s u g g e s tiv e
o f a h y p e r d y n a m ic c i r c u l a t o r y s ta te ( v a s o d ila te d ) . T h e tr e a tm e n t in s u c h a s c e n a r io w ill b e to
in c r e a s e th e h e m a to c r it. If th e p a tie n t h a d a d e c r e a s e d c a r d ia c o u tp u t, th e tr e a tm e n t w o u ld b e to
a d m in is te r v o lu m e /c r y s ta ll o id s . L e ft- h e a rt f a ilu r e (L H F ) w ill h a v e a h ig h P C W P a n d p u lm o n a r y
a r te r y p r e s s u r e . R ig h t- h e a r t f a ilu r e (R H F ) w ill h a v e a h ig h C V P a n d n o r m a l o r lo w P C W P . B o th
L H F a n d R H F w ill h a v e lo w C O .
43. B. P r o ta m in e b in d s a n d e f f e c tiv e ly in a c tiv a te s h e p a r in b e c a u s e th e p o s itiv e c h a r g e o f
p r o ta m in e n e u tr a liz e s th e n e g a tiv e c h a r g e o f h e p a r in . T im in g o f p r o ta m in e a d m in is tr a tio n
s h o u ld b e d e te r m in e d b y c lo s e c o m m u n ic a tio n w ith th e s u r g e o n . T o o e a r ly a d m in is tr a tio n m a y
le a d to c lo t f o r m a ti o n in th e c a r d io p u lm o n a r y b y p a s s c ir c u it. T h e e le c tr i c a lly n e u tr a l h e p a r in ­
p r o ta m in e c o m p le x e s a r e r e m o v e d b y th e r e tic u lo e n d o th e lia l s y s te m . P r o ta m in e d o s in g is b a s e d
o n th e a m o u n t o f h e p a r in in itia lly r e q u ir e d to p r o d u c e th e d e s ir e d a c tiv a te d c lo ttin g tim e ;
p r o ta m in e is th e n g iv e n in a r a tio o f 1 to 1.3 m g p e r 1 0 0 U o f h e p a r in . A n o th e r a p p r o a c h
c a lc u la te s th e p r o ta m in e d o s e b a s e d o n th e h e p a r in d o s e - r e s p o n s e c u r v e a n d th e e s tim a tio n o f
h e p a r in c o n c e n tr a tio n u s in g s p e c ia l m o n ito r s (H e p c o n ) .

44. C. T h e a c tiv a te d c lo ttin g tim e s h o u ld r e tu r n to b a s e lin e f o ll o w i n g r e v e r s a l o f h e p a r in w ith


p r o ta m in e ; s o m e tim e s , a d d itio n a l d o s e s o f p r o ta m in e ( 2 5 - 5 0 m g ) m a y b e n e c e s s a ry .
C o a g u lo p a th y o f te n f o llo w s lo n g b y p a s s p e r io d s (> 2 h o u r s ) a n d is d u e to m u ltif a c to r i a l c a u s e s :
s u r g ic a l b le e d in g s ite s , in a d e q u a te r e v e r s a l o f h e p a r in , r e h e p a r in iz a tio n , th r o m b o c y to p e n ia ,
p la te le t d y s fu n c tio n , h y p o th e r m ia , p r e o p e r a t iv e c o a g u la tio n d e fe c ts , o r n e w ly a c q u ir e d d e fe c ts
m a y b e r e s p o n s ib le . R e h e p a r in iz a tio n ( h e p a r in r e b o u n d ) a fte r a p p a r e n t a d e q u a te r e v e r s a l is d u e
to a r e la tiv e h e p a r i n - p r o t a m i n e c o n c e n tr a tio n m is m a tc h a n d c a n b e c a u s e d b y a r e d is tr ib u tio n
e ith e r o f p r o ta m in e to p e r ip h e r a l c o m p a r tm e n ts o r o f p e r ip h e r a lly b o u n d h e p a r in to th e c e n tr a l
c o m p a r tm e n t. H y p o th e r m ia (< 3 5 ° C ) o f te n e x a c e r b a te s s u c h b le e d in g p r o b le m s .

45. A. D D A V P, 0 .3 |ig /k g ( in tr a v e n o u s ly o v e r 2 0 m in u te s ), c a n in c r e a s e th e a c tiv ity o f f a c to r s V III


a n d X II a n d th e v o n W ille b r a n d fa c to r. D D A V P fa c ilita te s th e ir r e le a s e f r o m th e v a s c u la r
e n d o th e liu m . H e n c e , a s e c o n d d o s e is u s u a lly n o t e ffe c tiv e . D D A V P is v e r y u s e f u l in r e v e r s i n g
q u a lita tiv e p la te le t d e fe c ts , b u t is n o t r e c o m m e n d e d f o r r o u tin e u s e .

46. A. I m m e d ia te ly f o ll o w i n g c a r d ia c s u r g e r y , th e e m p h a s is is o n m a in ta in in g h e m o d y n a m ic
s ta b ility a n d m o n ito r in g f o r e x c e s s iv e p e r io p e r a tiv e b le e d in g . S e d a tio n u s in g
p r o p o f o l/f e n ta n y l /titr a te d d o s e s o f m o r p h in e /d e x m e d e to m id in e is u s e d in d if f e r e n t in s titu tio n s
to e n s u r e a c a lm , c o m f o r ta b le p a tie n t. C h e s t-tu b e d r a in a g e m o r e th a n 10 m L /k g /h o u r in th e f i r s t
2 h o u r s o f te n r a is e s a r e d f la g a n d p r o m p ts c o a g u la tio n s tu d ie s a n d s o m e tim e s r e q u i r e c h e s t
r e e x p lo r a tio n . A v e r y d e a d ly s ite f o r p o s to p e r a tiv e m o n ito r in g is in to th e p e r ic a r d iu m c a u s in g
c a r d ia c ta m p o n a d e . T h is is u s u a lly s ig n a le d b y e q u a liz a tio n o f d ia s to lic p r e s s u r e s a n d
h e m o d y n a m ic c o m p r o m is e a n d n e e d s im m e d ia te s u r g ic a l in te r v e n tio n . A fte r th e f i r s t 2 h o u r s ,
a n y d r a in a g e f r o m c h e s t tu b e > 1 0 0 m L /h o u r s h o u ld b e c lo s e ly o b s e r v e d .

47. A. N O is a p o te n t v a s o d ila to r , w h ic h c a n b e g iv e n a s in h a le d n itr ic o x id e , w h ic h c ir c u m v e n ts


th e u n w a n te d s id e e f f e c t o f d e c r e a s e d S V R a n d s y s te m ic b l o o d p r e s s u r e , a t th e s a m e tim e
r e ta in in g th e th e r a p e u tic p o te n tia l o f d e c r e a s in g p u lm o n a r y h y p e r te n s io n . I n o d ila to r s lik e
d o p a m in e a n d m i l r in o n e m a y h e lp in s itu a tio n s w ith r ig h t - v e n t r ic u la r (R V ) f a ilu r e s e c o n d a r y to
p u lm o n a r y h y p e r te n s io n . V a s o d ila to r s lik e n itr o g l y c e r i n w ill a ls o d e c r e a s e th e P V R , b u t th e y
p r o d u c e d r o p in s y s te m ic b l o o d p r e s s u r e . In h a le d p r o s ta g la n d i n E 1 ( P G E 1) is a ls o v e r y s p e c if ic
in d e c r e a s in g P V R w ith o u t a f f e c tin g S V R . A d v a n c e d R V f a ilu r e m a y n e c e s s ita te a R V - a s s is t
d e v ic e o r a n in tr a - a o r ti c b a l l o o n p u m p , w h ic h w o r k s b y in c r e a s in g th e p e r f u s io n to th e r i g h t
s id e o f th e h e a r t. In h a le d N O a t 10 to 6 0 p p m a n d P G E 1 a t 0 .0 1 to 0.2 |i g /k g /m in a r e v e r y
e f f e c tiv e p u lm o n a r y v a s o d il a to r s .

48. D. E n d - s ta g e h e a r t d is e a s e p a tie n ts h a v e a n o p tio n to g e t a d e s tin a tio n v e n tr ic u la r - a s s is t d e v ic e


th e r a p y o r g e t a c a r d ia c tr a n s p la n ta tio n . T h e ir p o s itio n in th e tr a n s p la n t lis t is h ig h e r if th e y a r e
u n lik e ly to s u r v iv e th e n e x t 6 to 12 m o n th s . S u r v iv a l r a te s a fte r c a r d ia c tr a n s p la n ta tio n a r e
u s u a lly h ig h a t a 5 -y e a r s u r v iv a l r a te o f 6 0 % to 9 0 % . H ig h p u lm o n a r y v a s c u la r r e s is ta n c e > 6 to
8 W o o d u n its (1 W o o d u n it = 8 0 d y n [ m id d o t] s /c m 5) is a p r e d ic to r o f r ig h t - v e n t r ic u la r f a ilu r e ,
w h ic h h a s a h ig h e a r ly p o s to p e r a tiv e m o r ta lity . H e n c e , i r r e v e r s i b l e p u lm o n a r y v a s c u la r d is e a s e
is c o n s id e r e d a c o n tr a in d ic a tio n to o r th o t o p ic c a r d ia c tr a n s p la n ta tio n . T h e y s till q u a lif y f o r a
c o m b in e d h e a r t - l u n g tr a n s p la n ta tio n , w h ic h is a llo c a te d f r o m a s e p a r a te lis t. S iz e , A B O b l o o d -
g r o u p ty p in g , a n d c y to m e g a lo v ir u s s e r o l o g y a r e u s e d f o r d o n o r - r e c i p i e n t c o m p a tib ility te s tin g .
H o w e v e r, tis s u e c r o s s m a tc h in g is g e n e r a lly n o t p e r f o r m e d . D o n o r o r g a n s f r o m p a tie n ts w ith
h e p a titis B o r C o r H IV in f e c tio n a r e e x c lu d e d .

49. B. T h e C V P w a v e f o r m is c h a r a c te r is tic in c a r d ia c ta m p o n a d e . C a r d ia c ta m p o n a d e is
c h a r a c te r iz e d b y e q u a liz a tio n o f d ia s to lic p r e s s u r e s th r o u g h o u t th e h e a r t: L A P = R A P = L V E D P
= R V E D P. T h is p r o d u c e s a r e d u c e d s tr o k e v o lu m e a n d h ig h c e n tr a l v e n o u s p r e s s u r e . T h e
e x te r n a l c o m p r e s s io n o n th e c o lla p s ib le c h a m b e r s p r e v e n ts e m p ty in g , a n d th e s e p a tie n ts
c o m p e n s a te b y h a v in g ta c h y c a r d ia a n d a n in c r e a s e in c o n tra c tility . H o w e v e r, in th e p r e s e n c e o f
im p a ir e d e m p ty in g , th e c o n tr ib u tio n f r o m s tr o k e v o lu m e is v e r y lim ite d . T h is is p a r tic u la r ly
im p o r ta n t to th e a n e s th e s io l o g is t w h ile in d u c in g g e n e r a l a n e s th e s ia in s u c h p a tie n ts . T h e y d o n o t
to le r a te th e s w itc h f r o m n e g a ti v e - p r e s s u r e to p o s itiv e - p r e s s u r e b r e a th in g . C h a r a c te r is tic C V P
w a v e f o r m in c a r d ia c ta m p o n a d e is d e s c r ib e d a s im p a ir m e n t o f b o th d ia s to lic f il lin g a n d a tr ia l
e m p ty in g a b o lis h e s th e y d e s c e n t; th e x d e s c e n t ( s y s to lic - a tr ia l f il lin g ) is n o r m a l o r e v e n
a c c e n tu a te d . A r te r ia l v a s o c o n s tr ic tio n ( in c r e a s e d s y s te m ic v a s c u la r r e s is ta n c e ) s u p p o r ts
s y s te m ic b l o o d p r e s s u r e , w h e r e a s v e n o c o n s tr ic tio n a u g m e n ts th e v e n o u s r e tu r n to th e h e a r t.

50. C. C o n s tr ic tiv e p e r ic a r d itis is c h a r a c te r iz e d b y a s tif f p e r ic a r d iu m th a t lim its d ia s to lic f illin g


o f th e h e a r t. P a th o p h y s io lo g y c o n s is ts o f a th ic k e n e d , f ib r o tic , a n d o f te n c a lc if ie d p e r ic a r d iu m
s e c o n d a r y to a c u te o r r e c u r r e n t p e r ic a r d itis . T h e a d h e r e n t p a r ie ta l p e r ic a r d iu m a llo w s th e h e a r t
to f ill o n ly to a f ix e d v o lu m e . F illin g d u r in g e a r ly d ia s to le is ty p ic a lly a c c e n tu a te d a n d
m a n if e s te d b y a p r o m i n e n t y d e s c e n t o n th e C V P w a v e f o r m . T h is is in c o n tr a s t to c a r d ia c
ta m p o n a d e , w h ic h c a u s e s a f illin g d e fe c t. T h is p a th o p h y s i o lo g y is r e s p o n s ib le f o r K u s s m a u l
s ig n — p a r a d o x ic a l r i s e in v e n o u s p r e s s u r e d u r in g in s p ir a tio n . C h e s t X - r a y m a y s h o w s o m e
p e r ic a r d ia l c a lc if ic a tio n s , a n d E K G m a y s h o w a tr ia l f ib r illa tio n , c o n d u c tio n b lo c k s , l o w Q R S
v o lta g e , a n d d if f u s e T -w a v e a b n o r m a litie s . C lin ic a l s ig n s in c lu d e r a is e d j u g u l a r v e n o u s
p r e s s u r e , h e p a to m e g a ly , a s c ite s , a n d a b n o r m a l liv e r fu n c tio n .

51. D. T h e g o a l d u r in g m a n a g e m e n t o f a n e s th e s ia f o r p a tie n ts w ith h y p e r tr o p h ic c a r d io m y o p a th y


is to d e c r e a s e th e p r e s s u r e g r a d ie n t a c r o s s th e le f t- v e n tr ic u la r o u tf lo w o b s tr u c tio n . D e c r e a s e s in
m y o c a r d ia l c o n tr a c tility a n d in c r e a s e s in p r e lo a d ( v e n tr ic u la r v o lu m e ) a n d a f te r lo a d w ill
d e c r e a s e th e m a g n itu d e o f le f t- v e n tr ic u la r o u tf lo w o b s tr u c tio n . I n tr a o p e r a tiv e h y p o te n s io n is
g e n e r a lly tr e a te d w ith in tr a v e n o u s f lu id s o r a n a a g o n is t s u c h a s p h e n y le p h r in e . D r u g s w ith p-
a g o n is t a c tiv ity a r e n o t lik e ly to b e u s e d to tr e a t h y p o te n s io n b e c a u s e a n y in c r e a s e in c a r d ia c
contractility or heart rate could increase left-ventricular outflow obstruction. When
hypertension occurs, an increased delivered concentration of isoflurane or sevoflurane can be
used. Vasodilators, such as nitroprusside or nitroglycerin, should be used with caution because
decreases in systemic vascular resistance can increase left-ventricular outflow obstruction.

52. B. PCWP is an indirect measure of LVEDP, with many false positives and negatives:
PCWP > LVEDP
• PEEP/positive-pressure ventilation
• Increased intrathoracic pressure
• Left-atrial pathology— myxoma
• Mitral-valve pathology— stenosis/regurgitation
• Pulmonary hypertension
• Chronic obstructive pulmonary disease
LVEDP > PCWP
• LVEDP >25 mm Hg
• Premature mitral-valve closure (usually an aortic regurgitation jet causing this)
• Left-ventricular diastolic dysfunction (left-ventricular hypertrophy/ischemia)

53. B. Mixed venous oxygen tension refers to the oxygen tension in a venous sample with blood
mixed from both inferior vena cava and superior vena cava. Ideally, this sample is drawn from
the tip of a pulmonary artery catheter. It is a good measure of tissue oxygen supply relative to
its demand. A reduction in delivery (decreased cardiac output) or an increase in consumption
(increased BMR) can both cause a reduction in Pvo2. Normal Pvo2 is about 40 mm Hg, with a
saturation of 75%.

54. C. Site of previous MI, history of coronary artery bypass grafting, site of procedure for
procedures <3 hour, and type of anesthesia used (general anesthesia vs. regional) have no
influence on perioperative myocardial reinfarction.
Only three pharmacologic measures have been proven to produce a decrease in
cardiovascular m orbidity and mortality: P-blockers, clonidine, and statins. P-Blockers started 7
to 30 days prior to surgery and continued for 30 days postoperatively reduce the risk of cardiac
m orbidity (M I or cardiac death) by 90%. If started just prior to surgery and continued for 7
days, it w ill still confer a reduction in mortality risk by 50%. Perioperative clonidine
administration reduces the 30-day and 2-year mortality risks. Statin therapy with fluvastatin for
30 days before and after surgery, in addition to P-blockade, reduces risk of M I and death by an
additional 50%.
Intraoperatively, strict hemodynamic control using an intra-arterial catheter and prompt
pharmacologic intervention or fluid infusion to treat physiologic hemodynamic alterations
from the normal range may decrease the risk of perioperative cardiac m orbidity in high-risk
patients.

55. C. Perioperative risk-reduction therapy with medications is superior to risk stratification with
invasive testing, angioplasty, and CABG. P-Blockers, clonidine, statin, and aspirin have all been
used for this. A single 1-minute episode of myocardial ischemia detected by 1-mm ST-segment
elevation or depression increases the risk of cardiac events 10-fold and the risk for death 2­
fold. Tachycardia for 5 minutes above 120 bpm in the postoperative period can increase the risk
of mortality 10 times. The incidence of perioperative myocardial reinfarction does not stabilize
at 5% to 6% until 6 months after the prior myocardial infarction. Thus, elective surgery,
especially thoracic and upper abdominal, or other major procedures used to be delayed for a
period of 2 to 6 months after a myocardial infarction. However, recently this has reduced to 6 to
8 weeks follow ing the ACC/AHA guidelines. Perioperative myocardial reinfarctions occur
most frequently in the first 48 to 72 hours postoperatively. However, the risk of myocardial
infarction remains increased for several months after surgery.

56. D. Careful preoperative evaluation is the most effective method of predicting a perioperative
cardiac event. Risk stratification based on preoperative history and physical examination
followed by some series of tests (if deemed necessary) predicts perioperative cardiac morbidity
and mortality risk. Invasive testing adds little information, which can be used to produce a
change in outcome. The risks of interventional procedures like angiography and an
intracoronary stent or even coronary artery bypass graft (CABG) surgery adds to the already-
existing risk of the proposed surgical procedure and does not reduce total risk. The combined
risk of two procedures exceeds that of the original operation. The American College of
Cardiology (ACC) and American Heart Association (AHA) have developed a protocol entitled
ACC/AHA Guideline Perioperative Cardiovascular Evaluation for Noncardiac Surgery. The
ACC/AHA guidelines have not been shown to actually reduce perioperative risk. Perioperative
medical optimization of the patient with P-adrenergic blockers, clonidine, statins, and aspirin
may be superior to invasive approach with angioplasty and/or CABG.

57. A. Deep anesthesia and brief duration of direct laryngoscopy (<15 seconds) is important in
minimizing the hemodynamic changes associated with intubation. If you anticipate a longer
intubation or if the patient has uncontrolled hypertension preoperatively, addition of other
drugs should be considered. Lidocaine can be given IV (1.5 mg/kg IV) or topically (2 mg/kg)
on the airway. Other pharmacologic options include esmolol 0.5 mg/kg and fentanyl 2 to 5
|ig/kg. However, brief duration of laryngoscopy seems to be the most effective method in
avoiding the sympathetic response to intubation.

58. D. Barlow syndrome, as it is sometimes called, refers to mitral valve. It is an abnormality of


the mitral-valve structure (suspected to be myxomatous in origin) that permits prolapse of the
mitral valve into the left atrium during left-ventricular systole. Any condition that increases
cardiac emptying can accentuate this prolapse: (1) sympathetic nervous system stimulation, (2)
decreased systemic vascular resistance, and (3) performance of surgery with patients in the
head-up or sitting position all of these conditions predispose to increased cardiac emptying.
Adequate preload and a sudden prolonged decrease in systemic vascular resistance must be
avoided during induction of anesthesia in these patients to prevent the worsening of prolapse.

59. D. Anesthetic considerations in patients with regurgitant lesions:


• Keep the heart rate high— decreases the duration of systole
• Keep SVR high
• Avoid decrease in myocardial contractility
• V wave is a reflection of mitral-regurgitant flow

60. B. QTc >440 ms in EKG is considered a predisposing factor for ventricular dysrhythmias,
syncope, and sudden death due to delayed repolarization. Common congenital syndromes
associated with these conditions are Jervell and Lange-Nielsen syndrome (with deafness) and
Romano Ward syndrome (no deafness). Any condition that increases the heart rate predisposes
these patients to arrhythmias— avoidance of sympathetic stimulation during anesthetic induction
is vital. Care should also be taken to avoid the drugs that prolong the QT interval like
phenothiazines. If the patient is hemodynamically stable, these patients can be treated with p-
blockers. Unstable ventricular arrhythmias need electrical cardioversion. Left-stellate ganglion
block has been shown to have some therapeutic benefit, suggesting an autonomic nervous
system imbalance as possible etiology for this syndrome.

61. D. Anesthetic considerations in patients with aortic stenosis:


• Maintaining a high SVR
• Optimal preload
• Avoiding extreme fluctuations in HR (60-80 bpm is ideal)
• Avoiding arrhythmias
• Rapid availability of a agonists to counter the drop in SVR with induction
• Accurate BP measurements preferably with an intra-arterial catheter

62. D. VPCs are recognized on the ECG by (1) premature occurrence, (2) the absence of a P wave
preceding the QRS complex, (3) a wide and often bizarre QRS complex, (4) an inverted T wave,
and (5) a compensatory pause that follows the premature beat. The primary goal with VPCs
should be to identify any underlying cause (myocardial ischemia, arterial hypoxemia,
hypercarbia, hypertension, hypokalemia, mechanical irritation of the ventricles) if possible and
correct it. VPCs can be treated with lidocaine (1 to 2 mg/kg IV) when they (1) are frequent
(more than six premature beats/min), (2) are multifocal, (3) occur in salvos of three or more,
or (4) take place during the ascending lim b of the T wave (R-on-T phenomenon) that
corresponds to the relative refractory period of the ventricle.

63. C. WPW syndrome is characterized by a short PR interval (less than 120 ms), a wide QRS
complex, and 5 wave in EKG. The short PR interval is due to conduction along the bundle of
Kent, which does not have a physiologic delay like conduction across the atrioventricular node.
The composite of cardiac impulses conducted by normal and accessory pathways is the reason
for 5 wave and wide QRS complex. WPW is the most common preexcitation syndrome, with an
incidence of approximately 0.3% of the general population. Atrial arrhythmias like paroxysmal
atrial tachycardia (most frequent) and supraventricular may lead to hemodynamic collapse in
patients with WPW syndrome.
Anesthetic management in the presence of a preexcitation syndrome is to avoid increase in
sympathetic nervous system activity events (anxiety) or drugs (anticholinergics, ketamine,
pancuronium) that might predispose to tachydysrhythmias. A ll cardiac antidysrhythmic drugs
should be continued throughout the perioperative period.
Ketamine with its sympathomimetic property w ill be a poor choice for induction. Intravenous
P-blockers (atenolol, metoprolol, propranolol, or esmolol) can be used to avoid tachycardia
during induction of anesthesia. Histamine-releasing agents like mivacurium/atracurium are also
preferably avoided. In case of a sudden onset of tachycardia, adenosine or procainamide w ill be
a good choice to treat the arrhythmia. Digitalis and verapamil may decrease the refractory
period of accessory pathways responsible for atrial fibrillation, resulting in an increase in
ventricular response rate during this dysrhythmia and should be avoided.

64. A. The magnet mode of many implanted devices, especially the newer AICDs, is now
programmable and does not always default to asynchronous pacing. Hence, it should not be
considered “ safe.” The specific magnet mode for a patient’s device should be identified by
interrogation prior to surgical procedures as some magnet modes change with device state or
are programmable. Electrosurgical cautery is interpreted as spontaneous cardiac activity by the
artificial cardiac pacemaker when the ground plate for electrocautery is placed too near the
pulse generator or with use of a unipolar cautery. For this reason, the electrical return plate
(wrongly called ground plate) should be placed as far as possible from the pulse generator.
Other techniques to improve safety include using a bipolar cautery, and placement of external
pads prior to the beginning of the case.
Magnet mode for many pacemakers (not AICDs) is asynchronous at 99 bpm. However, in some
devices, the magnet mode shifts to asynchronous at 50 bpm at the end of battery life.
Asynchronous pacing at such a low heart rate with the sensing function o ff may lead to R-on-T
phenomenon if the patient has a spontaneous heart rate above 50 bpm.

65. B. Cardiac tamponade is characterized by (1) decreases in diastolic fillin g of the ventricles,
(2) decreases in stroke volume, and (3) decreases in systemic blood pressure due to increased
intrapericardial pressure from accumulation of fluid in the pericardial space. Inadequate
ventricular fillin g leads to a decreased stroke volume, which in turn results in activation of the
sympathetic nervous system (tachycardia, vasoconstriction) in attempts to maintain the cardiac
output. These patients need to be kept “ fu ll and fast” as the right-sided fillin g occurs only when
central venous pressure exceeds the right-ventricular end diastolic pressure.

66. D. If the aortic valve is not competent, the regurgitant flo w from the aorta w ill keep
distending the left ventricle, impairing perfusion and myocardial preservation. This can be
avoided (1) by a drain placed from the right superior pulmonary vein into the left ventricle, (2)
by aspirating from the antegrade cardioplegia line placed in the proximal ascending aorta, or
(3) via a pulmonary venous drain. The goal is to keep the ventricle from overdistention when it
is not pumping. Venting of blood returning via the Thebesian or bronchial veins may also be
necessary.

67. D. The bypass pump serves to pump the oxygenated blood back to the arterial side of the
patient. They are of two types: centrifugal and roller pump. The centrifugal pump has three
disks rotating at 3,000 to 4,000 rpm that use blood viscosity to pump blood. Centrifugal pumps
are less traumatic to blood cells, do not pump air bubbles secondary to air being less dense than
blood, and are afterload-dependent, avoiding the risk of line rupture with clamping of the
arterial inflo w circuit. Roller pumps generate flo w by compression of fluid-filled tubing
between the roller and curved metal back plate and can pump air. Because of their mechanism,
they can cause tube rupture with arterial inflo w clamping. The flo w is determined by a dial on
the cardiopulmonary bypass machine, and usual flows for normothermia or m ild hypothermia
aim for a cardiac index between 2 and 4 L/min/m 2.

68. D. In the clinical scenario described, the patient has an increased CO2 in the blood
(irrespective of temperature correction).
Paco2 is a balance of CO2 production and removal. If removal exceeds production, Paco2
decreases. If production exceeds removal, Paco2 increases. The resulting Paco2 is expressed by
the alveolar CO2 equation:

Paco2 = k. VCO2/VA

In the equation, k is a constant (0.863) that corrects units, V C o 2 is carbon dioxide production,
and VA is alveolar ventilation. Since the patient is on cardiopulmonary bypass, increasing the
fresh-gas flo w to the oxygenator w ill wash out more C o 2. None of the other options has any
role in C o 2 production or elimination during cardiopulmonary bypass.

69. B. This patient has a drop in Pao2 from 90 to 70 mm Hg despite having a Fio 2 of 70. Along
with the relative hypoxemia, he also developed an increase in peak inspiratory pressure with no
change in plateau pressure. The lack of change in plateau pressure rules out any intrinsic
change in the lung compliance. Both ARDS and left-ventricular failure (pulmonary edema) w ill
result in a change in lung compliance. The clinical scenario described can result from both
tension pneumothorax and bronchial mucous plugging. But the fact that it occurred after 2 days
of mechanical ventilation and without any change in the hemodynamic status makes bronchial
mucous plugging the most likely cause.

70. C. The drop in mean arterial pressure at the beginning of CPB is caused by a sharp decrease
in systemic vascular resistance caused by the drop in hematocrit caused by the priming solution
on pump. This drop in blood pressure along with decreased hematocrit may cause a drop in
tissue oxygen delivery. This is very important for tissues with high oxygen consumption like
myocardium and brain. Use of a agonists to keep the mean arterial pressure may aid the
cerebral perfusion. The correct blood pressure during bypass is often decided based on the
patient’s coexisting conditions, carotid stenosis, etc. Lower pressures may reduce cerebral
blood flo w and emboli load to the brain. Higher pressures may improve cerebral blood flow
and reduce watershed infarction but higher pressures come from higher flows and more emboli
per unit time. Pressures less than 40 mm Hg are avoided if possible in adults. Pressures higher
than 60 mm Hg are used during rewarming. Pressures up to 80 to 90 mm Hg may be used in
patients with cerebral vascular disease.

71. B. A mixed venous Po 2 lower than 30 mm Hg associated with metabolic acidosis suggests
inadequate tissue perfusion. Temperature correction of Paco2 and pH is probably not necessary.
Urine output may serve as a guide to the adequacy of renal perfusion, with an output of 0.5 to 1
mL/kg/hr, indicating adequate renal perfusion.

72. C. Monitoring of coronary sinus pressures during retrograde administration is used to assess
proper catheter placement. The anatomical location of coronary sinus ostia makes it very
difficult for proper visualization by the surgeon. A properly placed coronary sinus catheter w ill
have pressure of 40 to 60 mm Hg during a 200-mL/min infusion. If the pressure at the distal tip
of the coronary sinus catheter during cardioplegia administration at 200 mL/min is equal to
central venous pressure, the catheter is not in the coronary sinus but is most likely in the right
atrium or in the superior vena cava. Being up against a wall produces a very high (>100 mm
Hg) pressure. Positioning of the coronary sinus catheter should be checked with
transesophageal echocardiography and manual feel by the surgeon. If the catheter is too deep,
cardioplegia to the right ventricle w ill be compromised, resulting in poor right-ventricular
protection.

73. C. At 30°C, the heart muscle consumes oxygen at a rate of 8 to 10 mL/100 g/min, provided it
is normally contracting. Oxygen consumption in the fibrillating ventricle at 22°C is 2 mL/100
g/min. The electromechanically quiet heart at 22°C consumes oxygen at a rate of 0.3 mL/100
g/min.

74. A. The extra volume of crystalloid used in priming the CPB circuit may produce a sudden
dilution of circulating drug concentrations. This creates a high chance of patient recall/
movement. Supplemental anesthetics, such as benzodiazepines or opioids, and an additional
dose of nondepolarizing muscle relaxant, may be administered prophylactically. Volatile
anesthetics delivered using vaporizers incorporated into the CPB circuit have largely negated
this problem along with the use of BIS monitors. The effect of hemodilution on drug
concentrations is likely to be offset by a decreased need for drugs during hypothermia. On the
contrary, anesthetic requirements seem to be minimal if the patient was adequately rearmed at
the conclusion of CPB. Therefore, additional anesthesia is not routinely required during
rewarming at the termination of CPB.

75. B. Low SVR is a very common hemodynamic abnormality after CPB. This makes weaning
from CPB very difficult. SVR is usually calculated using the formula mean arterial pressure
(mm Hg) - central venous pressure (mm Hg)/pump flo w (L/m i) x 80.
SVR should be between 1,200 and 1,400 prior to CPB separation. The units of SVR are dyn
s/cm5. SVR can be normalized with a vasoconstrictor prior to weaning from CPB. By this, we
are attempting to match the vascular input impedance to the cardiac output impedance and
optimizing energy transfer.
76. D. Acute mitral regurgitation (MR) post-CPB is often noticed as a prominent V wave in
PAOP tracing. If there is a transesophageal echocardiograph (TEE) in place, we may be able to
see a wide MR jet, with observation of an echogenic mass attached to the mitral valve or when a
mobile mass is seen to prolapse into the left atrium during systole and to move back into the left
ventricle during diastole. The posterior papillary muscle, along with the posterior wall, is
entirely perfused either by the right coronary artery (RCA) or by the third obtuse marginal
branch, usually by a single artery unlike the anterior, which derives its blood supply from two
arteries. It is usually a complication of acute mitral infarction but maybe seen at the end of CPB
due to inadequate myocardial protection (warm blood in the adjacent descending aorta
providing inadequate protection) during CPB or air entry into the RCA. Acute MR due to
volume overload from excessive fluid administration is usually a central MR as evidenced in
TEE with a distended ventricle and can be managed by decreasing the preload.
Thoracic Anesthesia
Deppu Ushakumari and Ashish Sinha

1. Which of the follow ing is not a characteristic feature of asthma?


A. Chronic inflammatory changes in the submucosa of the airways
B. Airway hyper responsiveness
C. Reversible expiratory airflow obstruction
D. Elastase deficiency in the airways

2. A 55-year-old male presented to you with a pulmonary function test report, which shows an
increase of FEV: percent predicted of more than 12%, and an increase in FEV1 of greater than
0.2 L in response to bronchodilators. Which characteristic of his respiratory illness is depicted
here?
A. Bronchial asthma— acute bronchodilator responsiveness
B. Chronic obstructive pulmonary disease (COPD)— variability in airflow obstruction
C. COPD— acute bronchodilator responsiveness
D. A ll the above are correct

3. Which of the follow ing techniques is associated with a lower complication rate related to
bronchospasm in the asthmatic population?
A. Regional anesthesia
B. General anesthesia— laryngeal mask airway (LM A)
C. General anesthesia— endotracheal tube (ETT)
D. Combined general and neuraxial anesthesia

4. A 22-year-old patient with a history of moderate persistent asthma on medium-dose inhaled


corticosteroids and long-acting inhaled 0-agonist presents for an emergency appendectomy. On
clinical examination, he is actively wheezing, but maintaining an oxygen saturation of 99% on
room air. Which of the follow ing statements about this clinical scenario is most appropriate?
A. Presence of wheezing on physical examination indicates that he is having a severe attack of
asthma
B. Volatile anesthetics cause bronchodilation through catecholamine-independent mechanisms
C. Increased airway resistance that occurs intraoperatively is usually due to acute exacerbation
of asthma
D. A laryngeal mask airway (LM A) is more stimulating to the airway than an endotracheal
tube, and should be avoided in asthmatics

5. During the above case, the end-tidal sevoflurane concentration reads 3.5, but the anesthesia
ventilator is alarming because of high peak airway pressures. Which of the follow ing is the
most likely cause?
A. Acute bronchospasm
B. Anaphylactic reaction to intravenous muscle relaxant that you just administered
C. Mechanical causes of obstruction
D. Inadequate depth of anesthesia

6. At the end of the above case, the surgeon requests you to extubate the patient fu lly awake
because he found extensive intestinal adhesions and is afraid of retained gastric contents in the
stomach. Which of the follow ing w ill be your most likely plan of action?
A. Insert an orogastric tube, empty the stomach as much as you can, and proceed with a deep
extubation to avoid bronchospasm
B. Administer intravenous lidocaine to decrease the likelihood of airway stimulation and wait
till the patient is fu lly awake before extubation
C. Shut o ff the inhalational agent and use intravenous propofol to avoid transitioning through
a rocky stage-2 wake up
D. Transition to a laryngeal mask airway (LM A) under sevoflurane anesthesia and let the
patient wake up with an LM A

7. Which of the follow ing is true regarding administering general anesthesia to a chronic
obstructive pulmonary disease (COPD) patient?
A. Nitrous oxide + opioid technique is ideal
B. Use large tidal volumes
C. Use lower breathing rates to permit more exhalation time
D. Correct the hypercapnia intraoperatively to help extubate early

8. Anesthetic considerations for a patient with severe pulmonary hypertension include all the
following, except
A. Right heart catheterization is the gold standard for diagnosis
B. M ortality in pregnant patients undergoing vaginal delivery is very small as opposed to
cesarean section
C. Minimize tachycardia, hypoxemia, and hypercapnia during anesthetic management
D. Cardiac output from a failing right ventricle depends on fillin g pressure from venous
return and pulmonary pressure

9. Which of the follow ing is not a part of the “ STOP BANG” screening questionnaire for
obstructive sleep apnea (OSA)?
A. Snoring
B. Observed apnea
C. Exercise tolerance
D. High blood pressure

10. Risk factors associated with increased perioperative m orbidity and mortality in thoracic
surgery patients include all the following, except
A. Extent of lung resection
B. Age older than 70 years
C. Experience of the operating surgeon
D. Male sex

11. The follow ing is not necessarily a part of prethoracotomy respiratory assessment
A. Pulmonary capillary wedge pressure >18 mm Hg
B. Predicted postoperative FEV1 >40%
C. V o 2 max >15 mL/kg/min
D. Predicted postoperative diffusing capacity for carbon monoxide (D l c o ) >40%

12. Which of the follow ing is one of the benefits regarding cessation of smoking 12 to 24 hours
prior to surgery?
A. Shift of oxyhemoglobin dissociation curve to the right
B. Improvement in mucociliary transport
C. Decrease in sputum production
D. Improved small-airway function

13. In surgical cases requiring lung isolation


A. Measurement of tracheal width from a posteroanterior chest radiograph is of no use in
selecting the size of a double-lumen tube (DLT)
B. More frequent use of left-sided DLT is based on the anatomy of tracheobronchial tree
C. Uniform ventilation to all lobes is most likely achieved by a right-sided DLT because it has
a ventilation slot in the bronchial tube
D. Fiber-optic confirmation of correct DLT placement is not required if you have good
clinical confirmation

14. A 59-year-old lady is intubated with a 37 left-sided double-lumen tube (DLT) for wedge
resection of left lower lobe nodule. After intubation, you inflate the bronchial cuff and ventilate
the left lung through the bronchial lumen without any difficulty. Then you proceed to inflate the
tracheal cuff and ventilate through the tracheal lumen. You notice a very high resistance to air
flow. Which of the follow ing events is most unlikely with the said clinical picture?
A. Left DLT too deep with the tracheal outlet into the left main-stem bronchus
B. Left DLT displaced with the bronchial cuff herniated at carina
C. Left DLT entered the right bronchus with the tracheal outlet in the right main stem
D. DLT too far out with the bronchial lumen sitting just above the carina

15. What would you do if you have the follow ing situation with a bronchial blocker for left lung
surgery?

A. Appropriate positioning for this surgery


B. Withdraw the bronchial blocker a couple of centimeters
C. Insert the bronchial blocker a few centimeters farther down
D. Remove the bronchial blocker and reinsert it into the left side

16. In which of the follow ing situations is applying continuous positive-airway pressure (CPAP) to
the nondependent lung most ideal for improving oxygenation?
A. Bronchopleural fistula
B. Open lobectomy
C. Massive pulmonary hemorrhage
D. Sleeve resection

17. Which of the follow ing statements is false regarding ventilation/perfusion relationship in a
lateral decubitus position during spontaneous ventilation?
A. The ventilation/perfusion matching is preserved
B. Contraction of dependent hemi diaphragm is more efficient
C. Dependent lung is on a more favorable part of the compliance curve
D. The lower lung receives less ventilation and more perfusion than the upper lung

18. In an open pneumothorax, the major effect of mediastinal shift is to


A. Decrease the contribution of dependent lung to the tidal volume
B. Move air to and fro between the dependent and the nondependent lung
C. Decrease the perfusion to the dependent lung
D. Compress the big veins and decrease cardiac preload

19. Factors known to inhibit hypoxic pulmonary vasoconstriction (HPV) and thus worsen the
alveolar-arterial oxygen gradient include all of the following, except
A. Hypocapnia
B. Nitroglycerin
C. Hypercapnia
D. Pulmonary infection

20. A 64-year-old female is undergoing a left video-assisted thoracoscopy for a suspicious


pulmonary nodule. Immediately after positioning the patient laterally, which of the follow ing
alarms indicates a malposition of the double-lumen tube (DLT)?
A. High CO2 alarm
B. Low O2 alarm
C. Low tidal volume alarm
D. Unable to drive bellows alarm

21. A bronchial blocker is useful in all of the follow ing clinical situations, except
A. Patient to be left intubated post operatively
B. Anatomical abnormality precluding the placement of a double-lumen tube (DLT)
C. Tamponading bronchial bleeding in adult patients
D. To attain better collapse of the nondependent lung

22. Which of the follow ing statements about lung resection surgery is false?
A. M ortality rate for pneumonectomy is 5% to 7%
B. M ortality rate for lobectomy is 2% to 3%
C. M ortality is higher for left-sided pneumonectomy
D. Most postoperative deaths result from cardiac issues

23. Regarding lung resection surgery, which of the follow ing statements is false?
A. Perioperative arrhythmias are common
B. Supraventricular tachycardias (SVTs) are thought to result from surgical manipulation or
distension of the right atrium
C. Incidence of arrhythmia decreases with age due to the ageing of cardiac conduction system
D. Postoperative hypoxemia and acidosis due to atelectasis and shallow breathing are common

24. Which of the follow ing has the least effect on hypoxic pulmonary vasoconstriction (HPV)?
A. Nitrous oxide
B. Desflurane end tidal 5.5%
C. Sevoflurane end tidal 2.5%
D. Isoflurane end tidal 1.5%

25. Which of the follow ing statements is not true regarding “ lower lung syndrome” ?
A. It is caused by excessive fluid administration in a lateral decubitus position
B. It increases intrapulmonary shunting
C. It is gravity-dependent transudation of fluid into the dependent lung
D. It is due to volutrauma caused during one-lung ventilation

26. The first step recommended to improve oxygenation if a patient is exhibiting drop in oxygen
saturation during one-lung ventilation is
A. Apply continuous positive-airway pressure (CPAP) to the collapsed lung
B. Apply positive end-expiratory pressure (PEEP) to the dependent lung
C. Periodic inflation of the collapsed lung
D. Continuous inflation of oxygen into collapsed lung

27. During apneic oxygenation,


A. Adequate oxygenation can be maintained only for short periods of time
B. Arterial Pc o 2 rises 3 to 4 mm Hg in the first minute
C. Arterial Pc o 2 rises 1 to 2 mm Hg each subsequent minute after the first minute
D. Progressive respiratory acidosis limits the use of this technique to 10 to 20 minutes in most
patients

28. A 68-year-old male patient with a lung nodule underwent a right upper lobectomy. On
postoperative day 4, the patient develops a sudden large air leak from the chest tube associated
with increasing pneumothorax and partial lung collapse. The most likely cause is
A. Bronchopleural fistula on the right from necrosis of suture line
B. Bronchopleural fistula on the right from inadequate surgical closure of the bronchial stump
C. Atelectasis causing shifting of the mediastinum to the left
D. A normal finding

29. An 80-year-old female underwent a left lower lobectomy. In the ICU on postoperative day 2,
she develops hemoptysis. The vital signs are stable, but on the chest X-ray a homogenous
density is seen in the left lower lung area. After subsequent bronchoscopy, the left upper lobar
orifice is closed. The most likely diagnosis is
A. Acute herniation of the heart into the left lower lobe area
B. It is a normal finding and the homogenous opacity is due to accumulation of fluid in the left
lower lobe area
C. Torsion of the left upper lobe as the left upper lobe expanded to occupy the left hemithorax
D. Reexpansion edema of the left upper lobe

30. A 45-year-old recent immigrant from Vietnam is admitted to the emergency department with
massive hemoptysis (>600 mL in the last 24 hours). You are called to evaluate the patient for a
possible bronchial artery embolization or a rig id bronchoscopy. In your discussion with the
patient, which of the follow ing statements is not appropriate about his clinical condition?
A. Operative mortality exceeds 20%
B. It can be done as a semi-elective procedure, and there is no need to do it emergently
C. The most common cause of death is asphyxia secondary to blood in the airway
D. Medical management has a lower mortality rate than operative management

31. An 81-year-old chronic smoker, with a history of 60 pack year smoking, is admitted with
progressive dyspnea and a huge right-sided pulmonary cyst. The cyst is compressing her
remaining right lung, and she is brought to the OR for an emergency pulmonary cystectomy.
Which of the follow ing is right regarding anesthetic management of this patient?
A. The greatest risk of rupture of the cavity is during preoxygenation just prior to induction
B. These cavities allow to and fro movement of air and have a very low chance to
progressively enlarge
C. Maintenance of spontaneous ventilation is desirable until a double-lumen tube (DLT) is in
place
D. Assisted ventilation is not necessary immediately after induction and can be harmful

32. A 66-year-old patient with a history of severe tracheal stenosis is presenting for a tracheal
resection. The most unlikely clinical finding is
A. Progressive dyspnea
B. Wheezing evident on exertion
C. Dyspnea worse on sitting up and leaning forward
D. Patient may have a history of blunt/penetrating tracheal trauma

33. In the anesthetic management of the above patient, which of the follow ing statements is correct?
A. Flow-volume loops aid the clinician in evaluating the severity of the lesion
B. Right radial artery blood pressure monitoring is preferred over the left side for lower
tracheal resection
C. Slow-inhalation induction is not advisable and a rapid-sequence induction should be used
D. Early extubation is not advisable at the end of the procedure for risk of rupturing the suture
lines
34. Complications associated with mediastinoscopy include all the following, except
A. Vagally mediated reflex bradycardia
B. Cerebral ischemia
C. Pneumothorax
D. Thoracic duct injury

35. Anesthetic considerations for bronchoalveolar lavage include all the following, except
A. It is performed for patients who make excess quantities of surfactant and fail to clear it
B. It is performed under general anesthesia with lung isolation
C. It is usually performed in the supine position
D. It involves positioning the patient in a lateral position to aid active suctioning of the lavage
fluid

36. Considerations for lung transplantation include all the following, except
A. Cor pulmonale does not necessarily require combined heart-lung transplantation
B. Patients with diminished left-ventricular function can be transplanted as long as they have
normal right-ventricular function
C. Patients with Eisenmenger syndrome require combined heart-lung transplantation
D. Organ selection is based on size and ABO compatibility

37. After a double-lung transplantation


A. Loss of lymphatic drainage predisposes to pulmonary edema
B. Respiratory pattern changes to a slow deep respiration
C. Cough reflex is abolished above the carina
D. Hypoxic pulmonary vasoconstriction is abolished

38. Anesthetic considerations for esophageal surgery include


A. Very low risk of pulmonary aspiration
B. Mandatory pulmonary artery catheter monitoring
C. Diaphragmatic retractors interfering with cardiac function
D. Always performed with a double-lumen tube (DLT)

39. Lung-volume-reduction surgery (LVRS)


A. Has been demonstrated to have very good efficacy by the National Emphysema Treatment
Trial (NETT)
B. Necessitates lim iting of peak inspiratory pressure to 30 cm H2O follow ing intubation
C. A prolonged inspiratory time is recommended for facilitating exposure of the surgical
segments
D. Patients have a better outcome if kept intubated at the end of the surgery

40. Which of the follow ing flow-volum e loops w ill be expected in a child with variable
extrathoracic obstruction?

41. A 12-year-old boy with suspected lymphoma presents to you for a lymph node biopsy. When
you go to visit the patient, you notice that he has venous engorgement and edema of the head,
neck, and arms. He refuses to lie down, and is tachycardiac and tachypneic. The preferred
management for this boy would be
A. Safest thing w ill be to secure the airway immediately by using rapid-sequence induction
B. Preferably biopsy the lymph node under local anesthesia so that the patient can be sent for
radiotherapy immediately after a tissue diagnosis
C. Empiric treatment with steroids and surgery under general anesthesia only after the airway
compromise is alleviated
D. Get a chest X-ray and rule out mediastinal compression prior to any active management
CHAPTER 12 ANSWERS

1. D. Asthma is a type of reactive airway disease characterized by hyperresponsive airways,


reversible expiratory airflow obstruction, and chronic inflammation. Sudden bronchospasm in
response to external/internal stimuli and response to bronchodilators like P2-agonists are
important distinguishing features of asthma. Elastase deficiency in the airways is a feature of
emphysema.

2. A. Response to a bronchodilator drug resulting in relief of airway obstruction is highly


suggestive of bronchial asthma. A more than 12% increase in predicted FEV1 and an absolute
increase in FEV1 of more than 0.2 L suggest acute bronchodilator responsiveness and
variability in airflow obstruction. The reversibility of this magnitude is almost always
indicative of bronchial asthma. COPD patients do respond to bronchodilators but not to the
same extent. Early stages of asthma are diagnosed by decreased mid expiratory flo w rates
(effort independent) and decreased FEV1 and by its reversibility.

3. A. In a severely asthmatic patient, regional anesthesia is superior to general anesthesia with an


LMA, which is better than general anesthesia with ETT. The choice of anesthetic technique is
often influenced by the severity of asthma, history of previous intubations for asthma,
dependence on inhaled bronchodilators, and patient preference. The goal in any such
circumstance is to decrease airway manipulation and stimulation. If a general anesthetic
technique is pursued, inhaled bronchodilator therapy immediately prior to induction, use of
non-histamine-releasing drugs, airway manipulation only after deep anesthetic plane, and use
of intravenous lidocaine prior to intubation have all been proven to be useful.

4. B. History and physical examination can suggest presence of severe asthma if the patient has
had repeated intubations for asthma. Even though high-pitched, musical wheezes are
characteristic of asthma, they are not specific and they have no correlation with the severity of
obstruction. Spirometry is the only objective method to quantify the severity of obstruction.
Sudden severe bronchospasm can present as high airway pressures with absence of breath
sounds and very high resistance to mechanical ventilation. Mechanical causes of obstruction
such as a kinked endotracheal tube or a mucous plug can also present a similar clinical picture
and are more common. If bronchospasm is suspected, anesthesia should be augmented with an
intravenous anesthetic such as propofol. General anesthesia through a LM A is less stimulating
to the airway than through an endotracheal tube. Volatile anesthetics are potent bronchodilators,
and they act through catecholamine-independent mechanisms. They are rarely used as second-
line agents in cases of bronchospasm refractory to medical therapy.

5. C. Acute bronchospasm causes expiratory wheezing, increased peak inspiratory pressure or


decreased tidal volume (depending on the mode of ventilation), and a characteristic upslope of
the capnogram. Any airway stimulation can cause severe reflex bronchoconstriction and
bronchospasm in severely asthmatic patients with hyperactive airways. Mechanical causes of
obstruction such as a kinked endotracheal tube or a mucous plug can also present a similar
clinical picture and are more common. When troubleshooting such a scenario, an intravenous
anesthetic agent is very helpful to deepen the plane of anesthesia as the delivery of inhaled
anesthetic agents may not be effective.

6. B. A patient who has adequate return of neuromuscular function and has a regular spontaneous
breathing pattern with adequate tidal volumes can be considered a candidate for deep extubation.
After clearing out the secretions from the oropharynx and the endotracheal tube, extubation is
performed under a deep plane of anesthesia and ventilation continued by a mask/LMA. Careful
patient selection is very important, and it should not be considered in those at increased risk for
aspiration of gastric content and if the necessary airway management skills are not immediately
available. When extubation is delayed for reasons of patient safety, (presence of gastric contents
in a case with acute appendicitis), intravenous administration of lidocaine (1.5-2 mg/kg bolus)
may decrease the likelihood of airway irritation and bronchospasm. Thus, extubation can be
performed after the patient is awake and follow ing commands if airway irritation can be
avoided.

7. C. Balanced anesthesia using an inhaled anesthetic and opioid is a safe choice for anesthesia
for a COPD patient. Use of nitrous oxide (N2O) is normally safe but not strictly necessary. The
ability of N2O to diffuse into closed air spaces may lead to the enlargement of an
emphysematous bulla or a pneumothorax and possibly rupture. A ir trapping and development
of auto positive end-expiratory pressure can be decreased by providing a prolonged expiratory
time. This can be done by using a normal tidal volume and a slow respiratory rate and an I:E
ratio of >1:3. Care should be taken to avoid hyperventilation and creation of a respiratory
alkalosis as these patients tolerate marked hypercapnia secondary to hypoventilation. However,
high Paco2 levels w ill increase pulmonary artery pressure, which may be poorly tolerated in
patients with a compromised right-ventricular function and cor pulmonale. Bronchodilation
using inhaled P2-agonists and pulmonary toilet through blind suctioning or fiberoptic
bronchoscopy may facilitate safe extubation of the trachea.

8. B. Pulmonary hypertension is defined as an increase in mean pulmonary artery pressure


above 25 mm Hg at rest or 30 mm Hg with exercise in the presence or absence of an elevated
pulmonary capillary wedge pressure. Right-sided heart catheterization is the gold standard for
diagnosing and quantifying the degree of pulmonary hypertension. Care should be taken to
avoid all the factors that increase pulmonary vascular resistance in a patient with severe
pulmonary hypertension presenting for surgery. This includes avoiding hypoxia, hypercarbia,
hypothermia, light anesthesia, pain, dysrhythmias, and maintaining adequate cardiac output.
Progressive right-ventricular dilation and hypertrophy in response to an increased afterload
generated by chronic pulmonary hypertension w ill eventually lead to right-ventricular systolic
dysfunction, inadequate left-ventricular fillin g , and eventually biventricular failure. The
interventricular septal bulge decreases left-ventricular cavity fillin g , further worsening the left-
ventricular failure. Cardiac output from a failing right ventricle depends on the fillin g pressure
from venous return and pulmonary pressure. Pulmonary hypertension in pregnant patients has a
high mortality rate up to 50% for vaginal delivery and even higher for cesarean delivery.

9. C. Snoring, daytime sleepiness, hypertension, obesity, and a fam ily history of OSA are risk
factors for OSA. There is a high risk for OSA if >3 yes to the below questions.
S (snore) Have you been told that you snore?
T (tired) Are you often tired during the day?
O (obstruction) Do you know if you stop breathing or has anyone witnessed you stop breathing
while you are asleep?
P (pressure) Do you have high blood pressure or on medication to control high blood
pressure?
B (BMI) Is your body mass index greater than 28?
A (age) Are you 50 years old or older?
N (neck) Are you a male with a neck circumference greater than 17 inches, or a female with a
neck circumference greater than 16 inches.
G (gender) Are you a male?

10. D. The extent of lung resection (pneumonectomy > lobectomy > wedge resection), age older
than 70 years, and inexperience of the operating surgeon are risk factors associated with
increased perioperative m orbidity and mortality rates. In patients with anatomically resectable
lung cancer, pulmonary function tests— ppoFEV1, lung perfusion scanning, and exercise testing
to measure maximum oxygen consumption (Vo2max)— may predict postoperative pulmonary
function and outcome.

11. A. The prethoracotomy respiratory assessment has been labeled as a three-legged stool that
incorporates assessment of respiratory mechanics, cardiopulmonary reserve, and lung
parenchymal function. The follow ing findings are considered favorable: respiratory mechanics
assessment demonstrates ppoFEV1 >40%, MVV, RV/TLC, and FVC; the cardiopulmonary
reserve measurements show Vo2 max >15 mL/kg/min, 6-minute walk test, exercise Spo2 <4%,
stair climb >2 flights, and assessment of lung parenchymal function shows ppoDLCO >40%,
Pao2 >60, PacO2 <45 mm Hg. The choices B, C, and D are the most valid tests out of the “ three­
legged stool.”

12. A. Any patient presenting for elective surgeries with a history of smoking should be advised
smoking cessation regardless of the time available prior to surgery. Smoking can affect the
pulmonary system in multiple ways— increase in airway irrita b ility and secretions, decreased
mucociliary transport, and increased incidence of postoperative pulmonary complications.
Patients are more receptive toward interventions immediately prior to surgery and this provides
a good teachable moment. Prolonged abstinence (8-12 weeks) is required to improve
mucociliary transport and small-airway function and decrease sputum production. The
incidence of postoperative pulmonary complications decreases with abstinence from cigarette
smoking for more than 8 weeks in patients undergoing coronary artery bypass surgery, and
more than 4 weeks in patients undergoing pulmonary surgery. However, even 12 to 24 hours of
cessation may be beneficial because it decreases the level of carboxyhemoglobin and it shifts
the oxyhemoglobin dissociation curve to the right.

13. B. The angle between the right main bronchus and trachea is 25 degrees at the level of carina,
but the left main bronchus takes o ff at an acute angle of 45 degrees. Thus, right main bronchus
is shorter, wider, and more in line with the trachea. There is a good correlation between tracheal
and bronchial width (bronchial diameter is predicted to be 0.68 of tracheal diameter).
The right upper lobe bronchus takes o ff at an acute angle from the point of origin of the right
primary bronchus and is easily occluded if the ventilation port on the right-sided DLT is not
aligned properly. Because of these reasons, a left-sided DLT is most commonly used. Uniform
ventilation to all lobes can be achieved more easily with a left-sided DLT than a right-sided one.
Measurement of tracheal width from a posteroanterior chest roentgenogram can help select the
size of a left-sided DLT. In addition to physical examination, fiberoptic assessment should be
done to confirm proper position of a left-sided DLT because the malposition incidence if
confirmed with auscultation alone is considered to be 20% to 48%.

14. C. There is no single predictor that can accurately predict the appropriate size of a DLT. But a
general guideline is a woman shorter than 160 cm should be intubated with a 35-Fr tube, a
woman taller than 160 cm should be intubated with a 37-Fr tube, and a man shorter than 170 cm
should be intubated with a 39-Fr tube, and a man taller than 170 cm should be intubated with a
41-Fr tube. This tube size of 37 Fr may be too big for this lady. When we inflated the bronchial
cuff and attempted ventilation, the unoccluded outflow tract of the bronchial lumen made it easy
to ventilate. However, after inflating the tracheal cuff (with the bronchial cuff already inflated),
failure to ventilate suggests that something is occluding the tracheal lumen (the bronchial cuff
in this situation). The presence of breath sounds only on the right side with both the cuffs
inflated suggests that the bronchial lumen is patent and ventilating the right side. The ventilated
gas coming out of tracheal lumen is being trapped between the tracheal and the bronchial
balloons. This finding can be confirmed with fiberoptic bronchoscopy, and the tube needs to be
repositioned with bronchial cuff in the left main-stem bronchus. If with the same clinical
picture, you are hearing breath sounds only on the left side with both the cuffs inflated, it could
be that the DLT is too far into the left bronchus with the tracheal lumen opening into the left
main stem. It is also possible that the bronchial cuff is barely into the left main stem with a
herniated bronchial cuff preventing the inflation of right-sided lung via the tracheal lumen.
15. B. Figure 12-4A is a bronchial blocker for a left-sided lung surgery placed deeply into the left
main-stem bronchus. Figure 12-4B is a bronchial blocker advanced too far into the right main
stem, and the balloon of the bronchial blocker is occluding the right upper lobe takeoff. For
both situations, the bronchial blocker should be withdrawn a couple of centimeters to a level
just below the carina.
F igu re 12-4.
16. B. Difficulties with oxygenation are fa irly common during one-lung ventilation. If the Spo2 is
below acceptable range, various techniques that can be tried to improve the oxygenation include
increasing Fio 2 to 1, intermittent two-lung ventilation, applying positive end-expiratory
pressure (PEEP) to the dependent lung. However, the most effective method is the application of
5 to 10 cm H2O CPAP to the nondependent lung. This should be done prior to application of
PEEP to the dependent lung. This low level of CPAP results in minimal lung inflation and
generally does not interfere with surgery. A slow inflation of 2 L/min of oxygen into the
nonventilated lung for 2 seconds and repeated every 10 seconds for 5 minutes or until the
saturation rises to 98% has been shown to improve oxygenation during one-lung ventilation.
CPAP applied to the operative lung may be disadvantageous in some cases like thoracoscopy,
bronchopleural fistula, sleeve resection, or massive pulmonary hemorrhage.

17. D. During spontaneous ventilation in lateral decubitus position, ventilation/perfusion


matching is preserved because the lower lung receives more perfusion due to gravity and more
ventilation due to better contraction of the dependent hemidiaphragm. The dependent
hemidiaphragm gets a better displacement from a higher position in the chest. The dependent
lung has a better compliance as well— this improves ventilation.

18. A. Inspiration in a lateral position during spontaneous ventilation causes more negative
pleural pressure on the dependent side of the open pneumothorax. The relatively higher
pressure on the nondependent side causes a downward shift of the mediastinum during
inspiration. The reverse happens on expiration, and the mediastinum shifts upward. This results
in an ineffective respiratory exchange, but the major effect is by decreasing the contribution of
the dependent lung to the tidal volume.

19. C. HPV is a protective mechanism by which body shunts away blood from a nonventilated
lung. It plays a significant role in maintaining oxygenation during one-lung ventilation. Factors
inhibiting HPV include infection, pulmonary hyper/hypotension, low Paco2, changes in Svo2
(mixed venous oxygen saturation), and pharmacological agents like vasodilators—
nitroglycerin and nitroprusside, P-agonists, calcium channel blockers, and inhalational
anesthetic agents.

20. C. Left-sided DLTs are most commonly used in clinical practice. Any change in position of
the patient after the DLTs have been placed includes a risk of malpositioning of the DLT. Low
exhaled tidal volumes and poor lung compliance are the most common initial indicators. Left­
sided DLT may be malpositioned back into the trachea, into the right main-stem bronchus, or
too far into the left primary bronchus. If it is in the trachea, the inflated bronchial cuff is
preventing any ventilated gas from going past it. If it is in the right side or pushed too far into
the left bronchus, the bronchial cuff may be obstructing the left upper or left lower lobe
bronchus. These two situations can be immediately relieved by deflating the bronchial cuff.

21. D. DLTs are considered to be the best lung isolation device currently in use. In certain
situations, placement of a DLT is difficult and bronchial blockers are used for lung isolation.
They are similar to Fogarty catheters and are single-lumen devices with an inflatable balloon at
the tip. They are passed through a single-lumen endotracheal tube under fiberoptic guidance and
the balloon is inflated within the bronchus of the operative side. The cuff of the bronchial
blocker is a high-pressure-low-volume cuff. The single narrow lumen within the blocker
allows the lung to deflate (though slowly) and can be used for suctioning or insufflating oxygen
(below). The biggest problem is caused by the small size of the channel, which impairs
exhalation. However, in patients with a history of difficult intubation bronchial blockers
circumvent the need to reintubate a patient prior to transferring out of the operating room.

22. C. The mortality rate for pneumonectomy is about 5% to 7%, compared with 2% to 3% for a
lobectomy. M ortality is higher for right-sided pneumonectomy than for left-sided
pneumonectomy. This is attributed to the greater loss of lung tissue. Lung cancer resection
surgeries involve finding the right balance between resecting enough lung tissue to obtain a
tumor-free margin, at the same time leaving enough for residual postoperative pulmonary
function. Wedge resections for peripheral lesions, lobectomy for bigger tumors,
pneumonectomy for tumors involving the main bronchus, sleeve resections for patients with
proximal lesions, and limited pulmonary reserve are among the various choices the surgeons
can make.

23. C. The incidence of arrhythmias increases with age and with the amount of pulmonary
resection. Perioperative arrhythmias are fa irly common after thoracic surgery— atrial
fibrillations/SVTs and PVCs are all seen and are thought to be a result of the surgical
manipulation of the heart and distension of the right atrium as a result of decreased pulmonary
vascular bed. Pulmonary complications after surgery can be decreased by preoperative
incentive spirometry, bronchodilator therapy, and good pulmonary hygiene.

24. B. Halogenated agents generally have minimal effects on HPV in doses <1 minimum alveolar
concentration (MAC). Balanced anesthetic technique using a combination of inhaled anesthetic
agents and intravenous opioids is beneficial. Inhalational agents allow delivery of 100%
oxygen, are potent bronchodilators, and can be easily titrated to desired concentration and
opioids have minimal hemodynamic effects as well as providing analgesia. They complement
each other very well. However, use of long-acting opioids should be limited during surgery to
prevent excessive postoperative respiratory depression. The only choice with <1MAC is B.

25. D. Anesthetic management of pulmonary resections includes very tight fluid management.
Most of the time restrictive fluid management strategy facilitated by use of blood/colloids is
entertained. Lower lung syndrome refers to gravity-dependent transudation of fluid into the
dependent lung, which decreases effective oxygenation, increases ventilation-perfusion
mismatch, shunting, and promotes hypoxemia. This transudation is worsened by excessive
administration of intravenous crystalloids. On the nondependent side, reexpansion of the
collapsed lung can result in pulmonary edema due to alteration in the pressures on either side of
the Starling equation.

26. A. Hypoxemia is fa irly common after institution of one-lung ventilation in the lateral
position. Various interventions can be tried when it happens— some have better efficacy than the
others. These include periodic reinflation of the collapsed lung with oxygen, which interferes
with surgery; early ligation or clamping of the ipsilateral pulmonary artery— seldom used but
can be tried in pneumonectomies; and CPAP (5-10 cm H2O) to the collapsed lung causing
partial reexpansion of the lung and may interfere with surgery. The mechanism of action of
CPAP application is supposed to oxygenation as well as displacement of blood from the
pulmonary vasculature into the dependent lung. Other less efficacious methods that can be tried
include PEEP (5-10 cm H2O) to the ventilated lung, oxygen insufflation to collapsed lung
(diffusion respiration), and change in dependent lung minute ventilation (Vt 5 mL/kg is usually
recommended but can be increased). Application of CPAP to the collapsed lung should be done
before instituting PEEP to the ventilated lung, as the effect of PEEP depends on where the lung
falls on the PEEP-PVR curve. Persistent hypoxemia requires immediate return to two-lung
ventilation.

27. D. Apneic oxygenation refers to insufflation of 100% oxygen at a rate greater than the
oxygen consumption (>250 mL/min) while the ventilation is stopped. Progressive respiratory
acidosis limits the use of this technique to 10 to 20 minutes in most patients. Oxygenation can be
maintained in patients with normal D l c o for more than this time interval. During apneic
oxygenation, arterial Pc o 2 rises 6 mm Hg in the first minute followed by 3 to 4 mm Hg every
subsequent minute.

28. A. Bronchopleural fistula refers to a communication between the bronchial and pleural
spaces. It presents as a sudden large air leak from the chest tube that may be associated with an
increasing pneumothorax and partial lung collapse. Inadequate surgical closure of the bronchial
stump usually presents itself with a bronchopleural fistula in the first 24 to 72 hours. Necrosis
of the suture line (bronchial/parenchymal) caused by ischemia or infection usually presents
after 72 hours. This is a rare complication, but small air leaks are fa irly common after
segmental or lobar resection due to collateral ventilation from small channels at the sites of
incomplete fissures. They are usually smaller in volume, w ill not cause significant impairment
of ventilation, and w ill close by itself after a few days— after which chest tubes can be
discontinued.

29. C. This is the classical picture of torsion of a lung lobe as it expands to f ill up the space left
by resection of the other lobe. This is because the torsion results in occlusion of the pulmonary
vein, which drains blood flo w to that part of the lung and presents clinically as hemoptysis and
radiographically as an enlarging homogenous density. This can be confirmed by visualizing a
closed lobar orifice on bronchoscopy. On the other hand, an acute herniation of the heart into
the operative hemithorax is associated with hemodynamic changes and a shift in the cardiac
shadow on chest X-ray. This is caused by a large pressure difference between the two
hemithoraces. Herniation to the right causes severe hypotension and an elevated central venous
pressure due to torsion of the vena cava. Herniation to the left causes compression of the heart
at the atrioventricular groove, resulting in hypotension, ischemia, and infarction.

30. D. With the given history, the massive hemoptysis (defined as >500-600 mL of blood loss in
24 hours) is most likely infectious in origin with tuberculosis being a strong possibility. The
other causes of massive hemoptysis include bronchiectasis, aspergillomas, neoplasms, foreign
body in the trachea, and trauma. A potentially lethal hemoptysis with severe hemodynamic
compromise necessitates emergency surgery. Bronchial artery embolization may be attempted
if the patient is hemodynamically stable. Whenever possible, surgery is carried out in a semi­
elective way, but the operative mortality is still high, >20%. However, medical management is
associated with a much higher mortality, >50%. The most common cause of death is asphyxia
secondary to blood in the airway.

31. C. Pulmonary cysts or bullae are large cavitary lesions that behave as if they have a one-way
valve, gets progressively large and may compress the remaining lung tissue. They may also
rupture producing a tension pneumothorax. They can be congenital or acquired as a result of
emphysema. They are usually scheduled for lung resection surgeries when they cause recurrent
pneumothorax or progressive dyspnea. Positive-pressure ventilation results in further
expansion of such cavities and increased risk of rupture along with impaired oxygenation from
the affected lung. Maintenance of spontaneous ventilation (negative inspiratory pressure) is
recommended until the affected lung is isolated using a DLT or until a chest tune is placed.
Inhalational agents can be used to facilitate this, but the large dead space caused by the presence
of huge cyst may result in progressive hypercarbia. Assisted ventilation is helpful in such
circumstances. Care should be taken to avoid complete positive-pressure ventilation.

32. C. Tracheal stenosis is narrowing of the airway as a result of tracheal mucosal damage
followed by scarring. It can also be caused by tumors— squamous or adenoid cystic carcinoma.
The inciting factors for the mucosal damage include trauma or prolonged endotracheal
intubation. These patients present with progressive dyspnea, hemoptysis, and stridor on
exertion. The dyspnea is characteristically worse on lying down and is made better by sitting up
and leaning forward.

33. A. Anesthetic considerations for tracheal resection include invasive monitoring, use of
anticholinergics to prevent increased secretions, slow inhalational induction maintaining
spontaneous ventilation, airway stimulation after attaining a deep plane of anesthesia, return of
spontaneous ventilation, and early extubation. The left radial artery is preferred for lower
tracheal resections because of the potential for compression of the innominate artery. A
nonirritating inhalational agent like sevoflurane in 100% oxygen can be used along with short­
acting opioids like remifentanil. Care should be taken to decrease the Fio 2 to below 0.3 if the
surgeon is using laser to resect the scar tissue. After opening the stenosed segment, the surgeon
can insert a sterile endotracheal tube into the segment of trachea below the lesion and patient
can be ventilated through that. There w ill be a brief period of apnea as the surgeon is
anastomosing the anterior part of trachea after resection. Once the anastomosis is complete, the
initial endotracheal tube can be readvanced below the lesion. The neck is kept flexed in the
postoperative period to minimize tension on the tracheal suture line. Heliox offers a method to
avoid turbulence due to its lower density. Flow-volume loops confirm the location of the
obstruction and aid the clinician in evaluating the severity of the lesion.

34. D. Mediastinoscopy involves operating on an area covered with blood vessels and nerves.
The complications include reflex bradycardia due to vagal stimulation, bleeding from damage
to the great vessels, pneumothorax, air embolism, post-op hoarseness due to recurrent
laryngeal nerve injury, and phrenic nerve injury. A false drop in blood pressure may be
observed due to compression of the innominate artery if the arterial line is placed on the right
arm. A spontaneously breathing patient with head end elevated is also at risk for a
pneumothorax that presents postoperatively.

35. D. Pulmonary alveolar proteinosis is a condition in which patients produce excessive


quantities of surfactant and fail to clear it, producing bilateral lung involvement and recurrent
pneumonias. Bronchoalveolar lavage is performed in these patients for severe hypoxemia or
worsening dyspnea. They undergo sequential lung lavages interspaced by a few days with the
worse lung getting lavage first. It is an absolute indication for lung isolation. If both lungs are
lavaged during the same procedure, it significantly impairs effective oxygenation.
Lung isolation for unilateral bronchoalveolar lavage is obtained by a double-lumen tube
under general anesthesia. Proper positioning of the tube by bronchoscopy is essential prior to
the lavage to prevent contamination of the opposite lung. A water-tight seal with the cuffs is also
essential prior to the lavage. The procedure is normally done in the supine position; lavaging a
dependent lung in a lateral position helps to minimize soiling of the nondependent lung, but the
ventilation-perfusion mismatch caused by ventilating a nondependent lung which is not
perfused is severe and makes this clinically impossible. Warm normal saline is infused into the
lung to be treated and is drained by gravity; treatment continues until the fluid returning is clear
(about 10-20 L). Patient can be extubated after carefully suctioning out both the lungs or the
double-lumen tube is replaced by a single-lumen tube at the end of the procedure.

36. B. Right-ventricular failure caused by increase in right-sided afterload (increased pulmonary


artery resistance) may recover after isolated lung transplantation, and they do not require
combined heart-lung transplantation. Such is not the case in patients with Eisenmenger
syndrome who require combined heart-lung transplantation. However, normal left-ventricular
function and absence of significant coronary artery disease or other serious health problems is
ensured before lung transplantation, as the wait list of patients for the organs are long.
Respiratory failure caused by cystic fibrosis, bullous emphysema, or vascular diseases are
usually bilateral and necessitate a double-lung transplant. It can be done using cardiopulmonary
bypass or sequentially using one-lung ventilation depending on the pulmonary artery pressures
and the ventricular function. Single-lung transplantation is being increasingly performed for
patients with chronic obstructive pulmonary disease. Organ selection is based on size, ABO
compatibility, and cytomegalovirus serology matching.

37. A. A newly transplanted lung lacks the neural innervation, lymphatic drainage, and bronchial
circulation, which were present in the explanted lung. Central respiratory pattern generated by
centers in the brain stem is unaffected. Hypoxic pulmonary vasoconstriction, mediated locally is
also unaffected. However, loss of lymphatic drainage increases extravascular lung water and
predisposes the transplanted lung to pulmonary edema. Fluid restriction is fa irly common after
lung transplantation to prevent this from happening. Although some patients develop bronchial
hyperreactivity, cough reflex is abolished below the carina. These patients usually get
postoperative bronchoscopy to assess bronchial suture line, as they are prone for ischemic
breakdown in the absence of bronchial circulation.

38. C. Anesthetic considerations in patients with esophageal disease include the risk of
pulmonary aspiration, use of a DLT, invasive monitoring, intravenous access sufficient for
rapid fluid resuscitation, maintaining normothermia, and use of transcutaneous pads for
defibrillation if needed. The esophageal disease process predisposes them to aspiration due to
obstruction, altered motility, or abnormal sphincter function. The risk of aspiration continues
into the postoperative period. Even though a DLT (Double lumen tube) facilitates surgical
exposure, it is not always required. Invasive monitoring with arterial line and central venous
pressure monitoring help guide hemodynamic management. However, a PAC (Pulmonary
artery catheter) is used only for patients with significant cardiac disease. Substernal and
diaphragmatic retractors used during the transhiatal approach to esophagectomy can interfere
with cardiac function. Surgeons hand can interfere with cardiac fillin g while bluntly dissecting
the esophagus from the posterior mediastinum. Since the vagus runs very close to the
esophagus, marked vagal stimulation can result in profound bradycardia or even cardiac arrest
— transcutaneous pads helps in these situations. Hypothermia increases coagulopathy and
increases cardiac arrhythmias and should be avoided. The potential for rapid massive blood
loss is significant as the surgery is near the major blood vessels.

39. B. There has been a recent resurgence in LVRS, even though NETT, a trial of usual medical
therapy versus usual medical therapy plus LVRS, suggested lack of efficacy of LVRS.
Anesthetic considerations for LVRS include watching out for pneumothorax caused by a
ruptured bleb, use of double-lumen tubes to allow selective ventilation and to facilitate surgery,
using a lower Fio 2 to a goal Spo2 of 90%, lim iting the degree of positive-pressure ventilation
(<30 cm H2O peak inspiratory pressure), prolonging the expiratory time, and early extubation.
Total IV anesthesia techniques using propofol and remifentanil or inhalational agents like
desflurane with short-acting neuromuscular blocking agents help facilitate early extubation. If
the patient cannot be extubated at the conclusion of the procedure, the double-lumen tube is
exchanged for a single-lumen tube to decrease airway resistance.

40. B. Equal pressure point refers to the point in the airway where intraluminal pressure and
extraluminal pressure (pleural) are the same. This is normally beyond the 11th to 13th
generation of bronchioles where cartilaginous support is absent. This is the point where
dynamic airway compression can occur— this refers to the phenomenon in which collapsible
membranous portion of the airway gets compressed by the extraluminal pressure generated by
a forced expiration. It is facilitated by the large pressure drop across the airways causing a
higher gradient between extra and intraluminal pressures. Obstructive airway diseases
predispose the patients to dynamic airway compression. Elastase deficiency in emphysema
causes decreased elastic support in smaller airways. The bronchoconstriction and inflammation
of asthma predisposes to reversal of transmural gradients. Such patients usually adapt by
pursed-lip breathing and terminating the expiration early before functional residual capacity
falls below closing capacity (auto PEEP). However, the increase in lung volume and slowing of
expiration caused by such a maneuver helps to stent the airway open. The increase in lung
volume increases the intraluminal pressure and dilates the airways, and the slow expiration
reduces the decrease in pressure from the alveoli to the mouth because lower driving pressures
are sufficient for lesser flows. This shifts the equal pressure point to the noncollapsible larger
airways or to the mouth.

41. C. Any lesion causing a compression of the superior vena cava (SVC) and impedes blood
return from head and neck can cause venous engorgement and edema of the head, neck, and
arms. This is usually produced by a mediastinal tumor causing compression of the mediastinal
structures including the SVC. It can also be caused by an occlusive thrombus in the SVC.
Among the mediastinal neoplasms, lymphomas are the most common causes for SVC
syndrome. But other mediastinal tumors like germ-cell tumors or pulmonary lesions with
secondary lymphadenopathy may also be responsible. These cases are very difficult as
induction of anesthesia in a supine position causes severe airway obstruction and
cardiovascular collapse. The airway obstruction is due to direct mechanical compression as
well as mucosal edema. Attempts should be made to decrease the size of the mass and the degree
of mediastinal compression should be made prior to elective surgery. This includes radiation
therapy, chemotherapy, and steroids. An empiric treatment with steroids may be attempted prior
to a tissue diagnosis in this 12-year old. A preoperative echocardiogram can quantify the degree
of compromise in cardiac function, presence of a thrombus, or dynamic inflo w obstruction in
the presence of pericardial fluid. A CT scan/MRI w ill help diagnose the presence of
tracheomalacia/erosion and the level of the lesion. If induction of general anesthesia is required
in the presence of SVC syndrome, awake fiberoptic intubation is the preferred method and
inhalational anesthetics can be used to attain a deep plane of anesthesia in a spontaneously
breathing patient after intubation. A rig id bronchoscope and ability to go on cardiopulmonary
bypass are other precautionary measures that can be taken. <CT>
N euroanesthesia
Dipty Mangla and Ashish Sinha

1. Total normal cerebral blood flo w (CBF) is


A. 25 mL/100 g/min
B. 50 mL/100 g/min
C. 100 mL/100 g/min
D. 150 mL/100 g/min

2. The factor associated with maximum increase in intracranial pressure (ICP) is


A. Increased central venous pressure to 14 mm Hg
B. Hyper car bia with Paco2 of 50 mm Hg
C. Ventilation with positive end-expiratory pressure (PEEP) of 5 cm H2O
D. Bucking and coughing on endotracheal tube

3. Cerebral perfusion pressure (CPP) (mm Hg) in a patient with intracranial pressure (ICP) of 12
mm Hg, central venous pressure (CVP) of 15 mm Hg, and mean arterial pressure (MAP) of 70
mm Hg w ill be
A. 58
B. 55
C. 52
D. 48

4. Treatment of a patient with mannitol can lead to all the follow ing, except
A. Oliguria
B. Hypotension
C. Hypervolemia
D. Hypokalemia

5. A patient is undergoing craniotomy for subdural hematoma. During the procedure, the surgeon
requests lowering the intracranial pressure. A ll the follow ing can be used, except
A. Mannitol
B. Hyperventilation
C. Steroids
D. Furosemide

6. The desired level of Paco2 in a neurosurgical patient is

A. 30 to 35 mm Hg
B. 25 to 30 mm Hg
C. 20 to 25 mm Hg
D. 15 to 25 mm Hg

7. An absolute contraindication for electroconvulsive therapy (ECT) is


A. Hypertension
B. Pheochromocytoma
C. Aortic aneurysm
D. Stroke

8. Signs of air embolism in a patient include all, except


A. Hypertension
B. Heart murmur
C. Arrhythmia
D. Decreased EtCO2

9. A 65-year-old male is undergoing surgery for medulloblastoma in the posterior fossa of brain.
Approximately 1 hour into surgery you notice arrhythmias on the monitors. The next step w ill
be
A. Inform the surgeon
B. Give P-blockers
C. Administer lidocaine
D. Give 100% oxygen

10. Nitrous oxide should be avoided in patients with


A. Subdural hematoma
B. Brain tumor
C. Closed head injury
D. Pneumocephalus

11. The follow ing fluid should be avoided in a patient undergoing craniotomy
A. Lactated Ringerés
B. Normal saline
C. Dextrose 5%— normal saline
D. Hetastarch
12. Most sensitive method to detect air embolism is
A. Transesophageal echocardiogram (TEE)
B. Decreased end-tidal carbon dioxide
C. Increased end-tidal nitrogen
D. M ill wheel murmur

13. Best measure to reduce cerebral oxygen consumption includes


A. Administration of barbiturates
B. Hyperventilation
C. Administration of opioids
D. Institution of hypothermia

14. A ll of the follow ing decrease cerebral blood flo w (CBF), except
A. Etomidate
B. Propofol
C. Thiopental
D. Ketamine

15. In a patient undergoing craniotomy, the transducer of arterial line should be zeroed at the
A. Level of hypothalamus
B. Level of heart
C. Level of external auditory meatus
D. Level of atmosphere

16. Jugular venous oxygen saturation


A. Estimates oxygen extraction
B. Is unaffected by systemic hypoxia
C. Involves placement of catheter through inferior vena cava
D. Monitors global oxygenation of both cerebral hemispheres

17. The effect of ischemia on somatosensory-evoked potentials (SSEPs) is


A. Decreased latency, decreased amplitude
B. Increased latency, increased amplitude
C. Decreased latency, increased amplitude
D. Increased latency, decreased amplitude

18. A patient with spinal injury, sustained 3 hours ago, comes to the OR for exploratory
laparotomy. Anesthetic management of the patient includes which of the following?
A. Rapid-sequence induction with succinylcholine
B. Hypothermia for better neurologic outcome
C. Managing autonomic hyperreflexia
D. Avoiding corticosteroids

19. The electrophysiological monitor most resistant to anesthetic agents is


A. Somatosensory-evoked potentials
B. Motor-evoked potentials
C. Brain-stem auditory-evoked potentials
D. Electroencephalography

20. The most reliable monitor for neurologic monitoring in a patient undergoing carotid
endarterectomy is
A. Electroencephalogram
B. Jugular venous oxygen saturation
C. Awake neurologic exam
D. Stump pressure

21. Anesthetic management of a patient with multiple sclerosis (MS) includes


A. Avoiding hypothermia
B. Avoiding hyperthermia
C. Spinal anesthesia is safe
D. Use of succinylcholine can result in hypokalemia

22. A ll the follow ing are true for Guillain-Barre syndrome (GBS), except
A. Respiratory paralysis is frequent complication
B. Presence of labile autonomic nervous system
C. Ascending motor paralysis
D. Exaggerated reflexes

23. True statement about autonomic hyperreflexia is


A. Lesions below T10 is responsible for the reflex
B. It can be treated with deep general anesthetic
C. It is associated with vasoconstriction above the site of injury
D. It can be provoked by thermal stimulation

24. A 16-year-old patient with acute lysergic acid diethylamide (LSD) intoxication and head injury
comes to emergency room. A ll the follow ing can be used in anesthetic management, except
A. Propofol
B. Succinylcholine
C. Ketamine
D. Phenylephrine
25. A 25-year-old patient with severe depression is undergoing an electroconvulsive therapy
(ECT). The duration of seizure can be increased by
A. Hypoventilating the patient
B. Hyperventilating the patient
C. Administering succinylcholine
D. Administering rocuronium

26. A ll of the follow ing are contraindications of electroconvulsive therapy (ECT), except
A. Pacemaker
B. Recent stroke
C. Raised intracranial pressure
D. Severe osteoporosis

27. True statement regarding cerebral physiology is


A. Normal cerebral metabolic oxygen consumption is 5 mL/100g/min
B. Normal Intracranial pressure (ICP) is approximately 15 mm Hg
C. Normal cerebral blood flo w (CBF) is 50 mL/100g/min
D. Cerebral autoregulation is strictly maintained at blood pressures between 60 and 150 mm
Hg in all patients

28. True statement about cerebrospinal fluid (CSF) is


A. It is formed in the third ventricle
B. It is absorbed in arachnoid granulations present in fourth ventricle
C. Total volume of CSF is about 150 mL
D. Major mechanism of formation is by passive diffusion of ions

29. A precordial Doppler can detect a minimal o f ___mL of intracardiac air:


A. 0.1
B. 0.25
C. 0.5
D. 1

30. The only inhalational anesthetic that can cause an isoelectric EEG among the follow ing is
A. Isoflurane
B. Halothane
C. Enflurane
D. Nitrous oxide

31. Intraoperative anesthetic management of a patient undergoing cerebral aneurysm repair


includes all, except
A. Maintenance of hypotension
B. Mannitol for facilitating surgical exposure
C. Maintaining m ild hypothermia
D. Patient remaining intubated for 24 hours postoperatively

32. Which of the follow ing types of neuromonitoring can be done in a patient undergoing
transsphenoidal resection of a pituitary tumor?
A. EEG
B. Motor-evoked potentials
C. Visual-evoked potentials
D. Auditory-evoked potentials

33. The drug of choice for treating nausea and vomiting in a patient with parkinsonism would be
A. Ondansetron
B. Promethazine
C. Droperidol
D. Metoclopramide

34. A ll the follow ing anesthetic agents can cause seizurelike activity on the electroencephalogram
(EEG), except
A. Ketamine
B. Etomidate
C. Enflurane
D. Thiopental

35. The neuromuscular blocking agent relatively contraindicated in a patient with raised
intracranial pressure (ICP) is
A. Rocuronium
B. Vecuronium
C. Atracurium
D. Cisatracurium

36. The afferent input for somatosensory-evoked potentials is carried by which spinal cord tract
A. Corticospinal
B. Dorsal columns
C. Spinothalamic
D. Spinocerebellar

37. You are called to evaluate a 50-year-old patient for brain death. A ll the follow ing are criteria
for brain death, except
A. Apnea for 10 minutes
B. Absence of corneal reflex
C. Presence of spinal reflexes
D. Decerebrate posturing

38. A 30-year-old male is found unresponsive outside a supermarket. The emergency response
team finds him in ventricular fibrillation. After 10 minutes of CPR, the emergency response
team is successful in reviving the patient. In the emergency room, it is decided to cool the
patient to 34°C from 37°C. By this measure, the cerebral metabolic demand w ill decrease by
A. 12%
B. 18%
C. 24%
D. 30%

39. A ll the follow ing are relative contraindications to a sitting craniotomy, except
A. Right-to-left cardiac shunt
B. Patent foramen ovale
C. Ventriculoatrial shunt
D. Ventriculoperitoneal shunt

40. An 80-year-old female comes to the ER with closed distal radial fracture. On further
questioning, she gives a history of stroke about 2 weeks ago. How long should one wait before
it can be assumed that her risk of perioperative stroke is same as a healthy 80-year-old?
A. 6 days
B. 6 weeks
C. 6 months
D. 6 years

41. A 28-year-old male is being treated in the ICU for raised intracranial pressure (ICP). A ll the
follow ing measures can aid in decreasing ICP quickly , except
A. Corticosteroids
B. Hyperventilation to Paco2 of 30 mm Hg
C. Mannitol
D. Head elevation to 30 degrees

42. Which of the follow ing agents w ill have the least effect on somatosensory-evoked potentials
(SSEPs)?
A. Vecuronium
B. Propofol
C. Fentanyl
D. Nitrous oxide
43. Signs and symptoms of raised intracranial hypertension include all the following, except
A. Hypertension
B. Tachycardia
C. Bradycardia
D. Irregular respiration

44. Etomidate in a dose of 0.2 mg/kg can lead to all the following, except
A. Abolish ventilatory response to carbon dioxide
B. Increase amplitude and latency of somatosensory-evoked potentials (SSEPs)
C. Decrease cerebral metabolic oxygen demand
D. Decrease cerebral blood flo w (CBF)

45. The most important factor governing cerebral blood flo w (CBF) is
A. Cerebral metabolic oxygen demand
B. Paco2
C. pH
D. Cerebral perfusion pressure

46. The follow ing graph depicts the relationship between cerebral perfusion and

A. Paco2
B. Pao2
C. Mean arterial pressure
D. Cerebrospinal fluid pH

47. The follow ing graph depicts the relationship between cerebral perfusion and
A. Paco2
B. Pao2
C. Mean arterial pressure
D. Cerebrospinal fluid pH

48. A 45-year-old male is seen in the preadmission testing for pituitary adenoma resection surgery.
A ll the follow ing would be expected if this adenoma was causing acromegaly, except
A. Hypotension
B. Obstructive sleep apnea
C. D ifficult airway
D. Hyperglycemia

49. The fastest measure to decrease intracranial pressure (ICP) in a patient is


A. Mannitol
B. Dexamethasone
C. Furosemide
D. Hyperventilation

50. Therapy for cerebral vasospasm includes


A. Hypertension, hypervolemia, hemodilution
B. Normotension, euvolemia, hypocarbia
C. Hypotension, hypovolemia, hypocarbia
D. Hypertension, hypervolemia, hypocarbia
CHAPTER 13 ANSWERS

1. B. Normal total CBF is about 50 mL/100 g/min. CBF below 20 mL/100 g/min is associated
with cerebral ischemia. CBF is modulated by various factors, which include Paco2, Pao2, blood
pressure, intracranial pressure, etc.

2. D. Intracranial pressure is supratentorial CSF pressure measured in the lateral ventricles or


cerebral cortex. Normal ICP is 10 mm Hg or less. Between Paco2 values of 20 and 80 mm Hg,
CBF increases by 1 mL/100 g/min and cerebral blood volume increases by 0.05 mL/110g/min
per mm Hg increase in Paco2. Increase in CVP and adding PEEP w ill m inim ally increase ICP by
affecting venous return. Coughing and bucking can cause a much higher increase in ICP (acute
increase) than any of the above factors.

3. B. CPP = MAP - ICP or CVP, whichever is higher.


Thus, CPP = 70 - 15 = 55 mm Hg.

4. A. Mannitol, a six-carbon sugar, is the most commonly used diuretic in neuroanesthesia


practice. It is an osmotic diuretic and undergoes little or no reabsorption. It also improves renal
blood flow. Side effects include an initial increase in circulatory volume, which can cause
pulmonary edema. Diuresis attributed to mannitol can lead to hypovolemia and hypokalemia.

5. C. Treatment of intracranial hypertension includes hyperventilation to PaCO2 of 25 to 30 mm


Hg, improving CSF drainage by elevating the head by 30 degrees or surgical placement of CSF
drain, using an osmotic diuretic (mannitol), hypertonic saline, decompression craniectomy,
barbiturates, and corticosteroids. The latter have been used to decrease cerebral edema, and take
a few hours to have effect, but routine use of corticosteroids in managing intracranial
hypertension is not recommended.

6. B. PaCO2 is the most potent physiologic determinant of cerebral blood flow. Maximal
reductions in ICP can be achieved by decreasing PaCO2 to 25 to 28 mm Hg, and the reduction in
ICP lasts up to 24 to 36 hours.

7. B. ECT is commonly used for treatment of refractory major depression. It involves using
electricity to shock one or both cerebral hemispheres to induce a seizure lasting 30 to 60
seconds. Contraindications to ECT include pheochromocytoma, recent myocardial infarction
(<3 months), recent stroke (<1 month), intracranial mass or increased ICP, angina, poorly
controlled heart failure, significant pulmonary disease, bone fractures, severe osteoporosis,
pregnancy, glaucoma, and retinal detachment.

8. A. Clinical signs of venous air embolism include a decrease in end-tidal CO2, a decrease in
arterial oxygen saturation, sudden hypotension, m ill wheel murmur, and even sudden
circulatory arrest. Presence of a patent foramen ovale, which has an incidence of 20% in adults,
can lead to paradoxical air embolism, with the potential of causing coronary ischemia or a
stroke.

9. A. For posterior fossa tumor resection, the patient is frequently placed in the sitting or prone
position. Monitoring of the patient includes arterial blood pressure line, a central venous
catheter (for access, pressure monitoring, aspiration of any air— if required), and a precordial
Doppler to detect intracardiac air (venous air embolism). Operations on posterior fossa tumors
can injure vital brain-stem respiratory and circulatory nuclei, resulting in hemodynamic
fluctuations or depression of ventilation. The surgeon should be informed at the first sign of
cardiac arrhythmias.

10. D. Nitrous oxide can diffuse into closed air spaces, which may be of significant clinical
consequences. The blood/gas coefficient of nitrous oxide is 0.47, whereas that of nitrogen is
0.015. This means that nitrous oxide is about 33 times more diffusible than nitrogen. As a result,
at any given partial pressure, far more nitrous oxide can be carried into a closed gas space than
nitrogen removed. Thus, nitrous oxide can quickly expand closed gas spaces, such as middle
ear or a pneumothorax.

11. C. In a patient undergoing craniotomy, intravenous fluid replacement should be performed by


using glucose-free isotonic crystalloid or colloid solutions. Hyperglycemia is known to worsen
ischemic brain injury.

12. A. The most sensitive intraoperative monitor for detecting venous air embolism is TEE. The
second best monitor is precordial Doppler sonography, which can detect as little as 0.25 mL of
air. Changes in end-tidal respiratory gas concentrations, such as nitrogen and carbon dioxide,
and changes in pulmonary artery pressures are less sensitive. Hypotension and m ill wheel
murmur are late manifestations of venous air embolism.

13. D. Hypothermia is one of the most effective methods for protecting the brain against ischemia.
Hypothermia decreases both basal and electrical metabolic requirements throughout the brain,
unlike intravenous anesthetic agents or hyperventilation.

14. D. Propofol, barbiturates, and etomidate produce dose-dependent decreases in cerebral


metabolic rate and CBF. Ketamine is the only induction agent that dilates the cerebral
vasculature and thus increases CBF (50% to 60%).

15. C. In a seated patient, the arterial pressure in the brain differs significantly from left
ventricular pressure. Cerebral perfusion pressure is determined by setting the transducer to zero
at the level of the ear, which approximates the circle of W illis.

16. D. Jugular venous bulb oximetry involves placing a sampling catheter in the internal jugular
vein (IJV). The normal range for mixed venous oxygen saturation at IJV is 50% to 75%. It gives
an estimate of balance between oxygen supply and demand of the brain, and measures global
cerebral oxygenation (not focal).

17. D. SSEPs reflect the integrity of neuronal pathway from the peripheral nerves through the
spinal cord (dorsal columns) to the brain. SSEPs are electrical manifestations of the central
nervous system response to external stimulation. Intraoperative changes in amplitude or latency
or complete loss of waveforms are indicators of compromised sensory pathway integrity. SSEP
amplitude loss greater than 50% or a latency increase greater than 10% is considered
significant.

18. A. In the early management of acute spinal injury patients, particular emphasis should be
placed on preventing further spinal damage, which may occur during patient movement, airway
manipulation, and positioning. High-dose corticosteroids are often administered to help
improve neurological outcome. The head and neck should be stabilized using manual inline
stabilization, and awake fiberoptic intubation should be considered in high cervical injuries.
Patients with high cord transections may have impaired airway reflexes, hypotension, and
bradycardia and may be prone to hypothermia in view of generalized vasodilation.
Succinylcholine can be used safely in first 24 hours follow ing spinal injury.

19. C. Somatosensory- and motor-evoked potential monitoring is commonly used to detect


ischemia of spinal cord in spine surgeries. Brain-stem auditory-evoked responses monitor
ischemia during posterior fossa surgeries. Inhalational agents in general increase the latency
and decrease the amplitude of evoked potentials (if used at more than 0.5-0.75 MAC). The effect
of inhalational anesthetics on evoked potentials in decreasing order is visual > motor >
somatosensory > brain-stem auditory.

20. C. Awake neurological status is the most reliable method to detect cerebral ischemia. In
patients undergoing carotid endarterectomy under local anesthesia and m ild sedation, global
and focal neurological status can be continuously assessed. In patients undergoing carotid
endarterectomy under general anesthetic indirect methods to detect cerebral ischemia can be
used. These include EEG monitoring, transcranial Doppler, arteriography, and measurement of
blood flo w using xenon.

21. B. MS is characterized by progressive demyelination in the brain and spinal cord. Stress,
anesthesia, and surgery can have detrimental effects on the course of the disease. Elective
surgery should be avoided in acute relapse of MS. Regarding the effect of anesthetic technique
on MS, spinal anesthesia can exacerbate MS symptoms, epidural anesthesia usually does not
affect MS, succinylcholine should be avoided to prevent hyperkalemia, and hyperthermia
should be avoided as an increase in temperature may block nerve conduction. Advanced MS
may be associated with autonomic dysfunction.

22. D. GBS affects about 2/100,000 people. It is characterized by a sudden onset ascending motor
paralysis, areflexia, and paresthesias. Bulbar involvement with respiratory failure is a frequent
complication. Succinylcholine should be avoided in these patients, as it can cause hyperkalemia.
Regional anesthesia may make GBS worse. Anesthetic management may be complicated by
liab ility of the autonomic nervous system (hypotension or hypertension).

23. B. Autonomic hyperreflexia is seen in patients with spinal cord injury at or above T6. It is
characterized by acute generalized sympathetic hyperactivity in response to a triggering
stimulus. The triggering stimulus can be any stimulus occurring below the level of the lesion,
and is most commonly a distension of hollow viscera (bowel or bladder). Clinical signs include
severe hypertension, bradycardia, arrhythmias, profuse sweating, vasodilation above the level
of lesion, and pallor and vasoconstriction below the level of lesion. Antihypertensives may have
to be utilized to treat the hypertension. Spinal anesthesia (not preferred because of technical
difficulty and unpredictable level) or deep general anesthesia has been used in preventing
autonomic hyperreflexia.

24. C. LSD is a hallucinogen and causes CNS excitation, sensory distortion, delusions,
hallucinations, and euphoria. Autonomic effects, mediated via the hypothalamus, include
tachycardia, hypertension, mydriasis, piloerection, salivation, lacrimation, and vomiting. In
view of hypertension and tachycardia that can be caused by LSD, ketamine should be avoided.

25. B. Propofol when used for induction in patients undergoing ECT can increase the seizure
threshold and decrease the duration of the seizure. Hyperventilation and administration of
caffeine or etomidate can increase seizure duration. Muscle relaxants do not affect the threshold
or duration of the seizure.

26. A. Contraindications to ECT include recent myocardial infarction (<3 months), a recent stroke
(<1 month), an intracranial mass and raised intracranial pressure, angina, poorly controlled
congestive heart failure, significant pulmonary disease, bone fractures, severe osteoporosis,
pregnancy, glaucoma, and retinal detachment.

27. C. The cerebral metabolic rate is reflected by oxygen consumption, which is about 3 to 3.8
mL/100 g/min. Total CBF averages 50 mL/100 g/min. In normal individuals, CBF remains
nearly constant between mean arterial pressures of about 60 and 160 mm Hg. The cerebral
autoregulation curve is shifted to right in patients with chronic arterial hypertension. ICP by
convention means supratentorial CSF pressure measured in the lateral ventricles or over the
cerebral cortex, and the normal CSF pressure is 10 mm Hg or less.

28. C. CSF is formed by the choroid plexuses of cerebral lateral ventricles. In adults, normal CSF
production is about 20 mL/hour with a total volume of 150 mL. The CSF is absorbed in
arachnoid granulations over cerebral hemispheres. CSF formation involves active secretion of
sodium in the choroid plexuses, and not passive diffusion.

29. B. A precordial Doppler can detect as little as 0.25 mL of intracardiac air. A precordial
Doppler is the next best sensitive indicator to detect intracardiac air after a transesophageal
echocardiogram.

30. A. Isoflurane can produce an isoelectric EEG at 2 to 2.5 MAC, while enflurane typically
produces a spike and wave pattern at 2 to 3 MAC. Seizure activity may be seen on EEG with 3%
enflurane in a hypocapnic patient. Halothane causes slowing of EEG activity with increasing
concentration until 4 MAC, after which it produces uniform activity. Increasing sevoflurane
concentration from 2 to 5 MAC changes the cortical EEG pattern from a high-amplitude slow
wave to burst suppression to an isoelectric EEG interspersed with spikes.

31. D. Intraoperative management of cerebral aneurysms should include availability of blood,


avoidance of hypertension during induction, central venous pressure and arterial blood
pressure monitoring, mannitol after the dura is opened to help surgical exposure, elective
hypotension as it decreases transmural pressure across the aneurysm (avoiding rupture),
administration of thiopental and m ild hypothermia for cerebral protection, and awake
extubation depending on neurological status.

32. C. The transsphenoidal or bifrontal craniotomy approach may be used to gain access to
pituitary gland. The former (transsphenoidal approach) has several advantages including
elimination of frontal lobe retraction, microsurgical removal of small adenomas, reduced
blood loss, and shorter hospital stay. Patients are intubated endotracheally (oral), and
oropharyngeal packing is done to prevent bleeding into the esophagus. Additionally,
epinephrine or cocaine may be injected submucosally to reduce bleeding. The cavernous sinus
forms the lateral border of the sella turcica and includes the internal carotid artery, venous
structures, and cranial nerves III, IV, V, and VI. Therefore, visual-evoked potentials may be
monitored in the OR for early detection of visual pathway damage.

33. A. Parkinson disease is a movement disorder that affects individuals 50 to 70 years of age. It is
caused by progressive loss of dopamine in the nigrostriatum. Patients have bradykinesia,
postural instability, rigidity, facial masking, and a resting p ill-ro llin g tremor. Antidopaminergic
activity associated with butyrophenones, phenothiazines, and metoclopramide can worsen
symptoms and thus these should be avoided.

34. D. Ketamine, etomidate, and enflurane can cause seizurelike activity on the EEG. Thiopental
increases the threshold and decreases the duration of seizure activity.

35. C. In a patient with increased intracranial pressure, a nondepolarizing muscle relaxant is


commonly used to facilitate controlled ventilation and tracheal intubation. Rocuronium and
vecuronium are commonly used as they provide the greatest hemodynamic stability.
Succinylcholine and atracurium (due to associated histamine release) may increase ICP,
particularly if intubation is attempted before deep general anesthesia. Hyperventilation prior to
intubation is utilized to decrease the ICP.

36. B. Somatosensory-evoked potentials are transmitted through the follow ing pathway:
peripheral stimulus peripheral nerve dorsal root ganglia first-order fibers in the ipsilateral
posterior column to dorsal column nuclei second-order fibers crossing to the opposite side
medial lemniscus to the thalamus third-order fibers continuing to the frontoparietal sensory-motor
cortex.

37. D. Brain death is irreversible cessation of all brain activity. Generally accepted clinical
criteria for brain death include presence of coma, absence of motor activity, absence of brain­
stem reflexes (papillary, corneal, vestibule-ocular, and gag/cough), absence of ventilatory
effort (Paco2 >60 mm Hg), exclusion of hypothermia or effect of sedatives, isoelectric EEG,
and absence of cerebral perfusion by angiography.

38. B. Cerebral metabolic rate decreases by 6% per degree Celsius decrease in body temperature
below 37°C. Hence, a 3°C drop in temperature w ill decrease the cerebral metabolic rate by
18%.

39. D. The incidence of venous air embolism in sitting craniotomies is about 20% to 40%. The
presence of right-to-left shunt can cause paradoxical air embolism. A ir embolism can have
catastrophic consequences, such as coronary ischemia and stroke. Thus, sitting position should
be avoided in patients with a right-to-left shunt, patent foramen ovale, or ventriculoatrial shunt.

40. B. Regional blood flo w and metabolic rate are normal after 2 weeks follow ing a stroke.
Alterations in CO2 responsiveness and blood-brain barrier abnormalities require more than 4
weeks to be corrected. Thus, most clinicians postpone elective surgery for at least 6 weeks
follow ing stroke.

41. A. Definitive treatment of intracranial hypertension is ideally directed at the underlying cause.
Treatment modalities include fluid restriction, head elevation, osmotic agents and loop
diuretics, moderate hyperventilation (up to 24-36 hours), avoidance of hypotension, hypoxia
and hypercarbia, and corticosteroids. The latter is used to decrease cerebral edema in patients
with known intracranial tumors, and take a few hours to take effect.

42. A. Inhalational volatile anesthetics produce an increase in latency and decrease in amplitude of
evoked potentials. Nitrous oxide produces a decrease in amplitude with no change in latency.
Propofol decreases amplitude and an increase in latency of SSEPs. Muscle relaxants have no
effect on SSEPs. Narcotics cause dose-dependent decrease in amplitude and increase in latency.

43. B. Increased intracranial pressure (ICP) can lead to altered mental status, intractable vomiting,
and focal or global neurological deficits. Clinical signs include hypertension, bradycardia,
irregular respiration, and pupillary changes (papilledema may be seen on fundoscopy). Cushing
triad consists of raised ICP, hypertension, and bradycardia.

44. A. Etomidate decreases cerebral metabolic rate, CBF, leading to a decrease in intracranial
pressure. It enhances SSEP. It is a sedative hypnotic but lacks analgesic properties. Ventilation is
affected to a lesser extent with etomidate when compared to barbiturates or benzodiazepines.
Induction doses usually do not result in apnea.

45. A. Increased metabolic activity leads to an increase in CBF. Regional CBF parallels metabolic
activity and can vary from 10 to 300 mL/100 g/min. For example, motor activity of a lim b is
associated with a rapid increase in regional blood flo w of the corresponding motor cortex.

46. A. CO2 gas tension has the greatest influence on cerebral blood flo w (CBF). Between a Paco2
of 20 and 80 mm Hg, CBF changes approximately 1 to 2 mL/100 g/min per mm Hg in Paco2.

47. B. Marked changes in Pao2 affect cerebral blood flo w (CBF), although minimally. Hyperoxia
is associated with only minimal decreases in CBF. On the other hand, hypoxemia (Pao2<50 mm
Hg) greatly increases CBF.

48. A. The acromegalic patient suffers from general overgrowth of skeletal, soft, and connective
tissues. This results in coarse facial features and enlarged hands and feet. Patients may also have
a difficult airway because of overgrowth of soft tissues of upper airway, enlargement of tongue
and epiglottis, overgrowth of mandible with increased distance from lips to vocal cords, and
glottic and subglottic narrowing. These changes may also lead to obstructive sleep apnea.
Patients also are prone to hyperglycemia, hypertension, congestive heart failure, increased lung
volumes, increased ventilation-perfusion mismatch, peripheral neuropathy, skeletal muscle
weakness, osteoarthritis, and osteoporosis.

49. D. The quickest way to reduce ICP in a patient is hyperventilation, often to a PaCO2 of 25 mm
Hg. Reduced PaCO2 (hypocarbia) causes cerebral vasoconstriction leading to a reduction in
cerebral blood flo w and cerebral blood volume. However, hyperventilation is only used as a
temporizing measure only in periods of acute raised ICP.

50. A. Cerebral vasospasm occurs in about one-third of patients surviving the initial aneurysmal
rupture, and carries a high degree of m orbidity and mortality. The degree of vasospasm
depends on the degree of initial subarachnoid hemorrhage. Vasospasm usually develops 3 to 14
days postsubarachnoid hemorrhage results in narrowing of cerebral blood vessels and
decreased blood flo w distally. This may lead to an ischemic deficit and cerebral infarction, if
left untreated. Therapies for cerebral vasospasm include “ triple-H therapy”
(hypertension/hypervolemia/hemodilution), balloon angioplasty, and intra-arterial nicardipine
and other vasodilators.<CT>
Gastrointestinal, Liver, and Renal Diseases
Thoha Pham

1. A 38-year-old woman with a history of diverticulosis is scheduled for an exploratory


laparotomy for lysis of adhesions. Which of the follow ing is the best way of maintaining core
body temperature during the initial hour of general endotracheal anesthetic?
A. Providing warm and humidified inspired gases
B. Increasing ambient temperature
C. Administration of warm intravenous fluids
D. Use of warm irrigating fluids

2. Each of the follow ing would be expected in an otherwise-healthy 125-kg (BM I 40 kg/m2) man
undergoing open cholecystectomy, except
A. Decreased functional residual capacity
B. Increased intra-abdominal pressure and risk of reflux
C. Increased metabolism of volatile anesthetics
D. Decreased metabolism of atracurium

3. Which of the follow ing has a dual effect of increasing gastric pH, and decreasing the gastric
volume to minimize risks associated with aspiration?
A. Prochlorperazine
B. Ranitidine
C. Ondansetron
D. Metoclopramide

4. This finding is indicative of microatelectasis on the second postoperative day after major
abdominal surgery:
A. Hypercarbia
B. Hypoxemia
C. Diffuse wheezing
D. Tactile fremitus

5. A m orbidly obese 60-year-old man with a 65-pack year history of tobacco smoking is awake
after an uncomplicated general anesthetic with sevoflurane for routine endoscopy and
colonoscopy screening. After 45 minutes in the recovery room (PACU), while breathing 6
L/min of oxygen via nasal cannula, his pulse oximetry drops to 88%. His rest of the vital signs
are stable, and the lungs are clear to auscultation. The most effective management at this point
is
A. Coughing with deep breathing
B. Reintubation of the trachea
C. Intravenous administration of doxapram
D. Continuous positive-airway pressure

6. During rapid-sequence induction of anesthesia for emergent laparotomy to explore multiple


stab wounds, a 45-year-old man vomits a large quantity of undigested food particles. During
intubation of the trachea, food particles are noted near the cords. After instituting ventilation
with 100% oxygen, the most appropriate next step in this patient’s management is
A. Place patient in Trendelenburg position
B. Ventilate with positive end-expiratory pressure of 15 cm H2O
C. Administer corticosteroids
D. Administer antibiotics

7. A 71-year-old female develops a severe case of diarrhea with multiple loose bowel movements
since awakening this morning. When she arrives preoperatively for her surgery, an arterial
blood gas (ABG) is obtained. The most likely finding would be
A. pH = 7.30, Paco2 = 50, Pao2 = 60, HCO3 - = 24
B. pH = 7.35, Paco2 = 32, Pao2 = 85, HCO3 - = 18
C. pH = 7.45, Paco2 = 30, Pao2 = 80, HCO3 - = 28
D. pH = 7.40, Paco2 = 45, Pao2 = 85, H C o3 - = 15

8. A 65-year-old patient is noted to have excessive bleeding during a colectomy with an activated
clotting time (ACT) of 200 seconds. The most unlikely reason for this oozing is
A. Undiagnosed factor V II deficiency
B. Prior administration of heparin 5,000 U subcutaneously
C. Preoperative ingestion of aspirin and ibuprofen
D. Dilutional thrombocytopenia

9. During laparotomy, a patient has required infusion of 4 L of lactated Ringer’s and 4 U of


packed red blood cells (pRBCs). As the fifth unit of pRBCs begins infusing, patient has sudden
onset of tachycardia and hypotension. Within a few minutes, Foley bag reveals dark urine. The
most likely cause of unexplained oozing is
A. Hemolytic transfusion reaction
B. Leukoagglutinin reaction
C. Dilutional thrombocytopenia
D. Dilutional coagulopathy
Questions 10 to 12
A 26-year-old male patient with a history of severe ulcerative colitis, unresponsive to conservative
measures, presents for elective open total abdominal colectomy with end ileostomy. He has been
unable to eat for the last 2 weeks and was started on total parenteral nutrition (TPN) several days
prior.

10. Intraoperative effect that should be expected and monitored for is


A. Dilutional anemia
B. Hyperglycemia
C. Sepsis
D. Hyperphosphatemia

11. At the conclusion of the surgery, the patient fails to regain consciousness. The metabolic
complication of TPN (Table 14-1) that is likely is
Table 14-1 Metabolic Complications of TPN.
Glucose (hypoglycemia, hyperosmolar nonketotic coma)
Protein (hyperammonemia)
Hypercalcemia
Hypophosphatemia
Essential fatty acid deficiency
Vitamin toxicity

A. Azotemia
B. Hyperkalemia
C. Hyperosmolar ketotic hyperglycemia
D. Hyperosmolar nonketotic hyperglycemia

12. Consider that the patient opens his eyes and is extubated in the operating room. However, 15
minutes after arriving to the recovery room (PACU) he is unable to maintain adequate
ventilation and oxygenation. Physical exam reveals profound global weakness with absent
reflexes. The specific electrolyte abnormality that should be evaluated considering his TPN
requirement is
A. Potassium
B. Phosphate
C. Sodium
D. Glucose

13. Each of the follow ing statements about the preoperative management of an adrenal
pheochromocytoma is true, except
A. Adequate blockade can be assessed by in-house blood pressures <160/90 mm Hg for 24
hours prior to surgery
B. ^-Blockers should be administered only in conjunction with adequate a-blockade
C. Administration of a-blocker can decrease operative mortality
D. Nasal congestion is a sign of inadequate a-adrenergic block
14. A 40-year-old man undergoing an open resection of a pheochromocytoma under isoflurane
general endotracheal anesthesia suddenly develops tachycardia, hypertension, and
multifactorial ventricular ectopy. Each of the follow ing could be considered an appropriate
treatment option, except
A. Switching from isoflurane to sevoflurane
B. Intravenous vasodilator
C. Intravenous a-blocker
D. Intravenous lidocaine

15. An otherwise-healthy 38-year-old female patient is undergoing repair of a large ventral hernia
under intrathecal anesthesia. A T2 sensory level is obtained with hyperbaric bupivacaine prior
to incision. A false statement concerning this situation includes
A. Effective cough is preserved
B. The cardioaccelerator nerves are blocked
C. Examination of the biceps reveals fu ll strength bilaterally
D. Bupivacaine binds to the intracellular portion of sodium channels

16. A patient with cholestasis presents for preoperative evaluation with laboratory findings
revealing normal aspartate aminotransferase (serum glutamic-oxaloacetic transaminase) and
prothrombin time but with a markedly elevated alkaline phosphatase. He w ill need a muscle
relaxant for upcoming colon surgery. Which of the follow ing anesthetic scenarios should be
considered?
A. Prolonged duration of vecuronium action
B. Increase intubating dose of atracurium
C. Prolonged duration of succinylcholine action
D. Shortened duration of pancuronium action

17. An alcoholic 62-year-old male patient is noted to have jaundice one day after a laparoscopic
cholecystectomy under halothane/fentanyl general endotracheal anesthesia. B ilirubin and
alkaline phosphatase are elevated, but alanine aminotransferase (serum glutamic-pyruvic
transaminase [SGPT]) and aspartate aminotransferase (serum glutamic-oxaloacetic
transaminase [SGOT]) are within normal ranges. Of note, all values were within normal limits
in this patient preoperatively. The most likely cause of his jaundice is
A. Idiopathic halothane hepatic injury
B. Worsening of underlying chronic liver dysfunction
C. Posthepatic biliary obstruction
D. Intravenous acetaminophen administration

18. An initial bolus of pancuronium was administered to a patient with end-stage liver disease with
associated ascites for general anesthesia. Appropriate anesthetic considerations include all of
the following, except
A. Increased sympathomimetic activity due to vagolysis
B. Intense histamine release immediately after administration
C. Larger volume of distribution requiring initial larger doses
D. Longer duration of action requiring smaller maintenance doses

19. A chronic alcoholic patient with liver cirrhosis is likely to demonstrate all of the follow ing
during administration of anesthesia, except
A. A high minimum alveolar concentration (MAC) for desflurane
B. Opioid hyperalgesia
C. Resistance to the hypnotic effects of thiopental
D. Resistance to the analgesic effects of opiates

20. A woman with long-standing alcoholic cirrhosis (Child-Turcotte-Pugh B) presents to the


emergency room for chronic shortness of breath and abdominal pain. A review of her lab
findings reveal a hematocrit concentration of 36% (hemoglobin 12.4 g/dL) with an arterial
blood gas revealing a Pao2 of 65 mm Hg breathing a Fio 2 of 0.5 via face mask. Her vitals are a
blood pressure of 135/60 mm Hg and a heart rate of 88 bpm. The most likely cause of her
hypoxemia is
A. Intrahepatic arteriovenous shunts
B. Intrapulmonary arteriovenous shunts
C. Anemia
D. Decreased cardiac output

21. Which of the follow ing cardiovascular abnormalities is least likely to be present in a patient
with end-stage alcoholic cirrhosis
A. Resting tachycardia
B. Widened pulse pressure
C. Increased peripheral vascular resistance
D. Increased cardiac output

Questions 22 to 23
A 120-kg diabetic male is scheduled for emergent pinning of his mandible after a motor vehicle
accident. His wife reports that he snores loudly every night with occurrences of breathing cessation.
Medical history is also significant for hypertension controlled with a diuretic. On physical
examination, he has a large tongue and a wide neck with inadequate mouth opening revealing a
Mallampati grade 4 view. His BMI is 38 kg/m2 with a neck circumference of 44 cm.

22. Arterial blood gas (ABG) finding that would confirm Pickwickian syndrome is
A. pH = 7.44, Paco2 = 44, Pao2= 90, HCO3 = 24
B. pH = 7.35, Paco2 = 44, Pao2= 65, HCO3 = 26
C. pH = 7.42, Paco2 = 36, Pao2= 80, HCO3 = 22
D. pH = 7.37, Paco2 = 55, Pao2= 67, HCO3 = 28
23. The dose of thiopental required for rapid-sequence induction would be increased , as compared
with what would be required at his ideal body weight, because of changes in
A. Decreased basal metabolic rate
B. Increased blood volume
C. Increased muscle mass
D. Decreased liver metabolism

24. A patient with chronic liver disease is scheduled for a laparoscopic abdominal operation. The
risk of mortality during surgery for this patient is assessed using
A. Mayo end-stage liver disease
B. Child-Turcotte-Pugh score
C. Ranson criteria
D. Alvarado score

25. The variable not used to calculate an MELD (model for end-stage liver disease) score to
prioritize patients for liver transplantation is
A. Creatinine
B. INR (international normalized ratio)
C. Bilirubin
D. Albumin

Questions 26 to 28
A 30-year-old male patient without preoperative renal dysfunction is undergoing a primary
orthotopic liver transplant (OLT) for failure due to inherited a1-antitrypsin deficiency.

26. During cross-clamping of the suprahepatic inferior vena cava (IVC), the most accurate effect
created by use of venovenous bypass (VVB) is that it
A. Induces urinary retention
B. Prevents metabolic acidosis
C. Requires heparinization
D. Supports cardiac output

27. Immediately before unclamping and reperfusion of the transplanted liver, sodium bicarbonate
and calcium chloride are administered intravenously to counteract
A. Coagulopathy
B. Decreased cardiac output
C. Increased systemic vascular resistance
D. Hypertension

28. At the end of the case as the drapes are taken down, diffuse microvascular bleeding is noted in
this patient who required 15 U of blood during his intraoperative course. Platelet count is
40,000/mm3, prothrombin time is 18 seconds, activated partial thromboplastin time (PTT) is 54
seconds, D-dimer is 2,000 ng/mL, and serum fibrinogen concentration is 40 mg/dL. The most
likely cause of bleeding is
A. Disseminated intravascular coagulation (DIC)
B. Abnormal platelet function
C. Depressed levels of factor V III
D. Citrate toxicity

29. A patient presents for preoperative evaluation for upcoming surgery. He has a history of liver
transplantation 2 years ago, otherwise feeling well. Which of the follow ing is most likely to be
present during preoperative evaluation?
A. Elevated serum creatinine concentration
B. Hypoalbuminemia
C. Prolonged partial thromboplastin time
D. Hypocalcemia

30. Following a gastric bypass procedure, a 130-kg woman is extubated and breathing
spontaneously in the recovery room (PACU). She is breathing at a rate of 24 breaths/min on 10
L/min of oxygen via nasal cannula, and is complaining of continued subjective dyspnea.
Arterial blood gas analysis shows Pao2 = 95 mm Hg, Paco2 = 44 mm Hg, and pH = 7.37. The
parameter most closely related to her increased alveolar-arterial oxygen-tension gradient is
A. Decreased minute volume
B. Decreased functional residual volume
C. Decreased expiratory reserve volume
D. Decreased respiratory drive

31. During laparoscopic cholecystectomy, the risk of failure to visualize contrast material entering
the duodenum during intraoperative cholangiogram is highest with the administration of
A. Buprenorphine
B. Nalbuphine
C. Morphine
D. Naloxone

32. Drugs that can decrease or reduce opioid-induced biliary spasm include all of the following,
except
A. Diltiazem
B. Atropine
C. Metoclopramide
D. Glucagon

33. Each of the follow ing is associated with delayed gastric emptying, except
A. Diabetes mellitus
B. Celiac plexus block
C. Vagotomy
D. |i-Receptor agonism

Questions 34 to 39
A 33-year-old otherwise-healthy female suffering from moderately severe abdominal pain of unclear
etiology is set to undergo an exploratory laparoscopy. The abdominal cavity is insufflated using
carbon dioxide (CO2).

34. A ll of the follow ing are correct statements regarding pathophysiologic changes associated with
creation of the pneumoperitoneum, except
A. Increased risk of reflux and aspiration
B. Decreased venous return
C. Decreased systemic vascular resistance (SVR)
D. Increased intrathoracic pressures

35. Inherent risks of abdominal laparoscopy include


A. Renal failure
B. Bronchospasm
C. Gas emboli
D. Hypothermia

36. The patient is placed in a steep Trendelenburg position. Her oxygen saturation begins to
gradually decline over the course of several minutes while being ventilated with 100% oxygen
(Fio 2 = 1.0). The initial step in the management of her hypoxemia is

A. Add positive end-expiratory pressure (PEEP)


B. Intravenous bolus of 500 mL saline
C. Reposition the patient
D. Switch to pressure support ventilation

37. The exploratory surgery progresses slowly. Over the next 3 hours, her EtCO2 begins to
gradually rise, requiring increasing minute ventilation. A ll of the follow ing contribute to the
degree of systemic CO2 absorption, except

A. Solubility of the gas


B. Intra-abdominal pressures (IAP)
C. Duration of surgery
D. Blood pressure

38. Each of the follow ing is hemodynamic change associated with hypercarbia, except
A. Arrhythmias
B. Bradycardia
C. High cardiac output
D. Low systemic vascular resistance (SVR)

39. The surgery continues on with a request to increase the pneumoperitoneum to 30 mm Hg to


improve the surgical view. A ll of the follow ing are appropriate in the differential diagnosis for
hypotension during laparoscopy, except
A. Compression of the inferior vena cava
B. Increase cardiac afterload
C. Too small blood pressure cuff
D. CO2 embolism

40. This physical exam finding is inappropriately paired with the possible nerve injury resulting
from ill positioning during surgery:
A. Inability to evert the foot common peroneal nerve
B. Inability to stand on toes sciatic nerve
C. Difficulty climbing stairs femoral nerve injury
D. Foot drop saphenous nerve injury

41. A 50-year-old male patient is to undergo an open nephrectomy for renal carcinoma. The patient
requests an epidural for perioperative pain management, as he is strongly intolerant to |i-
agonist opiate therapy with nausea and vomiting. After a T2 sensory level is obtained, the
patient is induced with propofol 200 mg and rocuronium 70 mg, followed by tracheal
intubation. The expected response to intubation in this patient includes
A. Hypertension
B. Tachycardia
C. Tachypnea
D. Mydriasis

42. A 24-year-old female status postrecent living-related renal transplant requires chronic
immunosuppression with cyclosporine and steroids to combat organ rejection. She now
presents for right-knee arthroscopic anterior cruciate ligament repair and mentions significant
history of postoperative nausea and vomiting (PONV). The most appropriate next step in
planning her anesthetic management is
A. Proceed with total IV anesthesia (TIVA), avoiding inhaled anesthetics
B. Avoid regional anesthesia
C. Liberally infuse intravenous fluids
D. Use metoclopramide to decrease gastric secretions

Questions 43 to 45
A 70-year-old 70-kg male with benign prostatic hypertrophy and difficulty with urination presents for
a transurethral resection of his 65-g prostate (TURP). His other pertinent history includes
hypertension and hyperlipidemia, both well controlled. He has a remote history of a lumbar spinal
fusion with no current lumbar symptomatology. The patient requests a general anesthetic for the
procedure and refuses spinal anesthesia.

43. Assuming the use of a hypotonic irrigant, these factors w ill contribute to the amount of fluid
absorbed by the patient, except
A. Venous pressure
B. Hydrostatic pressure of the irrigation infusion
C. Lithotomy position
D. Size of prostate

44. In the recovery room, he complains of bothersome localized suprapubic pain and is requesting
pain medicine. He denies pain or discomfort anywhere else. His review of systems is negative
for fevers or chills. The relatively common complication of this procedure that should be ruled
out at this time is
A. Hyponatremia
B. Glycine toxicity
C. Extraperitoneal perforation
D. Transient bacteremia

45. The patient is administered hydromorphone intravenously, and 20 minutes later is feeling well
with minimal pain complaints. At this time, his postoperative laboratories have returned,
revealing a serum sodium value of 130 mEq/L. The most appropriate next step in the
management of his hyponatremia is
A. Hypertonic saline infusion
B. Fluid restriction
C. Demeclocycline administration
D. Insulin and glucose administration

46. Effects of furosemide administration in the perioperative period include


A. Hypernatremia
B. Decreased risk for acute tubular necrosis
C. Metabolic alkalosis
D. Hyperkalemia

Questions 47 to 51
A 38-year-old woman is set to undergo extracorporeal shock wave lithotripsy to disintegrate a
painful stone trapped in her upper ureter. The patient is requesting an epidural anesthetic and is
choosing to be otherwise awake and cooperative with her positioning and procedure.

47. The step of the epidural placement that should be avoided in this patient is
A. Loss of resistance to air
B. Loss of resistance to hanging drop
C. Test dose injection
D. Bolus dose of local anesthetics

48. Once the epidural is adequately placed and the patient is immersed sitting in the water tank, the
physiologic change that should be expected is
A. Decreased central venous pressure
B. Increased vital capacity
C. Increased functional residual capacity
D. Lower extremity peripheral pooling

49. Extracorporeal shock wave lithotripsy therapy proceeds with the shock wave synchronized with
what ECG phase of the cardiac cycle?
A. The P wave
B. The Q wave
C. The R wave
D. The S wave

50. Which of the follow ing statements would be considered false with regard to extracorporeal
shock wave lithotripsy (ESWL)?
A. Delivery of the shock wave is timed to coincide with the ventricular refractory phase
B. Neuraxial anesthesia up to T2 sensory level is adequate
C. If able to control ventilation, use high tidal volumes and low respiratory rate
D. Removal of the patient from the bath water can be accompanied by a decrease in the blood
pressure

51. A ll of the follow ing are contraindications to immersion extracorporeal shock wave lithotripsy,
except
A. Harrington rod implants
B. Abdominally placed rate-responsive cardiac pacemaker
C. Positive pregnancy test
D. Large calcified abdominal aortic aneurysm

52. Which of the follow ing is considered the most sensitive indicator of impending traumatic renal
failure?
A. Decreased creatinine clearance
B. Decreased central venous pressure
C. Decreased fractional excretion of sodium
D. Increased urine osmolality

53. A 26-year-old male patient with A lport syndrome requires hemodialysis (every third day) and
presents for an arteriovenous fistula creation. His last dialysis treatment was yesterday. Patient
requests general anesthesia for this procedure. Which of the follow ing drugs w ill have a
prolonged duration of action?
A. Fentanyl
B. Neostigmine
C. Atracurium
D. Methadone

54. Each of the follow ing is associated with acute tubular necrosis, except
A. Hyaline casts
B. Urine specific gravity <1.010
C. Muddy casts
D. Fractional excretion of sodium of 4%

Questions 55 to 56
A 75-year-old patient who is awaiting urgent laparotomy has had oliguria for the past 12 hours since
the onset of his acute abdominal pain last night. His medical history includes well-controlled
hypertension. Vital signs include a BP of 120/65 mm Hg and a HR of 72 bpm. His laboratory findings
reveal

Urine osmolality: 550 mOsm/L


Urine specific gravity: 1.020
Urine sodium concentration: 15 mmol/L
Fractional excretion of sodium: 0.5%
Ratio of urine-to-plasma urea concentration: 10

55. The most appropriate treatment of his oliguria is


A. Fluid restriction
B. Fluid challenge
C. Renal ultrasound
D. Foley placement

56. Fluid resuscitation is done with 4 L of normal saline. The potential acid-base abnormality that
can occur is
A. Hyperchloremic acidosis
B. Metabolic alkalosis
C. Hyperkalemic acidosis
D. Respiratory alkalosis

57. A 67-year-old patient with chronic renal failure presents for hip arthroscopy to address and
treat his labral tears and associated hip pain. The best option for opioid therapy in this patient is
A. Meperidine
B. Codeine
C. Dextropropoxyphène
D. Fentanyl
CHAPTER 14 ANSWERS

1. B. The initial reduction in core temperature during general anesthesia is caused by


redistribution of heat from the core to the periphery, which can be attenuated by increasing
ambient temperature to minimize the gradient.

2. D. Perioperative m orbidity related to obesity is associated with changes in respiratory (e.g.,


difficult airway, decreased functional residual capacity), cardiovascular (e.g., increased cardiac
output), and gastrointestinal (e.g., gastroesophageal reflex disease, increased abdominal
pressure) systems that w ill impact the delivery of anesthesia. Given that metabolism of
inhalational agents is increased over normal weight patients, higher minimum alveolar
concentrations may be required. Atracurium (including cis-atracurium) is metabolized via
Hofmann degradation and is unaffected by the obese state.

3. B. Aspiration of acidic gastric juices poses a potential threat during induction and intubation.
H2-blockers (e.g., cimetidine, ranitidine) can decrease gastric volume and raise pH to a level
that should be protective from fatal aspiration. Metoclopramide promotes gastrointestinal
m otility without directly affecting pH itself. 5-HT3 (serotonin) receptor antagonism (e.g.,
ondansetron) and D2 (dopamine) antagonism (e.g., prochlorperazine) are useful antiemetics,
with no effect on gastric pH or volume.

4. B. Atelectasis likely occurs in all patients who undergo general anesthesia, in particular
those postabdominal surgeries. Changes of microatelectasis develop routinely and do not
significantly delay discharge for most patients despite the relative state of hypoxia (decreased
Pao2). Deep breathing, use of an incentive spirometer, early mobilization, and adequate pain
control are all measures used to expand lung volumes and promote improved oxygenation.

5. A. Postsurgical atelectasis is treated by physiotherapy, focusing on deep breathing while


encouraging coughing. An incentive spirometer is often used to promote fu ll expansion of the
lungs. Ambulation is also highly encouraged to improve lung inflation. These measures are
considered first-line options for his presumed microatelectasis. In the smoker, coughing w ill
also clear the airways of mucous to improve aeration. Doxapram stimulates chemoreceptors in
the carotid bodies, which in turn stimulates the respiratory center in the brain stem to increase
tidal volume and respiratory rate.

6. A. Initial management involves the recognition of a possible aspiration event when there are
visible gastric contents in the oropharynx. Once diagnosis is suspected, the patient should be
placed in Trendelenburg position to lim it pulmonary contamination, followed by suctioning of
the oropharynx. Empirical antibiotic therapy is strongly discouraged unless it is apparent that
the patient has developed a subsequent pneumonia. Corticosteroids should not be given
prophylactically, as there is no evidence to support this practice.
7. B. Gastrointestinal secretions, including diarrhea and intestinal fistulas, are rich in
bicarbonate and, therefore, losses w ill cause a metabolic acidosis. However, respiratory
compensation for metabolic processes w ill occur almost immediately by increasing ventilation
to blow o ff CO2 to reduce the acidosis, effecting change in as quick as 15 to 30 minutes.
Therefore, one would expect ABG findings of a metabolic acidosis with fu ll respiratory
compensation.

8. A. The ACT enables one to monitor the anticoagulant effect of unfractionated heparin. ACT
prolongation can also indicate coagulation-factor deficiency, severe thrombocytopenia, or
severe platelet dysfunction. The ACT is sensitive to a deficiency or dysfunction of all the
clotting factors (except factor V II)— indicating problems with the intrinsic or common
pathways. Factor level must be less than 5% of normal to prolong the ACT.

9. A. An acute hemolytic transfusion reaction is associated with hemolysis of transfused blood,


usually related to ABO incompatibility with associated hemoglobinuria. Pulmonary
leukoagglutinin reaction is related to the presence of antileukocyte antibodies in donor plasma
leading to transfusion-related acute lung injury.

10. B. Malnourished surgical patients are at greater risk for postoperative m orbidity and mortality
compared to a well-nourished patient undergoing similar operations for similar indications.
However, providing TPN to the malnourished patient in the perioperative period carries its own
inherent risks, such as greater risk of infection, hyperglycemia, and electrolyte abnormalities.

11. D. For those on TPN, the anesthesiologist must monitor blood glucose levels meticulously to
avoid hypo- or hyperglycemia. Hyperosmolar, nonketotic, hyperglycemic coma has been
reported in patients who fail to regain consciousness after anesthesia.

12. B. Ensuring the presence of normal serum phosphate levels in the patient receiving TPN is
essential, as hypophosphatemia has been associated with acute respiratory failure due to
profound areflexic muscle weakness.

13. D. The most critical element to safe perioperative care of the pheochromocytoma patient is
adequate preoperative blockade against the effects of the circulating catecholamines. The main
goals of preoperative blockade are to normalize blood pressure and heart rate, restore volume
depletion, and prevent surgery-induced catecholamine storm. A sign of adequate a-blockade is
the development of nasal congestion due to smooth-muscle relaxation of nasal mucosal
arterioles.

14. A. Switching from isoflurane to sevoflurane is not an appropriate method to treat the
catecholamine storm, which can occur during direct surgical manipulation of the tumor. An a­
blocker, vasodilator, and lidocaine are appropriate options to counter the effects of
catecholamine storm.
15. A. Sympathetic preganglionic fibers originate in the intermediolateral cell column of the
spinal cord from T1 to L2. Cardiac innervation is principally via sympathetic fibers from T1 to
T4. As such, high thoracic blockade up to T2 w ill block the cardioaccelerator nerves, leading to
bradycardia and hypotension. The respiratory system is usually unaffected, as diaphragmatic
breathing alone can maintain relatively normal arterial blood gases. However, patients may feel
unable to breath and are often unable to cough effectively.

16. A. The pharmacokinetics of many nondepolarizing muscle relaxants in the presence of


cholestasis and obstructive jaundice may be altered. The prolonged duration of action likely
results from both inhibition of hepatic uptake by the accumulated bile salts and a general
deterioration of liver transport function. Succinylcholine, atracurium, and cis-atracurium have
theoretical advantages because their elimination occurs via plasma cholinesterases and
Hofmann degradation, respectively, mostly independent of renal or hepatic function.

17. C. Postoperative liver dysfunction is common, but is generally m ild and asymptomatic (Table
14-2). M ild transient increases in serum levels of liver enzymes (SGOT/SGPT) are often seen
within hours of surgery, but rarely persist >2 days. Subclinical hepatocellular injury can occur
in up to 50% of those receiving an inhaled anesthetic with halothane. Though volatile
anesthetics are often implicated as the cause of postoperative jaundice, there are many other
causes to consider. A surgical cause is likely if the operation involved the liver or biliary tract.
Drugs, including antibiotics, and other metabolic or infectious causes must also be ruled out.

Table 14-2 Postoperative Liver Dysfunction—Causes and Differentiation.


P R E H E P A T IC IN T R A H E P A T IC P O S T H E P A T IC

E tio lo g y H em o ly sis Severe a rte ria l h y p o x e m ia S tones


H e m a to m a re a b s o rp tio n C irrh o sis Sepsis
B iliru b in o v e rlo a d C o n g e stiv e h e a r t failu re C ancer
Sepsis
V iru se s
D r u g -in d u c e d

B iliru b in T (u n c o n ju g a te d ) T (c o n ju g a te d ) t (c o n ju g a te d )

A m in o tra n sfe ra s e s U nchanged TT U nchanged

A lk a lin e p h o sp h a ta s e U n c h a n g e d U nchanged Tt
P ro th r o m b in tim e U nchanged T U nchanged

A lb u m in U nchanged i U nchanged

18. B. Chronic liver disease may interfere with the metabolism of drugs due to decreased number
of functional hepatocytes or decreased hepatic blood flo w that typically accompanies cirrhosis
of the liver. Prolonged elimination half-life times for morphine, diazepam, lidocaine,
pancuronium, and, to a lesser degree, vecuronium have been demonstrated in this population.
Cirrhotic patients w ill require a larger initial dose of pancuronium due to increased volume of
distribution for this hydrophilic agent with smaller maintenance doses for prolonged duration
of action. Pancuronium has slight vagolytic activity resulting in increased heart rate and cardiac
output. Mivacurium and atracurium are associated with histamine release.

19. B. Certain physiologic and pathologic states may alter MAC of inhaled anesthetics. MAC is
higher in infants and lower in the elderly. Also, MAC increases with hyperthermia, alcoholism,
and thyrotoxicosis. Furthermore, hypothermia, hypotension, and pregnancy seem to decrease
MAC, while duration of anesthesia, gender, height, and weight seem to have little effect on
MAC. Those with chronic liver disease are also at increased risk of arterial-venous shunting.

20. B. Those with chronic liver disease are at increased risk of arterial-venous shunting. The
presence of intrapulmonary shunting w ill result in hypoxemia.

21. C. Cirrhosis is typically associated with several cardiovascular abnormalities including a


hyperdynamic circulation characterized by increased cardiac output and decreased peripheral
resistance. Other cardiovascular changes include a resting tachycardia, warm peripheries, a
bounding pulse, and a widened pulse pressure.

22. D. Obesity hypoventilation syndrome (aka Pickwickian syndrome) is a state in which the
severely overweight patient fails to breathe rapidly or deeply enough, resulting in hypoxia and
hypercarbia. If Pickwickian syndrome is suspected, the most important initial test is the
demonstration of elevated carbon dioxide in the blood. This requires either an ABG or a
measurement of bicarbonate levels in venous blood. Expected ABG findings would reveal a
chronic, compensated respiratory acidosis.

23. B. Redistribution of thiopental to inactive tissue sites rather than metabolism is the most
important determinant of early awakening follow ing a single intravenous injection.

24. B. The Child-Turcotte-Pugh score is used to predict mortality during surgery in patients with
chronic liver disease, namely, cirrhosis. The Mayo or model for end-stage liver disease was
in itia lly developed to predict death within 3 months of surgery in patients who had undergone a
transjugular intrahepatic portosystemic shunt procedure and was subsequently found to be
useful in determining prognosis and prioritizing patients for liver transplant. Alvarado score is
used for appendicitis, while the Ranson criteria assess pancreatitis.

25. D. The MELD score is a formulaic calculation utilizing three variables: creatinine, INR, and
bilirubin. For dialysis-dependent patients, the creatinine score is automatically set to 4 mg/dL
despite true serum levels.

MELD score = 10 x [0.957 x log e (creatinine) + log e (bilirubin)


+ 1.12 x log e (INR)] + 6.43

26. D. Standard technique of OLT causes changes in hemodynamics during the anhepatic phase
because of cross-clamping of the suprahepatic IVC. Interruption of the IVC and portal vein flow
causes a decrease in preload, cardiac output, and arterial blood pressure. VVB has been used to
achieve hemodynamic stability by avoiding venous congestion, promoting venous return with
decrease incidence of renal dysfunction.

27. B. Postreperfusion syndrome is the most common hemodynamic derangement in liver


transplantation, manifesting mainly as decreased heart rate, mean arterial pressure, and
systemic vascular resistances. Ventricular function, both right and left, has been shown to be
normal during reperfusion, in which case the visceral and liver vasodilation that occurs would
be the main cause of arterial hypotension. Prophylaxis with atropine prevents bradycardia but
not hypotension. Administration of calcium chloride and sodium bicarbonate together with
hyperventilation mitigates the symptoms related to the reduced cardiac output.

28. A. Coagulopathy follow ing massive transfusion is a consequence of posttraumatic and


surgical hemorrhage. Bleeding follow ing massive transfusion can occur due to hypothermia,
dilutional coagulopathy, platelet dysfunction, fibrinolysis, or hypofibrinogenemia. Transfusion
of 15 to 20 U of blood products causes dilutional thrombocytopenia contributing to the
bleeding. Excessive fibrinolysis and low fibrinogen are further causes of bleeding in these
patients. The hemostatic signatures of DIC are low platelets, low fibrinogen, prolonged
prothrombin, prolonged PTT, elevated D-dimers, and low antithrombin.

29. A. Long-standing insufficient liver function is believed to cause changes in the circulation that
changes vessel tone and blood flo w in the kidneys. The likely presence of renal insufficiency is
a consequence of these changes in blood flow, rather than direct damage to the kidney itself.

30. C. Dyspnea is a common complaint in individuals with class II or III obesity, especially
follow ing a general anesthetic. As such, individuals present with a pronounced reduction in
expiratory reserve volume and an increase in the alveolar-arterial oxygen gradient.

31. C. ^-Receptor agonism may contribute to sphincter of Oddi spasm, preventing passage of
contrast with fu ll ^-agonist more likely to contribute versus partial ^-agonists (e.g.,
buprenorphine) and agonist-antagonist (e.g., nalbuphine). Naloxone, as a ^-antagonist would
alleviate any opioid-induced spasm.

32. C. A variety of agents that can produce smooth-muscle relaxation have been used. Nitrates and
calcium channel blockers have been the most extensively studied. Anticholinergics, including
atropine and glucagon, are additional agents that can provide sphincter of Oddi relaxation.
Metoclopramide is a prom otility agent that enhances sphincter smooth-muscle contraction.

33. B. Sympathetic celiac plexus blockade leaves parasympathetic fibers unopposed with
associated increased gastrointestinal m otility and possible diarrhea.

34. C. Abdominal laparoscopy requires insufflation of the abdominal cavity, most commonly
using CO2, to create a pneumoperitoneum. Increase in intra-abdominal pressures w ill place the
patient at a greater risk of reflux and aspiration; thus, general anesthesia with an endotracheal
tube is required. High pressures in the abdominal cavity can also compress both small and large
blood vessels, hampering venous return to the heart. Intrathoracic pressures are also increased,
associated with diaphragm elevation, compromising cardiac output further. Increase in SVR
occurs during pneumoperitoneum, reflected as an increase in afterload for left-sided heart
chambers.

35. C. Abdominal laparoscopy, though relatively safe, is associated with a few inherent dangers
including gaseous embolism, potential inability to control bleeding, an increase in CO2 partial
pressures and changes in arterial blood pressure and heart rate. CO2 absorption from the
peritoneal cavity can result in a state of acidosis as Paco2 rises.

36. A. The supine position under general anesthesia results in a decrease in functional residual
capacity (FRC). Pneumoperitoneum and the Trendelenburg position shifts the diaphragm
cephalad, further decreasing FRC. If FRC becomes less than closing capacity, airway collapse,
atelectasis, and ventilation/perfusion mismatch can further compromise respiratory function.
The judicious use of PEEP can be helpful to mitigate end-expiratory alveolar collapse;
however, too much PEEP can contribute to deterioration in right-sided cardiac performance.

37. D. Systemic absorption of gas from pneumoperitoneum is determined by factors including


solubility of the gas, IAP, and duration of surgery. Therefore, CO2 laparoscopy may produce
hypercarbia, particularly during long surgeries under high IAP unless minute ventilation is
increased. In those with severely compromised cardiopulmonary function and restricted CO2
clearance, severe hypercarbia can occur despite aggressive hyperventilation.

38. B. Hypercarbia causes hemodynamic changes by its direct action on the cardiovascular system
and indirect actions through the sympathetic nervous system. Manifestations while under
general anesthesia include tachycardia, arrhythmias, high cardiac output, increased arterial
blood pressure, and low SVR with flushed skin.

39. C. The effects of pneumoperitoneum include compression of the inferior vena cava resulting
in poor venous return and low preload. Systemic vascular resistance increases proportionately
when the intra-abdominal pressure is elevated, providing a larger afterload against which the
left ventricle must function. During insufflation, a gas embolus can occur, entering the venous
system to create an “ air lock” with mechanical obstruction of the right-side chambers. A blood
pressure cuff that is too small for the arm w ill result in erroneously high blood pressure
readings.

40. D. The saphenous nerve is the largest and longest branch of the femoral nerve that supplies
sensory innervation to the medial aspect of the lower leg. Movement of the foot is unaffected.

41. D. High thoracic epidural blockade up to T2 blocks the cardiac accelerators, providing
adequate sympathectomy to prevent hypertension and tachycardia. Sympathetic outflow to the
pupil travels via the intermediolateral cell column at the C8 to T2 cord level and remains intact;
thus, the sympathetic surge can still result in mydriasis.

42. A. Transplant recipients are always under various regimens of immunosuppression to prevent
organ rejection. Clinically significant reductions in serum levels of these medications can be
caused by dilution with massive fluid resuscitation perioperatively, as well as with
cardiopulmonary bypass. Many immunosuppressants are metabolized in the liver via the
cytochrome P450 system such that drugs administered during anesthesia (or perioperatively)
may affect blood levels including increased concentrations with cimetidine and metoclopramide
and decreased levels with octreotide. Regional anesthesia and/or TIVA are reasonable options to
minimize PONV in this patient.

43. C. Normally, about 20 mL/min of irrigation fluid is absorbed (1-1.5 L for a normal case with
resection time about 45-60 minutes), which increases as the duration of the surgery increases.
In clinical practice, it is almost impossible to accurately assess the volume absorbed. The
amount of fluid absorbed depends on several other factors as well, including the hydrostatic
pressure of the irrigation infusion (determined by the height of the bag), venous pressure (more
fluid absorbed if patient is hypotensive), the size of the prostate to be resected (associated with
longer time required), blood loss (implies a large number of open veins), and surgical skills of
the surgeon (efficiency with time management and hemostasis).

44. C. Another relatively common complication of TURP is perforation of the bladder.


Perforations usually occur during difficult resections and are often made by the cutting loop or
knife electrode. The tip of the resectoscope can also cause injury, as well as overdistention of
the bladder with irrigation fluid. Most perforations are extraperitoneal, and in the awake patient,
they result in pain in the periumbilical, inguinal, or suprapubic regions; additionally, the
urologist may note the irregular return of irrigating fluid. Less often, the perforation is through
the wall of the bladder and thus intraperitoneal. In such cases, pain may be generalized, in the
upper abdomen, or referred from the diaphragm to the shoulder. Bacteremia is usually
asymptomatic and easily treated with commonly used antibiotic combinations that are effective
against gram-positive and gram-negative bacteria.

45. B. For normovolemic, asymptomatic hyponatremic patients, free water restriction is generally
the treatment of choice. There is no role for hypertonic saline in these patients. The volume of
restriction should be based on the patient’s renal diluting capacity. If patient is unable to adhere
to fluid restrictions, consider use of a loop diuretic (e.g., furosemide) to increase free water
excretion in the kidneys. Demeclocycline is a tetracycline antibiotic that has a secondary effect
of reducing the responsiveness of the collecting tubule cells to antidiuretic hormone, thus
improving free water loss.

46. C. As with many diuretics, furosemide can cause dehydration and electrolyte imbalance,
including loss of potassium, calcium, sodium, and magnesium. Excessive use of furosemide
w ill most likely lead to a metabolic alkalosis due to hypochloremia and hypokalemia.

47. A. With epidural anesthesia, consider avoiding the use of loss of resistance to air for
identifying the epidural space, as air w ill provide an interface and cause dissipation of shock
wave energy resulting in local tissue injury. Animal experiments have shown epidural tissue
damage follow ing injection of air followed by exposure to shock waves.
48. D. Water immersion produces significant changes in the cardiovascular and respiratory
systems. Cardiovascular changes include an increase in central blood volume, with an increase
in central venous and pulmonary artery pressures, which are directly correlated with the depth
of immersion. The sitting position, together with either general or epidural anesthesia, would
tend to cause peripheral pooling and decreased venous return. Respiratory changes with
immersion up to the clavicles are significant: functional residual capacity and vital capacity are
reduced by 20% to 30%; pulmonary blood flo w has been shown to increase; and tight
abdominal straps and the hydrostatic pressure of water on the thorax impart a characteristic of
shallow, rapid breathing pattern.

49. C. Shock wave-induced cardiac arrhythmias occur in up to 10% to 14% of patients


undergoing lithotripsy despite the fact that shock waves are purposefully synchronized with the
patient’s ECG and are delivered in the refractory period of the cardiac cycle (R wave).

50. C. An advantage of providing a general anesthetic for ESWL is that ventilatory parameters can
be controlled using high frequency and low volumes to decrease stone movement with
respiration.

51. A. Contraindications for lithotripsy include the follow ing: pregnancy, a large aortic
aneurysm, certain bleeding conditions, and certain skeletal deformities that prevent accurate
focus of shock waves. Patients with abdominally placed cardiac pacemakers should notify their
doctor. Rate-responsive pacemakers that are implanted in the abdomen may be damaged during
lithotripsy. Orthopedic prostheses, including hip prostheses and even Harrington rods, are
generally not a problem as long as they can be kept out of the blast path.

52. A. Creatinine clearance test evaluates how efficiently the kidneys clear creatinine from the
blood. Creatinine, a waste product of muscle energy metabolism, is produced at a constant rate
that is proportional to the muscle mass of the individual. Because the body does not recycle it,
all of the creatinine filtered by the kidneys in a given amount of time is excreted in the urine,
making creatinine clearance a very specific measurement of kidney function.

53. B. Succinylcholine, atracurium, and cis-atracurium have theoretical advantages because their
elimination occurs via plasma cholinesterases and Hofmann degradation, respectively, mostly
independent of renal or hepatic function. Fentanyl and methadone are also considered relatively
safe in renal failure as they have no active metabolites. Methadone has limited plasma
accumulation in renal failure as it is prim arily eliminated in the feces. In terms of reversal
agents, renal excretion accounts for approximately 50% of the clearance of neostigmine and
approximately 75% of elimination of edrophonium and pyridostigmine. Renal failure allows
some protection against residual neuromuscular blockade because renal elimination half times
of anticholinesterase drugs is prolonged.

54. A. Acute tubular necrosis is classified as a “ renal” (e.g., not prerenal or postrenal) cause of
acute kidney injury. Diagnosis is made by a fractional excretion of sodium >3%, greater than
expected urine sodium concentration with low osmolality and presence of muddy casts on
urinalysis. A sensitive indicator of tubular function is sodium handling because the ability of an
injured tubule to reabsorb sodium is impaired, whereas an intact tubule can maintain this
resorptive capacity. If the patient has tubular damage for any reason, the urinary sodium w ill be
greater than expected. Keep in mind that the use of diuretics, however, can complicate the
interpretation of these results. Low urine flow, concentrated urine, or an acidic environment can
contribute to the formation of hyaline casts, pointing to hypovolemia and prerenal failure
(Table 14-3).

Table 14-3 Differentiation between Prerenal and Intrinsic Renal Failure.


PARAM ETER P R E R E N A L F A IL U R E A C U T E T U B U L A R N E C R O S IS

Urine Na+ (meq/L) <20 >40


Urine osmolality (mOsm/kg) >500 <350
FENa % <1 >2
Urea % <35 >35
Urine specific gravity >1,020 <1,010
Urine:plasma urea ratio >10:1 <7:1

55. B. Urinalysis reveals a prerenal state. Treatment focuses on correcting the cause of the
prerenal acute renal failure, most often with a fluid challenge. Depending on the cause, the
condition often reverses itself within a couple of days after normal blood flo w to the kidneys
has been restored. But if it is not reversed or treated successfully and quickly, prerenal acute
renal failure can cause tissue death in the kidneys and lead to intrinsic (intrarenal) acute renal
failure.

56. A. Hyperchloremic acidosis is a well-recognized entity as a consequence of large volume


administration of some intravenous fluids. Normal saline (0.9% sodium chloride solution) and
colloids suspended in normal saline are often infused because they are easily available, and are
isotonic with plasma. When a patient is given normal saline (a hyperchloremic solution),
chloride levels can significantly increase. It is the chloride anion that is the ultimate cause of the
acidosis. Consider this equation: sodium chloride combines with water: NaCl + H2O HCl +
NaOH. The strong acid (HCl) and the strong base (NaOH) should cancel each other out, with no
effect on pH. However, because the normal concentrations of Na+ and Cl - in the serum are 140
and 100, respectively, adding normal saline (154 mEq Na and 154 mEq Cl) causes the chloride
to increase proportionately more than the sodium. This increase in chloride tips the acid-base
balance toward HCl, thereby causing a metabolic acidosis.

57. D. Chronic pain is common in chronic kidney disease and most w ill rate their pain as
moderate to severe. The absorption, metabolism, and renal clearance of opioids are complex in
renal failure. However, with the appropriate selection and titration of opioids, patients with
renal failure can achieve analgesia with minimal risk of adverse effects. Meperidine is not
recommended in renal failure due to accumulation of normeperidine, which may cause
seizures. Morphine is not recommended for chronic use in renal insufficiency due to the rapid
accumulation of its active nondialyzable metabolite (morphine-6-glucuronide). Codeine has
been reported to cause profound renal toxicity, which can be delayed and may occur after trivial
doses. Dextropropoxyphene is associated with central nervous system and cardiac toxicity and
is not recommended for use in patients with renal failure. On the other hand, fentanyl is
considered relatively safe in renal failure, as it has no active metabolites.
Endocrine Diseases
Jean Kwo and Edward Bittner

1. Type 1 diabetes mellitus


A. Is characterized by a relative lack of insulin plus resistance to endogenous insulin
B. Always requires insulin
C. Affects 95% of patients with diabetes
D. Can be controlled with diet, weight loss, and oral hypoglycemic agents

2. Preoperative assessment of patients with diabetes mellitus should include


A. An assessment of functional status
B. 24-Hour creatinine clearance
C. Pulmonary function testing
D. Cancellation of the surgical case if HbA1c >10%

3. Preferred anesthetic agent in a patient with hyperthyroidism includes


A. Desflurane
B. Ketamine
C. Sevoflurane
D. Meperidine

4. Multiple endocrine neoplasia (MEN) I syndrome includes


A. Pheochromocytoma, medullary thyroid carcinoma, parathyroid hyperplasia
B. Pancreas tumors, medullary thyroid carcinoma, pituitary adenoma
C. Pheochromocytoma, medullary thyroid carcinoma, mucosal neuromas
D. Pancreas tumors, pituitary adenoma, parathyroid hyperplasia

5. Laboratory findings in primary hypothyroidism are


A. Low TSH, elevated T3, elevated T4
B. Low TSH, low T3, low T4
C. Normal TSH, low T3, low T4
D. Elevated TSH, low T3, low T4
6. Obese patients may experience rapid oxygen desaturation during induction of general
anesthesia because of
A. A decrease in lung compliance
B. A reduction in functional residual capacity (FRC)
C. A history of obstructive sleep apnea
D. Restrictive lung disease

7. A 39-year-old woman with a history of headaches, hypertension, palpitations, and


nephrolithiasis is undergoing a parathyroidectomy for parathyroid adenoma. During induction,
she develops severe hypertension and tachycardia. The most likely diagnosis for these signs is
A. Adrenal insufficiency
B. Carcinoid syndrome
C. Thyroid storm
D. Pheochromocytoma

8. Phenoxybenzamine is a
A. Selective ^-receptor antagonist and a nonselective P-adrenergic receptor antagonist
B. Reversible ^-receptor antagonist
C. Irreversible, nonselective a-adrenergic receptor antagonist
D. Selective a2-receptor agonist

9. A 40-year-old woman with a history of Graves disease is in the recovery room after
undergoing a CT scan under general anesthesia. While in the recovery room, her blood
pressure drops to 80/55 mm Hg, her heart rate increases to 140 bpm, and she becomes agitated
and complains of difficulty breathing and feeling hot. The most likely diagnosis for these signs
is
A. Thyroid storm
B. Carcinoid syndrome
C. Malignant hyperthermia
D. Pheochromocytoma

10. Treatment of thyroid storm includes


A. Dantrolene
B. Phenoxybenzamine
C. Octreotide
D. Propylthiouracil

11. During a postoperative check on a 53-year-old patient who underwent a total thyroidectomy
earlier in the day, you notice that he is stridorous and is complaining of muscle cramps. The
best treatment for these symptoms is
A. Administration of calcium gluconate
B. Opening the neck wound
C. Reintubation for airway protection
D. Administration of sodium bicarbonate

12. Patients with obstructive sleep apnea (OSA)


A. Are at increased risk of left-heart failure
B. Have the same perioperative complication rate as patients without OSA
C. May have an increased likelihood of difficult intubation
D. Rarely require continuous positive airway pressure (CPAP) after bariatric surgery

13. A 39-year-old patient with a BMI of 45 kg/m2 is scheduled for a Roux-en-Y gastric bypass. She
has a history of hypertension. Your perioperative concerns include
A. Preparation for a rapid sequence induction, since she is at increased risk for aspiration of
gastric contents
B. Placing her in the reverse Trendelenburg position to reduce atelectasis in dependent areas
of the lung and move the chest and breast tissue caudally to allow easier access to the mouth
for endotracheal intubation
C. Need to dose water-soluble drugs (e.g., neuromuscular-blocking agents) to actual body
weight
D. More frequent administration of lipid-soluble drugs w ill be needed

14. During the preoperative evaluation of a critically ill patient with ischemic bowel scheduled for
a second look laparotomy and possible abdominal closure, you notice multiple electrolyte
abnormalities including hypophosphatemia, hypokalemia, and hypomagnesemia. A possible
cause for these electrolyte abnormalities is
A. Renal failure
B. Hypoventilation
C. Hypoparathyroidism
D. Refeeding syndrome

15. Complications of cricoid pressure include


A. Esophageal obstruction
B. Displacement of thoracic spine
C. Worsening of view of airway in patients with difficult airway
D. Need for less pressure in parturients

16. You are evaluating a 55-year-old patient with type 2 diabetes mellitus for a total knee
replacement. His diabetes is controlled on a regimen of Glucophage (metformin), NPH insulin
twice a day, and insulin sliding scale. Perioperative instructions for glucose management
should include
A. Give half of the NPH dose if morning blood glucose level is at least 150 mg/dL
B. Give regular insulin dose according to morning blood glucose level
C. Holding metformin for 48 hours preoperatively to avoid risk of fatal lactic acidosis
D. Starting insulin infusion with target glucose range of 81 to 108 mg/dL

17. Carcinoid tumors


A. Grow rapidly, and patients are often symptomatic with carcinoid syndrome
B. Synthesize epinephrine and norepinephrine
C. Can cause left-sided heart failure due to mitral and aortic valve damage
D. Can cause right-sided heart failure due to tricuspid and pulmonary valve damage

18. You are taking care of a 67-year-old patient undergoing a parathyroidectomy. The patient is
hypercalcemic with a serum calcium of 20 mg/dL. Anesthetic considerations should include all
of the following, except
A. Hypoventilation to decrease ionized calcium level
B. Careful titration of neuromuscular-blocking agents
C. Hydration with normal saline and diuresis with furosemide
D. Care with laryngoscopy because of risk of vertebral compression

19. Clinical manifestations of mineralocorticoid excess include


A. Hypotension
B. Metabolic acidosis
C. Hypokalemia
D. Tetany

20. Normal daily cortisol production (mg/day) in adults is


A. 10 to 15
B. 20 to 30
C. 50 to 60
D. 75 to 100

21. A 75-year-old patient with coronary artery disease, hypertension, and chronic obstructive
pulmonary disease (COPD) is undergoing a left colectomy for cancer. He had a COPD
exacerbation 4 months ago and was on steroids for a week at the time. Steroid replacement
A. Should be given at a dose greater than 10 times the normal daily cortisol production rate
B. Should not exceed 100 to 150 mg of cortisol equivalent per day
C. Is not necessary in this patient
D. Should include 100 mg of cortisol, tapered over 5 to 7 days

22. Physiologic effects of chronically elevated corticosteroid levels (Cushing syndrome) include
all of the following, except
A. Hypotension
B. Muscle wasting
C. Hypokalemia
D. Glucose intolerance

23. You are taking care of a 45-year-old patient undergoing a left adrenalectomy for a
pheochromocytoma. Intraoperative management includes
A. Use of ketamine as an induction agent to counteract the effects preoperative of a-
adrenergic blockade
B. Long-acting antihypertensive agents should be available to treat hypertension
C. Judicious fluid replacement as these patients are usually volume-overloaded
D. Magnesium sulfate infusion to treat hypertension

24. A 75-year-old, 110-kg patient is scheduled for a radical prostatectomy. He has a history of
hypertension and type 2 diabetes mellitus. His preoperative ECG is significant for Q waves in
leads II, III, and aVF, though the patient denies having a previous myocardial infarction. His
medications include insulin, Glucophage (metformin), a P-blocker, and an angiotensin-receptor
blocker. Upon induction, his blood pressure drops from 150/80 to 65/40. The most likely cause
of hypotension is
A. Use of angiotensin-receptor blocker
B. Diabetic autonomic neuropathy
C. Volume depletion
D. Myocardial ischemia

25. Patients with type 1 diabetes mellitus may be difficult to intubate because of
A. Increased supraglottic soft tissue due to chronic hyperglycemia
B. An association between type 1 diabetes and an anterior larynx
C. Limited jo in t m obility
D. An increased incidence of obesity in patients with type 1 diabetes
CHAPTER 15 ANSWERS

1. B. Type 1 diabetes mellitus results from the autoimmune destruction of insulin-producing p


cells of the pancreas and thus these patients always need insulin to prevent hyperglycemic
ketoacidosis and other complications. Most patients carrying the diagnoses of diabetes (95%)
have type 2 diabetes, which is characterized by a relative lack of insulin plus resistance to
endogenous insulin. Type 2 diabetes can be controlled with diet and weight loss, and oral
agents, though these patients may also require insulin.

2. A. Complications of diabetes result largely from microangiopathy and macroangiopathy.


Diabetes is a well-recognized risk factor for coronary artery disease (CAD). Cardiac autonomic
neuropathy may mask angina pectoris and obscure the presence of CAD. Hence, a careful
assessment of functional status and any symptoms such as increasing dyspnea on exertion and
fatigue may be indicative of significant CAD. While diabetes is a leading cause of renal failure,
there is no evidence that a preoperative evaluation with a 24-hour creatinine clearance is
helpful. While the risk of complications of diabetes increases with increasing HbA1c levels, and
there is evidence that higher HbA1c levels are associated with adverse outcomes follow ing a
variety of surgical procedures, there is insufficient evidence to recommend an upper lim it of
HbA1c prior to elective surgery. The risks associated with poor glycemic control should be
balanced against the necessity for surgery.

3. C. In patients with hyperthyroidism, the goal of anesthesia is to avoid an increase in heart


rate or sympathetic activation. Ketamine, desflurane, and meperidine cause sympathetic
stimulation and tachycardia. Conversely, anesthetics and techniques that reduce or blunt
sympathetic activity are preferred. Sevoflurane for anesthesia, and fentanyl and its congeners
for analgesia would be favored. Regional anesthesia, when practical, might also be efficacious
in avoiding sympathetic activation.

4. D. MEN I syndrome includes the triad of tumors of the pancreas, pituitary, and parathyroid
glands and is inherited as an autosomal-dominant trait. Medullary thyroid carcinomas are a
component of the MEN II endocrine syndromes, of which there are several subtypes.

5. D.

Table 15-1

T h y r o id d iso r d e r TSH T3 T4

P rim ary h ypothyroidism T 4 4


Secondary* h ypothyroidism 4 4 4
P rim ary hyp erth y ro id ism 4 t 4
Secondary* h y p erthyroidism t t t
Subclinical hypothyroidism T N orm al N orm al

Subclinical h y perthyroidism 4 N orm al N orm al

Sick euthyroid N o rm al 4 4
6. B. Obesity is associated with obstructive sleep apnea, decreased pulmonary compliance, and
lung volumes suggestive of restrictive lung disease. Total pulmonary compliance decreases due
to a decrease in both chest-wall compliance and lung compliance. Chest-wall compliance
decreases because of excessive adipose tissue over the thorax, while lung compliance decreases
because of the increased abdominal mass, which pushes the diaphragm cephalad causing an
increase in pulmonary blood volume. The FRC of the lung is the volume of air present in the
lungs at the end of passive expiration and reflects a balance between the elastic recoil of the
lungs and the pleural pressure. With obesity, there is a shift in this balance due to adipose tissue
in the chest wall and abdomen, resulting in a decreased FRC. The FRC is the reservoir of
oxygen during the apneic state associated with the induction of general anesthesia. Thus, the
reduction of FRC associated with obesity results in greater oxygen desaturation during the
induction of general anesthesia.

7. D. While inadequate anesthesia and thyroid storm may result in intraoperative hypertension
and tachycardia, the most likely diagnosis is pheochromocytoma. Pheochromocytoma is a
catecholamine-secreting tumor and is part of the multiple endocrine neoplasia (MEN) type II
syndrome, which consists of pheochromocytoma, medullary thyroid carcinoma, and
parathyroid adenoma. Symptoms associated with pheochromocytoma include paroxysmal
headache, hypertension, diaphoresis, and palpitations.

8. C. Phenoxybenzamine is an irreversible, nonselective a-adrenergic receptor antagonist used


preoperatively for adrenergic blockade in patients with pheochromocytomas. It blocks both the
postsynaptic a! and presynaptic a2 receptors in the nervous system, thereby reducing
sympathetic activity. Clinical signs of the optimal dose of phenoxybenzamine are a stuffy nose
and slight dizziness due to postural hypotension. Doxazosin is a reversible, selective a!-
receptor antagonist that is an alternative to phenoxybenzamine for treatment of
pheochromocytoma. In patients with pheochromocytoma, a-blockade is always started prior to
P-blockade. Starting P-blockade first w ill lead to unopposed a stimulation causing further
increase in the blood pressure.

9. A. Thyroid storm is characterized by fever, tachycardia, altered mental status, and


hypertension, presenting most often in the postanesthesia care unit or in the immediate
postoperative period (24 hours). Hypertension may be followed by congestive heart failure that
is associated with hypotension and shock. Thyroid storm is a state of severe hypermetabolism
induced by excessive release of thyroid hormones. It can be precipitated by surgery, stress,
infection, and drugs including chemotherapeutic agents, anticholinergic, and adrenergic drugs
such as pseudoephedrine, amiodarone, and iodinated contrast media. Unlike malignant
hyperthermia, it is not associated with muscle rigidity, an elevated creatinine kinase, or acidosis.

10. D. Thyroid storm is a medical emergency and if untreated, often fatal. Supportive treatment
includes cooling, hydration, and P-blockers to control heart rate. Propranolol has the additional
benefit of inhibiting the peripheral conversion of T4-T3. Propylthiouracil and methimazole
inhibit the synthesis of T4 by blocking the organification of tyrosine residues. Iodide blocks the
release of preformed thyroid hormones, but it should be given only after the loading dose of
antithyroid medication to prevent the utilization of iodine in the synthesis of new thyroid
hormones. Administration of cortisol is also recommended to prevent complications from
potential coexisting adrenal insufficiency.

11. A. Hypoparathyroidism resulting from the unintentional removal of the parathyroid glands is
a potential complication of thyroidectomy. Low blood calcium levels interfere with normal
muscle contraction and nerve conduction, and can result in muscle cramps, weakness, tetany,
laryngospasm, and stridor. Treatment consists of normalizing the serum calcium level with
intravenous calcium. While a neck hematoma can cause airway compromise due to
compression, it is unlikely to cause muscle cramps. Stridor due to bilateral vocal cord paralysis
is evident immediately on extubation and would require reintubation to establish a patent airway.
Sodium bicarbonate would cause a metabolic alkalosis and potentially worsen symptoms of
hypocalcemia by decreasing ionized calcium levels.

12. C. Patients with OSA may have an increased likelihood of difficult intubation, since the upper
airway abnormalities associated with OSA (increased neck circumference, large tongue,
decreased cross-sectional area of the upper airway) may also predispose to difficult intubation.
Hypercapnia associated with severe OSA can lead to right-heart failure. OSA is associated with
increased perioperative complications including cardiac arrhythmias, hypertension, myocardial
ischemia, respiratory failure, and stroke. Supine positioning and sedative agents make the upper
airway even more prone to obstruction. Thus, patients with OSA may require CPAP in the
immediate postoperative period.

13. B. Preoperative preparation is essential for caring for the obese patient. Perioperative
concerns include difficult intravenous access, possible need for arterial blood pressure
monitoring, positioning, difficult endotracheal intubation, and appropriate dosing of
medications. Nondiabetic obese patients are not at increased risk of aspiration of gastric
contents, as they may have smaller gastric fluid volumes at higher pH than do lean nondiabetic
patients. However, obesity may increase the risk of a difficult laryngeal intubation, especially in
males and patients with a higher Mallampati score. Placement of the patient in the reverse
Trendelenburg position during intubation is advantageous because it reduces atelectasis,
increases time to oxygen desaturation after preoxygenation, and moves the chest and abdominal
tissue caudally to allow easier access to the mouth for endotracheal intubation. Obese patients
have a smaller volume of distribution for water-soluble drugs. Thus, dosing of these drugs
should be based on ideal body weight to avoid overdosing. Larger fat stores provide an
increased volume of distribution for lipid-soluble drugs. For lipid-soluble drugs, while a
loading dose should be based on actual body weight, clearance w ill be slower because of the
larger volume of distribution, and thus, maintenance doses should be administered less
frequently.

14. D. Refeeding syndrome can occur in malnourished patients who are acutely fed (either
enterally or parenterally). It is caused by increased adenosine triphosphate production and
metabolic rate. Hypophosphatemia is the hallmark biochemical feature of refeeding syndrome.
Other metabolic and electrolyte disturbances may include abnormal sodium and fluid balance;
hypokalemia; hypomagnesemia; thiamine deficiency; and changes in glucose, protein, and fat
metabolism. Refeeding syndrome can be avoided by slowly increasing the nutritional intake
toward caloric goals.

15. C. Cricoid pressure can be associated with several complications. These complications are
more likely in the elderly, children, pregnant women, patients with cervical injury, patients with
difficult airways, and when there is difficulty palpating the cricoid cartilage. The technique
involves the application of backward pressure on the cricoid cartilage to occlude the esophagus
and thus prevents the aspiration of gastric contents during induction of anesthesia. However,
strong downward pressure can also displace an unstable cervical spine and worsen visualization
of the airway by occluding the glottis. In contrast, parturients may need more pressure to
effectively occlude the esophagus.

16. A. The primary goal of intraoperative blood sugar management is to avoid hypoglycemia.
The most common perioperative management regimen consists of giving the patient a fraction
(usually half) of the morning intermediate-acting insulin dose. If hypoglycemia is a concern, an
infusion of dextrose may be started. Short-acting insulin preparations are held because of an
increased risk of hypoglycemia and their short duration of action. Metformin has a duration of
action of 6 to 24 hours (up to 48 hours with the extended release formulation). While it was
previously recommended that metformin be discontinued 48 hours preoperatively to avoid risk
of fatal lactic acidosis, more recent data suggest that this risk is low. The optimal level of
glucose control in the perioperative setting remains controversial. The American Association
of Clinical Endocrinologists (AACE) and the American Diabetes Association (ADA)
recommend keeping blood glucose between 140 and 180 mg/dL in critically ill patients. For
noncritically ill patients treated with insulin, premeal glucose targets should generally be <140
mg/dL and random blood glucose values should be <180 mg/dL. The NICE-SUGAR trial in
critically ill patients showed an increased mortality and increased incidence of severe
hypoglycemia in patients randomized to intensive glucose control (target glucose range 81-108
mg/dL).

17. D. Carcinoid tumors are slow-growing tumors that secrete serotonin, kallikrein, and
histamine. Excess serotonin secretion can result in carcinoid syndrome, which is characterized
by diarrhea, flushing, palpitations, and bronchoconstriction. However, most patients with
carcinoid tumors are not symptomatic because the liver detoxifies the excess serotonin. Patients
are symptomatic if they have tumors arising outside of the hepatic portal venous system or
when liver metastatic disease has compromised hepatic synthetic function. The sclerosing effect
of serotonin on the tricuspid and pulmonary valves can result in right-heart failure. The left
heart is generally not affected because of lung metabolism of serotonin. Preoperative
echocardiography should be considered in patients with carcinoid syndrome.

18. A. Patients with a serum calcium >14 mg/dL should be managed with saline and diuresis to
decrease their calcium level. Neuromuscular-blocking agents should be titrated carefully as
severe hypercalcemia can result in muscle weakness. Prolonged hypercalcemia can result in
osteoporosis and risk of vertebral compression fractures with laryngoscopy and bone fractures
during transport. Hypoventilation should be avoided as acidosis increases ionized calcium
levels.

19. C. Hypersecretion of aldosterone results in increased sodium reabsorption in the distal renal
tubule in exchange for potassium and hydrogen ions. This results in fluid retention,
hypertension, metabolic alkalosis, hypokalemia, and muscle weakness.

20. B. Adults normally secrete 20 to 30 mg of cortisol daily. This may increase to over 300 mg
under conditions of stress.

21. C. Patients who have received the equivalent of 5 mg of prednisone or more for a period of
more than 2 weeks within the previous 3 months may not be able to respond appropriately to
surgical stress due to adrenal suppression. These patients should receive perioperative steroid
replacement therapy. The dose of steroids needed is controversial though. One recommended
approach is to give a dose between 1 and 5 times the daily cortisol production (no more than
100 to 150 mg of cortisol equivalent) per day, beginning at the time of surgery and taper the
replacement over 48 to 72 hours.

22. A. Cushing syndrome is characterized by muscle weakness/wasting, glucose intolerance,


hypertension, hypokalemia, weight gain, hypercoagulability, and osteoporosis.

23. D. Intraoperative management of pheochromocytoma resection includes avoidance of drugs


(e.g., ketamine, ephedrine) or techniques that may stimulate the sympathetic nervous system.
Intubation should be performed after a deep level of anesthesia is achieved and hypoventilation
should be avoided. Despite adequate preoperative a- and ^-blockade, hypertension may still
occur. These should be treated with short-acting, easily titrated agents such as nitroprusside or
nicardipine. Phentolamine may also be useful because it blocks a-adrenergic receptors.
Magnesium infusions have been shown useful in managing hypertension by inhibiting
catecholamine release and by altering adrenergic receptor response. Patients with
pheochromocytomas are often hypovolemic and become hypotensive, and hypoglycemic (lack
of catecholamine-induced glucose synthesis) after tumor ligation and resection.

24. B. While all of the above may cause hypotension on induction of anesthesia, the most likely
cause in this patient is diabetic autonomic neuropathy. Diabetic patients with hypertension,
longstanding diabetes, coronary artery disease, and old age are more likely to have autonomic
dysfunction. Patients with autonomic neuropathy are unable to compensate for intravascular
volume changes with an increased heart rate, and thus are more likely to have hemodynamic
instability and even sudden cardiac death. This risk is increased by concomitant use of P-
blockers, angiotensin-converting enzyme inhibitors, and angiotensin-receptor blockers.

25. C. Limited jo in t m obility syndrome is due to glycosylation of tissue proteins due to chronic
hyperglycemia. It is characterized by hand stiffness, though other joints (wrists, elbows, feet,
spine) may be involved. Involvement of the temporomandibular jo in t and the cervical spine can
result in difficult endotracheal intubation.
Ophthalmic, Ear, Nose, and Throat Surgery
Thoha Pham

1. The most accurate statement regarding absorption of topically administered ophthalmic drugs
is that they are absorbed
A. Slower than subcutaneous absorption
B. Faster that intravenous absorption
C. Similar to oral absorption
D. Slower than intravenous absorption

2. Drainage of aqueous humor occurs at all of these sites, except


A. Canal of Schlemm
B. Trabecular network
C. Episcleral venous system
D. Tear ducts

3. The normal intraocular pressure (IOP) is _______(mm Hg):


A. 5
B. 10
C. 25
D. 30

4. Correct consequence of respiratory variables on intraocular pressure (IOP) is


A. Decrease in Pao2 w ill decrease IOP
B. Increase in Pao2 w ill decrease IOP
C. Decrease in Paco2 w ill increase IOP
D. Increase in PacO2 w ill increase IOP

5. A ll of the follow ing w ill serve to decrease intraocular pressure (IOP), except
A. Nitrous oxide
B. Acidosis
C. Morphine
D. Vecuronium
6. Increases in intraocular pressure (IOP) follow ing succinylcholine administration for tracheal
intubation can be minimized by all of the following, except
A. ^-Adrenergic blocker
B. Nondepolarizing relaxant
C. Detachment of extraocular muscles from the globe
D. Lidocaine

7. The ocular effects of ketamine includes


A. Pupillary constriction
B. Blepharospasm
C. Decrease in intraocular pressure
D. Myoclonus

8. An 82-year-old female patient who resides in a nursing home facility presents for breast biopsy.
She states that she uses eye drops to treat glaucoma, but does not know exact names. Patient
denies other medical issues, however states that she frequently has acid reflux. Potential
anesthetic considerations as a result of eye drops include all of the following, except
A. Hyperchloremic metabolic acidosis
B. Hypokalemic metabolic acidosis
C. Prolonged neuromuscular block with succinylcholine
D. Atropine-resistant bradycardia

9. An air bubble is injected into the posterior chamber at the conclusion of retinal surgery
(pneumatic retinopexy) to facilitate anatomically correct healing. The most appropriate
anesthetic management, before the air bubble is injected, is
A. Increase depth of anesthesia
B. Discontinue nitrous oxide (N2O)
C. Ensure adequate muscle relaxation
D. Hyperventilate the patient

10. Compared with air, sulfur hexafluoride (SF6) bubble injected follow ing vitreous surgery

A. Has a longer duration of action


B. Is more soluble in blood than nitrogen
C. Is inert and w ill not expand
D. Is contraindicated in outpatient surgery

Questions 11 to 14
A 22-month-old 14.5-kg “ preemie” is undergoing strabismus repair under general endotracheal
anesthetic (GETA). Following an uneventful inhaled induction with sevoflurane, peripheral IV was
obtained, and by oversight, patient was given 20 mg of succinylcholine prior to intubation. Masseter
spasm was noted moments later.
11. What parameter is considered the earliest sign and symptom of an ensuing hyper metabolic state
follow ing succinylcholine administration?
A. Hyperthermia
B. Hypotension
C. EtCO2 increase
D. Low oxygen saturation

12. Midway through the surgery, when surgical traction in the operative field is applied, patient’s
heart rate plummets from 110 bpm down to 55 bpm. The pairing that accurately reflects the
afferent and efferent limbs, respectively, of this reflex is
A. Trigeminal nerve vagus nerve
B. Optic nerve vagus nerve
C. Vagus nerve trigeminal nerve
D. Trochlear Nerve optic nerve

13. The most appropriate first step in the management of this hemodynamic instability is
A. Epinephrine
B. Atropine
C. Remove traction
D. Phenylephrine

14. At the conclusion of the surgery, postoperative nausea and vomiting should be anticipated and
can be minimized by all of the following, except
A. Serotonin (5-HT3) antagonist
B. Propofol infusion
C. Lim iting opioids
D. Deep extubation

15. The true statement regarding an oculocardiac reflex is


A. It does not occur in enucleated patients
B. Incidence is increased in the setting of hypercarbia
C. Intensity increases with repeated stimulation
D. Suppressed by general anesthesia

16. A ll of the follow ing anatomic structures may participate in triggering an acute and abrupt
bradycardia during ophthalmic surgery, except
A. Trigeminal nerve
B. Vagus nerve
C. Globe
D. Optic nerve
17. Appropriate anesthetic management for ophthalmic surgery requires tight control of
intraocular pressure (IOP) before, during, and after the procedure. The accurate effect of an
anesthetic drug or maneuver on IOP is
A. Decreased by glycopyrrolate
B. Increased by hyperventilation
C. Decreased by nitrous oxide
D. Increased by nondepolarizing muscle relaxants

18. A ll these nerves can be disrupted by injection of local anesthetics into the retrobulbar space,
except
A. Optic nerve
B. Oculomotor nerve
C. Trochlear nerve
D. Abducens nerve

19. The eye movement that is preserved, or unaffected, follow ing a retrobulbar block with 0.5%
bupivacaine is
A. Abduction
B. Rotation
C. Adduction
D. Elevation

20. Possible complications of a retrobulbar block include all the following, except
A. Central retinal artery occlusion
B. Oculocardiac reflex
C. Puncture of the globe
D. Horner syndrome

Questions 21 to 22
A patient is given propofol 20 mg intravenously just before placement of a retrobulbar block (0.5%
bupivacaine— 3 mL) to provide ocular akinesia for ocular surgery.

21. As the surgeon attempts to place a lid speculum, the patient squints, preventing adequate
placement. Additional blockade of which muscle can provide additional akinesia?
A. Orbicularis oculi
B. Temporalis
C. Zygomaticus minor
D. Levator anguli oris

22. Moments later, apnea occurs followed by complete loss of consciousness. The most likely
etiology to explain this event is
A. Subarachnoid injection of local anesthetic
B. Effects of propofol
C. Oculocardiac reflex
D. Intravenous injection of local anesthetic

Questions 23 to 27
A 57-year-old otherwise-healthy male was leaving a dinner party when he was involved in a rollover
car accident during which a foreign object became lodged into his right eye. He is taken to the OR for
emergent surgical repair of a penetrating wound to his right globe.

23. The most appropriate anesthetic plan to consider is


A. Retrobulbar block followed by monitored anesthesia care (MAC)
B. IV induction of general anesthesia avoiding muscle relaxants
C. Rapid-sequence induction of anesthesia using large dose rocuronium
D. Secure the airway with an awake fiberoptic intubation

24. Anesthetic strategies that can minimize intraocular pressure (IOP) increase and lessen his risk
of ocular extrusion include all of the following, except
A. Ketamine
B. Hyperventilation
C. Inhaled volatile agent, 2.0 MAC
D. Controlled hypotension

25. Fifteen minutes after the start of surgery, while the surgeon is retracting the medial rectus
muscle, the patient becomes hypotensive and bradycardic. The first-line therapy to address this
cardiovascular derangement is
A. Atropine 1 mg IV
B. Phenylephrine 100 |ig IV
C. Ask the surgeon to stop
D. Glycopyrrolate 1 mg IV

26. The patient’s vital signs normalize and anesthesia is maintained with desflurane and nitrous
oxide. Later in the case, conjunctival instillation of a phenylephrine (10%) solution results in
immediate escalation of blood pressure from 105/70 to 220/115 mm Hg, while his pulse falls
from 86 to 35 bpm. The ECG reveals new onset of ectopic ventricular complexes. The most
appropriate treatment option at this time is
A. Ask the surgeon to stop
B. Administer nitroprusside
C. Administer atropine
D. Discontinue nitrous oxide

27. At the conclusion of the surgery, patient is extubated and brought to the recovery room (PACU)
in a stable condition. Thirty minutes later, when he is more awake, he notes unilateral eye
discomfort in the nonsurgical eye. He has associated tearing, conjunctivitis, photophobia, and
pain, which is worsened with blinking. These eye symptoms are most likely caused by
A. Retinal hemorrhage
B. Oculogyric crisis
C. Angle-closure glaucoma
D. Corneal abrasion

28. True statement regarding laryngospasm is


A. Associated risk of pulmonary edema
B. The false vocal cords do not spasm
C. Mediated through the recurrent laryngeal nerve
D. Increased risk of aspiration

29. A patient in the intensive care unit (ICU) with pulmonary failure requires tracheal intubation.
Compared with nasotracheal intubation, oral tracheal intubation carries a higher incidence of
A. Patient discomfort
B. M axillary sinusitis
C. Tr ansient bacter emia
D. Otitis media

30. When compared to an adult, the airway anatomy of a 6-week-old infant reveals
A. Tongue is smaller and floppy
B. Airway is narrowest at the glottic opening
C. Position of the larynx is more anterior in the neck
D. Epiglottis is flat and firm

Questions 31 to 32
A 3-year-old patient arrives for rescheduled tonsillectomy and adenoidectomy with another acute
upper respiratory tract infection (URI). Her initial surgery was postponed 3 weeks ago as she had a
URI at that time as well. Exam reveals a runny nose with greenish-yellow discharge with an
intermittant wet cough. She is afebrile with normal vital signs.

31. Postponement of surgery w ill reduce the risk of


A. Laryngospasm
B. Hemorrhage
C. D ifficult intubation
D. Gastroesophageal reflux

32. Surgery proceeded without incident; however, 2 hours later in the recovery room (PACU), she
vomits a large blood clot followed by ongoing bleeding. She appears pale and anxious. Vitals
reveal heart rate = 130 bpm, respiratory rate = 25 bpm, and blood pressure = 77/35 mm Hg. Her
capillary re fill time is 4 seconds. The most appropriate next step in management at this time is
A. Insertion of orogastric tube to empty the stomach of blood
B. Emergent return to the operating room
C. Administer anxiolysis medication
D. Provide liberal fluid resuscitation

Questions 33 to 35
A 65-year-old male requires transoral laser microsurgery to address his laryngeal webs. His medical
history reveals remote tobacco smoking and recreational drug use in college.

33. Minimizing airway fire hazards associated with laser surgery can be accomplished by use of all
of the following, except
A. Intermittent mode laser emissions
B. An air/oxygen anesthetic technique
C. A polyvinylchloride (PVC) endotracheal tube
D. Saline-soaked sponges over exposed tissues

34. Ten minutes later, the surgeon yells “ FIRE!” The most appropriate next step is to
A. Ventilate with air
B. Increase Fio2 to 1.0
C. Instill saline down the endotracheal tube lumen
D. Remove the endotracheal tube

35. One hour later while recovering in the PACU, the patient is noted to have stridor and difficulty
breathing. At this time, the most appropriate next step in his airway management includes
A. Administration of aerosolized epinephrine
B. Endotracheal intubation
C. Administration of helium and oxygen
D. Intravenous injection of dexamethasone

36. A 10-year-old g irl with hoarseness presents for laser microsurgery to address laryngeal
papillomas. She is otherwise healthy. The surgeon is requesting a general endotracheal
anesthetic (GETA). The gas mixture least likely to support combustion is
A. Oxygen 35%, air 65%
B. Oxygen 30%, helium 70%
C. Oxygen 20%, nitrous oxide (N2O) 80%
D. Oxygen 30%, nitrogen (N2) 70%

37. A 55-year-old woman with a 35 pack-year history of tobacco smoking is undergoing


laryngobronchoscopy utilizing the Sanders jet ventilation technique. The principle behind
apneic oxygenation is
A. Contrasting density of inhaled gases
B. Maintenance of spontaneous ventilation
C. A ir entrainment
D. Use of helium-oxygen mixtures

38. During apneic oxygenation via a rig id bronchoscope, anesthetic considerations include all of
the following, except
A. Duration of the procedure is limited by the increase in carbon dioxide
B. Denitrogenation should be performed prior to apnea
C. Paco2 remains unchanged for the first 15 minutes
D. Functional residual capacity and body weight influence the rate of desaturation

Questions 39 to 43
A 35-year-old male with a toxic multinodular goiter presents for thyroidectomy with radical neck
dissection. He denies any other significant medical history. Review of systems reveals orthopnea and
dysphagia with a recent change in the caliber of his voice.

39. True statements about this patient include all of the following, except
A. A flow-volum e loop on spirometry can evaluate tracheal compression
B. The airway may obstruct with sedation
C. The trachea may collapse postoperatively
D. An abnormally low forced expiratory volume in 1 second (FEV1) would be diagnostic of
an upper airway obstruction

40. To attenuate risk of a “ cannot ventilate, cannot intubate” scenario, an awake airway intubation is
discussed. The neural structure that does not need to be blocked in order to provide adequate
airway analgesia for a nasal intubation is
A. Hypoglossal nerve
B. Sphenopalatine ganglion
C. Superior laryngeal nerve
D. Recurrent laryngeal nerve

41. At the conclusion of a complicated 4-hour resection, the patient is extubated and brought to the
recovery room. One hour after extubation, the patient complains of dyspnea with stridorous
respiration. Initial steps include all of the following, except
A. Intravenous administration of calcium
B. Nebulized racemic epinephrine
C. Inspection of the surgical site
D. Direct laryngoscopy

42. If bilateral recurrent laryngeal nerves were unintentionally severed, the likely finding on direct
laryngoscopy would be
A. Paralysis of the cricothyroid muscles
B. Intermediate position of the cords
C. Midline, closed position of the cords
D. Pure adductor vocal cord paralysis

43. Instead, postoperative direct laryngoscopy reveals normal position of the cords at rest, widely
open glottic opening at maximal inspiration, and symmetrically moving cords during quiet
breathing but with weak phonation and inability to speak loudly or shout. The most likely
etiology is
A. Recurrent laryngeal nerve paralysis
B. Superior laryngeal nerve (SLN) paralysis
C. External airway compression
D. Vagus nerve paralysis

Questions 44 to 45
A 27-year-old male arrives to the operating room with laryngotracheal injuries stemming from a
motorcycle collision. He presents with hoarseness and dyspnea while sitting, but is unable to lie flat
due to worsening dyspnea. He is unable to swallow, and is drooling/spitting moderately blood-stained
sputum. His anterior neck is diffusely swollen and exquisitely tender with notable subcutaneous
emphysema. Oxygen saturation is 100% with supplemental oxygen via face mask. Review of imaging
reveals a thyroid cartilage fracture horizontally and crossing the midline.

44. The most appropriate approach to his airway management is


A. Tracheostomy
B. Laryngeal mask airway
C. Nasotracheal intubation
D. Cricothyroidotomy

45. His injury would be consistent with trauma to this zone of his neck:
A. Zone I
B. Zone II
C. Zone III
D. Zone IV

Questions 46 to 47
During thyroidectomy for carcinoma, a 22-year-old patient develops tachycardia to 115 bpm while
blood pressure intensifies to 145/100 mm Hg. The inhaled anesthetic is deepened and minute
ventilation is increased. Thirty minutes later, tachycardia and hypertension persists despite all efforts
(Table 16-1).

Table 16-1
P r e -o p 15 M in u te s 3 0 M in u te s

H eart Rate (bpm ) 72 115 134


B lood p ressure (m m Hg) 118/56 145/100 174/108
0 2 S aturation (%) 99 (room air) 97 (Fio 2 = 0.60) 94 (Fio 2 = 1.0)
E tC 0 2 (m m Hg) - 48 69
T em perature (°C) 36.6 38.5 41.8
E lectrocardiogram N o rm a l sinus rh y th m Sinus tachycardia Trigem iny

46. The appropriate treatment to consider at this time is


A. Propranolol
B. Acetaminophen
C. Iodine
D. Dantrolene

47. Diagnosis of malignant hyperthermia is most commonly confirmed by


A. Caffeine halothane contracture test (CHCT)
B. Urinalysis
C. Arterial blood gas
D. Core temperature >42°C
CHAPTER 16 ANSWERS

1. D. Topically applied drops are quickly absorbed by the mucosal lining of the nasolacrimal
duct as well as by blood vessels in the conjunctival sac with a potential to produce systemic
effects. Absorption is rapid, faster than oral or subcutaneous administration, but still slower
than intravenous.

2. D. Intraocular pressure (IOP) is a reflection of the eye’s ability to form and drain aqueous
humor. The posterior chamber’s ciliary body is the major producer of aqueous humor.
Obstruction of the drainage system, whether it is at the canal of Schlemm, the trabecular
network, or the episcleral venous system, w ill elevate IOP. Tear ducts do not contribute to the
drainage of aqueous humor.

3. B. Normally, IOP of the eye varies between 10 and 22 mm Hg, and is generally considered
abnormal when >25 mm Hg. This pressure is not static, as it can vary by 1 to 2 mm Hg with
each cardiac contraction. Diurnal variations of up to 5 mm Hg also exist, with a higher pressure
noted upon awakening.

4. D. Hypoventilation (tPaco2) along with hypoxemia (iPao2) w ill result in increased IOP,
whereas hyperventilation (lPacO2) w ill serve to minimize choroidal blood flo w to decrease
IOP. Hyperoxemia (TPaO2) does not affect IOP significantly.

5. B. Inhaled and injected anesthetics (with the exception of ketamine) along with opioids tend
to lower IOP. Nondepolarizing muscle relaxants w ill decrease IOP, presumably via their
relaxant effects on extraocular muscles. Hypoventilation (TPacO2) results in respiratory
acidosis, which w ill increase IOP (Table 16-2).

Table 16-2 Factors Affecting Intraocular Pressure (IOP)


• H ypertension (sym pathetic stim ulation) as occurs during laryn goscop y and intu
• A cid osis
Increased IOP
• H ypoxia
• Increased central venous pressure (co u g h in g , valsalva m aneuver)
• H ypotension
• Inhalational anesthetics (volatile and nitrous oxide)
Decreased IOP
• O pioids
• N ondepolarizing m u scle relaxants

Table 16-3 Signs of Malignant Hyperthermia


E a r ly L ate

Increased EtCO2 Hyperthermia


Tachycardia Elevated creatine phosphokinase
Skeletal muscle spasm/rigidity Myoglobinuria
Tachypnea Cyanosis
Sweating Disseminated intravascular coagulation
Acidosis—respiratory and metabolic Cardiac arrest

6. C. The use of succinylcholine for eye surgery is controversial. Succinylcholine can increase
IOP by about 5 to 10 mm Hg for about 5 to 10 minutes after intravenous administration (longer
duration of HOP follow ing intramuscular administration). Pretreatment with nondepolarizing
muscle relaxants, lidocaine, or P-blockers may reduce the ocular hypertensive response to
minimize increases in IOP. The increase in IOP after succinylcholine persists whether or not the
extraocular muscles are intact, suggesting that cycloplegic effects, rather than physical
contraction, are responsible for IOP elevation.

7. B. Ketamine may cause nystagmus and blepharospasm and may not be suitable for
ophthalmic surgery. Studies with respect to the effect of ketamine on intraocular pressure (IOP)
have shown conflicting results, but it appears more likely to increase, as opposed to decrease,
ocular pressures. This may depend on whether ketamine is administered through the IM or IV
route. Ketamine is not known to affect pupil size. Myoclonus is commonly associated with
etomidate and likely should also be avoided when IOP control is essential.

8. A. Topical ophthalmic medications undergo sufficient and prompt absorption to produce


systemic effects and may cause adverse cross-reactions to medications used in routine
anesthesia care. Acetazolamide drops, due to its action as a carbonic anhydrase inhibitor, can
induce a hypokalemic metabolic acidosis. Topical echothiophate iodine, an irreversible
cholinesterase inhibitor, can reduce plasma cholinesterase activity, prolonging the duration of
action of succinylcholine and mivacurium. Absorption of tim olol, a nonselective P-adrenergic
blocker has been associated with atropine-resistant bradycardia, hypotension, and
bronchospasm during general anesthesia. Hyperchloremic acidosis is largely related to large
volume resuscitation with normal saline.

9. B. In the presence of N2O, air bubbles w ill increase in size as N2O is 35 times more soluble
compared to molecular nitrogen (the major component of air), allowing it to diffuse into an air
bubble more rapidly than nitrogen is absorbed out of the bubble. If the bubble expands after the
incision is closed, intraocular pressure w ill rise. This complication can be avoided by
discontinuing N2O at least 15 minutes prior to the bubble injection, as the washout of N2O from
the lungs is 90% complete within 10 minutes. Additionally, repeat general anesthesia with N2O
should be avoided until the bubble is fu lly absorbed, which for air can take up to 5 days.

10. A. SF6 is an inert gas that is much less soluble than nitrogen (the major component of air) in
blood and, therefore, w ill have a longer duration of action (10 days) compared to an air bubble.
Bubble size doubles within 24 hours after injection of SF6 because nitrogen from inhaled air
w ill enter more rapidly into the bubble than sulfur can diffuse out of it. This slow bubble
expansion usually does not pathologically affect IOP. However, inspired N2O, which is 117
times more diffusible than hexafluoride (compared to 35 times more than nitrogen), w ill
rapidly enter the SF6 bubble such that IOP w ill rise significantly within 30 minutes after the eye
is closed. As with air, repeat general anesthesia with N2O should be avoided until the SF6 bubble
is fu lly resorbed.

11. C. Although still quite rare, an increased incidence of malignant hyperthermia (MH) has been
reported in patients with strabismus (underlying myopathy) such that a high index of suspicion
should be maintained. EtCO2 is considered the earliest indicator of a hypermetabolic state with
unexpected increases in CO2 despite constant minute ventilation. Avoiding known triggers can
negate the risk of inducing MH, such that succinylcholine is not recommended during
strabismus surgery involving infants and children.

12. A. Trigeminovagal reflex: the afferent lim b of the oculocardiac reflex is via the trigeminal
nerve (CN V), prim arily through the ophthalmic division (V1). The impulse travels along the
long and short ciliary nerves (LCN and SCN) to synapse on the ciliary ganglion. The impulse
then continues through the trigeminal ganglion arriving at the sensory nucleus of the trigeminal
nerve. The convergence between the afferent and efferent limbs is at the motor nucleus of the
vagus nerve (CN X) of the brain stem. From here, the efferent lim b is via the vagus nerve,
which eventually synapses on the sinoatrial node of the heart, resulting in an abrupt bradycardia
(Fig 16-1).

Figure 16-1.

13. C. The oculocardiac reflex (OCR) occurs frequently during strabismus surgery. It can occur
follow ing traction of the extrinsic eye muscles, or placement of pressure on the globe. The
OCR is most commonly manifested as bradycardia, which regresses almost immediately after
the stimulus is removed. Bigeminy, ectopy, nodal rhythms, atrioventricular block, and cardiac
arrest have also occurred. Traction on any of the extraocular muscles can evoke this reflex, but
it appears that manipulation of the medial rectus muscle is the most consistent trigger. Though
the prophylactic use of an anticholinergic (atropine or glycopyrrolate) before the potential
evoking stimulus may be recommended, the most effective treatment is the removal of the
stimulus.

14. D. The incidence of nausea and vomiting follow ing strabismus surgery can be high, ranging
anywhere from 48% to 85%. Minimizing the use of opioids, substituting propofol for inhaled
anesthetics, along with the prophylactic use of antiemetics can reduce nausea and vomiting after
surgery. Deep extubation has no impact on postoperative nausea and vomiting, and may place
patient at risk for aspiration.

15. B. The afferent lim b of the oculocardiac reflex (OCR) is the trigeminal nerve such that
pressures on the globe, conjunctiva, or orbital structures and traction on the extraocular
muscles are potential triggers. This reflex occurs even with an empty globe. Hypercarbia and
hypoxemia are factors believed to augment the incidence and severity of the reflex. This reflex
is noted to fatigue with repeated stimulation and is not suppressed by general anesthesia.

16. D. The afferent lim b of the oculocardiac reflex is the trigeminal nerve such that triggers
include pressure on the globe, conjunctiva, or orbital structures as well as traction of the
extraocular muscles. The vagus nerve is the efferent lim b with connections to the sinoatrial
node triggering a reflex bradycardia. The optic nerve is not involved in this reflex activity.

17. C. Anticholinergics (e.g., glycopyrrolate) may include mydriasis of the pupils, leading to an
increase in intraocular pressure. Unlike atropine, however, glycopyrrolate is completely
ionized at physiologic pH; thus, the occurrence of CNS-related side effects is lower, as it has
difficulty crossing the blood-brain barrier. Anesthetic agents, whether inhaled or injected,
reduce IOP, with the possible exception of ketamine. Nondepolarizing neuromuscular-blocking
agents produce a slight decrease, while depolarizing relaxants increase IOP. Hyperventilation
w ill cause vasoconstriction with decrease in choroidal blood flo w and intraocular pressures.

18. C. Nerves blocked are those within the optic cone (annulus of Zinn), which include optic (CN
II), oculomotor (CN III), and the abducens (CN VI). The trochlear nerve (CN IV) is not affected,
since it is located outside of this muscle cone.

19. B. The trochlear nerve (CN IV) remains intact follow ing a retrobulbar block, since it is
located outside of the muscle cone. The trochlear nerve innervates the superior oblique muscle;
thus, rotational movement of the eye remains intact.

20. D. Common complications attributed to a retrobulbar block include retrobulbar hemorrhage


with possible central artery occlusion, oculocardiac reflex, puncture of the posterior globe,
penetration of the optic nerve, and inadvertent intrathecal injection. Horner syndrome is not
commonly seen follow ing retrobulbar blocks; instead, it results from an interruption of the
sympathetic nerve supply to the head/face, resulting in a triad of miosis, ptosis, and anhidrosis.

21. A. Blockade of the orbicularis oculi muscle, which is a sphincter muscle around the eye, can
further provide adequate surgical conditions for any ocular procedure with consequent inability
to squeeze the lids shut. This can be achieved by blockade of the facial nerve (CN VII).

22. A. There is 1% to 3% risk of complications with retrobulbar block, ranging from m ild to
severe. Possible complications include accidental subarachnoid injection, which can cause a
“ total spinal” leading to apnea, unconsciousness, and cardiorespiratory collapse.

23. C. Eye injuries commonly occur as a result of trauma, which frequently means providing
emergent general anesthesia for patients with fu ll stomachs. It is important to avoid any sudden
increases in intraocular pressure (IOP) that may cause extrusion of the ocular contents.
Although awake tracheal intubation provides the greatest margin of safety to prevent aspiration,
it may in fact promote increase in IOP with inadequate orotracheal anesthesia. Placement of a
retrobulbar block is not advised as inadvertent globe puncture may lead to extrusion of orbital
contents. For most cases, rapid-sequence or modified rapid-sequence induction is utilized. The
choice of succinylcholine offers the advantage of rapid onset of muscle relaxation, but may
acutely cause elevation in IOP. Alternatively, the use of a large dose of nondepolarizing
neuromuscular-blocking agent w ill reduce IOP and facilitate tracheal intubation as long as
adequate blockade is confirmed prior to laryngoscopy.

24. A. Hyperventilation, hypotension, and hypothermia decrease IOP, whereas arterial hypoxemia
and hypoventilation elevate IOP. External pressure can also be generated by venous congestion
of orbital veins, which is accentuated during a valsalva, coughing, and vomiting. Additionally,
most inhaled and injected anesthetics (with the exception of ketamine) can also serve to reduce
IOP.

25. C. The oculocardiac reflex best explains this cardiovascular presentation during
ophthalmologic surgeries. First-line therapy is always to remove the stimulus, which is
mediated via trigeminal afferents.

26. B. Hypertensive episodes during anesthesia should be tackled logically. Common causes are
light anesthesia, hypoxia, and hypercarbia. In this case, excessive systemic uptake of the
phenylephrine precipitated severe hypertension. Elevated diastolic pressures with ECG
pathology necessitate immediate action to prevent further cardiovascular decline.
Administration of sodium nitroprusside is beneficial to quickly reduce the blood pressure and
decrease cardiac afterload.

27. D. Corneal abrasions produce a foreign body sensation with associated tearing, conjunctivitis,
and photophobia. This pain is made worse by blinking. Protection against this occurrence
includes application of nonionic petroleum-based ophthalmic ointment to the eye, securely
taping the eyelids shut during anesthesia, and discouraging patients from rubbing their eye on
emergence. Abrasions can be diagnosed by fluorescein staining, and treatment options include
saline flushes, antibiotic ointment, and patching the eye.

28. A. Laryngospasm can complicate any routine airway management and is especially prevalent
around the time of extubation. It often occurs during stage 2— “ excitement stage” — of general
anesthesia in combination with an airway irritant such as blood, mucus, laryngoscope blade,
suction catheter, surgical debris, or other foreign objects. This protective reflex is mediated by
the superior laryngeal nerve and manifested as sustained closure of the glottis. Laryngospasm
with complete airway obstruction can be associated with negative pressure pulmonary edema,
as patients can create a significant amount of negative intrathoracic pressure during attempts to
breathe against an obstructed upper airway. The management consists of positive pressure
ventilation, increasing the depth of anesthesia, and occasionally a small dose of a muscle
relaxant with or without reintubation.

29. A. Tracheal intubation to facilitate mechanical ventilation is common in ICU patients to


appropriately manage failure of adequate spontaneous ventilation and/or oxygenation. Both
nasal and oral tracheal tubes are relatively safe, for at least several weeks, while patients
convalesce. When compared with prolonged oral intubation, nasotracheal intubation may be
more comfortable for the patient, more secure (fewer occurrences of accidental self-
extubations), and less likely to cause laryngeal damage. Nasal intubation, however, has its own
significant adverse events, including significant nasal bleeding, transient bacteremia, sinusitis,
and otitis media (from obstruction of the auditory tubes).

30. C. Recognizing the anatomical differences between an adult and a pediatric airway is
important. One of the most obvious differences is the tongue itself. The pediatric tongue is
larger, in relation to the amount of free space in the oropharynx, when compared to the adult
tongue. With regards to the pediatric epiglottis, it tends to be large and floppy with a more
oblong configuration, making epiglottis control with a laryngoscope blade more challenging.
Additionally, the position of the adult larynx is at about the level C5-C6; the pediatric larynx is
more cephalad, at about the level of the C3-C4. This is an important anatomical airway
consideration, since the higher larynx tends to be more anterior as well (Fig 16-2).

Child's upper airway

Figure 16-2.

31. A. Laryngospasm associated with airway manipulation is more likely to occur in the presence
of a URI such that surgery is typically postponed until resolution of symptoms, typically 1 to 2
weeks. Young children, however, have frequent URIs such that risk:benefit ratio should be
c o n s id e r e d w h e n d e te r m in in g a p p r o p r ia te n e s s o f p r o c e e d in g v e r s u s f u r th e r p o s tp o n e m e n t.

32. D. H e m o r r h a g e f r o m a b le e d in g to n s illa r b e d in th e p o s to p e r a tiv e p e r i o d is a h a z a r d o u s


c o m p lic a tio n . H e r v ita ls r e v e a l h y p o v o le m ia a n d a s s u c h , in itia l m a n a g e m e n t s h o u ld b e to
r e s u s c ita te th e p a tie n t p r i o r to r e tu r n in g to th e o p e r a tin g r o o m to m in im iz e m o r b i d ity
a s s o c ia te d w ith a n e m ia a n d h y p o v o le m ia in th e s e ttin g o f r e p e a t g e n e r a l a n e s th e s ia . A ls o
a s s u m e th a t p a tie n t w ill n o w h a v e a d if f ic u lt a ir w a y w ith a “ fu ll s to m a c h .”

33. C. A n e s th e s ia d u r in g la s e r s u r g e r y m a y b e a d m in is te r e d w ith o r w ith o u t a n e n d o tr a c h e a l tu b e .


If in tu b a tio n is n e e d e d , a p p r o p r ia te la s e r - r e s is ta n t e n d o tr a c h e a l tu b e s s h o u ld b e u tiliz e d . In th is
r e g a r d , r e m e m b e r th a t all P V C tu b e s a r e f la m m a b le a n d c a n ig n ite w h e n c o n ta c te d b y th e la s e r
b e a m . U s in g th e la s e r in te rm itte n tly , v e n tila tin g th e p a tie n t w ith a lo w c o n c e n tr a tio n o f
c o m b u s tib le g a s e s , a lo n g w ith p r o te c tin g a d ja c e n t tis s u e s w ith s a lin e - s o a k e d s p o n g e s a r e a ll
a p p r o p r ia te a p p r o a c h e s to m in im iz e th e f ir e h a z a r d s .

34. D. A ir w a y f ir e s a r e a n in h e r e n t r i s k w ith la s e r s u r g e r y , s u c h th a t a p la n o f a c tio n s h o u ld b e


c o n s id e r e d b e f o r e th e c a s e b e g in s . T h e c u f f o f th e e n d o tr a c h e a l tu b e m a y b e f ille d w ith s a lin e ,
a s o p p o s e d to a ir, to m in im iz e f la m m a b ility s h o u ld th e la s e r b e a m r u p tu r e th e c u ff. I n s p ir e d
o x y g e n c o n c e n tr a tio n is m in im iz e d a s to le r a te d ( u s u a lly F i o 2 o f < 0 .5 0 ), a s o x y g e n r e a d ily
s u p p o r ts c o m b u s tio n . In th e e v e n t o f a n a ir w a y f ir e , th e a n e s th e s ia c ir c u it s h o u ld b e im m e d ia te ly
d is c o n n e c te d to in te r r u p t f u r th e r d e liv e r y o f o x y g e n , f o ll o w e d b y r e m o v a l o f th e tu b e f r o m th e
p a tie n t’s a irw a y . If th e f la m e p e r s is ts , th e f ie ld s h o u ld n e x t b e f lo o d e d w ith n o r m a l s a lin e .

35. B. P o s t a ir w a y f ir e s , it is m o s t a p p r o p r ia te to le a v e th e p a tie n t in tu b a te d f o r c o n tin u e d


o b s e r v a tio n a s th e p r e s e n c e o f la r y n g e a l a n d p h a r y n g e a l e d e m a c a n r e s u lt in f a ile d e x tu b a tio n .
T h e r e f o r e , th is p a tie n t s h o u ld b e r e in tu b a te d w ith a r e g u l a r e n d o tr a c h e a l tu b e a n d m o n ito r e d f o r
th e n e x t 2 4 h o u r s . C o r tic o s te r o id s c a n b e c o n s id e r e d f o r s e v e r e e d e m a w ith a b s e n t c u f f le a k , b u t
g e n e r a lly is n o t g iv e n p r o p h y la c tic a lly .

36. B. T h e m ix tu r e o f g a s e s d e liv e r e d in to th e e n d o tr a c h e a l tu b e m a y a f f e c t th e r i s k o f c o m b u s tio n


d u r in g g e n e r a l a n e s th e s ia a n d la s e r s u r g e r y o f th e a irw a y . N 2O is h ig h ly c o m b u s tib le a n d
s h o u ld b e s tr ic tly a v o id e d . F i o 2 s h o u ld b e r e d u c e d to a s l o w a s p o s s ib le w ith a n a i r - o x y g e n
m ix tu r e . H e liu m , if a v a ila b le , is id e a l a s it is in e r t a n d n o n c o m b u s tib le . T h o u g h N 2 is a ls o
c o n s id e r e d s a fe , th e m e a n tim e to ig n itio n w ith n itr o g e n h a s b e e n f o u n d to b e s ig n if ic a n tly
s h o r te r w h e n c o m p a r e d to th e s a m e c o n c e n tr a tio n o f h e liu m .

37. C. In 1 9 6 0 s, S a n d e rs d e s c r ib e d v e n tila tio n te c h n iq u e u s in g a 1 6 - g a u g e j e t p la c e d d o w n th e s id e


a r m o f a r i g i d b r o n c h o s c o p e , r e ly in g o n a ir e n tr a p m e n t to c o n tin u e o x y g e n a tio n w ith a n o p e n
b r o n c h o s c o p e . A n in te r m itte n t j e t o f o x y g e n a d m in is te r e d f r o m a h ig h - p r e s s u r e s o u r c e (5 0 p s i)
e n tr a in s r o o m a ir to m a in ta in s u p r a n o r m a l o x y g e n c o n c e n tr a tio n s in th e u p p e r a ir w a y s , w h ic h
c r e a te s a d if f u s io n g r a d ie n t to th e a lv e o la r s p a c e s . T h is g r a d ie n t is m a in ta in e d a s a lv e o la r
o x y g e n is c o n s ta n tly c o n s u m e d .
38. C. Apneic oxygenation relies on mass movement oxygenation. With the onset of apnea, a low
pressure develops in the airspace of the lungs, as more oxygen is absorbed (230 mL/min) than
CO2 is released (200 mL/min). If the airways are open, 100% oxygen supplied to the upper
airways w ill fo llo w the pressure gradient and flo w into the lungs, replacing the oxygen
consumed. The uptake of oxygen into the blood w ill then remain at relatively normal levels,
recognizing that the lack of ventilation w ill eventually cause marked hypercapnia and acidosis.

39. D. The configuration of the flow-volum e curve during spirometry testing can be used to
demonstrate abnormalities of the larger central airways (larynx, trachea, and main stem
bronchi). The FEV1/FVC ratio can provide diagnostic value, as disproportionate reduction in
the FEV1 as compared to the FVC is the hallmark of obstructive lung diseases. Concern should
be made regarding airway collapse follow ing sedation or induction of anesthesia when
extrathoracic lesions are present. If long-standing, tracheomalacia may leave the trachea weak
and collapsible postoperatively (Fig 16-3).

40. A. Anesthesia of the nasal mucosa and nasopharynx is achieved via blockade of trigeminal
branches, particularly the sphenopalatine ganglion and ethmoid nerves. Blockade of the
glossopharyngeal and superior laryngeal nerves provide anesthesia to the mouth, oropharynx,
and base of the tongue. The hypopharynx, larynx, and trachea are innervated via a branch of the
vagus nerve (CN X), specifically the recurrent laryngeal nerve, which can be blocked via a
transtracheal approach. On the other hand, blockade of the hypoglossal nerve (CN XII) w ill
only serve to paralyze the intrinsic muscles of the tongue without adding to anesthesia of the
airway (Fig 16-4).
A irw a y innervation

Trigeminal (V) n. Glossopharyngeal (IX) n. Vagus (X) n

Figure 16-4.

41. A. Inspection of the neck is generally considered the first step, as it may reveal a life-
threatening and reversible cause of airway obstruction such as a compressing hematoma. Direct
visualization of vocal cords may point toward recurrent laryngeal nerve damage contributing
to dyspnea. Though hypocalcemia due to removal of the parathyroid glands can occur, signs
and symptoms w ill usually present much later in the perioperative course (24-96 hours), and
unlikely to be contributing to dyspnea in the PACU. Inhaled racemic epinephrine is commonly
used when stridor is present after extubation.

42. B. The recurrent laryngeal nerves provide motor innervation to all the intrinsic muscles of the
larynx, except the cricothyroid muscle, which is innervated by the superior laryngeal nerve.
Damage to bilateral recurrent laryngeal nerves w ill affect abduction and adduction of the cords,
resulting in both vocal cords adopting an intermediate, or paramedian, position. Patient would
also have associated aphonia with risk of airway obstruction with inspiration as the cords flap
together. Unilateral damage w ill present with hoarseness.

43. B. In the case of lesions to the SLNs, adduction and abduction of the vocal cords remain intact.
SLN lesions instead lead to weak tensor strength (cricothyroid muscle), leaving the voice
hoarse, weak, breathy, and with the inability to scream or shout. Other associated findings
would be loss of sensation above the cords, leaving patient vulnerable to inhalation of any
material present in the pharynx.

44. A. Blunt-neck trauma is most commonly a result of a motor vehicle collision associated with
rapid acceleration or deceleration injuries, which may include crushing injuries of the trachea,
esophagus, vascular structures, and cervical spine. A laryngeal fracture can lead to life-
threatening airway obstruction and as such should be treated in an emergent manner. Signs and
symptoms of dyspnea, emphysema, and inability to lie flat reflect a fragile airway. Definitive
airway management follow ing airway trauma is a surgical airway, most commonly a
tracheostomy. Cricothyroidotomy is not recommended follow ing laryngotracheal injuries, as
the landmarks are usually difficult to assess, since the cricoid is often the level of the injury.
45. B. The neck is divided into three zones: zone I, including the thoracic inlet, up to the level of
the cricothyroid membrane, is treated as an upper thoracic injury. Zone III, above the angle of
the mandible, is treated as a head injury. In this case, fracture of the thyroid cartilage represents
an injury of the neck in zone II. For ease of memory, consider that the cricoid cartilage
demarcates the border between zones I and II and the angle of the mandible separates zone II
from zone III (Fig 16-5).

Figure 16-5.

46. D. Increasing EtCO2 and temperature may reveal possible malignant hyperthermia. With onset
of cardiac arrhythmias, and the increasing likelihood of the development of malignant
hyperthermia with rapidly climbing EtCO2 and hyperthermia, treatment with dantrolene should
be considered and pursued. Other signs that can strengthen the diagnosis are muscle rig id ity and
myoglobinuria.

47. A. More than 30 different mutations are linked to malignant hyperthermia susceptibility.
Genetic testing is available to establish a diagnosis, but the CHCT remains the criterion
standard.
Obstetric Anesthesia
Thoha Pham

1. Beyond midgestation, pregnant women are at increased risk of gastroesophageal reflux and
aspiration of gastric contents for all these reasons, except
A. Decreased competence of the lower esophageal sphincter
B. Delayed gastric emptying associated with the onset of labor
C. Delayed gastric emptying due to opioid administration
D. Increased incidence of constipation

2. Changes in the cardiovascular system associated with pregnancy include


A. Increase in central venous pressure
B. Increase in cardiac output
C. Increase in systemic vascular resistance
D. Increase in blood pressure

3. During pregnancy, the disproportionate increase in plasma volume versus erythrocyte volume
accounts for
A. Increase in the mean arterial pressure
B. Increase in stroke volume
C. Increase in cardiac output
D. Relative anemia of pregnancy

4. By the third trimester of pregnancy, cardiac output increases to nearly 50% due to which of
these alterations?
A. Increase in stroke volume and increase in heart rate
B. Decrease in stroke volume and increase in heart rate
C. Increase in stroke volume and decrease in heart rate
D. Decrease in stroke volume and decrease in heart rate

5. The largest increase in cardiac output is seen during this peripartum period:
A. During induction of anesthesia
B. During the start of labor
C. Immediately after delivery
D. A t c o n c e p tio n

6. A 20-year-old G!P0 female at 425 weeks of gestation presents to labor and delivery floor with
rupture of membranes and onset of early labor. She appears uncomfortable and becomes
extremely anxious with peripheral IV placement, and begins to hyperventilate. If allowed to
continue hyperventilation, it w ill cause
A. Increased placental perfusion
B. Decreased maternal arterial pH
C. Increased fetal arterial pH
D. Decreased maternal uterine artery flow

7. In the above patient, labor is nonprogressive with signs of fetal distress on heart rate
monitoring. Spinal anesthesia with 2-chloroprocaine 3% (2 mL) is provided for emergent
cesarean section. On postpartum day 2, she complains of leg numbness, which quickly
progressed to flaccid paralysis. On examination, inability to move her lower extremities with
complete loss of pain and temperature sensation below T4 with normal sensation to light touch
was noted. The most likely cause of this complication is
A. 2-Chloroprocaine neurotoxicity
B. Inadvertent subdural injection
C. Anterior spinal artery syndrome
D. Brown-Sequard syndrome

Questions 8 to 9
A 23-year-old female, in early labor, was transferred from an outside hospital at 37 weeks’ gestation
with a history of a congenital bicuspid aortic valve. The patient reports dyspnea throughout her
pregnancy, and had a recent syncopal event. Subsequently, transthoracic echocardiogram revealed a
mean aortic valve gradient of 45 mm Hg and an aortic valve area of 1.2 cm2.

8. Two hours later, she endorses abdominal pain (8/10) and is requesting analgesia. The most
appropriate option for her pain management during labor and delivery is
A. Spinal anesthetic with bupivacaine
B. Epidural anesthesia with adequate volume preloading
C. Inhaled nitrous oxide
D. Oral analgesics

9. Despite an appropriate increase in her cardiac output and plasma volume, her systemic blood
pressure does not increase during the course of her pregnancy because of
A. Decrease in systemic vascular resistance
B. Compression of the vena cava
C. Decrease in venous capacitance
D. Decrease in heart rate
10. Iatrogenic contributions to maternal supine hypotension syndrome can be minimized by
A. Left hip elevation
B. Left-uterine displacement
C. Regional anesthesia
D. General anesthesia

11. The most significant change in maternal lung volume that occurs in the third trimester of
pregnancy includes
A. Decrease in vital capacity
B. Increase in residual volume
C. Decrease in functional residual capacity (FRC)
D. Decrease in closing capacity (CC)

12. Which of the follow ing is not associated with oxytocin administration?
A. Myocardial ischemia
B. Respiratory depression
C. Hypotension
D. Tachycardia

13. During maintenance of a general inhaled anesthetic for an urgent nonobstetric surgery, one
would expect this difference in the pregnant patient versus a nonpregnant patient:
A. Slower emergence from anesthesia
B. Minimal changes in depth of anesthesia
C. There is to be no difference
D. Faster induction of anesthesia

14. When providing general anesthesia during pregnancy, minimum alveolar concentration (MAC)
is
A. Increased
B. Decreased
C. Unchanged
D. Unclear

15. The speed of time to hypoxia follow ing apnea is faster in the late-trimester parturient due to all
of the follow ing factors, except
A. Reduced functional residual capacity
B. Increased minute ventilation
C. Preoxygenation
D. Increased oxygen consumption

16. The correct respiratory physiologic change associated with pregnancy is


A. Increase in arterial pH
B. Increase in HCO3
C. Increase in Paco2
D. Increase in tidal volume

17. The P50 for maternal hemoglobin

A. Increases due to elevated levels of 2,3-diphosphoglycerate (DPG)


B. Remains unchanged
C. Increases to maintain pH
D. Decreases to enhance oxygen delivery to tissues

18. At sea level, the most likely arterial blood gas (ABG) sample of a parturient at 35 weeks’
gestation when she rests in the supine position breathing room air is
A. pH = 7.35, Pao2 = 90, Paco2 = 45, HCO3 = 20
B. pH = 7.40, Pao2 = 100, Paco2 = 40, HCO3 = 24
C. pH = 7.44, Pao2 = 90, Paco2 = 30, H C o3 = 20
D. pH = 7.50, Pao2 = 105, Paco2 = 30, H C o3 = 20

19. A 27-year-old G2P! at 392 weeks’ gestation is electing to have spinal anesthesia for a repeat
cesarean section. Five minutes after bupivacaine spinal injection, the patient becomes
hypotensive and is complaining of tingling in her fingers with subjective difficulty breathing.
Her oxygen saturation remains 100% and blood pressure is 95/55. The most likely etiology is
A. Engorgement of epidural veins contributed to inadvertent intravascular injection of the
local anesthetics
B. Decrease in volume of CSF in the subarachnoid space facilitated higher spread of local
anesthetics
C. Severe patient anxiety
D. Increased peripheral nerve sensitization to local anesthetics

20. During pregnancy, hepatic changes contribute to


A. Decreased albumin levels contributing to higher free blood levels of highly protein-bound
drugs
B. Decreased liver function tests due to decreased blood flow
C. Decreased concentration levels of coagulation factors leading to easy bruisability
D. Decreased activity of plasma cholinesterase resulting in significantly longer duration of
action of succinylcholine

Questions 21 to 23
After 18 hours of laboring and adherence to a strict nonpharmacologic natural birth plan, the patient
experiences late decelerations and fetal distress, requiring emergent cesarean section.
21. To minimize the risk of aspiration and resultant pneumonitis,
A. Place patient in left-uterine displacement
B. Give H2-receptor antagonist to decrease the pH of gastric fluid present in the stomach
C. Give metoclopramide to reverse opioid-induced gastric hypomotility
D. Give a nonparticulate antacid to decrease the pH of the gastric fluid

22. The most common cause of late decelerations in fetal heart rate (FHR) (down to 90 bpm) is
A. Fetal vagal reflex
B. Compression of the fetal head
C. Umbilical cord compression
D. Fetal alkalosis

23. After performing a single-shot intrathecal anesthetic consisting of 7.5 mg of preservative-free


bupivacaine and 25 |ig of fentanyl, the surgical incision is made and systemic hypotension
(78/44 mm Hg) ensued. To avoid significant decreases in uterine blood flow, first-line therapy
to consider is
A. Provide additional inhaled nitric oxide (NO) to vasodilate the uterine vasculature
B. Increase maternal cardiac output with use of epinephrine
C. Increase intravascular volume with fluids
D. Use reverse Trendelenburg to decrease aortocaval compression

Questions 24 to 25
With increasing concern of variable decelerations, a male fetus is delivered with vacuum assistance.
The amniotic fluid was noted to be meconium stained. Initial evaluation reveals a cyanotic lim p infant
with a heart rate of 80 bpm, poor respiratory efforts, and grimacing in response to suctioning.

24. Patient’s Apgar score would be


A. 0
B. 3
C. 5
D. 10

25. Appropriate initial steps in the resuscitation efforts would include all of the following, except
A. Tracheal suctioning
B. Provide radiant heat source
C. Positive-pressure ventilation
D. Supplemental oxygen

26. Regarding forceps-assisted delivery


A. High-forceps delivery has the highest success rate
B. Prevents clavicle fracture associated with dystocia
C. Hastens postpartum maternal recovery
D. Is associated with increased incidence of fetal facial nerve trauma

27. True statement regarding fetal circulation includes


A. The ductus venosus shunts blood away from the pulmonary circuit.
B. Deoxygenated blood is carried in the umbilical vein.
C. The foramen ovale shunts blood from right to left ventricles.
D. Intracardiac pressures are equalized across both right and left ventricles.

28. Successful transition from fetal to neonatal circulation is required after birth to support
extrauterine life. This depends prim arily on these factors, except
A. Removal of the placenta
B. Decreased systemic vascular resistance
C. Decreased pulmonary vascular resistance
D. Closure of the intra- and extracardiac shunts

29. In considering placental exchange and fetal uptake, all statements are true, except
A. Minimizing the maternal blood concentrations of a drug is the most important method of
lim iting the amount that ultimately reaches the fetus
B. Drugs that readily cross the blood-brain barrier w ill also cross the placenta
C. Placental exchange of substance occurs principally via ion transport from the maternal
circulation to the fetus
D. Ion trapping explains why fetal-to-maternal lidocaine ratios are higher during fetal
acidemia than during normal fetal well-being

30. Which of the follow ing best explains why lidocaine has a higher fetal-to-maternal plasma ratio
when compared with bupivacaine?
A. Bupivacaine has a smaller molecular weight
B. Lidocaine has higher protein-binding
C. Bupivacaine has a lower dissociation constant (pKa)
D. Lidocaine is less lipid soluble

31. In order to provide analgesia for all stages of labor, one must accommodate the evolving and
varied course of labor and delivery. The least accurate statement regarding the anatomy of
labor is
A. Pain during labor and delivery is often described in two stages
B. Somatic and visceral innervation of the uterus and cervix enters the spinal cord via T10 to
L1
C. Innervation of the perineum is prim arily via the pudendal nerve
D. Somatic and visceral afferent sensory fibers from the uterus and cervix travel with greater,
lesser, and least splanchnic nerves via the celiac plexus
32. The regional or neuraxial technique that would not be expected to provide appropriate
analgesic benefit during the first stage of labor is
A. Lumbar epidural
B. Pudendal nerve block
C. Lumbar sympathetic block
D. Paracervical block

Questions 33 to 37
A 37-year-old G9P4 patient at 38 weeks of gestation presents for management of labor and delivery.
She denies any medical history and admits to minimal prenatal care. The patient is moderately
hypertensive (160/95) with associated pitting edema at her ankles.

33. The statement about her disorder that is most likely true is
A. Eclampsia is imminent
B. Preeclampsia is a syndrome manifested after the 36th week of gestation
C. HELLP syndrome is the mildest form of eclampsia
D. Definitive treatment of preeclampsia is delivery of the fetus and placenta

34. The patient is started on oxytocin to augment her labor, and the patient is now requesting a
labor epidural. Anesthetic considerations include
A. The presence of hypertension and edema requires further workup before proceeding
B. No workup is required prior to performing epidural anesthesia, as this w ill treat her
hypertension
C. Neuraxial anesthesia should be avoided, as there is increased risk of bleeding
D. Avoid systemic opiates, as the risk of respiratory depression is too high

35. After a review of her laboratory results, a lumbar (L3-L4) epidural was placed without incident
(including lack of CSF, and negative test dose after administration of 45 mg lidocaine with
1:200,000 epinephrine). Epidural anesthesia is then initiated with a bolus of 15 mg of
bupivacaine. Variable decelerations are noted minutes later on fetal heart rate monitoring. If
scalp pH reveals fetal acidosis, compared with a normal pH, the anesthetic absorbed by the fetus
w ill be present in
A. Higher concentration, most in ionized form
B. Lower concentration, most in ionized form
C. Higher concentration, most in unionized form
D. Unchanged concentration, equal fraction of ionized and nonionized

36. The patient has now been receiving a dilute infusion (bupivacaine 0.125% with 2 |ig/mL
fentanyl) for the past 3 hours and reports good pain relief with a bilateral T5 sensory level. Her
blood pressure is now 85/45 mm Hg, and her heart rate is 120 bpm. The fetal heart rate pattern
begins to show late decelerations. The most appropriate management in this patient includes
A. Immediate bedside cesarean delivery
B. Administration of phenylephrine
C. Administration of ephedrine
D. Discontinuation of the epidural infusion

37. As augmentation of labor continues, patient’s blood pressure slowly climbs again, with waning
epidural analgesic benefit. Highest pressure was noted to be 166/112 mm Hg with heart rate
sustained over 100 bpm. The most appropriate pharmacologic option for acute treatment of
severe hypertension in a preeclamptic patient is
A. Magnesium
B. Dopamine
C. Labetalol
D. Hydralazine

38. Four hours postdelivery, and after the epidural is removed, the patient now requires emergent
anesthesia for surgical removal of retained placental products. The appropriate anesthetic
management includes all of the following, except
A. Antibiotic administration
B. Total intravenous anesthesia
C. General endotracheal inhaled anesthetic
D. Sodium citrate

39. Forty-eight hours postdelivery, the patient is febrile, complaining of chills with severe occipital
and neck pain worsened with sitting and standing, but not improved when lying in bed. The
finding you would not expect to find on examination is
A. Urinary retention
B. Low back pain
C. Nausea and vomiting
D. Normal white blood cell (WBC) count

40. Postdural puncture headache (PDPH) occurs more frequently


A. In elderly (>50 year old) vs. young patients
B. In underweight vs. overweight patients
C. With a cutting-point vs. pencil-point spinal needles
D. With larger- vs. smaller-gauge spinal needles

41. Decrease in fasciculations can be seen follow ing induction doses of succinylcholine for
emergent cesarean section. The factor that can blunt this response is
A. Increased cardiac output
B. Prior magnesium administration
C. Prior nitrous oxide inhalation
D. Metabolic alkalosis
42. Administration of all the follow ing w ill provide uterine relaxation, except
A. Sevoflurane
B. Nitrous oxide
C. Nitroglycerine
D. Terbutaline

43. Adverse effects of inhaled P-tocolytic therapy for preterm labor to the mother include all of the
following, except
A. Hypoglycemia
B. Pulmonary edema
C. Tachycardia
D. Ventricular arrhythmias

44. During a general anesthetic for emergent cesarean section, administering of all of the
follow ing could contribute to increased operative blood loss, except
A. Nitroglycerine
B. Ritodrine intravenously
C. 1 MAC Desflurane
D. Hyperventilation

45. With regard to sodium thiopental, the follow ing statements are accurate, except
A. Peak concentration in the brain occurs at 1 minute postinjection
B. Rapid redistribution allows for return of consciousness in <10 minutes
C. Infusions maintain appropriate surgical conditions with fast recovery due to ultra-short
action
D. Repeating the induction dose results in fetal depression

46. The follow ing statements are true regarding umbilical cord blood, except
A. Provides a picture of the acid-base balance in the infant at the moment of birth
B. Double clamping of the umbilical cord at birth w ill preserve a segment of cord blood in
isolation, which can remain stable for up to 24 hours
C. Cord blood that is still in continuity with the placenta w ill have shifting acid-base balance
due to ongoing placental metabolism and gas exchange
D. Normal paired arterial and venous specimens can provide evidence against an intrapartum
hypoxic-ischemic event to the newborn

47. Maternally administered drugs that decrease beat-to-beat variability of fetal heart rate include
all of the following, except
A. Ritodrine
B. Atropine
C. Prochlorperazine
D. Bupivacaine

48. A 24-year-old G4P2 parturient is undergoing a general anesthetic for emergency cesarean
section due to uterine rupture. A ll these findings would suggest an amniotic fluid embolism
(AFE), except
A. Decreased EtCO2
B. Increased maternal pH
C. Bleeding diathesis
D. Upsloping EtCO2 tracing

49. A 42-year-old G!P0 at 294 weeks’ gestation is undergoing intracranial clipping of a large
arteriovenous malformation, follow ing sudden onset of a severe headache with associated
nausea/vomiting. Patient is intubated in the interventional radiology suite and ventilated with
settings of T V = 500 mL, respiratory rate = 14 bpm, PEEP = 5 cm H2O, and Fio 2 = 1.0. Arterial
blood gas (ABG) 30 minutes later reveals pH = 7.55, Pao2 = 502, Paco2 = 19, and HCO3 = 21.
These findings are associated with all of the following, except
A. Decreased fetal cerebral oxygen delivery
B. Decreased placental transfer of oxygen
C. Rightward shift of the oxygen dissociation curve
D. Decreased umbilical blood flow

50. True statement concerning hyperglycemia during pregnancy is


A. Increases risk of fetal microsomia
B. Fetal oxygen requirements remain decreased
C. May contribute to neonatal hypoglycemia
D. Increases risk of sepsis during cesarean delivery

51. True statement regarding neuraxial opioids for labor and delivery is
A. Opioids should never be used as a sole agent
B. Most common side effect is fetal bradycardia
C. Intrathecal morphine is associated with quick peak in concentration and early onset
maternal respiratory depression
D. Systemic absorption is similar to intramuscular (IM) administration

52. A ll of the follow ing drugs readily cross the placenta, except
A. 0-Agonist antagonists
B. Local anesthetics
C. Insulin
D. Morphine
53. Following a 0.6 mg/kg intravenous dose of rocuronium to facilitate rapid-sequence induction
in a parturient requiring surgical delivery, one would expect
A. Minimal placental transfer of rocuronium to the newborn
B. Shorter duration of relaxation with concurrent magnesium administration
C. Unsuitable intubating conditions as recommended doses are 1.5 mg/kg
D. Use of rocuronium has been shown to affect Apgar scores and fetal muscle tone at birth
and should be strictly avoided

54. During cesarean section under general endotracheal anesthesia, venous air embolism (VAE)
A. Is associated with high end-tidal CO2
B. Should be treated with nitrous oxide
C. Is associated with expired nitrogen
D. Induces severe hypertension

Questions 55 to 58
A 30-year-old otherwise-healthy G2P0 (167 cm, 68 kg) presents at 341 weeks’ gestation with the
rupture of membranes, single footling in breech presentation with fetal bradycardia. The decision for
emergent cesarean delivery under general anesthesia is made, and the patient is quickly prepared for a
rapid sequence induction. However, patient’s larynx is noted to be very anterior, and is unable to be
intubated after multiple direct laryngoscopy attempts.

55. The appropriate next step considering persistent fetal bradycardia (<80 bpm) is
A. Administer 1 mg/kg of rocuronium intravenously
B. Use bag-mask ventilation and allow surgical delivery to proceed
C. Wake the patient up for awake fiberoptic intubation
D. Reposition the patient in Trendelenburg with left-uterine displacement

56. The fetus is quickly delivered (skin-to-skin time of 18 minutes). However, 10 minutes after
delivery, her uterus is noted to be boggy and bleeding persists. The appropriate treatment
option is
A. Bolus oxytocin (Pitocin) 20 U intravenously
B. Bolus methylergonovine (Methergine) 0.2 mg intravenously
C. Misoprostol (Cytotec) 800 mg intramuscularly
D. 15-Methyl PGF2a (Hemabate) 0.25 mg intramuscularly

57. Two hours later, the patient remains apneic and intubated in the intensive care unit. She is
sedated and mechanically ventilated (TV = 450, RR = 12, Fio 2 = 0.4) with the arterial blood gas
revealing a pH of 7.45, Pao2 of 100 mm Hg, and Paco2 of 37 mm Hg with a base excess of
zero. Her examination reveals absent deep-tendon reflexes throughout. ECG reveals
intermittent ventricular bigeminy. This situation could be explained by
A. Hypermagnesemia
B. Severe hypovolemic shock
C. Hypocalcemia
D. Pituitary necrosis

58. At 3 weeks’ postpartum, the patient has absence of lactation and denies return of her menstrual
cycle. Review of systems is positive for intolerance to cold, constipation, hair loss, and 2-
pound weight gain. The best explanation for this constellation of symptoms is
A. Amenorrhea-galactorrhea syndrome
B. Sheehan syndrome
C. Fibromyalgia
D. Meigs syndrome
CHAPTER 17 ANSWERS

1. D. I n c r e a s in g le v e ls o f p r o g e s t e r o n e a lo n g w ith a n e n la r g in g u te r u s c o n tr ib u te s to
in c o m p e te n c e o f th e lo w e r e s o p h a g e a l s p h in c te r p la c in g p a r tu r ie n ts a t in c r e a s e d r i s k o f
a s p ir a tio n . T h is r i s k in c r e a s e s f u r th e r a s d e la y e d g a s tr ic e m p ty in g is a s s o c ia te d w ith b o th th e
o n s e t o f la b o r ( s y m p a th e tic e ffe c ts ) a n d !! -o p io id a d m in is tr a tio n f o r a n a lg e s ia . A s p ir a tio n
p r e c a u tio n s m u s t b e u tiliz e d w h e n p r o v id i n g a n e s th e s ia f o r w o m e n b e y o n d m id g e s ta tio n .

2. B. D u r in g p re g n a n c y , c a r d io v a s c u la r c h a n g e s in c lu d e in c r e a s e in b l o o d v o lu m e , p la s m a
v o lu m e , c a r d ia c o u tp u t, s tr o k e v o lu m e , a n d h e a r t ra te . D e s p ite th e s e in c r e a s e s , th e s y s te m ic
b l o o d p r e s s u r e , d u r in g a n o r m a l u n c o m p lic a te d p re g n a n c y , d o e s n o t in c r e a s e d u e to d e c r e a s e in
s y s te m ic v a s c u la r r e s is ta n c e . S im ila r ly , th e r e is n o c h a n g e in c e n tr a l v e n o u s p r e s s u r e s d e s p ite
th e in c r e a s e in p la s m a v o lu m e d u e to in c r e a s e in v e n o u s c a p a c ita n c e (T a b le 1 7 -1 ).

Table 17-1 Normal Hemodynamic Changes during Pregnancy


P a r a m e te r P regnancy Labor

Blood volume Increase of 50% Increase


Heart rate Increase of 10-15 bpm Increase
Blood pressure Decrease of 10 mmHg Increase

Stroke volume Increase—1st and 2nd trimester Increase of 300 mL/contraction


Decrease—3rd trimester
Cardiac output Increase of 30%-50% Additional increase of 50%
Systemic vascular resistance Decrease Increase

3. D. M a te rn a l in tr a v a s c u la r f lu id v o lu m e in c r e a s e s in th e f i r s t tr im e s te r , a n d a t te r m , th e
p la s m a v o lu m e is in c r e a s e d b y a b o u t 4 5 % , w h ile th e e r y th r o c y te v o lu m e in c r e a s e s o n ly 2 0 % ,
a c c o u n tin g f o r th e r e la tiv e a n e m ia o f p r e g n a n c y d e s p ite th e h ig h e r h e m a to c r it. T h is s e r v e s to
d e c r e a s e b l o o d v is c o s ity a n d im p r o v e flo w . T h is in c r e a s e in m a te r n a l b l o o d v o lu m e a ls o a llo w s
w o m e n to b e tte r to le r a te th e b l o o d lo s s a s s o c ia te d w ith d e liv e ry .

4. A. C a r d ia c o u tp u t in th e th ir d tr im e s te r is in c r e a s e d b y n e a r ly 5 0 % d u e to a n in c r e a s e in b o th
th e s tr o k e v o lu m e a n d h e a r t r a te to m e e t a u g m e n te d m a te r n a l a n d fe ta l m e ta b o lic d e m a n d s .

5. C. T h e l a r g e s t in c r e a s e in c a r d ia c o u tp u t is s e e n im m e d ia te ly a fte r d e liv e r y a s th e in c r e a s e in
b l o o d v o lu m e p e r s is ts w ith a n a d d itio n a l in c r e a s e in in tr a v a s c u la r v o lu m e ( 3 0 0 - 5 0 0 m L ) f r o m
th e c o n tr a c tin g u te ru s . T h is a u to tr a n s f u s io n f u r th e r in c r e a s e s c a r d ia c o u tp u t. P a tie n ts w ith fix e d
s te n o tic v a lv u la r le s io n s th e r e f o r e s h o u ld c o n tin u e to b e m o n ito r e d c lo s e ly a fte r d e liv e ry .

6. D. H y p e r v e n tila tio n w ith o x y g e n c o n s u m p tio n c r e a te s s ig n if ic a n t c h a n g e s in a c i d - b a s e sta tu s


th a t c a n b e h a z a r d o u s to th e fe tu s . E x tr e m e ly l o w P a c o 2 le v e ls r e s u lt in v a s o c o n s tr ic tio n a n d
g lo b a l r e d u c tio n in p la c e n ta l p e r f u s io n a n d b l o o d flo w . T h e a lk a le m ia a ls o s h ifts th e o x y g e n -
h e m o g lo b in d is s o c ia tio n c u r v e to th e left, i m p a ir in g th e r e le a s e o f o x y g e n f r o m m a te r n a l b lo o d
to fetal blood. Both factors w ill decrease the availability of oxygen delivery to the fetus (Fig 17­
1).

7. C. The anterior spinal cord achieves its blood supply from the anterior spinal artery, which
is single and unpaired. Due to this, injury or thrombosis can lead to a unique constellation of
symptoms consisting of loss of motor function, pain, and temperature below the level of the
injury, bilaterally. The posterior columns, carrying fine touch and proprioception, are
preserved as paired posterior spinal arteries supply.

8. B. The presence of moderate-severe aortic stenosis makes it especially crucial to minimize


sympathetic output and hemodynamic deterioration. Though a regional anesthetic is capable of
attenuating the release of catecholamines during painful labor, one must be vigilant to prevent
hypotension. The abrupt hypotension follow ing spinal anesthesia with local anesthetics may
result in cardiac hypoperfusion and ischemia. Epidural anesthesia, with its slow onset to effect,
is usually well tolerated, especially with adequate volume loading. Inhaled nitrous oxide and
oral analgesics are unlikely able to provide adequate analgesia to maintain hemodynamic
stability in this patient with a fixed valve lesion.

9. A. Although there is an increase in cardiac output and plasma volume, the systemic blood
pressure in normal maternal physiology does not actually increase due to decreases in systemic
vascular resistance. In fact, the mean arterial pressures generally decrease by approximately 10
to 15 mm Hg. Certainly, her fixed stenotic lesion also contributes.

10. B. The mechanism of supine hypotension syndrome is decreased venous return as a result of
aortocaval compression by the gravid uterus when the pregnant woman assumes the supine
p o s itio n . R e m o v a l o f th e c o m p r e s s io n w ith le f t- u te r in e d is p la c e m e n t c a n m in im iz e th is
in c id e n c e , w h ic h is p a r tic u la r ly im p o r ta n t f o r p a tie n ts u n d e r g o in g r e g i o n a l o r g e n e r a l
a n e s th e s ia b e c a u s e u s u a l c o m p e n s a to r y in c r e a s e s in s y s te m ic v a s c u la r r e s is ta n c e w ill b e
b lo c k e d .

11. C. W ith in c r e a s in g e n la r g e m e n t o f th e u te ru s , th e d ia p h r a g m is f o r c e d c e p h a la d , w h ic h is
r e s p o n s ib le f o r d e c r e a s in g th e F R C . W h ile s u p in e , F R C c a n b e c o m e le s s th a n C C f o r m a n y
s m a ll a ir w a y s r e s u ltin g in a te le c ta s is . E x p i r a t o r y r e s e r v e v o lu m e a n d r e s id u a l v o lu m e a r e a ls o
d e c r e a s e d w h ile b o th v ita l c a p a c ity a n d C C r e m a in u n c h a n g e d .

12. B. O x y to c in r e m a in s a f ir s t- lin e a g e n t in th e p r e v e n tio n a n d m a n a g e m e n t o f u te r in e a to n y .


O x y to c in h a s im p o r ta n t c a r d io v a s c u la r s id e e ffe c ts , in c lu d in g h y p o te n s io n a n d ta c h y c a r d ia ,
s e ttin g th e s ta g e f o r m y o c a r d ia l is c h e m ia . S lo w c o n tin u o u s in tr a v e n o u s a d m in is tr a tio n o f
o x y to c in m in im iz e s m a te r n a l h e m o d y n a m ic in s ta b ility a n d a ls o e n c o u r a g e s m a in te n a n c e o f
u te r in e to n e .

13. D. D e c r e a s e d f u n c tio n a l r e s id u a l c a p a c ity (F R C ) w ith in c r e a s e d m in u te v e n tila tio n r e s u lts in a n


e s c a la tio n in r a te a t w h ic h c h a n g e s o f a lv e o la r c o n c e n tr a tio n o f in h a le d a n e s th e tic s c a n b e
a c h ie v e d , in c r e a s in g s p e e d o f in d u c tio n , e m e r g e n c e , a n d c h a n g e s in d e p th o f a n e s th e s ia .

14. B. M A C p r o g r e s s i v e l y d e c r e a s e s d u r in g p r e g n a n c y — a t te r m b y a s m u c h a s 4 0 % — f o r a ll
v o la tile a n e s th e tic a g e n ts .

15. C. I n c r e a s e d o x y g e n c o n s u m p tio n a n d d e c r e a s e d r e s e r v e d u e to r e d u c e d f u n c tio n a l r e s id u a l


c a p a c ity c a n r e s u lt in a r a p id f a ll in a r te r ia l o x y g e n te n s io n d u r in g a p n e a . T h is o c c u r s d e s p ite
c a r e f u l p r e o x y g e n a tio n , w h ic h im p o r ta n tly p r o v id e s a tim e b u ff e r.

16. D. M in u te v e n tila tio n is in c r e a s e d a b o v e p r e p r e g n a n t le v e ls p r i m a r i l y b y a s ig n if ic a n t in c r e a s e


in tid a l v o lu m e w ith s m a lle r in c r e a s e s in r e s p i r a t o r y ra te . R e s tin g m a te r n a l P a CO2 d e c r e a s e s
f r o m 4 0 to a b o u t 32 m m H g , th o u g h a r te r ia l p H a p p r o a c h e s n o r m a l le v e ls w ith in c r e a s e d r e n a l
e x c r e tio n o f b ic a r b o n a te io n s . A t te r m , P a o2 g e n e r a lly d o e s n o t c h a n g e s ig n if ic a n tly , th o u g h
m a y b e s lig h tly d e c r e a s e d , r e f le c tin g a ir w a y c lo s u r e a n d a te le c ta s is .

17. A. E le v a te d le v e ls o f 2 ,3 -D P G d e c r e a s e s m a te r n a l h e m o g lo b in a f f in ity f o r o x y g e n , s h iftin g


th e P 50 c u r v e to th e right ( in c r e a s in g it f r o m 2 7 to 3 0 m m H g ) to e n h a n c e o x y g e n d e liv e r y to
tis s u e s .

18. C. A n in c r e a s e d m e ta b o lic r a te in a d d itio n to th e p r e g n a n c y - in d u c e d in c r e a s e in m in u te


v e n tila tio n r e s u lts in a m a r k e d ly d e c r e a s e in P a CO2 f r o m 4 0 to 3 0 m m H g . S u c h th a t a P a CO2 o f
4 0 to 4 5 o n A B G w o u ld in d ic a te C O 2 r e te n tio n . H o w e v e r, r e s p i r a t o r y a lk a lo s is is m in im iz e d
d u e to m e ta b o lic c o m p e n s a tio n w ith in c r e a s e d r e n a l e x c r e tio n o f b ic a r b o n a te a n d th u s a lo w e r
H C O 3 a s c o m p a r e d to th e n o n p a r tu r ie n t. A t 3 5 w e e k s ’ g e s ta tio n , a n o r m a l m a te r n a l p H is
approximately 7.44. Lastly, maternal position can affect Pao2, even in healthy parturients. In the
upright position and breathing room air, Pao2 w ill be slightly greater than 100 mm Hg, as the
increase in cardiac output is greater than the increase in oxygen consumption. However, as
functional residual capacity decreases during pregnancy and is often less than closing capacity
in the supine position, Pao2 w ill frequently fall below 100 mm Hg, while supine likely
reflecting atelectasis.

19. B. Engorgement of epidural veins occurs with progressive enlargement of the uterus
contribute to decrease in size of the epidural space and can predispose to intravascular injection
with attempted epidural anesthesia. Additionally, CSF volume is decreased in the subarachnoid
space, facilitating the spread of local anesthetics. Therefore, in pregnancy, there is a decrease in
dose requirements of local anesthetics for neuraxial procedures.

20. A. Similar to physiologic anemia of pregnancy, dilution of serum albumin w ill result in
higher free blood levels of highly protein-bound drugs (e.g., fentanyl and midazolam),
resulting in a more robust clinical effect as compared to the nonpregnant state. Though plasma
cholinesterase activity is also decreased, this is unlikely to be of clinical significance with
regard to the duration of action of succinylcholine.

21. D. Cephalad displacement of the pylorus, decreased gastrointestinal motility, and decreased
pH of gastric contents all contribute to significant risk of aspiration and resultant pneumonitis
during labor and delivery. H2-receptor antagonists, unlike antacids, do not alter the pH of
gastric contents already present in the stomach. Avoidance or particulate antacids can minimize
pulmonary damage should aspiration occurs. Though metoclopramide can increase gastric
m otility to decrease the gastric volume, opioid-induced hypomotility is resistant to this
treatment.

22. C. Late decelerations are shallow, uniform ly shaped decelerations that are characterized by a
gradual decrease from and return to baseline of the fetal heart rate. The nadir of late
decelerations usually is between 5 and 30 bpm below the baseline. Late decelerations typically
begin near the end of a contraction; with return to baseline, FHR always occurring after the
contractions have ended. Uteroplacental insufficiency (e.g., umbilical cord compression,
maternal supine hypotension syndrome) contributes to late decelerations. Cephalopelvic
disproportion and fetal head compression are associated with early decelerations (Fig 17-2).
Late Decelerations

Mom’s Contractions

Figure 17-2.

23. C. The uterine vasculature is not autoregulated and remains essentially maximally dilated
under normal conditions during pregnancy. Epidural or spinal anesthesia does not
pathologically alter uterine blood flo w as long as maternal hypotension is avoided. Acceptable
options include left-uterine displacement to minimize aortocaval compression, increase IV
fluids to improve volume status, Trendelenburg position to encourage venous return, and a-
adrenergic agents to increase maternal arterial blood pressure. Prompt correction of maternal
hypotension w ill lead to best neonatal outcome.

24. B. The Apgar is a scoring system for evaluating an infant’s physical condition at birth. The
infant’s heart rate (1 pt), respiration (1 pt), muscle tone (0 pt), response to stimuli (1 pt), and
color (0 pt) are rated at 1 minute and again at 5 minutes after birth. Each factor is scored 0, 1, or
2; the maximum total score is 10 (Fig 17-3).

0 1 2

H e a r t ra te A bsent S lo w < 1 0 0 /m in > 1 0 0 /m in

R e s p ir a to r y ra te A bsent S lo w , w e a k c r y G o o d cry

M u s c le to n e F la c c id S o m e fle x io n o f e x tr e m it ie s W e ll fle x e d

R e fle x , ir r ita b ility N o resp o n s e G r im a c e C ry

C o lo r B lu e , p a le B o d y p in k , e x t r e m it ie s b lu e C o m p le te ly p in k

S c o re

Figure 17-3.
25. C. A team that is skilled at reviving newborn infants should be at the delivery if meconium
staining is found in the amniotic fluid. Newborns should be placed under radiant heat sources to
support body temperature, as heat loss can be rapid. If the baby is active and crying, no
treatment is needed. If the baby is not active and crying right after delivery, the trachea should
be suctioned. Avoiding positive-pressure ventilation before suctioning can minimize further
aspiration into the lungs.

26. D. The invention of the forceps has had a profound influence on obstetrics, providing an
alternative modality to surgical delivery during cases of difficult, nonprogressing, or
obstructed labor. High-forceps delivery refers to attempted delivery prior to head engagement,
which carries high risk and is no longer an accepted practice. Certainly, there is risk to both
mother and child. Maternal risks include increased postpartum recovery time and pain, while in
the fetus, forceps assistance can cause minor injuries such as cuts and bruises, or more serious
damages such as facial nerve injury, clavicle fracture, and intracranial hemorrhage.

27. D. The fetal circulation is markedly different from the adult circulation as gas exchange does
not occur in the lungs but instead occurs in the placenta. The placenta provides oxygen-rich
blood via the umbilical vein to the fetal circulation and removes deoxygenated blood via
umbilical arteries. In addition, the fetal cardiovascular system is designed in such a way that the
most highly oxygenated blood is delivered preferentially to vital organs (brain and heart) while
minimizing flo w to nonvital fetal organs (liver and lungs). The presence of intra- and
extracardiac shunts achieves these circulatory adaptations in the fetus; the ductus venosus shunts
oxygenated blood away from the liver, while the ductus arteriosus shunts blood away from the
fetal pulmonary bed. The foramen ovale effectively shunts blood from the right to the left
atrium, resulting in equalization of right and left sides of the heart (Fig 17-4).
F igu re 17-4.

28. B. Successful transition from fetal to neonatal circulation starts when the umbilical cord is
clamped and cut such that the placenta no longer acts as the “ lungs” to provide oxygen. The
ductus venosus closes physiologically as soon as the umbilical vein is obstructed with the
clamping of the cord. With the loss of the placenta, a large low-resistance bed, systemic
vascular resistance rapidly increases. With the first breath, the lungs inflate with a fall in
pulmonary vascular resistance and increase pulmonary blood flow. The rise in pulmonary
venous return results in left-atrial pressure being slightly higher than right-atrial pressure to
close the foramen ovale. Decrease in circulating prostaglandins and the higher blood oxygen
content of blood result in vasoconstriction of the ductus arteriosus.

29. C. Placental exchange of substances occurs principally by diffusion from the maternal
circulation to the fetus and vice versa, which depends on maternal-fetal concentration gradients,
maternal protein-binding, molecular weight, lipid solubility, and the degree of ionization of that
substance.

30. D. Local anesthetics easily cross the placenta, which is affected by several independent factors,
including maternal-fetal hemodynamics, permeability of the placenta, concentration of free
drug in the maternal plasma, and physiochemical properties of the drug itself. Lidocaine is less
lipid soluble than bupivacaine, which is reflected in their lipid-water partition coefficients.
P r o te in - b in d in g a ls o p la y s a r o l e in th e d if f u s io n o f d r u g s a c r o s s th e p la c e n ta . T h e u n b o u n d
f o r m o f th e d r u g is f r e e ly tr a n s f e r r e d , w h e r e a s p r o te in - b in d in g lim its d if f u s io n . H ig h p r o t e i n ­
b in d in g , a s in th e c a s e o f b u p iv a c a in e , le a d s to m u c h lo w e r f e ta l- to - m a te r n a l p la s m a r a tio . L o c a l
a n e s th e tic s a r e w e a k b a s e s a n d th e r e f o r e h a v e m in im a l io n iz a tio n a t p h y s io lo g ic p H . T h e c lo s e r
th e p K a is to th e p h y s io lo g ic p H , th e m o r e it w ill b e a ffe c te d b y th e a c i d - b a s e sta tu s o f th e fe tu s .

31. D. S o m a tic a n d v is c e r a l a f f e r e n ts f r o m th e u te r u s a n d c e r v ix tr a v e l w ith s y m p a th e tic n e r v e


f ib e r s e n r o u te to th e s p in a l c o r d . T h e s e f ib e r s p a s s th r o u g h th e i n f e r io r , m id d le , a n d s u p e r io r
h y p o g a s tr ic p le x u s e s to a r r i v e a t th e s y m p a th e tic c h a in . T h e f i r s t s ta g e o f la b o r is l a r g e l y
v is c e r a l ( T 1 0 - L 1 ) d u e to u te r in e c o n tr a c tio n s . A s la b o r p r o g r e s s e s , th e p a r tu r ie n t e n c o u n te r s
th e s e c o n d s ta g e o f la b o r w ith a d d itio n a l s o m a tic p a in c o m p la in ts a s th e fe tu s d e s c e n d s in to th e
p e lv is c a u s in g d is te n s io n o f th e v a g in a , p e r in e u m , a n d p e lv ic f l o o r m u s c le s . T h is s o m a tic p a in
is tr a n s m itte d v ia th e p u d e n d a l n e r v e ( S 2 - S 4 ) .

32. B. L u m b a r s y m p a th e tic b lo c k s a n d p a r a c e r v ic a l b lo c k s , th o u g h r a r e l y p e r f o r m e d f o r la b o r
a n a lg e s ia , a r e a p p r o p r ia te ta r g e ts d u r in g th e f i r s t s ta g e o f la b o r . H o w e v e r, p a r a c e r v ic a l b lo c k s
a r e r a r e l y u s e d in c u r r e n t p r a c tic e b e c a u s e o f a n a s s o c ia tio n w ith fe ta l b r a d y c a r d ia . E p id u r a l
a n d c o m b in e d s p i n a l - e p i d u r a l te c h n iq u e s a r e id e a l, in th a t th e y a r e a b le to b lo c k th e v is c e r a l
a f f e r e n t f ib e r s r e s p o n s ib le f o r th e f i r s t s ta g e o f la b o r a n d th e s o m a tic n e r v e f ib e r s th r o u g h
w h ic h th e s e c o n d s ta g e is tra n s m itte d . T h e o b s te tr ic ia n c a n a ls o p r o v id e p u d e n d a l b lo c k s d u r in g
d e liv e r y to m itig a te s o m a tic p a in d u r in g th e s e c o n d s ta g e , th o u g h c a n n o t m itig a te v is c e r a l f i r s t
s ta g e p a in .

33. D. P r e e c la m p s ia is a s y n d r o m e m a n if e s te d a fte r th e 2 0 th w e e k o f g e s ta tio n , w h ic h is


c h a r a c te r iz e d b y s y s te m ic h y p e r te n s io n (> 1 4 0 /9 0 m m H g ), p r o te in u r i a (> 0 .5 g /d a y ), g e n e r a liz e d
e d e m a , a n d c o m p la in ts o f a h e a d a c h e . H E L L P s y n d r o m e ( h e m o ly s is , e le v a te d liv e r e n z y m e s ,
low p la te le ts ) is a s e v e r e f o r m o f p r e e c la m p s ia . E c la m p s ia is p r e s e n t w h e n s e iz u r e s a r e
s u p e r im p o s e d o n p r e e c la m p s ia , a n d it is p o te n tia lly lif e - th r e a te n in g . C a u s e s f o r m a te r n a l
m o r t a lit y in w o m e n w ith p r e e c la m p s ia in c lu d e c o n g e s tiv e h e a r t f a ilu r e , m y o c a r d ia l in f a r c tio n ,
c o a g u lo p a th y , a n d c e r e b r a l h e m o r r h a g e . D e f in itiv e tr e a tm e n t is th e d e liv e r y o f th e fe tu s a n d
p la c e n ta , a fte r w h ic h p r e e c la m p s ia u s u a lly a b a te s w ith in 4 8 h o u r s .

34. A. T h e p r e s e n c e o f h y p e r te n s io n w ith a s s o c ia te d e d e m a r e q u ir e s f u r th e r w o r k u p in c lu d in g
c o m p le te b l o o d c o u n t w ith p la te le ts a n d p r o t h r o m b i n tim e /in te r n a tio n a l n o r m a liz e d r a tio to
e n s u r e a d e q u a te h e m o s ta s is c a n b e a c h ie v e d . U s u a lly , p la te le ts > 1 0 0 K / |iL c a r r y little in c r e a s e d
r is k , a n d o n e m a y s a f e ly p r o c e e d w ith e p id u r a l p la c e m e n t. P la te le t c o u n t < 5 0 K / |iL is g e n e r a lly
c o n s id e r e d a c o n tr a in d ic a tio n to n e u r a x ia l in te r v e n tio n s d u e to h ig h r i s k o f e p id u r a l h e m a to m a .
E p id u r a l a n e s th e s ia is o f te n v ie w e d a s th e te c h n iq u e o f c h o ic e f o r la b o r p a in a s th e p a r tu r ie n t
r e m a in s a w a k e a n d a le r t w ith o u t s e d a tiv e s id e e ffe c ts . H o w e v e r, s y s te m ic o p ia te s a r e r e a s o n a b le
if e p id u r a l is c o n tr a in d ic a te d f o r w h a te v e r r e a s o n , in c lu d in g p a tie n t r e f u s a l. A g e n e r a l o r
r e g i o n a l a n e s th e tic s h o u ld n o t b e u s e d in a tte m p ts to lo w e r m a te r n a l b l o o d p r e s s u r e .

35. A. T r a n s f e r o f d r u g s f r o m m o th e r to fe tu s ta k e s p la c e a t th e le v e l o f th e p la c e n ta m a in ly b y
diffusion. Thus, keeping maternal blood levels of drugs as low as possible is a major strategy
for decreasing the amount of drug that reaches the fetus. In addition, since most of the blood in
the umbilical vein travels directly to the liver, a large portion of the drug w ill be metabolized
before reaching vital fetal organs. Furthermore, drug in the umbilical vein that bypasses the
liver via the ductus venosus to access the inferior vena cava w ill be diluted with blood from the
lower extremities, and this further reduces concentration of drugs in the fetal blood. Two things
work against these “ safety features” : (1) fetal acidosis during times of distress causes increased
perfusion of the heart and brain and thus increases delivery of drug to these important organs.
(2) Fetal pH is lower than maternal pH and results in basic drugs (such as local anesthetics)
becoming more ionized when they reach fetal circulation. This effectively traps them on the
fetal side of the circulation, since ionized molecules cannot easily cross the placenta. This also
maintains a gradient for diffusion. This is known as “ ion trapping” and can be quite a
significant effect especially during times of fetal distress (when pH gets even lower).

36. B. Late decelerations are worrisome as it is a sign of fetal hypoxemia, which requires prompt
treatment. Uteroplacental resuscitation measures should be implemented immediately in an
attempt to improve uteroplacental perfusion and oxygen delivery to the fetus. Supplemental
oxygen should be provided to the mother, and she should be placed in a lateral position to avoid
aortocaval compression. Maternal hypotension should be treated promptly with an IV fluid
bolus and/or administration of a vasopressor. In this case, phenylephrine may also improve her
tachycardia. Emergent cesarean delivery is indicated only if these utero resuscitative measures
are not successful. Discontinuation of the epidural infusion is recommended only if the patient
has an excessively high sensory level.

37. C. Definitive treatment is delivery of the fetus and placenta. In the interim, magnesium and
antihypertensive drugs may be required. Magnesium is effective by decreasing the irrita b ility of
the CNS to decrease the risk of seizures. Though it m ildly reduces blood pressure due to its
vasodilatory effect, it is not an effective agent for severe hypertension. Antihypertensives are
usually required when the diastolic pressure is >110 mm Hg. Hydralazine and labetalol are the
most commonly administered. Hydralazine has the advantage of being a vasodilator; thus, it can
improve uteroplacental and renal blood flow. Labetalol, with its adrenergic blockade may
improve tachycardia. Keep in mind that labetalol has a much faster onset of action (5 minutes)
vs. hydralazine (30 minutes) as such may be more appropriate for acute management of severe
hypertension.

38. B. A ll women in the peripartum period should be given a nonparticulate antacid such as
sodium citrate 30 mL to neutralize gastric contents. A rapid-sequence induction should be
performed follow ing adequate preoxygenation. If a woman is in shock, etomidate is preferable
to thiopental or propofol as an induction agent. Equipotent doses of all the volatile agents
depress uterine contractility to an equivalent, dose-dependent extent. Following retained
placenta, there is an increased incidence of endometritis; however, there is no consensus
opinion on whether antibiotic prophylaxis is routinely indicated.

39. D. Epidural abscesses are associated with headache, fevers/chills, nausea/vomiting, low back
pain, and bowel or bladder dysfunction that can range from retention to incontinence.
Hematologic evaluation would likely reveal an immune response with elevated WBC.

40. C. Since the first reported case in 1898, PDPH has been a problem for patients follow ing dural
puncture. Research over the last 30 years has shown that use of larger-gauge needles,
particularly of the pencil-point design, is associated with a lower risk of PDPH than larger
traditional cutting-point (Quincke) needle tips. Keep in mind that gauge and bore diameter of a
needle are inversely related such that a 22G is smaller compared to a 16G. A careful history
should rule out other causes of headache. A postdural component of headache is the sine qua
non of PDPH. High-risk patients include those <50 years, postpartum, and puncture with small
gauge (large bore diameter) needles.

41. B. Magnesium acts as a physiologic calcium blocker to provide uterine relaxation in addition
to electrical conduction disruption such that levels can be predicted strength of deep-tendon
reflexes. Similarly, postsuccinylcholine fasciculations are blunted.

42. B. Nitroglycerin may be used as an alternative to terbutaline sulfate (P2 agonist) or general
endotracheal anesthesia with halogenated agents for uterine relaxation. Inhaled anesthetics
produce dose-dependent uterine vasodilatation with a decrease in uterine contractility. Uterine
relaxation produced by inhalation agents may be helpful for removal of retained placenta.
However, uterine vasodilatation might lead to increased blood loss during obstetric surgery or
delivery. Nitrous oxide does not change uterine contractility in doses provided during delivery.
Initiating treatment with incremental doses of nitroglycerin may relax the uterus sufficiently
while minimizing potential complications (e.g., hypotension).

43. A. Maternal side effects due to P2-agonist therapy (e.g., terbutaline, ritodrine) for tocolysis
include cardiopulmonary complications (e.g., arrhythmias, tachycardia, hypotension, and
pulmonary edema) and metabolic hyperglycemia.

44. D. Alkalization of the blood causes vasoconstriction, to provide a semblance of hemostasis.


Inhaled anesthetics, ritodrine, and nitroglycerine are all potent vasodilators that can contribute
to her ongoing blood loss.

45. C. Sodium thiopental is an ultra-short-acting barbiturate commonly used to induce general


anesthesia prior to intubation. Following a low dose, the drug rapidly reaches the brain and
causes unconsciousness within 30 to 45 seconds. At 1 minute, the drug attains a peak
concentration of about 60% of the total dose in the brain. Thereafter, the drug distributes to the
rest of the body, and in about 5 to 10 minutes, the concentration is low enough in the brain such
that consciousness returns. Thus, a one-time bolus displays first-order kinetics. Larger doses,
or infusions, undergo slow zero-order elimination kinetics such that thiopental is not used to
maintain anesthesia in surgical procedures (T/2 11.5- 26 hours) due to slow recovery. As such,
larger or repeated doses can depress the baby.
46. B. U m b ilic a l c o r d b l o o d g a s a n a ly s is is r e c o m m e n d e d in a ll h i g h r i s k d e liv e r ie s . F o r m o s t
a c c u r a te in te r p r e ta tio n , p a ir e d u m b ilic a l a r te r ia l a n d v e n o u s s a m p le s s h o u ld b e ta k e n s o o n a fte r
b ir t h f r o m a s e g m e n t o f c o r d th a t h a s b e e n d o u b ly c la m p e d to is o la te it f r o m th e p la c e n ta . T h is
c o r d b l o o d w ill r e m a in s ta b le f o r u p to 1 h o u r . In fa n ts w ith p H < 7 .0 a t b ir t h w h o a r e n o t
v i g o r o u s a r e a t h ig h r i s k o f a d v e r s e o u tc o m e . A n a ly s is o f p a ir e d a r te r ia l a n d v e n o u s s p e c im e n s
c a n g iv e in s ig h ts in to th e e t i o l o g y o f th e a c id o s is . In c o m b in a tio n w ith o th e r c lin ic a l
in f o r m a t io n , n o r m a l p a ir e d a r te r ia l a n d v e n o u s c o r d b l o o d g a s r e s u lts c a n u s u a lly p r o v id e a
r o b u s t d e fe n s e a g a in s t a s u g g e s tio n th a t a n in f a n t h a d a n in tr a p a r tu m h y p o x ic - is c h e m ic e v e n t.

47. A. T h e fe ta l h e a r t r a te v a r ie s b e a t to b e a t, w ith a n o r m a l h e a r t r a te r a n g in g b e tw e e n 110 a n d


1 6 0 b p m . T h is n o r m a l v a r ia b ility is th o u g h t to r e f le c t th e in te g r ity o f th e v a g a l n e u r a l p a th w a y
f r o m th e fe ta l c e r e b r a l c o r te x to th e c a r d ia c c o n d u c tio n s y s te m . F e ta l w e ll- b e in g is s a f e g u a r d e d
w h e n th is b e a t- to - b e a t v a r ia b ility is p re s e n t. C o n v e r s e ly , fe ta l d is tr e s s is a s s o c ia te d w ith m in im a l
( o r a b s e n t) v a r ia b ility o f th e fe ta l h e a r t ra te . O p io id s , b e n z o d ia z e p in e s , s e d a tiv e - h y p n o tic s , lo c a l
a n e s th e tic s , p h e n o th ia z in e s , a n d a n tic h o lin e r g ic s a d m in is te r e d to th e m o th e r h a v e b e e n s h o w n to
e lim in a te o r r e d u c e th is v a r ia b ility , e v e n in th e a b s e n c e o f d is tr e s s . T h is d r u g - r e l a te d e f f e c t d o e s
n o t a p p e a r to b e d e le te r io u s , b u t m a y c a u s e d if f ic u lty in in te r p r e tin g fe ta l h e a r t r a te m o n ito r s .

48. B. N o r m a lly , a m n io tic f lu id d o e s n o t e n te r th e m a te r n a l c ir c u la t io n b e c a u s e it is c o n ta in e d


s a f e ly w ith in th e u te ru s , s e a le d o f f b y th e a m n io tic s a c . A n A F E o c c u r s w h e n th e b a r r i e r b e tw e e n
a m n io tic f lu id a n d m a te r n a l c ir c u la t io n is b r o k e n , a llo w in g it to a b n o r m a lly e n te r th e m a te r n a l
v e n o u s s y s te m . T h e d e v a s ta tin g c o n s e q u e n c e o f c ir c u la tin g fe ta l d e b r is ( c a r r ie d b y a m n io tic
flu id ) o c c u r s o n ly r a r e ly . N e w e r th e o r ie s c o n te n d th a t A F E m o r e c lo s e ly r e s e m b le s a n
a n a p h y la c tic r e a c tio n to fe ta l d e b r is r a th e r th a n a tr u e e m b o lic e v e n t. C a r d in a l s ig n s in c lu d e
b r o n c h o s p a s m /w h e e z in g , h y p o x e m ia , s h o c k /m a te r n a l a c id o s is , c o a g u lo p a th y /d is s e m in a te d
in tr a v a s c u la r c o a g u la tio n , a n d a lte r e d m e n ta l sta tu s.

49. C. H y p e r v e n tila tio n a n d h y p o c a p n ia h a v e p r o f o u n d e ffe c ts o n c e r e b r a l b l o o d f lo w (C B F ),


r e s u ltin g in a 2 % d e c lin e in C B F f o r e a c h 1 to r r d e c lin e in P a c o 2. In th e s a m e m a n n e r , l o w C O 2
c o n c e n tr a tio n s w ill c a u s e v a s o c o n s tr ic tio n o f th e u te r in e v e s s e ls i m p a ir in g u te r in e b l o o d flo w .
In a d d itio n , m a te r n a l h y p o c a p n ia w ill s h if t th e o x y h e m o g lo b in d is s o c ia tio n c u r v e to th e left,
f u r th e r i m p a ir in g o x y g e n r e le a s e a n d tr a n s f e r to th e fe tu s . C a r b o n d io x id e te n s io n s in a r te r ia l
b l o o d p la y s a n im p o r ta n t r o l e in d e te r m in in g b l o o d f lo w a n d d e liv e r y to th e p r e g n a n t u te ru s ;
h y p o c a r b ia c a n le a d to p o o r o x y g e n a n d n u tr ie n t d e liv e r y w ith a r e s u ltin g fe ta l h y p o x ic d is tr e s s
( F ig 1 7 -5 ).
50. D. Hyperglycemia during pregnancy contriXbutes to higher birth weights (macrosomia) with
cephalopelvic insufficiency and shoulder dystocia requiring surgical delivery via cesarean
section. Fetal hyperglycemia induces fetal hyperinsulinemia, resulting in elevated metabolic
rates that lead to increased oxygen demand, risking ischemia. This hypoxemia in turn
contributes to the development of a metabolic acidosis and may trigger polycythemia with
increased production of red blood cells. At birth, however, with separation from the maternal
hyperglycemic circulation, neonatal hypoglycemia develops secondary to high levels of
neonatal insulin.

51. D. Opioids can be added to local anesthetics or alternatively can be used as a sole agent
(epidural or intrathecal) to provide peripartum analgesia. Side effects of neuraxial opioids
include pruritus, nausea/vomiting, hypotension, urinary retention, fetal bradycardia, and
maternal respiratory depression. Among these, pruritus occurs most commonly. The epidural
space contains an extensive vascular plexus with extensive systemic absorption such that
absorption of epidural morphine, fentanyl, or sufentanil produces opioid serum concentrations
that are similar to an equivalent dose of IM injection.

52. C. Glycopyrrolate, insulin, heparin, and neuromuscular blockers are unable to cross the
placenta, whereas opioids, local anesthetics, atropine, ephedrine, and P-blockers readily cross
the placenta to enter the neonatal circulation.

53. A. Reversal of neuromuscular blockade may be unsatisfactory in patients receiving


magnesium for preeclampsia as magnesium can enhance the neuromuscular blockade. Though
doses of 1 mg/kg have been used safely for rapid sequence induction of anesthesia, this dose
has not been tested for safety in pregnancy. Therefore, only a dose of 0.6 mg/kg is
recommended in this patient population despite the knowledge that rocuronium (similar to all
paralytics) does not readily cross the placental barrier, as evidenced by low umbilical-to-venous
plasma concentrations.
54. C. VAE has a very high incidence during cesarean sections secondary to entrainment of room
air from ruptured or severed veins. The sudden development of hypotension, hypoxia, and a
drop in end-tidal CO2 suggests the presence of a VAE. Supportive therapy includes flooding the
surgical field with normal saline and placing the patient in Trendelenburg position with a left-
lateral tilt. Medical gases such as carbon dioxide, nitrous oxide, and helium w ill aggravate this
condition by expanding the pulmonary vascular bubbles that can create an air lock with
mechanical flo w obstruction. If central venous catheter or pulmonary artery catheter is in place,
the trapped air may be aspirated. Expired nitrogen is the most sensitive VAE-detection method,
as the largest component of air is nitrogen.

55. B. General anesthesia is used most commonly in emergency surgical deliveries for fetal
distress. Endotracheal intubation follow ing rapid-sequence induction remains the principal
approach. Pregnancy-related features (obesity, enlarging breasts, short neck) make intubation
challenging such that inability to intubate the trachea is a major cause of m orbidity and
mortality. Current opinion is that after two failed attempts at direct laryngoscopy, one should
proceed with surgery as repeated trauma and edema may prevent ability to ventilate as well.
Severe fetal distress requires emergent delivery as opposed to preparing for an awake
fiberoptic intubation.

56. D. 15-Methyl PGF2a is a smooth-muscle constrictor with additional sites of action on


bronchial smooth muscles, which may promote bronchospasm. Direct intravenous bolus
injection of oxytocin (>5 units) has been associated with maternal hypotension and death, thus
should only be infused over time in a dilute solution (e.g., 40 units/L). Methylergonovine is a
potent vasoconstrictor that can cause vasospasm and severe hypertension if given intravenously,
such as usual administration intramuscularly. Misoprostol is a prostaglandin analog that can be
given up to 1 mg rectally or sublingually, but is to be used with caution in patients under
general anesthesia due to aspiration risk.

57. A. Overdose of magnesium during the peripartum period can lead to arreflexia and cardiac
conduction abnormalities, resulting in bradycardia and ectopy. Metabolic acidosis would be
seen in cases of severe hypovolemic shock. Examination findings expected with hypocalcemia
would include hyperreflexia, as opposed to arreflexia.

58. B. During pregnancy, hypertrophy and hyperplasia of lactotrophs result in enlargement of the
anterior pituitary, but without a corresponding increase in blood supply. As such, major
hemorrhage or hypotension during labor and delivery can result in anterior pituitary ischemia
and necrosis. Sheehan syndrome specifically is hypopituitarism caused by ischemic necrosis
during the peripartum period.
Pediatric Anesthesia
Dipty Mangla and Ashish Sinha

1. Correct statement regarding neonatal physiology is


A. Neonates have a greater volume of distribution for water-soluble drugs
B. Total body water is higher in adults
C. Dose of propofol (mg/kg) is lower in neonates than in adults
D. Neonates have a higher body fat content than adults

2. A 4-year-old child weighing 16 kg is scheduled for hernia repair under general anesthesia.
Assuming he was NPO for 8 hours, his total fluid deficit w ill be about_____(mL):
A. 380
B. 420
C. 460
D. 500

3. The total dose of midazolam that may be given orally as premedication is


A. 0.2 mg/kg, maximum 10 mg
B. 0.2 mg/kg, maximum 20 mg
C. 0.5 mg/kg, maximum 15 mg
D. 0.5 mg/kg, maximum 20 mg

4. A newborn baby of 37 weeks of gestation has a heart rate of 90 bpm, is crying, is pink with blue
extremities, and shows some flexion. Her Apgar score would be
A. 6
B. 7
C. 8
D. 9

5. After initial evaluation of the baby described above, the next step in managing her would be
A. Provide positive-pressure ventilation
B. Chest compressions
C. Warming blanket
D. Cardiology consult
6. A ll of the follow ing drugs can be given through endotracheal tube, except
A. Epinephrine
B. Lidocaine
C. Surfactant
D. Calcium

7. The disease or syndrome with known association with malignant hyperthermia is


A. Huntington chorea
B. Fabry disease
C. King Denborough syndrome (KDS)
D. Burns

8. An 8-year-old child is brought to the emergency room with testicular torsion. The parents tell
you he ate a sandwich 6 hours ago. Surgeon wants to operate immediately. Your response
should be
A. Take him to the OR, deem it emergent, rapid-sequence intubation
B. Wait for 2 more hours, deem it urgent, rapid-sequence intubation
C. He is adequately fasting, elective, intubation
D. Wait for 2 hours, elective, intubation

9. Which of the follow ing statements about pediatric airway is true?


A. More caudal position of larynx as compared to adult
B. More acute angulation of epiglottis
C. Glottic opening is the narrowest part of airway
D. Longer trachea as compared to adults

10. A 10-week-old baby, who was born prematurely at 30 weeks of gestation, undergoes
circumcision uneventfully under general anesthesia. After the baby recovers from anesthetics in
postanesthesia care unit, he can/should be
A. Admitted and monitored for 24 hours
B. Discharged home with parents
C. Discharged home if parents live within a 30-minute radius
D. Admitted to the ICU

11. Hypertrophic pyloric stenosis is associated with


A. Metabolic acidosis
B. Metabolic alkalosis
C. Hyperkalemia
D. Hyperchloremia

12. A child with which of the follow ing diseases/syndromes should be evaluated for heart disease?
A. Omphalocele
B. Gastroschisis
C. Hypertrophic pyloric stenosis
D. Tracheobronchitis

13. The earliest and the most pathognomic feature of malignant hyperthermia (MH) is
A. Increased temperature
B. Increased end-tidal CO2
C. Increased heart rate
D. Increased respiratory rate

14. The most common type of tracheoesophageal fistula (TEF) is

15. Down syndrome is associated with all of the follow ing, except
A. Large tongue
B. Atlantooccipital instability
C. Hyperthyroidism
D. Increased incidence of seizures

16. The first sign of intrathecal injection follow ing the placement of caudal epidural with 0.25%
bupivacaine in a 1-year-old child would be
A. Hypotension
B. Bradycardia
C. Falling oxygen saturation
D. Tachycardia

17. A 2-year-old child weighing 13 kg is scheduled for inguinal hernia repair. The calculated dose
of 0.25% bupivacaine for a caudal epidural would be approximately______(mL):
A. 13
B. 7
C. 10
D. 20
18. A ll the follow ing are physiologic changes that occur at birth, except
A. Closure of foramen ovale
B. Closure of ductus arteriosus
C. Decreased right-ventricular afterload
D. Decreased left-ventricular afterload

19. Neonates lose heat by all the follow ing mechanisms in the operating room, except
A. Conduction to cold surfaces
B. Exposure to cold operating room
C. Dry airway gases
D. Metabolism of brown fat

20. A 4-year-old child with tetralogy of Fallot is scheduled for incision and drainage of a foot
abscess. A ll the follow ing measures can be used to improve his oxygenation, except
A. Phenylephrine
B. Nitroglycerine
C. N itric oxide
D. Epinephrine

21. Which of the follow ing heart rates is inappropriate for the age?
A. 50 bpm at 12 years of age
B. 120 bpm for a neonate
C. 100 bpm for a 1-year-old
D. 80 bpm for a 3-year-old

22. The age at which the glomerular filtration rate in a child is same as in adults is
A. 6 months
B. 1 year
C. 1.5 years
D. 2 years

23. Normal blood glucose level in a neonate is ______(mg/dL):


A. 20 to 40
B. 40 to 60
C. 60 to 70
D. 50 to 80

24. The recommended size of an endotracheal tube for a 1-year-old child is


A. 2.5
B. 3.0
C. 4.0
D. 5.0

25. As compared to a 10-year-old child, a 1-year-old child w ill have higher


A. Oxygen consumption
B. Functional residual capacity
C. Tidal volume
D. Vital capacity

26. The total blood volume in a preterm is ______(mL/kg):


A. 90 to 100
B. 70 to 80
C. 50 to 60
D. 80 to 90

27. A 2-year-old is scheduled for elective tonsillectomy and adenoidectomy. His mother tells you
he has runny nose. Your decision whether to proceed w ill be based on all the following, except
A. If he is afebrile
B. If he is not actively wheezing
C. Cancel the surgery since it is elective
D. Reluctance of parent for admitting the child, if needed

28. Urine output in a 6-year-old child undergoing surgery under general anesthesia should be
(mL/kg/h):
A. 0.5
B. 1
C. 1.5
D. 2

29. Perioperative management of a child with a femur fracture and sickle cell disease includes all
of the following, except
A. Hydration
B. Treat infections
C. Transfuse to hemoglobin of 14 mg/dL
D. Avoid metabolic acidosis

30. Anesthetic management of a 12-year-old with Down syndrome includes all of the following,
except
A. Continue antiseizure medications
B. Heavy sedation since all such patients are combative
C. Prepare for manual in line neck stabilization
D. Radiographs of the neck should be reviewed to rule out atlantooccipital instability

31. An 8-year-old boy, weighing 30 kg, is undergoing resection of a Wilms tumor in the operating
room. His starting hemoglobin is 12 g/dL. If the threshold for transfusion is 8 g/dL, the
allowable blood loss is _____ (mL):
A. 820
B. 840
C. 860
D. 880

32. A 5-year-old otherwise-healthy child is undergoing strabismus surgery with a laryngeal mask
airway (LM A) in place. Thirty minutes into the procedure, his heart rate is 60 bpm, blood
pressure is 90/60 mm Hg, and the pulse oximeter reads 98%. The next step in management
should be
A. Replace the LM A with an endotracheal tube
B. Inform surgeon, administer atropine
C. Nothing, this is normal for this child
D. Increase the Fio 2 to 1.0

33. The afferent lim b for oculocardiac reflex is


A. Vagus nerve
B. Trigeminal nerve
C. Glossopharyngeal nerve
D. Facial nerve

34. Positive-pressure ventilation with a face mask is contraindicated in which of the follow ing
condition?
A. Laryngospasm
B. Congenital diaphragmatic hernia
C. Trauma
D. Asthma

35. Treatment of postintubation croup in a child who underwent adenoidectomy is


A. Inhalation of mist
B. Steroids
C. Racemic epinephrine
D. A ll of the above

36. The most important measure to avoid subglottic edema in children is


A. Use of an appropriate-size endotracheal tube
B. Lubricating the endotracheal tube prior to intubation
C. Administering intravenous lidocaine for all intubations
D. Administering intravenous steroids for all intubations

37. Important difference between epiglottitis and laryngotracheobronchitis (croup) is


A. Croup responds to racemic epinephrine and steroids
B. Croup occurs in older children
C. Higher temperatures are seen in croup patients
D. Etiology of croup is bacterial

38. Anesthesia in a patient with Pierre Robin syndrome can be complicated by


A. Renal failure
B. Tendency to develop malignant hyperthermia
C. Cardiac failure
D. D ifficult airway

39. Basic metabolic rate in children is


A. Least at 1 year of age
B. Same as adults
C. Highest till 2 years of age
D. Decreases after puberty

40. The percentage of patients developing malignant hyperthermia (MH) after masseter spasm is
A. 0% to 24%
B. 25% to 49%
C. 50% to 74%
D. 75% to 100%

41. A ll of the follow ing are true for children with congenital diaphragmatic hernia (CDH), except
A. Pulmonary hypoplasia may be present
B. Dextrocardia is common
C. Bag and mask ventilation is contraindicated
D. Surgical management takes precedence over medical management

42. To protect lungs in a child with tracheoesophageal fistula, all the follow ing should be done,
except
A. Avoid feeding
B. Upright position
C. Intermittent suction of upper blind esophageal pouch
D. Prophylactic intravenous steroids

43. The main factor responsible for physiologic closure of a patent ductus arteriosus is
A. Increased Paco2
B. Increased Pao2
C. Increased pulmonary artery pressure
D. Administration of nonsteroidal anti-inflammatory agents

44. The most effective method for maintaining normothermia in an operating room is
A. Warm humidified gases
B. Warm intravenous fluids
C. Warming blankets
D. Increasing the room temperature

45. A 2-year-old child undergoing myringotomy develops laryngospasm in the operating room.
The patient is breathing spontaneously with face mask at an Fio 2 of 0.6. Next step in the
management would be
A. Increasing Fio 2 to 1.0
B. Jaw thrust
C. Endotracheal intubation
D. Intramuscular succinylcholine

46. Normal pulmonary dead space in a neonate is ______(mL/kg):


A. 1
B. 2
C. 3
D. 4

47. Which of the follow ing statements regarding fetal hemoglobin is true?
A. It is composed of two a and two P chains
B. It has more affinity for oxygen than adult hemoglobin
C. Patients with sickle cell disease and high fetal hemoglobin have poor prognosis
D. None of the above

48. Compared to adults, oxygen desaturation is more frequent in pediatric population because of
A. Lower functional residual capacity (FRC) in children
B. Higher oxygen consumption in adult
C. Lower heart rate in adults
D. Lower functional residual capacity in adults

49. The most consistent sign of intravascular injection follow ing caudal epidural with 0.25%
bupivacaine with 1:200,000 epinephrine is
A. Tachycardia
B. ST segment changes
C. Bradycardia
D. Hypertension

50. The dose of nondepolarizing muscle relaxants in a neonate is


A. Decreased as compared to adults
B. Increased as compared to adults
C. Same as adults
D. Cannot be predicted
CHAPTER 18 ANSWERS

1. A. Total body water in a term neonate is 75% of the total body weight, as compared to 60% in
adult males and 55% in adult females. Water-soluble drugs w ill have an increased volume of
distribution because of increased body water. Propofol dose (mg/kg) w ill be higher in neonates
and infants than adults.

2. B. Maintenance fluid requirements to replace fluid deficits accounting for a period of fasting
can be calculated by the follow ing formula.

Table 18-1 Calculation of maintenance fluid requirements


W e ig h t H o u r ly F lu id R e q u i r e m e n t

<10 kg 4 mL/kg
10-20 kg 40 mL + 2 mL/kg > 10 kg
>20 kg 60 mL/kg + 1 mL/kg > 20 kg

Thus using above formula, total fluid deficit would be 40 + (6 x 2) = 52 mL/h.


Accounting for 8 hours of fasting, total fluid deficit w ill be 52 x 8 = 416 mL.

3. D. Preoperatively, midazolam is the most common medication given for sedation and anxiety.
Midazolam can be given orally in a dose of 0.25 to 0.5 mg/kg (maximum dose of 20 mg) in
children. Sedative premedication is generally omitted for neonates, infants, and sick children.
Oral ketamine (4-6 mg/kg) can also be used as premedication. For uncooperative children,
intramuscular midazolam (0.1-0.15 mg/kg, maximum of 10 mg) and ketamine (0.02 mg/kg) can
be used.

4. B. Apgar scores recorded at 1 minute and 5 minutes after birth remains a valuable method for
assessment of the well-being of a neonate.

Table 18-2 Apgar Score


S ig n P o in ts

0 1 2

H e art rate (bpm ) A bsent <100 >100


R espiratory effort A bsent Slow, irregular G o o d crying
M uscle tone Flaccid Som e flexion Active m o tio n
Reflex irritability N o response G rim ace C rying
C olor Blue o r pale B ody p ink, blue extrem ities All p in k

Apgar score in this case would be 1 + 2 + 1 + 2 + 1 = 7.

5. A. Indications of positive-pressure ventilation in a newborn include apnea, gasping


respirations, persistent central cyanosis with 100% oxygen, and heart rate less than 100 bpm.
Assisted ventilation by bag and mask should be at a rate of 30 to 60 bpm with 100% oxygen. If
after 30 seconds the heart rate is less than 80 bpm, chest compressions should be started and the
neonate should be intubated (Fig 18-2).

F igu re 18-2.Reused with permission from Kattwinkel J, Perlman JM, Aziz K, et al. 2010 American Heart Association Guidelines for
Cardiopulmonary Resuscitation and Emergency Cardiovascular Care Science. Part 15: Neonatal Resuscitation. C i r c u l a t i o n .
2010;122:S909-S919.

6. D. Lidocaine, epinephrine, atropine, and vasopressin can be delivered down a catheter whose
tip extends beyond the endotracheal tube. The dose of drugs through endotracheal tube is 2 to
2.5 times the intravenous dose. Surfactant can be given through endotracheal tube in children
with severe bronchopulmonary dysplasia.

7. C. Musculoskeletal diseases associated with a relatively high incidence of malignant


hyperthermia include Duchenne muscular dystrophy, myotonia, and KDS. KDS is seen
prim arily in young boys who exhibit short stature, mental retardation, cryptorchidism,
kyphoscoliosis, pectus deformity, slanted eyes, low-set ears, webbed neck, and winged scapulae.
8. A. A male presenting with sudden onset of acute scrotal pain in the absence of trauma should
be suspected to have testicular torsion. Testicular torsion requires immediate investigation and
possible surgery to preserve potentially viable testis. Surgery should be performed within 6
hours of onset of pain to save the testicle. The salvage rate decreases to 50% if surgery is
delayed between 6 and 12 hours. Children with suspected torsion of testis are assumed to have a
fu ll stomach and should have a rapid-sequence endotracheal intubation. The surgery is
emergent and the patient needs to be taken to the OR.

Table 18-3 Preanesthesia Fasting Guidelines for Pediatric Patients


P roduct M in im a l F a s tin g T im e

Clear liquids 2 hours


Breast milk 4 hours
Infant formula, Jell-O 6 hours
Solid food 8 hours

9. B. Neonates and infants have a larger head and tongue, an anterior and cephalad epiglottis and
larynx, and a short trachea and neck. The larynx is at a vertebral level of C4 versus C6 in adults.
The narrowest portion of larynx in children is at the level of cricoid cartilage as compared to
glottic opening in adults. An adult’s epiglottis is flat and broad, and its axis is parallel to that of
trachea, whereas an infant’s epiglottis is typically narrower, omega-shaped, and angled away
from the axis of trachea.

10. A. Premature infants who are less-than-50-weeks postconceptional age at the time of surgery
are prone to postoperative apneic episodes for up to 24 hours. Besides prematurity, other risk
factors for postanesthetic apnea include hematocrit <30% (anemia), hypothermia, and
neurological abnormalities. Thus, elective or outpatient procedures should be deferred until the
preterm infant reaches the age of at least 50 weeks’ postconception. These patients should be
monitored for 12 to 24 hours postoperatively with pulse oximetry.

11. B. Hypertrophic pyloric stenosis causes stasis of gastric contents and thus leads to persistent
vomiting. This can lead to depletion of sodium, potassium, chloride, and hydrogen ions,
causing a hypochloremic metabolic alkalosis. Patients are first medically stabilized (correction
of volume-deficit and metabolic alkalosis), and then a pyloromyotomy is performed. Hydration
should be done with a sodium chloride solution supplemented with potassium (avoidance of
ringer lactate as it is metabolized to bicarbonate).

12. A. Both gastroschisis and omphalocele are congenital disorders characterized by defects in
the abdominal wall. Omphaloceles have a hernia sac, and are often associated with other
congenital anomalies (trisomy 21, diaphragmatic hernia, cardiac and bladder anomalies).
Gastroschisis, on the other hand, does not have a hernia sac, and is often an isolated finding.
The latter is a more serious condition, as the absence of a hernial sac can lead to dehydration,
hypothermia, and infection.
13. B. M H is a r a r e b u t p o te n tia lly fa ta l h y p e r m e ta b o lic d is o r d e r t r i g g e r e d b y e x p o s u r e to v o la tile
in h a la tio n a l a n e s th e tic s o r s u c c in y lc h o lin e . T h e in c id e n c e o f M H is 1 :1 5 ,0 0 0 in p e d ia tr ic
p o p u la tio n a n d 1 :5 0 ,0 0 0 in a d u lts . S ig n s o f M H in c lu d e m a s s e te r m u s c le r ig id ity , ta c h y c a r d ia ,
ta c h y p n e a , h y p e r c a r b i a ( in c r e a s e d C O 2 p r o d u c tio n — e a r lie s t s ig n ) , a n d h y p e r th e r m ia (la te s ig n ) .
H y p e r te n s io n a n d a r r h y th m ia s m a y b e s e e n ( s y m p a th e tic o v e r a c tiv ity ) . G e n e r a liz e d m u s c le
r i g i d i t y is n o t c o n s is te n tly p r e s e n t, a n d p r e s e n c e o f d a r k - c o l o r e d u r in e in d ic a te s m y o g lo b in u r ia .

14. C. A m o n g th e d if f e r e n t ty p e s o f T E F , th e m o s t c o m m o n is th e ty p e TTTB . T h is is w h e r e th e
u p p e r e s o p h a g u s e n d s in a b lin d p o u c h a n d a lo w e r e s o p h a g u s th a t c o n n e c ts to th e tr a c h e a . A t
b ir th , T E F is s u s p e c te d b y f a ilu r e to p a s s a c a th e te r in to th e s to m a c h a n d v is u a liz a tio n o f th e
c a th e te r c o ile d in th e b lin d u p p e r e s o p h a g e a l p o u c h . T y p ic a lly , b r e a th in g le a d s to g a s tr ic
d is te n s io n a n d f e e d in g le a d s to c h o k in g a n d c y a n o s is . T E F p a tie n ts a re , th e r e f o r e , p r o n e to
p u lm o n a r y a s p ir a tio n . C o e x is te n c e o f c a r d ia c c o n g e n ita l a n o m a lie s is c o m m o n . T E F p a tie n ts
m a y h a v e a s s o c ia te d v e r te b r a l d e fe c ts , a n a l a tr e s ia , a n d r a d ia l d y s p la s ia , k n o w n a s th e V A T E R
s y n d r o m e . A d d itio n o f c a r d ia c a n d limb a n o m a lie s is c a lle d th e V A C T E R L v a r ia n t.

15. C. D o w n s y n d r o m e o r tr i s o m y 21 is o n e o f th e m o s t c o m m o n c o n g e n ita l s y n d r o m e s in
p e d ia tr ic p o p u la tio n . A n e s th e tic c o n s id e r a tio n s in th e s e p a tie n ts in c lu d e p r e s e n c e o f s h o r t n e c k
a n d l a r g e to n g u e ( p o s s ib le d if f ic u lt a ir w a y ) , i r r e g u l a r d e n titio n , m e n ta l r e ta r d a tio n , h y p o to n ia ,
c o n g e n ita l h e a r t d is e a s e in 3 0 % to 4 0 % o f p a tie n ts ( p a r tic u la r ly e n d o c a r d ia l c u s h io n d e fe c ts a n d
v e n tr ic u la r s e p ta l d e fe c t), s u b g lo ttic s te n o s is , tr a c h e o e s o p h a g e a l fis tu la , c h r o n ic p u lm o n a r y
in f e c tio n s , s e iz u r e s , d u o d e n a l s te n o s is , a n d d e la y e d g a s tr ic e m p ty in g .

16. C. U n lik e o ld e r c h ild r e n a n d a d u lts , s u b a r a c h n o id a n d e p id u r a l b lo c k a d e in in fa n ts a n d s m a ll


c h ild r e n is c h a r a c te r iz e d b y h e m o d y n a m ic s ta b ility , e v e n w h e n th e le v e l o f b lo c k r e a c h e s u p p e r
d e r m a to m e s . Y o u n g c h ild r e n r e l y m o r e o n th e d ia p h r a g m f o r m a in ta in in g tid a l v o lu m e s ; th u s,
a p n e a m a y b e th e f i r s t s ig n o f to ta l s p in a l in in fa n ts a n d s m a ll c h ild r e n .

17. B. A r m ita g e f o r m u l a c a n b e u s e d f o r c a lc u la tio n o f c a u d a l b u p iv a c a in e in a c h ild w ith


a p p r o p r ia te w e ig h t f o r h is a g e .

0 .5 m L /k g f o r a lu m b o s a c r a l b lo c k
1 m L /k g f o r a th o r a c o lu m b a r b lo c k
1.2 5 m L /k g f o r a m id th o r a c ic b lo c k
0 .2 5 % B u p iv a c a in e u p to a m a x im u m o f 2 0 m L

18. D. F e ta l c ir c u la t io n is a s s o c ia te d w ith in c r e a s e d p u lm o n a r y v a s c u la r r e s is ta n c e , d e c r e a s e d
p u lm o n a r y b l o o d flo w , d e c r e a s e d s y s te m ic v a s c u la r r e s is ta n c e , a n d r i g h t to le f t b l o o d f lo w
th r o u g h p a te n t d u c tu s a r te r io s u s a n d f o r a m e n o v a le . A t b ir th , th e o n s e t o f s p o n ta n e o u s
v e n tila tio n a n d e lim in a tio n o f p la c e n ta l c ir c u la t io n d e c r e a s e s p u lm o n a r y v a s c u la r r e s is ta n c e a n d
in c r e a s e s p u lm o n a r y b l o o d flo w . S im u lta n e o u s ly , s y s te m ic v a s c u la r r e s is ta n c e in c r e a s e s , le ft-
a tr ia l p r e s s u r e in c r e a s e s , f o r a m e n o v a le c lo s e s fu n c tio n a lly , a n d r ig h t - to - le f t s h u n tin g c e a s e s .
W h e n a n a to m ic c lo s u r e is a c h ie v e d a n d th e c a r d ia c a n a to m y is n o r m a l, s h u n tin g th r o u g h d u c tu s
a r te r io s u s c e a s e s .
19. D. Neonates are susceptible to increased heat losses due to thin skin, low fat content, and a
higher relative body surface area. Cold operating room, wound exposure, unwarned
intravenous fluid administration, dry anesthetic gases, and the direct effect of anesthetic agents
on temperature regulation can further accelerate heat loss. Hypothermia is associated with
delayed awakening from anesthesia, cardiac irritability, respiratory depression, increased
pulmonary vascular resistance, altered drug responses, delayed wound healing, and coagulation
and platelet dysfunction. Metabolism of brown fat is responsible for heat production in infants.

20. D. Tetralogy of Fallot consists of right-ventricular obstruction, right-ventricular hypertrophy,


and a ventricular septal defect with an overriding aorta. About 20% of patients also have
pulmonic stenosis. Anesthetic management of a child with tetralogy of Fallot includes adequate
preoperative hydration, avoiding factors that can increase pulmonary vascular resistance,
maintaining systemic vascular resistance (SVR), and avoid increases in heart rate that may
worsen infundibular stenosis. Hypercyanotic spells are treated by volume administration,
sedation, and administration of drugs that increase SVR such as phenylephrine. Propranolol
may be given to relieve infundibular spasm. Epinephrine in this situation may worsen cyanosis
by increasing HR and decreasing SVR.

21. A. Normal cardiovascular variables in children:

Table 18-4

P a r a m e te r N e o n a te In fa n t 5 Years A d u lt

O xygen co n su m p tio n (m L /kg/m in) 6 5 4 3


Systolic b lo o d pressure (m m Hg) 65 90 -9 5 95 120
H e art rate (bpm ) 130 120 90 80

22. D. Premature neonates have decreased creatinine clearance, impaired sodium retention,
glucose excretion, and bicarbonate reabsorption, and poor diluting and concentrating ability.
Normal kidney function may develop anywhere from 6 months to 2 years of age. Therefore, it
is extremely important to pay meticulous attention to fluid management in children less than 2
years of age.

23. B. Laboratory value of blood glucose in children:

Table 18-5

24. C. The approximate diameter inside the endotracheal tube can be estimated by a formula based
on age:

Tube diameter in mm = (Age in years/4) + 4

Exceptions include premature neonates (2.5- to 3.0-mm tube) and full-term neonates (3.0- to
3.5-mm tube).
Formula for the length of endotracheal tube at the lip:

Length = 12 + Age/2

25. A. Metabolic rate and oxygen consumption are higher in infants than in older children. Rest of
the parameters in the question remain the same per weight basis in younger and older children.
Respiratory rate is increased in neonates and gradually falls to adult levels by adolescence.
Airway resistance is increased in neonates due to a relative paucity of small airways. The high
metabolic rate and oxygen consumption lim it oxygen reserves during periods of apnea (e.g.,
intubation) and predispose neonates and infants to atelectasis and hypoxemia.

26. A. Developmental changes in blood volume:

Table 18-6

A ge B lo o d V o lu m e (m L /k g )

Preterm 90-105
Termneonate 78-86
1-12 Months 73-78
1-3 Years 74-82
4-6 Years 80-86
7-18 Years 83-90
Adults 68-88

27. C. Presence of an acute purulent upper respiratory infection, fever, change in mental status, or
signs of lower respiratory tract infection (wheezing, rales), especially in a child, is sufficient to
postpone the surgery.
Factors affecting decision for elective surgery in a child with upper respiratory tract infection
are as follows:
Pros

• Presence of runny nose alone


• Active, happy child
• Older child
• Clear lungs

Cons

• Recent development of symptoms within 1 to 2 days


• Lethargic child
• Purulent nasal discharge
• Wheezing, rales
• Child <1 year, ex-premature
Major surgery

28. A. Urine output should be monitored in all children undergoing surgeries involving major
fluid shifts. The fluid therapy should be aimed at maintaining a urine output of 1 mL/kg/h.

29. C. Optimal preoperative preparation in patients with sickle cell anemia includes adequate
hydration, treatment of infections, and an acceptable hemoglobin concentration. Preoperative
transfusion therapy in sickle cell patients is individualized to the patient and to the surgical
procedure. The goal of transfusion therapy is to achieve a hematocrit of 35% to 40%, with 40%
to 50% normal hemoglobin. Hemoglobin desaturation or low -flow states (stasis) should be
avoided in sickle cell patients. Therefore, tourniquet use should be avoided during surgical
procedures. Conditions that could cause hemoglobin desaturation or stasis include hypothermia
or hyperthermia, acidosis, hypoxemia, hypotension, or hypovolemia.

30. B. Patients with Down syndrome exhibit a short neck, irregular dentition, mental retardation,
hypotonia, large tongue, congenital heart disease (in 40% of patients, endocardial cushion
defects, ventricular septal defect), subglottic stenosis, tracheoesophageal fistula, chronic
pulmonary infections, and seizures. Down syndrome patients often have a difficult airway (use
of smaller size of the endotracheal tube). Excessive neck flexion during laryngoscopy or
intubation may result in atlantooccipital dislocation because of the laxity of the ligaments.
Postoperative stridor and apnea are common in these patients. Antiseizure medications should
be continued perioperatively.

31. B. Maximum allowable blood loss during surgery can be calculated by the follow ing formula:

Maximum allowable blood loss = Patient’s hemoglobin - Allowed hemoglobin/Average of the two x
EBV

EBV or expected blood volume = 70 mL/kg x weight of the child

32. B. Traction on extraocular muscles or pressure on the eyeball can result in cardiac
dysrhythmias (bradycardia, ventricular ectopy, ventricular fibrillation). This reflex, called the
oculocardiac reflex, consists of a trigeminal afferent and a vagal efferent pathway. The reflex
can occur in patients undergoing ocular procedures such as cataract extraction, enucleation, and
retinal detachment repair. In awake patients, the oculocardiac reflex may be associated with
somnolence and nausea. Management of the oculocardiac reflex consists of (1) immediate
notification to the surgeon and temporary cessation of surgical stimulation, (2) confirmation of
adequate ventilation, oxygenation, and depth of anesthesia, (3) administration of intravenous
atropine (10 |ig/kg) if the conduction disturbance persists, and (4) infiltration of the rectus
muscles with local anesthetic. Also, retrobulbar block performance can elicit the oculocardiac
reflex.

33. B. Traction on extraocular muscles or pressure on the eyeball can result in cardiac
d y s r h y th m ia s ( b r a d y c a r d ia , v e n tr ic u la r e c to p y , v e n tr ic u la r f ib r illa tio n ) . T h is r e f le x , c a lle d th e
o c u lo c a r d ia c r e f le x , c o n s is ts o f a tr i g e m in a l a f f e r e n t a n d a v a g a l e f f e r e n t p a th w a y . T h e r e f le x
c a n o c c u r in p a tie n ts u n d e r g o in g o c u la r p r o c e d u r e s s u c h a s c a ta r a c t e x tr a c tio n , e n u c le a tio n , a n d
r e tin a l d e ta c h m e n t r e p a ir . In a w a k e p a tie n ts , th e o c u lo c a r d ia c r e f le x m a y b e a s s o c ia te d w ith
s o m n o le n c e a n d n a u s e a . M a n a g e m e n t o f th e o c u lo c a r d ia c r e f le x c o n s is ts o f (1 ) im m e d ia te
n o tif ic a tio n to th e s u r g e o n a n d t e m p o r a r y c e s s a tio n o f s u r g ic a l s tim u la tio n ; (2 ) c o n f ir m a tio n o f
a d e q u a te v e n tila tio n , o x y g e n a tio n , a n d d e p th o f a n e s th e s ia ; (3 ) a d m in is tr a tio n o f in tr a v e n o u s
a tr o p in e (1 0 |ig /k g ) if th e c o n d u c tio n d is tu r b a n c e p e r s is ts ; a n d (4 ) in f i ltr a tio n o f th e re c tu s
m u s c le s w ith lo c a l a n e s th e tic . A ls o , r e tr o b u lb a r b lo c k p e r f o r m a n c e c a n e lic it th e o c u lo c a r d ia c
r e f le x .

34. B. P o s itiv e p r e s s u r e w ith a fa c e m a s k c a n b e a lif e s a v in g te m p o r iz in g m e a s u r e in s itu a tio n s


s u c h a s la r y n g o s p a s m , h y p o x ia , a n d e v e n d if f ic u lt in tu b a tio n . H o w e v e r, it is c o n tr a in d ic a te d in
s itu a tio n s w h e r e th e r e is a n in c r e a s e d r i s k o f a s p ir a tio n . In p a tie n ts w ith c o n g e n ita l
d ia p h r a g m a tic h e r n ia a n d tr a c h e o e s o p h a g e a l fis tu la , p o s itiv e - p r e s s u r e v e n tila tio n w ith a fa c e
m a s k is r e la tiv e ly c o n tr a in d ic a te d .

35. D. P e r io p e r a tiv e p o s tin tu b a tio n c r o u p o c c u r s in 0 .1 % to 1 % o f c h ild r e n . F a c to r s a s s o c ia te d


w ith in c r e a s e d r i s k o f c r o u p in c lu d e a l a r g e r o u te r d ia m e te r o f e n d o tr a c h e a l tu b e r e la tiv e to
a irw a y , f r e q u e n t p a tie n t p o s itio n c h a n g e s , m u ltip le in tu b a tio n a tte m p ts , tr a u m a tic in tu b a tio n , a n d
c h ild r e n a g e d 1 to 4 y e a r s . T r e a tm e n t in c lu d e s h u m id if ie d m is t, n e b u liz e d r a c e m ic e p in e p h r in e ,
a n d d e x a m e th a s o n e .

36. A. P e r io p e r a tiv e p o s tin tu b a tio n c r o u p o c c u r s in 0 .1 % to 1 % o f c h ild r e n . F a c to r s a s s o c ia te d


w ith in c r e a s e d r i s k o f c r o u p in c lu d e a l a r g e r o u te r d ia m e te r o f e n d o tr a c h e a l tu b e r e la tiv e to
a irw a y , f r e q u e n t p a tie n t p o s itio n c h a n g e s , m u ltip le in tu b a tio n a tte m p ts , tr a u m a tic in tu b a tio n , a n d
c h ild r e n a g e d 1 to 4 y e a r s . T r e a tm e n t in c lu d e s h u m id if ie d m is t, n e b u liz e d r a c e m ic e p in e p h r in e ,
a n d d e x a m e th a s o n e .

37. A. C r o u p is u p p e r a ir w a y o b s tr u c tio n c h a r a c te r iz e d b y a b a r k in g c o u g h . It c o u ld b e
p o s tin tu b a tio n c r o u p o r a r e s u lt o f v i r a l in f e c tio n . In c id e n c e o f in f e c tio u s c r o u p is in c r e a s e d in
c h ild r e n a g e d 3 m o n th s to 3 y e a r s . I n f e c tio u s c r o u p p r o g r e s s e s s lo w ly , a n d p a tie n ts r a r e l y
r e q u i r e in tu b a tio n . It is tr e a te d w ith n e b u liz e d r a c e m ic e p in e p h r in e a n d d e x a m e th a s o n e . A c u te
e p ig lo ttitis is a b a c te r ia l in f e c tio n c o m m o n ly d u e to Haemophilus influenzae ty p e B . It a ffe c ts
c h ild r e n o f 2 to 6 y e a r s o ld . A c u te e p ig lo ttitis c a n r a p id ly p r o g r e s s f r o m a s o r e th r o a t to
c o m p le te a ir w a y o b s tr u c tio n . E n d o tr a c h e a l in tu b a tio n ( s p o n ta n e o u s b r e a th in g in h a la tio n a l
in d u c tio n in s ittin g p o s itio n ) a n d a n tib io tic th e r a p y c a n b e lif e s a v in g .

38. D. P ie r r e R o b in s y n d r o m e is a g e n e tic d is o r d e r c h a r a c te r iz e d b y h y p o p la s tic m a n d ib le ,


p s e u d o m a c r o g l o s s i a , a n d h ig h a r c h e d a n d c le f t p a la te . L a r g e to n g u e a n d s m a ll m o u th c a n le a d
to b o th d if f ic u lt v e n tila tio n a n d in tu b a tio n .

39. C. C h ild r e n h a v e a h ig h e r m e ta b o lic r a te th a n a d u lts u n til 2 y e a r s o f a g e . P e d ia tr ic p a tie n ts


have a larger surface area per kilogram than adults (increased surface area/weight ratio).
Besides the higher surface area accounting for the higher metabolic rate, children (especially
neonates) also loose heat to a greater extent.

40. B. Masseter muscle spasm (MMS) may be seen in pediatric patients after the administration of
succinylcholine. About 50% of patients in whom MMS develops prove to be susceptible to MH
by muscle testing.

41. D. During fetal development, the gut can herniate into the thorax through diaphragmatic
defects, with left-sided herniation is the most common type (90%). The reported incidence of
diaphragmatic hernia is about 1:5000 live births. Clinical features of diaphragmatic herniation
include hypoxia, a scaphoid abdomen, and evidence of bowel in the thorax (confirmed with
auscultation or radiography). Pulmonary hypoplasia and malrotation of the intestines are
commonly associated. While the ipsilateral lung is particularly affected, the herniated gut can
compress and retard the maturation of both the lungs. Pulmonary hypertension is common.
CDH is often associated with dextrocardia. The goal of initial management of CDH is to avoid a
surgical intervention when the infant is hypoxic and acidotic. Instead, medical management is
directed to stabilizing the infant’s cardiorespiratory status by improving oxygenation,
correcting metabolic acidosis, reducing the right-to-left shunting, and increasing pulmonary
perfusion.

42. D. Interventions to protect the lungs from aspiration in the presence of tracheoesophageal
fistula include

• Avoidance of feedings
• Upright positioning of the infant to decrease the likelihood of gastroesophageal reflux (30-
degree elevation)
• Antibiotic therapy and physiotherapy if pneumonia is diagnosed
• Intermittent suctioning of the upper blind esophageal pouch

43. B. At birth, initiation of spontaneous ventilation and elimination of the placental circulation
decrease pulmonary vascular resistance and increase pulmonary blood flow. At the same time,
systemic vascular resistance increases, left-atrial pressure increases, foramen ovale closes
functionally, and the right-to-left shunting ceases. When anatomic closure is achieved and
cardiac anatomy is normal, shunting through ductus arteriosus also ceases.

44. C. Hypothermia is defined as a body temperature less than 36°C. Prewarming for half an hour
with convective forced-air warming blankets effectively prevents phase I (initial rapid decline
in body temperature) hypothermia by eliminating the central-peripheral temperature gradient.
Methods to minimize phase II hypothermia (slower decrease in body temperature) from heat
loss include use of forced-air warming blankets and warm-water blankets, heated
humidification of inspired gases, warming of intravenous fluids, and raising ambient operating
room temperature. Passive insulators such as heated cotton blankets or the so-called space
b la n k e ts h a v e little u tility in p r e v e n tin g h y p o th e r m ia .

45. A. L a r y n g o s p a s m is a f o r c e f u l a n d in v o lu n ta r y s p a s m o f th e la r y n g e a l m u s c u la tu r e c a u s e d b y
s tim u la tio n o f th e s u p e r io r la r y n g e a l n e r v e . In itia l tr e a tm e n t o f la r y n g o s p a s m in c lu d e s g e n tle
p o s itiv e - p r e s s u r e v e n tila tio n w ith 1 0 0 % o x y g e n a n d f o r w a r d j a w th ru s t. I n tr a m u s c u la r o r
in tr a v e n o u s s u c c in y lc h o lin e a n d c o n tr o lle d v e n tila tio n m a y b e r e q u ir e d in r e c a lc itr a n t
la r y n g o s p a s m .

46. B. N o r m a l d e a d s p a c e in a n e o n a te is 2 m L /k g .

47. B. A p p r o x im a te ly 7 0 % to 8 0 % o f th e h e m o g lo b in a t b ir t h is fe ta l h e m o g lo b in (H b F ). T h e
c o n c e n tr a tio n o f H b F d e c r e a s e s s ig n if ic a n tly b y 3 to 6 m o n th s o f a g e . H b F h a s a h ig h a f f in ity
f o r o x y g e n , w h ic h s h ifts th e o x y h e m o g lo b in s a tu r a tio n c u r v e to le ft. S ic k le c e ll p a tie n ts w ith
m o r e H b F h a v e a b e tte r p r o g n o s is .

48. A. A lv e o la r m a tu r a tio n is n o t c o m p le te u n til a b o u t 8 y e a r s o f a g e . I n c r e a s e d a ir w a y r e s is ta n c e


a n d d e c r e a s e d c o m p lia n c e le a d to in c r e a s e d w o r k o f b r e a th in g a n d th u s r e s p i r a t o r y m u s c le s
e a s ily fa tig u e . T h e c h e s t w a ll c o lla p s e s d u r in g in s p ir a tio n , a n d r e s id u a l lu n g v o lu m e s a r e l o w a t
e x p ir a tio n . T h e r e s u ltin g d e c r e a s e in F R C a n d in c r e a s e d o x y g e n c o n s u m p tio n le a d to r a p id
d e s a tu r a tio n in th e e v e n t o f h y p o x ia . In a d d itio n , th e h y p o x ic a n d h y p e r c a p n ic v e n tila to r y d r iv e s
a r e n o t w e ll d e v e lo p e d in n e o n a te s a n d in fa n ts .

49. B. B u p iv a c a in e is c a r d io to x ic , a n d a n in a d v e r te n t in tr a v a s c u la r in je c tio n c a n le a d to S T
s e g m e n t c h a n g e s , c a r d ia c a r r h y th m ia s , a n d c a r d ia c a r r e s t. W h e n e p in e p h r in e is a d d e d to
b u p iv a c a in e , ta c h y c a r d ia is u s u a lly s e e n , b u t a t tim e s th is s ig n c a n b e u n r e lia b le ( h ig h e r h e a r t
r a te in in fa n ts ).

50. A. B e c a u s e o f s h o r te r c ir c u la t io n tim e s th a n a d u lts , a ll p e d ia tr ic p a tie n ts h a v e a s h o r te r o n s e t


tim e (u p to 5 0 % le s s ) o f m u s c le r e la x a n ts . N o n e th e le s s , in tr a v e n o u s succinylcholine ( 1 - 1 .5
m g /k g ) h a s th e f a s te s t o n s e t a m o n g m u s c le r e la x a n ts . S ig n if ic a n tly l a r g e r v o lu m e o f
d is tr ib u tio n is a ttrib u te d to l a r g e r d o s e r e q u ir e m e n ts in in fa n ts . W ith th e n o ta b le e x c lu s io n o f
s u c c in y lc h o lin e , m iv a c u r iu m , a n d p o s s ib ly cisatracurium, in fa n ts r e q u i r e s ig n if ic a n tly le s s
m u s c le r e la x a n t th a n o ld e r c h ild r e n . M o r e o v e r , b a s e d o n w e ig h t, o ld e r c h ild r e n r e q u i r e h ig h e r
d o s e s th a n a d u lts f o r s o m e n e u r o m u s c u la r b lo c k in g a g e n ts (e .g ., m iv a c u r iu m a n d atracurium).
A s w ith a d u lts , a m o r e r a p id in tu b a tio n c a n b e a c h ie v e d w ith a m u s c le r e la x a n t d o s e th a t is 1.5 to
2 tim e s th e E D 95 d o s e a t th e e x p e n s e o f p r o l o n g i n g th e d u r a tio n o f a c tio n .
Critical Care
David Stahl, Daniel Johnson, and Edward Bittner

1. A 78-year-old otherwise-healthy woman arrives in the postanesthesia care unit after an urgent
cystoscopy and ureteral stent placement for an impacted ureteral stone. In the operating room,
there were no complications and only minimal blood loss. One hour later, she is febrile to
102.3°F, tachycardic with a heart rate of 117 bpm, and hypotensive with a noninvasive blood
pressure of 73/42 mm Hg. Blood cultures are drawn and broad-spectrum antibiotics are
initiated. A central venous catheter is placed, and the central venous pressure is measured at 2
mm Hg. The best next step in the management of her shock is
A. Start dobutamine for increased inotropy
B. Fluid resuscitation to restore adequate preload
C. Blood transfusion to a goal hemoglobin concentration of 12 g/dL
D. Initiate nitroglycerin infusion to off-load the right ventricle

2. Shock is most accurately defined as


A. Inadequate tissue perfusion to meet the oxygen demand of end organs
B. Hypotension not responsive to intravenous fluid administration
C. An irreversible process of multisystem organ failure
D. Decreased blood flo w resulting from inadequate cardiac output

3. A 73-year-old man with a history of chronic obstructive pulmonary disease (COPD) on home
oxygen was initia lly admitted to the medical floor for a COPD exacerbation. Over the past few
hours, he has developed altered mental status and hypotension. He is transferred to the ICU,
intubated, and vasopressors are started to support his blood pressure. A pulmonary artery
catheter is placed via the right internal jugular vein. Initial readings reveal central venous
pressure = 23 mm Hg, positive airway pressure = 34/15 mm Hg, pulmonary capillary wedge
pressure = 4 mm Hg, and CO = 1.9 L/min. The most likely diagnosis is
A. Hypovolemic shock from inadequate fluid resuscitation
B. Septic shock from pneumonia
C. Anaphylactic shock from medications given during intubation
D. Cardiogenic shock from right-ventricular failure

4. A 54-year-old man is postoperative day 1 after a pancreaticoduodenectomy for pancreatic


cancer, complicated by a small intraoperative bile leak. He is febrile to 39.5°C, rigorous, and
hypotensive with a blood pressure of 71/32 mm Hg. He is admitted to the ICU. Laboratory work
reveals a leukocytosis with bandemia. Despite 4 L of intravenous crystalloid, he remains
hypotensive. The most accurate diagnosis for his condition is
A. Postoperative infection
B. Sepsis
C. Severe sepsis
D. Septic shock

5. Dopamine acts on all of the follow ing receptors, except


A. a!
B. Pi
C. P2
D. D A1

6. A ll of the follow ing may be caused by P-agonist effects of vasopressors, except


A. Increased inotropy
B. Bronchodilation
C. Inhibition of renin secretion
D. Uterine relaxation

7. You are called to the ER to assist in the intubation and management of a 26-year-old man who
sustained significant closed head injury during a motorcycle collision. Following uneventful
intubation, you accompany the patient and neurosurgery team to the CT scanner where you see
a large subarachnoid hemorrhage with effacement of the sulci and 9-mm midline shift. While
preparations are made to proceed directly to the operating room, the neurosurgeon asks if you
can increase the patient’s mean arterial blood pressure (MAP) from 70 to 90 mm Hg to
improve cerebral perfusion. The best vasopressor to accomplish this increase in MAP is
A. Dopamine
B. Phenylephrine
C. Norepinephrine
D. Epinephrine

8. Acute renal failure is defined as


A. Urine output of less than 0.5 mL/kg/hr or increase in serum creatinine by 50% in 24 hours
B. Urine output of less than 1 mL/kg/hr or increase in serum creatinine by 100% in 24 hours
C. Urine output of less than 1 mL/kg/hr or increase in serum creatinine by 200% in 24 hours
D. Urine output of less than 0.25 mL/kg/hr or increase in serum creatinine by 50% in 24 hours

9. A 28-year-old man is admitted to the intensive care unit after a motorcycle collision from
which he suffers multiple injuries including traumatic aortic injury requiring open repair,
multiple long-bone fractures, and a closed head injury. On arrival, his blood pressure is
maintained on a norepinephrine infusion. His urine output has been <5 mL/hr for the past 8
hours despite adequate fluid resuscitation and a renal ultrasound study that was normal. His pH
on arterial blood gas analysis is 6.9 with a base deficit of 16 and a potassium of 5.4 mEq/L. The
decision is made to institute renal replacement therapy for recalcitrant acidosis. The best course
of action is
A. Institution of continuous renal replacement therapy (CRRT) as it has been shown to
improve mortality at 30 days when compared to intermittent hemodialysis (IHD)
B. Institution of IHD as it has been shown to improve in-hospital mortality when compared to
CRRT
C. Institution of IHD as it has been shown to more effectively clear acidosis
D. Institution of CRRT as it has been shown to be more hemodynamically stable than IHD

10. Delirium as defined by the DSM-IV includes which of the follow ing major tenants?
A. Decreased attention and altered cognition
B. Agitation and pulling at lines
C. Altered mental status and dementia
D. Chronic perceptual disturbances and depressed mood

11. Delirium in the ICU setting is


A. A relatively benign condition
B. Associated with increased mortality
C. Associated with a decreased risk of eventual development of dementia
D. Often successfully treated with benzodiazepines

12. An 88-year-old man is admitted to the intensive care unit after a right-hip hemiarthroplasty to
repair an intertrochanteric femur fracture sustained during a fall from standing. On
postoperative day 1, he is confused and intermittently agitated with a disorganized thought
process. His nurse completed the CAM-ICU screen and reports that the result was positive. The
next steps in the management should include all of the following, except
A. Continually reorienting the patient to his surroundings
B. Minimizing sedatives if possible
C. Removing all opioids from his pain regimen
D. Optimizing sleep health by minimizing nighttime wakeups and encouraging daily
wakefulness

13. A ll of the follow ing conditions are associated with delirium in the ICU, except
A. Advanced age
B. Orthopedic surgery
C. Sepsis
D. Sleep deprivation

14. Which of the follow ing is a benefit of enteral nutrition when compared to parenteral nutrition?
A. Decreased cost
B. Decreased length of mechanical ventilation
C. Decreased rates of infection
D. A ll of the above are benefits of enteral nutrition

15. Enteral nutrition should be initia lly avoided in a


A. 54-year-old man who presents with acute alcoholic pancreatitis
B. 23-year-old G1P0 with hyperemesis gravidarum
C. 76-year-old woman with a full-thickness esophageal perforation
D. 34-year-old woman hospitalized with an acute exacerbation of ulcerative colitis

16. A 36-year-old G3, now P3, after a normal spontaneous vaginal delivery is complicated by
postpartum hemorrhage. Her vitals are checked, and she is noted to be tachycardic with a HR of
132 bpm and hypotensive with a BP of 76/35 mm Hg. The rapid response team is called. As a
result of calling the rapid response team, which of the follow ing outcomes can most reasonably
be expected?
A. She is less likely to have a cardiopulmonary arrest on the postpartum floor
B. She is less likely to have a cardiopulmonary arrest in the hospital
C. She is less likely to be transferred to an ICU
D. She is more likely to survive to hospital discharge

17. The most significant risk of intensive insulin therapy (goal blood glucose 80-100 mg/dL) when
compared to moderate glucose control (goal blood glucose <180 mg/dL) is
A. Myocardial infarction
B. Seizure
C. Patient dissatisfaction
D. Hypoglycemia

18. A 54-year-old man is admitted to the intensive care unit for monitoring after a complicated left
colectomy for diverticulitis. He has a history of type 2 diabetes mellitus on metformin. On
arrival to the ICU, his blood glucose on an arterial blood gas is 254 mg/dL. One hour later, it is
435 mg/dL. The next appropriate step in his management is
A. Recheck blood glucose in 1 hour
B. Restart home metformin
C. Start IV insulin therapy with a goal glucose <180 mg/dL
D. Start IV insulin therapy with a goal glucose <120 mg/dL

19. A 93-year-old woman is admitted to the ICU with a leaking 7.8-cm abdominal aortic aneurysm.
A multidisciplinary discussion is initiated between the patient, family, bedside nurse, ICU team,
and surgery team to decide on the next course of action. Select the answer which best identifies
the ethical principle at hand in each quote:
A. Autonomy— patient: “ I accept that refusing an operation means I w ill likely die soon, but I
want to die at home with my fam ily around me if at all possible”
B. Beneficence— ICU attending: “ I w orry that if you have this operation it w ill be unlikely that
you w ill ever return to living at home without significant assistance”
C. Nonmaleficence— surgeon: “ The best chance of you surviving is to have the aneurysm
repaired”
D. Justice— patient’s daughter: “ Is there another way to do the operation that is less risky?”

20. An 86-year-old man with end-stage congestive heart failure and chronic obstructive pulmonary
disease is admitted to the intensive care unit after a fall down one fligh t of stairs from which he
sustains a large subarachnoid hemorrhage. After lengthy discussion with the family, including
the patient’s wife who has been previously designated his health-care proxy, a decision is made
to change goals of care to comfort measures alone. The patient is started on a morphine
infusion for pain and to control dyspnea; he is extubated and the fam ily is present at the bedside.
About an hour later, the patient’s daughter emerges from the room, and tearfully asks, “ How
much longer can this go on? Can’t you do something to speed up the process?” You correctly
reply
A. “ We can add additional sedation which w ill make him pass more quickly”
B. “ I can give him a bolus of morphine to stop him from breathing”
C. “ We can increase the rate of the morphine infusion if he appears to be in pain”
D. “ We can give him a strong muscle relaxant called rocuronium, which w ill stop him from
breathing”

21. An 89-year-old woman is postoperative day 23 from an open repair of a thoracoabdominal


aortic aneurysm. She is bacteremic for a second time, and continues to require vasopressor
support, mechanical ventilation, and continuous renal replacement therapy. After the most
recent of many multidisciplinary fam ily meetings, the decision is made to withdraw life-
sustaining treatment and focus on comfort measures alone. As you are discussing with the
bedside nurse the logistics of removing vasopressor support, a nursing student asks, “ Isn’t that
going to k ill her?” The most correct response is
A. “ Since our goal is not to end her life, it is not technically killin g her”
B. “ As long as her heart beats after we turn o ff the medications, it is not euthanasia”
C. “ As long as the fam ily has given us permission it is OK”
D. “ Stopping these treatments simply discontinues our prolongation of her natural death”

22. A 45-year-old construction worker falls from a three-story building suffers multiple traumatic
injuries including a partial amputation of his left leg, resulting in substantial hemorrhage. He is
admitted to the intensive care unit directly from the emergency room having received 4 U of
packed red blood cells and 2 U of fresh-frozen plasma. Following further resuscitation and
operative repair of his left leg, his serum creatinine is noted to be 2.13 mg/dL, and urine output
is 0.3 mL/kg/h. The most likely etiology of his renal failure is
A. Prerenal failure from hypotension
B. Prerenal failure from thromboembolic disease
C. Intrinsic renal failure from rhabdomyolysis
D. Postrenal failure from ureteral obstruction

23. A 28-year-old woman with a history of Hodgkin lymphoma and external beam radiation is
admitted to the intensive care unit after repair of an esophageal perforation. She is maintained
strictly NPO. Addition of dextrose to her maintenance intravenous fluids w ill most likely
A. Fail to suppress protein catabolism
B. Improve her blood glucose control
C. Increase her insulin requirement
D. Improve her cardiac metabolic balance

24. Which of the follow ing tissues does not rely on glucose metabolism in the setting of
starvation?
A. Neural tissue
B. Cardiac tissue
C. Renal medullary tissue
D. Erythrocytes

25. Which of the follow ing parenteral nutrition orders would be least likely to precipitate
hypercarbic respiratory failure in a patient with severe chronic obstructive pulmonary disease?
A. Protein = 40 g/L, dextrose = 125 g/L, fat = 0 g/L
B. Protein = 30 g/L, dextrose = 150 g/L, fat = 0 g/L
C. Protein = 50 g/L, dextrose = 60 g/L, fat = 50 g/L
D. Protein = 50 g/L, dextrose = 100 g/L, fat = 25 g/L

26. A 25-year-old man is admitted to the intensive care unit after exploratory laparotomy and
repair of multiple bowel injuries from several gunshot wounds. Several hours after admission,
the respiratory therapist calls to alert you that his peak airway pressures have increased
significantly. The bedside nurse also reports an increase in his vasopressor requirement, and a
decrease in urine output. On examination, his abdomen is tense, with a midline dressing intact,
and clear breath sounds bilaterally. The most likely diagnosis is
A. Acute myocardial infarction leading to pulmonary edema
B. Hypovolemia from inadequate fluid resuscitation
C. Hemorrhage from an unrecognized injury
D. Abdominal compartment syndrome from bowel edema or hemorrhage

27. One of the feared adverse effects of reinstituting nutrition in a malnourished patient is
refeeding syndrome. The most common electrolyte abnormality seen in refeeding syndrome is
A. Hypophosphatemia from an increase in intracellular movement of phosphate
B. Hypokalemia from extracellular buffering of alkalosis
C. Hypomagnesemia from renal losses
D. Hyponatremia from excess free water retention by the kidney
28. A 59-year-old man with acute chronic pancreatitis complicated by pseudocyst and necrotizing
pancreatitis has been receiving total parenteral nutrition (TPN) for 3 weeks. An error in the
ordering system prevents the pharmacy from receiving his order in time to make that day’s
supply. If his TPN is abruptly discontinued, he is at highest risk for
A. Hyponatremia
B. Hypokalemia
C. Hypoglycemia
D. Hyperkalemia

29. Acute respiratory distress syndrome (ARDS) patients’ plateau pressures should be maintained at
or below
A. 50 cm H2O
B. 60 cm H2O
C. 40 cm H2O
o

D. cm H2O

30. Helium-oxygen mixtures can be useful therapies for patients with upper airway obstruction.
Compared with air, helium-oxygen mixtures have lower
A. Density
B. Viscosity
C. Oxygen content
D. Nitrous oxide content

31. In assist-control ventilation (ACV)


A. Breaths triggered by the ventilator result in the fu ll preset tidal volume being delivered,
while breaths triggered by the patient are unsupported by the ventilator
B. A ll breaths result in the fu ll preset tidal volume being delivered, regardless of whether they
are initiated by the ventilator or by the patient
C. A ll breaths must be initiated by the patient
D. The patient is incapable of triggering breaths

32. A 78-year-old man is admitted from the surgical floor to the intensive care unit for respiratory
distress. He is postoperative day 1 from open reduction and internal fixation of a right femoral
shaft fracture sustained during a motor vehicle collision. His heart rate is 118 bpm, blood
pressure is 104/62 mm Hg, Spo2 is 68% on a non-rebreathing mask at 15 L/min of oxygen, and
respirations are 42/min. On examination, he is unresponsive to commands and to sternal rub.
The ICU team is deciding whether to initiate noninvasive positive-pressure ventilation (NIPPV)
or to perform endotracheal intubation. NIPPV is contraindicated because
A. The patient’s neurologic examination suggests that he is incapable of protecting his airway
B. NIPPV is incapable of improving significantly low oxygen saturations
C. The patient is claustrophobic
D. The patient has certainly suffered a pulmonary embolism (PE), and NIPPV is not helpful in
this situation

33. In the first several days follow ing traumatic brain injury requiring mechanical ventilation, an
optimal regimen for anxiolysis includes
A. Diazepam bolus every hour
B. Lorazepam infusion
C. Hydromorphone infusion
D. Propofol infusion

34. In pressure-support ventilation (PSV), inspiration ends (and expiration begins) when
A. A preset tidal volume has been achieved
B. A preset airway pressure has been achieved
C. Flow decreases to a preset level
D. A preset amount of time has passed

35. A patient can be diagnosed with acute respiratory distress syndrome (ARDS) if he or she has an
acute onset of illness, bilateral infiltrates on chest X-ray, lack of evidence of left heart failure,
and a Pao2/Fio2 (P/F) ratio of less than or equal to

A. 500 mm Hg
B. 400 mm Hg
C. 350 mm Hg
D. 200 mm Hg

36. A 25-year-old woman has been in the ICU for 6 days after sustaining multiple life-threatening
traumatic injuries. She is suffering from septic shock and acute respiratory distress syndrome
(ARDS). On examination, she is dyssynchronous with the ventilator, coughing, grimacing in
pain, and tearful. Spo2 is 88% on a Fio 2 of 100%, and the BP is 108/58 mm Hg on a low-dose
infusion of norepinephrine. The most appropriate plan for management of the patient’s pain,
anxiety, and ventilator dyssynchrony is
A. Hydromorphone PRN and a continuous infusion of cisatracurium
B. Fentanyl infusion and a continuous infusion of cisatracurium
C. Midazolam and fentanyl infusions
D. Lorazepam PRN

37. A patient is at greatest risk for requiring endotracheal intubation and mechanical ventilation if
the Spo2 is 91% while breathing

A. Room air
B. 4 L/min of oxygen via nasal cannula
C. 15 L/min of oxygen via a non-rebreathing mask with reservoir bag
D. Noninvasive positive-pressure ventilation with an Fio 2 of 35%

38. Weaning from mechanical ventilation is expedited by


A. Daily spontaneous breathing trials
B. Synchronized intermittent mandatory ventilation (SIMV)
C. Administration of bronchodilating medications around the clock
D. Daily bronchoscopy

39. Synchronized intermittent mandatory ventilation (SIMV) was an improvement on intermittent


mandatory ventilation (IMV) because it
A. Can provide fu ll ventilation support to an apneic patient
B. Can utilize volume-preset or pressure-preset ventilation
C. Allows the patient to breathe spontaneously
D. Reduces the likelihood of breath stacking and volutrauma

40. Tracheostomy should be considered to reduce the risk of subglottic stenosis after an
endotracheal tube has been in place for
A. 5 days
B. 10 days
C. 2 to 3 weeks
D. 8 to 10 weeks

41. A 45-year-old alcoholic male was admitted to the medical floor with severe pancreatitis. On
hospital day 5, his respiratory status significantly deteriorates and he is transferred to the ICU.
The Spo2 is 89% on a non-rebreather mask at 15 L/min oxygen. Upon arrival in the ICU, he is
sedated and intubated. Initial ventilator settings should include a set tidal volume of
A. 2 mL/kg
B. 6 mL/kg
C. 10 mL/kg
D. 14 mL/kg

42. A 20-year-old trauma patient requires a large dose infusion of propofol while intubated in the
ICU. When the propofol is reduced to attempt a spontaneous breathing trial, the patient thrashes
w ild ly and tries to pull out his arterial and central venous lines. A titratable agent that could
prove useful for management of this patient’s agitation while not depressing his respiratory
drive is
A. Methadone
B. Dexmedetomidine
C. Nitrous oxide
D. Fentanyl transdermal patch
43. For pressure-preset ventilation (also known as “ pressure-control ventilation” ), the independent
variable and dependent variable, respectively, are
A. Tidal volume and Fio 2
B. Tidal volume and frequency
C. Spo2 and airway pressure
D. Airway pressure and tidal volume

44. A mechanically ventilated, 70-kg patient has an arterial blood gas of pH = 7.06, Pc o 2 = 83 mm
Hg, and Po 2 = 140 mm Hg on volume control ventilation (tidal volume = 450 mL, respiratory
rate = 8, Fio 2 = 50%, and positive end-expiratory pressure [PEEP] = 8 cm H2O). The most
appropriate next step in the management is
A. Increase PEEP
B. Increase Fio 2
C. Increase the respiratory rate
D. Administer sodium bicarbonate

45. The primary benefit of positive end-expiratory pressure (PEEP) during mechanical ventilation
is
A. Improved elimination of CO2
B. Improved venous return and cardiac output
C. Prevention and reversal of alveolar collapse (atelectasis)
D. Reduction in peak inspiratory pressure

46. A patient with a chronic obstructive pulmonary disease exacerbation has an initial arterial blood
gas (ABG) with pH = 7.05, Pc o 2 = 95 mm Hg, and Po 2 = 54 mm Hg on 6 L of oxygen via nasal
cannula. The patient is awake, alert, and in moderate respiratory distress with significant
wheezing. Bronchodilators and continuous positive airway pressure (CPAP) 10 cm H2O via
face mask with Fio 2 of 50% are initiated. One hour later, the ABG is pH = 7.10, Pc o 2 = 90 mm
Hg, and Po 2 = 92 mm Hg. The patient remains awake and alert and is now in less distress. The
most appropriate next step in the management is
A. Increasing the Fio 2
B. Increasing CPAP to 15 cm H2O
C. Changing the mode to bi-level positive airway pressure (BiPAP)
D. Stopping CPAP and delivering oxygen via high-flow nasal cannula

47. An ICU patient with severe acute respiratory distress syndrome (ARDS) remains
dyssynchronous with the ventilator despite administration of high-dose propofol and fentanyl
infusions and changes in the mode of ventilation. The patient’s gas exchange has deteriorated
over the course of the day, and hypotension requiring vasopressor support has developed in the
setting of increasing the propofol dose. The next best step is to
A. Increase propofol
B. Change to pressure-support ventilation
C. Aggressively diurese the patient
D. Administer a nondepolarizing neuromuscular-blocking agent

48. A 35-year-old man is receiving care in the ICU after sustaining an 80% total body surface area
burn from a house fire 5 weeks ago. The surgical team wishes to transition from large dressing
changes in the operating room to smaller dressing changes in the ICU. The patient’s analgesic
regimen consists of extended-release morphine 60 mg by mouth every 12 hours and
hydromorphone 2 to 4 mg IV every 2 hours PRN for breakthrough pain. The most appropriate
agent for providing sedation and analgesia during the dressing changes in the ICU is
A. Oral gabapentin
B. Oral clonidine
C. Intravenous morphine
D. Intravenous ketamine

49. A 30-year-old man is admitted to the ICU intubated after a 14-hour spine surgery in the prone
position. When he emerges from anesthesia, he bites down on his endotracheal tube, tries
frantically to breathe, and panics when he is unable to draw a breath. The nurse boluses
propofol and achieves adequate sedation. Five minutes later, the Spo2 rapidly falls from 99% to
65% and the patient appears cyanotic. Pink froth is seen in the endotracheal tube. This
complication could have been avoided by
A. Placing a bite block between the patient’s teeth
B. Administration of less IV fluid during the spine surgery
C. Omitting the propofol bolus
D. Positioning the patient in reverse Trendelenburg

50. Effective treatment for carboxyhemoglobinemia includes


A. Sodium nitrite
B. Ventilation with 100% oxygen
C. Ventilation with air
D. Sodium thiosulfate
CHAPTER 19 ANSWERS

1. B. The patient has clinical evidence of shock for which the most common cause is sepsis
(likely urosepsis in this case). Primary treatment of vasodilatory shock consists of repleting
intravascular volume until adequate preload can be restored, followed by, vasopressor support
to maintain adequate end-organ perfusion. The patient’s history as otherwise healthy does not
rule out cardiogenic shock but makes it less likely, particularly in the setting of fever. Institution
of dobutamine for increased inotropy may be useful in cardiogenic shock but is unlikely to
improve vasodilatory shock where cardiac output is typically already elevated. Nitroglycerin
infusion can be useful in cases of cardiogenic shock secondary to right-ventricular failure, but
in such a case, an elevated central venous pressure would be expected. Finally, while blood
transfusion may be indicated to increase oxygen-carrying capacity and oxygen delivery in
certain shock states, there is no evidence for a goal hemoglobin concentration of 12 g/dL. Fluid
resuscitation should begin with crystalloid until more data can be gathered on the patient’s
condition.

2. A. Shock is a common state in the intensive care unit which has many causes. The common
result is inadequate tissue perfusion to end organs, resulting in an imbalance between oxygen
supply and demand. If treated early, shock is reversible. However, if untreated, shock can
progress to irreversible multisystem organ failure and death. While hypotension is a common
component of shock, lack of fluid responsiveness is not a part of the definition, nor is
inadequate cardiac output.

3. D. In medically complex patients, the etiology of shock can be difficult to diagnose. While by
history, one could presume this patient has hypovolemic, septic, or anaphylactic shock, the
pulmonary artery catheter indicates elevated right-sided fillin g pressures with relatively low
left-sided pressures, and hypotension, which are consistent with right-ventricular failure and
cardiogenic shock.

4. D. The American College of Chest Physicians/Society of Critical Care Medicine


(ACCP/SCCM) Consensus Conference Definitions for sepsis would classify this patient as
having septic shock. The consensus definition for sepsis is a confirmed or suspected infection
plus two of the SIRS criteria (temperature <36°C or >38°C, heart rate >90 bpm, respiratory rate
>20 breaths/min or PaCO2<32 mm Hg, leukocyte count <4,000 cells/L or >12,000 cells/L).
Severe sepsis is defined as sepsis together with dysfunction of at least one organ system. Septic
shock is defined as sepsis plus hypotension (systolic blood pressure <90 mm Hg) despite fluid
resuscitation. This patient has a suspected infection, meets at least two of the SIRS criteria, and
remains hypotensive despite fluid resuscitation, making the most correct answer septic shock.

5. C. Dopamine has direct agonist action on a1‫־‬ and D A1 receptors, as well as indirect
agonism of a1 and P1 receptors via release of endogenous norepinephrine. D A1 effects are
predominately seen in low doses of dopamine and cause renal arteriole dilation. p1 effects are
s e e n a t m o d e r a te d o s e s o f d o p a m in e a n d in c r e a s e m y o c a r d ia l c o n tr a c tility a n d h e a r t ra te .
I n c r e a s e d m y o c a r d ia l w o r k a n d o x y g e n d e m a n d r e s u ltin g f r o m th e a g o n is t a c tio n s o f d o p a m in e
c a n le a d to m y o c a r d ia l is c h e m ia . a ! E ffe c ts a r e s e e n a t h ig h d o s e s o f d o p a m in e a n d le a d to
in c r e a s e d s y s te m ic v a s c u la r r e s is ta n c e .

6. C. A g e n ts th a t a c t o n p r e c e p to r s h a v e e ffe c ts o n a w id e v a r ie ty o f o r g a n s . P! a c tiv a tio n c a u s e s


a n in c r e a s e in in o t r o p y a n d c h r o n o t r o p y a s w e ll a s s tim u la tio n o f r e n in s e c r e tio n . p 2 a g o n is m
r e s u lts in b r o n c h o d ila tio n a s w e ll a s d ila tio n o f o th e r s m o o th m u s c le s , in c lu d in g th e u te ru s .

7. B. P h e n y le p h r in e a c ts d ir e c tly o n a ! r e c e p to r s to in c r e a s e s y s te m ic v a s c u la r r e s is ta n c e ,
a r te r ia l b l o o d p r e s s u r e , a n d c e r e b r a l b l o o d flo w . P h e n y le p h r in e d o e s n o t c r o s s th e b lo o d - b r a i n
b a r r i e r a n d th e r e f o r e d o e s n o t a f f e c t th e c e r e b r a l v a s c u la tu r e , m a k in g it th e v a s o p r e s s o r o f
c h o ic e in b r a in - in j u r e d p a tie n ts .

8. A. T h e r e a r e m a n y d e f in itio n s o f a c u te r e n a l f a ilu r e ( o r a c u te k id n e y in ju r y ) . T h e m o s t
c o n s is te n t d e f in itio n is a u r in e o u tp u t o f le s s th a n 0 .5 m L /k g /h r o r a 5 0 % in c r e a s e in s e r u m
c r e a tin in e o v e r 2 4 h o u r s . B o th th e A c u te D ia ly s is Q u a lity In itia tiv e (A D Q I) c r i t e r i a (a ls o k n o w n
a s th e R IF L E c r ite r ia ) a n d th e A c u te K id n e y I n ju r y N e tw o r k (A K IN ) c r i t e r i a in c lu d e th e s e in th e
f i r s t s ta g e o f a c u te k id n e y in ju r y .

9. D. C R R T h a s n o t b e e n s h o w n to b e m o r e e f f ic a c io u s o r im p r o v e m o r t a lit y in IC U p a tie n ts
w h e n c o m p a r e d to IH D ; h o w e v e r, C R R T is a s s o c ia te d w ith le s s h y p o te n s io n p r e s u m a b ly
b e c a u s e o f s m a lle r in tr a v a s c u la r f lu id s h ifts w h e n c o m p a r e d to IH D . In th is p a tie n t w h o h a s a
p e r s is te n t v a s o p r e s s o r r e q u ir e m e n t, C R R T w ill lik e ly p r o v id e m o r e h e m o d y n a m ic s ta b ility ,
w h ile e n a b lin g c le a r a n c e o f b o th h is a c id o s is a n d h y p e r k a le m ia .

10. A. D e lir iu m is d e f in e d b y th e DSM-IV a s a n a lte r a tio n o f c o n s c io u s n e s s w ith (1 ) d e c r e a s e d


a b ility to f o c u s o r s u s ta in a tte n tio n a s s o c ia te d , (2 ) a d is tu r b a n c e in c o g n itio n o r p e r c e p tio n n o t
a c c o u n te d f o r b y b a s e lin e d e m e n tia . A g ita tio n a n d p u llin g a t lin e s m a y b e s ig n s o f h y p e r a c tiv e
d e lir iu m b u t a r e n o t d ia g n o s tic o f d e lir iu m . D e m e n tia is a c h r o n ic c o n d itio n th a t is
d ia g n o s t ic a lly s e p a r a te f r o m d e lir iu m , w h ic h is a n a c u te c o n d itio n . D e p r e s s e d m o o d is n o t p a r t
o f th e d e f in itio n o f d e lir iu m , a lth o u g h f la t a f f e c t m a y b e s e e n in h y p o a c tiv e d e lir iu m .

11. B. W h ile p r e v io u s ly th o u g h t o f a s a r e la tiv e ly b e n ig n c o n d itio n o r a m e r e in c o n v e n ie n c e to


IC U p r o v id e r s , d e lir iu m h a s b e e n a s s o c ia te d w ith p r o l o n g e d m e c h a n ic a l v e n tila tio n , p r o l o n g e d
IC U a n d h o s p ita l stay , a n d in c r e a s e d m o r ta lity . D e lir iu m is a ls o a s s o c ia te d w ith e v e n tu a l
d e v e lo p m e n t o f d e m e n tia . R is k o f d e lir iu m is in c r e a s e d w h e n b e n z o d ia z e p in e s a r e u s e d f o r
s e d a tio n .

12. C. W h ile o p io id s m a y in d e e d c o n tr ib u te to d e lir iu m , in a d e q u a te p a in c o n tr o l is a ls o a s s o c ia te d


w ith d e lir iu m . It m a y b e b e n e f ic ia l to o p tim iz e p a in c o n tr o l w ith o th e r n o n o p io id a d ju n c ts s u c h
a s a c e ta m in o p h e n , b u t r e m o v a l o f a ll o p io id s m a y n o t b e p r a c tic a l o r h e lp f u l. E n c o u r a g in g
sleep health, reorienting the patient to his surroundings, and minimizing sedatives are all
important treatments for delirium.

13. B. Advanced age, sepsis, and sleep deprivation have all be associated with delirium.
Orthopedic surgery has not been independently associated with delirium. Other conditions
associated with delirium include baseline cognitive impairment, increasing severity of illness,
multisystem organ failure, immobilization, pain, mechanical ventilation, and use of sedatives
(especially benzodiazepines).

14. D. Enteral nutrition is less expensive, easier to administer, and maintains normal enteric
physiology and flora better than parenteral nutrition. Enteral nutrition is also associated with
lower rates of infection, and more recently has been shown to decrease the length of mechanical
ventilation and hospital stay (even in a subset of patients who have limited enteral intake).

15. C. While previous concerns have been raised about enteral feeding in a number of disease
states, more recent evidence demonstrates the benefits outweigh the risks in cases of acute
pancreatitis, hyperemesis gravidarum, and inflammatory bowel disease, as well as cases of
enteric fistulas, short-bowel syndrome, and bone marrow or other chemotherapy patients.
Patients with an esophageal perforation are managed without enteral nutrition in the acute
setting.

16. A. The institution of multidisciplinary rapid response teams were in reaction to the high
number of cardiopulmonary arrests seen in the in-hospital setting. Despite this intervention,
there are no data to demonstrate decreased incidence of cardiopulmonary arrest in the hospital,
prevention of ICU admission, or decreased mortality. However, patients evaluated by the rapid
response team are more likely to be moved to the ICU sooner, and therefore less likely to have
a cardiopulmonary arrest outside of the ICU, such as on the postpartum floor.

17. D. Initial studies of intensive insulin therapy (goal blood glucose approximately 80-100
mg/dL) suggested decreased ICU mortality. Unfortunately, subsequent trials have failed to
reproduce the benefit of decreased mortality and have demonstrated substantial increases in
rates of hypoglycemia. While myocardial infarction and seizure may be presentations of
hypoglycemia, the more likely underlying cause in the case of intensive insulin therapy is
hypoglycemia.

18. C. The patient has a rapidly increasing blood glucose level that warrants control immediately.
Restarting home metformin, while reasonable, is unlikely to have an acute effect on his
hyperglycemia. IV insulin therapy is indicated in this patient. While earlier studies demonstrated
a mortality benefit to intensive insulin therapy, subsequent data have shown an increased risk of
hypoglycemia and failed to show a mortality benefit. It would be most reasonable to start IV
insulin with a goal blood glucose of <180 mg/dL.

19. A. End-of-life discussions should protect a patient’s right to die with dignity. Understanding
the ethical principals at play can help guide balanced solutions. Autonomy recognizes the right
of the individual (in this case, the patient) to self-determination. In this case, accepting refusal of
treatment with a rational understanding respects the patient’s autonomy. Beneficence is the
principle of taking the action that best serves the patient’s interest. The ICU attending, while
expressing beneficence is also expressing nonmaleficence, is the concept of avoiding harm to
patients. The surgeon is more clearly expressing the ideal of beneficence. Justice refers to the
distribution of scarce resources. The question from the patient’s daughter of doing the
procedure in a less-risky manner is more likely reflective of beneficence and nonmaleficence.

20. C. The goal of medications at the end of life should be the treatment of specific symptoms, not
the direct hastening of death. In this case, treating pain or dyspnea with morphine is quite
appropriate. The principle of the “ double effect,” that is, two consequences caused by a single
action, is also appropriate at the end of life. In this instance, morphine may have the desired
effect of treating pain (following the ethical principle of beneficence) but may also hasten death
(following the ethical principle of nonmaleficence). It is not ethically appropriate to add
sedation to hasten death, or to bolus morphine or a paralytic to stop breathing.

21. D. It is important to communicate to fam ily members and providers that withdrawal of life-
sustaining treatment is no different from having a patient (or fam ily member) refuse life-
sustaining treatment in the first place, and respects the principle of autonomy. Furthermore,
withdrawal of life-sustaining treatment is not active euthanasia but rather cessation of the
prolongation of natural death. This is true regardless of the physiologic consequences.
Whereas, medications at the end of life are often given with the understanding of the “ double
effect” principle, namely that medications given to alleviate pain and suffering may also speed
death; withdrawal of life-sustaining treatment does not have a specific goal other than to respect
the patient and fam ily member’s autonomy, or to cease medically futile care.

22. A. In this vignette, the patient’s massive hemorrhage likely leads to prerenal failure from both
hypotension and hypovolemia. Evaluation of the urine would likely reveal an elevated specific
gravity and a fractional excretion of sodium of <1%. Thromboembolic disease is possible,
particularly with long-bone fractures, although much more rare. Rhabdomyolysis is also
possible due to muscle injury but less likely. Finally, while urine output does not rule out
obstruction, there is no mention of an injury that would increase suspicion of renal-outflow
obstruction.

23. A. Addition of dextrose to intravenous fluids during acute illness does not suppress protein
catabolism. Even the addition of adequate calories may not entirely prevent protein catabolism
in a critically ill patient. The amount of dextrose in standard maintenance fluids is unlikely to
improve her glucose control or substantially alter her insulin requirements. The heart
preferentially relies on fatty acid metabolism, which is likely unchanged in the setting of
additional IV dextrose.

24. B. Within approximately 24 hours of the onset of fasting, glycogen supplies are depleted and
gluconeogenesis becomes increasingly important. Neural tissue, renal medullary tissue, and
erythrocytes continue to utilize glucose, sparing tissue proteins, while other cells preferentially
utilize fatty acids such as cardiac myocytes.

25. C. The respiratory quotient (RQ) is defined as the ratio of the amount of carbon dioxide
produced relative to the amount of oxygen consumed (V c o 2/V o 2 ). The RQ changes with the
type of caloric intake. Carbohydrates have an RQ of 1, whereas proteins have an RQ of 0.8 to
0.9, and lipids have an RQ closer to 0.7. Given that a higher RQ reflects greater CO2
production, and therefore, increased need for CO2 elimination, the parenteral nutrition with the
least calories from carbohydrates w ill be least likely to precipitate hypercarbic respiratory
failure.

26. D. While all of these diagnoses are important to exclude, the most critical in this case is
abdominal compartment syndrome. The increase in peak airway pressures, worsening
hypotension, and decreased urine output in the setting of recent abdominal surgery with a tense
abdomen raise concern for abdominal compartment syndrome. Bladder pressure can be
measured as an indicator of intra-abdominal pressure. Myocardial infarction is unlikely in a 25-
year-old, and clear lung sounds make pulmonary edema also unlikely. Hypovolemia and
hemorrhage are also both possible, but intra-abdominal compartment syndrome should be
ruled out first.

27. A. Hypophosphatemia is commonly seen in critically ill patients. As a part of the refeeding
syndrome, there is increased synthesis of intracellular adenosine triphosphate, resulting in
increased intracellular transport of phosphates. Hypokalemia may also be seen in refeeding
syndrome but is related to intracellular transport and secondary effects of endogenous and
exogenous insulin. Hypomagnesemia may also occur in refeeding syndrome. Hyponatremia is
not commonly associated with refeeding syndrome.

28. C. Given the patient’s long-term TPN use, he is at highest risk for hypoglycemia, as his
constant source of glucose has been acutely removed. If TPN is to be discontinued, then it
should be slowly tapered over a period of hours. If TPN is to be abruptly discontinued, then it
should be replaced with a dextrose-containing fluid. Patients who receive insulin in their TPN
are also at risk of hyperglycemia.

29. D. Multiple studies have shown a mortality benefit when patients with ARDS are ventilated
with tidal volumes of 6 mL/kg ideal body weight and have plateau pressures maintained at or
below 30 cm H2O.

30. A. Helium-oxygen mixtures have a lower density than air, which promotes improved laminar
gas flo w across obstructed airways. The mixtures have a similar viscosity to air. Helium-
oxygen mixtures do not have oxygen content lower than 21% (the oxygen content of air).
Neither the mixtures nor the air contains nitrous oxide.
31. B. T h e e s s e n c e o f A C V is th a t all b r e a th s r e c e iv e th e fu ll p r e s e t tid a l v o lu m e r e g a r d l e s s o f
w h e th e r th e b r e a th s a r e in itia te d b y th e v e n tila to r o r b y th e p a tie n t. W ith A C V if th e v e n tila to r is
s e t a t V t = 5 0 0 m L , th e f r e q u e n c y is s e t a t 10 b r e a th s /m in , a n d th e p a tie n t e x h ib its n o r e s p i r a t o r y
e f f o r t, th e v e n tila to r w ill d e liv e r 5 0 0 m L b r e a th s 10 tim e s p e r m in u te . If th a t s a m e p a tie n t m a k e s
8 r e s p i r a t o r y e f f o r ts in a d d itio n to th e 10 m a n d a to r y b r e a th s , th e v e n tila to r w ill d e liv e r 5 0 0 m L
b r e a th s 18 tim e s p e r m in u te . C h o ic e A is n o t c o r r e c t b e c a u s e in a s s is t- c o n tr o l m o d e , a ll b r e a th s
a r e f u lly s u p p o r te d . C h o ic e C is n o t c o r r e c t b e c a u s e in a s s is t c o n tr o l, if th e p a tie n t m a k e s n o
r e s p i r a t o r y e f f o r t, th e v e n tila to r w ill p r o v id e b r e a th s a t th e p r e s e t f r e q u e n c y . C h o ic e D is n o t
c o r r e c t b e c a u s e a s s is t c o n tr o l d o e s a llo w p a tie n ts to t r i g g e r b re a th s .

32. A. U n r e s p o n s iv e p a tie n ts a r e p o o r c a n d id a te s f o r N IP P V b e c a u s e o f th e r i s k o f r e g u r g i t a t i o n
a n d a s p ir a ti o n o f s to m a c h c o n te n ts . W h e n c o n s id e r in g N IPPV , o n e m u s t c o n s id e r w h e th e r th e
p a tie n t w ill b e a b le to p r o te c t th e a irw a y . C h o ic e B is i n c o r r e c t b e c a u s e N IP P V c a n , in fa c t,
im p r o v e s ig n if ic a n tly l o w o x y g e n s a tu r a tio n , p a r tic u la r ly in c a s e s o f c h r o n ic o b s tr u c tiv e
p u lm o n a r y d is e a s e e x a c e r b a tio n s o r c o n g e s tiv e h e a r t f a ilu r e . C h o ic e C is i n c o r r e c t b e c a u s e th e
q u e s tio n d o e s n o t m e n tio n c la u s tr o p h o b ia . W h ile it is d if f ic u lt f o r p a tie n ts w ith c la u s tr o p h o b ia
to to le r a te N IPPV , m o d e r n d e v ic e s th a t f it o v e r th e n o s e a lo n e h a v e a lle v ia te d th is p r o b le m to
s o m e d e g r e e . C h o ic e D is i n c o r r e c t b e c a u s e it is n o t c e r ta in th a t th e p a tie n t h a s s u f f e r e d a P E .
T h e c lin ic a l s c e n a r io is s u g g e s tiv e o f P E (tr a u m a , lo n g - b o n e f r a c tu r e , sta tu s p o s ts u r g e r y ) , b u t
p a tie n ts w ith p o s to p e r a tiv e p u lm o n a r y e d e m a a n d o th e r e tio lo g ie s o f r e s p i r a t o r y f a ilu r e c a n
p r e s e n t in a s im ila r w ay .

33. D. P r o p o f o l in f u s io n is a n e x c e lle n t c h o ic e f o r a n x io ly s is in m e c h a n ic a lly v e n tila te d p a tie n ts


a fte r a c u te b r a i n in ju r y b e c a u s e o f its f a v o r a b le p h a r m a c o k in e tic s a n d p h a r m a c o d y n a m ic s . T h e
IC U te a m c a n s to p th e p r o p o f o l in f u s io n f r e q u e n tly a n d r e e v a lu a te th e p a tie n t’s n e u r o l o g i c
e x a m . C h o ic e A is i n c o r r e c t b e c a u s e h o u r l y b o lu s e s o f d ia z e p a m w o u ld r e s u lt in a c c u m u la tio n
o f th e d r u g a n d i n te r f e r e w ith n e u r o l o g i c e x a m in a tio n s . C h o ic e B is i n c o r r e c t b e c a u s e a
lo r a z e p a m in f u s io n is a ls o lik e ly to i n te r f e r e w ith n e u r o l o g i c e x a m in a tio n . C h o ic e C is
i n c o r r e c t b e c a u s e th e q u e s tio n is a b o u t a n x io ly s is , n o t a n a lg e s ia .

34. C. In PSV , in s p ir a tio n is t r i g g e r e d b y a p a tie n t’s r e s p i r a t o r y e f f o r t. A c o n tin u o u s a ir w a y


p r e s s u r e is m a in ta in e d b y g a s f lo w th a t d e c r e a s e s th r o u g h o u t in s p ir a tio n . W h e n f lo w d e c r e a s e s
to a p r e s e t f r a c tio n o f th e p e a k f lo w ( u s u a lly 2 5 % o f p e a k flo w ) , g a s f lo w in to th e in s p i r a t o r y
lim b e n d s a n d e x p ir a tio n b e g in s . C h o ic e A d e s c r ib e s v o lu m e - p r e s e t v e n tila tio n , o f te n c a lle d
“v o lu m e c o n tr o l.” C h o ic e B is i n c o r r e c t b e c a u s e in PSV , a p r e s e t a ir w a y p r e s s u r e is maintained
th r o u g h o u t in s p ir a tio n . C h o ic e D is i n c o r r e c t b e c a u s e in PSV , d e c r e a s e in f lo w ( n o t a p r e s e t
tim e ) d e te r m in e s th e le n g th o f in s p ir a tio n .

35. D. T h e A m e r ic a n - E u r o p e a n C o n s e n s u s C o n f e r e n c e (A E C C ) o n A R D S d e f in e d A R D S a s (1 )
a c u te o n s e t a n d (2 ) P /F < 2 0 0 m m H g , b ila te r a l in f iltr a te s o n c h e s t X -ra y , a n d a p u lm o n a r y a r te r y
o c c lu s io n (w e d g e ) p r e s s u r e < 1 8 m m H g o r c lin ic a l a b s e n c e o f le f t- a tr ia l h y p e r te n s io n . T h e
p u lm o n a r y p r o c e s s w a s d e f in e d a t A c u te L u n g I n ju r y if th e a b o v e c r i t e r i a w e r e m e t b u t th e P /F
w a s 2 0 1 to 3 0 0 m m H g . In 2 0 1 2 , th e A R D S D e f in itio n T a s k F o r c e r e m o v e d A c u te L u n g I n ju r y
f r o m th e d e f in itio n a n d r e p la c e d it w ith th e c a te g o r ie s m ild , m o d e r a te , a n d s e v e r e A R D S
defined by P/F ratios of 201 to 300, 101 to 200, or <100, respectively. The new definition is
known as the Berlin definition of ARDS. Only choice D has a P/F ratio low enough to be called
ARDS by the AECC or the Berlin definition.

36. C. The combination of an opioid with a benzodiazepine addresses both pain and anxiety and is
effective in helping many ARDS patients achieve synchrony with the ventilator. While they can
have a significant effect on hemodynamics, both fentanyl and midazolam (in modest doses) are
relatively well tolerated by the septic patient suffering from hypotension requiring
vasopressors. Choices A and B are incorrect because they involve neuromuscular blockade
without any amnestic/anxiolytic agent. Choice D is incorrect because the patient appears to be in
pain and lorazepam lacks analgesic properties. Additionally, cough suppression is a quality of
opioids that are lacking in benzodiazepines.

37. C. The non-rebreathing mask with reservoir bag can deliver an Fio 2 of nearly 100% when
oxygen flo w is 15 L/min or greater. An Spo2 of 91% on an Fio 2 of 100% should alert the
clinician to the likely need for endotracheal intubation and mechanical ventilation. Choice A is
clearly incorrect as the Fio 2 is only 21%. Choice B is incorrect because the Fio 2 is only 35%
and can be further increased. Choice D is incorrect assuming that other variables are safe and
stable (Paco2, mental status, ability to protect airway). Many chronic obstructive pulmonary
disease patients in the ICU benefit from short-term support from noninvasive positive-pressure
ventilation and do quite well with Spo2 readings in the low 90s.

38. A. Many studies have sought to determine the factors that result in the fastest time to weaning
from mechanical ventilation. The daily spontaneous breathing trial has emerged as a simple
maneuver that consistently results in fewer days on mechanical ventilation. Choice B is
incorrect because SIMV has not been shown to reduce days on mechanical ventilation. Choice C
is not correct because blanket administration of bronchodilating medications has not been
shown to expedite weaning from the ventilator. Choice D is not correct because unnecessary
bronchoscopy exposes the patient to the risks of the procedure without benefit.

39. D. Choices A, B, and C describe features that are common to both IM V and SIMV. SIMV
allows a patient’s respiratory effort to initiate mandatory ventilator-supported breaths; while in
IMV, the mandatory breaths are delivered on a preset schedule without regard for the patient’s
spontaneous breaths. Therefore, ventilation with IM V has a higher risk of breath stacking and
volutrauma, as a patient can take a large spontaneous breath just prior to the ventilator
delivering a mandatory breath.

40. C. The risk of subglottic tracheal stenosis increases after an endotracheal tube has been in
place for 2 to 3 weeks. There is a trend toward performing early tracheostomy, which may
facilitate weaning from the ventilator. Waiting 8 to 10 weeks to perform tracheostomy on an
intubated patient would put the patient at undue risk for tracheal stenosis.

41. B. This severe pancreatitis patient is at high risk for acute respiratory distress syndrome
(ARDS). When compared with the tidal volume of 12 mL/kg, 6 mL/kg was shown to reduce
mortality in patients with ARDS. Choice A is incorrect because 2 mL/kg is not larger than
anatomic dead space, so it would lead to progressive respiratory acidosis and hypoxia. Choices
C and D are incorrect because these tidal volumes are too large for a patient at risk for ARDS.

42. B. Dexmedetomidine is a titratable, intravenous a2-receptor agonist with sedative and


anxiolytic effects. Compared with propofol, benzodiazepines, and barbiturates, it causes less
respiratory depression. This makes the drug well suited for cases in which the patient oscillates
between apnea/unresponsiveness on large doses of propofol or benzodiazepines, and extreme
agitation on lower doses. Young trauma patients being weaned from mechanical ventilation
often tolerate the endotracheal tube very poorly, and some benefit from dexmedetomidine in the
period leading up to extubation. Choice A is incorrect because methadone, with its complex
pharmacokinetic profile, is difficult to titrate over a short period of time and would not provide
direct anxiolysis. Choice C is incorrect because nitrous oxide is an impractical agent for use in
the ICU. Choice D is incorrect because a transdermal fentanyl patch would be difficult to titrate
over a short period of time and would not provide direct anxiolysis.

43. D. With mechanical ventilation, the “ independent variable” is manipulated or “ set” by the
operator, while the dependent variable results from the system and cannot be “ set” by the
operator. In pressure-preset ventilation, the inspiratory airway pressure is set to a desired value,
and thus is an independent variable. The tidal volume is not set; rather, it results from a
combination of variables within the system, including the set inspiratory airway pressure, lung
compliance, chest-wall resistance, and the set inspiratory time. Thus, tidal volume is a
dependent variable. In volume-preset ventilation, or “ volume control ventilation,” the opposite
is true; the independent variable is tidal volume, and the dependent variable is airway pressure.
Choice A is incorrect because tidal volume is not an independent variable and Fio 2 is not a
dependent variable. Choice B is incorrect because tidal volume is not an independent variable
and frequency is not a dependent variable. Choice C is incorrect because Spo2 is never an
independent variable and airway pressure is not a dependent variable in pressure-preset
ventilation.

44. C. This is a case of nearly pure respiratory acidosis. The pH is very low as a result of a
significantly elevated Pc o 2 . The management of a respiratory acidosis consists of increasing
the minute ventilation by increasing either respiratory rate (choice C) or tidal volume (not
given as an answer choice). Choices A and B are incorrect because neither would result in an
increased minute ventilation. Choice D is incorrect because giving bicarbonate w ill temporarily
increase the pH, but w ill not address the underlying problem of inadequate elimination of CO2.

45. C. PEEP helps to expand collapsed alveoli, which improves ventilation/perfusion matching by
reducing shunt. Choice A is incorrect because PEEP does not directly impact ventilation, which
determines the elimination of CO2. Choice B is incorrect because PEEP typically reduces
venous return, which in turn reduces the cardiac output. Choice D is incorrect because in
general, the peak inspiratory pressure w ill increase when PEEP is added.
46. C. The patient is tolerating CPAP well, and oxygenation has improved, but ventilation remains
inadequate. By changing modes to BiPAP, inspiratory pressure w ill be added, which w ill likely
increase ventilation and improve the hypercarbic respiratory acidosis. Choices A, B, and C are
incorrect because the patient’s arterial oxygen level is adequate, and increasing the FiO2 or
CPAP would cause an increase in Pao2.

47. D. In cases of severe ARDS and ventilator dyssynchrony refractory to high-dose sedatives and
opioids, neuromuscular blockade is appropriate. The risk of polymyoneuropathy of critical
illness is increased by the use of neuromuscular-blocking agents, but the immediate risks of
hypoxemia and hypoventilation are more pressing. Choice A is incorrect because the patient is
already on high-dose propofol with adverse effect (worsening hypotension). Choice B is
incorrect because the patient is already failing on fu ll ventilator support. A change to pressure-
support ventilation would almost certainly result in worsened oxygenation. While diuresis has a
role in the care of patients with refractory hypoxemia, choice C is incorrect because the patient
is hypotensive. Adding hypovolemia to a distributive shock patient could result in
hemodynamic collapse.

48. D. Ketamine is an N-methyl-D-aspartate antagonist with dissociative and analgesic effects. It


also has sympathomimetic effects, so it w ill often cause an increase in heart rate and blood
pressure. It has minimal effect on respiratory drive. This combination of qualities makes it well
suited for procedural sedation in ICU patients, especially for those patients who have developed
tolerance to opioids and GABAergic agents. Choices A and B are clearly incorrect because oral
medications are impractical for procedural sedation in the ICU. Choice C is incorrect because
the patient is already on large doses of hydromorphone and morphine, is likely to have
developed tolerance to opioids, and therefore, is unlikely to have a significant response to
additional morphine administration.

49. A. This patient is suffering from negative-pressure pulmonary edema resulting from the
patient’s strong respiratory effort, which generates a large negative intrathoracic pressure in the
setting of an occluded airway. In this setting, the large negative pressure in the alveoli pulls
fluid from pulmonary capillaries into the airspaces. This can result in the immediate onset of
severe pulmonary edema, shunt, and hypoxia. A bite block prevents occlusion of the
endotracheal tube.

50. B. The half-life of carboxyhemoglobin is reduced significantly by ventilation with 100%


oxygen. Choices A and D are treatments for cyanide poisoning. Ventilation with air would
greatly prolong the time to resolution of carboxyhemoglobinemia.
Postoperative Anesthesia Care
Sheri Berg and Edward Bittner

1. The most common cause of postanesthesia care unit (PACU)-related malpractice claims is
A. Under treated pain
B. Critical respiratory incidents
C. Nerve injury from regional blocks
D. Cardiovascular events

2. Which of the statements regarding the American Society of Anesthesiologists (ASA) Standards
for Postanesthesia Care is true?
A. A physician is responsible for the discharge of a patient from the postanesthesia care unit
(PACU)
B. Medical supervision and coordination of patient care in the PACU should be the
responsibility of an anesthesiologist
C. Use of a PACU scoring system is recommended
D. A ll of the above

3. The most common cause of postoperative airway obstruction is


A. Loss of pharyngeal tone in a sedated patient
B. Weak diaphragmatic contraction
C. Redundant pharyngeal tissue
D. Laryngeal edema

4. A 36-year-old man who underwent a laparoscopic cholecystectomy develops upper airway


obstruction in the PACU. You suspect that residual neuromuscular blockade is a major
contributing factor. Which of the follow ing would exclude the presence of residual
neuromuscular blockade?
A. Oxygen saturation of 98% on 2-L nasal cannula
B. Normal tidal volumes while spontaneously breathing
C. Normal end-tidal carbon dioxide concentration while spontaneously breathing
D. None of the above

5. After reversal of neuromuscular blockade, pharyngeal function returns to baseline when the
adductor pollicis train-of-four (TOF) ratio is greater than
A. 0.9
B. 0.7
C. 0.5
D. 0.4

6. Which of the follow ing is considered the “ gold standard” when using clinical assessment to
evaluate for residual neuromuscular blockade?
A. Tongue protrusion
B. A b ility to lift the head o ff the bed for 5 seconds
C. A b ility to lift the legs o ff the bed for 5 seconds
D. Hand-grip strength

7. Which of the follow ing metabolic states can contribute to residual neuromuscular blockade?
A. Hypocalcemia
B. Hypomagnesemia
C. Hyperthermia
D. Alkalosis

8. You are called to evaluate a 14-year-old g irl in the postanesthesia care unit (PACU) with
decreased oxygen saturation. The nurse tells you her anesthesia team “ extubated her deep.” You
determine that she is in laryngospasm. Which of the follow ing would be the most appropriate
first step in her management?
A. Wait for 5 minutes, watch her, and reassess
B. Administer 2 mg/kg of propofol
C. Provide a jaw thrust and apply continuous positive airway pressure (CPAP)
D. Administer 0.5 mg/kg of succinylcholine

9. A 40-year-old woman undergoes an 8-hour spine surgery and is left intubated for concern of
airway edema. Which of the follow ing statements regarding the assessment of airway edema is
correct?
A. The absence of facial edema excludes the presence of airway edema
B. The presence of air movement around the endotracheal tube with the cuff deflated excludes
the presence of airway edema
C. The absence of scleral edema excludes the presence of airway edema
D. The cuff-leak test cannot exclude the presence of airway edema

10. Strategies to reduce the risk of airway obstruction in patients with obstructive sleep apnea
(OSA) include all of the following, except
A. Administration of benzodiazepines in place of opioids to reduce anxiety
B. Application of postoperative continuous positive airway pressure (CPAP)
C. Use of continuous regional anesthetic techniques
D. Preoperative screening to identify patients at high risk

11. You are called to evaluate a 65-year-old man in the postanesthesia care unit who underwent a
left-carotid endarterectomy earlier in the day. He is having difficulty breathing, and you notice
that the left side of his neck appears swollen. As you examine him, he becomes agitated and his
oxygen saturation decreases to 92%. You ask for the surgeon be called “ stat” and attempt bag
mask ventilation. The next step to take is
A. Wait for the surgeon to arrive
B. Release the sutures and evacuate the hematoma
D. Administer naloxone
E. Apply noninvasive ventilation

12. The most common cause of transient postoperative hypoxemia in the postanesthesia care unit
(PACU) is
A. Micro aspiration
B. Pneumothorax
C. Alveolar hypoventilation
D. Pulmonary embolus

13. For every 1-mm Hg increase in arterial Paco2, minute ventilation increases by

A. 0.5 L/min
B. 1 L/min
C. 2 L/min
D. 4 L/min

14. Which of the follow ing is not a cause of arterial hypoxemia in the postanesthesia care unit?
A. Decreased alveolar partial pressure of oxygen
B. Ventilation-to-perfusion mismatch
C. Shunt
D. Decreased venous admixture

15. A healthy 21-year-old college football player is admitted to the postanesthesia care unit with
hypoxemia after undergoing an Achilles tendon repair. The patient developed laryngospasm
after extubation in the operating room, which resolved after application of positive pressure.
What is the likely cause of his pulmonary edema?
A. Cardiovascular dysfunction
B. Aspiration
C. Postobstructive pulmonary edema
D. Volume overload
16. Administration of 5 L/min of oxygen by nasal cannula results in an Fio 2 delivery of

A. 0.45
B. 0.41
C. 0.34
D. 0.28

17. Which of the follow ing postoperative hemodynamic abnormalities is the most predictive of
unplanned ICU admission and mortality
A. Tachycardia
B. Bradycardia
C. Hypotension
D. Hypertension

18. The most common cause of systemic hypotension in the postanesthesia care unit (PACU) is
A. Intravascular volume depletion
B. Myocardial ischemia
C. Residual anesthetic effects
D. Vasodilation

19. A sympathetic block above which level can result in bradycardia and hypotension?
A. T4
B. T6
C. T8
D. T10

20. Which of the follow ing statements regarding a perioperative anaphylactic reaction is most
correct?
A. Vasopressin is the drug of choice to treat anaphylaxis
B. The absence of bronchospasm and rash excludes the diagnosis of anaphylaxis
C. Low serum tryptase concentrations can differentiate between anaphylactic and
anaphylactoid reactions
D. Neuromuscular-blocking drugs are the most common causes of anaphylactic reactions in
the perioperative setting

21. You are called to the bedside to evaluate new ST-segment depressions on a routine
postoperative EKG of a 42-year-old woman who underwent a partial colectomy. She is
asymptomatic. Her heart rate is 80 to 90 bpm, and her blood pressure is 135/60. Your next step
in the management includes
A. Wait and monitor her
B. Send o ff one set of troponins
C. Administer metoprolol for HR control
D. Call a cardiology consult

22. Which of the follow ing patients warrants a routine postoperative EKG?
A. An 85-year-old male with hypothyroidism who underwent a cystoscopy and ureteral stent
placement
B. A 72-year-old male with coronary artery disease (CAD) and hypertension who underwent
an ankle fusion
C. A 52-year-old male with hypertension, hyperlipidemia, and diabetes who underwent a
radical prostatectomy
D. A 50-year-old male with rheumatoid arthritis who underwent bilateral knee replacements

23. You are called to evaluate a 68-year-old woman who underwent a right upper lobectomy for
lung cancer. She is complaining of chest pain and palpitations and explains to you that she has
never had this problem before. Her EKG demonstrates atrial fibrillation with a rate of 152. Her
blood pressure is currently 65/40. Which of the follow ing is the most appropriate first step in
managing her?
A. Repeat EKG in 15 minutes
B. Administer 150 mg IV amiodarone
C. Administer 5 mg IV metoprolol
D. Electrical cardioversion

24. Postoperative premature ventricular contractions (PVCs) most commonly are a result of
A. QT prolongation
B. Excessive P-blocker administration
C. Increased sympathetic system stimulation
D. Residual volatile anesthetics

25. Which of the follow ing could result in bradydysrhythmias in the postoperative period?
A. Opioid administration
B. Bowel distention
C. Increased intraocular pressure
D. A ll of the above

26. What percentage of patients over the age of 50 who undergo elective surgery w ill experience
postoperative delirium within the first 5 days follow ing their surgical procedure?
A. <1%
B. 5%
C. 10%
D. 25%

27. Which of the follow ing intraoperative factors is predictive of postoperative delirium?
A. Blood loss
B. Anesthetic technique
C. Intraoperative hypotension
D. Intraoperative hypertension

28. Each of the follow ing increases the risk of postoperative delirium, except
A. Advanced age
B. Preexisting cognitive impairment
C. Alcohol abuse
D. Chronic pain

29. Which of the follow ing statements regarding emergence excitement is most correct?
A. It is most common in children aged 6 to 8 years
B. Less than 5% of children experience emergence excitement
C. It is associated with long-term cognitive sequelae
D. Preoperative midazolam administration is associated with an increased incidence

30. Oliguria is defined as urine output less than


A. 0.2 mL/kg/hr
B. 0.5 mL/kg/hr
C. 0.7 mL/kg/hr
D. 1.0 mL/kg/hr

31. Postoperative urinary retention (POUR) is the inability to void despite a bladder volume of
A. 100 to 200 mL
B. 300 to 400 mL
C. 500 to 600 mL
D. 700 to 800 mL

32. The most common cause of oliguria in the immediate postoperative period is
A. Low cardiac output
B. Acute tubular necrosis
C. Renal vascular obstruction
D. Hypovolemia

33. An intra-abdominal pressure higher than which of the follow ing is required to impede renal
perfusion?
A. 10 cm H2O
B. 15 cm H2O
C. 20 cm H2O
D. 3 0 c m H 2O

34. A 42-year-old m orbidly obese male undergoes a laparoscopic gastric bypass. The surgical
procedure lasts 8 hours. Estimated blood loss is 200 mL, and he receives 4.5 L of crystalloid. In
the postanesthesia care unit, his urine output is 5 to 10 mL/hr despite an additional 1 L of
crystalloid. The most likely etiology of his oliguria is
A. Contrast-induced nephropathy
B. Rhabdomyolysis
C. Hypovolemia
D. Surgical injury of ureters

35. A ll of the follow ing are consequences of moderate hypothermia (33-35°C), except
A. Inhibition of platelet function
B. Prolongation of neuromuscular blockade
C. Inhibition of drug metabolism
D. Increases coagulation-factor activity

36. The most accurate measurement of core body temperature is obtained via
A. A xillary
B. Tympanic membrane
C. Rectal
D. Nasopharyngeal

37. The most effective treatment for abolishing postoperative shivering is


A. Clonidine
B. Doxapram
C. Meperidine
D. Fentanyl

38. A 22-year-old nonsmoking woman with no previous anesthetic history undergoes a


laparoscopic ovarian cystectomy. Her risk of postoperative nausea and vomiting (PONV) is
most closely approximated by
A. 5%
B. 10%
C. 20%
D. 40%

39. A 30-year-old woman who underwent a knee arthroscopy has postoperative nausea and
vomiting (PONV) in the postanesthesia care unit (PACU). Per report, she received ondansetron
4 mg IV 30 minutes prior to the conclusion of her procedure. Which of the follow ing
treatments is most appropriate for managing her PONV in the PACU?
A. Scopolamine patch
B. Dexamethasone
C. Ondansetron
D. Promethazine

40. Which of the follow ing is the most frequent cause of delayed awakening in the postanesthesia
care unit (PACU)?
A. Hypothermia
B. Hypoglycemia
C. Residual effects of sedatives
D. Hypercarbia

41. Which of the follow ing general principles regarding discharge from the postanesthesia care
unit (PACU) is correct?
A. A mandatory minimum stay in the PACU is not required
B. Patients should not be discharged until they are pain-free
C. Patients need to void prior to PACU discharge
D. Patients need to demonstrate the ability to drink and retain clear fluids prior to PACU
discharge

42. According to the ASA Standards for Postanesthesia Care, which of the follow ing statements is
correct?
A. A patient is to be transported to the postanesthesia care unit (PACU) by at least one
physician
B. A patient must be monitored by continuous pulse oximetry during transport to the PACU
C. A patient who solely received regional anesthesia may routinely bypass the PACU
D. A patient must be discharged from the PACU by a physician

43. An otherwise-healthy adult male breathing room air receives a large dose of opioid that
depresses his ventilation to the point that his alveolar PaCO2 is 80 mm Hg. What is his predicted
alveolar Pao2?

A. 40 mm Hg
B. 50 mm Hg
C. 70 mm Hg
D. 90 mm Hg

44. If the patient described in the previous question is administered 2 L of oxygen by nasal cannula,
then his alveolar Pao2 increases to what amount?

A. 60 mm Hg
B. 80 mm Hg
C. 100 mm Hg
D. 120 m m Hg

45. A 42-year-old woman complains of pain and inability to dorsiflex the first toe. The nerve most
likely to be involved is the
A. Sciatic
B. Femoral
C. Tibial
D. Peroneal

46. Which of the follow ing clinical criteria is associated with transfusion-related acute lung injury
(TRALI) as compared to transfusion-associated circulatory overload (TACO)?
A. Pulmonary edema
B. Hypoxemia
C. Leukopenia
D. Leukocytosis

47. Which of the follow ing statements regarding postoperative shivering is most correct?
A. Occurs with general anesthesia but not epidural anesthesia
B. Is always associated with a decrease in body temperature
C. In normothermic patients is related to a hypothalamic depressant effects of opioids
D. In normothermic patients results from uninhibited spinal reflexes

48. A 49-year-old woman with nephrolithiasis develops tachycardia, low-grade fever, and
hypotension after a ureteral stent placement. Urine output is 5 to 10 mL/hr. A ll of the follow ing
can be used to treat the patient, except
A. Antibiotics
B. Fluids
C. Diuretics
D. Vasopressors

49. A ll of the follow ing are advantages of high-flow nasal cannula delivery systems, except
A. Humidification of the gas
B. Gas delivery up to 6 L/min
C. Deliver of gas throughout the respiratory cycle
D. A b ility to deliver warm gas (37°C)

50. Which of the follow ing procedures is most likely to be associated with postoperative
hypertension?
A. Craniotomy
B. Colectomy
C. Gastric bypass
D. Hip arthroplasty
CHAPTER 20 ANSWERS

1. B. Critical respiratory events accounted for more than half of the PACU malpractice claims in
the US closed-claims database.

2. D. The ASA has adopted Standards for Postanesthesia Care that delineate the minimum
requirements for PACU monitoring and care. A ll of the statements are contained within the ASA
Standards for Postanesthesia Care.

3. A. Airway obstruction is a common and potentially devastating complication in the


postoperative period. The most frequent cause of postoperative airway obstruction is the loss of
pharyngeal tone due to the residual depressant effects of inhaled and intravenous anesthetics and
the persistent effects of neuromuscular-blocking drugs.

4. D. The possibility of residual neuromuscular blockade must be considered as a potential cause


of upper airway obstruction in any patient who received neuromuscular-blocking drugs during
anesthesia. Residual neuromuscular blockade may not be clinically evident because the
diaphragm recovers from neuromuscular blockade before the pharyngeal muscles do. End-tidal
carbon dioxide concentrations, oxygen saturation, and tidal volumes may indicate adequate
ventilation and oxygenation, while the ability to maintain a patent upper airway and clear upper
airway secretions remains compromised.

5. A. Measurement of the TOF ratio is commonly used to assess reversal of neuromuscular


blockade. Significant clinical weakness may persist to a ratio of 0.7, and pharyngeal function
does not return to baseline until an adductor pollicis TOF ratio is greater than 0.9.

6. B. In awake patients, clinical assessment of reversal of neuromuscular blockade is preferred


to the application of painful train-of-four or tetanic stimulation. Clinical evaluation includes
grip strength, tongue protrusion, the ability to lift the legs o ff the bed, and the ability to lift the
head o ff the bed for a fu ll 5 seconds. Of these maneuvers, the 5-second sustained head lift is
considered the gold standard because it reflects not only generalized motor strength but, more
importantly, the patient’s ability to maintain and protect the airway.

7. A. If persistence or return of neuromuscular weakness in the postanesthesia care unit is


suspected, prompt review of possible etiologic factors is indicated. Metabolic states that can
contribute to prolonged neuromuscular blockade include hypocalcemia, hypermagnesemia,
hypothermia, respiratory acidosis, and hepatic and renal failure.

8. C. Laryngospasm refers to a sudden spasm of the vocal cords that completely occludes the
laryngeal opening. Although it is most likely to occur in the operating room at the time of
tracheal extubation, patients who arrive in the PACU asleep after general anesthesia are also at
risk for laryngospasm. Jaw thrust with CPAP is often sufficient stimulation to “ break” the
laryngospasm. If jaw thrust and CPAP maneuvers fail, then administration of propofol and
providing muscle relaxation with succinylcholine are effective treatments.

9. D. Airway edema is a possible complication in patients undergoing prolonged procedures in


the prone position and in procedures with large amounts of blood loss, requiring aggressive
fluid resuscitation. Although facial and scleral edema are important physical signs that can alert
the clinician to the presence of airway edema, significant edema of pharyngeal tissue is often
not accompanied by visible external signs. If tracheal extubation is to be attempted in these
patients in the postanesthesia care unit, evaluation of airway patency must precede removal of
the endotracheal tube (ETT). The patient’s ability to breathe around the ETT can be evaluated by
a “ cuff-leak” test. By deflating the ETT cuff and with occlusion of the proximal end of the ETT,
the patient is asked to breathe around the tube. Good air movement suggests that the patent’s
airway w ill remain patent after tracheal extubation. Though helpful, the cuff-leak “ test” does not
fu lly exclude the presence of airway edema.

10. A. Patients with OSA are particularly prone to airway obstruction and, therefore, deserve
special consideration in the postanesthesia care unit (PACU). Patients with OSA are exquisitely
sensitive to opioids, and when possible, continuous regional anesthesia techniques should be
used to provide postoperative analgesia. Benzodiazepines can have a more intense effect on
pharyngeal muscle tone than opioids and, therefore, can contribute to airway obstruction in the
PACU. When caring for a patient with OSA, plans should be made preoperatively to provide
CPAP in the immediate postoperative period. The m ajority of patients with m ild to moderate
OSA are undiagnosed at the time of surgery; therefore, care should be taken to identify at-risk
patients based on preoperative clinical suspicion, a history of snoring, and daytime sleepiness.

11. B. An obstructed upper airway requires immediate attention. It may not be possible to mask-
ventilate a patient with severe upper airway obstruction as a result of edema or hematoma. In the
case of hematoma after carotid surgery, an attempt should be made to decompress the airway by
opening the wound and evacuating the hematoma. This maneuver may not effectively
decompress the airway if a significant amount of fluid or blood has infiltrated the tissue planes
of the pharyngeal wall. If emergency tracheal intubation is required, it is important to have
ready access to difficult airway equipment and surgical backup for performance of an
emergency tracheostomy.

12. C. Alveolar hypoventilation and atelectasis are the most common causes of transient
postoperative hypoxemia in the PACU. Microaspiration, pneumothorax, and pulmonary
embolus are less common causes of postoperative hypoxemia.

13. C. Under normal conditions, minute ventilation increases by approximately 2 L/min for every
1-mm Hg increase in arterial PacO2. This normal ventilatory response to carbon dioxide can be
significantly depressed in the immediate postoperative period by the residual effects of drugs.

14. D. Increased venous admixture, decreased alveolar partial pressure of oxygen, ventilation-to-
perfusion mismatch, and shunt are causes of arterial hypoxemia in the postoperative period.
Increased venous admixture is due to mixing of desaturated venous blood with oxygenated
arterial blood. Normally, only 2% to 5% of cardiac output is shunted through the lungs, and this
small amount of shunted blood with a normal mixed venous saturation has a minimal effect on
Pao2. In low cardiac output states, or conditions in which the shunt fraction increases (such as
pulmonary edema and atelectasis), there is mixing of a greater amount of desaturated shunted
blood with saturated arterialized blood, which decreases the Pao2.

15. C. Postobstructive pulmonary edema is a rare, but significant complication resulting from
upper airway obstruction produced by the exaggerated negative pressure generated by
inspiration against a closed glottis. This exaggerated negative intrathoracic pressure increases
venous return, which further promotes the transudation of fluid. Muscular healthy patients are at
increased risk because of their ability to generate significant inspiratory force. Laryngospasm
is the most common cause of upper airway obstruction leading to postobstructive pulmonary
edema, but pulmonary edema may result from any condition that occludes the upper airway.
Arterial hypoxemia is usually manifested within 90 minutes after development of
postobstructive pulmonary edema and is accompanied by bilateral flu ffy infiltrates on the chest
radiograph. The diagnosis depends on clinical suspicion once other causes of pulmonary
edema are ruled out. Treatment is supportive and includes supplemental oxygen, diuresis, and
positive-pressure ventilation.

16. B. As a general rule, each L/min of oxygen flo w through nasal cannula increases Fio 2 by 0.04,
with 5 L/min resulting in approximately 0.41 Fio 2 [0.04 x 5 = 0.2 + 0.21 (room air) = 0.41].

17. D. Hemodynamic alterations in the postoperative period can have a negative impact on
outcome. Surprisingly, postoperative hypertension and tachycardia are more predictive of
unplanned admission to the critical care unit and mortality rate than are hypotension and
bradycardia.

18. A. Intravascular volume depletion (hypovolemia) is the most common cause of hypotension in
the PACU. Common causes of decreased intravascular volume in the immediate postoperative
period include ongoing third-space translocation of fluid, inadequate intraoperative fluid
replacement, and loss of sympathetic nervous system tone as a result of spinal or epidural
blockade. Bleeding should be ruled out as a cause of hypovolemia in patients who have
undergone a surgical procedure in which significant blood loss is possible.

19. A. A high sympathetic block (T4) can result in bradycardia and hypotension secondary to
blockade of the cardioaccelerator fibers. This should be treated promptly with vasopressors,
such as ephedrine, as cardiac arrest secondary to bradycardia and hypotension can ensue.
Epinephrine is used when there is severe bradycardia and hypotension.

20. D. Anaphylactic (or anaphylactoid) reactions may be the cause of postoperative hypotension.
Anaphylaxis should be considered in cases of sudden refractory hypotension even when not
accompanied by the classic signs of bronchospasm and rash. Increased serum tryptase
concentrations confirm the occurrence of an allergic reaction, but this change does not
differentiate anaphylactic from anaphylactoid reactions. Neuromuscular-blocking drugs are the
most common cause of anaphylactic reactions in the operative setting. Epinephrine is the drug
of choice to treat anaphylaxis.

21. A. Postoperative ECG changes should be interpreted in light of the patient’s cardiac history
and risk index. In low -risk patients (<45 years of age, no known cardiac disease, only one risk
factor), postoperative ST-segment changes on the ECG do not usually indicate myocardial
ischemia. Relatively benign causes of ST-segment changes in these low -risk patients include
anxiety, gastroesophageal reflux disease, hyperventilation, and hypokalemia. In general, low-
risk patients require only routine postanesthesia care unit observation unless associated signs
and symptoms warrant further clinical evaluation.

22. C. A routine postoperative ECG is only recommended for patients with known or suspected
CAD who have undergone high- or intermediate-risk surgery. High-risk surgery includes
emergency surgery, major vascular surgery, peripheral vascular surgery, and unanticipated
prolonged procedures associated with large fluid shifts or blood loss. Intermediate-risk
procedures include intra-abdominal and thoracic surgery, carotid endarterectomy, head and
neck surgery, orthopedic surgery, and prostate surgery.

23. D. Control of the ventricular response rate is the immediate goal in the treatment of new-onset
atrial fibrillation. While most patients can be treated pharmacologically, hemodynamically
unstable patients require prompt electrical cardioversion.

24. C. PVCs and ventricular bigeminy are common in the postoperative period. PVCs most often
reflect increased sympathetic nervous system stimulation, as many occur with hypoxemia,
hypercapnia, and acidemia.

25. D. Bradycardia in the PACU is often iatrogenic. Drug-related causes include administration of
a-blockers, opioids, anticholinesterase agents, and treatment with dexmedetomidine. Procedure-
and patient-related causes include bowel distention, increased intracranial or intraocular
pressure, and spinal anesthesia.

26. C. Approximately 10% of adult patients older than 50 years who undergo elective surgery w ill
develop postoperative delirium within the first five postoperative days.

27. A. Intraoperative factors that are predictive of postoperative delirium include surgical blood
loss, the number of intraoperative blood transfusions, and hematocrit less than 30%.
Intraoperative hemodynamic derangements and the anesthetic technique do not seem to be
predictors of postoperative delirium.

28. D. Many adult patients at risk for postoperative delirium can be identified preoperatively. The
most significant preoperative risk factors include (1) advanced age, (2) preoperative cognitive
impairment, (3) decreased functional status, (4) alcohol abuse, and (5) a previous history of
delirium. Chronic pain is not a risk factor for postoperative delirium.

29. D. Emergence excitement is a transient confusional state that is associated with emergence
from general anesthesia. It is common in children, with more than 30% experiencing agitation
or delirium at some period during their postanesthesia care unit stay. The peak age of
emergence excitement in children is between 2 and 4 years. Unlike delirium, emergence
excitement typically resolves quickly and without long-term cognitive sequelae. Preoperative
midazolam administration has been associated with an increase in the incidence and duration of
emergence delirium in children.

30. B. Postoperative oliguria can result from prerenal, renal, and postrenal causes. Frequently, the
cause is multifactorial, with a preexisting renal insufficiency that is exacerbated by an
intraoperative insult. Oliguria is defined as urine output less than 0.5 mL/kg/hr.

31. C. POUR is defined as the inability to void despite a bladder volume of more than 500 to 600
mL. Risk factors include male gender, age older than 50 years, intraoperative fluid volume,
duration of surgery, and bladder volume on admission. Certain types of surgery are also
associated with a higher risk of POUR, including anorectal and jo in t replacement surgery.
Commonly used perioperative medications such as anticholinergics, P-blockers, and narcotics
can also contribute to POUR. Diagnosis can be made by clinical examination, bladder
catheterization, or ultrasound assessment. Bladder volumes measured by ultrasound imaging
correlate well with volumes obtained by urinary catheterization.

32. D. The most common cause of oliguria in the immediate postoperative period is hypovolemia.
A fluid challenge is usually effective in restoring urine output. Volume resuscitation to
maximize renal perfusion is particularly important in order to prevent the development of acute
kidney injury. If an intravascular fluid challenge is contraindicated or oliguria persists,
assessment of intravascular volume or cardiac function is indicated to differentiate
hypovolemia from low cardiac output states.

33. D. An intra-abdominal pressure higher than 30 cm H2O can impede renal perfusion, leading to
renal ischemia and postoperative renal dysfunction. Bladder pressure should be measured in
patients in whom intra-abdominal hypertension is suspected so that abdominal decompression
can be performed to relieve intra-abdominal pressure and restore renal perfusion.

34. B. Rhabdomyolysis is a recognized cause of postoperative renal insufficiency in m orbidly


obese patients, particularly those who have undergone gastric bypass procedures. Risk factors
include the body mass index and duration of surgery. Volume loading, diuretics, and
alkalinization of urine to flush the renal tubules can prevent ongoing renal tubular damage and
subsequent acute renal failure.
35. D. M ild to moderate hypothermia (33-35°C) is a recognized cause of a number of
postoperative complications, including inhibition of platelet function, reduced coagulation-
factor activity, and decreased drug metabolism. In addition, it exacerbates postoperative
bleeding, prolongs neuromuscular blockade, and may delay awakening.

36. B. Core body temperature can most accurately be measured at the tympanic membrane.
Axillary, rectal, and nasopharyngeal temperature measurements are less accurate and may
underestimate core temperature.

37. C. A number of opioids and clonidine are effective in stopping shivering once it starts, but
meperidine is the most effective treatment. Doxapram, a central nervous system stimulant, is
somewhat effective in abolishing postoperative shivering.

38. D. A simple risk score consisting of four factors can be used to identify high-risk patients for
PONV. The four risk factors are (1) female gender, (2) history of motion sickness or PONV, (3)
nonsmoking, and (4) the use of postoperative opioids. The incidence of PONV correlates with
the number of these factors present: zero, one, two, three, or four factors correspond to an
incidence of 10%, 21%, 39%, 61%, and 79%, respectively. The patient in the vignette has two
risk factors (female, nonsmoker), so her approximate risk of PONV is 40%.

39. D. When choosing a rescue antiemetic for patients with PONV, both the class of drug and the
timing of administration are factors. If an adequate dose of antiemetic given at the appropriate
time proves ineffective, simply giving more of the same class of drug in the PACU is unlikely
to be of significant benefit. If no prophylactic drug was given, the recommended treatment is a
low-dose 5-HT3 antagonist, ondansetron. Of the choices provided in the vignette, promethazine
is likely to be the most effective rescue antiemetic. Since the patient received ondansetron for
prophylaxis, additional ondansetron is unlikely to be effective. A scopolamine patch is unlikely
to take effect rapidly enough to be beneficial. Dexamethasone, while effective for prophylaxis,
is less beneficial for rescue.

40. C. Even after prolonged surgery and anesthesia, a response to stimulation in 60 to 90 minutes
should be expected. The etiology of delayed awakening after anesthesia can be divided into the
general categories of pharmacologic, metabolic, and neurologic causes. Of these, residual
sedation from drugs used during anesthesia is the most frequent cause of delayed awakening in
the PACU.

41. A. Specific PACU discharge criteria may vary, but certain general principles are universally
applicable. These principles include mandatory minimum stay in the PACU is not required,
patients must be observed until they are no longer at risk for respiratory depression, and their
mental status is clear or has returned to baseline; hemodynamic criteria are based on the
patient’s baseline hemodynamics without specific systemic blood pressure and heart rate
requirements. To facilitate PACU discharge, discharge scoring systems have been developed
and modified over time to reflect current anesthesia practice.
42. D. The Standards for Postanesthesia Care are intended to ensure the quality of postanesthetic
patient care. They include the following:

• Standard I: “ A ll patients who have received general anesthesia, regional anesthesia, or


monitored anesthesia care shall receive appropriate post anesthesia management”
• Standard II: “ A patient transported to the PACU shall be accompanied by a member of the
anesthesia care team who is knowledgeable about the patient’s condition. The patient shall be
continually evaluated and treated during transport with monitoring and support appropriate to
the patient’s condition”
• Standard III: “ Upon arrival to the PACU, the patient shall be reevaluated and a verbal report
provided to the responsible PACU nurse by the member of the anesthesia care team who
accompanies the patient”
• Standard IV: “ The patient’s condition shall be evaluated continually in the PACU”
• Standard V: “ A physician is responsible for the discharge of the patient from the PACU”

43. B. At sea level, a normocapnic patient breathing room air w ill have an alveolar oxygen
pressure of 100 mm Hg. Review of the alveolar gas equation demonstrates that hypoventilation
alone is sufficient to cause arterial hypoxemia in a patient breathing room air. In this case, a rise
in PaCO2 from 40 to 80 mm Hg (alveolar hypoventilation) results in an alveolar oxygen
pressure (Pao2) of 50 mm Hg.
Pao2 = FIO2 x (Patm - PH2 0 ) - PaC02/R)
= 0.21 x (760 - 47) - 80/0.8
= 50 mmHg

(Patm = atmospheric pressure mm Hg, PH20 = water vapor pressure mm Hg, R = respiratory
quotient— 8 C 0 2 molecules produced for every oxygen molecule consumed)

44. C. In the setting of isolated hypoventilation, modest increases in inspired oxygen are
remarkably effective at restoring alveolar oxygenation. For this patient, if 2 L of supplemental
oxygen is administered by nasal cannula, then the Fio 2 increases to approximately 28% and the
calculated alveolar Pao2 is 100 mm Hg.
Pao2 = FIO2 x (Patm - PH2 0 ) - PaC02/R)
= 0.28 x (760 - 47) - 80/0.8
= 100 mmHg

45. D. An assessment and written documentation of the patient’s peripheral nerve function on
discharge from the postanesthesia care unit may become useful information should a new
peripheral neuropathy develop in the later postoperative period. The peroneal nerve provides
the motor innervation for dorsiflexion of the first toe, while the tibial nerve allows plantar
flexion of the first toe.

46. C. TRALI can occur up to 6 hours after transfusion of blood, coagulation factor, or platelet
transfusions. Therefore, it should be included in the differential diagnosis of pulmonary edema
in the postanesthesia care unit, among patients who received intraoperative transfusions. The
resulting noncardiogenic pulmonary edema is often associated with fever, systemic
hypotension, and the presence of exudative pulmonary fluid. If a complete blood count is
obtained with the onset of symptoms, an acute decrease in the white blood cell count
(leukopenia) reflecting the sequestration of granulocytes is seen within the lung. Initially, it may
be difficult distinguishing TRALI from TACO caused by volume overload resulting from the
blood products transfused. In either case, treatment is supportive and includes supplemental
oxygen, dieresis, and mechanical ventilation, if needed.

47. D. Postoperative shivering commonly occurs after both general and neuraxial anesthesia and
is usually, but not always, associated with a decrease in the patient’s body temperature. Although
thermoregulatory mechanisms can explain shivering in a hypothermic patient, a separate
mechanism has been proposed to explain shivering in normothermic patients. The mechanism
of normothermic shivering is thought to be a result from uninhibited spinal reflexes, which are
manifested as clonic activity.

48. C. Urinary tract manipulation can result in sepsis in the postanesthesia care unit. In these cases,
hypotension is often accompanied by fever and rigor. If sepsis is suspected, fluid resuscitation
and vasopressor support should be initiated, blood should be obtained for culture, and antibiotic
therapy should be administered. The patient’s low urine output should be improved with
hemodynamic support. Diuretics are not indicated for a hypovolemic patient with sepsis.

49. B. Delivery of oxygen by traditional nasal cannula is limited to 6 L/min flo w to minimize
discomfort and complications that result from inadequate humidification. Alternatively, oxygen
can be delivered up to 40 L/min by high-flow nasal cannula systems, which humidify and warm
the gas to 99.9% relative humidity and 37°C. Unlike non-rebreather masks, these devices deliver
oxygen directly to the nasopharynx throughout the respiratory cycle.

50. A. A number of patient, procedural, and postoperative factors can contribute to the
development of postoperative hypertension. Patients with a history of essential hypertension are
at greatest risk for significant systemic hypertension in the postanesthesia care unit. Advanced
age, history of cigarette smoking, and preexisting renal disease are other patient-related risk
factors for postoperative hypertension. Surgical procedures that predispose the patient to
postoperative hypertension include craniotomy and carotid endarterectomy. Other common
postoperative causes of hypertension include pain, hypoxemia, hypoventilation and associated
hypercapnia, emergence excitement, shivering, bladder distension, drug withdrawal, and
hypervolemia.
Miscellaneous Topics
Paul Sikka and Thomas Halaszynski

1. A 90-year-old male is presented to the operating room for surgical repair of a right femoral
neck fracture. His medical history is significant for chronic obstructive pulmonary disease (60
pack year smoking history) and is prescribed 4 L/min of continuous home oxygen. A note from
his pulmonologist states that this patient is a high-risk candidate for general anesthesia and w ill
prove to be d ifficult to wean from mechanical ventilation. To properly assess the respiratory
risk for this patient, which of the follow ing w ill provide the least beneficial value?
A. Stat pulmonary function tests
B. Baseline chest radiograph
C. Thorough history and physical examination
D. Baseline arterial blood gas

2. A 65-year-old female, status post coronary artery bypass grafting (CABG) 2 weeks ago, is
scheduled for a fem-fem bypass. The patient has been recovering well since her routine two-
vessel cardiac bypass surgery, but continues to experience intermittent claudication symptoms
of the left lower extremity. The surgeon informs you that the patient was scheduled for the
vascular bypass surgery several weeks ago, but could not undergo the surgery due to her poor
cardiac function. Now that cardiac pathology has been resolved, he would like to proceed with
the vascular procedure as soon as possible. Your recommendations to the vascular surgeon
would be
A. Provided she is without cardiac symptoms, the vascular surgery can now be performed
B. The vascular procedure should be delayed for another 2 weeks
C. The surgeon needs to obtain cardiology clearance prior to the procedure
D. The vascular surgery should be delayed for at least 6 months follow ing the CABG
procedure

3. A 76-year-old female comes to the preadmission clinic for anesthetic evaluation prior to a right
total hip replacement (THR) scheduled in 2 weeks. Her medical history is significant for
coronary artery disease (status post stent placement 6 months ago) and baseline unstable angina
one to two times per month. The patient indicates that her symptoms are relieved by sublingual
nitroglycerin. A recent echocardiogram (30 days prior) showed an ejection fraction of 30%
along with evidence of inferior-wall-m otion abnormality. Examination of the most current
EKG shows diffuse T-wave inversions with a heart rate of 60 to 65 bpm (on metoprolol) and a
blood pressure of 125/60 mm Hg. In addition, the patient has severe chronic obstructive
pulmonary disease, is dependent upon 2 L/min home O2, and has obstructive sleep apnea (on
bi-level positive-airway pressure at night). In order to maximize the preoperative condition of
this patient, you w ill order all of the follow ing diagnostic tests/examinations/consultations,
except
A. Repeat the cardiac catheterization and confirm whether or not the patient requires coronary
artery bypass grafting (CABG) surgery prior to THR
B. Communicate with cardiologist to confirm patient is medically optimized
C. Would not introduce any more coronary interventions unless new symptoms are present
D. Maintain hemodynamic stability during THR surgery

4. A 74-year-old patient undergoes a lumbar sympathetic blockade to improve blood flo w after
sustaining a frostbite injury to the left lower extremity. Clinical findings that would suggest a
successful block include
A. Inability to dorsiflex the foot
B. Piloerection on the legs
C. Numbness from the knee to the toes
D. Temperature increase in the legs

5. The nerve that needs to be blocked to obliterate the gag reflex when applying pressure to the
posterior portion of the tongue during an awake fiberoptic intubation is the
A. Recurrent laryngeal nerve
B. Glossopharyngeal nerve
C. Superior laryngeal nerve
D. Inferior laryngeal nerve

6. A 74-year-old patient undergoes a stellate ganglion block secondary to extreme hot flashes and
night awakenings secondary to a long history of breast cancer. Potential complications include
all of the following, except
A. Recurrent laryngeal nerve paralysis
B. Subarachnoid block
C. Pneumothorax
D. A ll of the above

7. Incorrect statement regarding metabolic equivalent (MET) is


A. 1 MET = consumption of 3.5 mL O2/min/kg of body weight
B. 5 MET = climbing one to two flights of stairs, dancing, or bicycling
C. 4 MET = equivalent to gardening
D. 2 MET = equivalent to getting dressed

8. A 35-year-old G2P1 at 30 weeks gestational age is coming to the OR within the next hour for
open reduction internal fixation of an ankle fracture. The patient’s blood type is O+ and has
hematocrit of 32. A ll of the follow ing should be arranged, except
A. Prepare for a perioperative obstetrical (OB) consultation
B. Type screen and crossmatch for blood
C. Intraoperative RhoGam injection prior to surgery start
D. Prepare for perioperative fetal monitoring

9. An E-cylinder of oxygen with a pressure of 1,000 psig and being used at a rate of 2 L/min w ill
run out in
A. 2 hours
B. 3 hours
C. 4 hours
D. 6 hours

10. A 49-year-old patient is undergoing a craniotomy for tumor resection. Intraoperatively, the
patient received drugs including thiopental, vecuronium, isoflurane, and fentanyl. The patient is
brought to the postanesthesia care unit with a HR of 58/min, BP of 196/96 mm Hg, and oxygen
saturation of 98%. A few moments later the patient has two episodes of vomiting. You would
then
A. Give ondansetron
B. Give metoclopramide
C. Give fentanyl
D. Call the neurosurgeon

11. Parkinsonism is associated with


A. Loss of dopaminergic neurons alone
B. Loss of cholinergic neurons alone
C. Loss of cholinergic and increase in dopaminergic activity
D. Loss of dopaminergic and increase in cholinergic activity

12. A 36-year-old patient with multiple sclerosis (MS) is to undergo an exploratory laparotomy.
The best anesthesia technique to prevent a flare-up of symptoms would be
A. General anesthesia with endotracheal intubation using a nondepolarizing muscle relaxant
B. General anesthesia with endotracheal intubation using a depolarizing muscle relaxant
C. Spinal anesthesia
D. Combined spinal-epidural anesthesia

13. The primary aim of using succinylcholine for anesthesia for electroconvulsive therapy (ECT)
is to
A. Prevent loss of airway
B. Control excessive seizure activity
C. Control cardiovascular sympathetic discharge
D. Prevent musculoskeletal injuries

14. Cardiovascular response follow ing an electroconvulsive therapy (ECT) is characterized by


A. An initial parasympathetic discharge followed by a sympathetic discharge
B. An initial sympathetic discharge followed by a parasympathetic discharge
C. Sympathetic discharge alone
D. Parasympathetic discharge alone

15. Nondepolarizing muscle relaxants block which of the follow ing receptors?
A. Adrenergic
B. Calcium
C. Muscarinic
D. Nicotinic

16. Ipratropium acts to relieve bronchospasm via which of the follow ing receptors?
A. Nicotinic
B. Muscarinic
C. a-Receptors
D. ^-Receptors

17. A ll statements regarding neostigmine are true, except


A. It inhibits acetylcholinesterase
B. It inhibits pseudocholinesterase
C. It shortens the duration of action of succinylcholine
D. It can cause neuromuscular blockade

18. When using neostigmine to reverse neuromuscular blockade in the presence of severe renal
disease, you would use the follow ing dose when compared to a normal patient
A. Same
B. Higher
C. Lower
D. Titrated

19. Fastest acting neuromuscular reversal agent is


A. Edrophonium
B. Neostigmine
C. Pyridostigmine
D. Physostigmine

20. Highest plasma concentration of a local anesthetic w ill occur if infiltrated via which of the
follow ing routes?
A. Tracheal
B. Caudal
C. Intercostal
D. Brachial plexus

21. A 27-year-old 38 weeks pregnant female presents with painless vaginal bleeding. The best step
in the management of this patient is
A. Direct examination with a vaginal speculum and then take the patient to OR for cesarean
section
B. Cesarean section
C. Bed rest and observation
D. Epidural after bleeding stops

22. The most frequent cause of delayed emergence in the postanesthesia care unit is
A. Residual anesthetic agents
B. Hypoventilation
C. Hypotension
D. Hypothermia

23. Emergence from inhalational anesthetics is prim arily dependent on


A. Type of agent used
B. Cardiac output
C. Ventilation
D. Adjunct anesthetic drugs

24. Emergence from intravenous anesthetics is prim arily dependent on


A. Redistribution
B. Elimination half-life
C. Type of agent used
D. Hepatic or renal disease

25. A 35-year-old patient is brought to the postanesthesia care unit (PACU) after undergoing an
appendectomy. His anesthetics included propofol 140 mg, isoflurane 2.0 MAC, vecuronium 6
mg, and morphine 6 mg. In the PACU, the patient is shivering. The most likely cause of his
shivering is
A. Use of isoflurane
B. Presence of infection and dehydration
C. Use of unwarmed fluids
D. Use of morphine
26. Best method to prevent shivering is
A. Use warmed fluids
B. Warming lights
C. Meperidine
D. Forced-air-warming device

27. A 56-year-old patient, with a tracheostomy, is undergoing a radical neck dissection under
general anesthesia. The induction is uneventful and you proceed to replace the tracheostomy
tube with an endotracheal tube for the procedure. The patient’s peak airway inspiratory
pressures increase suddenly. The most likely diagnosis is
A. Bronchospasm
B. Pneumothorax
C. Malposition of the endotracheal tube (ETT)
D. Patient attempting to breath

28. Laryngospasm (LS) is due to stimulation of the


A. Superior laryngeal nerve
B. Internal laryngeal nerve
C. Recurrent laryngeal nerve
D. External laryngeal nerve
CHAPTER 21 ANSWERS

1. A. Pulmonary function test results have not been shown to be beneficial or to guide treatment
when planning for intraoperative anesthesia. History and physical exam are the basics and
important in anesthesia plan formulation. Baseline chest film s along with arterial blood gas
results are not indicated in every pulmonary patient, but may be helpful in anesthesia decision­
making and intraoperative anesthetic management.

2. B. With the exception of emergency surgery, current guidelines suggest waiting at least for a
1-month time interval follow ing a coronary intervention, before proceeding with any elective
surgical procedure.

3. A. Generally speaking, the indications for cardiovascular investigations are the same in
surgical patients as in any other patient. Unless the combined risk of coronary intervention and
surgery is less than surgery alone without coronary intervention, preoperative CABG/stent, etc.,
is not generally suggested.

4. D. Indications for a lumbar sympathetic blockade include diagnosis, prognosis, and therapy of
circulatory and pain conditions such as inoperable peripheral vascular disease, vasospastic
disease (lower), reflexive sympathetic dystrophies and herpes zoster (lower), and the presence
of pain (neuropathic, urogenic/pelvic, cancer pain, and phantom limb). Contraindications for a
lumbar sympathetic blockade include anticoagulant therapy, hemorrhagic disorder, allergy to
injected medications, infection, local neoplasm, and local vascular anomalies. Lumbar
sympathetic chain includes L3-L5 ganglia, and is positioned anterior to L2, L3, and L4
vertebral bodies, anterior to the psoas muscle margin and fascia, posterior to the vena cava on
the right, and posterior to the aorta on the left. Complications of a lumbar sympathetic blockade
include blockade of the L2 somatic nerve root, injection into the
subarachnoid/epidural/intravascular (vena cava/aorta/lumbar vessels) spaces, damage by needle
or neurolytics to the kidneys/renal pelvis/ureters/intervertebral disks, infection, backache,
neuropathic pain, hematoma, sympathalgia, destruction of sympathetic fibers
(cramping/burning pain to anterior thigh), sympathectomy-mediated hypotension, intravascular
steal (especially arteriosclerotic patient), and failure of ejaculation.

5. B. Airway blockade techniques: For anesthesia of nasal mucosa and nasopharynx, and nasal
intubation, the sphenopalatine ganglion and ethmoid nerves need to be anesthetized. For
anesthesia of the mouth (oropharynx and tongue base), the glossopharyngeal and superior
laryngeal nerve blocks need to be performed. For anesthesia of the hypopharynx, larynx, and
trachea, the recurrent laryngeal nerve needs to be blocked by performing a transtracheal block.

6. D. Complications of stellate ganglion block include hematoma formation (vascular injury to


carotid artery, internal jugular vein), nerve injury (vagus, brachial plexus roots),
pneumothorax, esophageal perforation, intravascular injection (carotid or vertebral artery,
internal jugular vein), epidural or intrathecal injection, hoarseness of voice (recurrent
laryngeal nerve), elevated hemidiaphragm (phrenic nerve), infection, and Horner syndrome
(ptosis, anhidrosis, miosis).

7. D. 1 MET = consumption of 3.5 mL O2/min/kg of body weight. Typically, 1 MET = dressing


or eating; 2 MET = walking downstairs or cooking; 4 MET = gardening; 5 MET = climbing
one to two flights of stairs. A patient unable to achieve the level of 4 to 5 MET is at an
increasing risk of perioperative complications, typically cardiopulmonary adverse reactions.

8. C. The patient is Rh O+; therefore, there exists no need for RhoGam immunoglobulin
injection. OB consultation should be initiated with any pregnant patient, and the obstetrician
should decide the need for appropriate perioperative monitoring (continuous monitoring
versus pre- and postoperative monitoring) of the mother and the fetus based upon the stage of
pregnancy.

9. B. An E-cylinder of oxygen at 1,000 psig is approximately half full, that is, it has about 330 L
of oxygen. If being consumed at a rate of 2 L/min, it w ill be exhausted in about 3 hours.

10. D. Vomiting in patient who has undergone an intracranial procedure may indicate raised
intracranial pressure. Therefore, the patient needs to be evaluated immediately, and the
neurosurgeon needs to be notified.

11. D. Parkinsonism of Parkinson disease (called when no identifiable cause) is associated with a
loss of dopaminergic activity and a reciprocal increase in cholinergic activity in the brain.

12. A. General anesthesia is most often used in patients with MS. Regarding muscle relaxants, the
use of succinylcholine should be avoided, as demyelination and denervation may increase the
risk of succinylcholine-induced hyperkalemia. Nondepolarizing neuromuscular blockers are
safe to use, but patients of MS may have altered sensitivity and prolonged duration of action,
which may necessitate postoperative ventilation. Therefore, nondepolarizing muscle relaxants
should be administered in minimal doses. Regarding regional anesthesia, spinal and epidural
anesthesia and peripheral nerve blocks have been successfully used in patients with MS.
Although spinal anesthesia has been implicated in postoperative exacerbations of MS
symptoms, the finding is not fu lly confirmed. Furthermore, intraoperatively the patient’s
temperature should be closely monitored, as even slight increases in body temperature may
cause a decline in neurologic function postoperatively.

13. D. ECT is performed under general anesthesia. The patient is preoxygenated, and general
anesthesia is induced with a hypnotic (methohexital or propofol). Once the patient is asleep,
succinylcholine is administered to relax the muscles. Seizures produced by ECT have been
known to cause musculoskeletal injuries and jo in t dislocations. Therefore, succinylcholine is
used to relax the muscle and prevent such injuries. Airway is maintained with mask ventilation.
14. A. Cardiovascular response follow ing an ECT consists of an initial parasympathetic response
followed by a sympathetic response. The parasympathetic response may lead to severe
bradycardia in some. Glycopyrrolate administered pre-ECT may attenuate the parasympathetic
response and also decrease secretions. The sympathetic response leads to tachycardia and
hypertension, which may lead to deleterious effects in patients with coronary artery disease. The
sympathetic discharge can be attenuated by using P-blockers (esmolol, metoprolol) or labetalol.

15. D. Nondepolarizing muscle relaxants inhibit neuronal transmission to the muscle by blocking
the nicotinic acetylcholine receptors. They act as competitive antagonists to acetylcholine (Ach)
and prevent the binding of Ach to the receptors.

16. B. Ipratropium (atrovent) is a bronchodilator and acts on the muscarinic acetylcholine


receptors in the smooth muscles of the bronchi in the lung when inhaled. It is a derivative of
atropine, but has a quaternary amine structure and thus it does not cross the blood-brain barrier
to cause central effects. Although ipratropium is commonly combined with albuterol as a rescue
agent for bronchospasm, it should not be used as a replacement for albuterol.

17. C. Neostigmine is a reversible acetylcholinesterase inhibitor, the enzyme that breaks down
acetylcholine. This leads to more acetylcholine being available for neuromuscular
transmission, which can now competitively displace the nondepolarizing muscle relaxant
molecules to cause the return of neuromuscular activity. Since succinylcholine is broken down
by a similar enzyme (pseudocholinesterase), neostigmine administration leads to the
prolongation of duration of action of succinylcholine. It should be remembered that
neostigmine, when given (unintended) without the prior administration of a nondepolarizing
muscle relaxant, can directly act as a muscle relaxant when given in sufficient dose.

18. A. Renal excretion accounts for about 50% of excretion of neostigmine (about 75% of that of
pyridostigmine and edrophonium). It is important to note that the presence of renal failure
decreases the plasma clearance of not only neostigmine (and pyridostigmine, edrophonium) but
also nondepolarizing muscle relaxants. Therefore, if neostigmine is administered in the usual
dosage, and overdoses of muscle relaxants are avoided, renal failure should not be associated
with recurarization.

19. A. Edrophonium is given in a dose of 0.5 to 1 mg/kg and has an onset of action in 30 to 60
seconds. Peak action occurs in 1 to 5 minutes and duration of action of is about 5 to 20 minutes.
Because of its short duration of action, patients should be monitored for the effects of
recurarization. Onset of neostigmine’s action (0.04-0.07 mg/kg) is in 1 to 3 minutes, peak effect
occurs in 5 to 7 minutes, and duration of action is 40 to 60 minutes. Pyridostigmine is not used
for neuromuscular reversal, and physostigmine has no role in neuromuscular blockade
reversal.

20. C. Local anesthetics, when infiltrated into tissues, get absorbed into the circulation to some
extent. The amount of local anesthetic absorbed into circulation depends upon the vascularity of
the area. Highest blood concentration occurs with intercostal infiltration due to the high
vascularity of the area.

21. B. Painless vaginal bleeding is most commonly due to placenta previa. A full-term parturient
who presents with active painless vaginal bleeding should be taken to the operating room for
cesarean section under general anesthesia. Examination with a vaginal speculum may initiate
massive hemorrhage and hence should be not performed. Patients should have large-bore IVs
(even a central line) for adequate fluid resuscitation, and blood should be available for
transfusion. Patients with placenta previa, who are less than 37 weeks of gestation, and with m ild
bleeding, may be managed with bed rest and observation.

22. A. The most common cause of delayed emergence is residual anesthetics. These can be
sedatives, analgesics, muscle relaxants, or volatile inhalational agents. Overdose of narcotics
can be reversed by naloxone, benzodiazepines can be reversed by flumazenil, muscle relaxants
are reversed with an appropriate dose of neostigmine-glycopyrrolate and administered as per
the train-of-four twitch monitoring, and volatile agents are washed out by adequate ventilation.
Hypoventilation can lead to hypoxia and hypercarbia. Hypothermia potentiates the effects of
CNS depressants, and can be prevented by using forced-air-warming devices, using warm
intravenous fluids, and raising the ambient room temperature. Other causes of delayed
emergence include hypotension and metabolic abnormalities.

23. C. Once the administration of volatile agent is stopped at the end of the surgery, the washout
or elimination occurs prim arily through the lungs. Hence, adequate ventilation is the main route
of elimination of volatile inhalational agents. Hypoventilation due to any cause w ill decrease the
washout of volatile anesthetics and delay emergence from anesthesia.

24. A. Emergence from intravenous anesthetics depends prim arily on redistribution from the
brain. However, as the intravenous drugs accumulate, due to repeated administration or
infusion, emergence becomes dependent on metabolism and elimination half-life. Presence of
hepatic or renal disease, and the pharmacokinetics of the agents, also affects emergence from
anesthesia.

25. A. Volatile inhalational agents cause peripheral vasodilation and cause redistribution of heat
from the body core to the peripheral compartment. Using isoflurane in such a high
concentration (2 MAC) is the most likely cause of shivering in the PACU in this patient. Other
causes that can cause shivering are cold ambient operating room temperature, using unwarmed
intravenous fluids, and an open large wound (exploratory laparotomy). Shivering tries to raise
the body’s temperature by causing intense vasoconstriction. In addition, shivering can increase
the oxygen demand tremendously, which can be of issue in patients with coronary artery
disease. Shivering can be treated with meperidine (12.5-25 mg IV). Hypothermia should be
treated by raising the room temperature or by using a forced-air-warming device.

26. D. One of the best methods to prevent hypothermia and shivering is using a forced-air-
warming device intraoperatively or in the postanesthesia care unit. Meperidine is commonly
used to treat shivering, 12.5 to 25 mg IV. Warming lights, raising the room temperature, and
using warm intravenous fluids are other methods to prevent or treat hypothermia.

27. C. Patients undergoing radical neck dissection for laryngeal cancer often have a tracheostomy
tube. After induction, the tracheostomy tube is commonly replaced with an ETT, which is
sutured into place by the surgeon. The ETT should be placed carefully, and adequacy of
ventilation should be checked by ausculating breath sounds and the presence of end-tidal CO2. A
malposition of the ETT, including placement in a false passage w ill lead to high peak
inspiratory pressures. Other causes listed can also lead to high inspiratory pressures.

28. A. LS is a forceful involuntary spasm of laryngeal muscles. It is due to stimulation of the


superior laryngeal nerve. LS occurs commonly due to intense stimulation during light
anesthesia (during extubation). Also, the presence of oral secretions can lead to LS. Treatment
of LS is done by providing positive-pressure breaths (bag and mask) with 100% oxygen. This
usually breaks the LS. However, if LS persists, succinylcholine is administered in a small dose
(0.25 mg/kg) to relax the laryngeal muscles.
Side effects of inhaled ipratropium are minimal and include dry mouth, skin flushing,
tachycardia, palpitations, and headache. It is contraindicated for use in patients with narrow
angle-closure glaucoma. In patients with prostatic hypertrophy, it can lead to urinary retention,
and hence should be used with caution in these patients.
Parkinsonism is characterized by progressive loss of motor function resulting from the
degeneration of neurons in substantia nigra region of the brain. The onset of Parkinson disease
typically occurs between the ages of 60 and 70 years. Clinical signs include a slight tremor of
the thumb and forefinger (p ill-ro llin g tremor), muscular rig id ity (arms, legs, neck),
bradykinesia (difficulty in initiating movement), postural instability, a shuffling gait, lack of
facial expression (masked face), and difficulty in swallowing or speaking. The disease slowly
progresses over 10 to 20 years, resulting in paralysis, dementia, and death.

Вам также может понравиться